Thùy trang next top model ảnh sex full năm 2024

See other formats

TAP CHÍ TOÁN TUỔI THƠ TỔNG TẬP ToânTuổiThơ NẢM 2016 TRUNG nọc CỚ NĂM THỨ MƯỜI BẢY ISSN 1859-2740 01/2016 Giá: lữOOOđ NHÃ XƯÂT BẦN GIÃO Dực VIỆT NAM ■ Bộ GIAO DỤC VÃ ĐAO TẠO Chiỉdren's Fun Maths Journaỉ NHÀ XUẤT BÁN ŨIẢO ĐỤC VIỆT NAM - BỘGIÁO Dựữ VÀ BÃO TẠO CH|U TRÁCH NHIỆM XUẤT BÀN Chù lịcti Hội dong Thành viên MẠC VAN THIỆN Táng Giâm đóe MỀm Tftnfl Mèn lặp om Vũ VÃN HÙNG HỘI ĐỒNG BIÊN TẬP Tống biên Lập. ThS .VỦ Kĩ M THỦY NGNP, VŨ HỮU BÍNH TS. GIANG KHÁC BÍNH TẸ r TRẨN ĐÍNH CHÃU TS. VŨ DÍNH CHUẨN TS. NGUYỀN minh đức ThS. NGU YẾN ANH DŨNG ĨS. NGUYỄN MINH HÀ PGS, TS. LẺ QUỐC HÁN PGS.TSKH. VŨ ĐÌNH HÒA TS. NGUYÊN ĐƠC HOÀN 5 ThS. NGUYỄN VÚ LOAN NGUYỆN DỬC TAN PGS. TS. TÔN THÂN TRƯƠNG CỒNG THÀNH PHẠM VAN TRỌNG ThS. HỐ QUANG VINH TÒA SOẠN Tlng 5 , F.D 361 đưởreg Trựủng Chinh, qưãn ThẸĩ] h Xuãrn, Hã NÃÈ Đsẹn inoại (Tel): 04 - 3569-2701 Điện sao í'Fax): 04,35662702 Diộn thư (Email): toantuũilhũ@vrrn.vn Trang mạng {Wtìũ£it&). //www.toanỉuolthũ.vn OẠI DtỀN TẠI MIẾN NAM NGUYỄN VỄFT XUÂN 55/1 2 Trán Đinn Xu. p. cáu Kho. Q.1 . TP. HCM ĐT. &&.Ễ6B211&9. DĐ 0973 308 199 Blôn tập NGU YẾN NGỌC HÂN, PHAN BƯONG Trị sự - Phá L nành: TRINH THỊ TUYẾT TRANG, VŨ ANH THƯ, NGUYỀN HUYÊN THANH ChỂ bàn: ỡỏ TRUNG KIẺN Mĩ thuât: Tủ ÂN TRONG SỐ NÀY Đành cho học sinh Iđp 6Ằ 7 Kỉ năng vận đụng dấu hiệu chia hết với học sinh lớp 6 Thát Hữu Huệ Học ra sao? Giai toán thê nào? Khai thốc bàf loản hình học trong sách giáo Khoa Đậu Cộng Nhữ Com pa vui tính CóchÍB hết Khủng? Nguyễn Đửc Tấn Phá án cùng thảm tử Sẽlỏccõc Đỗi hoa tai biến mất Nguyễn Quang Hiếu Đến vởi tiếng Hán Bài 65. Đà Năng hống hơn Há Nội Nguyễn Vũ Loan Học Toán bềng tiếng Anh Lines Vũ Kim Thủy Sai ỏ đâu? sứa chữ đúng Giả trị lớn nhất Nguyễn Đoàn Võ Oảnh cho các nhà toán học nhò Phân chĩa hình chữ nhật để ghép lại thành hình vuỗng (Tiếp theo Ki trước) Nguyễn Việt Hải Để thi cãc nước Ạustralian Mathematics Compelìtion - AMC 201 3 (Junior Division) nếp theo kì trước Đỗ Trưng Hiệu Tr 13 Tr 19 Tr 20 KĨ NĂNG VẬN DỤNG DẤU HIỆU CHIA HẾT VỚI HỌC SINH LỚP 6 THÃI HỮU HUỆ (GV. THCS Quang Lộc. Can Lộc. Hà Tỉnh} Càc bái toàn về chia hết ồ lòp 6 cô nội ơưng rẩĩ phong phủ. Trong bài viết này chúng tỏi giời thiệu một sô ơạng toàn chia hết thường gặp ƠỂ hoc sinh có những kĩ ráng tổt hơn khi giải toán. Bái toàn li Tim số tự nhiên có hai chữ 50 giong nhau, biết rằng số đó chia hết cho 2 vá chia cho 5 dư 3. Lời giái. Gọi số cán tim lá aa (a lá chữ sỗ khãc 0). Vì aa chia gho 5 dư 3 nên a = 3 hoặc 3 - ỡ. VI aa chia hẽt cho 2 nền a = s. SỐ cần Um là 86. 8ãi toán 2. có bao nhiêu 50 tự nhiên n chia hết cho 2 vá 5, biết 136 < n < 1 &2. Lởi giải. Các &D chia hốt cho 2 và 5 là các số có tận củng là 0, Mà 136 < lì < 162 nên n bàng 140, 150, 160, 170 và ISO. Vầy có tã L cả 5 số. Bải toán 3. Tìm cảc chữ 50 a. b biết a63b chia hết cho 2, 3, 5 và 9. Lới giải. Đổ a63b chia hếl cho 2 vả 5 thi b ù, Ta oò số a63G chia hẽt cho 9, suy ra a 4 9 : 9. Do đó -a - 9, Vậy Sũ cẩn tìm ỉè 9630, Bảl toản 4. Tim cảc chữ sõ a. b sao cho a - b = 4 vi â7ab : 9. Lồi giải. Vì 87ab : 9 nẽn a + b + 8 + 7 = a + b + 15:S, suy raa+bc {3; 1 2}. Mà a Ì- b > a-b = 4. Do dố a 4 b = 12. Suy ra 3 = 8. b = 4. Bài toán 5, Viết các số tự nhien Nén tiếp từ 1 0 đốn 99 la đưọc mọt 50 tự nhiên. HÒI 50 đó có chia hết cho 9 không? Tại sao? Lời giải. GọiAlả 50 đưoc viết bài 90 SỂ 10. 11, 12, .... 99. Tổng các chừ số hàng dơn vị của 90 số đõ bâng {0 r 1 4 2 4 ... 4 9).9 = 45.9 = 405. Tổng các chữ sđ háng chục của 90 s6 dó bằng (1 - 2 + ... 4 9}. 10 = 45.10 = 450.. Tổng các chữ số cua sổ A lá 405 4 450 = 855. Vi 855 : 9 nên A 9, Bái toàn 6. Cho các chữ số 3, b Nhác 0. Chứng minh rằng

  1. abba : 11;
  2. aaabbb : 37;
  3. ababab : 7. Lài giải, a) Tạ có abba = 1 0OOa 4 1 QOb 4 1 0b 4 a
  4. 100 la 4 1 lOb - 1 1(9la *10b) 1 11.
  5. Ta có aaabbb -■ lOOŨOŨa ilOOŨOa rlŨOOa itŨOb Ilũb ib = 1110003 -11 1b =111(10003 +b} -37.3.(1O0Qa 4b}i 37.
  6. Ta có □ babab = lOOOOati 4l0ùab tab = 1 0101 ,ãb = 7.1443.ãb !7. 8àl toán 7. Tím các chừ số H, y sao ch ũ 34x5y chia hết cho 36. Lời giải. Vì 34x5y:36 nên 34x5yi9. Suy ra 3 4 4 + X 4 5 + y = (12 4 X 4 y) : 9. Do đò X 4 y = 6 hùãc X 4 y = 1 5. Mà 34x5y:36 => 34x5yl4 ■=> 5yl4. Suy ra y = 2 hõàc. y = 6. m Nếu y = 2 thi X = 4. • Nấụ y = 6 thi X = 9 hoặc X =: 0. Vây các câp Sũ (x, y) cãn tim lả (4, 2), (9, 6} hoạt {0. 6). Bài tập Bàí 1. Chủng minh rằng ab 4 ba : 1 1 . Bàí 2. Chúng minh ràng nếu ab = 2cd thì abcd í 67. Bàl 3. Chứng minh rằng nếu (abc 4deg) ; 37 thl abcdeg :37. Bãi 4. Chửng minh ràng A = 1 00000 - ,, 0000 40 chia hất cho 18. 2015 chữ stìo VE cụọc THI TOAN PHAT HIẸN TÀI NĂNG CỦA AUSTRALIA (AMC) AUSTRALIAN MATHEMATICS COMPETITION TẠ NGỌC TRỈ (Hà Nội) Đ ổi với học sinh Việt Nam chùng ta, đất nước Austraỉia tturỡng được nghĩ đễn lả xử sở cùa chuột túi (kangaroo) hay cẩu cảng Sydney nổi tiếng với những màn pháo hoa rực rỡ khi đón chào năm mửi. Tuy nhiên đối vứỉ những nguủi quan tâm đến các kỉ thi Qlympic Toán Quốc tế IMO (International Mathematics Olympiad) thì Australia là nơi giữ ki lục của thí sinh nho tuổi nhất đoạt huy chương vàng. Terence Tao giành huy chương vàng IMO 1933 tại Canberra khi 13 tuổi. Trên thực tế Terence Tso khi đoạt huy chương vàng đả dự thi Toán Quốc tế hai lển trước đó (năm 1986 đoạt huy chương đống vá năm 1937 đoạt huy chương bạc}. Sa ư nãy, khi trưdng thánh vá đạt được nhiều thánh công trong nghiên cứu toán học, được công nhận bởi nhiều giải thưởng toán học uy tín, trong đó có giải thưởng Fteltís nãm 2006, GS. T. Tao, khoa Toán - Đại học Calitornia tại Los Angeles (Hoa Kỳ) vẫn dành thời gian viết lại những kinh nghiệm học toán thời tuổi trẻ của minh ị trũng cuốn sách T. Tao (2000). Soỉving Mathematics Probiems , a Personal Perspective, Oxỉord ưníverstty Press). Trong cuốn sách này GS, T. Tao dẫn nhiều vi dụ lả các bài toán trong các cuộc thí Toán của Australia (AMC) để trình bày các ý tưỏng của mình. Trên thực tế AMC chính là nơi đã giúp Ausỉraíia vá thế giới tìm ra được một nhã toán học lởn, một Mozarl của toản học thế giới hiện nay như nhiếu người ca ngợi! Terence Teto iúc 1 2 tuổi, năm 1987 AMC lển đểu tiên đươc tổ chức năm 1978 và cho đến nam 2015 đã có 14,5 triệu học sinh từ 30 nưỏc trên thể gáởi tham dự. Cuộc thi náy hiện được tài trợ bài Ngân hàng CornmonwẼaflh và được Quỹ ủy thác Toán học Australia (Australian Mathematics Trust, AMT) quản lí. AMT tìm kiếm, phát hiện và từ dó bổi dương các tài nãrtg toàn hạc, tin học cho Ausưalia [hóng gua càc cuộc thi nhưAMC, CAT. Cuộc thi AMC có các bài thi cho học sinh khối lớp 3-4, khối lởp 5-6, khối lớp 7-3, khối lớp 9-1 0, vá khối lốp 11-12. Mũi bài thi có 30 câu hỏi làm trong 60 phút (đối vởi căc bãi thi khối lớp 3-4 vá khối 5-6} hoặc 75 phút với các khối lởp còn lạỉ, Các bàl toán được cảc chuyên gia toán học thiết kế theo đúng tiêu chí của cuộc thi Tỉm kiếm vả phát hiện tàỉ nâng ỉữán học. Chinh vì vây các báỉ toản được sắp xểp theo thử tự từ dễ đến khó, phù hợp VỚI tất cả các trinh độ học sinh. Các bài toán từ 1-10 được chấm 3 điểm/bài. từ 11-20 được chấm 4 điổm/bài. từ 21-25 đưạc chấm 5 đlểm/bài. Các bài toán "khó nhất", từ 26^30 được chấm tướng ứng 6, 7, B, 9 vã 10 điểm. Điểm cao nhất của hãi thi có thể đạt được của thí sinh \ề 135 điểm. Thí sinh dự thi sẽ được làm quen vởi cách làm bài toán thướng thấy ờ các kì thí chuẩn Quốc tế: đó là *tô'" chi vào Chữ cái đạt trước câu trẻ lời đúng. Riêng các cảu 26-30 thí sinh sẽ "tõ ,r vào cảc õ chỉ mõi số tự nhiên có ba chữ sổ mà Ihí sinh cho lả đáp số của bài toán đó. Việc chấm thi hoàn toán do máy ví tính thực hiện. Sau cưộc thi, moi thi sinh sẽ được AMT gửi cho một report (báo cáo) về kết quả bài thỉ cùa minh, kết quả chung của tất cẳ Cổc thí sỉnh cùng nhỏm dự thi cho từng bãi toán cũng như chung cho cả bái thi. Mâí thi sinh cũng sẽ nhận được chứng nhận của AMT vế thành tích cũa mình trên cơ sở thánh lích của các bạn khác ở cùng bang (đỗi với thí sinh của Austraỉia), hoậc cùng nưởc tham gia dự thỉ. Những thỉ sinh xuát sác nhất của mỗi nưức dự thi sẽ được nhận Huy chương (Medal) trong một buổi lẻ đạc biệt (kem thèm ở [1]). Nhừhg nãm Yừ& qua dã cò nhiếu học sinh người Việt Nam tham gia thi AMC khi học ỏ các trường ò Singapore hay Australia và đạt thánh tích rất tốt (xem [2], hoặc xem kết quả AMC từ những nâm trưỡc ở ( 1 ]>- Trong thói gian làm việc ở Austratia từ tháng 5^9/2015 tại Viện Chương trình, Kiểm tra đánh giá và Co quan bão câo giao dục của Auslralia (ACARA) lôi đả làm việc với AMT. Dược sự đống ý của AMT chúng tôi trẳn trọng giới thiệu với các bạn học sinh yêu toán cửa Việt Nam chúng ta vể AMC và mang muốn Tạp chi Toán Tuổi thơ sẽ hợp tác cùng với AMT tổ chức cuộc thi AMC tại Việt Nam từ nãm 2016. Ngoài cuôc thi này, chúng tôi cũng mong muổn các cuộc thi khác của AMT tổ chức như CAT hay AI MO cũng sẽ được giới thiệu tại Vìệ! Nam. Mục dích chung lá giới thiêu vái các bạn học sinh những bài toán, cách thi bổ ích bằng tiếng Anh, góp phần tim ksếm, kịp thời phát hiện những tài năng để boi dưỡng nhân tái cho đất nước. Tài liệu tham kháo [1] Trang của Quỹ ủy thác Toán học Australia: h ư p ://w ww.amt.edu. au/ [2 j http :fỉứ uhoc. dantrì .com .vn/d u-hoc/CQ-gaỉ- be-hat-tieu^aHiooHMmg-5"7HHJong-cten-dh- stanford"danh-tieng-201 509 1 2 1 2481 1 83.htm BE CUỘC THI .. . (Tiếp theo trang 2 6) Suy ra ngữ giác COOHE nội tiếp => CồH=CDH =90 ủ , Nhận xét. Bài toán nảy khó nên khủng cố bạn nào giải đúng. NGUYỄN MINH HA Các bạn sau được thưởng ki này: Kim Thị Hồng J;. tr/ỉb, 9E1, Phan Huyền Ngọc, 98, THCS Vĩnh Tương, Vĩnh Tường, VVnh Phúc; Bùi Thùy Lỉnh, 0A1; A igụyên Thùy Dưững, 5A3, THCS Lãm Thao, Lắm Thao; Lê Nguyễn Quỳnh Trang , 9C, THCS Vân Lang, TP. Việt Tri, Phú Thọ; Thành Tù Oanh, 9D, THCS Trung Dô, TP. Vinh, Nghệ An. Ẳnh các bạn dược thương ỡ bìa 4. I © wm số NÀO MỚI ĐÚNG DÂY? Bài 1 . Tim phân số tiếp Itieo của dãy phân số -L-L-L- ... z 7 13 21 31 Bài 2, Hãy tìm số thích hạp dổ điền váo dấu ? cho hạp lỏgic. NGUYỄN Đừc TẤN (TP. Hố Chi Minhị ismm Sô NÀO ĐÚNG NHỈ? (TTT2 SỐ 1 53) Nhận xét. Ki này càu a) hoi khó, rất it bạn phát hiện ra quy luật. Câu b) tuông đoi dề, tuy nhiên nhiêu bạn tim đũng dấu hiệu đặc trưng của các sổ hang trong dãy, nhưng ghi kết quà sai, cho rằng số 91 lá so nguyền lổ {mà tìl = 13.7 lả họp sổ). Quy luật, a) Ta viểt lại dãy số dã cho thánh _ 5, 13. 39. 151 ,
  7. ? T i ; V- - . , , , II ,6 21 . Ta thấy 2 = — — + 1; = T + 1; 2 1 - 1 3 2 2 - 1 13 3! . -£ = ■-- + 1; ... 7 2 3 -1 Số hạng tổrvg qưát của dãy có dang u n = — - 1 1 . 2" -1 Theo quy luật đó, sỗ hung tiếp theo của dây (số hạng Lhứ 3) lả {TTT2 số 153) THẾ CỞ (Kì 76) 1 ,lh6 gxf6 [1 ...gxh6 2.Êe7#| 2.Í167+ *h8 3.®xf6# r Các bạn sau được thưởng kì này: P/ĩan li.l v.:. Vãn Tài, 7B. THCS Hoàng Xuân Hãn. Đức Thọ, Hà Tĩnh; Nguyễn Tuệ Ân. 7C. THCS Bạch Liéu, Yèn Thánh. Nghệ An; Phan Thu Trang. 9A1, THCS Chat lưọng cao, Mai Sơn ; $ơn La: Nguyễn Thanh Tụng, 7D. THCS vinh 1 ưừng Vĩnh Tường- Lựu Trượng Giang, 7A, THC$ vinh Yên, TP. Wnh Yền, Vĩnh Phúc. LÊ THANH TÚ 6! . 720 J 703 87 .„3 w + 1 = -=-L + 1 = -1 = -± = 12r- 2-1 63 63 7 7
  8. Dây số 11; 31; 41; 61; 71; ... gom các sấ nguyên tố liên tiếp có tận cùng bềng 1. Vậy só Bếp theo của dãy là 101. Xin trao thuởng cho bạn: Lã Nguyển Quỳnh Trang, 9C. THCS Vân Lang, TP. Việt Tri Phũ Thọ: Đổ Tiến Dũng. Hà Báo Linh. 6D, THCS Vĩnh Yên, TP. Vĩnh Yên; Vữ Đức Duy, SE2, THCS Vĩnh Tutìng. Vĩnh Tunởng, Vĩnh Phúc. NGUYỄN XUÃN BÌNH Ban cố biết 10 HOỊỊĨ ĐỆHG uò Sự HIỆn cùn m TUỔI THO otìlĩl 2015
  9. Bắt đầu hoạt động Câu lạc bộ Toán Tuổi thơ cấp trường, huyện, tĩnh, tạo không khí mới cho dạy - học toán ử TH và THCS.
  10. Tố chức hội thảo Toán Tu ni thợ tại Đống bằng sòng củu Long; đại biểu các đ--a phương: TP. Hồ Chí Minh, Cấn Thơ. Bạc Liẻu, Cà Mau. Hậu Giany, Kién Giang, Sóc Trắng, Tiền Giang. Trà Vĩnh, Vĩnh Long vể dự tại cần Tho.
  11. Tổ Chức Cuộc thi tỉm hiểu Cộng dóng ASEAN hướng tới ngáy thánh lập Cộng đồng ASEAN 31.12.2015.
  12. Tồ chức Cuộc thi Đặc biẹt nhân 15 nám Toán Tuổi thơ (2S.1Ò.2ŨỮỮ ra so đẩu tiên. 30,1,2002 thành lâp đdn vị).
  13. Hợp tảc VỚI Oníine Maih. Clas&book để xuất bản càc ấn bản điện Lử.
  14. Các hoạt dộng kỉ niệm 15 năm Toán Tưổi thơ như chuẩn bị cho Ngáy Toán Tuổi thơ, ra KÌ yếu Toán Tuối thơ theo dóng thơi gian, Thi liẻn tỉnh CLB.
  15. 0] công tác nhiều sình [hanh Nam Qịnh. Thái Bình, Hà Nam, Hải Dương, Bác Ninh, Phú Thọ, Hài Phòng, Quảng Ninh, Ba Ri;a " Vũng Tàu, Đã Nắng. TP Hố Chí Minh, cán Thơ. ô. Tham dự các Hôi thảo toán của Hội Toán học Việt Nam, Hổi ỉháó toán Quốc tế ICME 2015 tai ĐH Bách Khoa.
  16. Tái bẳn 2 cuđn sách Tuyển chọn 10 nằm Toàn ruổi thơ, 279 Bái toán hỉnh học phàng Qlympic các nước được bạn đọc yèu thích.
  17. Tạng sách cha thư viện các trưởng ô Nam Ldi. Nam Trực, Nam Định. Xuân Hòa. Hả Quảng và Quảng Hưng, Quáng Uyên, Cao Báng. Tăng quá Tết cãc gia đinh chính sách ở Mộ Lao. Hã Đỏng. Há Nội. VŨ ĐÔ QUAN EXPRESSION, VARIABLE AND P0LYN0M1AL (TTT 2S Í, 53 , Đại số là một ngãnh irong toán học, nõ dựa trên những phép toàn- cộng, trử, nhãn, chia của sổ học và dựa trẽn khái niệm của đại lượng chưa biết hoặc biến, Những chừ cái như X hoặc y dược sử dụng để biểu thị nhũĩig đại lượng chua biết. Một sự kết hợp gi im các chứ cài và phép toán số học, như B 3, 6x 2 - 5x + 1956 vả —————— được 1981X-19&4 gọi là biểu thức đại số. Biểu thức 6x 2 - 5x - 1956 bao gồm các số hạng 6x 2 , -5x vã 1 956; 6 lá hệ số của X 2 , —5 [ã hệ sò của X và 1 956 lá một háng số [hoộc lá hệ sứ của x°). Biểu thức B - 3 lã da thửc bậc nhất cùa B vì lũy thừa cao nhất cũa B là 1. Biểu thức B + 3 Eâ một da thúc tuyến tính của B, Biểu thức 6x 2 - 5x + 1956 dược gọi lá đa thúc bậc hai của X vi lũy thừa cao nhất của X lá 2. Biểu thức 6x 2 - 5x + 1956 dược gọi lã đa (hức bậc hai của X. 8ieu thủc — — — - — — không phâi lá một đa thúc, 19S1X-19&4 Nhãn xét. Ki nãy có rất nhiều bạn ... tham gia dịch vã gửi bái vé tòa soạn. ■ Hau hết các hạn deu hiểu nội dung vá lời dich tượng dối gặy gọn. Các bạn xuất sắc nhất đuợc nhận quà ki này: Trân Diệu Linh, 9B, THCS Nguyễn Thượng Hiển, Ưng Hòa, Hà Nội; Kiếu Bảo M ỳ. 9A2, THCS Vận Phong, Yèn Phong, Bẩc Ninh: Phạm Thùy Lữìh, Nguyễn Đức Tấn, 7A3, THCS Lâm Thao, Lâm Thao, Phú Thọ; Nguyễn Nhặt Linh, BẼ. THCS Lê Quý Đon, TP. Tuyên Quang, Tuy én Quang; Hoàng Hà My. BA, THCS Chu Ván Ãn, Nga Sơn, Thanh Hóa; Vứ Thài Thùy Linh. 6B; Hoàng Thị Trang, ec, THCS Bạch Liêu, Yên Thành; Nguyễn Trinh Tuấn Đại, 7D, THCS Lý Nhật Quang, Dô Lương; Nguyễn Thị Mai Anh, 7D; Thài Anh Quàn. SA, THCS Đặng Thai Mai TP. Vinh, Nghệ An; Nguyễn Hưng Phát, 6B, THCS Hoàng Xuân Hãn, Đức Thọ, Hà Tĩnh; Thôn Hoài Thương. 7/7, THCS Võ Như Hưng. Điện Bán. Quàng Nam. Các bạn sau dược khen ki náy. Tử Tắn Dũng, 7D, THPT chuyên Hà Nội - Amsterdam, cầu Giấy. Hả Nội; Nguyễn Thị út Thơm, SAI: Ngỏ Th> Thuyết 8Ẩ2, THCS Yên Phong. Yên Phong. Bắc Ninh; tê Đúc Thài, ẼA2. THCS Yên Lạc r Yên Lạt, Vĩnh Phúc; Hoảng Nguyễn Ngọc Giang, 70, THCS Ván Lang, Việt Trì, Phú Thọ: Nguyễn Thị Bâng Bàng, 7C; Vô Trà jWy r Phạm Thị Ngọc ũìệp, BC, THCS Bạch Liêu, Yên Thánh; Thành Tù Oanh, 9D, THCS Trung Đố. TP. Vinh, Nghệ An. ĐÕ TRUNG KIỀN 6 I KẾT QUẢ PHIẾU THĂM DÒ DO ĐỘC GIẢ GỬI TỚI những chuyên mục được yẻu cầu VỚI tỉ lệ Tạp chí Tũản Tuổi thơ muốn mang đến bạn đọc nnột hinh ảnh mới vể cả nội dung và hình thức. PHIẺU THĂM DÒ là nơi đóng góp y kiến của bạn đọc giúp chúng tõỉ định hirông. diếu chỉnh nội dung ch ũ phù hợp. Sau đây là kết quả mà Tạp chí đã tổng hợp. Bạn dọc Toán Tuôi thơ 2 lẩn đau tiên khi nào? Oặt mua dài hạn qua bưu diện chiếm tỉ lệ cao nhất 61% tống số phiếu. Đánh giá chung mức độ dể thi giái toán qua thư: Sau khi chúng tỏi tổng hợp lạl thi các bài toãn trong chuyên mục nãy được độc giả đánh giá là ở múc độ vừa phái chiếm 54,3% và ỏ mứt độ khỏ chiếm tỉ lệ thấp.. Bạn thích chuyên mục nào nhất? Cảc phiếu cho thấy hẩu hết các chuyên mục của Tạp chí Toán Tuổi thơ 2 đếu được các bạn yêu thích, đặc biệt lả các chuyên mục: Thám tử sẻlổccốc; Dế thi giải toán qua thư, Thế cà; Đo tri thông minh; Vảo thảm vườn Anh; Học Toán bầng Tiếng Anh; Dành cho học sinh lớp 6 & 7; Học ra sao? Giải toán thế nào?; Sai ở đâu? sửa cho đủng; Để thi học sinh giòi “ Đế thi trưởng chuyên; Già ra chơi; Cuộc thi vui hè; Đẽ thi các rước; Góc Olympic: Rubic hỏi đáp; Nhỉn ra thế giới; Trưởng Olympic; Trang thơ; Dành cho cảc nhà toán học nhỏ; õn tập cùng bạn; Những đưởng cong toán học; Thách đấu; Bong bóng thì chìm ... Chuyên mục nào cần tăng thêm dỉèrt tích, tân sô xuất hiện hàng tháng? Cỏ rất nhỉếu chuyên mục được cảc bạn đọc yêu cầu cẩn được tăng thêm. Sau đây là cao nhất: Thám từsèlốccốc; Đề thi giải toán qua thư; vào thâm vườn Anh; The cờ; Học ra sao? Giải toán thế riào?; Đé thi học sinh gioi - Đế thi trường chuyẻn; Dành cho học sinh lớp 6 & 7; Đo trí thông minh; Giờ ra choi; Họctoỏn bằng Tỉếng Anh; TỪZero đến vồ cùng; ôn tập cùng bạn; Toán quanh ta; Đề thi các nước, khu vực; Compa vui tinh; Trò chuyện; Cuộc thỉ giải toán dành cho nữ sinh ... Chuyên mục nào cẩn rút gọn số trang, tần số xuất hiện hàng tháng? Rất ỉl chuyên mục độc giả yêu cầu cắn í giâm, tỉ lệ thống ké được là khỏng đảng kể. VỂ hỉnh thức mua Tạp chí thí phương án mưốn đạt Tạp chí dài hạn qua bưư điện Chiếm 45%, mua ở trưctng chiếm 45%. Hai phương ản này chiếm tỉ lệ khá cao so với cảo phương án còn lại. Vẽ phẩn đảnh giá chung Tạp chí thì hẩu hểt đẽu [á cảc phản hổi tích cực: Nội dung phong phú đa dạng, nhiểu điẽu mới mẻ, hình thức bảo đẹp; nhờ chuyên mục Dành cho học sinh lớp 6 & 7 đã giúp các bạn học lớp 6 & 7 học môn Toán tốt hơn ... Có rất nhiếu bạn còn yêu cẩu mỗỉ tháng, Tạp chí nên ra 2 sđ vã thêm chuyên mục Học tin học, trang Giao lưu Toán học, ... Thật ra tạp chí đả có chuyên mục Kết nối 3T. Kết quả phiếu thảm dò trên sẽ ỉà cán cứ để chúng tòi xem xét, thay đổi cho phù hợp hơn. Hi vọng Tạp chi Toán Tuổi thơ sẽ lư&n I là cuốn Tạp chí mà các bạn mong chở đón đọc hàng thảng. TTT / © Lời GIẢI ĐỂ JHI ỌLYMPIC TOÁN HỌC TRẺ QUỐC TẾ CIMC TẠI TRUNG QUÔC 2015 (CIMCỈ (77ềp theo ki vườcị ThS. PHUNG KIM DUNG. CAI VIẾT LONG (GV. THPT chuyêỉỉHà Nội - Amsíerổam) TS. NGUYỄN VIỆT HẢI (Hà Nậí) (Sưu tầm và giòi thiệu) 11 . Gọi ha số đang xẻt là Hi = aa, n£ = ba, =b với a, b là các chữ sỏ khác D.
  18. Giâ sửn, lã so nguyên lố ỉhi n n = 11, Giắ sửn 3 lả SŨ nguyên tố Ihỉ n 3 cố thể bằng 2 hoặc 5 vỉ 21 và 51 không phải là so nguyên íố, nhưng không phải là 3 hay 7 vi 31 và 71 là số nguyên tố. Giả sử n 2 lã SỂ nguyên lố thi n 3 không là sổ nguyên tố. Vi thế n 3 gó thể là 4 hoệc 6 vi 41 và 61 là số nguyện tố, nhưng không phai 8 hoặc 9 vì 61 và 91 kh&ng phải là sổ nguyên tố. Ta đã có 4 cách chọn. • Giả sử n 1 không phải là số nguyên tổ Ihì p„ vã đều lá 50 nguyên tổ. Nếu n 3 - 2 Ihì n 2 lá 23 hoặc 29. Nếu n 3 = 3 thi n 2 = 37 vi n 1 # 11. NỂU n 3 = 5. thì n 2 Eà 53 hoặc 59. NỂU n 3 = 7 thi n a có thể là 73 hoặc 79. Ta có thâm 7 cách chọn, Vậy tọng cộng có 11 cách chọn. 12 . Theo già Ihiết a . = 5b + r = 3r 4 b VỚỊ 1 < b < 2 vã 1 < r < 4 . Tửdó 2r = 4b <=} r = 2b. Điéu nãy xay ra khi và chỉ khi (b, r) bầng (1,2) hoặc {2, 4). tức là a 0Ó thể bằng 7 hoặc 14. Ticb cần tìm lá 7. 14 - 93. 13 . A E B Ta có S DEZ = 2 S ữen = 4S mem = 20 (cm 2 } và 5 MEZN “ S DE2 “ S DMN = S DEZ “ S EMN = 1 5 í cm )■ Dù AB = 3BE và 8C = 2BZ nên S ABCD = = ' ÌS AED vả S ABCD = 2S BCD “ 43 ZCD củn S ABCD = 2S abc = 6S ece = 12S qez . T^C=C - $ADE - S ZCD - S BEZ = S ABCD ỉ 1 “ 3 ~4 _ Ĩ2 ) = s abcd. Suy rs = ^^DẼZ = (cm) vs = ^ v *y S MEBZN = S MEZN + S BEZ = 1 5 + 5 = 20 (cm 2 ).
  19. Ta chia 12 so thành 3 lớp. Lãp A gồm 4 số x^, x 2 . x 3 , x 4 có dạng 3k VỚI k - 1, 2, 3, 4. Lớp B góm 4 số y 1 , y 2 , y 3 , y 4 có dạng 3k I 1 vứi k = 0, 1, 2, 3. Lớp c gổm 4 số z r z 2 , z.y z 4 có dạng 3k + 2 vtìi k = 0, 1, 2. 3. Để chia 12 số thành 4 nhỏm, mỗi nhóm gốm 3 sổ có tổng chia hét cho 3 Ihi trong một nhỏm bất ki chỉ cỏ Ihể có 1 sđ hoặc 3 sỏ thuộc lớp A. Cách chĩa thứ nhất có dạng {x r x 2 , x 3 }; {x 2 , y 2 , z 2 }; {x 3 , y 3 , z 4 }; (x fll y 4 , z 4 }. Lúc đó có 4.4.4 =s 64 câch chọn nhóm đau tiên và ba nhóm còn lại được xác định. Cách chia Ihứ hai cỏ dang (x 1 , x^, x^; {x 4 ; y^ z 1 }; {y 2 , z 2 . ỉg); {y 3 , y 4 . z 4 }, Lúc dò có 4,4.4 = 64 cách chọn nhóm {x 4 , y r z,>. 3 cách chọn y 2 và 3 cách chọn {z 3 . z 4 } nên câ tất cả 64.3.3 - 576 cảeh chọn. Vậy tổng số có $4 I 576 = 640 cách chọn,
  20. Kí hỉẻu số b "theo sau" sế a là b = s(a). Theứ giả thiẽt b - s(a) nếu: 1 <b-a<9líltlâb-9< a < b - 1. o hoặc 10 < a - b < ie lửt là b 4 10 < a<b-18.H
  21. với mỗi số b mà 10 < b < 19 Ihl có 9 số "theo sau* b_là b - 9, b- s b - 1. Với mỗi số b má 1 < b < 9 thi có 9 số "theo sau" b lá b 4 10, b 4 11, .... b 4 10.
  22. Trang 9 so "theo sau" so b ban dầu ta chọn ra hai sổ a = 5(b) vả c = s(b). xót các trư&ng hợp sau;
  23. Nếu 1 £ a < 0 < b < 19 thỉ xảy ra n đối với a, b vá o đố! với b, c nên có1<c-a<{b-1)-(b“9) = &, do đố c = s(a). KHAI THAC BAI TOAN HINH HỌC M trong sách giáo khoa ĐẶU CỒNG NHO [GV. THCS CaoXuắn Huy. Diễn Chân, Nghệ An) Tử những bài toán trang sách giảo khoa nểu nghiên cừu sâu vá khai thác thi sẽ giúp các em học sinh củng cố, khác sãu kiến thức và khơi dậy tươuy sáng tẹo khi học môn toán. Chúng ta cùng khai thác một bài toán hinh học trong sách giáo khoa Toán 9 {tìéi 9, trang 70. Toàn ô, tập Ị, NXBGD Việt Nam nặm 2005). Bài toán 1. Cho hình vuông ABCD. Gọi I lá một điểm nấm giữa A vá B, Tia DI và tia CB cắt nhau ■ở K. Ki đưừng thlng qua D, vuông góc vội DI vá cắt đường thẳng BC tai L. Chửng minh râng ã) Tam giác DIL lá tam giác cán.
  24. Tđng — 1-H — í— không đổi khì I thay đổi trẻn ' DI 2 DK 2 cạnh AB, Lời
  25. Ta có ADAI - ADCL {g.c.g) Suy ra DI - DL, do dó ADIL cân tạl D.
  26. Áp dụng hệ thức vể canh vả đường cao trong tam giảc vuông DLK, ta cò 1 1 1 1 „, kJ . r? + —7 = T77 ► —77 - —7- (không đoi) DI 2 DK 2 DL 2 DK 2 DC 2 Nhận xét. Từ bài toản T hếu thây hình vuòng ABCD bàtìg hỉnh chữ nhật rìi AB = tBC !â có bài tõátĩ sau: Bải toàn 2. Cho hình chữ nhặt A6CD cớ A8 tBC. Trên cạnh BC lấy điểm E. Tia AE cât đường ihẳng lít CD tại F. Chửng minh rằng — ~ + — ... .. AB 2 AE 2 t 2 AF 2 Lời giải. Qua A kẻ đường thằng vuông góc với AE cắt dương thẳng CD tại G, Ta co AABE iADG (g.g) AE AB _ . ĂÉ => -~rzr = -77T = t =s AG=— . AG AD t A e Áp dụng hặ Ư 1 ÓÍC vé cạnh vá đương cao trong tam giác vuông ta có 1 L AG 2 f AF 2 1 1 1 1
  27. Ị, ,

    AD 2 " AE Ap* AS ■ t J 1 1.1

    Ị Ị Ị AB 2 - A£ 2 + t 2 AF 2 ' Cõc bạn hãy giải các bái íàp saụ nhó Bàl 1. Cho hình vuông ABCD. Gọi I là một điểm nầm giữa A vá B. Tia DI và tlá CB cắt nhâu ồ K. Qua D kẻ đường thẳng vuông g&c với DI, cắt đương thẳng BC tại L Trẽn tia dơi của tla DL lấy điểm E sao cho DE = DK. Gọi M, N lẩn lượt lá trung điểm của EK. LI. Chởng minh rầng M, N nằm trôn đường thẳng cố định khi I thay đổi trên cạnh AB. Bài 2. Cho hình vuông ABCD. Gọi I là mọt điềm nằm giữa A vả 3. Gọi M và N là các điểm đối xúng vởl I lần iưọl qua AC vả BD. Qua I kẻ dương thảng vuổng gốc VỚI MN tại H. Chúng minh rằng khi I di đọng tr&n AB thi dường thãng IH luỡn dí qua mọt điểm cố d|nh. • Nếu 1 < b < a < c < 1 9 thì xáy ra ("} đối với a, b và (') đối với h, c nên có 1 < c - a < (b 4 1 9) - (b 4 10) - 6, do đó c - s[a). • Nấu 1 < a < b < c s 19 thì xày ra () đối với a, b và () ddi vúi b, c nén có c - a > íb + 1 ữ) (b - 1 ) = 11 và c - a < 16, do đó a = s(c).
  28. Như vậy khi chọn hai số bất ki trong 9 số má chúng đều "Iheo sau - b thi luõn có bộ ba số thỏa mãn a = 5 (b), c - 5(b) và c = &(a) hoặc a - 5(c), ớo đó cò c| = 9.4 = 36 cảch chọn. Kết hợp vơi lấy 1 9 sở b ban đáu thi củ tãl cả 19.36 = 664 cách chọn. © ĐỀ THI CHỌN ĐỘI TUYÊN HỌC SINH GIỎI LỚP 9 A V V Nám học 2014-2015 Bải 1, 1) a) Ta cò M -2(Ja + 1) ab-1 Ị_ ab-1 -2-Jãb(-Jă + 1) - ^/ãb
  29. Ta có M = 4-4 Vab 4 MI f !.**
    /b I 4 1 Dáu bằng xảy ra khi a = b - A 9 Vậy MaxM = 9 khi a = b = -Ị;. 9
  30. Ta cỏ 3{2-&yJ(& + 2) z 3(4-5) 1 ■V5+V(V5-3) 2 ~ì/& + 3-^ - Do đóÁ =F-1. Bàt 2. ạ) Khí m = -1 t3 cớ -X - y = — 3 I X = s „ <=> -X -4y = 3 |y = -2.
  31. Ta cố -K + my = -3 Ị X = rry 4 3 mx-4y*=m + 4 [(m 2 _4)y = -2 tth-4, Suy ra hệ phường trinh cú nghiệm duy nhái khi .X „ m + 6 r „ -2 ni 4± 2. Khi dó X = y - — . m 4 2 m + 2 3 Do đỏ X 4 y - — <=> m - -3 {Vi m x±2) m
  32. Xẹt m = -2; m = 2: m X ±2. m Vòi m > -2 thi hê phương trinh có nghiệm duy nhât như trên. • Vãi m = 2 thỉ hệ phưtmg trình cỏ vỏ âũ nghiệm với -3 < 2y < 0, còn X i= 2y 4 3, Bải 3, 1) Gọi 50 xe trọng tải 4 tấn, 11 tấn lấn lươt Já X (xe), y (xe) [Điểu kiện X, y Ể N*). Ta có 4x 4 Iiy = 47. Suy ra y < 5 và (y - 1} : 4 nén y = 1 r từ đô X = 9. Vảy oò 9 xe trọng tài 4 tấn. 1 xe trọng tải 11 lấn.
  33. Ta có (ax 4 by 4 cz)(x + y4z)=0(Vìx+y4z = 0) => (ax 2 4 by z - cz 2 ) 4 xy(a 4 b) 4 yz(b 4 c) 4 zxfc + a) = ữ. Doa-t-b4c = ũ nẻn (ax 2 + by 2 4 CZ Z ) - cxy - ayz - bzx =0. =í (ax 2 4by 2 4CZ 2 )- xyz| — 4— 4 =0 { V -)
  34. Càc điểm o, M. A, N, I cùng nảm trẽn dưửng trôn đường kính OA.
  35. Ta chửng minh dược AI.AK = AH.AO = AM 2 = AB.Â.C. Suy ra AK = không đổi.
  36. Ta chứng minh được EM HM , MP HM 2MP 2HM MQ “ DQ : MQ - HQ MQ HQ 2MP HM trh Do đủ ” = -11. Sưy ra EM = 2MP r MQ DQ Vậy p lá trung điếm của MF". Vẽ AH BC. OK ± BC. Ta Cữ AM S ABC 1 . S QAB r 5 OAC OM OK Êọac Sqbc Sqbq Chúng minh tưdng lư rối cộng theo Lùhg VỂ các dâng thức đủ, áp dụng bất dẳng thức AM-GM la duợc đpcm, ĐỂ THI CHỌN HỌC SINH GIỎI MÔN TOÁN LỚP 7 QUẬN 9 TP. HỔ CHÍ MINH Năm học: 2014-2015 Thài gian /ám bài: 12Ũ phút Bải 1. (4 điềm)
  37. Tìm các số nguyên a. b, c, d sao cho [a - b| 1 |b - c| + |c - d| I |d - a[ = 2015. 72011 72013,1
  38. Cho A = B = Hãy @0 sánh A và B, 7 2013 +1 7 2015 +1 Bái 2. (.5 điểm) Tính . , ( 1 1 a A =
  39. + ■ 1009 1010 ' 4 JLJ + 1 1 2015 2016 10x 2 -3y 2 3 5 í 1 11 1 1 2 3 4 Ị Ị_J 2015 2016
  40. biẽt X, y, z * 0 vá X - y - z = 0. Bái 3. (4 điểm) ■ 3 D c
  41. Tlm ba số a, b, c bíẽl rằng VÉ abc = 20. y 12 9 5 Cj g 2 b Ị Tim ba sổ có lổng bằng 420; biet rảng y sổ Lhứ nhất bàng Sũ thử hai và bằng số thử ba. Bài 4. (4 điẩm) Cho lam giác ABC vu&ng tại A vời ACB =15°. Tr&n tia BA lấy điểm o sao cho BO = 2AC. Chừng minh ràng tam giác OBC cản. Bái 5. (2 điểm) Chú lam gĩảc ABC vuỏng tại A VỚI BD là đường phân giác. Đưdng thảng vuồng gốc với AC lai c cát ba BD tại E. Chứng minh rang chu vi tam giác ABD nho hơn chu vi tam giác CI3E. Bài 6. (1 điểm) Cú 10 hợp thuốc, mơi hộp có 10 gói. mói gói nặng 100 g. Bict rãng trọng 10 hộp đó có một hộp làm sai quy định, mỗi gói chì có 90 9 . Dùng một cái cán (loại cân đồng hố) vá chỉ cân một lần. hãy (im ra hộp nào chứa các gùi thuốc làm sai quy định. Giải toán qua thiẩ Bải 1(153). Tính . _ I 2 2 2 ? 100 + 5000 2 2 200 + 5000 99 2
  42. 1 — — T 99 2 -9900 + 5000 Lởi giải, xẻt k e H* La có (100 - k} 2 - (1Ũ0 - k).1QQ - 5000
  43. 1 Oữ 2 - 2,1 oo.k + k 2 - 1 QO 2 + IQQk + 5000
  44. k 2 - 100k + 5000. Lần lượt thay k = 1; 2; 3; ... ; 99 ta có I 2 - 1 00 + 5000 - 99 2 - 9900 + 5000: 2 2 - 200 + 5000 = 9S 2 - 9600 + 5000: 99 2 - 9900 -I 5000 - I 2 - 100 + 5000. Ta cố n . 1 a +99 2 2 2 98 a I 2 - 100 + 5000 2 2 - 200 + 5000 99 2 + 1 2 99 2 - 9900 ■ 5000 Mạt khác k a + (100 - k) 2 = k 2 + 100 2 - 2.100k k 2 - 2(k a - lOOk i 5000), Do đó -7— - 2. k 2 -lOOk 5000 Bài 2(1 53). Cho tam giác ABC cân Lại A. dương cao AH. Từ điểm D bất ki liên cạnh AB hạ DE VLiỏng góc với BC. Trên đoạn thầng HC íấy drểm F sao cho FC = EH. Qua c vẽ dưứng Ihàng vuông góc với AC cẳt AH tại G, Chửng minh rằng L>FG = 90°, Tứ già thiết, ta có HB = HC. Mặt khác FC = EH nẽn BE = FH và EF = BH = CH. Áp dụng đính lí py^ta-go, la có DF 2 + FG 2 = (DE 2 + tF 2 ) + (HF 2 + HG 2 )
  45. DE 2 + BH 3 + BE 2 + HG 2 ±± DE 2 4 BH 2 4 BE 2 - BG 2 - 8H 2 = B0 2 +BG 2 .(1) Xét AABG vả AACG có Suy ra 2A = 2 + 2 + 2 + . . . + 2 (có 99 sổ hạng là 2). 2.99 Do đó A = = 99. 2 Nhận xèt. Đây là một bài toán hay, nhiều bạn tham gia và giảa đúng. Các bạn trinh bây lốt: Lê Phạm Yến Linh, 6A8, ĨHCS Chu Vãn An. Ngô Quyén: Phùng Quang Minh, 9A1, THCS Hống Bàng, Hõng Bàng: Nguyền Binh Nguyên , 7C10, THCS Trần Phủ, Lê Chân. Hái Phỏng: Đường Minh Quán . 7C, THCS Bạch Liẽu, Yên Thảnh, Nghệ An Cao Thị Khành Linh, Nguyễn Trung Kiõn, Nguyễn Oàng Doanh, Bùi Nguyễn Nhật Minh, Trận Đừc Tùng, Nguyễn Hưng Phét, Ễ?B, Phan Lê Vân Nhi, Pham Hiấu Ngăn, Bùi Thị Minh Thư Phạm Yến Nhi, Nguyễn An Na. 7A. THCS Hoàng Xuân Hản, Đức Thù, H ã Tĩnh. PHÚNG KIM DUNG AB = AC. BG = CG. AG chung nên AABG = aACG (c.c.c). Suy ra ẢBG = ẤCG = 9Ữ Ủ . Từ đó vá (1) la có DF Z + FG 2 = GD 2 . Theo định tl py-ta-go cỉâo Ihi tam giác DFG vụộng tại F, suy ra DFG - 9Q r . Nhặn xét. Cú nhiểú bạn gửi bãi về tủa soạn, Các bạn sau cố lài giải tốt: Đỗ Quang Đàng, 7A. THCS vinh Yên, TP, Vĩnh YỄn, Vĩnh Phúc; Nguyền Đức Hiếu, 7C10, THCS Trân Phú, Q. Lè Chân. Hài Phông; Hoàng Mạnh Nghĩa. Lẽ Xưùn Toàn. 7D, THCS Lý Nhật Quang, Do Luông; Phan Hà Thanh, Nguyễn thị Kim Chi, ĩrẩn Sỹ Tiến, Nguyễn íhị Béng Bănỹ , 7C, THCS Bạch Liêu, Yên Thành, Nghệ An; Phạm Hiểu Ngân. Phạm Yến Nhi, Nguyên Hâi Ly, Phan Thị Thu Hoài. Phạm Ánh Nguyệt, Nguyễn Ngọc Ánh r Bùi Thị Minh Thư, 7A. THCS Hoang Xuân Hán, Đửc Thọ, Há Tĩnh. HÓ Quang vĩnh Bai 3(153). Chủ X, V, 2 thỏa mãn 4x 2 +.4z 2 = 17 4y(x t 2) - 5 2ũy 2 +27 - -18z, Tính giá tr[ của biểu thirc p = 30x + 4y + 2Ũ13z. Lởi giải. Hệ phương trình có thể viết thánh 4x 3 + 4z 2 -17 = 0 Ị -8xy- I6y f10 = ũ 20y 2 4l6z 427 = 0 Cộng theo vế cảc phương trình trorlg hệ, tã được 4x 2 4 4z 2 - 8xy - 16y + 20y 2 4 1 Bz 4 20 = 0 <=> 4(x 2 - 2xy + y 2 ) T 4(4y 2 - 4y + 1 ) + 4{z 2 + 4z + 4) = 0 » (x - y) 2 * {2y - I) 2 4 ịz 4 2Ỷ = ữ x-y-Q <-? 2y-1 = 0 4-> z*2 = ũ 1 Y = -ì ỉ 2 z = -2 Thử lại, ta thấy (X, y. z) =1 X -2 j (hỏa mãn hệ dã chữ. K 2 2 ) Do đó p = 30. ị +4 +2013.Í-2) = ”4000. 2 2 Nhặn xéỉ. Cú nhiều hạn giai đủng thao cách trên. Thực chất đây là hài toán giãi hệ phương trinh. S3U dó ta Ihay giẳ trị của X, y. 2 dể tính p. Cảc hạn sau dây có hài giải tốt' Hoàng Hè My Vũ Hoàng Kiện, SA, THCS Chu Vân An, Nga Sơn. Thanh Hóa. Ngụyễn Trung Hiểụ , 7A3, ĨHCS Lâm Thao, Lâm Thao. Phú Thọ; LỂ Đinh Thánh, 7D, THCS Lý Nhật Quang. Dô Lượng. Nghệ An; Nguyễn Huy Quang, Phạm Hiếu Ngân. Phan Lê Vân Nhí' Phan Thị Thu Hoài r Phạm Ýến Nhi' Hoàng Tuấn Tài. Nguyễn An Na, Nguyễn Ngọc Ảnh, Bùi Thị Minh Thư, Trẳn Thị Kim Oanh, Théì Thi Thu Sang. Nguyễn Minh Anh, Lê Thị Hàng Nhi, 7A, THCS Hoàng Xuân Hãn, Đúc Thọ, Hà lĩnh; Nguyên Cống Huấn, Chu Vãn Việt, Tạ Nam Khảnh, 8E1, THCS Vĩnh Tương, Vĩnh Tường, Vĩnh Phúc; Vù Anh Đút, ÊC, THCS Nguyễn Cao. Qué Võ, Bắc Ninh, NGUYỄN ANH DŨNG Bãi 4(153). Cho các số thực dương X, y, z. Chừng ì-u r~K . / - y" ] % > 3 s V z 4 3x Vx-3y Vỵ+3z 2 Lơi gi ái. Áp dụng bất đẳng thửt AM-GM ta củ X 1^2 ỉ ĩ" - ị —

    Z43x 4 V z+ 3x 4 " Y z 4 3x " Tương tự

    rỊ> CxI;ĩ t Ị ỉ .CZI.

    X4 3y 4 \x43y y43z 4 \y 4 3z Do đó

    4 1* 1

    rr~ 1( z 4 3x \ Ịx 4 3y V y 4 3z

    3 , X y , 2 í — 4 " — 4 4 I 4 z+3x X43y y+3z Ta sẽ chủíng minh — - ị — z + Zx X4 3y z 3 — — S“, y + 3z 4 Thật vậy, ta đặt a = — ,h = “,c = -=> abc = 1. y 7 X Khi đó bẩí dẳng thức trà Ihảnh 1 1 1' 3 ■ ” + ” "7 C4 3 3 + 3 b + 3 4 « (3 + 3)(b + 3}+(b +3)(c +3) 4 (c 4 3}(a 4 3) < |(a 4 3Kb +3)(c -3} 4 3(a 4 b 4 c) + 5{ab 4 bc 4 ca) > 24, (1) (vỉ abc = 1) Bat đẳng thúc (1} luôn đũng vì a 4 b 4 c > 3^abc = 3; ab 4 bc 4 ca > 3^a 2 b 2 c 2 - 3. Vậy bất đing thức được chrìhg m?nh và dẩu "=" xây ra khi X = y = 2. Nhận xét. Đây là bái toán không quả khỏ vi thể có nhiều bạn tham gia giải bài. môt số bạn biển đỏi dái dòng mới di đẽn kết quả. Những bạn sau dầy có lởi giẫá đúng và ngân gọn: Trán Đứt Duy, 9A4. THCS Yèn Lạc, Yên Lạc; Kim Huy Hoàng, 9B. Trấn Binh Minh, ÔEl, Nguyễn Hoài Phương, SE1 r THCS Vĩnh Tướng, Vĩnh Tưàng, Vĩnh Phủe: Nguyễn Hữu Ttvng Kiên, Nguyễn Xuân Kiên, 8A3, THCS Lãm Thao, Lảm Thao: Phan Thị Thuỳ Linh. 9A2, THCS Giấy Phong Chãu, Phù Ninh; Hã Ngoe Khang, &tì, I HCS I hanh Hà, I hanh Ba. Phủ Thọ: Chu Thị Hàng. 9A1 , THCS Yên Phong. Yên Phqng. Bắc Ninh; Phùng Quang Minh, $A1. THCS Hâng Bâng, Hồng Bàng, Hảỉ Phỏng, Cao Việt Hải Nam, BE. THCS Dặng Thai Mai, TP. Vinh, Nghệ An: Trương Caữ Minh, 9AG, THCS cẩu Giấy,’ Cầu Giấy, Há Nộỉ. CAO VĂN DŨNG Bải 5(153). Cho mợ) bảng gổm 20IG X 2015 õ vcũng nho. DiỂn vảo mỗi ủ một sổ 4l huỆõ -1. Cạnh mãi tỉórtg i ghi tích các số của còng đò vã gọi lo X . ũựới mãĩ CỘI i ghi tích các số của cội dó vã gọi là Yj (ĩ - 1 . 2. 3, 2015), ịxem hình). Chứng 201 & y, v 5 '2015 ntỉỉĩỉỊ. L^nựng minh rằng 4030 số X . y : nhận dược luồn có tang khác 0. Lời giải, Giâ sử tổng cùa 4030 số Kị. y, bằng 0. Ta cú X 1 + Xj + . . . X^OỊS + + Vg + ‘ ■ - + yãOlâ — mà mồi sđ Xj vá Yj đếu bằng -1 hoặc 1 nên trong 4030 SỐ X., Yj CÓ 2015 sổ bằng -1 vả 2015 số bằng 1. Suy ra tích x r x 2 . . . x 2ữ1 g-y^yg . . . y 2&lS = -1 (vì có một sở lè số thừa sỏ bằng -1). (1) Mát khac x r x 2 ... ^2015 = 1 , l 2 ^2015 lích của lất cả các số trong bàng). Suy ra x .^2 ... x 2ữí5 -y ! .y 2 ... y 2 0 15 = (! ... X 2Ũ15 ) 2 = 1 (mâu thuần vài (1)). Suy ra điểu gỉả sử tã sai. Vậy tổng của 4030 số X , yj luôn khác 0, Nhặn xẽt. Có nhiểu bạn gửi bái dến. tòa soạn, hầu hết các bãi dlu giải đúng. Các bạn sau dây có lời giải tợt; Tran Hữu Dừc Mạnh. 0A, Cao Khắc Tàn, 7A, '\ HCS Qậng 1 hai Mai, TP. Vinh; Ngựyễn Đinh Quần , ÊB. THCS Bạc Lièu. YỄn Thành; Lẻ Xuán Tùần. Hoàng Mạnh Nghĩa, Lê Đình Thành. Nguyên Sỹ Trọng, Nguyễn sọ Quyến, Nguyễn Thị Hương Giang, 7D, THCS Lý Nhặt Quáng, Bõ Lương, Nghệ An: Đãng ũwanh Anh, &A. THCS Nguyễn Chích, Đóng Sơn. Thanh Hóa; Ngõ Đặng Cõng Vinh, ĨB9, THCS Chu Vãn An, Ngõ Quyến, Hải Phòng; Trắn Quang Tài, 7A1, Đồ Thúy Hổng, 8A1, THCS Yẻn Phóng, Yên Phong; Trân Minh Quấn, 7A1, THCS Từ son. Từ Son; Tạ Viết Hoàn, 7C, THCS Nguyên Cao. QuỂ Võ. Bẳc Ninh: Dương Quang Tùng, 9A4, THCS Yên Lạc,. Yên Lạc: Lẽ Ngoe Hoa. ỠE1, Trần Thề Vinh, Đỉnh Thị Thanh Huyên, Nguyễn Hoài Phương, Đinh Van Thài, Lê Anh Dũng. Kim Thị Hổng Lĩnh, 9E1, Nguyễn Kim Dãn, Búi Anh Vũ, Kim Huy Hoàng, Phan Huyền Ngoe, Nguyễn Vũn Huấn, Nguyễn Vàn Hoảng. 0B, THCS Vinh Tuớng. Vĩnh Tuộog; Phạm Ngọc Hoa. &A1, THCS Sông Lô; Lê Hồng Nhung, 7A, THCS Vĩnh Yên, TP VSnh vẻn, Vĩnh Phúc: Nguyên Đức Tân, Cao Đức Học. Nguyền Chi Cõng, 7A3, THCS Lảm Thao, Lãm Thao; Nguyẻn Thị Ngọc Huyền. 9A, THCS Hùng Vương, T.x Phú Thọ, Phú Thọ Nguyễn Nhặt Linh. SE, THC5 Lẽ Quý Đôn, TP. Tuyên Quang, Tuyên Quang. TRỊNH HOÀI DƯƠNG Bái 0(153). Cho đường tròn {Ọ; Rị và dây BC - R v3 Cữ định. Trên cung íởn 8C lấy đif?m A Ỉỉẩt ki sao cho tam giác ABC nhọn, Vế các đường cao Bb và CF của , " . 1 14 lam giác ABC. Chúng minh rang — 4 — > — BE CF 3R Lòi giải. Gọi F là hình chiếu của c trên AB. Vì BC = nên BAC =ÊŨ ữ , do đó 2AF = AC, Theo định Ễí Pjrla-go, ta cỏ BC 2 = FB 2 4 - FC 2 = (AB - AF) 2 4 AC 2 - AF 2 = AB 2 - 2AB.AF + AF 2 4 AC 2 - AF 2 = AB 2 t AC 2 - AB.AC. Suy ra 4BC 2 = (AB + AC} 2 + 3(AB - AC) a (AB + AC) 2 . Do đó 2BC > AB + AC, Ta có 1 | 1 1 1 __2_j BE + CF BA sin 60° 1 CAsin60° s\ BA CA j Tử SỐ tháng 9 năm 2015 , Cõng ty cổ phấn Dịch vụ Giảo dục Việt Nam sẽ tạng các khóa học trực tuyến trẽn website: hocmaLvrĩcho cãc bạn học sinh duạc thưủng toong các chuyên mục và các bạn học sinh được khen trong chuyên mục Kết quà thi giải toán que thư, Các bạn học sinh sau khi nhận được mã cụng cấp thỉ đặng ki tại địa ghỉ: ttìỌẸ.hocmaị.vnÃoantuoiího (Xin Hèn hệ $ĐT 0955464644 để đươc giải đáp). <£ M CÓ CHIA HÍT KHÔNG? Bải toán. Tại mồi đĩnh của môt đa giác íĩéu 11 cạnh la ghi một số bất ki (rong các số 31; 32; 61; 62; 91; 92; 331; 332; 361; 362; 961 (mỗi so dùng dứng một lần). Bạn Toàn nói với ban Thơ rằng "Luồn tân tại ba đỉnh cũa đa giác là ba đĩnh cùa một tam giác cân và lồng các sọ ghi Irệr các đình của tam giác đớ lã mộl sỏ chĩa hỂl cho 3". Hỏi Toán nôi đúng hay sai? NGUYỄN DỰC TẮN ị TP. Họ Chỉ Minh) 2M5nEB > KET QUA TRẠN DAU (TTT2 số 153) Gọi các học sinh thi đấu là A 1 , Ag,. . .,A t1 , Â 12 vá số điểm tuơng ừg \à a 1 > a 2 >■■■> a 1t > a 1£ . sỏ trận đấụ của 5 học sinh ít đicm rhất khi đẩu vâi nhau lá 10 trận với lổng số điểm lâ 2Ũ điểm. TÍT dô vè gia thiết thi - a 8 + + a io +■ a n + a T 2 -
  46. Nếu a 2 = 21, lứt lả thẳng 10 trận và hòa 1 trận, lúc dó a 1 = 22. tức là A 1 tháng lất câ 1 1 trặn nẻn tháng cầ A^, dần đến mâu thuấn. Vậy a 2 = 20 = a g + a 9 + a 10 + a n + a 12 , túc lá Ag, Ag, A 10 , A^, A 12 đấu với bết ki bạn náo từA 1 đển Aj đểu thua. SỂ trận đấu cùa 3 bạn A 10 , , A 12 VỄÍ nhau là 3 trận nên a 10 + a^ + a 12 ầ. 6. suy ra a s + a & < 20 - 6 = 14, Nếu a 9 i7 thì a g > ô, không Ihỏa mãn, Theo giả thiết a g > a 1ữ + a 1T + a 12 > 6 nèn a g = 6 = a 10 + a 1f
  47. 9^, tức là A l0 , A ir A 12 dẩu với bất ki bạn nào từ A 1 đen Ag đéu Ihua, cỏn A 0 chĩ tháng 3 trận đô' vở: A 1Ữ , A ir A i£ vã thua tất cả các bạn còn lại, do dó Ag thua Ag và a a = s, Nhặn xét. Rất liếc lá không có bạn nào giáí đúng bài toán náy, Phấn thu6ng xin gác Èại kì sau. ANH COMPA 2 1 „ / 1 1 = -== — — (BA+CA) -Ị-+-V V3 ba+ca' (ba ca -ị ■ —Ị—2 JbÃÕÃ .2,/ - - Ị- - s 2BC VBA.CA 4 14 14 1 s BC 2BM 2BOsJn60° 3P (Theo hát đầng (hức AM-GM) Đẳng thức xày ra khi vá chĩ khí A6 = AC. Nhện xét. Có nhiếu bạn tham gia giải bài. Xin nếu tèn một vãi bạn có lởi giải tổt: Nguyễn Váfì Hoàng, Nguyền Kim Dân. Kitn Huy Hùàng. Hạ Trung Hiếu. Phan Huyển Ngọc, 9B, Nguyễn Hoài Phi/ơng, Là Anh Dũng, 9E1, THCS Vĩnh Tưàng, VĩnhTương; Dương Quang Tùng, Nguyễn Vãn Hiếu, 9A4, THCS Yên Lạc, Yên Lạc, Vĩnh Phúc; Cao V7ặf Hải Nom, 9E, THCS Đạng Thai Mai. TP. Vinh, Nghệ- An. NGU YẾN MINH HÀ ĐƯƠC THƯỞNG KÌ NÃY Thi giải toán qua thư
  48. . ạ Nguyễn Đữc Hiếu . 7C 1 ơ. T HCS Tran li... Phu Q. Lê Chần. Hàl Phóng; Cao Hải Nam, 9E, THCS Đặng Thaĩ Mai, TP. Vinh. Hoàng Manh Nghĩa, Nguyễn Sỹ Trụng. Nguyền Sỹ Quyến, 7D. THCS Lý Nhật Quang, Dô Luông. Nghệ An; Lê Anh Dũng. 9E1. THCS Vĩnh Tưàng. Vĩnh Tuàng; Đương Quang Tùng, 9A4, THCS Yên Lạc, Yên Lạc: lê Hồng Nhung, 7A, THCS VTnh Yên’ TP. Vĩnh Yên. Vĩnh Phúc. Phan Lõ Vãn Nhi. Phạm Hiểu Ngăn, 7A THCS Hoàng Xuân Hân. Đức Thọ. Hả Tĩnh: Vũ Hoàng Kiên, 6A. THCS Chu Vãn An. Nga Sơn. Thanh Hõa; Nguyễn Trung Hỉếư, 7A3, THCS Lâm Thao, Lâm Thao. Phú Tho: Chu Thi Hàng. 9A1, THCS Yên Phong, Yên Phong, Bắc Ninh: Nguyễn Nhật Linh , 8E, THCS Lẽ Quý Đòn. TP.Tuyên Quang, Tuyên Quang; Trương Cao Mỉnh, 9A6, THCS Cầu Giấy, Cẩu Giấy, Ha NỘI. B à Sarah - vợ mộl doanh nhản nổi tiếng - hổt hoảng gọi điện nhở thám tử Sélôccôc tim giúp đôi hoa tai đẩí giá. Như moi khi. thám tử vui vẻ nhận lõi vả khuyên bà giữ im lăng, đừhg vội lãm to chuyện. Khoảng hơn một giờ sau, thâm tử Sẽíõccôc đã có mệt tại nhá bồ Sarah.
  49. Nào, bà hãy kể lại mọi chuyện cho tôl nghe! Cứ bình tĩnh kể nhé! VỘI vàng là hay nhẩm đấy!
  50. Vâng! cảm dn ông đã tới! Chuyện thế nảy ông ạ. Trưa nay, lủc khoảng 11 giờ. tỏi tháo đôi hoa tal ra để đi bơi. Một lủc sau, tòi vào nhà thì không thấy đâu nữa.
  51. Bà bơi ở bể bơi gsa đinh trong vữờn nhà chữ gì?
  52. Vàng.
  53. Bã bơi bao lâu?
  54. Chắc chỉ 20 phúí thôi vi nưức hơi lạnh.
  55. Bá để hoa tai ò đàu?
  56. Tôi để í rên kệ nhà tắm â cạnh phàng của vợ chổng tôi.
  57. Từ bể bơi lèn, bà vào thẳng nhá tẳm hay cởn làm gỉ nữa? "Tòi váo kiõn nhà lầm. -Chác đôi hoa tai đểt giá lắm thì bà mớì phải nhở tô I đúng không?
  58. Vâng, đủng vậy. Vừa đểt, vùa là một kĩ niệm vô giá của tôi f thám tử ạ.
  59. Khi bà đi bơi, trong nhà cố nhửrsg ai?
  60. vẫn như mọi ngảy thỏi. Bá Kerry, anh John và cậu Aeron. Họ đểu d nhá tòi lâu nam rồi, tôi rất tin tưởng và hoàn toàn không nghi ngờ họ,
  61. Nhưng tỏi vẫn phải gặp từhg người. Bà goi giúp tôl chứ?
  62. Vàng, tất nhiên rồi, Tôi sẽ gọi họ tới đây. NGUYỄN QUANG HIỂU {mì., mc$ Nguyễn Bàng Đạo. TR Bấc Ninh. Bắc Ninh) Đẩy tiên Eá bà Kerry, nội trợ: “ Bà đã làm gl trong lúc bả Sarah đi bơi? -Tôi à trong bếp chuẩn bj bữa trưa. Tôi luôn Cỗ gắng để khi bà chủ ngoi vào bàn ãn thì mọi Lhứ vẫn nóng sổi. Tiếp theo là arth John - làm vườn kiêm bảo vệ:
  63. Lúc bà chủ bơi, anh biết chứ?
  64. Vâng, bể bơi trong vườn mà.
  65. Anh đă lảm gỉ, ở đâu lúc đố?
  66. Tỏi ngói thư giãn trên ghế đá trong vưãn và tranh thủ dọc quyển truyện mớỉ mua.
  67. Truyện gi thế?
  68. Harry Potter.
  69. ỏ, truyện đỏ rất nổi tiếng nhung tôi chưa đọc. Của tác giả náo anh nhĩ?
  70. Cùa nhà vân nổi tiếng người Mỹ, chuyến viết truyện trinh thảm, ồng ta tén là J.K. Rovvling. Cuối cùng là Aeron, Lái xe kiêm thợ sửa chữa.
  71. Anh đã làm gi, ỏ đàu trưa nay, trước bữa cơm?
  72. Trang lúc chờ bà chủ vể ãn trưa, tôi tranh thủ hướng dẵn bà Kerry sử dụng mấy món đồ gia dụng mới mua.
  73. Anh Lá người chọn mua à? -Vâng, tất nhiên róì.
  74. Anh mua cua hãng nào? -Toi mua đố cửa Sharp, một hãng nổi tiếng của Nhặt Bán, Sau đó. thâm tử sẻtôccôc gặp riẻng bồ Sarah:
  75. Tôi đã tim ra người đáng nghỉ trong vụ này roi. Tất nhiên, để kết luận chắc chan thi phải tiếp tục tim hiểu. Bả Sarah hết sức ngạc nhiỄn khi thám tử nghi ngờ một trong ba người thân cặn vtì bã nhất. Bá cũng khõng thể đoán ra người đó lá ai. Các thảm tử Tuổi Hổng củ thể giúp bà không? Theo các bạn, vì sao thám tử Sêlôccôc lạt nghi người đó? tmr nr LỞI KHAI (TTT2 Số 153) Thám tử Sèlôccõc nghi ngờ lới khai của chính ông Giêm bởi vi nếư cẩu dao diện bị ngẳt thi âng ta không thể xem TV Ềiẽn tục trong 2 tiếng được. Ki nảy bạn náo cũng phản đoán chính xác, tuy nhiên, những cảu trả lới rành mạch, súc tích, chặt chẽ thi chưa nhiểu lắm. Các bạn hãy chú ý traư dổi thẻm khả náng diẻn đạt của mình, sao cho cử đọng mả đủ ý, ngản gọn mà dể hiểu nhé. Phân thưởng sẽ được gửi tới: Đinh Vãn Thái Sơn, 61, THCS Lê Quỹ Dôn, Nghĩa Do, Gầu Giấy, Hả Nội: Đinh Đinh Hải Việt , 6A9, THCS Chư Ván An, Ngô Quyển, Hả ị Phòng; Đặng Hổng Phúc, 6E, THCS Dạng Thai Mai, Vinh, Nghệ An; Nhốm bạn Phan Vãn Nam, Cao Tam An, Nguyễn Thị Việt Trà , Nguyền Võ Hưng, Trần Hà Nhi , Nguyễn Trí Dũng, Hà Trung Chiến. 6B. THCS Hoàng Xuân Hãn, Đức Thọ, Hà Tĩnh; Ngõ Võ Hoàng Việt, 6A3, THCS Thực hành Sài Gòn, P.4, Q.5, TP. Hố Chí Minh Thám tử Sêlôccòc 9ẾN V- tải +IẾNE +MN \\ Bài 65: Đà Nang 1 nóng hơn Hà Nội ThS. NGUYỄN VÙ I.OAN LTS. Nếu bsẻt tiếng Hẩn bạn sè:
  76. Hiếu các tù' Hán Vsệl, sử dụrvg tốl hơn tiếng Vièl của mình. Trong kho lừ vựng liếng Vièt Tấl nhiều từ Hồn Việt
  77. Đọc õược sách cồ, van bia bang chữ Hãn vã Hãn Nôm, Ihẽíri hiều vãn chuông, lịch sù' nước Nam minh
  78. Hléu ngồn ngũ' mà cử 5 nguửi trên thế giói cỏ hơn 1 nguôi dùng. Dễ dâng hợp lác, lám ãn vời các nước vả vùng lãnh Ihò Trung Quốc, Hống Kòng. Dãi l oan, Singapore vá cá Nhặt sàn Hán Quổc. Néu biềt cà liếng Anh vã tiềcg Hán thi thật iá tuyệt. Từ mói. ditú: [địa cJỒJ bán đỗ H iSxiãtiãn: [hạ thiênj mùa hẻ ÍUặícngịĩng: [phongcãnh] phong cành JfêHfcShùnhuà: [Ibuận hỏa] Thừa Thiên - Huc mmhă imn: [hài than] bãi bien $1 Xiàiìgãng: [hiộn câng] Đà Nằng ĩỄ,ịìn: [cận] gản iìlyuàn: [viễn] xa bùdélíão: [hẩidẳcĩíèu] cực kì, võ cùng, vuợt trội f# dc: [dac] (trợ tử) Mầu câu.
  79. A: #«««*, w> (Shíij ià \vò xiàng qú Xiángấĩig, ni ne?) Nghỉ hè mình muốn di Đá Nang, còn cậu? B : íi -Ế ® "dĩ ( Wõ X i ắng q ù H LL7.hlm í ng sh ì . ì M i n h muốn đ i thà nl 1 phổ Hồ Ch í Mi nh .

    Ả:mầìtặiầmwn0. áiiỀmmmỉ (Xi àn găng bỉ Mú/himmg fihì jin dcduõ, nĩqủguò Xiàngảngma?) Đà Nầng gần hon thành phồ Hù Chí M inh nhiều, bạn dà đến Đá Nầng chưa? T 11 (VVồqùguòXỉỉmggănghéShìinhuàshi le.) Mình đã dển Dà Nảitg và thảnh phổ Hué rối. A: íKíllĩlĩ ? (Shủnhuả 5 hi dá bủ dà?) Thánh phổ Huề cỏ to lớn không? B: ®Hfc'fỊf T> j!v "Ấ (Shùnhuà shi bù lài dả.) Thành phó Huékhồrtg lo lấm.

    . ầsKãttkiiiĩỉ. *jttt«ẾWTWE#í. jfíèT. Muyfs*sf, (Mary hé gẽgẽ jTfmiản xiàtiãn yảo qù Xiàngăng, Tãmen xiàng qủ Xiảngàng hẻ Shủnhuà, xiingiìng hê Shùnhuà dõu yõu piầoỉiang de íengjĩng. J'iệiie yào qù Húzhìming shi. Tã gẽn péngyộu yiql qù hầitãn. Xiàngảng bĩ HÚ7hỉmíng shì jìn déduõ. Shủjiảkuảiyảo kãi.shĩle, Mary hc gcgejicjicdỡu gãoxìng dc bùdélião.) M ùa hò năm nay Mary và anh trai muốn di E>à N^tig, Hụ uiuổn di H ué và Đà Nãng. Huế và f)ả Nang đêu lá nơi cồ phong canh đụp. Chị gái Mary muốn đi thảnh phố I lô Chí Minh, chị ẩy vả bạn cúng di đến bó biến, Đà Nầng gần hon thành phố Hồ Chí Minh nhiều. Kỉ nghỉ hè sắp bdt đầu rồi, Mary vảanh trai, ch Ị gải bạn ầy vô cùng sung sướng. (Kỉ sau đảng tìêp) 1 . Lines LINES ĐƯỜNG THẲNG vũ KIM THỦY In geometry, the word ỉỉne reĩers to a straight li ne that extends vuithoLit enđ in both directions. The line can be reỉerred to as line AB Qr line d. A I d_ The part of the line From A tủ B is called a lihe segment. A and B are the endpoinis of the segment- The notation AB is used to denote li ne segment AB and AB IS useđ to denote the length ofthe segment.
  80. Intersectmg Lines lf two lines inìerseci, the opposite àngleâ are called vertical angles and ha ve the same measure. ZAOB and ZCOD are vertical angles and ZBOC and ZAOD ãre vertical angles. Also, x ũ + y° = 1B0 Ữ ,
  81. Maths Terms geometry ỉi ne straight ỉine extend diroction part intersect vsrticaỉ angies hình học đường, đường thẳng đưdng thẳng mở nộng , kéo dải hưởng phẩn cắt các góc đối đỉnh 4, Dựa vào gdi y từ vựng ỏ mục 3, bạn hây đỊch mục 1 vả 2 sang liếng Việt để học tốt Toán bằng tiếng Anh. Bãi dịch tốt sẻ có thưởng vả được nèu tẻn trèn báũ, PHÂN CHIA NHIỀU HÌNH CHỮ NHẬT m ĐỂ GHÉP LẠI THÀNH HÌNH VUÔNG (Tiếp theo kì trưõc) TS. NGUYỄN VIỆT HẢI (Há Nội) B ài này liếp ỉục xèí việc Phản chỉâ nhiều hình chữ nhật đế ghép lại thành hình vuông. © Định 1ỉ 3 {Bái toán Py-ta-go). Tồn tại cách phên chia 2 hình vuông ra thanh 5 đã giác để ghép chủng lai thảnh rnỏl hình vuông. Chứng minh. Đặt hai hinh vuông ABCD vã EFGH cẩn chia sao cho điểm B trùng điểm E, điểm B nấm gitta hai điểm A vá F, tia BC và tia EH trùng nhau. Trên tia AẼ (ấy điểm p sao cho ÀP = £F thi AB : PF. Trẽn tia BC lấy điểm G sao cho BQ = BC 4 CQ - BC + BH thì HQ - BC . Dề thấy ửlDAP - APFG - AQHG - ADCG nên DP - PG - QG - DQ, ẮDP - FPG => DPG - Sữ ữ do đó DPGQ là hình vuông (hình 6). Từ đủ haỉ hình vuông ABCD và EFGK (già sử AB > EF) dược phàn chia ra 5 đa giác DAP, DPMC, PBM. GHM. EFGM, rối dịch chuyển các đa giác DAP, PBM, EFGM theo thứ tự đến vị trí AQHG, ADNV, NCQV dể ghép thánh hình vuông DPGŨ. - Cách khác: Đạt hai hinh vuởng ABCD vá EFGH cần chia sao cho điểm A trùng với điếm F, điểm A nằm giữa hai diểm E và 8, tia AD và tia PG trừng nhau. Trân tia EF lấy điểm p sao cho EP AB thi EA = PB. Dụng hình vuông HPCQ thi S ABCD 4 ®EFGM = , ®HPCQ - Chứhg minh tương tự. Chú ý ràng nếu hai hình vuông ABCD và EFGH bằng nhau thi cách náy trùng với cà ch trong chừng minh định lí 3. Trên hỉnh 6 kè PJ vuóng gỏc với CD tại J, kẻ- GK vuồng góc với PJ tại K thi CJ = PB CH nên CHKJ la hĩnh vuông' cạnh HC = d. DÃ thấy KG = HG = CD = JP = d 4 b. Te thấy bốn tam giác vuông bằng nhau: AKPG = iHGQ = ACQD = AJDP. Vậy DPGQ lá hình vuởng dược ghép tư hỉnh vuông HCJK cạnh d vá bốn tam gtác vuông bằng nhau. Ghép tửng cặp hình tam giác đó được hai hinh chữ nhải PFGK cạnh lử b FG vá d 4 b = PF = AB. Như vậy ta đằ chứhg minh được định lí 4 sau đằy, © Đỉnh lí 4, Tổn tại cách phân chia một hinh vuông cạnh d và hai hinh chữ nhật chỉéu rộng b, chiều dàl a = b -I- d ra thành & đa gỉảc dể ghẻp chủng lại thảnh một hình vuông với diện lích bằng c 2 = d 2 4 2b(d + bj. Gọi c lá cạnh hình vuông lân DPGO thỉ la có c 2 = d 2 + 2b(d 4 b) - d 2 4 2db 4 b 2 - b 2 = (d 4 bj 2 4 b 2 = a 2 4 b 2 , như vậy S DPGQ = S ABCD 4 S BFGH (hình 6), nhung cách giải trong định li 3 (ch«a thánh 3 tam giác và 2 tứ giác) khác vởi cách giải (rong định lí 4 (chia Ihành 1 hình vuông và 4 lam giác). Ngươi ta đa thấy hình gốm một hình vuông cùng 4 tam giác vuông tạo thành một hĩnh vuông lởn trên tầm dá váo khoáng năm 1000 (rước Gông nguyên do nhá toán học Ãn Độ Bhaskara tim ra. Việc phân chia nhiêu hình vuông quy vể phản chia lần lượt hai hìi Bại toán 3. Háy phân chia một hình ghũf nhật VỚI chiểu rộng ã và chiều dái b ra làm nhiêu đa giảc đề ghép lại thảnh một hình vuông Irong mSi trưòng hợp sau đây:
  82. b = lõa, 2, b = 13a
  83. b=17a. 4. b= ápa. Lời giai,
  84. Cáctĩ 1. Ảp dụng dịnh lí 2 cho cách phân chia ra 5 tía giác. c ách 2, Nếu dùng định lí 4, từ 10 = 2 2 + 2.1.(1 2) Két hình vuông cạnh 2 vá 2 hình chữ nhật kích thước 1,3 = 3 {hinh 6) cho cách phan chia ra 5 đa giác. 2, Tử 13 - I 2 + 2,2, (2 + 1) Két hình vuông cạnh 1 vá 2 hỉnh chữ nhật kích ihước 2.3 = 6.
  85. TCí 17 = 3 2 + 2.1. (1 + 3) Két hình vuông cạnh 3 và 2 hỉnh chũr nhật kích thước 1 .4 = 4, 4, Tií 20 = 2 2 + 2.2, (2 + 2) xét hình vuông cạnh 2 vá 2 hỉnh chữ nhật kich (hước 2.4 s. Việc vẽ các hinh này dành cho bạn đọc. Dựa váo 4 định lí trẽn, kết hợp vội việc xét các hình bâng nhau, ta có nhiếu cảch khác nhâu phân chia hai hỉnh vuông ra các da Ọtác dể ghép tại ỉhànb một hlnh vuông, xin giởi thiệu mữt số cảch má sò da giác dược phàn chia ra kh&ng quá 5, việc chứng minh xin dành cho bạn dọc. Cách 1 Hình 6 trong chứng minh dịnh li 3. Cá ch 2. Dụng hình vuong như đã nói trong chúng minh đinh li 4. Cách 3. Hình 7. Đạt hai hĩnh vuông ABCD và EFGH (AB > EF) sao cho điểm F là tắm của hình vuồng ABCD, đổng thói EF // AB. Glả sử hình Vuông EFGH nằm trên nửa mát phàng chủa dỉểm D bờ AF. Trén cạnh BC láy diểm M sao cho BM = FG. trẽn tia CD fày các điếm p. Q sao cho CQ = FE vá CP- CQ 4 QP = FE + AB. n Dựhg hình vuông PAMN. Dựhg duủhg thầng đi qua
  86. cắt AD, BC tưưng ứng tại J, K sao cho JK ti AM. Dựng đường Ihẳng đi qua F. cát AB. CD tương úhg tạl T. s sao cho TS ,v AP. Các đường tháng GF. HG. EH. FE cái AM, MN. NP. PA tương Ong tại X, Y, u. V thi các điểm náy_theo thử tự Eà trung điểm của AM. MN, NP. PA. Dễ thấy các tam giàcABM, PQN, ADP bằng nhau, các da giác FJAT, FTBK, FKCS, F$OJ theo thử tự hằng các đa giác NU HY, PVEU, AXFV, MYGX. Chuyên cáo đa giác FjAĨ, FTBK, FKCS, FSDJ theo thừ tự đến chổ NUtìY, PVEU. AXFV. MYGX, lủc đủ = S ABCD + S EFGH . Cãch 4. Hình B, Đạt hai hĩnh vuũng ABCD vã EFGH {AB > EF) sao chữ diểm B trùng với điểm
  87. diểm B- nám giữa hai điếm A vá F, tia BC vá tía EH trùng nhau. Trên tia AB lấy điểm P sao cho AP = EF thì AB = PF, trẽn tia BC lấy điểm Q sao cho BQ = BỌ CQ = BC I BH thì HO = BC, Trong nửa mặt phẩng bử AB không chúa diểm G lấy điểm V sao cho APVF - iAPD. Đưừng thẳng FV cất DP tại M vả cắt ŨG tại N. Đường thẳng qua c vả song song vởi FV cát ẤD, DP, QG lường ứhg tạl s, K, J. Dể thấy cảc tam giảc APD. PVF. HGQ, DSC bằng nhau, các tam giác KDS, JQC, MPV. NGF bầng nhau, suy ra MNJK lá hình vuòng, Ta chuyển các đa giác KDS, CDK. APKS theo thứtựđến cho ANGF, AtPM và HGJC, lúc đố SịytnuK - $Ạ BCỮ + §EFGH- Q Bái 3. Chữ một hình chữ nhật với chiếu rộng bằng 2a, chiều dài bằng b = 3a vá một hình vuóng diện tích bằng 2a 2 . Hãy phân chia hai hình đó ra lãm nhiêu đa giác để ghớp lại chánh một hình vuông, Bài 4. Cho một hình chữ nhật vúi clìiẽu rộng bằng 2a, chiểu dài báng b = 3a vá một hình vuông diện tích bàng a 2 . Hãy phân chia hai hình đở ra lảm nhiều đa giác để ghép lại thánh một hình vuông. Hỉnh 7 AU STRALIAN MATHEMATICS COMPETITION - AMC 2013 JUNI0R DIVISION Tiếp tírao kỉ trước PG$. TS, DỖ TRUNG HtỆU (Hà Nõi, Sưu tám vá giộị thiệu) 24, Con si de r the following 4x4 squares with a 1 X 1 square deỉettìtí (shown in black). Con si de r tiíỉng í he squares p, Q and R using liỉes ti ke the one below. Which oỉ the followsng staíements is true? (A) Only p can be tilsd this way r {B) Qnly Q can be tiled Ihis way. (C) Only R can be tiled this way, (D) Only p and Q can be tiled Ihis way. {E) All the shapes can be liled Uiis way.
  88. A number is íormed by writing the numbers 1 to 30 in onder as shovvn. 1 234S07891 0111213 2930 Sỉmeon removred 45 of these 51 digits leaving 6 rn their onginal ardcr To imake the largest 6"dÉgit number pôssible. What IS the sum ừf the thgits ớf this number? (A)33 (B)3& {C)41 (D)43 (EJ 51 For questions 26 to 30, shade the answer 35 an integer Íroiíi 0 to 999 in the space províded on the arcswer sheet. Question 26 is 6 marks, question 27 is 7 marks, question 28 i$ B marks, question 29 Í5 9 marks and questỉort 30 Is 10 marks. 26, Consider a âequence oí ietters where each tetter is A or B. We call the seqnence stab/e if, when we tally the ngmber of As and thạ number oí Bs in the Sequence, wõrking from teít to right, the ditterence is never greater than one. For example, the sequence ÀBBABAts gtahle but Ihe sequence AABBAB is not, bccause atter counting the tĩrsl twữ lelters, the diíterence is two. How many stable sequences with cighreen lettors are there?
  89. Whenever Callum neads a datc like 1/8/2013, he incoríectlỵ interprets it as two divisỉons, with the second one evaluated betore the íirst one: 1 + (3+20 13) -251 ị 6 For some dates. like íhis one, he cíoes not gei an integer, while for othere, kke 28/7/2013, hs gel5 2Ô (7 + 2013) = 8052, an integer. Húw many dates this year {day/monlh/year) give him an integer? 2B. Whaí ỉs the smallest positive integer that can be exprẽ&sed aa the sum uf nine conseeutive integers, the sum of ten consecưtive integers and the sum of eleven cnnsecutive iníegers? 29, Each ọf fhe foưr circíes betow has a whole number vaiue. X IS the value of the topHeít ciíde. A number witten on the tigure indicates the product of the values of She circles íl lics within. Wh(it is the valưe of X 4 k? 30, Three diíĩerent non-zero digils are used to form sixditterent 3-digit numbers, The sum of tive of them is 3231, VVhat is the sixth nụmber? MATHEMATICS ESSAY PROBLEMS IMSO 2015 Tiếp theo kì trước TRỊNH HOÀI DƯƠNG (GV THCS Giảng Vổ. Ba Đỉnh, Hả Nội J
  90. In â four-digil rumiber, the- Ihũusands digit is larger than the units digit. vvhich is not zef 0 . vvhile the hundreds digit is larger than the tens digit. A newtbur~digit numberis obtained from the original number by reversing the order of the drgits. How many possihle ditíerences of the original and now number are thene?
  91. Thene are three lowesHerm ữactions, Ihe ratio of their numerator are positive integers in the raữo of 3 : 2 : 4 white the ratio of the ít denoininalor are positive integers in- the ratio oí 5 ; 9 : 15. The 20 sum of these three íractions is 4Ỗ What Í5 the sum of their denaminator? 1 1. Sixléen poĩnts are on the sides ữf a 4 X 4 grid so thai the eenter portlon of 2 X 2 are remiQved. How many iriangles are there in total thai ha ve vertices chosen from tliose remaining pointa and at leasl 1 interior angle equal lo 45°?
  92. From a 16 cm by 10 cm pieoe of paper, a

    3 em by 3 cm square is cul off From each corner. At most how many 3 cm by 4 cm rectangles can be cut off from the nemaining part of this piece of paper?

    • •

  93. In AABC. posnlã D and E are on BC such that

    BD : DE : EC -2:1 : 1. The púinl M Í5 on .4C such that = — . BM interseets AD, AE at point MA 3 H, G respsntively. F»nd BH : HG : GM. A Êảí 25NS. CÓ bao nhiêu SQ nguyên dương n nhỏ hơn 2Ù1Ệ thỏa mãn s = 1 n + 2 n + 3 n + 4" chia hết cho 5? KIỂU Dính minh (GV. THPT chuyên Húng Vương, Phũ Thọ) Bàp 2GNS. Giải phương (rình (jĩ 4 4)(-v/x + 2 +2) = {x + 1}(j( 2 -2* + 3). CAO NGỌC TOAN (GV. THPT Tam Giang. Phong Điéỉì, Thùa Thiên - Huế) Báí 27NS, Cho doạn Ihầrsg AB có trung dilm o. Kẻ hai dưửng thẳng d và ú' thứ lự vuông góc với AB tại A vã 8. Trên d vá d' lẩn luạt lấy M. N sao cho MON = 90°. Kẻ OH vuông góc với MN tại H. Đường trcn ngoại tiếp tam giác MHB cắt dựởng thrVig ti tại K, Chúmg minh rằng không đổi khi dưííng thểng MN thay đồi. AK ĐOÁN CÁT NHƠN { Phỏng Gìâo ÚụC và Đàu Ịậu An Nhơn. Bỉnh Đinh)

    . + + + + n i + ■■ ■ 1 I n i (TTT2 80 153) Bái 19NS. Ta cỏ x?ịý*z - X 2 - 5) = y{x 4 + z) ->■ x 2 (x 2 + x 2 y + 5) = yz(x 2 y - 1) X 2 4 x 2 y + 5 : (x 2 y - 1) {vi |x, x 2 y - 1) = 1 ) -ỈX 2 6:{x 2 y-1}(x 2 6) + - 1 ) i (x 2 y - 1 J =v x 2 {6y 4 1) : (x 2 y - 1) = 6y 4 1 > x 2 y - 1 > ũ. [1) Nếu X 2 9 thi x 2 y - 1 > 9y - 1 = 6y 4 1 + 3y - 2 Ôy + 1 (mâu thuãn với (1)). Sưy ra Jt e {1; 2}. Xét các trường hợp, ta được (X. y, z} = (1; 2; 4). Nhận xót Chỉ có bạn Kim Thị Hóng Lĩnh, 9Ê1, THCS Vĩnh Tưdng, Vĩnh Tường!. Vĩnh Phúc có lởi giải đủng. Bải 20NS. Ta có (a-b} 2 ỉữ«a 3 - ab t b 2 £ ab oa 3 4b 3 È a 2 b + ab ? <-> 5a^ - 4a 3 Ỷ b 2 s 11a 2 b 1ũa 2 b 4 ab 2 5a 3 - lOa^b > 4a 3 - b 3 4 11a 2 b 4 ab 2 Hiên, Bùi Thị Quỳnh, SA3; Nguyễn Thảo Chi, Trần Thị Thu Huyền. Bù Ị Thị Mỹ Hạnh, 9A3, THCS Lãm Thao. Lâm Thao; Nguyễn Thị Ngọc Huyên , 9A, THCS Hùng Vương, TX. Phú Tho; Lê Nguyền Quỳnh Trang, 9C, THCS Vân Lang, TP. việt Tri, Phủ Thọ. NGUYỀN NGỌC HÂN Bải 21 NS, Cảc bạn iự vẽ hình và chứng minh haỉ bổ đế sau: • Bố dể 1. Cho tam giác ABC nhọn vá không cân tại c. Gọi 0 lá tãm đường tròn ngoại tiếp. Các đường cao AD, BE cát nhau tại H. Gọi I và K thao thứ tự lả giao điểm của Aũ và QÊ. BE và Oũ. Khi ID KE ' n IA " KB' • Bổ âê 2. Cho tứ giác ABDE. Lay I, K thuộc các <=4 5a 3 1 0a 2 h > 4a 3 4 1 2a 2 b 4 4ab 2 - a 2 b -3ab"-b- <= 5(a 3 4 2a 2 b) > (4a - b)(a 2 4 3ab 4 b 2 ) a 3 +2a 2 b 4a-b =» , a 2 --3ab4b 2 & T ương tự ta có b 3 42b 2 c 4b-c c 3 4 2c 2 a 4c-a b 4 3bc 4 c b c 4 3ca 4 a 3 Cộng theo vể (1 J, (2) vá (3} ta được đpcm. Dấu bằng xảy ra khi và chỉ khi a = b = c. Nhặn xẻt, Bài này có rất nhiểu bạn tham gia giải vả giải đúng. Các bạn sau được khen: Kim Thị Hóng Lĩnh, 9E1, Phan Huyên Ngọc, 9B. THCS Vỉnh Tuởng, Vỉnh Tưởng; Tạ ỉhuy Tiên, 9A4, THCS Yên Lạt, Yên Lạc; Ngổ Thị Thu Hiến. Lê Thu Trang , 9D. THCS Lý Tự Trọng, Binh Xuyên, Vĩnh Phúc; Nguyễn Thị như Quỳnh A, 9A. THCS Lý Nhặt Qưang. Đõ Lương; Thành Tứ Oanh, 9D, THCS Trung Đò, TP, Vinh, Nghệ An; Bụi Thúy Linh. BAI Nguyễn Thúy Ouưng, Nguyễn Thu đoạn ÀD, BE sao tho — - -L-. Gọi M, N, L theú IA KB ' thử tự lâ trung điểm cua AB. DE, IK. Khí đó M, N, L thẳng hàng. Trỏ lại bài toàn ị bạn đọc tự vẽ hình) Không mất tính lổng guát giá sử CA< CB. Gọl N, L theo thử tự là trung điểm của DE. IK. Vi AD 1 CB, ĐE 1 CA và MA = MB nèn MD = ME, Do đó MN lá đưởng trung trực của DE. (1) Theo bổ đẽ 1 thì 1?=!^. IA KB Từ dó chú ý rằng M. N, L theo thứ tự là trung điểm của AB, DE. IK nên theo bổ để 2 thỉ M, N, L thảng háng. Kết hợp vứi M, I. K thảng hãng, Suy ra I, K thuộc doạn thẳng MN. (2) Từ [ 1 } vá (2}, suy ra IK lá đường trung trực của DE. Do dớ tứ gi ác DO HE nội tiếp (vì lá hình thang càn), Vì HDC=hÊC (=9Q Ũ ) nền tứ giác CDHE nại tiếp. (Xem tiếp trang 4) r DANH SÁCH ĐOẠT GIẢI cuộc THI TÌM HIỂU CỘNG ĐồNG ASEAN CẤP THC$ Ị Giải Nhất: Nguyễn Minh Trí, 6A2 THCS Yẻn Phong, Yên Phong, Bẳe Ninh; Nguyễn Đặng Sơn, 9A, THCS Nguyễn Trãi, Nam Sách, Hảỉ Dương. ị Giải Nhỉ; Kim Thị Hông Lĩnh, 9E1, THCS

    Vĩnh Tưởng, Vỉnh Tường, Vlrrh Phúc; Trần Thị Thu Huyền r 9D, THCS Lý Tự Trọng, Bình Xuyên, Vĩnh Phúc; Phan Thị Thào Ngôn , BC, THCS Bạch Liêu, Yên Thành. Nghệ An.

    Giải Bã; Mai Dứt Toàn, 9C, THCS Nguyền Cao. Quế Võ. Bắc Ninh; Thài Anh Quân, 7A, ị THCS Đàng Thai Mai, TR Vinh, Nghệ An; Phan An Khánh , 8A2, THCS Giảng Võ. Ba ! Đinh. Hà NỘI; Nguyễn CtĩíCõng . 6A3. THCS 2MESM KI 9(TTT2số153) Cảu 25. sâp xếp các vế phấn chữ ủng với ớảc vế phẩn đánh sổ nhự sau: a -»■ 4; b -> 3; c 2; d -■ 1 .
  94. NUỦC nằm trẽn cả Bắc hán cầu vã Nam hán cẩu là nữởc đông dân Ihử tư trên Ihế giới.
  95. Nước góm hơn 7000 hôn đảo (á nước có dân 50 gan với dãn sổ Việt Nam nhất (nhiẽu hơn một chút).
  96. Nuớc gốm hai phấn: Đông trên đào Calimantan, Tây tnẽn hán dân kéo dái từ 00 Cra tới vịnh Singapore, cảch nhau 75Ữ ken lã nươccó diện tích xấp xĩ Việt Nam.
  97. Nưdc gồm nhiểu đâo, diện tích nhò nhất lã nước cố hải cảng bân ràn nhất châu Á và sản bay lốt nhẩt thế giới. Câu 26. Tèn 5 thành phố dỏng dãn nhất trong ASEAN là: dakarta; Man lla; Bangkũk; Hô Chí Minh; Hà NỘI. Cãu 27. 1} ASEAN Summlti Hội nghị Thượng đỉnh ASEAN,
  98. ASEAN Miinistenal Meeíing - AMM; Hội nghị Bộ truủng ASEAN.
  99. ASEAN Ecunomic Ministers " AEM: Hội nghị Bộ trxíởng kinh tếASEAN. L ãm Thao, Lâm Thao, Phủ Thọ; Phan Thu Trang. 9A1 . THCS Chất lượng cao Mai Êơn ; thị trân Hảt Lót. Mai Sơn, Sdh La. • Gìảỉ Khuyên khích: Đặng Thị Hường, 9B, THCS Yẽn Phong, Yẻn Phong, Bẳc Ninh; Nguyền Hải Ly, 6A, THCS Hoàng Xuân Hãn, Đíre Thọ, Hả Tĩnh; Nguyễn Đinh Đạt. 7G, THCS Bạch Liêu. Yên Thành. Nghệ An; Nguyễn Vũ Hà, 7A3, THCS Làm Thao Lảm Thao, Phú Thọ: Lại Khánh Trang, 6A. THCS Vĩnh Yên, TP. vĩnh Yên, Vĩnh Phùe, Nguyễn Hỏi Khoa, 6A, THCS Lý Tự Trọng, Bình Xuyên, Vĩnh Phúc: Lè Hóng Nhung, 7A, THCS Vĩnh Yên. TP, Vĩnh Yên. Vĩnh Phúc.
  100. Joint Ministerial Meating - JMM: Hội nghị liên Bọ trưởng ASEAN.
  101. Senior ottìcials Meeting - SOM; Cuộc họp các quan chức cấp cao ASEAN.
  102. ASEAN Standing Comumittee - ASC: Oy han thướng trực ASEAN. Nhận xét. Da số các bạn có câu trả lài £A.„, . đúng. Cảc hạn được nhận quà kì này; Phan An Khàĩìtì , 9A2, THCS Giảng Vò, Ba Dinh, Hà Nội; Đặng Lan Hương, 7E1, THCS Vĩnh Tường. Vĩnh Tường; Nguyễn Hài Khoa, 6A. THCS Lý Tư Trong, Bình Xuyin, Wnh Phúc Bụi Thị Tướng Vi, SA, THGS Bạch Liêu, Yên Thánh; Nguyễn Thị Mai Anh. 70, THCS Đặng Thai Mai, Vinh; Nguyễn Trình Tuấn Đạt. 7D, THCS Lý Nhật Quang, Dô Lương, Nghệ An; Phan Thu Trang, 9A1, THCS chất lượng cao Mai Sơn, thị trấn Hái Lót. Mai Son, Sdn La: Nguyền Minh Trí. THCS Yên Phong, Yên Phong, Sắc Ninh; I Nguyễn Ngọc Lỉnh. 7A1 , THGS Lâm Thao, Lãm Thao, Phú Thọ. BTC Trang thơ Trang thơ * Trang thơ Trang thơ VŨ KIM THÚY £? > Bỗ ĐÀ Quanh co cổng lối kỉtó tim Rêu xanh iiiờng dai chùa chìm irọng căy Thời gian nyímy ờ nơ/ dãy Muốn tìm cố kính vẻ ngay Bó Dà Cách thú đó chang hao xa Giữ chữ hận thế bùng xòa íhờĩ gian Ngày xuân đến uãn cánh írổn Gíeơ ihém mdm ỉhiện íhêmXtiđn mỗi ngùời 26.2.2015 Việt Yên, Bẩc Giang CAO NGỌC TOẢN fGV. THPT Tam Gtang, Phong Điền, Thửa Tĩitển - Huế) y Đôi bàn íay Bàn iay ihon Liitít nhẹ phím đàn Ăm tfưmh ưẩm bồng tíểng ca cho đời Bàn lay thô Chai sđn vết náng mdh Chồi xanh trĩu quà tốt tươi bốn mùa Bàn ray ẩm Tay nủm ỉđy hàn íay Đường xa chăng ngạt, sòng dà í cũng qua Đói bán Lay sườì ấm những bán Lay Tinh yêu muôn thuở, tìm nồng yêu thương Diệu fcì thay Bđn ray ỉdm tất cả Vinh qưang nào chẩng có dấu bàn tay. TIN HOẠI ĐỘNG CẲU LậC BỘ TOÁN TUỔI IHO
  103. Ngày 19.1.2016, tạp chi Toán Tuổi thơ tổ

    chức Ngày Toán Tuổi thơ tạỉ trướng: tiểu học Đoán Thị Điểm, Mỹ Đình, Hả Nội. Nội dung gổm ba phẩn chính: +■ Kỉ niệm 15 năm Toán Tuổi thơ (25.1Ồ.2ỮỮỒ ra số dầu tiên , 30.12002 thành tập TTT}\

    Trao thưởng các cuộc thi Iren Tạp chí; TỂ chức thi li&ri tỉnh Gãu lac bộ Tữốn Tu& thơ

    giOa các Càu lạc bộ đen từ: Nam Oịnh, Thái Binh, Sdn La, Hưng Yên, Quảng Ninh r Vĩnh Phúc, Hà Nội. oé thi gồm 2 nội dung; Phẩn 1 là các dế toán tiếng Anh, chì yêu cẩu ghi tĩap số; phấn 2 lá các câu hỏi tiếng Việt vể IQ, câu đố toàn, lịch sử toán, trù chơi toán. Có hai vòng thi:

    Vòng 1, Thi tiếp sức đồng đội (Gốm có 2 hiêp )
  104. Hiệp 1. Tiép sửc toán
  105. Sáu thí sinh cùa mỗi Câu lạc bộ lấn lượt giãi 6 bái toán chỉ ghi dáp sổ. Thời gian tối da lá 3Ũ phút chạ Ê bái toán.
  106. Hiệp 2. Du lịch Toán học
  107. Có 6 thành phổ cho các bạn học sinh đẽn "tham quan" là: Hà Nội, Hải Phàng, Nam Định, Huế, Đà Nâng, TP Hỗ Chỉ Minh. Hai giảm khảo sẽ là chú nhằn của bán đại diện thành phổ đó.
  108. Các em học sinh trong Câu lạc bộ củng giải & bải toản vui. Một em học sinh lá đội trưởng đến thảnh phố thứ nhất để láy để bài 1 {Thăõ chỉ định của Ban tổ chức) sau dó các em học sinh của đội cùng giải bài rói đội trưởng nộp kểt quả cho gi ám khảo ở thánh phố thứ nhất, neu kết quã chưa đủng thi giảm khảo sẻ yêu cẩu làm lại đến khi náo Cau lạc bộ đó đưa ra được kết quả đủng bài 1 thì Gãu lạc bỏ đó mới cỏ được địa chỉ để đến thành phố thứ hai nhận đl bảì 2 để gia! tiếp, cứ tiếp lực như thé cho đỂn bài 6. Tổng thời gian tối đa để làm cả 6 bài toán là 30 phút. Hiệp 2 kết thúc khi hẽl già hoặc đã cố 2 đội có kết quà đúng ò bái thủ 6 vả vé đích.
  109. Hai Càu lạc bộ có tong điểm vỏng 1 cao nhất

    đuợt vào thi đâu vòng 2 để tranh giải Nhất (Nếu có câc Càu lạc bộ bàng diềm nhau thi sẽ dùng cáu hòi phụ để phân bại).

    Võng 2. Tranh giàí nhất
  110. Gốm 3 hi&p rnỗi hièp hai Câu lạc bộ cùng giải một bãi toán.
  111. Mâi đội nhện một bảng để ghi đáp sò. Thài gian giải mỗi bái không quá 5 phứt. Thực hiận theo hiệu lệnh trống của Ban bẩ chức,
  112. Ban tổ chức sẽ cộng điểm ở cả hai vòng thi đẩu dể chọn ra Câu lạc bỏ đuực trao giải Nhát, Câu lạc bộ được trao gỉải Nhì (JVếư các Càu tạc bộ bàng điểm nhau thi sè dùng cảu hỏi phụ dể phân bại).
  113. Ngoài cãc Cảu lạc bộ dã dược nêu lên, cốc trương sau đa dãng kí Càu lạc bộ Toán Tuổi thơ vơi Tạp chi: TH An Ninh. An Ninh, TH An Dục : An Dục, Quỳnh Phụ. TH Tân Lặp 1 , Tán Lập, Vũ Thư, Thãi Binh: TH Phan Chu Chinh. Nam Dà, Krông Nô. Đắk Nông; TH Mộc Lỵ, Thị trân Mộc Châu. Mộc Châu. Sơn La; Tt-t Hòn Ma, Thg Sơn; TH Tàn Hưng. My Lâm, TH Hiệp Tên, Mỹ Hiệp Sơn, Hòn Đất; TH Minh Hóa 7, Minh Hòa; TH Mọng Tho Aĩ, Mong Thọ A. Cháy Thánh: PTCS Sơn Hải. Sơn Hải, Kiên Lương; TH Thị trấn Thứ Ba, thị trấn Thứ Ba r An Biền; TH Bình An. Binh An; TH Thị trẩn Kién Liíơng í, TH Thị trấn Kiên Lương 2, Ttì Thị Irấn Kiên Lương 4, Thị trấn Kiên Lương, Kĩén Lương; TH Lương Thế Vinh. Rạch Sổi; TH Chảu Vẳn Liêm, Vĩnh Hiệp; TH Ắu Cơ, ỈH Hồng Bàng. Vĩnh Thanh Vàn; TH Lê Vàn Tém, Vĩnh Lạc; TH Lý Tự Trọng, TH Đinh Bộ Lĩnh, TH Lè Thị Hổng Gấm, TH Lê Hóng Phong. TH Hạnh Phưởc. Vĩnh Thanh; TH Lẽ Lợi, Vinh Hiẹp; TH Trấn Nhật Duậl, TH Trần Quốc Toẩn. Phi Thông; TH Trương Định, TH Lỳ Thường Kiệt, An Bình; TH Trần Vắn ơn, vĩnh LỢi: TH Mạc Đĩnh Chi . Rạch Sòi; TH Nguyễn Hiển, Vỉnh Quang; TH Kim Đống. Vĩnh Bấo; TH Nguyễn Thải Binh . Vĩnh Quang, TH Trán Khánh ùư, ĨH Trưng Vương !, An Hoa; TH Phạm Ngũ Lão. Vĩnh Tliâng: TH Nguyễn Chl Thạnh, Rạch sòí, TP. Rạch Giá, Kiên Giang, CLB TTT NOEL VA TET 1 Bất đáu tư Iháng 12, các thành phò phương Tây vá nhiều thành phố phuHìng Dồng như Maniia, Singapore, đã dược trang hoàng đèn màu lọng lẫy và cảc cãy thủng No&l lớn fựfc rỡ đưọc dựng lẽn. Ngưới tạ di mua sám táp nập. Nhiếu thành phố không ngủ. 'ớ vàn phóng, các cồng ty số ngứũi xin nghỉ phép nhiều lẻn vã nhữhg ngươi dĩ làm cũng không bận bịu. bận rộn vi nhiều người ờ cãc bọ phận đã nghỉ hoặc làm cầm chừng. Noẽl thực sự đa thành ngày sum họp gís dinh. Tiếp đó, chi sau dùng 1 luẩn là ngày bổt dấu nãm mủi. Vi thế IV đêm 24.12 dến hết ngáy 1.1 hàng nàm thực SƯ là nhưng ngày Le, Hội lân nhất trong nàm. Ngày 25.12 la quen gọi là Noèl. Ngày 26.12 lã ngây tặng quà, Các món ân truyền thống của các nước phương Tây trơng dtp này là gá lây, sưân nướng, gan ngòng, xúc xích, salad r khoai lảy. pho mat, bơ vá bành ngọt. Nhưng ấn lượng hơn câ lá các cây thông từ ngoại thánh được chỏ vào thánh phở. Cây th&ng đuọc tí eo câc quà bỏng mồu. cốc thiếp mùng nSm mới vè cây Irâ nên sinh động hơn VỚ! các dây màu, bóng bay ... Trả con vus nhận quà của ũng già NũỂI cả trong thụt te vả irong tưởng tượng. Truyền thuyết kế rằng 6ng di xe Tuẩn tộc kéo, lniợt tuyết đs khằp nơi. váo ong khói các gea đính dể bõ quà vào tất trè con khi đêm đẽn. Ngày Nãm mới được đãnh dấu bằng mãn phãữ hoa, Ở một số cõng ty nhãn viên cũng được sếp tặng phong bi tiểu cảm tín sau một nãm làm Vỉệc tốt. Ở Việl Nam. các thảnh phố Há Nôi . TP. Hố Chi Minh, Nam Đinh, Đá Nắng, Đà Lạt, .... hay các thị trấn như Sa Pâ những năm gắn đây cũng lưng bừng trưởc Noẽl chừng nửa thãnp, 2 Ở Việt Nam vã một số nước châu À lại cỏ Tếl theo lịch Mại Irãng. Tếl Việt Nam đao động từ ngày 21.1 dén ngày 19.2 Dương lịch. Cứ thế, nếu Tết rơi váo 21.1 lá sớm nhất và thuừng là lạnh, còn nếu rơi vảo 19.2 như năm ngoài lá muởn nhất vã thường là ấm. Còn lỉếi Lập xuãn thường rơi vào ngáy 3.2 hoặc 4.2. Sau tiết Lập xuẳn trdi thướng ấm hơn, Nâm ngoái và nãm nay Tét dểu sau Lập xuân là tròi đéu ấm. Nâm nay 1.1 Tết váo ngáy 8.2 lã giong như nâm 1958. Tính ngược trước Tết 1 luấn, ngày 23 tháng Chạp lả vùng nông thôn Bác B& trống dây nêu. VẾ hình BÍNH NAM HẢ mùi cung tận bằng vôi ỏ san nhà để duọi quỷ ra biển Đồng. Bần thớ tể tiên củng dược dọn trong ngáy nảy. Sau đó màm ngũ quâ dược bày lẽn bân Ihở và hương bắt đâu thắp tử ngày đó, Ngũ quả tượng trưng cho ngũ hành và úhg với nảm đửc tổt: Nhãn. Nghĩ3. LỄ, Trí. Tín. Ngũ qua cũng đủ cá các màu xanh (chuối }, vàng (quất, quýt), đỏ (cam), náu (hóng xiêm]. Nải chuối là thử quả không thể thiếu trong mâm ngũ quà miển Bắc vá dược số quả lè (15, 17, 1 9) thi chủ nhã rgit vui. Nguíii miến Nam thì bày màng cẩu, dừa, đu đủ, xoài do cách ohảt àm chệch đi cua: cầu vừa đù xài, Do cách phái ãm chưòi gần giống chúi vã quất gọi lá tác nên miển Nam ít dùng hai loại quả này. Lễ cúng ủng Táo thưởng lãm trưốc 12 giờ trưa 23 thảng Chạp và có 3 con cá chép de ba ông bà Táo có phương tiện VẾ bầm Ngoe Hoàng. DÉìm 30 Tết (cô n&m lịch thiếu chĩ lã 29 Tểt) lả [ỉ&m trừ tịch, nhiểu gia đình cúng Tất niên vá bây cả màm cỗ ngoái sân tụy Iruyốn í hổng giạ đình, Cùng vội cây nêu và hình cung lèn. hoa đào được bảy trong nhà để xua duổi quỷ. Gần dây hoa lay ơn, cay quất được nhiếu gia đình chơi t^ong dịp Tẹí, Cây quất có xu hưởng tạo dâng cây thũng Noèl ở Hả Nội, Quất phải đủ lá xanh, lá non, hoa, quà xanh vá quả chín lá đạp nhất, ờ Nem Định Ihl cây quất để dáng tròn tự nhiên. Những năm cuối thẽ kĩ trưỡc, ngày Tễl còn có hoa thược dược, hoa violet cẳm chung một lọ Shặt to. Hoa đáo Ihì dược mang về từ rửng Việt Bác vá Tây Bic. Sau đõ dào Nhật Tân. Tảy Tựu rối Hưng Yẻn, Vĩnh Phúc, ... dần dln có nbiểư láng trổng đèo, Day lá dáo trổng có cắt tỉa tạo dáng. Bảy gltí nổi íiểng hơn cả vẫn là hoa Tây Tựu, Quãng Bá (Há Nội), Vy Khẽ, Mỹ Tân (Nam Định), Vãn Giang (Hung Yên), Mè Linh (Vĩnh Phúc xưa. nay thuộc Hà Nội), Dà Lạt, ... Nhiều gia dinh cồ thú chơi hoa thủy liên là lọ hoa để bàn sống hãng nước, thưàng nở vào mồng 1 Tết rất thơm. Mấy năm lại đây nhiéu loại hoa mới của phương Tây rihư tuy lip, hoa [y du nhập váo Tél Việl. Mùa Xuân kém theo mưa xu an và gió Bấc cuối mùa, gió Đông đấu mùa dế cho vi rút, VI trúng phát triển néri các thành phô thưởng quét vâl vìâ hè. gốc cây ven đưdng phố. Tục lệ náy nay côn thấy ử Nam Định vả một sõ phố Hà Nội. (Còn uếp ỉ Hỏi: Anh Phó di! Em gửi chung cả hái Thi giải toán qua thư và bài Phá ản củng thảm tử Sêiôccôc có được không ạ? NGUYỄN NHẬT LINH (8E. THCS Lé Quỷ Đòn, TP. Tuyên Quang , Tuyên Quang) Đáp: Chung tà chung một phong bi Viết thi riêng giấy đừng ghi chung tò Mỗi bài ọửị một thầy cô Viết tiền tò giấy biểt đưa thầy nảo? Hỏh Khi làm bái Thf giảt toán qua thư, học sinh lớp trên có được làm bài của lốp dưâi khồng ạ? NGUYỀN NGỌC LINH (7 AI, ĨHCS Lâm Thao, Lăm Thao, Phú Thọ ) Đáp: Lôp tíưới tàm bài tóp trên Xưa nay điều ấy được khen Lóp trên /ám bải tớp dưôi Hoàn toàn tà chuyện không nên. Hỏí; Anh Phó ơi! Nếu em gủỉ đáp án của những mục khác bằng phong bi của Thi giải toán qưa thư thì có được không ạ? NGUYỀN TRƯỜNG THỊNH (7 AI. THCS Lâm Thao , Lảm Thao, Phú Thọ) Đáp: Điểu này anh hói nhiểu rỗi Phong bi thi 1. bải nhồi thât cẳng Miền đừng chép lằn trong trang Mỗi trang mỗi mục đàng hoàng nhò chưa. ANH PHÒ CÁC LỚP 6 & 7 Sải 1(165), Tìm các sò nguyên dương a, b, c thỏa mãn đổng thời các điểu kiện sau: í) ab I b - a! = í; ii) cb I c - b! = 1 ; íii) a 2 - £b 2 Ị 2a - 4b = 2. LƯU LÝ TƯỞNG (GV. THCS Văn Lang, TP. Việt Trí, Phú Thọ) Bài 2(1 55). Cho tam giác ABC có AB + AC - 2BC. Goi i là giaọ điểm Cát đường phân giác trong của tam giác. Gợi M. N theo thử tự lâ trưng diểm cũa AB, AC. Chứng minh rằng ẤMI + ẤNÌ - 1 80° NGUYỀN MINH HÀ (GV. trường THPT chuyên Đai hạc $ư phạm Hè Hội) CÀC LỞP THCS Bãi 3(155). Giã sử n JỂt sã nguyên dương sao cho lân tại ũảc sổ ngưyẻn dương a, b, c thỏa mân ab + a 2 c+ t> 2 c + abc 2 = 101 n , Chứng minh ràng n lã sổ chân. NGUYỄN DUY LIÊN (GV. THPT chuyên Vĩnh Phức) Bài 4(155), Cho a, b, c lả các số thục dương thỏa mãn a I b I 5 = 3, Chứng minh rèng: a 2 +ab 2 b 2 +bc 2 c 2 +ca 2
  114. — + — — — 1 - — >2 b 2 +a + b c^ + bi-t a 2 +c + â CAO MỈNH QUANG (GV, THPĨ chuyện Nguyễn Bỉnh Khtêm, Vĩnh Long) Bài 5(155), Một đa đỗ thị G(V, E) bao gõm một tập hợp V cát đính vá một tập hợp E các cạnh, trong đò E cỏ thể báó gồm cầc cạnh kèp vá khuyên, Xem ví dụ (e 4 , e 5 lá cạnh kép, e 7 lá khuyên). Hầy vẽ biểu đó cho mòi đã đố thị G(V, E), trong đó V = {P v P z . P 3 . P 4 , Pg) và
  115. E = (ÍP 2 , p 4 }, {p 2 , fP 3 , Pg}, (P 5 . p 4 }}; b> E={{p 1( p,}, {P 2 , P 3 >, {P 2 , p 4 }, {P 3 . P 2 }, {P 4 , p.}, {P 5 . P 4 }}, VŨ KIM THỦY Bái 6(155). Cho tam giác ABC nội tiếp đưàng tròn (O: R). Gọi I lá điểm nằm trong tam giác ABC (I không nằm trên cạnh cua tam giác). Các tia AI. BI, C1 Ihứtựcắt BC, CA. AB lại M, N, p, Chúng minh rằng — + — < 4 y AMSN BN.CP CP.AM 3(R-OỊ) 2 NGUYỄN KHÁNH TOÁN (GV. THCS Bắc Hải, Tiên Hèi, Thái Binh) SOLVE VIA MAIL COMPETITlOn QUESTIOHS Transỉõted by Nam Vỡ Thành 1(155). Find al! positive
  116. afi + ủ - a’ = 1; ii) Ị I I í ỉ PHIẾU ĐĂNG KÍ 1 THAM Dự Ị CUỘC THI GTQT NĂM HỌC 2015-2016 integers a. b, and c satisíying the toilovving eqưations. db -Mĩ-b! = 1; iii) a z -2 + 2a-4í? = 2. 2(155). Given a trĩangle ABC satislying Ẩ6 + /tc = 2BC. Let / be the mtersection oí its intemal sngie bisectors, Lel M and N he the midpolnts of AB and AC. respectivelỹ. Pnove thai ỵAMI + ỵANt = 180°, 3(155). Lel n be a positive integers such that there exist positive Êntegers a, b. c satisíying âJÍ> + a 2 c + Jb 2 c + abc 2 = 1ữ1 n . Prove Ihai fì is an even number, (Xem tiếp trang 20} Bạn hãy vàn webtìite: htt|i://ulin.viỉ/hieii-siU'h-iỉnline ik đục rạp chí Toán Tưíii thơ biin điên tú nhớ. • Tớ chức Cuộc thi sàng, lác càu hòi và bài lập phát triển nâng lực mỡn toán cua học sinh bậc TtlCS vã cấp Tiểu học. Đẫy là cuộc thi dành cho giáo vién. • Tỏ chức Câu lạc bộ Toán Tưởi thơtiì cáp trường dến cấp huyện, quặn, tmh, thánh ở cẩp Tiéu học và bậc. Trung hợc cơ sớ đé tạo phong trào dạy vã học: mởn Toán o các nhà trường, Liểrt Lỏí cuộc Ihỉ toàn qưóc. • Mó chuyên mục cứa số AC đang các Lhồng tin vẻ 10 nước ASEAN Khi Cộng dồng ASEAIH chinh thtíc dược thánh lộp tứ 3 1 . 1 2,2015, dé glũp bạn dọc có thèm nhiẻu hỉéti biết vê các nước bạn. • Hớ chuyên mục Vè tranh theo chú đề, Các em học sính được sáng tác các bilc tranh vẻ quệ hương, díit nước. Dành cho các thấy cô giảo ỮUỘC THI SRÍIGTÁỮ CPU HỦI ụp BÀI TẬP PHÁT TRỔ ntìnG Lực môn Tũtìn cũn HỌC « BẬC THCS Nhằm tạo ra ngân bủng câu hỏi giúp phát ĩribn năng lực cưa học sinh dỏng I.hời dộng vỉẽn, khuyến khích cãc thảy cõ gỉáo sảng tạo nhiều hơn nữa để có những gỉờ dạy hiệu quá cao, có thệ thống cáu hỏi, bải tập có chất lượng tốt, tạp chi Toán Tuổi thớ tổ chức Cuộc thi sáng tác câu hò ĩ và bài tập phát triền năng lực môn toán của hpc sinh bậc THCS Đãy lù một cuộc thi mdi vò là một cuộc thi lớn trong nãrti 2016 Lrẽh] tạp chi.
  117. Nội dung bải tạp, cãu hỏi. Các bãi tập. càu hủl môn toán giúp phát triển năng lực học toàn của học sinh. I3an tố chức hoan nghênh các bàl tập. câu hỏi cõ binh vẽ minh họa để các em học sinh thấy rằng mõn toãn thật ihũ vị. Các bái tập. cãu hỏi phái là các bài mới chưa xuất hiện trẽn bất kì sách, báo nào, Mỗi cá nhãn, tập thể gửĩ một lẳn hệ Ihống bài lặp và cáu hả ỉ gỗm ĨI nhãi 40 bài lặp, cãu hỏi cho cả bốn lớp 6, 7, 8 vã 9 (mồi bãi lập, câu hỏi cán ghi rỏ danh cho lởp may) .
  118. Đối tượng dự ỉhỉ. Cãc thẳy, cỏ giảo, cãc cán bộ quản ]j giáo dục,
  119. Thời hạn nhạn bài dự thi. Kể từ tháng 1.2016 đến hết tháng 12.2016. Các bài dự Ihỉ cản viết rõ trẽn phong bi: Tham dự Cuộc thỉ sáng tác câu hỏi và bải tập phát triển năng lực môn toán cua học sính bậc THCS, Trong bài dự thi ghi rõ: Họ vã Lẽn. địa chỉ, số điện thoại, email vá gdí về: Tộp chí Todn Tuổi thơ, tâĩtg 5, số 36 ì Trường chinh. Thanh Xuđn, Há NỘL Tạp chi Toãn Tuổi thơ se díing các bã ỉ tập, cầu hồi hay vã cãc tác giả sẽ dược nhận nhuận bũt.
  120. Tổng kết và trao giải. Hết thăng 12.2016, tạp chí Toàn Tuổt thơ sẽ tổng két cuộc thi. Ban tổ chức sẽ chấm các bâỉ thi dựa trẽn các tiêu chí: Sõ' lượng bàt tập, cũu hỏi, chấi lượng chuyên mồn của từng bài. Lững cáu và sự da dạng vẻ nội dung. Giải thưửng gòm Giấy chửng nhận, tiền mặt vã quá tặng. BAN Tổ CHỨC Thoại nhìn ta cử tưímg đang ƠLÍỢO ở một nhà hát lởn giữa trời Ẵu vào thế kì XIX hay XX. Diểu ăó đúng vì nhà hái Lân Hà Nội mõ phòng kỉẾn trúc I'.hả hát ũpéra Garnie (Paris. Ptiãp>. Điạsc khơi câng nám 1901 và hùản thành 1911, đín 2016 này nhà hát tròn 1Ữ5 tuổi. Chi khi nhìn õ tô, xe máy bõỉi ngoài ta mói trỡ lại vỡi tlìực tại tíLtôũ VẺ dẹp 'ì ý 3 nqầng. Bửc ảnh thật hải hòa bùi cự Lôn lẽn vè đẹp từ cây xanh và máy trài với diểm xuyết của lá cờ. Bạn hãy viết bài binh vể bức ảnh nhé. MORIS VŨ Anh: Phun Nịịỉịịm' Quang CÁC HỌC SINH DƯỢC KHEN THONG cuộc THI GIÀl TOÁN DÀNH CHO NỮ SINH Từ trái Sãĩĩg phàĩ: Kim Thị Hồng Lĩnh, Phan MuyỀn Hgọc, BÙE Thúy Linh, nguyên Thùy Duơng, Lẽ nguyên Quỳnh Trang. © (ịrcr vArỉmĩtetĩ rtùitĩ HỔNG HA -tíiiV Ỉfrỉiirtị tếtiỉrrv f ĩỉtứ ỈTrOtỉit ỈMỈ Cõng ty CP VFF Hổng Hả là nhá tài trợ cho 2 cuộc thU và Giấy phép xuất bản: s5 31/GP-BVHTT, cáp ngây 23/1 /2 D03 cũa Bộ Ván hóa vả Thõng tin. ỈVIã số; 8BTT156M16. ín tại: Công ly c6 phần in Còng Đoản Việt Nam. 167 Tây Son, Đống Đa, Há Nội. In xong và nộp lưu chiểu Ihángũl nám 2016. NĂM THỨ NHÃ XUẤT BẦN GIÁO DỤC VIỆT NAM ■ Bộ GIAO ■1 • i : - TRIÍNCI «irc cg ỈG Chil4ren'$ Maths Journal NHÀ XUẤT 8ÃN GIÁO DU c Y|ẸT NAM - BỘ GIÁO Dực VÀ MO TẠO CH|U trách nhiệm xuất bản Chú Ạch Hại dõng Thánh vEm MẠC VÍN THIỆN ĩlDn Giám nóc Ủ5. 1$. vu VÃN HÚNG Phỉ Tấng Giám dõc kiêm Tông biên lặn TS. PHAN mn THÁNH HỘI ĐÓNG BIÊN TẬP Táng biẾn tặp. Tha, VŨ KIM THÚT NGNB. vũ Hữu BÍNH rs. GIANG KHẮC BÍNH TS. TRĂN ĐINH CHÂU TS. VŨ ĐÍNH CHUẨN TS. NGUYỄN MINH ĐỬC ThS. NGUVẾN anh dũng TS. NGUYỄN MINH HÀ PGS. TS. LẺ QUOC HÁN PGS. TSKH. VÙ ĐĨNH HỎA TSr NGUYỄN Đifc HOÀNG ThS. NGUYỄN vũ LOAN NGUYỄN í>ừc TẤN PGS. TS. TÔN THÂN TRƯƠNG CÓNG THANH PHẠM VÃN TRỌNG ThS. HÓ QUANG VINH TÒA SOẠN Tổng 5, ẸQ 3$1 đutìng Trương Chinh, quặn Thanh Xuãn, Ha Nộĩ Điẹn Itioại ÍTel); 04.35682701 Điện sao (Fax): 04 .35882702 Diộn thư {Email): Lữantuoilho@vrm.vn ỉ rang mạrhg (WatislE&): //www.toanEijolliho.vn ĐẠI DIỆN TẠI MIẾN NAM NGUYỆN VIỂT XUÂN 55i''1 2 Trán Đinh Xu. p. cáu. Kho. Q.l. TP. HCM ĐT. 08.fi6a211ữa, DĐ. 0973 30âí99 Blôn tập NGUYẺN NGỌC HẰN. PHAN HƯƠNG Trị ỈLT- Ph,ál hành: TRỊNH THỊ TUYẾT TRANG, VŨ ANH THU, NGUYÊN HUYẼN THANH Ché bản: Đố TRUNG KIẼN Mi íhuiỊỉ: Tủ ÀN TRONG SỐ NÀY Dành cho học sình lớp 6 & 7 Các dạng toán vé góc trong hình học lởp 6 và 7 Võ Xuân Minh Học ra sao? Giảỉtoảri thẽ nào? Hệ hai phương trình ba ẩn Mai Vàn Nãm Com pa vui tinh Chia tĩ lệ đoạn trung tuyến Đan Quỳnh Phá án cùng thảm tư sẻ! ôc cóc Con rùa vàng Nguyễn Thị Lan Đến với tiêng Hán Bàỉ 65. Đá Nang nống hơn Há Nội {Tiếp theo kỉlrưức) Nguyễn Vũ Loan Học Vật I! bằng tiếng Anh Unĩt 18, Gas !aws and particles of matter {Tiếp theo kì trước) Vũ Kim Thùy Dành cho câe nhà toãn học nhỏ Tính chất đường tròn Éuler và một số bái toán áp dung Vù Công Minh ĐỂ thi cảc nước IMSO 2015 - Mathematics Essay problems sotution Trịnh Hoải Dương Lịch sửTỡán học Đại số Binh Nam Hả Tr 27 CAC DẠNG TOAN VE GOC TRONG HÌNH HỌC LỚP 6 VÀ 7 VÕ XUÂN MINH (GV. THCS Nguyễn Ván Trồi, Cam Nghĩa, Cam Ranh, Khãnh Hòa) Trông ùhương trinh Hỉnh học tôp ê và lớp 7 chùng tâ thường gặp cát bài toàn vé tinh gốc } chứng minh tia nàm giữa hai tia., hai lia đũi nhau. Bài viết nồy chúng tôi sã viết về các dạng toản trên thông qưa càc ví dụ. Dạng 1 r Chủng minh tia nằm giừa hai tia ■ Phương phàp giải. Tía Oz nằm giữa hai lia Ox vả Oy nếu cú một trong các điểu sau:
  121. xOy = xOz + zOy.
  122. Tia Oy vã tia Oz đếu thuộc niửa mặt phẳng bờ chứa tia Qx VỀ xOz < Xổy. -••Ton [ạ? đường thảng d cát các tia Ox, Oz và Oy thừ tự tại M, N và p vả N hằm giữa M và p. Vỉ dụ 1. VỄ hai gàc kế bù xõy vầ ỹÕz vứl xõy < 90°. VỀ Lia phân giác Ot của xOy vá tia phản giác Om cũa tÕ"z . Hòi trong ba lia Oy, Oz. ũm Ha náo nằm giữa tiai tiâ còn lại? Vi xOt vâ tbz là hai góc kể bìi nên ta có t6z=»180 ũ -xâl = zÕm = (ISO 0 - xốt) : 2 = 90° - < 90° Mẹt khảc zOy - 130° -xỡy > 90° (vỉ xOy<90°) Do đó zQm<zÕy. Mã tia Om vá tia Oy cùng thuộc nửa mặt phảng có bờ chứa tia Oz. Suy ra tia Om nằm giữa hai tia Oz vá Oy. Dạng 2. Tinh số do của góc • Phương pháp gìảt. Nốu lia Oy nám giữa hai tia Ox vá Oz thi xOy + ỹÕz = xÕz. Vỉ dự 2. Vẽ hai góc ké bú xõy và yũz thỏa man yOz = 48°. Vẽ lia phân giác Ot của xũy . Tíong góc tOz vẽ gỏc lOm = . 3
  123. Tính gót xõm.
  124. Tính gót yOm. Lòi giải.
  125. la có xOy + yQz=180° {hai gót kể bù) = x6ỹ = 130° - 48° = 1 32° => xõu tÕy =1 32° : 2 = 66° (vi Ot la tia phân giác cua xũy ) Ta cá tOz-i-tÕx=ieo a {hai góc kế bù) => lốz -180° -tõx -130° -66 ữ 7,114° -ĩ lõm = ^ =1 14° : 3 = 38°. 3 VI lia Ot nằm trong xôm nên xồm = xồt + tõm = 66° + 38° = 104°.
  126. Vì lia Om và tia Oy còng thuộc nứa mặt phẳng Cố bở chửa lid Ot và lòm < lOy nền tia Om nẳm giữa hai tia ot vá -Oy, suy ra yốrn = tõỹ - tũm = Sô - 33° = 23°. Dạng 3. Chứng minh hai tia dôi nhau • Phương pháp giéì Nếu hai tia Qy và ũz cúng thuộc nửa mặt phẳng có bờ chứa tia Ox sao cho xOy = xOz thì hai tia Oy và Oz trùng nhau. (Xam tiếp trang 6) HỆ HAI PHƯƠNG TRÌNH BA Ẩn MAI VĂN NĂM (GV. THCS Khánh Hông ; Yên Khánh. Ninh Binh) Khi giải hè phương ‘rình néu sô In bầng 50 phưong trình thì ta thường dùng phương pháp còng hay trừ hai phương (rỉnh hoãc dùng phương pháp 'ihế. Nhung đốd vđi một 50 bái tũán mà số In nhisu hơn 50 phương trinh thỉ phải có cách giải quyết riêng. Bài Viết này chúng tôi xin giởi Ihỉệu VỚI các bạn một số phương pháp giải hệ 2 phương trình 3 ẩn.
  127. $ử dụng đìlu Kiện có nghiệm của phương trình bậc 2 Vi dụ 1, Giẻi hệ phương trình 111,- — + — -t-— = 5 X y z ẻ-i- 25 - xy z 2 Lời gtál. ĐKXĐ X. y, 2 0. 111 Pặt - - a; — - b; - - c . hệ đã cho trỏ thánh ' X y ĩ J . 'a + b = 5 -c a - b + c = 5 3 1 25 + c 2 (I) 2ab-c 2 =25 'ab = 2 Khi đó a.. b la các nghiệm của phương trinh t 2 -{5 - c)t + — “ 0- (2) Để phương trinh (2) cỏ nghiệm thi . I r 2 , ntt ASO fc»(5-c) a -4. T 0 2 (c + 5) 2 < 0 « c = -5. Thay c = -5 váo hệ phương trinh (f) ta dược a-t-b = 10 a=5 ab = 25 b = 5 Từ đo ta tim được X = y = 4; £ = — 5 5 Vậy hệ đâ cho có nghiêm duy nhất (x; y; z) là f\' 1. T

    J'ỉ r ~5) Vi dụ 2. Giải hệ phương trình ịx 2 + y 2 = 4z-5 -l-2xy |x 4 +y 4 = 9z-ẽ-4z z -2xV Lời giải. Ta biộn đồi hệ đa cho vẽ dạng I X 2 + y 2 - 2xy = 4z - 5 I X 4 + y 4 + 2x 2 y 2 = 9z - 5 - 4z 2 c* Ị[x-y) 2 =4z-5 i(x 2 + y 2 ) 2 = 9z-5-4z 2 Hệ (II) có nghiệm suy ra (II) |4z-5 > 0 |-4z 2 -h 9z - 5 > 0 «4 4 -z-ị 1< Z <§ 4 4 Thay 2=4 váo hệ (ll) ta tim được X =y = Q. 4 Thừ lại thấy dúng, Vậy hệ đã cho có nghiệm duy nhất lạ (x; y; z) là

    5 ữ; 0; " . 4 Vi dụ 3. Giải hệ phương trình I X 2 + y 2 + z 2 + 2xy -zx - zy = 3 I X 2 y 2 + yz - zx - 2xy = "1

    Lờì giải. Biển dổi hệ đã cho VỂ dạng j(x + y) 2 -z(x + y) + z 2 -3 -ũ Ỉ<K- yf -z(x-y) + 1 = 0 Đặt a - X +■ y, b - X - y, khi dó hệ trên có dạng |a 2 -za + z ? -3=0 (1) (|||} |b 2 -zb+1^0 (2) Hệ {III) oó nghiệm khi và chỉ khi phương trinh (1) có nghiệm a và phương trinh (2) có nghiệm b suy ra Aị ao I z z - 4(z 2 - 3) > 0 jz 2 <4 A 2 > 0 I z 2 - 4 > 0 I z 2 > 4 «z 2 -4oz-±2. • Với z - 2, thay vào hệ phương trình (III) ta có a = b= 1 =9 x= 1; y=ữ.

    Vtìi 2 = -2 thay vào hệ phương trinh (III.) la có a = b = -1 ->x = -1; y = 0. Vậy hệ đâ cho có hai nghiệm (x; y: z) lá (1: 0: 2); Hí 0; -2). 2 . $ử dụng bâ’t dảng thứq Vỉ dụ 4. Giài hệ phương trinh 1 X 4 + y 4 + z 4 - 125(x + y 4 z} (1) i xyz - 1 25 (2) Làt giiL Âp dụng bất dẳng thức a * 2 4 b 2 + c 2 > ab
  128. bc -I- ca, đẳng thúc xây ra khi a = b = c. Do dó X 4 4 y 4 4 z 4 > x 2 y 2 4 y 2 z z + z z x 2 > xy.yz 4 yzjx 4 zx.xy = xyz(x + y + z) = 1 25(x 4 y 4 z). (3) Dấu bằng ỏ (3) xảy ra khi X = y = z = 5. Vạy hệ dã cho có nghiệm duy nhất {x; y; z) là (5; 5; 5). Ví dụ 5. Giả? hệ phương trinh I Vx + 1 1 4 + 1 1 + 4z 4 1 1 =12 Ivs-x + ^s-y - = 6 Lòi giải. ĐKXĐ: -11 < X < 9: -1 1 í y < 9; -11 <z<9. Áp dụng bất đảng thức Bunhiacũpxki ta cù: 12 a = (WxTĨĨ +1-Jv -t-11 + 1 V 2 + 11 ) 2 < [ì 2 4i 2 4i 2 ]Ị(Vx +11) 2 +{Ãĩf H-Jz +11 'Ý Ị = 3(33 4 X 4 y 4 z) = X4y4ZSl5.fl) Dáu bầng ở (1) xảy ra khi Vx 4 1 1 - t/vTTĨ = Vz4ii <=f X = y = z. 6 2 = (W9-x 4 4 lì/^7) 2 < [ì 2 4l 2 4l 2 ”[(V9“X) 2 4(V&-y) 2 + (Vs-z) 2 ] -3[27-{x + ỹ+z)| =v X 4 y4Z < 15.(2) Páu bang ở (2) xảy ra khi Vỹ - X = ^9-y = Vỹ-Z fex = y = z, Tứ (1) VÉ (2) suy ra fx + y 4Z = 15 _ 4 X = y = z = 5.

    [x = y = z Vạy hệ có nghiệm duy nhất (x; y; Z) lá (5; 5: 5). VI dụ 6. Tim các sổ dương X, y, ĩ thảa mãn |x 4 y4Z = 15 I Vi 5 h + yz 4 Vi 5y 4 zx 4 Vi 5z 4 xy — 30 Lởi giải. Áp dụng bất đ^ng Ihírc AM-GM la có Vi Sx + yz - V(x 4y 4Z)X 4yz = +y)(x 4z}

    X 4 y + X 4 z 2x 4 y 4Z

    2 ” 2 ' Vi 5y 4 zx 2y 4 X 4 z

    ĩĩĩ<ĨLtlll. Cộng vế ỉh(?Q vế ba bất đẳng thữc trên Sa được ì/l 5x 4 yz 4 I5y 4ZX 4 /l 5 z 4 xy < 2(x +y tz}- 30. Đẩng thức xảy ra khi X - y - z - 5. Vậy hệ đã cho có nghiệm duy nhất (x; y- z) lã (5; 5; ịb Vỉ dụ 7. Giải hệ phương trinh Ị X 4 y 4 z = 0 (8*48*48* = 2* 4 2* 42* Lời giải. Đạt 2 K = a, 2* = b, 2 Z = c íhì a, b, c > 0 và abc = 2* + - z = 1. |abc = 1 Khi đó ta có - I a s 4 b - c s = a 4 b 4 c Vì a, b. c > 0 nên ãp dụng bấE đằng thúc AM-GM la cá a 3 +1 + 1>3^ = 3a b 3 4l4l>3ÍỈ? = 3b c 3 4l4l>síc 3 ' = 3c cỏng theo vế các bẩt đẳng thức trẽn la đươc a 3 + b 3 - c 3 4 6 > 3(a 4 b 4 c). (1) Mặt khác theo bất đãng thức AM-GM la cố a 3 4 b 3 4 c a s 3abc = 3.1 = 3 nên 2(a 3 4 b^+c 3 } se. (2) Cộng theo vế tũa (1) và (2) suy ra a 3 4 b 3 + c 3 > a T b 4 c. Đẳng thút xảy ra khi a = b - c = 1. Từ đỏ 2 - 1 => X - 0 2* - 1 => y - 0 2 Z =1 => z = 0, Vệy hệ đã cho có nghiệm duy nhất {x; y; z) là (0; O; 0), Bài tập vận đụng Bải 1. Tim các 50 dương X, y, 7 . thỏa mãn X 4 y 2 4Z 3 =3 1 2 3 — + — + — = 6 K y z Bài 2„ Tìm các SỔ dưang X, y, z thỏa mãn Jx 2 4y 2 4z 2 =3 Ix4y4-z-xy4yz4zx=6 Bài 3. Giải hệ phương trình jx 4 y4 z = 16 i i/x 4 y 4 y 4 z 4 Vz 4 X = 3 2 TIN TỨC - HOẠT Ngây 22,1 .201 5, NX8GD tại Há Nội đã tổ chức Lẻ kĩ niệm 10 nãm thành lặp. Tởi dư c6 TS. Ngoyẽn Thị Nghía, Thử - trưởng Bộ Giáo ởục & Đào tạo: 6ng Chu Vân Hốa. Cục trưởng Cục Xuất bán - In vá Phát hành, Bộ Thõng tln vá Tru yến thõng: NGUT. Ngô Trán Ái, Co vấn cao cấp HĐTV. BTGĐ. Trướng ban chì đạo biên soạn SGK mãi NXBGD Việt Nam; ông Mac vãn Thiện. Chù tịch HĐTV NXSGD Viậl Nam: ŨS. TS. Vú Vãn Hùng. Tổng Giám đốc NXBGD Việt Nam; Lãnh đạo các dơn vị thánh viên NXBGD Việl Nam; Nhản dịp này NXBGD lại Hà Nội đã đuọc Thủ tướng Chinh phủ tộng Bằng khen. Trứng hai ngày 22 và 23.1.2016, lại Hả Nội. ĐỘNG - GẬP Gỡ NXBGD Việt Nam dã tổ chức Hội nghị tổng kết cóng tác năm 2015 và tnển khai ke hoạch nãm 2016, Tạỉ Hội nghị, TS. Nguyễn Thị Nghĩa, Thứ trường Bộ Giáo dục & Đào tao đã trao Quyết định bổ nhiệm TS. Phan Xuân Thành, ùy viên HĐTV, Phó Tổng Giám đốc NX8GD Việt Nam. Giám đốc c&ng ty cổ phấn Dịch vụ xuất bản Giáo dục ta< Há Nôi giữ chức vụ Tổng biên Lập NXBGD Việt Nam, ' Ngáy 22.1. 20 16 Bộ trường Bộ Giáo dục và Dão tạo tặng danh hiệu Tập thể lao động xuất sác vá Bằng khen cho tạp chỉ Toàn Tuổi thơ và TBT tạp chi, TTT CMEEB TÌM SO CON THIÊU Bãl 1 , Cho các số 2. 5, 11 , 17, 23, 29, ... Hãy tim sò tiếp theo sao cho hợp lógic. Bàl 2. Hãy tim 50 còn thiéu trong hĩnh vuông. NGUYỄN Đửc TẤN (TR Hổ Chi Minh) ỵmm sõ NÀO THÍCH Hựp? [TTT 2 số 154Ỉ Nhận xét. Ki này cả hai bái dếu dẻ. Tẩtcâ các bạn đều tìm ra đúng quy luật của bài 1, l.ưu ý: lổng của ba số chia cho 3 gọi là trung binh cộng cùa 00 số đó. VỬI bãi 2, túy theo cách nhìn các hình dưới những khía cạnh khác nhau má ta phát hiện ra các quy luật khác nhau. Với quy luệl “vẽ Minh bầng một nét" cảc bạn nên chĩ ra cách vẽ cụ thể. Quy luđt. Bái 1 . SỐ nầm trong hình lam giác hãng trưng binh cộng của ha số nấm ngoái tam giác cộng vớĩ 1 . Bài 2. - Vứi cách nhìn sớ hình có mệt Ba hình A,
  129. c đcu có 4 tam giác đơn. hình D Cữ 5 lam gsác đem. Vậy hĩnh D không Ihíeh họp với cảc hình còn lại.
  130. Với cách nhìn ghép hình: Các hình A, B, D dược ghép bửi chỉ các hình tam giác, hình c dược ghép bỏi các lam giác vá hình vuông.
  131. Với cách nhìn vẽ hỉnh hằng một ỉĩẽt Các hình A, c, D đều đưọc vặ bằng một nét (không nhấc bút lẻn) hình B phải vẽ bằng 2 nét. vậy hinh B không thích hợp với các hình côn lại. ní . . Xin trao thưỏng cho các bạn có lội giải ỵ'.:i " chi tiết, dien đạt chính xác: Vũ nến Hải, 7A1; Nguyền Đăng Duy, 7A2 r THCS Yẽn Phong. Yẻn Phong. Bắc NÉnh; Nguyền Thành Dũng, 6D. THCS Vĩnh Yên, TP. Vĩnh Yên: Mai Thanh Tâm, 7A, THCS Lý Tự Trọng, Binh Xuyên, Vĩnh Phúc; Nguyễn Tiến Đứũ 6B. THCS Hoàng Xuân Hãn, Đúc Thọ. Hà Tĩnh. Các bạn saụ đuợc tuyên dương kì náy: Nguyễn Tiến Phong , 7A1. THCS Yên Phong, Yên Phổng, Bắc Ninh: Nguyền Thùy Mãi, Nhõm ba bạn Nguyễn Quang Thọ, Phùng Quốc Làm. Trần Tuấn Anh, 7E1 . THCS Vĩnh Tướng. Vĩnh Tưửng, Vinh Phức Nguyễn Chí Cũng, 7A3. THCS Lãm Thao, Lảm Thao, Phũ Thọ; Tệp ihềiởp 70. THCS Bạch Uêư, Yèn Thánh, Nghệ An. NGUYÊN XUÂN BÌNH UN1T 17. PARTICULATE MODELS FOR SOLIDS, LIQUIDS AND GASES Solld Llquld Gas Arrangemerki of particles Pixed pattern Free to mova about. No fixeơ positions Free tủ move about. No tixed positions □istance betwe@n particlas Closely packed togelhộr Nol as eloseíy packed Far apart Fởrées of attraetion between parlicles Very strong strong Very negligiblQ íorces MũtsQri of partỉcles Vibrate aboưt their fìxed positions Vibrate and move about their positions Move freely at high speeds Nhện xét. Củ rát nhiêu ban củ đâp ân đúng, Lỏa scạn sẽ ưao quá chữ bạn cú lời giải đúng . . vá trinh bày đẹp là: Phan Thị Việt Linh; Nguyễn Thị Kim Chì; Vũ Thẻo Nhi, 7C, THCS Bạch Liêu. Yền Thành, Nghệ An; Nguyễn Vẳn Nam, 7A, THCS Vĩnh Yên, TP. Vĩnh Yên; Lê Đức Thái, 6A2. THCS Yẽn Lạc. Yẽn Lạc. Vĩnh Phúc; Trần Diệu Unh. 9B. THCS Nguyễn Thuạng Hiền ửng hòa; Phan An Khành, SA2, THCS Giảng va. Ba Đỉnh, Hà Nội; Nguyễn Thị Thu Trang, &A1, THCS Yên Phong. Yên Phong. Bẩc Nỉnh; Phạm Thu Ttìùy, 6A. THCS Thị irấn II. YỂn Lặp. Phủ Thọ. MAI VÙ CÁC DẠNG TOÁN VẼ GÓC Vỉ dụ 3. Vẽ hai góc kế bù xOy và yOz thòa man yồz = 48°. Vẽ tia phân giác ot của xOy . Vẽ tỉa On lã tia đối của lia Oy. Vẽ tla phân giâc Ou cửa nOz . Chứng minh rằng tia Ũ J và ũt là hai tia đõì nhau. Vẻ tiã Ov là Lia đố! của tia Ot. Tương lự ví dụ 2 ta cố zồt = 114 ũ . Ta cộ zồv +zOt = 1BQ U =► z'Òv =1 60° - zồt = 1 80° - 1 14° = 66°. Mệt khác zỏy + zốn - 1 so 1 ' =» zÔrt = 180 U - zÔy = 180° - 48' J = 1 32°. (Tiếp theo trang 2) Vì tia Ou là t?a phân giác cũa zQn nên zốu = zũn : 2 = 1 32° : 2 = Ôồ ù . Vì tia Ou vá tia Ov cùng Ihuộc nửa mặt phẳng bở chứa tia Oz vả zOu = zOv nên tỉa Qv vả Ha Gu trúng nhau. Vậy hai tia Qu vá ũt là hai tia đổ! nhau, Các bạn hãy giằi các bãi toàn sau nhé. Bải 1 . Vẻ góc xõy = 1 20°. Trong góc xõy vẽ tia Oz sao chơ xũz > 2 zOy. Vẽ Isa phản giác Ot cũa xOz vả tia phân giác Om cũa tOy. aj Hỏi trong 3 tia GL, Om. Oz tia nào nằm giữa hai tia còn lại?
  132. Cho zÕy = 30°, hãy tính rĩi"õz. Bài 2. Vê hai gỏc kể bú xOy và ỹÕz, vẻ tia phân giác ot của yOz. Chímg minh rằng &) xõt =90° 2
  133. Cho xOt = 145°. tính yOz. tìến thành phổ lạ íĩrn người chứa quen Nhờ người Um giúp đường, gỉủp phổ Có nhỉỉng con đường đi qua ruộng lua Cùn khóm tre rừng cua thuở nguyên sơ Nhà í ấn khuất diỉời vòm xanh ỉã Người cầri gập chẳng dẻ tim UƠẺ lạ Phường tới phường qua mấy quá đồ/ xa Một chuyền đi dạo phổ trời mưa 12.4.1965 à LẼ THỊ NGỌC THŨY (Trường CDSP Nghệ An) Mỗ ị tàn gịớ Toán Tuối thơ bao nhiêu ỉ? É an bất ngờ hiện ra ngỡ như gập bạn gồn xa mỗi bài toán mpí bòng hoa ngạt nqăo Bao nhiêu cđu hỏi Vĩ 500 ? bỗng lung linh súng như sao trên trời bao nhiêu câu chuyện vui cười hóa dòng sõng lẩm mát thời ĩnộĩig mơ Mỗi trang báo Toàn Tuổi thơ kết thành đõ/ cánh từng gỉờ nâng ta 9.2015 1 ru Ị NGÂY TOÁN TUÕI THƠ Sáng ngảy 10.1.2013 lạỉ Há Nội, lạo thi Toát Tuổi thơ dã lố chitc Ngáy Toan Tưổi thơ. Đến dự có GS, TS. Vỡ Vãn Hửng, Tổng Giám dốc. Tổng biãri tá 3 NXBGO Viẻt Nam; bã Do Thi Phương. Phó Giám đốc NXBGD lại Há Nội; õng Phạíĩì Quýnh. Phó Giám đốc Cõng ty cổ phấíi sâch Giáo dục lại TP, Há Nôi: TS. Đinh Van Vang, Tding biên tập tạp chí Vãn học và Tuổi Irẻ: Đại diện Phòng Giấo dục tiểu học. Phòng Giáo dục Trung học của sỏ Giáo dụt và Đáo lạo tác tình phía Bểc; NGUT, Nguyễn Thị Hiển, Hiệu Vưởng trưởng tiểu học Đoán Thị Điểm Há NỘI; ỏng ' Hoàng Manh Anh, Phở Tổng Giâm dổc Cõng ty Co phấn Ván phỏng phẩm Hồng Hà; bả Đinh Hương Ly. Đại diện Trung lâm phát triển tư duy vá kĩ nang EG. ông Ệùi Minh Mãn, Giám đốc Công ty cổ phần dịch vụ Giáo dục Việt Nam; õng Phạm Thọ Hoán, đai diởn websit 0 : olm.vn; TS. Chu cẩm Thơ.. Giám dổc Tnung tăm Toán POMath: các thẩy cô giáo nguyên lá lãnh đạo Tạp chí; các ủy viên Hội dđngbién tập Tạp chí Tnân Tuổi tha. cát câu lạc bộ Toán Tuổi Ihơ dển lừ; Nam Dinh. Thái Binh, Sơn La, Hitag Yên, Quáng Ninh, Vĩnh Phúc, Hả NỘI vã các am học sinh khối lòp 5 trướng tiểu học Doãn Thị Giếm Hã Nỏi,
  134. Ki niệm 15 nám Toán Tuổi thơ
  135. ThS. VO Kim Thụy, Tổng biên tập lạp Chi Toán Tuổi Ihơ đã có hái bão cáo vé 15 năm hoại động cùa Toàn Tuổi thơ.
  136. GS.TS. Vũ Ván Hùng, Tong Giám đtìc. Tổng biốn tập NXBGD Việt Nam đa phát biếu ghi nhận những cổ gáng cùa Tạp chi trong 1 5 năm qua vá đế nghị Toán Tuổi thơ cẩn cố gắng để phục vụ bạn dọc ngáy câng lot hơn.
  137. Ông Trán Quốc Bình, Phó phòng GD - ĐT TP, Sơn La, dại diên các cộng tác viên của Tap chi phát biểu.
  138. Em Nguyễn Van Thanh Sơn, học sinh ỉởp 8/1, THCS Nguyên Khuyến. Đã Nang, đoạt giâ- Váng cuộc thi dặc biệl nhân 15 nam Toán Tuổi thơ đai diện các am học sinh oó đôi tài Lám sự. chia sê và nhãn học bong cùa Uiebsite: olm.vn.
  139. Trao thướng các Cuộc thi trẾb tạp chi
  140. Cuộc thi ra đé kiểm Ira, đề thi toán,
  141. Cuỏc thi dạc biệt nhãn 15 nam Toán Tuùi thơ.
  142. Cuộc thi Iìiti hiếu Cộng đồng ASEAN.
  143. Cuõc thi vui cháo hè 201 5.
  144. Ta Chức thi dấu câu lạc bộ loãn Tuổi thú IIẺn tinh chơ 9 cằu lạc bộ Lấn đáu hên đã thi toán lá bằng tiẽíng Anh. yèu cáu ghi đáp 5Õ có tên dơn VỊ, Cuộc thi gâm hai vrtng:
  145. Vỏng 1. Thi đổng đội Hiệp 1. Tiếp sức lộậiì

    Điếm ỈTÌÓÍ cua CLS Toàn Tuấi thơ tà Hiệp 2 {Du tịch Toán học) và Vòng 2 Ợranh giải Nhất, ị.

    Hiệp t. Du ÌỊCh Tơáíĩ hoe (Đây lá oách thi họgn toàn mới) ' Cố 6 thánh pnó cho các bạn học sinh đến tbạm quan là Hả NỘI, Hải Phòng, Nam Định, Huở, Đà Nàng, TP. HỂ Chí Minh. Hai giám khảo lá chủ nhằn cùa mo- thành phố dó.
  146. Các em hpG sính Vong c£u lật bộ eùng g-ải 6 bâi toán (nong (hài gian không quá 30 phút. Mỏi cãu lạc bộ phải giãi dừng bài 1 tni mỡ riưoc di chuyển đơn thành phũ Ihúr hai nhận để bài 2 để giải Liơp, cứ tiếp lục nhu Ihế cho đến bái 6. Diểm lơi đa cùa mõi cãu lạc bộ ỏ Hiệp 2 Eá 12 d:Ểm.
  147. Kếl quả cf vòng 1 đơạc tính bằng tổng điểm cá hai hiệp để chọn ra hai câu lạc bộ cố điểm cao nhất. • Vòng 2. Tranh giải Nhất
  148. Gốm 3 hiệp, mỗi hiệp hai cãu lạc bộ cùng giải một bài toán.
  149. Mỗi cẽu lạc bộ nhẠn một bảng để ghi kết quá váo bàng đâ. Thơi gian lãm mẻi bái toàỉi lá 5 phút.
  150. Mồi bái gài đúng được 2 điểm, giải sal đutít ọ điểm.
  151. Ban tổ chừc sẽ cộng đỉểm ở hai vòng Ihi để xep giẳi.
  152. Kết què; G ải Nhát; câu lạc bộ Hà Nội 1 dơn từ trưởng (lểu học Đoán Th[ Điểm Há Nội: giái Nhỉ' cáu ỉạc bọ Hã Nội 2. đến lử trướng tiểu hoc Doàn Thị ữiỂm Há Nội. Các đóng giải 8a lá cốc câu lạc bộ rĩếrt từ lấường TH Nam Đáo, Nam Trực, Nam Dinh; inlâng TH Lẽ Hổng Phong, TP. Thái Binh, Thái Bỉnh: vướng TH Thị Trấn Vĩnh Tiãng, Vĩnh Phúc; tnrông TH-THCS-THPTĐoàn Thi Điểm, TP. Hạ Long, Quáng Ninh: trương TH Đoán Thi Điểm Ecopark, Vàn Giang, Hưng Vận; Irưồng TH Ban Mai, Hà Dòng. Hả Nội; TP, Sơn La, Sơn La. TTT THƯ CẢM ƠN Trong Ngáy Toán T-Jố; 111 Ơ cò sự h ệr diện và lặng lãng hoa lang quà chúc mừng của: GS. TS Vũ Van Hụ na Tổng Giám đog, Tíỉrig tren lập NXẼGDVN lãnh dạo NXBGD tại Hã NỌi: lãnh dao Cõng ty Cơ phán sách Giác dục tại TP, Há Nôi lÃmh đạo Cõng ty Cổ phan VPP Hống Ha: dại diện Trưng tâm phát triển tu duy vã kĩ náng IEG; lãnh dạo Xí ngh : ệp In bán dớ 1 - Bộ Quốc phỏng: dại diện tạp chi Toán học vá Tuổi trè; lãnh dạo lạp chi' Van học vá Tuổi tre, Phó Trưỏng phùng Giảo dục va Dào tao TP. Sơn La. Cãc co quan, các công ty, đơn vị LÚI trạ. NXBGDVN. Câng ty cổ phấn VPP Hflng Hà. dutìng TH Đoàn Thi Điểm. Hú NÒI w£bãiLủ hóímai.vn websi Ltd : otrn.Yiì; ĩrung l-ảrr phút Lrẹn lự duy vú kĩ năng IẼG- diện Phóng Ộ.HƠ dục Tiểu họn v& Phóng Giác dục Tmng hợc tủa sd G : ?-ơ dục vã Đáơ tạo các lirỉh phía Rết; đại diện Phông Giác duc vá Đao tao Q. Nam Từ Liêm Há Nội; dại diặn lãnh dạo các cóng íy. các dơn vị: các ủy viên Hội đống biẻíi tập cứa Tạp chí; các tinh nguyện vièc; dại diện các Phòng giáo dục vào Đão tạo. Ban giám hiệu, giảo viẻn vã các om lìữc sinh trong các câu lạc bó Tũẳn Tutỉi tho đến lử cát tình. Ihảnh. Hung Yên. HỀ NỘI. Nam Đinh. Qui lìạ Ninh, Thái Binh, Sơn La, Vinh Phút; các: phóng Vicjn &à( Truyén Hình VỂI Nam. kênh VTV2; sáo Thanh niẻh, bào Giẩtodục Ihài dại. Đái I rũyền hirh Việl Nam kênh VTV2 đã oó 4 chuơĩig trinh phét sỡnợ irong các ngày 24, 25, 26 vá
  153. 1 .2016 dưb Ún vớ Ngáy Toán Tuổi thơ. Tạp chi irãn vọng càm ơn, TẠP CHÍ TOÁN TUỔI THƠ ĐỂ THI 2 VÒNG CLB TOÁN TUổl THỚ LIÊN TỈNH 19.1.2016 CHILDREN’S FUN MATHS 10URNAL COMPETITION 2016 ROUND 1: RE LAY RACE - Duration; 30 minutes for 6 problems Problem 1. Caieuiate _5 3 7 9 12x17 4 34x10 4 60x9 1 27x36' Probiem 2, Find the smahest whole nurmber that, when divídedi by 9, 5 and 4, teaves remainder of 1, 1 and 3 resp&ctívety. Problcm 3. Gi ven thai the da te ôf 20.11.2010 IS a Saturday. Which day of the week is 20.1 1 .201 8? Problem 4. A rhombus has diagonals of 60 cm and 30 cm, and a hesghloí 48 cm. Find the length of its 5td&5. Problom 5, R.nd two dislinct numbers, given that their sum ls three limes their differenee ? and their product is eight lirnes their difference. Preblem 6. The íolỉovving sequence ũf numbers was wrilten on a board: 1, 2, 3, 4 200. Uyen erased threeoonsecutáve rnụmbers and (he sum of thạ rẹmaining numbers ig. 19646. Pind the three numbers that Wí»re erased. ROUND 2: MATHEMATICS TOUR - Duratíon: 30 min Lite s for 6 problsms The first City: TP. Hõ Chí Minh Problem 1. Find the dĩgits a and b such thai the nunbnr 2Ù1Êab 13 divisióle by 2 and 9, and has a remainder of 3 when dsvỉded by 5. Proceed to: Đà Năng Problem 2. Find (he whole number X, given Ịhat: 1 + 5+ 9 + 13 + ... + X = 760. Procced to: Huê Problem 3. Find X Such thai. Ịu^Ịỉí 1+4 xíi+-l'x., 1 xỊi+lỊiooe4 ( 2 i L 3 J { V 1 2 Proceeđ to: Hàs Phông Probiem 4- The priee of a (ype of chaĩk in August dmpped by 10% compared lo Ihat in June ; but increased by 10% m Octoher compared to thai ỉn Augusl. How many percent has the price increased Oí decreaaed from June lo October? Proceed to: Nam Đinh Problem 5. Find the sroallest whole number such that when it is multiplied by 12345679, the nesuitiíig pnoduct Is a runbLT híiving all of Its dgits equal Eo 3. Proceed to: Hà Nội Prũblem 6. Reíer lo the follũwing diagram: Gsven that Sạbc =4 cm 2 p S ADE = 14 om 2 , DB- CE = 1 cm, BC = 2 cm. Find the lengih of DB, Proceed to: TP. Hó Chí Minh Trẽn dãy lả các để theo so dổ di chuyến của câu lạc hộ Son La ROUND 3: FINALS - Duratìon; 5 minutes for EACH protỉlem Problem 1. Find the number abcd given that abed3 - abờd = 653 * 5. Problem 2. A reclangutar coưrtyard has ỉls lenglh equal lo four limes its vvidlh. It is exlenđed on both its length and its widlh by 5 m each, The extendeđ reclangularcourtyard has an area biggerthan that of the original one by40ũ m 2 . Fỉnd the area of the original courtyandi, Probiem 3. Gsven a quadrilaieral ABCD. The I<nes AC and SD Éntersect & ị o. Given thai S OAE - 3 cm 2 , S ŨŨA - 6 cm 3 , S ABCD - 15 CÍĨ 1 2 . Find the area of the ỉriangle OBC. 9 ĐẼ THI CHỌN HỌC SINH GIOI MON TOAN LỚP 7 TỈNH BẮC GIANG Năm học: 2012-2013 Thời gian làm bài: 120 phút Cãu 1 r (4.0 ở iểm) , . (3 2,1 Y /3 2 1 ì
  154. RútgọnA = --9+ — . -T-T + — , ,2 5 10 1 ‘2 3 12, 2, Tim giá trị nhỏ nhất của biểu thức p - |x - 2012] |x - 201 3| vãi X lả số rự nhiên. Cáu 2 ,(5,0 (ỉiềm)
  155. Tìm X biết 2 + 2 ,y + '.$ = 10600.
  156. Ba bạn An. Bình và Cưởng có tổng số viên bi lá 74. Biết rằng số viên bi cũa An vá Binh tỉ ỉệ vỏi 5 và 6; sổ viên bi của Bình vá Cường tỉ lệ vin 4 vá 5. Tính số vien bi cũa mót bạn. Câu 3. (4.0 điểm)
  157. Cho p là số nguyên tổ lớn hơn 3. Chứhg minh rằng ọ 2 - 2012 tá hợp Ẹố.
  158. Cho n là số iư nhiên QÓ hai chữ sổ. Tim n biết n + 4 và 2n đáu tả cảc SQ chính phưưng. Câu 4r (6. Ồ điểmì Cho Lam giãc ABC cân lại A vá cò cầ ba góc đế ũ lá gốc nhọn.
  159. vể phía ngoài của tam giác vẽ lam giác A3E vuông cân ỏ B, Gọi H lá trung điểm của BC, bẽn tia đối của t*a AH lấy điểm I sao cho AI = BC. Chúng minh hai tam giác AB1 vá BEC báng nhau và BI ± CE. 2 . Phân giác cú 3 các góc ÃBC . âDC cắt AC. BC lãn Mợ! tại D. M. Phân gtác của góc BDA cằt BC tại
  160. Chúng minh rằng BO = Cáu 5. (1,0 điềm} 111 1 Cho S = 1-4+4-4 + ...+-4— - 2 3 4 2011 Tính {S - P) 2ũia . 2012 4 2013 Vá p = 1007 1000 ' J ' 2012 " 2013 ' 'mrm MÙA NOẼL ( TTT 2!S ,S4, Một mùa đông nữa lại vé. Theo nhịp diệu thời gian vã quy luật cũa tạo hòa. thu đi dõng trở lại. Khi cãi lạnh của nàng Đỏng bắt dấu đến cũng có nghĩa lá mùa Noèl đả vé. Vào dịp đđc biệt này, trong lúc bà chúa Tuyết đi khếp miển Hàn đửi và ủn đứi Bác bán cấu để thả những bỏng hoa 1 uy ốt thì ông già Nosl lại mang biếỉ bao món quá dành tặng các bạn nhỏ. Trẻ con háo hút dắp nhũng người tuyết vá thi nhau cất lẽn bái hát Menry Christmas. Hĩnh ánh nụ cười của những chú người tuyết như cháo đón đ&ng VỂ. Mùa dỏng âm dán lên nhờ những cày thỏng tháp đèn muôn màu, muôn sắc. Những ngày này chỉ cần ngắm cầnh đường phố vá ờ bên ngưừi thân cung đù làm cho ta cảm thăy ấm áp hơn. Nhử có múa dóng mà ta đã có những trải nghiệm luyệt vời như vặy. câm dn tạo hóa dã mang dõng dến với Trái đất nãy. . Nhặn Két. Cú rát nhiễu ban gửi bải. ụ . : ; , . Lòa soạn Xin được trao quà chữ cảc bạn cỏ lòi ván hay và giàu cảm xúc là: Dinh Thị Huyến Trang. EiA, THCS Nam Cao. Lỹ Nhân, Hà Nam; Lậ Anh Tuyềt, 7E1, THCS Vinh Tưàng. Vĩnh Tường, VTnh Phúc Nguyên Htíơng Ly. 6A, THCS Thị trấn Cao Thượng, Tãn Vền, Bắc Giang; Ngõ Minh Châu, 7C, THCS Xuản Diệu, Thị trán Nghèn, Can Lộc. Hả Tĩnh: Nguyền Thị Bàng Bàng, Đặng Phương Linh, Nguyển Tuệ An, 7C. THCS Bạch Liệu, Yên Thành. Nghệ An’ ĐỀ THI CHỌN HỌC SINH GIỎI MÔN TOÁN LỚP 6 QUẬN 9, TP. HỒ CHÍ MINH HUổnii đln Dlịl de lỉì Irưởc Năm học 2014-2015 BàM. a) A = 1 -I- 2 - 3 - 4 -I- 5 4 6 - 7 - 8 ■. 9 - ...
  161. 2Ũ11 2012 4 2013 + 2014
  162. 1 4 (2 - 3 - 4 4 5) 4 (B - 7 - B 4 9} 4 4 (2010 -2011 —2012 + 2013) 4 2014 = 1 + 2014 = 2015. . _ 2 2 2 2 b)B = 3.5 98 J 3 490 245' i Y . 1 Y* 1 Ì (« . lì l 2 Jl 3 Ẳ 4 J .. 2015 J 345 2016 2016 10ữ3 234 2015 2 234 2015 2 Bát 2. Thòi gian bạn Đức di dược lá 7 giờ 55 phứt - 7 giờ 10 phút = 45 phút = già. 4 Thời gian bạn An đi được lá 1 7 giờ 55 phứt - 7 giờ 25 phút - 30 phút - - giớ. 3 Quãng đường bạn Đứt đi đươc là 1 2,“ - 9 (km). 4 Quãng đưdng bãri An đi được là 16. - 8 (km). Quằng đường Aô dải 9 4 5 4 s - 22 (km). Bãi 3. a) 3(x - 2) - 60 - 24 » 3(x - 2} - 84 w X - 2 = 28 w X = 30.
  163. 2a1 5b : ode = 90 =í 2a1 5b - 90,cde =*2a1 5b : 90 =>2ÍSb:iO^b = 0. L|ỊCÓ 2a1 50 : 9 => (2 4 a 4 1 4 5 4 0) = (8 4 a) ; 9 => a = 1 . cữẽ = 21150: 90 = 235. Vậy abcde =102 35.
  164. Ta cò -T-.írr = -lzr là số tự nhiên nèn

    b 75 75b 16a : 75b=> a ; 75 vâ 16 ; b. 14 I4a Lại CỚ 7-- b 155 165b lá số tự nhiên nên I4a ; I65b =* Sk : 105 vã 14 : b. Để ị là phân stì lởn nhất thi a = BCNN(75 : 165) b = 825 và b = ƯCLN (16 ; 14) = 2. Vậy ệ = Bài 4. a)Ĩ 0 oó COD=-Ị-ÃdC; COÊ = ịôÕB 2 2 => DDE - ÕŨD 4 COE - ^(CQA 4ÕQB)

    1 DỒ E 1 = — AOB => = -r. 2 ÀGB 2
  165. Ta củ AOB í 1 80° rrên DỎE <-1-180° = 90°.

    2 Vậy giá irị lớn nhát cũa số do gúc DQE là 90°'. Bài 5. Giả sử khảng định Q tà đúng ->A I 51 có tận cùng lá 2 =■ Khẳng định p là sai (vì 50 có tận cùng lã 2 không là số chinh phương). Kht đó A- 3S có lân cùng là 3 =>■ R là khảng dịnh sai (vì số cô tân cúng là 3 Khàng lả sổ chính phương). Vậy Q là khẳrig dính sai và p. R lá hai khẳng dính đủng. © Giải toán qua thiẩ Bải 1(154). Tìm SỔ tự nhiên n thỏa mãn n -I- S(n) + $($(n}) = 96 (S(n) lõ tổng các ctíữ 50 của n), Lòi giải,

    Xét n - Ũ. không thỏa mãn, • Xét n > 1 . ta có 5(n} >1 = n < 95 =s S(n) < 17 =* S(S(n)) ẩ 9. Do đó n - 96 - S(n) - S(S(n}) > 96 - 17 - 9 70 -? 70 < n < 95, (1) Mật khác n = S{n) (mod 9); S(n) B S(S(n)> (mod 9) nẻn 96 = n + S{n) I S(S(n)) = 3n (mod 9) =: n = 2 (itiữd 3). (2) Tử{1) vả (2) suy ra n e (71: 74; 77: 60; 63: 66: 69; 92; 95}. Thử las chỉ có 77; 6Ũ; 63 thỏa mân. Vây cảt giá trị cán tìm cùa n là 77, 30. 83. Nhặn xét. Có nhiều em tham gỉa gíài bái vá có lới giải đúng. Xin nâu tên một số em trình bẽy dẹp hơn: Nguyễn Ngọc Mát, Nguyễn Tiến Đức, Trắn Đũít Tùng, 6B, THCS Hoàng Xuân Hãn, Đức Thọ: Phan Thị Thu Hoài , Nguyễn An Na, Lã Thị Hằng Nhi. Ph&m Ánh Ngựyệt, 8ùi Thị Minh Thư, Phạm Phương Chi, 7A. THCS Hoàng Xuân Hắn, Đức Thọ: Trần Phương Thảo, Nguyên Thị Thúy Trang. 6A, Lé T7?f' Phương Linh , 6B. THCS Xuân Diệu, Can Lộc, Hà Tĩnh: Nguyễn Tuấn Kiệt ; 7E, THCS Chu Vản An, Nga Sơn, Thanh Hóa: Phạm Phương Anh, 7A4, THCS Yên Lạc, Yên Lạc. Vĩnh Phúc; Phạm Thị Quỳnh Trang, 7C, THCS Bạch Liêu, Yên Thảnh, Nghệ An. PHỪNG KIM DUNG Bãi 2(154). Cho tam giác ABC có Â < 90°. M lã trung điểm cùa BC. ũựng các tam g-ác vuủng cân lại A lả BAD và CAE [D vả c cùng thuộc nửa mạt phàng bớ AB. B và E cùng Êhi*ộc nửa inàl phẫng bở AC). Chừng minh rằng AM vuông gòc với DE, Lời giải. Trân tia đối của tia MA lấy điểm F sac Cho MF = MA. Khi dó AAMB = ÁFMC (c.g.c). Suy ra CF = AB vá MẤB = MFC, Từ đá CF = AD vả CP//AB. Ta lại Cố BẤC + ẺÃD - EÃC + DÃB - 160° Mà BÃC +FC"A -180 Ử (do CF ỉ/ AB). Suy ra ẾAD ĨE FCÂ. Tử đó ADAE = AFCA (e.g.c). A Để ý rằng AB X AD, từ đó suy ra AF 1 DE hay AM l DE, Nhân xẽt CÓ nhiều bạn giải đúng bải náy. Sau dày là những bạn oố Idi giảỉ đúng hơn cả: Nguyễn Thúy Mai, Phùng Thị Thúy Dung, 7A1 THC5 Vĩnh Tilừng, Vĩnh Tilơng; Phúng Thị Thúy, 7A, THCS Vĩnh Yẽn, TP. VTnh Y&n, Vĩnh Phúc; Nguyễn Vũ Hà. Nguyền c óng Hiểu r p/ỉạm Thúy Linh. 7A3, THCS Lãm Thao, Lâm Thao. Phủ Thọ; Nguyễn Trọng Thuấn, 7C, THCS Nhữ Bá Sỹ. Hoàng Hóa, Thanh Hòa; Nguyễn Thu Hén, 7D, THCS Ly Nhật Quang, Dô Lương, Nghệ Ân: 8ùi Thỉ Minh Thư. Nguyễn Ngọc Ảnh, Afg(jyễữ An Na, 7A, THCS Hoàng Xuèn Hãn, Đức Ttìọ, Hà Tĩnh; Trần Huy Lị®, 7A, THCS Nhãn Hậu, LỄ Nhãn. Há Nam; Phan Thị Như Quỳnh , 7/5, THCS Nguyễn TTì[ MÉnh Khai. Cam Phúc Bác, Cam Ranh, Khánh Hóa. Hổ QUANG VINH Bát 3(15-4),. Giải họ phương trinh ị xỵ = X 4 2y - 3 [4x^ -y 3 = 24X 1, -45x I 15y I 4t* Lời giải. Hệ phương trinh cé Ihề viộì thành |y(x-2)-<-2)-S=0

    |4{x a -6x 2 +12K S) 3(x 2) y 3 15y 15-0 |y(x -2) ■ (x -2) 4 5 ° |4{x-2) 3 -3{x-2)-y J -15y-15 = 0 Đât t = X - 2. ta dươc hệ I yĩ = t + 5 (1) 1 4t 3 -3(t + 5 + 5y)-y a =0 <2} Thay t - 5 = yt. la ũỏ t 4 5 4 5y = yt 4 5y = y(t 4 5) = y 2 !, Phương (rình (2) Irử thành 4t 3 - 3y z t - y 3 = 0 <r> 3t(l 2 - y 2 ) 4 1 3 - y 3 = 0 »<t-y)(2t + y) 2 = 0 y = -2t

    THI, y = t, Ihay vào (1), ta được

    Do đó X = 1 4 2 - ; y = t = 1 ±y5ĩ 2

    TH2. y = -2t, thay vão (1), ta dược 2t z 4t4 5 = 0. Phương trinh này vô nghiệm, Vậy hệ phương trinh dã chữ Cữ has nghiệm (x; y) là: 5““v/2Ĩ 1-ìỄT ' ' 5 +ĩ, 1 4ĩ ' 2 2 V / 2 ; 2 \ / Nhận xét. Cỏ thể giãi bài toản bâng cách: Từ phương trinh thứ nhất ừong hệ ta có y = x+ 3 x-2 ' thay váo phương trinh thứ hai vá biển dõi. la dược (X 2 - 5x 4 1)(2x 2 - ?x + II) 2 - 0; sưy ra kết qưà như Irên. Các bạn saư đảy có bãi giai tốt: Tạ Nam Khánh, SE1, THCS VTnh Tường, Vĩnh Tướng, VTnh Phức; Nguyễn Minh Nghĩa . 9B. Nỳiiyễn Thị Huỹén Ngọc, 9C. THCS Nguyễn Thượng Hiên, ừng Hóa, Hà Nội, NGUYỄN ANH DŨNG Bai 4(154), Chơ a, b, c lá các số thưc không ânn cú Lổng bấítg 1. Chửng minh rằng a(b-c) 2 4 bfc -aỶ 4 e(a -b) 2 <. 4- 4 Lời giải, Không mất tinh tổng quát ta có Ihể gsả sử a - max{a, b, c). Khi dớ ta cô a(b - c) 2 4 b(c - a) 2 4 c(a - b ) 2
  166. a(b - c} 2 4 b(â - c) 2 4 c(a - b} 2 < a(b 4 c) 2 4 ba 2 4 ca 2 = a(b 4 c} 2 4 a 2 (b 4 c) = a(b 4 c)(a 4 b 4 c) = a(b 4 c). Ấp dựng bất đầng thức AM-GM ta có (a 4b + c) 2 1 a(b+c) £ — t = 4 4 Do dỏ a(b-c) 2 +b(c-a) 2 +c(a -b) 2 í 4 4 í 1 1 Đầng thức xảy ra khi (a.b.c) = ,-j ,0 hoán vị của nó. và các Nhặn xét. Bài toàn lr€n hay và khùng quả khõ vi thế cú rát nhiều bạn tham gia giài bái, hau hếl càc lồi giải dều dung. Những bạn sau dãy có lồi giải đúng vá ngắn gọn: Nguyễn Công Huấn, Lê Ngọc Hoa, 6E1, THCS Vĩnh Tưâng, Vĩnh Tưởng. Vỉnh Phữc: Trắn Sỹ Hoàng. 8C. Bùi Thị Minh Thư, Bùi Ektản Nhặt Totàng, 7A. THC5 Hoàng Xuân Hãn, Đừc Thọ, Há Tĩnh; Trấn Thị Trả My, 9C, THCS Ngưyẻn Thượng Hiến, ửng Hóa, Hà Nõỉ: Đinh Viết Tỵ, 9D, THCS Lý Nhật Quang, Đõ Lương. Nghệ An; £>ểng Quang Anh' 9A, Nguyễn Chích, Đông Sơn, Thanh H6a. CAO van dung Bái 5(154). Tinh số dãy chữ khác nhau thu được khi hoán vậ cãc chữ cái cùa đây chữ sau: TŨANTUOITHŨ, Trong các dãy chữ đó có bao nhiêu dãy chữ không có hai chừ “T' đứng liễn nhau? Lời giải. Dãy chữ TOAN TUOITHO cớ 11 ch ử cái trọng EÍÓ cớ 3 chữ cái T. 3 chữ cáỉ o và nam chữ cải A, N, u, I, H mọi chũr cái xuất hiện 1 lấn. Như vậy trọng 111 hoán vị các chữ cái náy ihì khi hoán vị các ch ử cái T và khi hoán vị các chừ cái o thì ta chỉ thu được dãy chữ như trước nên so dãy chữ khác nhau thụ được lả: 11! — =1105500. 3131 Ta bỏ 3 cho cái T thì tương tự như trên ta có &! 777 = 6720 dãy chữ khác nhau tử hoán vị cảc chữ 3Ỉ oải của riãy chừ OANUOIHQ. Tương I r mg vởl mồi hoán vị trên ta chọn 3- trong số 9 vị trí khãc nhau cho 3 chữ cái T (1 vị tữ ớ đau. 1 vị trí ò CLiổi và 7 vị trí xen giữa ô chữ cái của hoán vị đó). Số cách chọn vị trí cho 3 ch ừ cái T sao cho không có 2 ghữ cái T cạnh nhau là: cị = 91 = 84 . 9 3í(9-3)! Như vậy, có 672Ũ.84 - 564480 dây chữ cáĩ mả khởng có 2 chữ T 030 đứhg cạnh nhau. Nhận xét. Có nhiều bạn giải sai ý thử hai của bái toản này, Cãc bạn sau đây có lới giải lốt: Từ Tấn Dũng , 7D, THPT chuyên Há Nội - Amsterdam, Cáu Giẩy r Hà Nội: Đặng Quang Anh . 9A, THCS Nguyễn Chích. Đõng Sơn; Thanh Hóa; Lê Ngọc Hoa, BE1, THCS Vinh Tướng, Vĩnh Tưởng; Vinh Phửc, Nguyễn Vần cườn g, SA, THCS Hợp TiỂn, Nam Sách, Mải Dương, TRỊNH HOÀI DƯƠNG Sãi 6(154), Cho tam giác ABC có trục tâm H. Gọi M r N lẩn lượt ià trung điểm cùa AH, QC, Gọi p, G lần sượt là chăn đương vuông oóc hạ tử N xuống SH, CH. Chứng minh rang MN đi qua trung điểm của PQ. Ạ Lài giải. Gọi E lá giao đĩếm của BH vá AC; F lá giao điểm của CH và AB. R ỉà trung điểm của EF. Vi BE 1 AC; CF 1 AB; MA = MH; NB = NC nên ME = MF; NE = NF và QF := QC. Do đó MN Jã trLing trực cũa EF. Vây MN đi qua R. (1) Vì NO - NC; RE - RF; NP íỉ QE; NQ ịị BF nên NP CE // QR; NQ .7 BF ỉỉ PR. Do đủ NPRQ là hình bình hành. Vây NR di qua irung diểm của PQ. (2) Tử(1) vâ (2) sưy ra MN đi qua trung điểm của PQ, Nhận xẽt. Cà nhiẩu bạn Iham gia giải hái, tuy nhiên mộl vài bạn phải sủ dụng cỉịnh lĩ Thales và những kiến thiit VỂ đưởng tròn. Xin nẽu tên một sử bạn có lài giải tốt: Vũ Lỉnh Chì, 8A1 , Nguyền Lé Phưũng Ttìẳo, 9A2. THCS Làm Thao. Lâm Thao, Phú Thọ: Nguyễn Lẽ Anh Dũng. 9E 1 THCS vĩnh Tường, Vĩnh Tường, Vĩnh Phúc: Nguyền Minh Trang , 9A2. THCS Yên Phong, Yên Phong. Bầc Ninh; Nguyễn Vàn Cường, 8A, THCS Họp Tiến, Nam Sách. Hái Dương: Đặng Quang Anh , 9A, THCS Nguyễn Chích, Dong Sơn, Thanh Hoâ; Đỉnh Viết Tỵ, 9D, THCS Lý Nhật Quang, Đ& Lương. Nghệ An. NGUYỄN MINH HÀ í HOCMAI Từ số tháng 9 nSm 2015, Cõng ty cổ phần Dịch vụ Giáo dục Việt Nam sẽ tặng các khóa học trục tưyển trân web5Eĩe: hocmaì.vn cho các bạn học sinh được Ihưỡng trong các chuyên mực vã các bạn hạc sinh được khen trong chuyên mục KỂL quả thi giái (oán qua thư. Các ban học sinh sau khi nhận đuạc mà cung cáp Ihl đang ki tại địa chi. thcs.hocmal. Ưiì/toantưữitỉìù (Xin liên hệ SĐT 09&6464644 để được giãi đàp). 14 imm CHIA TỈ LỆ ĐOẠN TRUNG TUYẾN Bàr toán, Cho tam giác ABC (AB AC) vớt trung tuyến AD. Tron cạnh AB tấy điểm

    lữ sao cho AB = 3AM. trên cạnh AC lấy điểm N, Đoạn thầng MN cắt AO tạs E, ^LAU An ấ) Biết AC -2211 Tinh tỉ sổ

    2 AE An 7
  167. Tim vị trí điểm N trên cạnh AC để - ị. AẼ 3 ĐAN QUÝNH (Hà Nội) imsm có HAY KHÔNG? {TTT2 số 154) EB NB giác EF của tam giác BEC có 33 = 33 = 3. Như EC NC vậy bạn Vui và bạn Ve đểu noi đúng, Nhận xét. Vĩèc cho điéu kiện đối vúi gúc BÃC vả AC = 3AB Lã để dành cho các bạn Vui. vẻ đố nhau vui vẻ. Cách giài trẽn của bạn Nguyễn Phan ỊÉ!:.!!'. iLi Bão Tuyết, 9/1. THCS Nguyễn Thỉ Minh Khai, Carm Phúc Bắc, Cam Ranh, Khánh Hòa, kòm Iheo nhận xểt là không cấn sử dụng các già thiết về cạnh và gốc cùa tam giác ABC. Phẩn thưỏng kỉ hảy dành cho ban Bàù Tuyết. ANH CỦMPA Cho dường tròn tâm 0 và một dây BC. Gọi F lá điểm chính giữa của cung BC (cung lớn hay cung nhỏ hay nửa đưóng Inón đéư được). Lây các diồm M và N trên ổoạn thẳng 8C sao cho MC - 3MB vã NB = 3NC. Tia FM và tia FN cắt dưõhg fròn (O) lấn nửa tại D Vả E theo thứ tự. Theo tính chất của gòt nỏi tiểp dù FB = FC nên BDF = FDC và BEF = FEC. Ttieo linh chấl dường phân giác ĐF của tam giâc BDC ta cỏ 773 s = 3. Tưtmg tự vói dưóng phàn DB MB 3 ■ 3r G Kì NÀY Đãng Quang Anh, SA, THCS Nguyền Chlcb, Đông 5<Jn, Thanh Hõa; Đinh Viết Ty, 9D. THCS Ly Nhật Quang. Đõ Lưong, Nghệ An Nguyễn Thị Huyền Ngợc. 9C, THCS Nguyẻn Thượng Hiến, ừng Hóa. Hà Nội; Trấn Huy Lực. /A, THCS Nhân Hậu. ư Nhãn. Há Nam: Phan Thị Như Quýnh. 7/5, THCS Nguyên Thị Minh Khai, Cam Phúc Bắc, Cam Thi giải toán qua thư Ranh, Khánh Hòa; Lê Ngạc Hữá. ÔE1, THCS Vinh Tường, Vinh Tường, Vĩnh Phúc. Phạm ĩhùy Lình, 7A3. 7HCS Lầm Thao, Lãm Thao, Phủ Thọ: Nguyễn Minh Trang, 9A2. THCS Yên Phong. Yõn phong, Bắc Ninh; Nguyễn Ván cường, SA. THCS Hợp Tiến, Nam Sách, Hải □ưđng; Nguyễn An Na, Búỉ Thị Minh Thư. 7A. THCS Hoàng Xuân Hãn. Dửc Thọ, Hà Tĩnh. H òm nay thài tiết Ehặt đẹp, thám tử Sêlồccôc đống cửa Vân phòng sớm hdn một chút để tranh thủ dạo chtíi ngắm cảnh phố phường. ông khoan khoái ngẳm nhìn muôn vãn chỏi non đang nhii lẽn mơn mởn trên những háng cây bện dường. Khi thâm từ vừa dùng chân bén một gốc cày thì bất chợt chuông điện thoại vang lẽn, Hơi khó chịu một chút nhưng ông vẫn nghe máy. Thl ra là bà Nga, bạn cũ từ thời phđ thững, Bà Nga khẩn khoản nhà thám tử tới ngay nhà mình để giúp một vsệc quan trọng. Quên ca ngẳm cánh mùa xuân, thám tử lập tức đến nhá người bạn cũ. Bà Nga buồn bã kể:
  168. Bà bạn tôi mới mua một con rùa bằng vàng,

    rất đẹp. Thích quá nèn l

    i hỏm kia tôi đẫ

    mượn vé để bảo òng nhà tòi nay mai mua một con như vậy. Dịnh mang trả bà bạn luôn nhưhg bà ỷ lại đí váng đột xuất, thê’ Eà lỏi để tạm trong ngân kéo lủ. Ai ngở, kẻ nào dó đã lấy trộm con rùa đẳt giá đó.
  169. Bà phát hiện con rùa bị mất khi náo? “Trưa nay.
  170. Tức là sau khi bà mang con rùa vế nhà hơn một ngảy? -Vảng, đúng vậy. Tôi nghĩ con rùa chỉ có thể bị lấy cắp trong ngày hôm qua thôi.
  171. VÈ sao bà nghĩ vậy?
  172. Vi tổi hôm kia vợ chổng tỏi ò trong phóng suốt. Hòm qua thi chúng tòi đi váng cả ngày, chiếu muộn mởi vế nhà,
  173. Bà để con rùa trong ngãrì kéo tủ nào?
  174. Tủ áo trong phòng ngũ của vọ chồng tòi, -Từ lúc bà mang con rùa VỄ nhà cho tới khi phái hiện nó bị mất, trong nhà bá cố những ai?
  175. Ngoái hai vợ chong tõi thì chi có bá giúp việc vả hai đứa cháu thôi, -Hai dứa cháu â đấy thưởng xuyỀn ư?
  176. Vãng, chúng nà ở hần nhà tôi để đi học cho tiện, vì nhà tỏi rộng rãi, lại gán trường nữa, 'Tôi cln gặp riêng từhg người để hỏi xem sao.
  177. Vảng, tỏi sẽ gọi họ bảy giò. Trước ỉiên lá bã giúp việc tên Lan.
  178. Bà đă làm gì trong ngày hỏm qua khi óng bà Con rùa vàng NGUYỄN THỊ LAN (ỜA2. THCS Yên Phong. Yên Phong , Bắc Ninh) chủ đi vắng?
  179. TÔI đi chợ, don dẹp và tranh thủ tới phòng khám nha khoa để chữa râng.
  180. Phóng khảm có xa dãy không?
  181. Không, ở ngay đẩu pho, Mếu ông cấn địa chì, lởi cố thể đưa õng xem hỏa đơn. Tiép Iheo lả cậu Binh, cháu gọi bả Nga bâng di:
  182. Cậu có thl cho biết cậu đa làm gì và ờ đáu hôm qua khủng?
  183. Được ạ. Hôm qua cháu cùng mấy dứa bạn di xem tnển lãm, sau đó đá chơi lòng vòng đến tối mới vể ạ,
  184. Cãe cậu xem triển lãm gi thế? -Triển lãm các tác phẩm vãn thơ thời khàng chiến chống Pháp ạ.
  185. Thủ vị qua nhỉ? Chác mộl thự viện nao dó đa tổ chức cuộc triển lãm ý nghĩa đỗ?
  186. Vâng, thư viện "Sách vảng" ạ. Tnển lãm còn mỏ 2 ngày nữa, bảc có thể đén xem.
  187. Cám ơn cậu. Thế lác phẩm náo trong triển lãm gảy ấn tượng đặc biệt cho cậu?
  188. "Tiểu đội xe không kính" của Phạm Tiến Duật ạ,
  189. Hình như đớ lá một bải thơ phải không? Làu ngáy quà, tõi không nhỏ chinh xác nừa.
  190. Vâng, đó lả một bài thơ ạ.
  191. Tin bái thơ đỏ là “Tiểu đội xe khống kính" ả?
  192. Vãng, bác nhở chính xác dấy ạ. Cuối cùng Eã cậu Hùng, cháu gọi chống bá Nga bàng chú:
  193. Hỏm qua cậu đã làm yl, ử dãu, cỏ thể k| cho tõi dược chử?
  194. Vâng, tẩt nhiên là được ạ, Hôm qua cháu đi thi cả ngày, Bác có thể vào trang wẹb của trương cháu dể kierri tra lịch thi ạ. Sau khi trò chuyện với cả ba người, thám íử Sẽlôccôc nôi riêng với bà Nga: -Tội cám thảy nghi ngơ một trong ba nguội thăn cận của bá. Tất nhiên, chưa the kết luận chếc chẳn người đó lá kè đằ lẩy trộm. Bá cỏ thể đoán xem tồi <3ả nghi ai không? Bá Nga nghi mãi rổi ỉắc đầu quầy quậy. Các thám tử Tuổi Hống cỏ thề giúp bá dược không? am VỤ TRỘM TẠI BỮA TIỆC (TTT2 so 154} Thám tử đã nghi ngờ anh Tomy bởi người mãi nhổ rang thì không thể "chén tì ti" các món cay vả nòng. Nhiểu bạn cũng đoàn Tomy lá kê khã nghi nhưng với lí do lá anh ĩa vừa nói "sáng đi nhổ rãng* sau lại nói luồn “trưa nay...". Thực ra, nếu dọc thật kĩ, cấc bạn $0 Iháiy chí tiết này khõng hể màu thuẫn. Tomy nói "trưa nay tỏi vừa đi nhổ rang vế", tức lá sảng đi nhổ và trưa thỉ nhổ xong, dí vẽ. Khi "phá án", các thám tủ Tuổi Hỏng cẩn chú ý dén những chi tiết nhô dẽn mửc tưởng như không thể nhỏ hdn, các bạn nhé. Phấn thưởng kì này đuạc gửi tới: Đỗ Gia Nam, 70. THCS Vĩnh Tương, “ Vĩnh Tương, Vĩnh Phúc; Nguyễn Hương Ly vã Nguyễn Thị Minh Ngưỵệí. 6A, THCS Thị tran Cao Thượng, Tàn Yên, Bắc Gỉ ang; Nguyẻrt Đáng Duy, 7A2, THCS Yên Phong, Yèn Phong. Bẳe Ninh; Từ Tẩn Dũng, 7D, THPT chuyên Há Nội - Amsterdam, Cắu Giấy, Hả Nội; Trển Sỷ Hoàng, 8C, THCS Hoàng Xuản Hãn, Đức Thọ, Há Tĩnh. Thám tử SẽEôccâc Bài 65: Ại?§ f*ỉ $Ị( Đà Nang nóng hơn Hà Nội (Tiểp theo ki ĩruớc) Ths. NGUYỄN vù L .OAN LTS. Nểti biết liếng Hán bạn sẽ: Nam mình. 1 . Hiẻu các lử Hán Việt sử dụng lổt hon liếng 3. Hiẻu ngởn ngữ mả cử 5 người s/èn thế giới cứ Việt của minh. Trong kho từ vựng tiêng V5ệi rẳt hơn 1 người dùng. Dẻ dàng hợp tác, làm ẩn vởicâc nhĩẻutửHán Việt nưởc vả vủng lãnh thổ Trnng Quủc, Hòng Kõng,
  195. Đọcdươc sách oồ, ván bia bằng chữ Hán vá Đái Loan, singapore vã cả Nhật Bàn, Hân Quốc.

    Hán Nõm, thêm hiều vãn chirơng, lịch sử nước Nếu biềt cảtiéngAnh-vá tiéng Hán thi thật lã tuyệt Tập dọc. wỏ hé ptíngyỏu KMÌlúìn qủle hủi lãn, hàilíìn de fèngjĩng hỄn piảoliang. Wõmen zàỉ hãitãn sầnbú. kiin fengjing. Bẽijtng dedõngTiãn lèngdé bùdéle, Béijĩng dechũntían chãngchángyòu íềng. Bẽíjĩngđeqíũtiãn bĩchũntiãn hăodéduõ, qiútiânshỉ Bèijĩngziú hăodejỉjiẻ. Tỉ à í lập. Điển vảo chồ trống A§ Bít ôn E% Fi£ I.TL« f#<TÍ

    T .

  196. áÁn áat I»

    T .

  197. ,
  198. HI Ã ■ ẩ J I Ảp VĨÊL ig. yị 4 3 M ' Ầ \ ..ÍT 7 iE m m M T"T i m 1] Tị f lị 1 7: íỊr tr Ị ị Ị 1 “1”? ri I U ™ T ' 8 - GAS LAWS AND PARTICLES 0F MATTER (Tiếp theo ki trưỡc ) VŨ KÉM THỦY Questton 5 , tn one minute, a diver breathes 1 litre of air at an atmosphehc press ure of 10Ũ kPa. Tữ breathe in the same mass DÍ air ỉn one minute, how rrtuch air would he need to breathe whon the total pressure an him under water ỉs 3Ũ0 kPa? 1 ... „ 1
  199. ~r lỉtre B, -7 litre c. 1 litre 3 2
  200. 2 iitres E. 3 lilrss Qucstion 0, The diagram 3hows a model to demonstrate the behaviour of gas molecules. VVhen the piston is vibrated mo re rapidly, the vvooden disc is íorced turther Lip the tube. VVeights have to be placed on the disc to return it to iis original position. The madeỉ represents what happens to a gas when it is
  201. cooled then oompressed
  202. cooled then heated
  203. heated then comprẽSSed
  204. heated then cooỉeõ E- expanded Ihen cooled Question 7. The 8ỉr in a iarge paper bag is heated. Tho bag is then íound to rise through the SLiỉTữunding co!d air. This ts because.
  205. the air in the bag has bacome less dense
  206. the mass of the paper bag has decreaseđ
  207. heat akvays rises
  208. the mass of air in the l>ag has increased
  209. the Chemical composỉtion of the aìr in the bag has changed. Phystcs Terms diver breaiba alưiùspheric press ur& rapidíy compressed mod&! behaviour ĩorceơ bave to surrounding Chemical thợ lặn thở áp suổt khí quyển nhanh nén mẫu tính chất, dáng điệu bị đẩy phải xung quanh hóa học compữSTt/on thành phấn Answer. Tòa soạn chở bải làm của các bạn gửi vẻ. Bải làm tòl được nẻu tển trốn bào vã nhận quà tặng. Bạn nhớ ghi đấy đù địa chỉ để tiện liên lạc. Kì 21 Hăy thay Các chư cái bởi các chư số. Các chư khác nhau biểu diễn các chữ số khác nhau. Lởi giâi cán có lặp luận lôgỉc. Q R E E N 'ORANGE CO L o R s TRƯƠNG CÔNG THẢNH (Sưu tấm) KÌ 20 {ĨTT2 số 1 54) Ta đién các chữ số như sau- 68782 +68782 6 50 138214 Nhặn Xét. Kỉ này không có bạn nao giải đúng. Phán thưởng xin gác lại ki sau. NGUYỄN MINH VÂN SOLUE UEA MAIL ... tếp ítwo trang 32) 1 5(156). Is it posslble to arrange 9 squares (each contaừiing 4 small squares, tigure 1) and 7 L-shape tigures (each containing 4 small squares, íigure 2) ta caver an 8 X 8 chess board? Pigure 1 6(156). Given a trỉangle ABC { ZA > 90°) inscribed in a circie (O). Leỉ H be the orthooenter of the triangle. Let M be a pũint moving alang the minor arc BC. Let D be the intersection of BM and CH, and £ be the intersection of CM and BH. Prove that the midpotnl ữf DẼ lies on a fixed straight lỉne. THÁCH ĐẨU! THÁCH ĐÃU ĐÂY! TRẬN ĐẤU THỨ MỘT TRĂM BA MƯƠI TƯ Ngưữl thách đấu: Dương Đức Lãm, sv K59 CLC. khoa Toán - Tn. Đại học Sư phạm Hà Nội, Bảỉ toàn thách dâu: Cha các số thực dưtlng a. b c. Chúhg minh rằng a 2 b 2 c 2 (a+b+c) 2 b + c c+a a+b 2j3(ab-f-bc i-ca) Xuất xứ: Sáng tác, Thửl hạn: Trưữc ngày 08,3,2016 Iheo dấu bưu điện. TRẬN ĐẤU THỨ MỘT TRĂM BA MƯƠI (TTT2 SÕ I 52) Đệt a+ 2b + 3c = m(m > 0)=s — + 2. — + 3.-^- = 1 ' m m m Tiếp tục đặt: X - — ;y - — ;z - — => a - mx, b - my, c - mz . m m UI Tử đó ta cò: x,y,z > o.x 2 +y 2 + z 2 # 0 vá X +2y + 3z = 1. ^mx + 2my + 3mz Suy ra p - ^2mỹ+3mz +2mx +^3mzTmx +5nvy +^hix +2my +7miZ 0t + 2y + 3z ựx + 2 y + 3 z _ 1 ^2y + 3z-r2x 4^3z + x + 5y +2y +7z Vi -rX 4 ^1 +3y +Vl +4z Xét A = Vũx"^ f 3y 4 Vi + 4z =d 34 29 1 29 Y ĩ 34 Ấp dụng bất đẳng thức Bunhiacopxki ta có A 2 < 34 .34 I29 2 3 ỊặÃ 17 j 29 " 6 I §?-i [34 34
( 1 + J 0+ ■ + 2y IM „ , - , 17 , 29 X +x +2y + 3z +-7- + Jrr 12 V 34 < J V /34" ,17 'Ịi- .17 Ĩ2Õ" frr+ 0+1+-^-+,J7r =L (29 6 J| 12 ¥34 I Đâng thức xảy ra khi 2 3 £ + 2y 4 + 3 z 1 4- X 3 4

3 2 34 29 X =0 X 2y +■ 3z = i 4 3 : X - 0 125 y = z = 403 79 61 2' 1 IM 70 Vậy MlnP = 4, Chảng hạn kh a = 0, b = 44, c =44. 408 612 Nhận xè!. Đây lả bái toán bất dâng thức hay vã khá, thặl tiếc khỏng có bạn náo đãng quang trong trận đấu này. Phấn thưởng xin gác lại ki sau. LÊ Đửc THUẢN (?? TÍNH CHẤT ĐƯỜNG TRÒN EULER VÀ MỘT SÔ BÀI TOÁN ÁP DỤNG VŨ CÔNG MINH (GV. THCS Hống Báng. Hải Phòng} m Đường f tìẳng Ẽuter lã đưửng Ihlog di qua ba điểm: trọng tãm G, trực lãm H, tãm dường trôn ngoại tiếp o của tam giác. • Dường tròn Eưlẹr của lam giác là đường tròn đi qua chỉn điểm gom: trung điểm của ba cạnh, trung điểm cũa ba đõạn thảng noi trực lâm vâi dác đĩnh, chân của ba đường cao. Bái VĨỂÍ náy trinh bày một sỏ bài toán liên quan dẽn đưởng tròn dặc biệt náy. Đường trán Euler có các tính chất sau dãy. • Tinh chất 1. Cho tam giác ABC, các đương cao AA r BB r CC 1 cểt nhau tại H. Gọi A', B , c, M, N, p Eẩn lượt lá trung điểm của BC, AC, AB, HA, I1B, HC, Khi dó chín điểm A r B v c v A , B\ C\ M. N, p cùng hẳm trên một dưởng trôn (gọMà đường tròn Euíer hay dưỡng irôn chín điểm của tam giác ABC). Chứng minh. (Hỉnh 1} Hinh í Ta có MN íf AB, NA' ỉỉ CH. Từ đó MN 1 NA' hay MNA' = 9C°. Chứng minh tương tự IVÍPA' = 90°: MB'A' = 90 ử ; MƯA' = 90° Mặt khác theo tính chất đường trung tuyên thuộc cạnh huyẽn trong tam giác vuông ta có MA = MB 1 , A'B 1 = A'C nẽn tam giác MAB, cãn tại M vá tam gíảc A’B,C cãn tại A’. Do đủ AB-ịM 4 ÃB 1 C=MAB 1 +bỊcA' =90°. Suy ra: MByV = 90°. Chiihg minh tương tự M"C-jA' =90°. Như vậy bảy điểm A r B v c,. B'. c\ N p cùng nhìn MA' dưới mòt góc vuông. Vảy chín điểm A r 8, , c r A 1 , B’, c\ M. N, p củng nằm trên mệt đưdng trùn đương kính MA. • Nhặn xẽt. ‘ Tương tự trẽn NB'. PC’ là dương kính đường trôn Euỉercủa tam giác ABC. Như vậy cảc đương thẳng A'M, B'N, C‘P đống quy tại tãm đường trán Euler cũa tạm giáo ABC.

Chúhg minh trén vần đúng khi AABC tủ. Khi ,\ABC vuông Ihì một số điểm trang 9 điểm trúng nhau, Việc chửng minh dành chủ bạn đoc. • Tính Chat 2. Tâm dường tròn Euler của tam giác lá trung diểm của doạn thầng nối lãm đương tròn ngoại tiẽp và irực tâm của tam giác đỏ. Chứng minh. {Hỉntì 1 ) Gọi j vá o lần lượt lá lãm dường tròn Euler vá tâm đương tròn ngoạs tíòp tam giác ABC. Dé rièng ọhứhg minh hai tam giác HAtìl OA À'B' 1 vả OA'B' đổng dạng {g.g}. Từ dó: = Ỷ hay OA' - MH. Do đó tứ giàc MO ATI lá hĩnh binh hãnh má J lá trung diểm của MA' nõn J lá trung diểm cùa ŨH. Nhận xét. Theo tinh chất 2, lâm đường tròn Euler

của tam giác nằm trên đường thẳng Euler của tam giác đò.

Bán kính dường irủn Eulercủa lam giãc bằng một

nừa bán kinh đưởng tròn ngoại tiếp tam giâc dó (do MA' - AO). Ta sử dụng các tỉnh chất dtíờng tròn Euíer dể giầi một số bài toán. Bál toán 1. Cho tam giác ABC vuỏng tại A. Gọỉ M lá trưng diểm của BC. Lấy đf|m D nnên doạn thẳng BM, Đương thẳng qua D vuông góc vốỉ BC cắt đoạn thảng AC tại E. Gọi F tà giao điểm cưa tia BE và dương tròn ngoại tiep tam giác ADM sao cho E nằm giữa B và F. Chứng minh rằng DÃE I FÀE. Lòi giải. (Hình 2} Gọi N lả giao điểm của DE và AS. Ta có E lè irực tâm tam gsâc BCN nẽn BE 1 NC. Đương trùn 'đi qua ba điểm A, D. M chinh là đương trôn Eulercủa 22 tam gián &NC mà tì£ cắt đuàng tròn Iran tại p suy ra BF là đường cao cũa tam giác BMC. Các tứ giác BDEA, BDFN, AEFN nỘ! tiêip nèn DÃE = DBE =FNE =FÃE. Vây DÃE = FÃR B Hình 2

Nhận xét. Tưúng lự ta có DE ;á phản giác của góc ADR Vậy E lã tâm đưúng trởn nội liếp, QB và DC lẩn lượt lá lia phân giác ngoái góc D của tam giác ADR Từ đó B vá c lẩn lượt lá tâm đương tròn báng tiếp góc F và gòc A CÙ3 tam giác ADR Ta có tính chất: "Đường tròn ngoại tiếp của một tam giác đi qua trung điểm của đoạn thẳng nối tằm ha đưónc Irủn bảng tiếp của lâm giác đò". Bài toán 2. Cho tam giác ABC cò trực tâm H vá mội đuởng thẳng d náo đó qua tâm đưởng tròn Eulcr cũa tam giác sao cho A và H nấm vể một phía so với d. B và c nằm vé một phía so với d. Chừng minh tổng khoẫng tảch từ A và H lới d bằng lổng khoảng cách từ B và c lới cl. LỜI giải. (Hinh 3} A Hỉnh 3 Gọi A', M lan luợt lã Irưng dĩểm của BC, HA: gọi I. K, ! r K r l 2 ’ 2 la nh chiau ° a C-I
  1. A trẽn d. Theo tính chát dướng trung binh của hình thang ta có Bl t + CK, = 2A'I. Hl z + AK 2 = 2MK. 0(3 ỹ rằng vi d đi qua tàm đường tròn Euler cùa tam giác ABC củng là trung điểm của MA' nên MK = A'l. Từ dỏ BL + CK 1 = HL 2 + AK 2 Bài toàn 3, Cho tam gsác ABC nhọn, cỏ trực tâm H, nội tiếp đưílng tnồn (ũ). Võ đưõng kính Aũ. Gọi M là trưng điểm cỏa BC; I, J. K theo thử lự Eá hỉnh chiểu cửa D Irên BH, CH VÉ BC. Chứlìg minh bòri điểm I , IVÍ, K ; J cùng nằm trên một đưtìng tròn. Lời giải. (Hỉnh 4) Gọi p là giao điểm cùa DJ và BH, Q lả giao điểm của DI và CH. Thấy rằng Q là trực lâm của tam giác PHD. Mặl khảc do từ giảo BHCD lả hình bình hành nẽn M lá irung điểm của HD, Gọi R tá trung điểm của PG. khi đỏ theo tính chất dương tròn Euler cùa tam giác HPP, các điểm M, 1, J nằm trên đường tròn dường kính RM, Lại có ảHCD co .\PŨQ (g.g), CM và DR là hai trưng tuyến tưdng ứhg của hai tam giác đang dạng này nên APRD w iHMC. Từ đó RDP = MCH Vi tử giác KJCD nội dép nên KDP = MCH. Vây RDP = KDP hay R, K, D Ihlng hàng. Do dỏ MKR = 90° nên K nâm trên đưõng trôn đường kính MR. Từ đó ta có đíéu phải chứng minh. Bãi lặp Bãí 1. Cho đường tròn (0) ngoại tiếp tam giác ABC vá MN lã một đường kính thay dổi của đường Irdn. Gọi M v M 2 lan lượt lã hĩnh chiếu của M Irãn AB, AC: N 1 , N 2 lán lượt lã hĩnh chiểu của N lèn AB, AC. Các đường thảng IVt 1 Mi 2 và cắt nhau tại 1. Chúìig minh I nầm tnện mộl đựóng tròn cọ định, Bài £. Cho tam giác ABC có trực lâm H. Gọi I ià tám đường tròn Euler của tam giác. Đường thảng d bãl ki sao cho A. B. c. H nằm vể cùng một phía so với ư. Chủng minh rằng tổng khoáng cách từA, B„ c. H tới d bồng 4 lẩn khoáng cách tư I tới d. IMSO 2015 MATHEMATICS ESSAY PROBLEMS SOLUTION TRỊNH HOÀt DƯƠNG (GV. THCS Giảng Võ, Ba Đình. Hả Nội) (Sưu tầm Yà giòi ihiệu)
  2. If a = b = c. then there are 9 numbers. Qy Q ỉf □ = b or b = c then there ạre 2 X - - =72 9v1 numberă. ^ lf a < b < c, then there are C3 - 04 numbers. Hancs there are in toỉal 9 + 72 + 84 =■ 1 65 numbers that sati&ỉy the given conditions.
  3. The next few numbers on the other side of 4 , , 4 4, 114 1 ane 4 +3 - -7, -7 + 4 - -7 + -7 - - 3 3 3 3 3 4 3 3 1 - . 3 ,

    “■ p — +• — — 3 | 3 + — = 4 -. 1 1 4*3 4 4 4 4 The roappearance of 3followed by4 means Ihat the 24 numbers will repeat in 24 + 6 = 4 cycles ợf longtti 6, Hence the sum of all twenty-four 4 113 2 nụmberg is 4x(3 i-4 ị -T I L ị+— +— ) = 38—, 3344 3

    e . 111, 1 1 11.
  4. Since - 4 -4 - = 1 — and 3 b c 2 3 7 42 111 s _ x 1 = - 4 — + - is the smaliest denominator partition 2 3 6 1 intù three ditíerent unlt tractkms. Hence we can íet 111 1,111. 11 1 — 4 — i = Xl — +— 4 — J =. — — + — — — a b c 42 2 3 6 84 1 26 252 to gel the Êmatlest pũSăible value õf a + b + c = 84 4 1 26 4 252 - 462. 4, We numberedtheplatesas 1 . 2. 3. ..... 99. 100, and placed the fìrst candy in plate 1 . Since the greatest Cữrnmon divisor oí lữũ and 15 IS 5, Susân places a candy on all plates whose remainder is 1 wtien divitìed t>y 5, that is 1, 6, 11, 91 and 96, those lwenly plates have candy. hence there are 100 - 20 = 60 plates remain empty. 1 1 1 1 1 " e s " isas + 1988 + 1987 + ''' + 20l4 + 2015 ' then we can get 1 1 Ị 1 2015 + 2015 2015 ' ' , + 2015 31 temrr& 111 1 < s < — — + — + -■ + — — 1965 1985 1985 1983 lhat j s 131 lùrmtì 31 31 — — — < & < ——7- , 2015 1985 then we have 64— = —7-7- < - -=: _ ' = 65 . 31 31 s 31 Hencc the largest integer fe&5 than Qf equal to the expression is 64. 6 . Atter íirst round, the number ợí the Sữldiers left ùn the line is: 2, 5, 8, 11. 14, 17, 20. ... , 77, 80, th e re are 27 soldiers. Ãtìer second round, the number QÍ the sơídiers- leít on the line i&: 5 r 14, 23, 32 r 41, 50, 59, 68. 77, the re are 9 soldiers. Atìer third raund, the number of the soldỉers left on the line ia: 14. 41 , 68, there are 3 soldiers. Atter the tdurth round, the number ơt soldiers left on the line ÌÊ 41.
  5. A 3 E 3 s Let diagonal AC inlersecl DE, DF. DG at point H. M, ■Prespeclively. Suppose the area of PGC is z cm 2 .
  6. Since AEH co ŨCHso thai fihe height of AEH 15 ■'X6 =2 Ciĩk. Wể can get ihe area of Jt is 3 y 1 g -Ị?x2x2 = 3 cm . 2 1 . . , The are-a of AỠCis 24x1 6 = 192 cm , hencẹ 2 the area of occ is 64 cm^. Since ABC and A^c are congruenl, ZACB = ZA'C'B' = 45°, hence we know that OCC' is a right isosceles tnangle, the area oí OCC' is x^xCC* = 64.
  7. Sinoe RWC Cf> DMA sothatthe heightcĩ EMC 2 12 15 — XẼ = — cm, We can get the area of y + z 15 1 . 12 24 2 2 5 5
  8. Sínce GPC oo ÙPA 50 that the height of GPC 1 3 is — x6 - -- cm. We can get the area of 2 is 1x2x5=! cm 2 . 2 2 2 24 3 33 Hence we can get the area of y 15 - = , 5 2 10 and the total area of Ihe shaded region is „ 33 63 , 3+tt = 77 cm . lữ 10 8, A A' (TTT2SÕ1541 THẾ CỞ (Kĩ 77) 1.Êh6+ Ếh8 2.&f7+ ig8 3.She# r Cãc bạn sau dưực thưông ki náy: Vũ Hải Làm, 3A4, TH Vệ An. TP. 3ẩc Ninh. Bắc Ninh; Tạ Kiến Quốc, 9A. THRT chuyên Há NỘ3 - Amsterdam, cẩu Giấy. Hà Nội, Kim Mình Quang, 7A4, THCS Yèn Lạc, Yên Lạc, Vĩnh Phúc; Vũ Tri ĐUt. 6A1, ĨHCS Hỗng Báng Hóng Bàng. Hài Phòng; Nguyễn Vù Hà, 7A3, ĨHCS Lâm Thao, Lâm Thao, Phú Thọ. LÊ THANH rù We can get CC’ — 10 cm, hance BB' = 24 1 24 — 16 = 32 cm.
  9. abcd-dcba =(1ữũũa +1Ũ0b +1ŨC +d) -(lOOOđ +100C +1ữb +a) -999{a-d)+90(b c). Srnoe Ihe thousands digit is largerthan the units digit, a and rỉ cannot be 0, there are 6 ditterẹnt vakias of a d. Since the hundreds dỉgiì is targer than the tens dìgit, there are 9 different vạtues of b - c. Hence. abcd - tlcba haa 72 tMereni values. (Ki sau đăng uểpị THÊ CỜ (Kì 79) Trắng đi trước thẳng. g h LẼ THANH TU (Đại kiện tướng Quốc tể} , MnS Bại 2SN5. Tìm các sò nguyên dương n sao cho n 4 + n 3 + 1 Sá số chính phương. Lưu LÝ TƯỢNG (GV. trường THCS Vặn Ung, TP Việt Trì , Phù Thọ ) Bàỉ 29NS. Cho các so thực dương a. b, c thồa mãn abc = 1 , Tim giả trị nhỏ nhãt của bieụ thức p = L p — -I-' — . ab + bd-1-da â+b+c CAO VÂN ĐŨNG ịGV. trường THPT chuyên Hà Nội - Amsterctem, Hả Nội) Bài 30N5. Cho dường tròn (O; R) có dãy cung cố đinh BG (BC < 2R). Gọi A là điểm chuyển động trận cung lộn BC, Các siểp tuyên với dường tròn (O) tại B, tại c cầl nhau tại M. Đường thẳng đi qua M song Song VỚI AC cắt AB ỗ D, đường thẳng đí qua M song song vớí A8 cắt AC ử E. Chứng minh rằng đường thẳng DE tuôn tiếp xúc với mội đường tròn ủâ định. NGUYÊN tìửc HO-A (G V. trưóng Ẽổi ơư&ng v&n hõo Thặng Long, Q. Tạn tìình, TP, Hữ Chi Minh) CUỘC THI GtHI TOHn DHnH CHO nữ SinH (TTT2 số 1 54) Đảì 22NS. Ta cd 2ŨX 2 + 13y 2 + 3Qxy +■ 20x - 6y + 67 < 0 <-> 2Qk 2 + 1 3y 2 + 3ũxy t 20x - 6y + 68 <, 0 <=■ (6x t 5y} 2 + (2x + 10) 2 + (y - 6} 2 < 0 Suy ra 6x + 5y = 0: 2x - 1 Q = 0 vã y - 6 = 0. Vây bất phutmg trình củ nghiệm nguyên (x, y) lâ <-í 6)- Nhặn xét. Các bạn sau có lời giải đúng: Kim Thi Hóng Lĩnh, 9E1: Chu Thị Thanh. 0E1 . THCS Vĩnh Tưởng, Vĩnh Tưởng, Vĩnh Phúc: Chu Thị Hàng, 9A1, THCS Yên Phong, Yên Phong. Bắc Ninh; HỔ Gia Bào , 9A6, THCS Thốt Nốt. Q, 1 hốt Nốt, TP. cln Thơ, Bái 23N3. ĐKXĐ s < X < m Ta có 2V?it-2 s^ĩt^ãg + VĨÕ^K Ị (9 » = x s -15 se

    »(x-9) 4 30 1 \ 1 4 ■ 5 ^7*1

    Nếu 6 < X < 9 thỉ 4 30 1 -x + 6 ■>/2x -2+4 h Vex-29 + 5 ^10-x +1
  10. X + 6 1 + 3_1_3 = 0. 2 2 • Nếu 9 < X < 10 ihl 4 . 30 1 A j=== - + 1 “ — , -K + 6<0. V2X-2 + 4 Vex-29 + 5 VlQ-m-1 Vậy phương trình cỏ nghiệm duy nhất X - 9. SSIhận xét, Các bạn có lởi giải dũng bái toán trên: Chu Thị Hàng, 9A1, THCS Yên Phong, Yên Phong, Bấc Ninh; Nguyễn Thị Linh Đan. 7D, THCS Lý Nhật Quang, Đỏ Lương, Nghệ Ân: Bùi Th< Quỳnh, 8A3, THCS Lám Thao, Lâm Thao, Phú Thọ. Sái 24NS. Bạn đọc tự vê hình {Theo cã ch giải của bạn Nguyễn Ngọc Huyền, 9A, Tt ICS Hùng Vương, TX. Phũ Tho, Phũ Thọ) Gọi I lã giao điểm của DA vá CB. Vẽ dường trớn (O) ngoại tiểp ilDC có dưàng kính DK. CD Vi AR // CD vá AB = — nẻn AB lá đường trung binh của AIDC. Ta cú tứ giác HSKG là hỉnh bình hành {vi cỏ các cặp cạnh đối song song]. Suy ra 8 lá trurtg điểm của HK. Từ đủ K, B , H, E thẳng hàng. Do dỏ DẼk « 90. t ừ đó E thuộc đương Irùn (O). Tương tự F thuộc dưỡng tràn {O). Vậy c, D, E, F cúng thuộc đường tròn (O). Nhận xét. Các bạn sau giải đúng bài toán trên: Nguyên Ngọc Huyên, 9À, THCS Húng Vương, TX. Phủ Tho, Phú Thọ; Kim Thị Hổng Lĩhh, 9E1 THCS Vĩnh Tướng, Vĩnh Tưởng, Vĩnh Phúc. Các hạn sau ƠƯỌC thi/ỗng ki náy: Kim ỈẼÌ^J~L Thị Hổng Lĩnh, 9E1; Chu Thị Thanh . 8E1, THCS vinh Tưởng, Vĩnh Tường, Vĩnh Phúc; Chu Thị Hằng, 9A1. THCS Yên Phong, Yẽn Phong, Bác Ninh; Hô Gia Bảo, 9A6, THCS Thốt Nốt, Q. Ttiốt Nốt, TP. Cần Thơ; Nguyễn Thị Lỉnh Đan, 7D, THCS Lý Nhật Quang, Đủ Lương, Nghệ An Bùi Thị Quỳnh, 8A3, THCS Lãm Thao, Lãm Thao; Nguyễn Ngọc Huyên, 9A, THGS Hùng Vưcmg, TX. Phú Thọ, Phú Thọ. Ánh các bạn dược thưdng ở bìa 4. NGUYỄN NGỌC HÂN Đ ại số là mốt ngành lớn của toàn học được nghiên cửu từ rất xa xưa trũng buổi binh minh của lịch sứ toán học nhãn loại. Lúc đó Oại số coi đối tượng nghiên cítư của minh lá các phép tính (cộng. trử. nhàn, chia, khai căn lũy thừa) và mối quan hệ giữa chúng được biểu ắht; qua các phương trình. Cuốn Cửu chương toán thuật cùa nhá toán học Trưng Quốc Trán Sanh (nám 150 trước Cõng nguyên) có chường VIIE mang tên là Phương Irận (Giải má trận) giàí nhũhg hệ 5 phương trình tuyến tính nãm ẩn: a 11 x 1 +a 12 ỉí 2 + - +a ie X S = b 1 a 21 X 1 + a 22*2 + “■ + a 25 x s = a 51 x t + a 52 x 2 + ■" + a 55 X G “ h 5 Trước dó Sách Chu bễ toàn kinh (khoảng 350 nân* trước c&ng nguyên) cho chúng la thây trinh dò toán học của người bấy giờ vỏỉ vài phừđng pháp 5 Ơ khai cũa đại số. Trong bộ sách Nguyên ti CÙB Eudỉơ khoảng 3ỮŨ nSm trước Cõng nguyên dã giãi mội số phương trinh bãc nhất, bậc hai. Có thể nỏi Đại Êổ bất đlu cách dây chừng 2500 nãm. Dại sổ cổ điển !ập trung chú yếu nghiên cứu phươnxg trình vá bất phuơng trình bậc thấp (bậc nhất, bậc hai), một sâ phương trinh bậc cao đạc biệt (như phương trình bậc bân trùng phương tức lả cố thể quy vế bậc hai), các hàm dơn giản, các khái niệm vẽ số và giới hạn, các vấn đế vé tích to hợp. Đẩu thế kĩ XV! , những chữ cái đã xuất hiện thay cho các số, gọi là số dai diện. Bởi thế món học múi củ tèn là Đại số (số đại điện). Thuãl ngữ Algsbra do nhà toán học Ba TưAhKhwanzrn.i năm 820 đưa ra, với ý nghĩa: al'j 3 br lá hợp nhất, liên kết. Francois Vièle, nhà toán học Pháp (1540 - 1603) ĐẠI SỐ BÍNH Nì AM HÀ đã thống Iihât phương phảp truyển thõng trong hình hoe VỚI phương pháp mới của đại số. ồng dùng các chừ cái là nguyện âm biểu thị các ẩn 50, con nhGhg dại lượng dã biết nhưng không xác định dược thi biểu thị bằng các phụ ám, õng cũng giải quyỂt triệt để vấn đề của cân bậc hai, một sổ vấn để cân bậc ba vả đưa ra định nghĩa cân bậc hai dựa trẽn mối quan hệ giữa các hộ số vá khai cân. Ông thương dược coi lã õng tổ của dại sổ sơ cấp do dưa ra cách giãi thống nhất cảc phương trình bặt 2, 3 vả 4. mối quan hệ giữa các nghiêm mà ngày nay la gọi tà định li Viète. Cách dụng các chữ cái cuối bâng Alphabet lá X, y, 7 , để chỉ các ẩn sỡ lá bắt đầu tử René Descartes. Đại sò hiện đại được bát đlu vảo thế kĩ 20, nghĩa lả khi õại 50 đă có bế dày lịch sử hdn 20 thế kì, Công đẩu thuộc vé chàng thanh niên Pháp Évarisle Galois (1811 - 1832) với việc nghiên cứu nghiêm cùa phương trình đại sứ. ỏng dã tùng giao cảc công trình quan trọng của mình cho nhá toán học Cauchy, nhưng Cauchy đánh mất Ván bằn quan trọng khác õng đệ tanh cho Joseph Pouriar, thư ki của Viện hám lãm khoa học Phãp. rnhimg Fõurier mẩt thời gsan ngẳn sau dỡ. Sau náy Poisson còn nhận vãn bản thít 3 của Galois nhung tũ Ghdi xót giải thưởng với lí do nộp muộn. (Còn tiếp) NOẼL VÀ TET (Tiếp theo kì trưởc) BÌNH NAM HẢ Giữa tháng Chạp, dòng sõng hoa đào ngập ngừng chảy vào thành phổ. Đú lá những cây đào nở sâm theo gió Đòng vế cùng với cánh én. Ấy là mùa xưãn đang đến, Mỗi chiếc xe đạp chở mẩy cành và sau này dỏng sông hoa ấy làm nên tờ dóng xe máy trôi lử ngoài vào thánh nội. Ở Hà Nộí là từ các láng Ngoe Hà, Nhật Tán , sau này là Quảng Bá, Téy Tựu và từ cả Hưng Yên, VFnh Phúc về. ở Nam Định là từ làng Vy Khẽ, Mỹ Tãn vã cả tử cảc tinh miéri núi mạn ngược xuôi theo QL 21. Chợ hoa Hàng Lược à Há Nội vá chợ hoa giữa 2 chợ Rồng ờ Nam Định hình thánh từ 23 tháng Chạp, sõi động, rực fữ suốt một tuán. Sau nảy thi chợ hoa Nam Định chuyển ra chỗ Quảng trưởng Hòa Bình còn chợ hoa hàng Lược nhướng chẽ dần cho chợ hoa Âu Cơ, Lạc Long Quân, Quảng Bả và chợ noa mở ra khẳp các cửa ô, các quận. Nhã nhà rửa lá dong, vo gạo, đãi đo để gói bánh chưng. Các cửa hàng bán quẩn áo cũng dông đức, tổp nập đến tận khuya. Ớ vùng nông thôn, các chùa dược trang hoàng đẹp vả những cáy nêu. cây đu bằng tre được dựng lẻn. Nhiéu gia đình ở thành phố Nam Định còn rửa nhà chiểu 30 Tết. Tãt cả đã sẳn sàng đón Tết. Chiều 30 Tết cũng lá khi các gia dinh té tựu làm mâm cơm cúng tất nỉên. nguửi lớn chuẩn bị quá mừng tuổi cho trẻ con bằng những đóng tiền mới. Chiểu 30 Tết cũng lả thời điểm cuối cùng mua hoa lay-ơn, thứ hoa đẹp nhưng khố để được lâu phải mua cuối cùng cho hoa chơi được lâu trong Tết. Vậy lả đã có đủo, quất, lay-ơn hoặc thược dược, mâm ngũ quả và mâm cơm cứng chiếu 30 Tết. Những nhà sành chđi hoa còn Cũ Ihẽm hoa tưlip, hoa thủy tiên, hoa lan trong nhá. Hoa, quả là lộc của trài đất và con người muốn đến gần với thién nhiên hơn trong dịp Xuãn vế. Giao thửa, đỏ là thòi khầc thiêng liêng, Tiếng pháo đốt xưa và pháo hoa nay đễu là những hĩnh thúc đánh dấu phiit chuyển minh của thời g*an, cùa trời đất gtao hòa. ỏng bà bổ mẹ mừng lưổi con cháu. Con cháu mừng tuổi bã mẹ, ông bả và những lời chúc tốt đẹp vé sức khỏe, niềm vui và hạnh phũc củng sự thịnh vượng. Nhiéu người cao tuổi, người chủ gia đinh chọn thời diểm này để khai bút đáu xuân. Gia đĩnh náo có hoa thủy ĩiên nở vào thời khầc đó Ihì ỉá điểm may mắn. Chầng gi, hoa thủy tiên nở thường thơm dịu nhẹ giữa đém. Đó thực lá sự thưởng hoa cao sang nhất trong đêm bàn giao giữa hai con giáp. Sáng mổng Một Tết, thời gian như ngưng lại. Khác với tuần giáp Tết pho xá chật ních ngưủi thì sảng đáu nám mới phố xá sạch bong quang quẻ. Những cây xanh ngd ngác hỏi nhau người đi đâu mà vắng vậy? Nhiéu người đã về què ởón Tết cùng họ mạc bố mẹ anh em. Nhiẽu người sau đêm xem pháo hoa vả chương trinh giao thừa giờ còn ngủ bù. Nhiều người ngại trỏ thành người xóng nhà nên cũng chưa đi đầư. Chỉ cố cãc chừa lá đồng người tới thắp hương đẩLi năm cấu may và an lánh. Ai cũng mãc đẹp. Trẻ con xúng xính quẩn ảo mới . Món quà thích nhát của các bạn nhỏ là tiến mừng tuổ! và bóng bay được ba me mua cho khi ra phố. Những ai cầu ki thi còn mời ngưởi họp năm hợp tuổi chủ nhà xông nhá, xỏng đất cho yén tàm. Cảu cháo gạp nhau sáng mèng Một là: Chúc mửhg nám mới, hoặc: Năm súm, chúc ổng bã mạnh kho-e. Gẩn trưa, cảc gia đình tụ tập chúc nhau rôm rả và ân bCra cơm gia đình dẩu năm. Mồng Haỉ, mống Ba vẫn còn chính Tết. Đường phố vần sạch và đởng dẩn người lại qua, Ở Nam Đạnhụ thành phố không quá lớn nên nhiểư người chọn cách đĩ bộ chức Tết, cứ sau mỗi nhà. đoàn ngươi đi lạt dài thêm. Vài cửa hàng bún, phâ mỏ hãng chọn ngày đã bản ngay từ mống Hai Tết. Cử thế mống Bốn, mồng Nam đường phố gần đỏng bằng ngày thưởng. Gác chuyến du xuân củng bẩt đẩu, xưa, Tết thường có mưa xuân. Nay hình như bả Chứa thời tiết đă thay đổi. Mùa xuân ẩm và khỏ hơn. Sang ngày mong Bảy các con đưửng đổ vể Nam Định từ Hà Nội, Phủ Lý r Hải Phòng, Thái Bình, Thanh Hóa, Ninh Bình vã từ Quất Lâm, Hải Thinh tiều chệi ních ngưới xe. Dêm mổng Bảy bốn chợ Viếng đẽu khai mỏ. Nổi tiếng hơn cà lả hai chợ Viếng Phủ (Vụ Bản) và Viềng Chùa (Nam Trực), Cây cảnh, dụng cụ nhà nông, đố cổ vá thỊt bò chạy dãi theo quốc lộ 10, quốc lộ 38B, tỉnh lộ 390, quốc tộ 37 B khầp vùng rộng cả trâm km 2 . Sang ngày mồng Tám Tết thì tràn ngập thỊt bờ (bẽ) được bán dọc các con đường náy. Sau ngáy Tết ăn thịt gá, thịt lợn, gió, chả, nem đa ngấy thì thịt bò trở thành món thay thế thích hợp nhất. Tết được kểt thúc bằng lề củng hóa vàng tùy từng gia đinh lác đác từ mổng Ba Tết theo hoàn cảnh mòi gỉa đinh, Phiên chợ Viềng vừa vui xuân vừa bẳt đẩu mùa vụ làm àn sau Tết Tết thực sự chấm dứt bởi phiên Khai ấn dền Trần, Người đi Hộp xin ấn để cầu sức khỏe, cắu sự may mắn, an vui cho gia đinh vá bán ỉhỗn. Từ khi quổc lộ 10, quốc lộ 21 , quốc lộ 38B và đường cao tốc Phủ Lỷ - Nam Định được mở rộng, làm mới thi lề Khai ấn cáng đông, mỗi nam hàng tràm ngán ngutìi về dự. có người muũn ghép Tết Tây và Tết Ta vào dịp đẫu nam Dương lịch. Điều này khó xây ra vl khi hậu Tết Dưỡng lịch thường quá rét ỡ miến Bầc và cấy vụ Chiêm còn chưa xong, Bởi vảy vẵn cỏ Tết Tãy, Tết Ta như ngàn đời đâ có. TJU> E:ni TftẰH 'XCIUI ■ J KHh TỔNG TẬP nẮA ỉuểinta HẮH Ỉ01Í T3ÉU 5ẹc Dõng tặp 12 số tạp chỉ cả nãm 2015. Đóng bia củng. Tiện tra cííu cho tliẩy cỡ. Bỏ di£ỉnọ học sinh giỏi. Lưu trữ trong thư viện. Qiíã tặng học sinh giỏi. Giá bìa: 1650ŨQ dổrtg. TAT CHÍ r-íllAH 7(lổ] tho TỔNG TẬP TOÁN Tllổl THđ NĂM 2015 THI. Xí; học cư sò Kì 10 TẠ THẬP (TR Hõ Chị Minh} BàM. Chửng tỏ rang só 243 ,3J2 cỏ ít hơn 856 chữ sổ. Bái 2. Cho a, b, c khác ữ thoa mãn 2ab = c 2 , ac = 4b 2 . Tinh giả trị , . _J . . 5a + 4h h3c cua biếu thức — — 3a 4 2b + c Bái 3. Tìm giá trị nhỏ nhất của hiếu Ihức M - 9|x - 4| 4 |x - 1 : 4 X. Bài 4. Tim số abcdeg , biết rằng 201 5 a + bcde.O - 1968g. Bái 5. Cho tam giàc ABC cản tại A có A = 100°. Điểm M nam trong tam giâc ABC sao cho MBC = 30°, IŨCB =20°. Tinh sổ đo AMC. Góc OLYMPIC Kì 9 (TTT2 số 154) Bái 1, Gọi số chũr số cùa 2 n vá 5 n lan lượt lá a vá b. Ta cà 10 a - 1 < 2 n < 10 a ; 10 b - 1 < 2 n < 1Q b vả a 4 b = 2015. Do đó I0 a_1 .10 b 1 < 2 n .5 n < 1Q a .10 b = 10 3 + b “ 2 < 10" < 10 a - b r 10 2013 < 10" < 10 2015 => 201 3 < n < 2015. Vì n E N* nên n = 2014. Bài 2. Ta CÓ (a 5 - Sab 2 ) 2 4 (b 3 - 3a 2 b) 2 = 16 2 + 26 2 => a® - 6a 4 b 2 + 9a 2 b 4 4 b 6 - 6a 2 b 4 4 9a V = 1 000 => a fi 4 3a 4 b 2 4 3aV 4 b s = 1000 => (a 2 4 b 2 ) 3 = 10 3 Suy ra a 2 + b 2 =• 10. Bải 3. Tim càc sổ tự nhiên rĩ để (n 2 - 8) 2 4 36 là SỌ nguyên to. Ta có (n 2 - 8) a 4 36 = n 4 - I6n 2 4 100 = n 4 4 20 n 2 4 1 00 - 36n 2 = (n 2 + 1 0} 2 - (6n) 2 = (n 2 4 10 - Sn)(n 2 4 10 + 6n) là số nguyên tố má n 2 4 10 4 6n > 1 ơo đó n 2 4 10 - 6n = 1 và n 2 4 1 0 4 6(1 là số nguyên tố. Suy ra (n — 3) 2 = 0, tứ đó n = 3 (Thử lại thấy n 2 4 10 4 6n = 37 lã số nguyên tố). Vệy n = 3. Bải 4. Ta DÓ 201 6 : 3 vá 2014 không chia hết cho 3 nên 2Q14 2015 4 201 6 2ữ17 không chia hết cho 3. (1} Mát khốc X 3 - 2x = x(x - 1 )(x 4 1 ) 4 3x : 3. { 2 ) Từ(1) và ( 2 ) suy ra không tôn tại sổ nguyên X sao cho 2014 2ữ15 4 2016 2íí17 chia hel cho X 3 4 2x. Bài 5. K Trên lia AC lẩy điểm D sao cho AD BK. Vì -iMAB vuông tại H có ABH = 45^ nên AHAB vưõrg cãn tại H, từ đó AH = SH, Xét iAHD và iBHK cò AH = BH, HAD -HBK (cùng phụ VỚI góc AC8), AD = BK. Do đó iAHD = iBHK (c.g.c). Suy ra HD = KH. lử dó AHKD cần (ặi H. Do đó HKD =HDA, Ta tại oó HDA =HKB (vi 4AHD - iBHK). Suy ra B"Ì<H =HKD Vậy BKH-~ = 45 Ủ . r c- v - . Nhận xét Các bạn được Ihuâng ki này: Ịlliị.... i-é Ngọc Hoa, SẼ1 , THCS Vhh Tường, Vĩnti Tường, Vinh Phúc; Tạ Lê Ngoe Sáng, 9A, THPT chuyên Hà Nội - Amsterdam, Cấu Giãy Há Nội; Hố Gia Bào. 9A6, THCS Thốt Nốt Thá Nốt. Cần Thơ. Trển Như Quỳnh, 7A, THCS Hoàng Xuân Hãn. Đúc Thọ. Hà Tĩnh; Lẽ Nguyễn Phỉ Lỗ, 7C. THCS Nhừ Bá Sỹ, Hoấng Hóa. Thạnh Hóa, NGUYỄN HIỆP 30 Hỏí: Anh Phó tìã! Êm có đưạc viết bài cộng tác VỞÈ chuyên mục "Toán quanh ta" không ạ? Em còn có thể cộng tác vớt những chuyên mục nào nữa? Tử TẨN DŨNG (7D, THPT c huyên Hà Nội - Amslerờam, Cầu Giấy, Hả Nội) Đáp: Toán quanh ía Toán quanh em Mục nỏo củng được miễn em viết bàì Hay vả không chép của ai Hợp VỞ! bào vá dừng dãi lẽ thẻ Bọc íên thấy mội ở kê! Riêng mục Giải toàn qua thư Dề dành cho các thầy cô gửi về. Hỏi: Nếu em hỏt 2 câu cùng trong chuyên mục "Rubic hỏi đáp" thì Cũ đưtìc chấp nhận khòng ạ? NGUYỀN THÚY THANH Ợ AI, ĨHCS Lãm Thao, Lâm Thao, Phú Thọ) Đáp: Một câu chứ một nghìn câu Anh đây không ngại ngồi lâu trả J lời Trẳ xong Gâu mật được rỗi Anh ngỗi trả tiếp hai rối đến ba Nghìn cãu chắc đến tuổi già. Hổí: Anh Phố dií Khoảng bao lâu sau khi báo ra lã hết hạn để chúng em gủì bái ạ? Một bạn quên ghi tên Đáp: Báo ra ỏ thô đỗ Ngùy 8 của hàng tháng Cứ cho lá chậm tẩm Cuối tháng vê trường em Tuấn đắu của tháng sau Em giải bài gửỉ bào Cứ cho ià chậm iếm Cuổỉ tháng nữa tới Tòa.., Soạn ra chấm là vừa. CÁC LỚP 6 & 7 Bãi 1(1 56). Cho 2016 số nguyên đương a.j. ct^i 3^ a i!0iẾ mãn J J_ f J_
  11. ..,+ a 1 a 2 a 3 1 a 2Ù16 = 300. Chúng minh rằng trong 2016 sỏ đã cho ton tại ít nhát hai sỗ bằng nhau. NGUYỄN NGỌC HÙNG (QV. ĨHCS Hoàng Xuận Hãn, ũừo Thọ, Hà Tĩnh) Bãi 2(156). Cho tam giác ABC vãi à = 40°, c = 30°. Trên cạnh AC lảy ơiểm Đ sao cho CD = AB. Tĩnh số đo ÀBD. NGUYỀN KHẢNH NGUYỄN (GV. THCS Hóng Bóng. Hóng Báng, Hải Phòng) CẢC LỚP THCS Bái 3(1 56). Cho cảc sổ nguyên dương a, b thủa mản ab 4 1 lá sổ chính phương. Chửng minh rằng tồn tại số nguyên dương c sao cho ac 4 1 và bc -i- 1 dểu lã sđ chính phương. VŨ ĐINH HÒA (GV. Oại học Sư phạm Hà Nội) 8ải 4(156). Cho cáo số dương a, b ; c thủa mân abc = 1 . Chúhg minh rằng: b a ú 1 J . 1 1 1 ì £2â4b4C4-4 — - -- . [ a c bj ! ! a b C J KIỀU ĐÌNH MINH ịGV. THPT chuyên Húng vương, Phú Thọ) Sàí 5(156). CÒ thể xếp 9 hỉnh vuâng gom 4 ồ vuông nhỏ (hình 1 ) và 7 hình thước thợ gốm 4 â vuông nhỏ (hình 2) để phủ kín bàn cờ 6 X 3 ò vuông nhỏ hay không? Hình 1 Hỉnh 2 VŨ KIM THỦY Sài 6(15$). Cho tam giác ABC ( >90 ù ) nội tiếp đường tròn (Q). H là trực tâm cũa tam giác ABC. M là điểm chuyển động trên Cung nhỏ BC. Gọ? D là giao điểm của BM vả CH. E là giao điểm cũa CM vả BH. Chúng minh rằng trung điểm của ơoạn thẳng DE nằm trận một đường Ihãng cõ định. NGUYỄN ĐỰC TẨN (TR Hộ Chị Minh) SOLVE UIA MAIL COMPETITION QUESTIONS Transtateổ by Nam Vù Thành 1(156). Given 2016 posilive integers a. a 2' a 3- ,111 1 e™ R such that —4 — + — + ...4 — ■ — ®1 a 2 a 3 a) te = 300. Prove that at least two numbers am ong the gìven numhers are equai- 2(1 56).. Given 3 triangEe ABC havsng ZA = 40 & and zc = 3Q tì . Leĩ D be a point on AC such that CD = AB. Find ỉhe measure of ZABD. 3(156). Given the positive integera 3 and b such Ihat at? 4 1 15 a pertecl square. Prove that thene exists n posítive integer c suọh Ihat ăũ 4 1 and hc 4 1 are both pebect squares. 4(156).. Giveiì the pủsitive numbers ở, b, and c such that abc - 1 . Prove that PHIÊU ĐĂNG KÍ THAM Dự CUỘC THI GTQT NĂM HỌC 2015-2016 3 , b , e , 7-4 — 4 — 43 b c a 3 V 1 1 -4 — 1- — >2 a + ti + c + - + -+— (Xem lép tràng 20) Bạn hãy vào websifc: htt|ỉ:/ l ' l olm.vn l ,, híeii-sifh-imliỉie [lể dọc ựip chí Toán ĩtidl tho bán diện tú nhc. Trước mẳt bạn lầ bầu trời tưoi đẹp vầ hoa ngút ngàn. Bức tranh trở nẻn hoàn hảo hon vtìi các cô gái đang biểu diễn trên các phím đàn. Vậy bạn còn nghe thấy những âm thanh rộn rã của mùa xuân đang vé nữa. Tòa soạn chờ bài viết cùa bạn v| câm nhận thị giác và tưởng tuợng thính giác nữa. Bài viết hay sẽ dược dăng TTT và tác giả dược nhận thưởng. MGRIS VŨ /\ĩiỉ\: Phan Ngụ: Quầng CÁC HỌC 5INH ĐƯỢC KHEN TRONG cuộc THI GIÀI TOÁN DÀNH CHO NỮ SINH ị Từ trâisanq phaf: him Th[ hông Linh, Chu Thị Thanh. Chu Thị háng, Nguyên Thị Linh Đan, {^rcr'ÃIỈtíl£fíG HtẨK HỔNG HA SMCỄ 1Ợ£# rCtriy Ỉỉttựỉĩt ỈÌỉiGrỹ 'Wứ ỉrtOtỶụ ỉiìi Công ty CP VFF nồng Hà là nhà tài trọ cho 2 cuộc thi: và Giấy phép xuất bán: số 31/GP-BVHTT, cấp ngây 23/1/2003 của Bộ Van hóa vả Thông tin. Mã số: SBTT156M16. fn tại: cỏng ty cổ phẩn in Còng Đoàn Việt Nam. 167 Tảy Son, Đống Đa, Hả Nội. In xong vã nộp lưu ohiểu tháng Ữ2 nãm 2016, TRUNG HỌC CD sở m NĂM THỨ MƯỜI BẢY ISSN 1859-2740 Giá; lOOOOđ NHÀ XUẤT BẢN GIÁO DỤC VIỆT NAM - BỘ GIÁO DỤC VÀ ĐÀO TẠO ĩ §/3/20 ĩ & MƯỜI SA NĂM HA sq ĐẨU TIÊN Y V 1 Childrens Fun Maths Journaỉ NHÀ XUẤT SÀN GIÁO DỤC VIỆT NAM - BỘ GIÁO DỤC VÀ DÀO TẠO "1,1,7, L M ỊT-1 ■aM Tổng biên tạp: ThS- VŨ KIM THỦY ỦY VIỂN NGNO.VLPHỮU BÌNH TS. GIANG KHÁC BÌNH TS. TRẦN ĐÌNH CHÂU TS. VŨ ĐỈNH CHUẨN TS- NGUYỄN MINH ĐỨC ThS. NGUYÊN ANH DŨNG TS. NGUYỄN MINH HÀ PGS. TS. LỀ QUỐC HÁN PGS. TSKH. VŨ ĐĨNH HÒA TS. NGUYÊN ĐỬC HOÀNG ThS. NGUYỄN vú LOAN NGUYỀN Dực TẨN PGS.TS. TÒN THÂN TRƯƠNG CÔNG THÀNH PHẠM VÃN TRỌNG ThS. HỐ QUANG VINH TÒA SOẠN Tầng 5, sổ 361 đưtìhg Trưdng Chinh, quận Thanh Xuân, Hà NỘI Điện thoại (T«IJ- 04.36032701 Điện sao (Fast). 04 35662702 Điện thư {Email): lữaniuừithd@vnn.vn Trang mạng (VVebsÊte}: h://www.lữanrtwuitho.v^ DẠI DIÊN TẠI MIỂN NAM NGUYÊN VIẾT XUÂN 55/12 Trpn Đinh Xu, p. cẩu Kho, Q.1 r TP. HCM ĐT; 08.66821 199. DD' 0973 308199 Eỉièn tập: NGUYỄN NGỌC HÂN, PHAN HƯƠNG Trị sự - Phát hành' TRINH THỊ TƯYÉT TRANG. VŨ ANH THƯ, NGUYỄN HUYỂN THANH Chế bàn: Đỗ TRUNG KIÊN Mĩ thuật: TÚ ÂN CHỊU TRÁCH NHIỆM XUẨT BẢN Chế tịch HỘI dong Thánh vlẽn NXBGD việt Nam; MẠC VÃN THIỆN ĩổitg Giám đôc NXBCŨ việt Nam; GS.1S.VỦ VAN HÙNG PHA Tổng Giam ffõ’c mém Tồng biên Ịập NXBGD Việl Nam: Tỉ. PHAN XUÂN THÀNH TRONG SỐ NÀY Dành cho học sinh lóp 6 & 7 Phương pháp tính tổng dựa vào độ lệch Sũ vởi các tổng quen thuộc Lê Thị Ngọc Thúy Học ra sao? Giải toán thế nào? Tìm giá trị lớn nhất, giá trị nhò nhất của phản thức đại sổ Hoàng Vãn Long CỬasổAC CácnướcĐôngNamÁ Bính Nom Hà Nhìn rathếgiới Qua hai bài toán nhỏ trong cuộc thi Toán Australia AMC năm 201 5 Tạ Ngọc Trí Com pa vui tính Không vẽ đường tròn Phạm Tuấn Khắi Phả án cùng thám tửsêlóccôc Mất trộm vì ngủ quên Nguyễn Ngọc Sơn Đến với tiếng Hán Bài 6Ệ. Ôn tập Nguyễn Vũ Loan Học Toán hằng tiếng Anh Geometry (Hình học) Vũ Đố Quan Để thi các nước IMSO 2015 - Mathematics Essay problems solution (Tiếp theo kì trước) Trình Hoài Dương PHƯƠNG PHÁP TÍNH TổNG DựA VÀO ĐỘ LỆCH so VỚI CÁC TỔNG QUẼN THUỘC ThS. LÊ THỊ NGỌC THÚY (GV. Cao đẳng Sư phạm Nghệ An) Cỗ nhiêu phương pháp tính nhanh các tổng dạng đặc biệt, Ví dụ điển hình nhất là nhà toán học thiên tài người Đức là F. Gaoxơ khi mới bảy tuổi đã biết tính tổng S = 1 + 2+ 3 + . .. + 99 + 100 bằng cách viết lại s = 100 + 99 + ... + 3 + 2 + 1. Từ đỏ 2S - 101 4 101 4 101 4 ... 4 101 {cố 100 sổ hạng bằng 101). Suy ra s = 101.100 : 2 = 5050. Sau đây chúng tôi sẽ trình bày một phương pháp tìm công thửc tính giả trị biểu thức dạng tổng hoặc hiệu các sổ. Bài toán 1. Tìm cóng thức tính S n = 1 + 2a + 3a 5 4 ... + {n + 1)a M . Lòi giải. Với a = 1 , có S n = 1 +2 4 ... + (n h 1). Dựa vào cách tính tổng như trên của F, Gao xơ, .4 » (n + 1)(n 4 2) nhận được s n - 1 — — -. 2 • Vãi a 5È 1, ta nghĩ đến tống quen thuộc P i: = 1 4 a 4 a 2 4 ... + a n với cách tinh như sau: Ta có aP = a 4 a 2 4 ... 4 a n 4 a n “ 1 . Đo dộ lệch aP n với P n là aP n -P n = {a-1)P n = a n + 1 -1. J,n-1 - 1 Do đó p n =- — . a-1 Như vậy để tính S n , ta đo độ lệch của nó với aS n , nhận được s n - aS n = 1 + a 4 a 2 + ... + a n - (n + 1}a ní 1
  12. p n - (n 4 1)a n+1 - — 7 1 - (n 4 1)a n+1 . a-1 Từ đó (n + 1)a n ~' a n+1 - 1 (n 4 1)a - (n 4 2)a n "' 4 1 a-1 {a-1) 2 " (a-1) 2 Chú ý. Để tính tổng P n = 1 - a 4 a 2 - a 3 + ... 4 (-1) n a n r ta không đo độ lệch của nó vứi aP n mà với -aP , nghĩa là xét biểu thức P n - (-aP ) = P n 4 aP n và nhận được (1 + a)P n = 1 4 (-1 ) n a n 1 1 . Bải toán 2. Tính
  13. p = 1 4 3 4 3 2 4 ... 4 3 10ữ .
  14. s = 1 +2.3 + 3.3 2 + ... 4 101.3 100 . Lài giải. Sử dựng kết quả bài toán 1, ta có . . 101 3 102 -102.3™ 1 +1 201 3 1ữ1 + 1
  15. 5 = = . 4 4 Bài toán 3. Tinh tổng S n = 1.1! 1 2.2! I 3.3! 4 ... 4 n.nT, trong đó n! = 1.2.3...n là tích của n số nguyên dương đầu tiên. Lởi giải. Để tính được tổng đá cho ta cẩn sử dụng tổng P n - 1!-r2! + 3! 4 ... + nf. Mặc dù tổng náy chưa tính được, nhưng nó giúp ta tìm được tổng cần tính. Ta đo độ lệch của S n không phải vởi P n mà vửi -p , nghĩa là xét biểu thức S r . - (-P ft ) = S n 4 P n . Nhận xét rằng k.k! + k! = k!(k4l) = (k 4 1)1. Ta có s n + p= 2! 4 3! + ... + n! í (n 4 1)!
  16. P n + (n + 1)! - 1 . Suy ra S n = (n 4 1)! - 1. Bải toán 4. Tim công thức tính e 1 1 1 1 n 1.2.3 + 2.3.4 3.4.5 + ■ + n(n 4 1)(n t 2) ' Lài giải. Để tìm được công thức tính này, ta cần phải dựa vào tổng quen thuộc sau B i , 1 , 1 1

    1-2 2 3 3.4 n(n4l) Ta biẽt rãng k(k + 1) k k4l Suy ra

    ,1.11.11. 1 1 , 1 2 2 3 3 4 n n I 1 n + 1 Bây gid ta sẽ đo đạ lệch cùa P n vôi S n 1 1 k(k + 1) k(k 4 1)(k 4 2) 1 k(k 4 2) 1{1 2 v k 1 > k 42 , (Xem tiếp trang 6) TÌM GIÁ TRỊ LỚN NHẤT, GIÁ TRỊ NHỎ NHẤT CỦA PHÂN THỨC ĐẠI sô HOÀNG VÂN LONG {GV. THCS Lê Quý Đôn, TP. Hai Dương, Hắi Dương) \ ãi các bạn học sinh lâp s việc tìm giá trị lãn nhất, giá trị nhỏ nhất của biểu thức đại số là bài toán không đơn giản. Qua bài viết này chủng tôi muốn đưa ra một phương pháp tìm giả trị lớn nhất, giá trị nhỏ nhất của phân thức đại số cò dạng ax 3 -bx 2 4CX -KỈ M = ax 2 4gx + h Bài toán 1. Tìm giả trị lớn nhất của phân thức A 3x + 1 A - ——5 7 - 2x -x + 3 Lời giải. Ta có 3x + 1 A = - 2x -X43 2x 2 -X 43-2x 2 + 4x -2 2x 2 - x + 3 (2x 2 -x + 3)-2(x 2 -2x + 1) 2x 2 -X + 3 si- 2(x-1? 2 2x 2 -X + 3 = 1 2(x-1) a J 7 1,1 2 X rX4 Vì 2(x-1) 2 2ÍX-1Ì + ạ 4 1 8 V / (x-1) 2 >0 Vx V < 1 . 16 23 2(x - 1 f 1 ' X — - ‘ => Ỷ 0 Vx J 1 f V 4j l V 0 Vx. 23 8 Dâu “=" xảy ra khi X = 1 . Vậy MâxA = 1 khi X - 1 Phân tích. Vân đề đãt ra là làm thế nào để tim ra cách giải như trên. Ta để ý mẫu thức của phân thức trèn 2x' = 2 X -X + 3 = 2| X \ý 23 4 1.1 X + -

    2 16 23 T r J 4 — > 0 Vx. 8 Do đó ta nghĩ đến cách tách 3x + 1 A =■ a(2x 2 - X 4 3) 4 b(x - c} 2 2x 2 -x + 3 2x -X + 3

    a b(x-c) 2

    2x 2 - x + 3 Ta cần tìm các hệ Ẽổ a, b, c thỏa mãn 3x 4 1 - a(2x 2 - X 4 3) 4 b{x - c) 2 Vx <-> 3x 4 1 = (2a 4 b)x 2 - (a 4 2bc)x 4 (3a 4 bc 2 ) Vx Đổng nhất hệ số hai đa thức trên ta được 2a + b=0

    “(a 4 2bc) = 3 <14 3a 4 bc 2 = 1 2a + b = 0 C4> ị b(1 - 4c) = 6 n 2a + b = 0 2a - b - 2(a 4 2bc) =6 3(2a4b)-2(3a4bc 2 ) = -2 b(3-2c 2 ) -2 Suy ra 1-4c 3-2c 2 <=> (c - 1){6c 4 10) - 0 <=> = -3=>1-4c = 6c 2 c = 1 5 c = , 3 • Vởi c = 1 thay vào () ta được b = -2: a = 1 5 • Với c = thay vào n ta được b = 44; a =-7, 23 23 Hai bộ số (a; b; c) cho ta hai cách tách tử thức để giải quyết hai bài toán tìm giả trị lớn nhất và giá trị nhỏ nhất của phân thức A, Bài toán 2. Tìm giá trị lớn nhất, giá trị nhỏ nhẩt „.UÌ„ iL . r, 4x 2 -4xỷ12

    của phán thưc B = í 2 4 2x + 2 Phân tích. Ta có B = 4x 2 - 4X412 = 4 I -12X4 4 © X 2 I 2x ị 2 X 2 ị 2x I 2 -12x4 4 Ta sẽ tìm giá trĩ lân nhất của c = . X 2 +2x + 2 Vi X = Ũ thì c = 2 nên giá trị lớn nhất của c là số 1 dương. Giả sử giá trị lân nhất của c là 7 - với k > 0. k Ta sẽ tìm k để

    -12X + 4 J u . -12x í 4 1 rt .

    c=— — — — < - Vx — -<0 Vx ( 2 +2x I 2 k X 2 + 2x í 2 k « k(-1 2x 4 4} - (x 2 4 2x 4 2)< 0 Vx <=> X 2 + 2x{6k * t) + 2-4k >0 Vx [x 2 - 2x(6k 4 1) 4 (6k 4 1) 2 ] 4 2 - 4k - {6k 4 1) 2 >0 Vx » (X -6k -I) 2 -3Ếk 2 -I6k 4 1 > ũ Vx {vì k(x 2 4 2x 4 2} ;> 0 Vx) Ta sẽ chọn k thỏa mãn -36k 2 - 16k 4 1 =0. -1 , 1 Suy ra k = -ý- (loại) hoặc k = Lời giải. Ta có

    4x 2 -4x412 , -12x 4 4 B =— - — = 44- x z +2x + 2 X 2 +2x42 = 44' -2(x 2 +2x-i-2) + 2x 2 -3X48 X 2 +2x + 2 = 2 + ^g->2. (x 4 I} 2 4 1 vj ị(x-2) 2 > 0 Vx 2(x-2) 2 (x 4 1) 2 > 0 Vx (x + 1) 2 +1 2:0 Vx Dấu '=" xảy ra khí X = 2. Vậy MinB - 2 khi X - 2. Bài toán 3. Cho phân thức M = x z 4 7x - 28 X — 1
  17. Tìm giá trị nhỏ nhất của M VỚI X > 1 :
  18. Tìm giá trị lớn nhất của M vói X < 1 . Hướng dẫn. Ta nghĩ đến cách tách tử thức như sau: X 2 4 7X+2S a(x-1)4b{x-c) 2 a + b(x-c) 2 X — 1 X — 1 X — 1 Bài toán 4. Cho X > ữ r tìm giá trị nhô nhất của X 3 4 X 2 4 X 4 3 phản thức s = - X Hướng dẫn. Ta tách X 3 -X 2 4X43 (X4 a)(x 4b) 2 4 cx 0 — — = c 4 (x + a) (x 4 b) 2 Bài toán 5. Cho X > 0, tìm giá trị nhỏ nhất của biểu A X 3 - 3x 2 4 5x + 4 thức A = — - — . Hướng dẫn. Ta tách . x 3 -3x 2 45X44 A = — - — 7 4 = x -3x 45+— X = X 2 -(34k 2 )X45+k 2 X4- X 2 J» (3 4 k 2 } f 3 4 k 2 = x -2 - — X 4 45- 3 4k 2 'V 4 ; k 2 x 4
  19. í x _3+k 2 ì \ iVx 2 4 4xk44 'j -,2 3 4k 2 2
  20. l X Tí ! 2 ) k, / ÍM _jc; 4 ư> 1
' (kx4 2) 2 c Í3+k 2 ì [-

2 X V 2 -4k Ta se tìm k sao cho 3+k 2 k Bài toán 6. Cho x>2 tìm giá trị nhỏ nhất của phân

  • X 3 -9x 2 4 2ÔX-24

    thức s = — T . x-2 Hướng dẫn. Đặt t = X - 2. Bài tập vận dụng Bài 1. Tìm giá trị lớn nhất, giá trị nhỏ nhất cùa A = 2x 4 1 X 4 2 B = - X 2 ( 2x 4 10 X 4 2x 4 3

    3x 2 4l1x4l2, _36x 2 -4X421 o — " u — . X 2 4 3X4 4 X 2 4 X4l Bài 2. Cho X > 1. Tìm giá trị nhỏ nhất của 3x 2 -7X + 31 E = - x-1 Bài 3. Cho X > -1. Tìm giá trị nhà nhất của 18x 3 -53x 2 4 2X4 82 G = X 4 1 X ÍStm số CÒN THIẾU TRONG HÌNH VUÔNG Bải 1. Tìm số tiếp theo cùa dãy số 1; 2; 4; 7; B: 11; 13; 14; 16; 17; ... Bài 2. Hây thay dấu ? bằng số thích hợp. 5 9 4 3 13 7 18 16 1 7 10 12 15 11 14 ? NGUYỄN ĐỨC TẤN (TP. Hổ Chì Minh) số NÀO MỚI ĐÚNG ĐÂY? (TTT2 S «155> Nhận xét. Rất nhiều bạn tham gia gửi bài và tất cả các bạn đểu tim ra đúng quy luật. Riêng bài 1 các bạn cần chĩ ra cụ thể hơn nữa quy luật bằng còng thửc của số hạng tong quát. Quy luật. .11 1 1 1 Bải 1 . Xét dãy số 4; -k ị±i 4 \ ... 3 7 13 21 31 3 1.2 + 1 7 2.3 + 1 13 3.4 + 1 1 1 1 1 7Ã 4.5 4 1' ãĩ“ 5.6 + 1 1 S5 hạng thứ n cùa dãy cỏ dạng u n = — — — - n.(n + 1) + 1 Vậy số hạng tiếp theo (số hạng thứ 6) lả 1 1 67 + 1 43 Bài 2. Dế thấy mỗi số nằm trong tam giác bằng lập phương của tổng ba số nằm ngoài tam giác. Theo quy luật đò, số cần điền vào dấu ? là {3 + 2 + 1)3 = 216. Xin trao thưởng cho các bạn cỏ lòi giải ịH J chính xác, ngấn gọn; Nguyễn Thị Quỳnh Chi , 6A2, THCS Yên Phong. Yên Phong; Nguyễn Phương Mai, 6C, THCS Lê Vãn Thịnh, Gia Bình, Bắc Ninh; Phạm Thị Kiều Trang, 8A2, THCS Yên Lạc. Yên Lạc, Vĩnh Phúc; Phan Minh Ánh, 6A, Võ Hông Thái, 7A, THCS Hoàng Xuân Hãn, Đức Thọ, Hà Tĩnh. Các bạn sau dược tuyên dương: Phùng Quốc Lán, 7Ẽ1, THCS Vĩnh Tường, Vĩnh Tường, Vĩnh Phúc; Nguyễn Khảc Thài Binh, 8B, THCS Nguyln Thượng Hiển, ửng Hòa, Hà Nội; Lã Việt Cường, 7A, THCS Nam Cao, Lý Nhân, Hà Nam; Trần Nữ Tố Uyên , 7D, THCS Lý Nhật Quang, Đô Lương, Nghệ An; Nguyền Trúc Quỳnh, 7/1. THCS Lè Văn Thiêm, TP. Hà Tình, Hà Tĩnh- NGUYỄN XUẦN BÌNH ểaipra SAI T rong một cuốn sách có bài toán với lời giải sau: Bài toán. Cho 11 1 1 a c b-c a-b và ac 0, a b, b * c. Chứng minh rằng — = . c b - c LẦM ở ĐÂU? o 11.11 Suy ra — — + — = — a-b c b-c a x „ . . - 1 ,1 c-a-b Ta lại có — — 4 — = - — — — . (1) a-b c (a-b)c _J 2 a b + c ị2)

    b-c a (b-c)a Từ (1 ) và {2), suy ra (a - b)c = (b - c)a.

    4 ' a a — b Do đó — - . c b-c Lởi giải. Ta cỗ — a Ị Ị b-c a - b Theo bạn lời giải trên đa ổn chưa? Nếu chưa ổn thì giải thế nào mới đúng. GIÁ TRỊ LỚN NHẤT (TTT2 số 1 55} Theo lởi giải của bạn: Chu Tuấn Kiệt. 9A2, THCS Hạ Hòa, Hệ Hòa r Phú Thọ Ta viết A(x) = 1 3 = 1 I J với X* 16. i/x- 4 Vx-4 Để biểu thức A{x) đạt giá trị lớn nhất thi A (x) > 0 và Vx -4 cần lấy glá trị dưứng (khi X > 16} nhỏ nhất. Giả sử với giá trị tùy ý X a > 16 thì luôn , a n a 4 1 6 2a chọn đước sỏ thực b = — + 8 = — -—< — = 3. 2 2 2 mà b = -- + 8 > + 8 = 16, tức là 16 < b < a, khi 2 2 đóO<Vb-4<’v/ã-4 f nên A(b)>A(a), do đó không thể chọn được giá trị của số thực X để \X - 4 lấy giá trị dương nhỏ nhất. Vậy khi A(x) lấy các glá trị lè SỂ> thực thì không tón tại giá trị lứn nhất cùa A(x). Nhận xét. Một số bạn chỉ xét các giá trị nguyên của A{x) nén suy ra giá trị lớn nhất của A{x) là A(25) = 8. ..... Cảc bạn sau cỏ lời giải đủng được ịlỊ™ thưởng' kì này: Chu Tuấn Kiệl 9A2. THCS Hạ H t a Hạ Hóa PI Thọ : Chu Văn Việt, 8E1, THCS vĩnh Tường, Vĩnh Tường. Vĩnh Phúc; Đặng Thị Hoài Anh. 9B. THCS Nguyễn Thượng Hiền, ứng Hòa. Hả Nội; Mai Ánh Quỳnh, 8A, THCS Chu Vãn An, Nga Sơn, Thanh Hóa: Vỡ Hùng Tuấn, 9A r THCS Đặng Thai Mai, TP. Vinh; Lễ Thanh Hùng, Bùi Thanh Trúc, BA, THCS Bạch Liêu, Yẻn Thành, Nghệ An, ANH KỈNH LÚP PHƯƠNG PHÁP TÍNH TổNG (Tiếp thao trang 2) Dù vậy ««»,- 1 . 11 . 11 . 1 2(P n — Spi) = 1 - — + — — - + — — — + ... 4 n n 3 2 4 3 5 n-1 1 . 1 1 n+1 n n+2 ..ui-. 1 1 2 n-t-1 n + 2 11 1 2S n = 2P n -1-4 + — Í- + — 1- n n 2 n+1 n+2 2 2 1 1 | 1 | 1 n(n + 3) n + 1 2 n + 1 n + 2 2(n + 1)(n + 2) Vậy suy ra s n = n(n + 3) 4<n+1)(n +2) Bài tập Bài 1. Tìm công thức của mỗi biểu thức
  • P n = 1 — a + a 2 4 a 3 4 ... 4 (-1) n a n .
  • s n = 1 - 2a 4 3a 2 - 4a 3 4 ... 4 (n - 1)(-1) n a n . Bài 2 . a) Tim hai chữ số tận cùng của p — 1 — 3 4- 3? 33 |_ 3 I 00
  • Tìm chữ số tận cùng của s = 1 - 2.3 4 3.3 2 - 4.3 3 4 ... 4 101. 3 10ữ . Bài 3. Chứng minh công thức sau -2 . „2 „2 . . ..2 n{n- 1 )( 2 n 4 l) r 4 2 + 3 Z + ... 4 n - — - .

    © CÁC NƯỚC DÔNG NAM Á BÍNH NAM HÀ AC ià từ viết tắt của Cộng đồng ASEAN bằng tiếng Anb (ASEAN Community). Cộng đóng ASEAN thành iệp chính thức từ 31. 12.2015. Năn 1 2016 này tạp chí Tùán Tuổi thơ mở chuyên mục Cửù sổ AC để bạn đọc hiểu hơn vê vùng đất con người rộng tởn của 10 quốc gia vởi 625 triệu dân. Đ ông Nam Á gổm 11 quốc gia. 10 nước đi tham gia hiệp hội các quốc gia Đông Nam Á (ASEAN) gôm: Brunây Đarutxalam (Brunei), Vương quốc Campuchia (Cambodia), Cộng hòa Inđônêxia {Indonesia), Cộng hòa dân chủ Nhân dân Lào (Laos), Liên bang Malaixia (Malaysia), Liên bang Myanma {Myanmar}, Cộng hòa Philippin (Philippines), Vương quốc Thái Lan (Thaỉland), Cộng hỏa xắ hội chủ nghĩa Việt Nam (Vietnam), Cộng hòa Xingapo (Singapore) và một nước chưa gia nhập ASEAN: Cộng hòa dân chủ Đông Timo (Timoi^Leste). Nước rộng nhất là Indonesia với diện tích 1 913 000 km 2 . Nước có diện tích hẹp nhất là Singapore với gần 700 km 2 - NƯỚC đông dân nhất là Indonesia với 235 500 000 người (nãm 2010}. Nước ft dân nhất là Brunei 400 000 người. Núi cao nhất là Cacabo Radi ở Myanmar với 5 885 m. Sông lớn nhất ở Đông Nam Á là Mê Công chảy qua Lào, Thái Lan, Campuchia và Việt Nam. Mật độ dân số cao nhất là Singapore với hdn 7500 người/km 2 , thấp nhất là Lào vỏỉ 27 người/km 2 GDP cao nhẫt là Indonesia với 1057 tỉ USD, thấp nhất là Lào với 9 tỉ USD. Bỉnh quần GDP danh nghĩa theo đẩu ngưừi cao nhất là Singapore vâi 56000 USD/ngưài/năm và thấp nhất là Campuchia vởi 1081 USD/ngưòi/năm. Nếu tinh theo sức mua tương đương thì cơn số này còn cao hơn. vể quy mô các nền kình tế, xếp theo thứ tự là: Indonesia, Thái Lan, Malaysia, Singapore, Philippines, Việt Nam, Myanmar, Campuehia, Brunei, Lào (tính theo thứ tự tổng số GDP theo USD tức tổng sản phẩm quốc nội). Quốc gia chì gõm 1 thành phô là Singapore. Quốc gia có nhiều đảo là Indonesia với gần 17 000 hòn đảo chạy dài 5000 km từTảy sang Đòng và 2 000 km từ Bắc xuống Nam, Indonesia cũng là nước nằm cả ở Bắc và Nam bán cầu, Quốc gia gồm có 2 phần rõ rệt là Malaysia vỏi một phần phía Tây thuộc bán đảo Malacca và phần phía Đông thuộc đảo Calimantan. Quốc gia không gặp thiên tai: bão, lũ, sóng thần là Singapore, nẳm gần như trùng vào xích đạo. Đa số các nước dùng múi giử 8. Một số nưâc dùng múi già 7 trong đó cỏ Việt Nam. Riêng Myanmar dùng giờ chuẩn 6.30. Quốc gia có khí hậu nhiệt đới gió mùa, mùa đỏng lạnh rõ rệt là Việt Nam. Mười quốc gia luân phiên nhau làm Chủ tịch mỗi năm của Hiệp hộj T nay gọi là Cộng đồng. Năm nay Lào là Chủ tịch ASEAN Ván phòng Tổng thư kí ASEAN đặt tại Jakarta, Indonesia. QUA HAI BAI TOAN NHO TRONG CUỘC THI TOÁN AUSTRALIA AMC NĂM 2015 Chúng ỉa xét hai bài tũán sau đây trúng các bài thi AMC (Australian Mathomatícs Compẽtition) năm 2015 cho học sinh các nhóm lóp 7-8 và 9-10. Đối với mồi bài toán chúng ta sẽ cùng thảo luận về phương pháp dạy vả học khi tìm lời giải. Qua đây chúng ta cũng có thể thảy được ‘'dạng 1 " toán mà các cuộc thi tìm kiếm tàl nâng toán học kiểu như AMC thưởng sử dụng. Bải toán 1 (Cáu hỏi 29, AMC 2015 dành cho lớp 7-8) Zoltan has a list ũf whole numbers, all largar than 0 but smaỉlér than 1000, He notices that every number in his li si is either one-third of another number ìn the list or three times another number ìn the list What is the ìargest nurnber ùf diderent whote numbũrs that can ba ùn Zọttan’s íist? Bái dịch sang tiếng Việt: Zoltan cố một dãy các sổ tự nhiên, các số trong dăy đều lớn hơn 0 nhiỉng nhở hơn 1000. Cậu ẩy .1 nhận thấy rằng mỗi sổ trong dãy hoặc là bằng -ị 3 của một số khác trong dãy, hoặc lè gấp ba lần một số khác trong dãy. Hỏi sổ nhiều nhất các sổ tự nhiên trong dãy của Zoltan cò thế có là bao nhiêu ? Qua bài toán này học sinh được rèn luyện, phát triển kĩ năng phán tích, khám phá, suy luận. Chúng tôi xin để xuất một cách sau đây để thực hiện mục tiêu đó: hãy nghiên cứu từ các trường hợp đơn giản!

    Trường hợp đơn giản thứ nhất Trước hết, chúng ta yêu cầu học sinh nghiên cứu bài toán khì dãy của Zoltan gồm các số lãn hơn 0 nhưng nhổ hơn 10. Hãy đặt ra các câu hỏi để khám phá: Nếu muốn số 1 nằm trong dãy của Zoltan thì số nào cũng phải (có thể) nằm trong dãy đó? T ương tự như vậy đối với số 2, 3, .... cụ thể: Sự xuất hiện của sđ 1 trong dây sẽ dẫn đến số 3 phải nầm trong dãy; TẠ NGỌC TRÍ Sự xuất hiên của số 2 trong dày sẽ dẫn đến số 6 phải nằm trong dãy; Sự xuất hiện của số 3 trong dãy sẽ dẫn đến sô 9 cỏ thể nằm trong dãy; SỐ 4 không thể nằm trong dăy vì không có số nguyên nào bằng của 4 và ba lần của 4 vượt quá 10; 3 Tương tự số 5, 7, 8 không thể nầm trong dãy. Sô 6, 9 có thể nằm trong dày (nếu có số 2, 3 nằm trong dãy). Từ đó đối với trường hợp náy dãy số nhiểu nhất cùa Zoltan lá 1, 2, 3, 6, và 9. Tức là trong trưởng hợp nảy dãy của Zũltan có nhiều nhất là 5 phẩn tử! • Trường hỢp dơn giản thứ hai Chúng ta yêu cầu học sinh nghiên cứu bài toán khi dãy cùa Zoltan gỗm các sổ lởn hơn 0 nhưng nhỏ hơn 100. Tương tự như trên đặt ra các câu hối để học sinh khám phả: Nếu muốn sổ 1 nằm trong dãy cùa Zoltan thi nhữhg số nào củng phải {có thể) nằm trong dây đó? Tương tự như vậy đối vửi số 2, ... Sau mỗi lần trả lởi các câu hỏi đố dãy của Zoltan lại được làm ‘giàu 1 ’ lên, cụ thể: Sự xuất hiện của số 1 trong dãy sã dẫn đến sổ 3 phải nlm trong dãy: Sự xuất hiện của số 2 trong dãy sẽ dẫn đến sỏ 6 phải nằm trong dãy; Sự xuất hiện của sô 3 trong dày sẽ dan đến sô 9 co thể nằm trong dãy; Sự xuất hiện của số 33 trong dãy sẽ dẫn đến số 99 có thể nằm trong dãy; Số 34 không thể nẳm trong dãy vì không có số 1 , nguyên nào bằng Ỷ của 34 và ba lần của 34 vượt quá 100: Tương tự số 35 không thể nằm trong dây; Tuy nhiên số 36 có thể nằm trong dãy (nếu có số 12 nằm trong dãy); © Như vậy các số lân hơn 33 (từ 34 trô đi) sẽ không thể nằm trong dãy của Zoltan nếu số đố không chia hết cho 3, còn các số chia hết cho 3 có thể nằm trong dãy đó (vì có thê’ lấy trong dãy số là -ị số đá!)- Như vậy dãy của Zoltan nhiều nhất sẻ có các số 1, 2, 3, ... , 33, 36, 39 99. Như vậy dãy đó nhiều nhất có 33 + (99 - 36) : 3 + 1 = 33 + 21 + 1 = 55 sổ. • Trở lại bài toàn 1 Từ nhừdg gì đã tìm được cố thể thấy Zoltan có thể có dảy với các số 1 , 2, 3, 4 333, 336, 339, ... ,
  • Tức là tất cả các số tír 1 đèn 333 và các số chia hết cho 3 kể từ sô 334. Trường hợp đó dãy sẽ cỏ 333 ị (999 - 336) : 3 + 1 = 555 số. Bài toán 2 (Câu hỏi 27. AMC 2015 dành cho lớp 9- lũ) How many positive intergers n less than 2015 . 1 1 have tha proparty that — + — can ba simptifìad to 3 n a traction with dehơminator lass than n? Bài dịch sang tiếng Việt: Có bao nhiêu số nguyên dương n nhỏ hơn 2015 có , - . 1 1 " ~ tính chất là —+— rùt gọn được thành một phân 3 n SỐ vởi mẫu số nhỏ hơn n? Ta lai xuất phát từ các trường hợp đơn giản hơn để có hương giải quyết. • Trường hợp đơn giản thứ nhất Có bao nhiêu số nguyên dương n nhỏ hơn 10 có 11 ,- tính chất là -7-— biểu diễn thành một phân số tối ^ 3 n giản vói mẫu số nhỏ hơn n? Bằng cách lần lượt xét các số 1 , 2, 3 9 chúng ta thấy bài toán chí có thể được thỏa mãn khi n = 6 (khi đó chủng ta được kết quả cũa phép cộng 1 hai phân số trẽn là -T). Khi thay như vậy bài toán này được thỏa mãn nếu phân sô có dạng ị n+3 3n và giản ước được cho V /' J K một thừa số chung lớn hơn 3, tức là tử sô và mau số có thím sỏ chung lốn hơn 3. Đối với trường hạp đang xét điểu đó xảy ra khi n = 6 và thừa số chung đỏ la 9. • Trường hợp đơn giản thứ hai Có bao nhiêu số nguyên dương n nhỏ hơn 30 có 11 ,- tính chất lã -7 + — biểu diễn thành một phân số tối 3 n giản với mau sổ nhỏ hơn n? Đối với trường hợp này chúng ta chỉ cẩn xét thêm các trường hợp n = 10, 11 29 và thấy chỉ có được thềm khi n = 15 và n = 24. Như vậy trưàng hợp nãy chúng ta có 3 đáp số là 6, 1 5 và 24 và thừa số chung đó cũng là 9. • Trở lại bài toán 1 Đến đây chúng ta đâ có thể dự đoán rằng có thể chĩ các giá trị của n là 6, 1 5, 24, ... thì bải toán mãi được thỏa mãn và thừa số chung của cả tử số vả mẫu số của phân số cỏ dạng nêu trên là 9. Dãy sổ 6, 1 5, 24, ... có tính chết gi? Dễ dàng dự đoán thấy rằng dãy này gồm các sô' bắt đầu là 6 và mỗi số của dãy kể từ số thử hai bằng số liền trước nó cộng thêm 9. Do đó đối vôi bài toán đang xét dãy số mà n có thể được sẽ là 6, 15, 24 2013 và kết quả cần tìm cỏ thể là (2013 - 6) : 9 I 1 = 224 sổ. Liệu đây có thể là đáp án của bài toán? Mẩu chốt cùa vấn đề là chủng ta phải tìm các sổ n để tử số (n - 3) và mẫu số (3n) có thừa số chung lớn hơn 3 (để giản ước được) thì bài toán mới được thỏa mãn. Do đó chúng ta phải tìm hiểu xem ước chung của n - 3 và 3n có thể có là nhữhg số nào. Từ đó chúng ta có thể giải như sau: Giả sử d là ước chung của n + 3 và 3n. Như vây thì đ cũng là ưức của 3(n + 3) vả 3n. Do đó d là ưỏc cùa 3{n + 3) - 3n = 9. Do đó d chỉ cỏ thể là 1 , 3 hoặc 9. Để cho mẫu sô của phân sô n ~3 nhỏ 3n hơn n thì chủng ta phải giản ưóc được cả tù và mẫu cho một thừa số chung d lân hơn 3 Vì vậy thừa số chung đó chỉ có thể là 9. Từ đó chúng ta có n + 3 = 9k, hay n = 9k - 3, k = 1, 2, ... Và vì vậy n chì có thể là các số 6, 15, 24, ... , 2013 như chúng ta đã dự đoản được ở trên. Một ý xin nhấn mạnh ở đây là bải thi AMC chỉ yêu cầu tó vào kết quả là một số từ 0 đến 999 đối vãi các bài từ 26 đến 30. Do đó trong khi làm bài thi các bạn học sinh chì cần nghiên cứu các trường hợp dơn giản thứ nhất và thứ hai là có thể "dự đoán” được kết quả cẩn tìm (555 với Bài toán 1 và 224 với Bài toán 2). Việc glảl chi tiết chỉ để làm rẽ tại sao có kết quả đó mà thôi. Có thể thấy rằng hai bài toán trên không đòi hỏi quá nhiểu kiến thức vể toán (Bài toán 1 có thể dành cho học sinh Việt Nam lớp 4-5 và Bài toán 2 có thể dành cho học sinh lớp 6) nhưng có thể giúp rèn luyện cho các bạn học sinh tư duy phân tích, dự đoán và suy luận toán học. Đó có lẽ là những kĩ năng quan trọng giúp cho các bạn học sinh sau này trong cuộc sống! Bài 1. a) Nếu X e z thì I X I t' - 2x khi X > 0 0 khí x< 0 — > I X I + X là sổ chẫn Do đó |a - b| + |b - c| + |c - d| -I- |d - a| = (|a - b| + a - b) + (|b - c| + b - c) + (|c - d| + c
  • d) + {|d - a| + d - a) là số chẵn vãi mọi X e z. Mà 2015 là sõ lẻ nên khàng có giả trị nào của a r b, c, d thỏa mãn.
  • Ta có 49A = 1 I —7^ ' j2ũ13 => 49A > 49B => A > B. Bài 2. Ta cỏ 49B = 1 ► 48 7 2015 + 1 ' 1_J_ 1 1 1 Ị 1 2 3 4 2015 2016 71111 1 1 Wl 1 1 '
  • M + 2 + 3 + 4 + "' + 2ÕĨ5~2ÕĨ6 l~[ĩ2 + "' ĨÕÕã ; 1 1 1 = —7— ■ —7- + ... +—7—. Do đỏ A =1. 1009 1010 2016 x y „5x
  • Ta có 77 = 77 => y = -7-. 3 5 3 Thay vào B ta tính được B = 8.
  • Ta có X - y - z = 0 — >x - z = y; y - X = -z: y T z = X. : c — v x - ' v y z 1. X y z X y z a b c Bài 3. Đặt - 7 - = 77 = 7 > k 12 9 5 => a = 1 2k; b = 9k, c = 5k. Từ abc = 20 ta tìm được lí -1 ^ a — 4; b = 3; c = — . 3 3'
  • Gọi x; y; z lần lượt là 3 sỏ’ cần tìm vãi X + y + z
  • 420- Ta có 6 9 2 X y z 7 = íĩ y 3^7 rí = f X + y + z 7,11.3 — — 4 — — 4 - — 2 6 9 2 = 108 => X = 1 26; y = 1 32; z = 1 62. Bài 4. Gọi H là trung điểm của BO. Ta có OH = HB=AC. Vẽ tam giác đều MBC (M, A cùng phía với BC). Ta có ÁBC + ÁCB = 90° => ABC = 75°; HBM + MBC = ẤBC = 75° => HBM = 1 5°. Như vậy AABC = AHMB {c.g.c) Suy ra MHB = BÂC = 90° => HM J BO. Do đó BMO là tam giác cân tại M. => BMO - 1 50° => ÓMC - 1 50°. Từ các điểu trèn ta chứng minh được ABMO = ACMO (c.g.c) => QB - oc. Vậy ABQC cân tại o. ÀBD - BEC (so le trong) > BẾC - ẺBC (- ÃBD).

    Do đó ABCE cân tại C => BC = CE. Chứng minh được ABDA - ABDH — > AD = DH. Mà DH < DC, súy ra AD < DC. Theo định lí Pytago ta có BD 2 - AB 2 - AD 2 , de 2 =ce 2 +cd 2 . Ta lạl có AB < BC = CE và AD < DC => BD < DE. Vậy chu vi AABD nhỏ hơn chu vi ACDE. Bải 6. Ta lấy bất ki 9 trong 10 hộp thuổc đã chù và đánh số từ 1 đến 9, một hộp còn lạl không dành số. Lấy ra à hộp đánh sổ 1 , 2, 3 9 lần lượt 1 gói, 2 gói, 3 gói, .... 9 gói, Như vậy ta lấy ra 45 gói và đem cân chúng.

    Nếu khối lượng cân được là 4500 g thi hộp không đánh số lã hộp làm sai. Nếu khối lượng là (4500 - lOa) g thì hộp đảnh số a là hộp làm sai (với a bằng 1,2, 3). DỀ THI CHỌN HỌC SINH GIỎI MÔN TOÁN LỚP 9 HUYỆN VĨNH TƯỜNG, TỈNH VĨNH PHÚC Năm học: 2015 - 2016 Thời gian íàm bài: 150 phút Câu 1. (3 diểm) J(a 2 + 1)fb 2 +1)
  • Rút gọn biểu thức M = (a + b) - 1- — - ' với a, b, c > 0 và ab 4 bc 4- ca = 1 V c 2 + 1
  • Cho am 3 = bn 3 = cp 3 và — + — + — = 1 . Chứng minh rằng ìfã ■ v/b ■ Ịỉc = ỉm+bn^+cp 2 . m n p Câu 2. (2 điểm)
  • Giải phương trình X 2 + 3x + 1 - (x + 3 )\/x 2 +1.
  • Tim các số nguyên X, y thỏa mãn X 2 - xy - 6x - 5y - 8. Câu 3. (1 điểm) 12
  • Cho các số dương X, y thỏa mãn X + y = 1. Tìm giá trị nhỏ nhất của biểu thức A = — T— — - + — + 4xy. X 2 + y 2 xy
  • Cho các số dương a, b, c thỏa mãn abc < 1. Chứng minh rằng ^ + — - — > a + b- c. b c a Gàu 4. (3 điểm) Cho tam giác ABC có BC - a, AC = b, AB - c. Gọi (I) là đường tròn nội tiếp tam giác. Đưởng vuông góc với Cl tại I cắt các cạnh AC, BC theo thử tự ở M, N. Chửng minh rằng
  • AM.BN IM 2 IN 2 ; , IA 2 IB 2 IC 2
  • 1, bc ca ab Gâu 5. (1 điểm)
  • Mỗi điểm cùa mặt phẳng được tô bởi một trong ba màu Dỏ, Xanh. vàng. Chứng minh rằng tôn tại hai điểm A, B được tô bởi cùng một màu mà AB -2016.
  • Trong một trường học cố 1000 sinh viên, được đánh số từ 1 đến 1000. Một nhóm gồm 500 sinh viên

    được gọi là "Nhóm hay" nếu cỏ một sinh viên trong nhóm có số chia hết cho so của một sinh viên khác trong nhóm đó và gọi là ''Nhóm tói h nếu điều đỏ khóng thỏa măn. Ví dụ 500 sinh vièn có s6 từ 1 đến 500 là một "Nhóm hay" bồi vi 13 là ưâc của 26 và cà hai sinh viên mang số 13 vá 26 đều thuộc nhỏm đó. Một "Sinh viên hay" là sinh viên không thuộc bất kì "Nhóm tồi" nào. Tìm sinh viên cố số lớn nhất trong các "Sinh viên hay". Giải toán quã thiẩ Bài 1(155). Tìm các số nguyên dương a, b, c thỏa min đổng thời các điều kiện sau: (i) ab - b - a! = 1; (ii) cb + c - b! = 1; (iií) a 2 - 2b 2 + 2a -4b = 2. Ldi giải. Từ (iii) suy ra a là số chấn. Vì a là số chẵn nên từ (ì) suy ra b lá số lè. (1) Vì b là số lẻ nên b + 1 là số chẵn, suy ra c(b +1 ) là số chắn hay cb + c là số chẵn. Do đó từ (ii) suy ra b! là số lẻ. (2) Từ (1) và (2) suy ra b = 1.

    Với b = 1 thay vào (ii) suy ra c = 1 .

    • Với b = 1 thay vào (iii) suy ra a = 2. Vậy (a; b; c}=(2; 1; 1). Nhện xét. Bài toán này nhìn qua tưởng khó nhưng thực chất lời giải rất đơn giản. Có khá nhiều em tham gia giải bài nhưng nhiều lời giai dài dòng, phức tạp. Xin kể tên một số bạn cố lời giải tốt: Nguyễn Thị Mai Lình, 6B, THCS Lý Nhật ũuang, Đô Lương; Nguyễn Xuân Hưng, 6C, THCS Cao Xuân Huy, Diễn Châu, Nghệ An; Nguyễn Trung Hiếu, Nguyễn Thị Ngọc Trâm, Đặng Đình Hựy, Nguyễn Thị Việt Trà. 6B; Bùi Hồng Quân, Nguyễn cẩtn Vị, Phạm Trần Duy Khánh, 6C, THCS Hoàng Xuân Hãn, Đức Thọ, Hà Tĩnh; Nguyễn Quốc Trung, Ngõ Đặng Công Vinh, 7B9, THCS Chu Văn An, Ngô Quyền. Hải Phỏng; Nguyễn Quốc Thử, Nguyễn Đức Tân, Triệu Hóng Ngọc, 7A3, THCS Lâm Thao, Lảm Thao. Phủ Thọ. PHÙNG KIM DUNG A B p c Nhận xét. Có nhiều bạn gửi bài giải về tòa soạn. Các bạn sau cố lởi giải tốt: Từ Tấn Dũng, 7D. THPT chuyên Hà Nội - Amsterdam, cầu Giấy, Hà Nội; Nguyễn Quốc Trung, Cao Sơn Tùng, 7B9, THCS Chu Văn An, Ngõ Quyển, Hải Phòng; Phạm Thúy Linh, Triệu Hổng Ngọc, Ngô Binh Minh, 7A3, THCS Lám Thao, Lâm Thao; Trấn Quang Vinh, Nguyễn Đức Mạnh, 7A, THCS Hùng Vương, TX. Phú Thọ, Phú Thọ; Tạ Kim Thanh Hiển, 7A4, THCS Yên Lạc, Yèn Lạc Vĩnh Phúc; Lê Vản Mạnh, 6B; Nguyễn Xuán Hà, 7A; Nguyễn Thu Huyền, Trẩn Duy Minh, Nguyễn Trình Tuẩn Đạt, Nguyễn Thị Linh Đan, 7D, THCS Lý Nhật Quang, Đô Lương; Đường Minh Quân, Nguyễn Trọng Trung Phong, Đường Hải Sáng, 7C, THCS Bạch Liêu, Yên Thành, Nghệ An; Lẽ Thị Hằng Nhi, Thái Thị Thu Sang, Nguyễn An Na, 7A. THCS Hoàng Xuân Hãn. Đức Thọ, Hà Tĩnh. HỒ QUANG VINH Bãi 2(155). Cho tam giác ABC có AB + AC - 2BC. Gọi l là giao điểm các đường phân giác trong của tam giác. Gọi M, N ỉheo thứ tự là trung điềm của AB, AC. Chứng minh rằng ÃMI-ẤNI - 1âữ°. Lửi giải. Vì M, N theo thứ tự là trung điểm của AB, AC và AB - AC = 2BC nên 2M B + 2NC = 2BC, từ đó MB + NC = BC. Do đỏ tân tại điểm p thuộc đoạn thảng BC sao cho BP = BM vả CP = CN. Ta có ABMI - ABPI (c.g,c) và ACNI - ACPI (c.g.c). Do đó ẤMI-ẤNI = {1 80° - BMÌ) + (1 80° - cíìi) = 360° -{BPỈ + CPĨ) = 360° -180° =180° Sài 3(155). Giả sử n là số nguyên dương sao cho tổn tại các số nguyên dương a, b, c thỏa mãn ab t a 2 c r b 2 c I âbủ 2 — 1ữ1 n . Chửng minh rằng n lá sổ chắn. Lài giải. Ta có ab + a 2 c + b 2 c + abc 2 = 1 01 11

    (a + bc)(b + ca) - 101 n J . „ |a I bc = 1 0 1 k Vi 1 01 là 50 nguyên tô nên \ (k, m € !ỉ). I b • ca - 1 01 m Vì vai trò của a, b như nhau nên không mất tính tổng quát, có thể giả sử a > b. Khi đố (b + ca) - (a + bc) = (a - b)(c - 1} > 0 =4 101 m > 101 k =4> m > k. 12 Ta lại có (a + bc + b + ca = 1 01 k 4101" 1 [b + ca-a-bc = 1D1 m -101 k | {a + b)(c +1) =101 k í101 nr|_fĩ +1) (1) <=> ị i(a-b)(e-1) = l01 k {l0l m " k -1) (2) • NỂU c + 1 MOI thì (c - 1, 101) = 1 nèn từ (2) suy ra a - b : 1 01 k . MàO<a-b<a I bc = 1 01 k => a = b. Do đó 1 0 1 n = (a 4 ac) 2 , suy ra n là số chần. • Nếu (c + 1 , 101 ) = 1 , từ (1 ) suy ra a + b : 1 01 k . Mà a 4 b < a 4 bo = 101 k => c = 1. Do đó 101 n = (a + b) 2 , suy ra n là sô’ chắn. Vậy trong mọi trường họp, ta có n là số lự nhiên chắn. Bài toán được chứng minh. Nhận xét. Đây là để toán khó nèn không có nhiếu bạn giải đúng. Các bạn hãy suy nghĩ xem trong đầu bài, cố thể thay sổ 101 bằng một số nguyên tố bất kì hay không nhè. Các bạn có bài giải tốt: Nguyễn Quốc Trung, 7B9, THCS Chu Vãn An, Ngõ quyển, Hải Phòng; Tạ Nam Khánh, 3E1, THCS Vĩnh Tường, Vinh tường, Vĩnh Phúc; Cao Việt Hải Nam, 9E, THCS Đặng Thai Mai, TP. Vinh, Nghệ An. NGUYỄN ANH DŨNG Bãi 4(155}. Cho a, b, c là các sỏ’ thực dương thỏa mãn a — b - 0 - 3. Chứng mieìh rằng a 2 4ab 2 b 2 4 bc 2 | c 2 4 ca 2 2 b £ 4 a - b c 2 + b 4 c aMc + a Lởi giải. Bất đẳng thức cẩn chứng minh tương đương vãí a-- a 2 + ab 2 b- b z rbc 2 ^ c 2 4 b 4 c
  • íc- 7 . __2 \ c \ b +a+b' . cMbic.i ! / \ / \ ab be ca , H . 1 b A 4a + b c +b4c a +c*a a I c I a <1 <1. Áp dụng bất đẳng thức AM-GM, ta có b 2 4 a 4 b > 3\'ab 3 = 3b\''a. Do đỏ ab 1 âb \''a 2 1 , 2a 4 1
  • — <-— = ——<-(34341) = — b 2 4 a + b 3 bìĩã 3 9 9 Tương tự, ta có bo < 2b4l ca t , 2c4l c 2 +b + c 9 a 2 +c + a 9 Cộng theo vẽ các bất đẳng thức trên ta được ab be ca 2(a +b + c) 43 — 4 4 < — b 2 434b C 2 4b4C a 2 4C43 9 Suy ra đpcm. Đẳng thức xảy ra khi a - b - c - 1. Nhận xét. Cố rất nhiều bạn tham gia giải bài. Một sổ bạn biến đổi dài dòng mồi đì đến điều phải chứng minh. Các bạn sau đày có lời giải tốt: Đinh Thị Thanh Huyền, 9E1 . Nguyễn Thị Hương Nữ, 9B, THCS Vĩnh Tường, Vĩnh Tường; Trần Đức Duy, Phạm Thu Bẳc , 9A4, THCS Yen Lạo, Yên Lạc, Vĩnh Phúc; Bùi Thanh Trúc, 8A, THCS Bạch Liêu. Yên Thành: Trẩn Hữu Đửc Mạnh, Nguyễn Quang Nam, 9A, Cao Việt Hải Nam. 9E, THCS Đặng Thai Mai, TP. Vinh, Nghệ An; Phạm Huỳnh, 6B, Bùi Thị Minh Thư, 7A, Trển Sỷ Hoàng, 8C. THCS Hoàng Xuân Hãn, Đức Thọ, Hà Tỉnh; Nguyễn Thị Thuỷ Trang, Hoàng Thế Sơn, 9A1, THCS Hồng Bàng, Hổng Bàng, Hải Phòng; Nguỵễn Mình Nghĩa, 9B, THCS Nguyễn Thượng Hiền, ứng Hoà, Hà Nội; Nguyễn Thu Hiền, Nguyễn Hữu Trụng Kiên, 8A3, Nguyễn Thảo Chi, Tran Quốc Lập. Trần Thị Thu Huyền, 9A3, THCS Lâm Thao, Lâm Thao; Lé Nguyễn Quỳnh Trang, 9C, THCS Ván Lang, TP. Viẹt Trì; Phi Anh Cháu, 9A2, THCS Giấy Phong Chàu, Phù Ninh, Phú Thọ. CAO VĂN DŨNG Bài 5(1 55). Một đa đ6 thị G(V, E) bao gồm một tập hợp V các đỉnh và một tập hợp E các cạnh, trong đó E cố thể bao gồm các cạnh kép và khuyên. Xom ví dụ (e 4 , e 5 là cạnh kép, e ? là khuyên), Hãy vẽ biểu đồ cho mỗi đa đổ thị G{V, E), trong đó V = {p v p 2 , p 8 , p 4 . P 5 } vá
  • E - «P 2 , P 4 }. {P 2 , P 3 }, {P 3 , p g }, (P 5 , P 4 »;
  • E ={{P V p,}, (P 2 , P 3 HP 2 , P 4 }, {P 3 , P 2 l (P 4 , p,}, {P* PJ}. Nhận xét. Cố một số bạn vẽ sai đa đổ thị, nhiều bạn mắc lỗi thừa cạnh. Các bạn sau đây có lởi giải tốt: Từ Tấn Dũng, 7D, THPT chuyên Hà Nội - Amsterđam, cầu Giấy; Nguyễn Minh Nghĩa, 9B r THCS Nguyễn Thượng Hiền, ửng Hòa, Hà Nội; Tạ Nam Khánh, 3Ẻ1; Kìm Thị Hổng Lĩnh, 9E1 r THCS Vĩnh Tường, Vĩnh Tưởng, Vĩnh Phúc; Nguyễn Nhật Linh, 8A, THCS Lê Quý Đôn, TP. Tuyên Quang, Tuyên Quang. TRỊNH HOÀI DƯƠNG Bài 6(155). Cho tam giác ABC nội tiếp đương tròn {O; R). Gọi I là điểm nằm trong tam giác ABC (I không nằm trên cạnh của tam giác). Các tia AI. BI, Cl thứ tự cắt BC, CA, AB tại M. N, p. Chứng minh 11 A ràng — — — í — — — + — — < — -. AM.BN SN.CP CP.AM 3(R_0I) 2 Trước hết xin phát biểu không chứng minh hai bô’ để quen thuộc. • Bô’ để 1. Vởi mọi số a, b, c ta có {a + b + c) 2 > 3(ab + bc + ca). Mngh? Trấn Hữu Đức Mạnh, 9A; Cao Việt Hải Nam , 9E, THCS Đặng Thại Mai, TP. Vinh, Nghệ An; Phạm Trần Duy Khánh, 6C, THCS Hoàng Xuân Hãn, Đức Thọ, Hà Tĩnh; Nguyễn Quốc Trung, 7B9, THCS Chu Vàn An, Ngô Quyền, Hải Phòng; Triệu Hông Ngọc , 7A3, THCS Lâm Thao. Làm Thao; Lẽ Nguyễn Quỳnh Trang, 9C, THCS Văn Lang, TP. • BỔ đề 2, Nếu điểm I nằm trong tam giác ABC và AI, BI. Cl theo thứ tự cắt BC, CA, AB tại M, N, „... IM IN IP AM AN AP Trở lại bài toán Theo bất đlng thức tam giác và các bổ đề 1 , 2, ta có R - OI = AO - OI < AI. Tương tự R - OI < BL R - OI < Cl. .... 1 1 1 Từ đó —!— + —! Ị— AM.BN BN.CP CP.AM 1 1 AI BI BI Cl Cl AI “ (r_0I) z (ãm bn + bn'cp + CP ãm ị < 1 1 AI B| CỊ f 3(R-0I) 2 U M+BN + Cp i 1 f A IM . „ IN „ IP ? 3(R - OI) 2 Ị am BN cpj 1 M IN IP Ý 3(R-OI) 1 3(R - OI) : AM BN CP ■<3-D z =■ 3(R -01)^ Nhận xét. Các bạn sau có lỏi giải đúng: Lá Nguyễn Quỳnh Trang, 9C, THCS Văn Lang, TP. Việt Trì. Phú Thọ; Tạ Nam Khành, SE1, THCS Vĩnh Tưởng, Vĩnh Tưởng: Phạm Thu BẳG, 9A4, THCS Yên Lạc, Yên Lạc, Vĩnh Phúc; Trần Hữu Đức Hạnh, 9A, Cao Việt Hải Nam, 9E, THCS Đặng Thai Mai. TP. Vinh, Nghệ An. NGUYỄN MINH HÀ Thi giải toán qua thui Việt Tri, Phú Thọ; Nguyễn Minh Nghĩa, 9Đ, THCS Nguyễn Thượng Hiền, ửng Hòa; Từ Tấn Dũng, 7D, THPT chuyên Hà Nội - Annsterdam, Cầu Giấy, Hà Nộí; Tạ Nam Khánh, 8E1 THCS Vĩnh Tường, Vĩnh Tường; Phạm Thu Bắc, 9A4, THCS Yên Lạc, Yên Lạc, Vĩnh Phúc: Nguyễn Nhật Linh , 8A, THCS Lẽ Quý Đôn, TP. Tuyên Quang, Tuyên Quang. Từ sổ tháng 9 nám 2015, Công ty cổ phẩn Dịch vụ Giảo dục Việt Nam sẽ tặng các khóa học trực tuyến trèn vvebsite: hocmai.vh cho các bạn học sinh được thưdng trong các chuyên mục vá các bạn học sinh được khen trong chuyên mục Kết quả thi giải toán qua thư. Các bạn học sinh sau khi nhận được mã cung cấp thì đãng kí tại địa chỉ: thes.hocmai.ưnAoantuoitho (Xin liên hệ SĐT 0966464644 để được giải đáp). HOCMAI reraro KHÔNG VẼ ĐƯỜNG TRÒN Bài toán. Cho tam giác ABC nhọn, trực tâm H, nội tiếp đường tròn (0), Không vẽ đường tròn đương kính AH r hãy dựng giao điểm K khác Acủa đương tròn này vãi đường tròn (O). PHẠM TUẤN KHẢI (Hả Nội) 233X1 m CÓ CHIA HÍT KHÔNG? {TTT2 số 155) Trong 11 số đã ghi ồ đỉnh đa giác có 6 số chìa cho 3 dư 1, ta thay chúng bâi sđ 1, còn 5 số chia cho 3 dư 2 ta thay chúng bởi số 2. Nếu tìm được một tam giác cân mà ba đỉnh đếu là số 1 , hoặc ba đình đểu là số 2 thì dễ thấy tổng ba số ghi ở các đỉnh đó là 3 hoặc 6 do đó tổng ba sô ban dẩu đã ghi ử các đỉnh đó chia hết cho 3 (điểu cần chứng minh). Khi ghi 6 số 1 và 5 số 2 vào 11 đỉnh của đa giác thì không thể xảy ra mỗi số 1 xen giữa các số 2. do đó tồn tại ít nhất hai đỉnh liên tiếp ghi sõ 1, giả sử là A, B. Xét bốn đỉnh liên tiếp D, A, B, c,
  • Nếu ba đỉnh A, B, c (hoặc ba đình D, A, B) đểu ghi số 1 thì tam giác ABC (hoặc DAB) cằn do đa giác 11 cạnh là đa giác đểu, đó là tam giác cẩn tìm.
  • Nếu hai đỉnh D. c đều ghi số 2 thỉ do đa giác

    có 11 đỉnh nên tốn tạl tam giác ABE vỏi đĩnh E thuộc đường trung trực của AB và cũng thuộc đường trung trực của ĐC,

    Già sử đỉnh E ghi số 1 thì ABE là tam giác càn có ba đỉnh ghi số 1 nên là tam giác cẩn tìm. • Giả sử đỉnh E ghi số 2 thì DCE là tam giác càn có ba đình ghi sổ 2 nên là tam giác cần tìm. Vậy bạn Toán nói đúng. l[€ll»llpí (TTT2.Ó15S) THỀ CỜ (Kị 78) 1.Sa6+ &e7 2.Sa8 Sxh7 3.§a7+ và thắng Các bạn sau được thưởng kì này: Hoàng Phúc. 7B, THCS Hoàng Xuân Hãn, Đức Tho, Hả Tĩnh; Chư Vãn Việt , 8E1, THCS Vĩnh Tưòng, Vĩnh Tường, Vĩnh Phúc LÊ THANH TÚ Nhận xét. Chỉ có hai bạn giải đúng bài này được nhặn phẩn thưởng là: Võ Thị Bảo Anh, Hoàng Thị Thu Huệ, 8A1, THCS Nghi Hương, TX. cửa Lò, Nghệ An. ANH CGMPA THÊ CỜ (Kì 80) a b c d e f g h LỂ THANH TÚ (Đại kiện tướng Quốc tế) PhiBÊiciiDg MẤT TRỘM VÌ ngủ quên NGUYỄN NGỌC SƠN (9B, THCS Nguyễn Thượng Hiền , ứng Hòa ', Hà Nội) ôm nay thám tử Sêlôccôc hơi mệt nên ông quyết định nghỉ ở nhà. không đến vãn phòng, ông cùng vợ ngồi ãn sảng và uống cả phẻ ngay trước hiên nhà. Haì vọ chồng đang trò chuyện vui vẻ thì chuông điện thoại reo vang. Vừa ra hiệu cho chổng đừng nghe, vợ thám tử vừa nhanh tay nhấc điện thoại lên. Bình thường bà không bao giờ nghe điện thoại của chồng, nhưng hôm nay, bà muốn ông được nghỉ ngoi dưỡng sức. Tuy nhiên, chỉ sau một vài giây, bà đành phải đưa điện thoại cho chông:
  • Ồng Smith gọì, nhất định đỏi gặp ông. Thám tử Sêlôccỏc nghe máy rồi bảo vợ:
  • Tói lại không được nghỉ rổi. Phải đến nhả Smith thôi. Ông ấy đang bệnh tật thế, mình không thể không giúp. Hi vọng sê sớm xong việc để về nghỉ. Một lúc sau, thám tử đi có mặt tại nhà ông Smith, ngưài bạn thân từ thuở thiếu thời, hiện đang phải chống chọi vởi bệnh dạ dày và bệnh khớp. Óng Smith lo lắng kể với thám tử: -Tôi qua tôi đến ăn cơm ở nhà một người họ hảng. Về hởi muộn, lại uống chút rượu nẽn tôi mệt quá, lãn ra ngủ luôn. Sáng tỉnh dậy, chầng thấy ví và đổng hồ đeo tay đâu cả. Tôi nhớ là khi về tới nhà, tôi bỏ ví ra khỏi túi áo, tháo đổng hổ rồi nằm luôn ra ghê đí-văng ỏ phỏng khách.
  • Ổng ngủ cả đêm ở phòng khách luân à?
  • Ừ! Tồi ngủ một mạch, chẳng biết gì. Sau một hổi trò chuyện với ông Smith, thám tử Sêlỏccôc đã ‘khoanh vùng 1 ' được 3 đối tượng nghi vấn: Cõ giúp việc Anne, đứa cháu Aeron và ông hàng xóm sảt nhà David. Như mọi lần, thám tử lần lượt trò chuyện với từng người. Đầu tiên là cô gỉủp việc.
  • Tối qua lúc ông Smith vể T cô có biết không?
  • Không. Tôi chẳng biết óng chủ về lúc nào vi tối ở lì trong phòng mình trên tầng hai. Tối qua ông ấy không ăn cơm nhà nên tôi nấu nưóng đơn giản rói lên phòng nghỉ ngơi từ sớm. Sảng nay, xuống đi chợ tồi mới thấy ông ấy nằm trên đí-vằng. Tiếp theo IÉ đứa cháu đang ở nhờ nhà ông Smỉth.
  • Cậu đâ làm gì, ở đâu từ tối qua đến sảng nay?
  • Tối qua nhân chú Smith đi vắng, cháu tranh thủ đi chơi với đám bạn. Cháu về khả muộn nên ngủ li bì Sáng nay cháu dậy cũng muộn. Cháu vừa dậy thì bác đèn đây ạ. Cuối cùng là õng David hàng xóm.
  • Từ tối qua đến giờ ông có sang nhà ông Smith không?
  • Không! Tòi mới mượn được quyển sách hay quả nén cứ say sưa đọc, chẳng quan tâm đến việc gì nữa.
  • Vậy ư? Sách gi mà hấp dẫn thế?
  • Sách về một số loài thú sống dưới nước.
  • ô, thế thì thú vị thật! Tôi cũng biết một sô loài, rành rành là thủ - đẻ con và nuối con bằng sữa - nhưhg lại sông dưởi nưóc như cá.
  • Tối qua tôi vừa đọc vế một trong số những loài thú đặc biệt đố đấy - thú mỏ vịt, thám tử ạ. Sau khi hỏi chuyện cả ba người, thảm tử Sêlôccôc nói riêng với ông Smith: -Tỏi tìm được người khả nghi nhâ't rồi. Tất nhiên, chưa đủ cơ sở để kết luận người đỏ có đúng là kẻ đã lấy trộm không Tôi và ông sẽ cùng điểu tra thêm nhé. Ông Smith không thể đoán được thám tử Sêlỗccôc đă nghi ai? Các thám tử Tuổi Hổng có thể giúp được không? XIKIE* ĐÔI HOA TAI BIÊN MẤT (TTT2 số 155) Tất cả các bạn đều đoán được ngay; Thám tử Sêlôccõc nghi ngờ anh Jũhn. Li do là anh này nói rằng tác giả cúa Harry Potter là nhà vãn nổi tiêng người Mỹ, chuyên viết truyện trinh thám và ‘ ông ta" tên là J.K. Rowling. Sự thực thì J.K. Rovvlíng đúng là tác giả của Harry Potter nhưng nhà văn nổi tiếng này là nữ (chứ không phải "ông ta") và bà là người Anh (chứ không phải người Mỹ). il mũĩ Phần thưởng sẽ được trao cho: Nguyễn Trường Giang, 7A1, THCS Lâm Thao, Lâm Thao, Phú Thọ; Đinh Vàn Thái Sơn, 61, THCS Lẽ Quý Đôn, cáu Giấy, Hà Nội; Hoàng Trung Nguyên , 7A, THCS Đông Lâm, Tiển Hải, Thái Bình; Nguyễn Quốc Trung, 7B9, THCS Chu Vân An, Ngô Quyến, Hải Phòng; Thái Minh Dũng, 6B, THCS Đặng Thai Mai, Vinh, Nghệ An; Nhóm bạn Thái Thị Thanh Phương, Thái Thị Thu Thủy, Phạm Hồng Nam, 7C, THCS Hoàng Xuân Hân, Đức Thọ, Hả Tĩnh. Thám tử Sêlôccôc Bài 66: Ồn ThS. NGƯYẺN VŨ LOAN LTS. Nếu biểt tiếng Hán bạn sẽ: Nam mình. 1 . Hiểu các từ Hán Việt, sử dụng tốt hơn tiếng 3. Hiểu ngôn ngữ mà cứ 5 người trên thể giới có Việt của mình. Trong kho tù' vựng tiếng Việt rất hơn 1 người dùng. Dễ dàng hựp tác, lámàn với các nhiều từ Hán Việt. nước và vùng lảnh thổ Trung Quốc, Hồng Kông,
  • Đọc được sách cổ, vàn bia bằng chữ Hán và Đài Loan, Singapore và cả Nhật Bản, Hàn Quốc. Hán Nôm, thêm hiểu văn chương, lịch sử nước Nếu biểt cả tiếng Anh và tiếng Hán thì thật là tuyệt.
  • fe

    nạ?

  • iìg+^T + ^iiặl^mi, m mínmr^nm, &p . =f, SM*. »*»T*aas,
  • %Ả+ỉh\n+ìì+&ĩế+m c ì ntMa^sii?
  • 3EÀ+3m+ii+£ìi

    íẺ-Ễ-irt# tt

    3jÈMỉíÈ.

  • À + ìẫ ( w ) +$jìp] + ií + ìn te8t*ÈíKtt. ffeĩS*a m.
  • AitK+Bmn+n%
  • ±*+»S«+»+»T

    MiMSSíỉmT

    7.

    GEOMETRY HÌNH HỌC {Tiếp theo kì trước) VŨ ĐÔ QUAN
  • Perpendicular lines An angle that has a measure of 90° is a right angle. If two lines intersect at right angles, the lines are perpendicular. d 1 and d 2 are perpendicular, denoted by d 1 d 2' A right angle Symbol in an angle of intersection indicates that the lìnes are perpendicular.
  • Parallel lines lf two lìnes that are ìn the same plane do not intersecỊ the two lines are parallel. In the tigure d i d 2 lines d 1 and d 2 are parallel, denoted I/ d 2 . If two parallel lines are intersected by a third lire, as shown below, then the ang le measure are related as indicated, where x° + y° = 1 80°.
  • Maths Terms perpendicular lines haí đường thẳng vuông góc angíe góc measure sô đo right angle góc vu õng intersect cắt nhau denoted by đươc kí hiệu là symbo! kí hiệu, biểu tượng paratlel song song same plane cùng một mạt phẳng, đổng phlng ỉigure hình, chữ số third li ne đường thứ ba
  • Bạn hãy dựa vào từ khóa cho trong phẩn 5 để dịch phần 3, 4 gửi vế tòa soạn dự thi. Bài dịch tốt và gửí sớm (tính theo dấu bưu điện) sẽ có thưởng CÂU LẠC Bộ TOÁN TUẩl THƠ N goài cáo câu lạc bộ đã được nêu têr, các trưởng sau đã đăng kí Cảu lạc bộ Toán Tuổi thơ vôi Tạp chí: TH Tày Yên 2, TH Táy Yên AI, xã Táy Yên; TH Đông Yên 2, TH Đông Yên 1, xã Đông Yên; TH Đông Thái 2, xã Đóng Thái; TH Nam Yên 2, xã Nam Yên; TH Thị trấn Thứ Ba 1, TH Thị trấn Thứ Ba 2, Thị trấn Thứ Ba; TH Đông Thái 3, xã Đông Thái; TH Nam Thái 1, TH Nam Thái 2, xã Nam Thái; TH Hưng Yên 1, xã Hưng Yên r huyện An Biên; TH Đông Hổ, phường Đông Hổ, TX. Hà Tiên, Kiên Giang; THCS Nguyễn Khuyên, phường Khuê Trung, quận Cẩm Lệ, TP. Đà Nắng; THCS Nguyễn Hiển, xã Nam Hồng; TH Nam Đào, huyện Nam Trực, Nam Định: TH Tam Quang, xã Tam Quang; TH Minh Lảng, xã Minh Lãng, huyện Vũ Thư; TH Lê Hổng Phong , TP. Thái Bình, Thái Bình; TH Thị Trấn Vĩnh Tường , Vinh Tưởng, Vinh Phúc; TH Đoàn Thị Điểm, quận Nam Từ Liêm; TH Ban Mai, quận Hà Đông, Hà Nội; trường TH-THCS-THPT Đoàn Thị Điểm, TP, Hạ Long, Quảng Ninh; TH Đoàn Thị Điểm Ecopark, huyện Văn Giang, Hưng Yèn. Từ sổ báo này các Câu lạc bộ Toán Tuổi thơ các trường hãy tham giũ giải bài trong chuyên mục CLB Toán Tuẩi thơ (Tạp chí khuyên khích lời giải trình bày bằng tiếng Anh). Các bạn không là thành viên các CLB TTT vẵn tham gia giải bình thương như chuyên mục Góc Olympic truúc đây. Nhiều phần thưởng dang đón chớ các bạn dấy! CLB1. At which integer value of X, the expression „ 2015-x .. . . - M = TTTT — attains its maximuim value? 1963-x Find that maxímum value. CLB2. Let a and b be positive integers such that a 2 + b 2 is divisible by the product ab. Find â b the value oí the expression p = CLB3. Given the real numbers a, b, and c such that ab'C(a + b + c) = 1 - Find the value of c 2 {a + b} 2 (1 + a 2 b 2 ) the expressien Q = (1 + b 2 c 2 )(1 + c 2 a 2 ) CLB4. Find all whole numbers n, given that the sum of all digits of n equals n 2 - 201 5n + 8, CLB5. Given an equilateral trìangle ABC having an area of s. Let D and E be the points on BC such that ZBAD = ZDAE = ZEAC = 20°. Let M be the midpoint of AD, and N be a point on AB such that AN = AM. Find the sum of areas of the triangles ABD and ADN in terms of s. NGUYẾN ĐỨC TẤN (TP. Hỗ Chí Minh) THÁCH ĐẤU! THÁCH ĐẤU ĐÀY! TRẬN DÂU THỨ MỘT TRĂM BA MƯƠI LĂM Người thách đấu: Nguyễn Minh Hà, GV. trường THPT chuyên Đại học Sư phạnn Hà Nội. Bài toán thách dấu: Chủ hai điểm A, B thuộc đường tròn (O). Hai đường tròn (I), (K) tiếp xúc ngoài với nhau tại c, cùng tiếp xúc với AB và cùng tiếp xúc trong với đường tròn (O) (I, K cùng nằm trên một nửa mặt phẳng có bờ là đường thẳng AB}. Tìm quỹ tích điểm c khi hai đưòng tròn (I). (K) thay đổi. Xuất xứ: Sáng tác. Thời hạn: Trước ngày 08.4.2016 theo dấu bưu điện. TRẬN ĐẤU THỨ MỘT TRĂM BA MƯƠI BA (TTT2số 155} Bài toán này không khó nhưng chỉ có một bạn tham gia giải với lời giải quả dài. Do đó không có võ sĩ nào đãng quang trong trận đấu này. Sau đày là lời giải chuẩn của bàl toán. Ta cần có một bổ đề. • Bổ dề- Cho tam giác ABC và điểm M thuộc đoạn BC. Một đường thẳng bất kì theo thứ tự cắt các đoạn AB, AC, AM tại B' ; c\ M' (khác A), Khi 0 AM’ ' BC AB r 1 CB AC' Chứtìg minh. Dựng các điểm E r F thuộc AM sao cho BE ỉ/ CF // B’C' (hỉnh 1). Không mất tính tổng quát, giả SỬAE > AF, A Trổ lại giải bàì toán thách đấu. Đặt BC = a; CA b; AB = c. Gọi A 1 là giao điểm của AO và BC (hình 2) Vẽ AH 1 BC và IK 1 BC. A Theo bổ đề trên và tính chất đưởng phân giác, ta có AA-, A-ịB AC A-|C AB CB AB^ BC ACf a+c b a+b 2a+b+c ,(i) AI b + c b + c b h 1 b +c Vì IK // AH; S| BC - S| CA S| AB nên AI ỊK IK.BC 2 S 'BC (S| BC +S| Cft +S| AB ) J Hình 1 Vì BE // CF iì B'C' nén AB AE AC AF ME MF ÃB 7 ' ÃM 7 ’ ÃC^ÃM 7 ' MB MC‘ Do đó MC AB MB AC MC AE MB AF BC ÃB’ 1 CB ‘ÃC 7- B^ ÃM 7 * CẼTÃM 7 MC(AM + ME) +MB(AM -MF) (MC -MB) AM AM
  • ■ ■■ - AM’’ AAi AH AH.BC 1 2^ abc 2S A&c Do đó = 2. (2) AI Từ (1) và (2) suy ra — b + c - 2. b-c Vậy a = {a + b + c). BC.AM’ BC.AM' NGUYỄN MINH HÀ RÀNH CHS iẢt NKÁTUN H04 NHỔ \m ĐỊNH LÍ THALÈS CHO ĐƯỜNG TRÒN THÁI NHẬT PHƯỢNG (GV. THCS Nguyễn Văn Trỗi, Cam Nghĩa. Cam Ranh, Khánh Hòa) Trong chuyên mục Nhìn ra thế giới, Toán Tuổi thơ số 96 tháng 02 nãm 2011 có giãi thiệu một bài toán của "Cuộc thi đổng đội toán Baltic Way" như sau: Bãi toán. Cho 3 cung tròn (C 1 ), (C 2 ). (Cg) có 2 điểm chung A và B. cả 3 cung này ồ cùng một phía đổi vứì AB, trong đó (C 2 ) nằm giữa (C và (C 3 ). Tia Bx cắt (C n ), (C 2 ), (C 3 ) lần lượt tại M,|, M 2 , Mg. Tia By cắt (C,), (C 2 ). (C 3 ) lẩn lượt tại K v K 2 . íf :„u ,ì„„ M1M2 K-ịKị Ko. Chứng minh ráng - ... = ~ , , ■ 3 M 2 M 3 k 2 k 3 (Lời giải đăng trên TTT số 98. Hưởng dẵn: Xét các tam giác dồng dạng AMịMj và AK f Kj, i, ị F {í; 2, 3}) Kết quả cũa bài toán trén ta gọi là định lí Thalès trong đường tròn. Bài viết này sẽ mỏ rộng, bổ sung thêm một sổ tính chất khác của định lí Thalès trong đường tròn,
  • Vẽ tiếp các cung tròn (C 4 ), (C 6 ) (C ) có 2 điểm chung A và B vả gọi các giao điểm của các tia Bx và By với các cung tròn trên tương ứng là M 4 , Mg M n và K 4 , Kg K t1 . Ta sẽ chứng minh M|AMj không đổi vãi í, j fe {1. 2 n} và i j khi tia Bx thay đổi. (1) Gọi (C^), (C 2 ), (C 3 ) lần lượt là cung chứa góc a, [3, Y có 2 điểm chung A và B, Ta có MiẢM 2 = ẦMiB -ÃM 2 B =ot-p khủng đổi. Tương tự M 2 ÃM 3 = p -7 không đổi, MĨẦM 3 = a - Y không đổi, ....
  • Vì ẤK 1 M 1 = ẤBM 2 = ẤK 2 M 2 và MĨÀK 1 = xBy = M 2 ẦK 2 nẻn AAM 1 K 1 to AAM 2 K 2 . Tương tự ta có AAMịKị co AAMjK vó(i í, j e {1,2, .... n} và i j.
  • Xét tia Bx nằm giữa hai tia By và BA.

    Ta có ẨM 1 K 1 = ÀM 2 K 2 (vì AAM 1 K 1 co AAM 2 K 2 ), mà ÀM.|B > ẤM 2 B nên BM i K 1 < BM 2 K 2 , T ương tự ta có BM 1 K 1 < BM 2 K 2 < ỐM 3 K 3 < ... < BM n K n Suy ra M 1 K V M 2 K 2 , M 3 K 3 , M n K n đôi một cắt nhau. Gọi D, E, F thứ tự là giao điểm của M 1 K 1 với M 2 K 2 , M 2 K 2 vứi lvl 3 K 3 , M 1 K 1 với M 3 K 3 . TừiAM 1 K 1 co AAM 2 K 2 co aAM 3 K 3 -> ÃKĨD = ẤK 2 E = ẤK 3 F; mp = ẤM 2 E = ẤM 3 F. Suy ra các tứ giác sau nội tiếp AK 1 K 2 D. AK 2 K 3 E, ak 1 k 3 f, am^d, am 2 m 3 e, am 1 m 3 f. Do đó MiDM 2 - M-|AM 2 - oc — |3, M^EM^ - M 2 AM 3 |3 — y, MiFM 3 = MiAM 3 = ơ — y, Tức là các đường thẳng W 1 K 1 . M 2 K 2 , M„K 3 đòi một cẳt nhau tạo thành góc không đổi khi các tia Bx, By thay đổí. Gọi O-ị, 0 2 , O3 lần lượt là tâm của các cung (C^), (C 2 ) r (Cj) và vè các đường kính AO-ịN-p A 0 2 N 2 , A0 3 N 3 . Gọi I là trung điểm của AB, ta có 10*1, 0 1 0 2r 0 2 0 3 là các đường trung bình của các tam giac ABN r AN 1 N 2 , AN 2 N_. Mà I, 0 1; 0 2 , Oj thẳng hàng nên B, N-ị, N 2 , N 3 thẳng hàng vì 0,0 3 ỉl N 1 N 3 nên N 2 N S - kN 2 N 3 Ũ 2 Ũ 1 = kũ 2 0 3 . (2)

    . , , . - - , ... - . M1M5 AM 5 Từ{1) suy ra AAM.IVL co áAN.N, =? - 1 = —-7-7A w 12 12 AN 2 và AAM,M, eo AAN,N- => 3 - ' 2 2 3 2 3 n 2 n 3 AN 2 MhMo M2M3 M-|M ? N-iNt Do đó - . = - " <=> 7 ' ' = . . N 3 N 2 n 2 n 3 m 2 m 3 n 2 n 3 Kết hợp với (2) suy ra M;Mj NjNj _ OịOj MịMt = ìsụ^ = õÃ (không đổi) với i, j, t € {1,2, n). Nói riêng M 2 là trung điểm của M 1 M 3 khi và chỉ khi 0 2 là trung điểm của 0.|Q 3 - M;M| NịNị KịKị
  • Từ các kết quả trên suy ra - 77 2 = — -2- = — — 1 M,M t N|N t K|K| . „ M,M, MiMt ... , hay tì sỗ ' ' / = ' '™ không đổi với các chỉ số khác KjKj K|K t nhau i, j, t e (1; 2; 3; n)). A tì Ta cố A. M 1 , K 2 thảng hãng 4=> ÃM-|B - ÃK 2 B + xBy <=> xBy - a - p. Tương tự A, Mg, K 3 thẳng hàng <=> xBy = p - y. Vậy A, M 1 , K 2 thầng hàng và A, Mn, K 3 thẳng hàng <=> xBy = ct -p = p - Ỵ.
  • Đào lại nếu Cố a-p =p-Ỵ thì A, M 1r K 2 thẳng hàng A, M z , K 3 thẫng hàng. Thật vậyA, M r K 2 thẳng hàng <-> xBy = 0 ì-(Ị« xBy = (3 -y <=>A, M 2 , Kg thẳng hàng.
  • Giả sử GE-[Ì = {Ì-Y. Trén (C 2 ) và (C 3 ) lần lượt lấy K 2 và M 3 sao cho AM3 - AK 2 - AB. Tia BMg cắt (0 1 >. (C 3 ) lần lượt tại M v M 2 . Tia BK 2 cắt (C 3 ) tại K3. Thể thi A, M 1 , K 2 thẳng hàng và A, M 2 , K3 thẳng hàng. Chửng minh . Từ AM 3 = AK 2 = AB, suy ra ẢB'K 2 = ẨK 2 B; ÃBM 3 = ẤM 3 B = ẤK 3 B = ãk 2 b = ẤBK 2 = ãbM 3 + M 3 BK 3 = ấk 3 b + ívT 3 BK 3 =; "BK 3 -p-Y = ct-p. VặyA, M r K 2 thẳng hàng vàA, M 2 , K 3 thầng hàng.
  • Nếu A, M 1? K 2 thẫng hàng và A, M 2 , K 3 thẳng hàng thì MÃK-I - M 1 ÂM 2 = M 2 ẦM 3 - xBy. Bài toán trên còn có những tính chất thú vị và ứng dụng nào khác? Các bạn hãy tiếp tục khai thác nhé. Bài tập áp dụng Bải 1. Vẽ ba cung tròn (C^. {C 2 ). (C 3 ) thứ tự lá các cung chứa gốc 90° ,70°, 50° cùng dựng trên đoạn thẳng AB và nằm cùng một phía đối với AB. Lấy D c (C 2 ) sao cho AD = AB. Gọi E là giao điểm của AD và đưàng tròn (c.|), F là giao điểm của BE và đưàng Irỏn (C 3 ). Tính BFD. Bài 2. Vẽ ba cung tròn (C 1 ), (C 2 ), (C 3 ) thứ tự có tâm là 0 |, 0 3 , Qj sao cho 0.|0 2 = 0 2 0j cùng dỊtng trên đoạn thảng AB và nằm cùng một phía đối vởi AB, lấy M 6 (C 2 ). Tia AM cắt {c“), (63) thử tự tại c và D. Tia BM cắt (C.,), (C 3 ) thứ tự tại E và
  • Chứng minh rằng CF H DE- IMSO 2015 MATHEMATICS ESSAY PROBLEMS SOLUTION (Tiếp theo kì trưởc ) TRỊNH HOẢI DƯƠNG (GV. THCS Giảng Võ, Ba Đình, Hà Nội) (Sưu tầm vá giói thiệu)
  • Suppose those three tractions are - 7 -, 7,-7 27 10 12 49 3 y 13 their sum is — + — = — — . 45 45 45 45 Since the real sum of these three íractíons is - 77 , 45 which is 4Ệ-.ỊỆ = .l times of our supposed 45 45 7 three íractions. Hence these three tradions are 3 V 4J2 2 4 8 4 4 16 _ „ 5 7 35 9 7 63 15 7 105 of their denomiriatar i$ 35 + 63 - 105 - 203. A 10 Ag A b Aj A 4 A 10 and A 2 . Ag. Ag, Ag, Ag, A & , A^ or A 12 can form a triangle that has one interior angle equal to 45 ữ . A d A 10 and A 1 or A 7 can torm a triangle that has two interior angle equal to 45°. A^A^ and A 2 , A ộ , A fll A 12 can form a triangle that has one interior ang le equal ta 45°. A 3 A ri and A 1 can form a triangle thai has two interior angle equal to 45°. A 5 Ag and A 2 , A fil Ag, A 12 can form a triangle that has one interior ang le equal to 45°. A & A g + A 7 can form a trìangle that has two interior angle equal to 45°. A 2 A 12 4 Ag, A 0 can form a triangle that has one interior angle equal to 45°. A 2 A 12 ị A 1 can form a triangle that has two Interior angle equal to 45°. AgAg 1 A 3 , A ni can form a triangle that has one interior angle equal to 45°. AịịAg + A 7 can form a triangle that has two ìnterior angle equal to 45°. There are 8 + 2 + 4 + 1 + 4 + 1 + 2 + 1 + 2 + 1 = 26 tríangles that can be tound (rom this direction. We also can find 26 triangles from another diagongl diredion, thereĩore the re a re 26 + 26 - 52 triangles, and the re are the last 8 triangles A^AgAg, A^A f .A 3 , A 6 A 3 A 11' A 6 A 9 A 12’ A S A 11 A 2' A 9 A 12 A 3’ A 11 A 2 A 5' A 12 A 3 A 6 can form a triangle that has two interior angle equal to 45°. There are 52 - B = 60 triangles ìn total that have at least one interior angle equal to 45°. C; ABG = T = 3 = S BGC =4x=> S GEC = X S BGC 1 ^GEC - ^GOÊ ' ®GDB - 1 - 1 - 2, Sg DG = 2x. Hence 5 ABC AH g S ADG ®BDG Hư 2x Sagc = 1 => S ADG = 4x, 3 12 3 ... „ . 6 24 We have Sau {3 = 4x X— — — - -X, so 1 + 6 7 S ABH = S AGB -SahG = 12x-— x = — X Thus BH : HG : GM = 60x : 24x : 21x = 20 : s : 7.
  • Color the squares ùf the unit squares red and

    white as shown in the íigure. Each 3 om by 4 om rectangle will cover 6 squares of each color. However, there are 128 red squares and 124 white 124 squares. Note that — — = 20-67. Henoe, at most 6 20 rectangles can be cut off from the remaining part of the paper. The íigure at the bottom below is an example of 20 rectangles that have been cut off. 3 3

    XETEI ĐƯỜNG THẲNG (TTT2 số 155)
  • Đường thẳng Trong hình học, từ đường dùng để chỉ một dường thẳng kéo dài không kết thúc ò cả hai hưống. Đường thẳng có thể được gọi là đường thẳng AB hay đường thẩng d, Phần của đưởng thẳng từ điểm A đến điểm B được gọi là đoạn thẳng. A và B là hai đẩu mút của đoạn thẳng. Kí hiệu AB được dùng để biểu thị đoạn thẳng AB và AB được sử dụng để biểu thị độ dài của đoạn thẳng này. A B d
  • Các dường thẳng cắt nhau

    Nếu hai đường thẳng cắt nhau, các góc đối diện vói nhau gọi lả các góc đối đỉnh và cò cùng số đo. là hai góc đối đỉnh. Ngoài ra r x° + y° = 180°. Nhận xét. Có rất nhiều bạn đã gửi bài

    đến tòa soạn, hầu như các bạn đều

    dịch đúng. Xin được trao phần thưởng cho các bạn có bài dịch sát với nội dung hơn và trinh bày đẹp hơn lã: Tử Tấn Dùng, 7D, THPT chuyên Hà Nội - Amsterdam, cầu Giấy, Hà Nội; Thân Hoài Thương, 7/7, THCS Võ Như Hưhg, Điện Bàn, Quảng Nam; Ngõ Võ Hoàng Việt , 6A3, TH Thực hành Sài Gòn phường 4 quân 5, TP. Hó Chi Minh. Các bạn sau được khen: Hoàng Bảo, 71, THCS Lẻ Quý Đôn, Nghĩa Đô, cầu Giây; Đặng Thị Hoài Anh, 9B, THCS Nguyển Thượng Hiền, ứng Hòa, Hà Nội: Kiều Bảo My, 9A2; Ngô Thị Thuyết , 8A2, THCS Yên Phong, Yên Phong, Bắc Ninh: Nguyễn Trình Tuấn Đạt 7D, THCS Lý Nhật Quang, Đô Lương; Đào Thị Xuàn í-ộc, 7C, THCS Bạch Liêu, Yên Thành, Nghệ An; Cao Thị Khánh Linh, 6B, THCS Hoàng Xuan Hãn, Đức Thọ, Hà Tĩnh; Mai Ánh Quỳnh, SA, THCS Chu Văn An, Nga Sơn. Thanh Hóa; Nguyễn Nhật Linh, 8E, THCS Lẻ Quý Đôn, TP. Tuyén Quang, Tuyên Quang. 25 MAI vũ Bài 1NS. Tìm các số nguyên dương X, y thỏa mãn (X 2 4 4y 2 + 28) 2 = 17(x 4 + y 4 4 14y 2 4 49}. TRƯƠNG QUANG AN (GV. tnỉờng THCS Nghĩa Thẳng. Tư Nghĩa, Quáng Ngãi) Bài 2NS. Tìm tổng bình phựdng các nghiệm của phương trinh (x 2 4 2x) z -2013(x 2 4 2x) 4 2015 = 0. NGUYỄN ĐỄ (Hải Phòng) Bài 3NS. Cho điểm A nằm ngoài đường tròn (O). Vẽ các tiếp tuyến AB và AC đến đường tròn (0} {B, c là tiếp điểm). Gọi H là giao điểm của OA và BC, Đường thẳng qua H song song vãi AC cắt cung nhỏ BC tại D. AD cắt đường tròn (O) tại điểm thử hai E. Gọi R là bán kính đường tròn ngoại tiếp tam giác ODH. Chứng mịnh rằng BH < R. NGUYỄN ĐỨC TẤN (TR Hố Chí Minh) (TTT2 số 155} Bài 25NS. Ta thây I 4 , 2 4 , 3 4 , 4 + chia cho 5 đều có SỐ dư là 1. DođỏS = r + 2 n + 3 n + 4 n = 1 r + 2 r + 3 r 4 4 r (mod 5), với r là số dư khi chia n cho 4. Suy ra s chia hết cho 5 khi và chỉ khi n khỏng chìa hết cho 4. Có (2012 - 4) : 4 + 1 = 503 số nguyên dương nhỏ hơn 2016 vả chia hết cho 4. Do đó sô các số cẩn tìm là 2015 - 503 = 1512 (số). Nhận xét. Các bạn có lời glảl đúng bàì toán trên: Nguyễn Thị Thúy Trang, 9A1 , THCS Hổng Bàng r Hóng Bàng. Hải Phòng: Nguyễn Thị Hương, 9A4, THCS Yên Lạc, Yên Lạc, Vĩnh Phúc; Nguyễn Thảo Chi, Tran Thị Thu Huyền, 9A3; Bui Thị Quỳnh, Nguyễn Thùy Dương, 8A3, THCS Lâm Thao, Lâm Thao: Phan Trần Khánh Lình, SA. THCS Hùng Vương, TX. Phú Thọ, Phú Thọ. Bài 26NS. ĐKXĐ X > -2. Ta có BK lần lượt tại p và Q. Ta có các tứ giác AM HO và BNHO nội tiếp, ẤHB = 90°. Suy ra HÕM = HNÒ = HBÒ -> MOA = ỐHB = HBÒ. Do đó OM và ON thứ tự là tia phân giác của HOA, HOB. Suy ra MH = MA, NH = NB. Vì AM II NB nên

    =í HI // NB => HQ 1 AB. (1) IB NB HN Ta lại có ẤKB = NHB = NĐH => ẤBK = ẤBH. (2) Từ (1), (2) suy ra AB là đương trung trực cùa HQ nẽn HP - HO. (3) Mặt khác - - 77 - — > ỈH - IP, (4) MA NM BM MA (X 4 4)(VxT2 4 2) = (X 4 1)(x 2 - 2x 4 3) e* (y x + 2) 2 2 = [(X - Ị}+ 2][(x - 1} 2 + 2]. (1) Đặt a = v'x 4 2 ằ 0; b - X - 1 . Thay vào ( 1 ) ta được a â + 2a 2 + 2a = b 3 + 2b 2 + 2b (a - b)(a 2 4 ab 4 b 2 4 2(a 4 b) 4 2) = 0 <-> a = b (vì a 2 + ab 4 b 2 4 2(a 4 b) 4 2 > 0) Ị— — , 3 4 Vi 3 «vX42 = x-1<=?x = — . 2 Nhận xét. Các bạn sau có lời giải tốt: Nguyễn Thị Thùy Trang, 9A1, THCS Hổng Bàng, Hống Bàng, Hải Phòng; Đặng Thị Hoài Anh. 9B, THCS Nguyễn Thượng Hiền, ửng Hòa, Hà Nội; Nguyễn Thị Linh Đan, 7D r THCS Lý Nhật Quang, Đô Lương, Nghệ An; Nguyễn Thị Hương Nữ, Phan Huyền Ngọc, 9B, THCS Vĩnh Tường, Vĩnh Tường; Nguyễn Thị Hương , 9A4, THCS Yên Lạc, Yén Lạc, Vĩnh Phúc' Bài 27NS. Bạn ơọc tự vẽ hình. Gọi I là giao điểm của AN và BM. Tia HI cắt AB và Từ (3), (4) suy ra HP - PQ - 21 p. ' MA IP 1 Do đó — — =■ — — - — . AK PQ 2 Nhận xét. Các bạn cỏ lời giải tết: Kim Thị Hống Lĩnh, 9E1, THCS Vĩnh Tường. Vĩnh Tương; Nguyễn Thị Hương, 9A4, THCS Yén Lạc, Yên Lạc, Vĩnh Phúc; Đặng Thị Hùàì Anh , 9B, THCS Nguyên Thượng Hiển, ứng Hòa. Hà Nội; Nguyễn Thị Thùy Trang. 9A1, THCS Hồng Bàng. Hổng Bàng, Hải Phòng; Trần Thị Thu Huyền, 9A3, THCS Lâm Thao, Lâm Thao, Phú Thọ. . L\ r — Các bạn được ịh ường kì này: Đặng Thị Hoài Anh. 9B, THCS Nguyễn ThƯỢng Hiển, ửng Hóa, Há Nội; Phan Huyền Ngọc, 9B, THCS Vĩnh Tương, Vĩnh Tường, Vĩnh Phúc; Nguyễn Thị Thùy Trang, 9A1, THCS Hổng Bàng, Hổng Bàng. Hải Phòng: Trần Thị Thu Huyền, 9A3, THCS Lâm Thao, Làm Thao, Phú Thọ Ảnh các bạn được thưỏng ỏ bìa 4. NGUYỄN NGỌC HÂN
  • Cuốn sách: Sài giảng sõ học Các tác giả: Đặng Hùng Thắng, Nguyễn Vản Ngọc, Vũ Kim Thủy Không có gì cổ và đơn giản hơn số học. Nhưng nhỉểu bài toán mà các nhà toán học lởn thách đấu đến nay chưa giải được lại thuộc về số học. Để phục vụ tốt cho một phạm vi đông đảo các bạn đọc thì các chương 1 và 2 được trình bảy các kiến thức cơ bản nhẹ nhàng, dễ hiểu rất thích hợp với các em học sinh cấp THCS, giúp các em được trang bị các kiến thức cơ bản quan trọng để phục vụ cho các kì thi học sinh giỏi cũng như thi vào các trương THPT chuyên. Các thầy cô giảo cũng có thể dùng như các giảo trình phục vụ giảng dạy các đội tuyển học sinh giòi. Các chương 3. 4 vả 5 thích hợp để làm giáo trình giảng dạy cho các lớp chuyên khối THPT và các em học sinh lớp 9 muổn tìm hiểu sâu thẻm bộ môn sổ học. Có lẽ khó nhất là chương 5, đi sâu vảo phương trình Điôphãng, có thể dùng cho các em học sinh luyện thi Qlympic toán quổc gia lớp 12, thi Olympìc toán quốc tế và dùng lảm tái liệu bồi dưỡng cho giáo viên. Chúng tôi tin rằng tẩt cả mọi người yêu thích bộ môn số học cũng có thể tim hiểu một phần hoặc toàn bộ các nội dung của cuốn sách này và những ai chưa yêu bộ môn số học hay sợ bô môn này thỉ sau khi đọc xong cuốn sách bạn sẽ suy nghĩ khác. Nãm 1993 sách đã đƯỢc Bộ Giáo dục & Đão tạo quy đinh là tài liệu chính thức dành cho các lớp THPT chuyên toán. Sách đã tái bản lần thứ 5.
  • Tòa nhà hĩnh học phảng và cản nhà hương Tác giả: Nguyễn Minh Hà Hình học Euclid phầng (hình học phầng) là một trong những ngành khoa học cổ xưa của nhân loại. Từ thế kỉ thứ VII đến thế kỉ thứ III trước Công nguyên các kiến thức hình học phầng dần dần được hệ thống. Vào thế ki thứ III trước Công nguyên (TCN), các kiến thức hình học phẳng được tổng kết xuất sắc trong tác phẩm Nguyên II của Euclìơ (330 - 275 TON). Theo Euclid, để định nghĩa một khái niệm hình học phẳng ta buộc phải dựa vào những khái niệm đã được định nghĩa trước đó. Như vậy phải có những khái niệm đầu tiên không định nghĩa mà mò tả, đó là những khái niệm cơ bản, Để chứng minh một định lí hình học phảng ta buộc phái dựa vào những định lí đã được chứng minh trước đó và HAY CẦN ĐỌC như vậy phải có những định lí đầu tién không chứng minh mà công nhận, đó là nhữhg tiên đế Hệ thống các khái niệm cơ bản và các tiên đề của Euclid được gọi là hệ tiên để Euclid. Với hệ tiên để Euclid, tòa nhà hình học phầng đã được xây dựng, Suốt thời gian dài, nếu không kể tới việc tìm cách chửng mình tiên đề 5 (1) 2 , vể cãn bản người ta tin rằng tòa nhà hình học phẳng đã hoàn Chĩnh. Mãl đến thế kỉ XIX, khoảng 2200 năm sau thời của Eudíd, cùng với việc giải quyết dứt điểm về sự khẳng định tiên đề 5* 2 *, vân đề cơ sở lôgic của hình học phẳng lạl được nghiên cứu rộng rải và sói noi, Kết quả là một hệ tiên đề mâi, được nhả toán học vĩ đại Hilbert (1862 - 1943) trình bày trong tác phẩm Cơ sỏ hình học. Với hệ tiên để Hilbert, tòa nhà hình học phẳng đã dược nâng cấp căn bàn và toàn diện, Có lẽ vì Euclid vả Hilbert đều quá lổi lạc nên cho đến bây giờ người ta vẫn cho rằng tòa nhà hỉnh học phầng đã hoàn chỉnh. Không biết từ bao giơ, trưâc hay sau thời của Hilbert, khái niệm góc lưọng giác và kèm theo nó cái đổng hố xuất hiện trong hình học phẳng. với sự xuất hiện của cál đồng hồ. tòa nhà hình học phẳng vãn chưa hoàn chỉnh. Không thể định nghĩa cái đồng hổ bằng các khái niệm cơ bản cùa hệ tiên đề Hilbert. Coi hình học phầng là đam mê lãn nhất của cuộc đời, TS. Nguyễn Minh Hà đã trăn trở với vấn đề trên trong nhiểu nãm. Bát đấu từ năm 2000, ỏng quyết tâm tim cách bỏ cái đóng hổ ra khỏi hỉnh học phẳng (cách nối của GS, TSKH. Nguyễn Vãn Khuê) và kết quả là bộ sách hai tập Hướng trong hình học phẳng và Hình học phẳng định hướng vừa ra đài. Vâi bộ sách này, tòa nhà hình học phảng có thêm một cãn nhà nhỏ, cãn nhà cỏ hướng. Nói cách khác, lí thuyết vẽ hương đã được xây dựng chặt chẽ trong hình học phẳng. Chú thích: (1) Tiên đề 5 của Euclid được phát biểu như sau: "Qua điểm A không nằm trẽn đường thẳng b có vá chỉ có duy nhất một đường thẳng song song với đường thẳng b". (2) Phải chăng tiên đề 5 là một định lí. Cảu hỏi trên đã làm đau đểu biết bao thế hệ các nhà toán học và cuối cùng đă được giải quyết gần như cùng một lúc bơi ba nhà toán học: Lobachevski (1792 - 1856), Gauss (1777- 1855), Bolyai (1802 - 1860). đặc biệt xuất sắc là Lobachevski. BBT NHỮNG CÔNG TRÌNH MỚI VŨ ĐÔ QUAN Ngày 3.1 .201 6 tuyến Quốc lộ 1 đoạn Bắc Ninh - Bắc Giang dài 20 km được nâng cấp lên tiêu chuẩn đường cao tốc. Từ nay xe từ Bắc Giang vể Hà Nội chạy chưa hết một giờ đồng hồ. (S) 0 o 51 km Hả Nội Bắc Ninh Bắc Giang Ngày 8.1 .2016 hẩm chui nút giao giữa đường vành đaì 3 và đường xuyên tâm (Nguyễn Trãi. Hà Nội) với tên gọi nút giao Thanh Xuân đã khảnh thành. Nút giao thông này có 4 tầng: hầm chui, đường mặt đất, đường trên cao và đường sắt trên cao, là nút giao hiện đại nhất nước ta hiện nay. Với việc hoàn thành nút giao này xe từ cửa ngõ Hà Đông (siêu thị Co.opmart) đến Ngâ tư Sở Hà Nội chạy thẳng khỏng có đèn xanh, đèn đỏ, vãi 4 km chỉ hẽt chưa đến 10 phút vào già cao điểm. Cùng ngày hầm chui nút giao giữa đường vành đai 3 và đường xuyên tâm (Thăng Long) với tèn gọi nút giao Trung Hòa cũng hoàn thành. • Ngày 9.1.2016 khánh thành cầu Tân Phong bắc qua sông Đào, TP. Nam Định trên quốc lộ 21 B lã đường vãnh dai S2. Đây là cây cầu thứ 3 qua sông Đào {sau cầu Đồ Quan và cầu Nam Định). Nam Định trở thành thành phô đầu tiên của Việt Nam có đường vành đai hình tròn từ 30.4.201 6 cùng 13 đường xuyên tâm (trong đó có 9 đường lớn và 4 đường đê) có hệ thống giao thông đường bộ, đưàng sắt, đưửng thủy thuận lợi. Ngày 15.1.2016 Quốc lộ 19 {Pleiku, Gia Lai - Quy Nhơn, Bình Định} hoàn thành việc nâng cấp. Đây là con đưàng quan trọng kết nối Tây Nguyên với ven biển Trung Trung Bộ. 175 knrt An Khẽ Quy Nhdn Ngày 17 1 2016 tuyến đưởng cầu Giấy - Nhật Tân đường cao ‘ốc đ sân bay Nội Bài, dài 6,4 km được thông tuyến, Từ nay theo tuyến đường này đường đi sán bay tỉnh từ cấu Giấy chỉ còn 23,5 km vốỉ tốc độ xe chạy 100 km/h. Cao Bằng Nội Bài Nhặt Tân Cẩu Giấy m Cùng ngày cẩu Hòa Trung (Năm Căn - Đất Mũi) cây cẩu cuối cùng trèn toàn tuyến 2000 km từ Cao Bằng tới mũi Cà Mau thông xe TP. Hõ Chí Minh Ngày 18,1 ,2016 cẩu vượt nút Long Biên trên tuyến QL5 cũ nối vỏi QL18 mới và QL2 (thông qua cẩu Đỏng Trù) khánh thành, Đây là cầu vượt bằng sắt lởn nhất Việt Nam. Cẩu hỏi kì này: Bạn hãy kể tên các cây cẩu bắc qua sông Hồng tại Hà Nội theo thứ tự từ Bắc xuống Nam. irarTO» Nhà hát Lớn Nhà hát Lớn Hà Nội là một trong những trung tâm văn hỏa của Hà Nội, nơi thường xuyên diễn ra các sự kiện vãn hóa nghệ thuật. Nhà hát được khởi công năm 1901, hoàn thành năm 1911 và hiện nay, nó đã tròn 1 05 tuểi. Nơi đây được trang trí khá cầu kì, với thanh đỡ uốn lượn, các cửa sổ được uốn vòm. Trên mái nhà là các cột lonic La Mã với mái chóp cong lợp ngói đá. Tất cả đểu mang dáng dấp của kiến trúc Châu Âu. Trước vẻ đẹp đẩy cuốn hút đó, ta có cảm giác nhưđang lạc giữa trài Táy vậy. Qua góc Hà Nội (TTT2 sô' 155) ■ máy của tác giả, nhà hát hiện lẻn dưới màu xanh cùa lá, của mây trời, màu vàng của hoa và thêm điểm xuyết màu đỏ cua lá cà. Vởi sự hải hòa về màu sắc, bố cục của cảnh vật ấy càng lảm cho cảnh vật trong lòng Hà Nội trở nên thơ mộng, thanh bình một cách lạ kì. Nhận xét. Chì có bạn Nguyễn MẼÊỄL Tuệ An, 7C, THCS Bạch Liêu, Yẽn Thành, Nghệ An có lời bình hay nhất được trao thưởng kì nảy. MAI VŨ N gày 29.5.1 832 trưửc khi lao vào cuộc đấu súng định mệnh, ông đê’ lại bứt thư cho Auguste Chevalier nói về mối quan hệ giữa lí thuyết nhóm với lởi giải của các đa thức bằng căn thức. Mãi đến nám 1843 Joseph Liouville mới xem bản thảo của ông vá kết luận Galois đã giải được bài toán do Niels Henrik Abel đưa ra. Công trinh sau đó được công bố toàn vãn trong Tạp chí toán lí thuyết và ứng dụng nắm 1846- Nhưng phải 20 năm sau nữa mủi có nhữhg người hiểu được những ý tưởng của ông. Giáo trình Đại số cao cấp của Serret và sau này cuốn Nghiẻn cứu các phẻp thế của Jordan đã giài thích cặn kê các khái niệm và vấn đề Galois đặt ra. Các khái niệm nhóm, vành, trường chưa ai đặt ra nên Galols phải trình bày khá dài và diễn giải, ồng chứng mình được điểu kiện cẩn và đủ để một phương trình đại số giải được bằng cãn thức. Ông đưa ra các khái niệm Nhóm con phán biệt, Phép đẳng cấu nhóm, Nhóm thương, ông đã là người giải quyết được vấn để tìm nghiệm cùa các phương trình đa thức bậc 5 trỏ lên bằng việc xây dựng lí thuyết Nhóm (group (nhóm) cũng là từ do õng dùng đầu tiên). Ngày nay lí thuyết này gọi là lí thuyết nhóm Galois. Nói đến đại số, không thể không kể đến Nhenxơ Henrich Aben (Niels Henrik Abel), nhà toán học Na Uy (1 802 - 1829). Vào năm 22 tuổi ông đã cho in tặp sách trong đó có định lí noi tiếng nói rằng phương trình bậc lớn hơn bốn dạng tổng quát không thể có nghiệm dưâi dạng cãn thức, òng còn tách ra được một lớp các phương trình bậc lân hơn bốn cố thể giải được dưới dạng cãn thức, sau này gọi là lỏp phương trình Aben. Giống như số phận các công trình của Galoĩs, công trinh của Aben khi gửi đến viện hàn làm khoa học Parí đã chẳng được al xem xét và xếp vảo hổ sơ lưu trữ. Aben đã sống cuộc sống đói rét nghèo và mắc bệnh lao khi bước sang tuổi 27. Sự thiên tài và trong sáng cùa tâm hón cống hiến cho khoa học của õng đả được người đời sau ghi nhớ. Thủ đồ Na Uy đã tạc tượng ông tại một quằng trường lớn. Ngày nay đã cỏ glảl thưởng toán học mang tên Aben. Galois và Aben dẽu chưa vượt qua được ĐẠI SỐ (Tiếp theo kì trước) BỈNH NAM HÀ nửa đời người thòng thường nhưng sự nghiệp và các cóng trình khoa học để lại sừng sững như những tháp cao trong lâu đài Đại số của vương quốc Toán học. Đại số hiện đại được bắt đầu từ thế kì 20 VỚI sự khỏi đẩu vể nghiên cứu tìm nghiệm cùa phương trình bậc cao mà Galois khởi đầu. Từ đó ra đời những chuyên ngành của Đại số như Lí thuyết nhóm, Lí thuyết vành, Lí thuyết trường. Đạì sổ giao hoán là một chuyên ngành hẹp mà đổi tượng nghiên cứu của nó là các vành trong đó phép nhân thốa mân luật giao hoán, đặc biệt là vành các đa thức nhiểu biến. Tuy nhiên giới hạn của Đại sô không thật rõ ràng. ■Đại số trừu tượng là một ngành của toán học vói đốì tượng nghiên cứu là các nhỏm, các vành, cấc trường, các dàn ... Đó là một khoa học nghiên cứu các phép toán trên các phẩn tử của một tập hợp tùy ỷ, suy rộng các phép cộng và phép nhân các số thông thường. Ngày nay để hiểu được đại sổ hiện đại cần phải có rất nhiểu kiến thức vế số học Tài liệu tham kháo: Ị1J Nguyễn Trường, Kể chuyện các nhà toàn học, NXB Văn hóa Thõng tin, 2011. [2] Đặng Huấn, Kế chuyện về những nhà toàn học, NXB Vãn học, 1997. [3] Vũ Kim Thủy, Li thuyết biểu diễn thứ cấp của càc hiữđun trẻn vành giao hoàn, Viện Toán học, 1993, [4] Nguyễn Đức Thuần, Sơ lược Lịch sử toán học, Trần Tất Thẳng dịch, NXB Khoa học và Kì thuật,
  • ANH PHÒ 31 Hỏi: Anh Phó thân mến í Em râ't muốn được cộng tác với TTT2 trong một số chuyên mục nhưng em chưa biết phải viết như thể nào. Em có cần phải viết mỗi bài và lời giải chi tiết trên từng tở giấy không ạ? Mật bạn ở ứng Hòa, Hà Nội Đáp: Mỗi bài trên một tờ Ghi họ tên đầy đủ Cả lớp trường váo nữa Quặn huyện hay tỉnh nào Rồi tất cả bỏ vào Một phong bì tất tạt Nếu giải bài kì trước Cần dán Phiếu dự thi Hây mạnh dạn lên đi Gửi ngay vê tờa soạn. Hỏi: Anh Phó ơi! Một số bạn chỉ được nêu tên trong một bài của mục Thi giải toán qua thư mà đã được thưởng. Còn em thì đã được khen như vậy vài lần nhưng lại không được thưởng. Thế là sao ạ? Một bạn giấu tên Đáp: Cò phần khen, phần thưởng Em cân đọc kĩ vào Xem tên được mức nào Nếu chỉ khen cùng tốt Được thưởng thì phẳi khao. Hỏi: Em xin hỏi trình tự để gửi bài tham dự các chuyên mục của TTT là như thế nào ạ? NGUYỄN THỊ TÚ ANH (6D, THCS Đặng Thai Mai, Vinh, Nghệ An) Đáp: Phải dê rõ cả họ tên Lởp trường huyện tỉnh ở trẽn mỗi bài Dán phiếu dự thi bên ngoài Phong bi để biếỉ là bài dự thi. Bạn hãy vào website: //olm.vn/hieu-sach-online để đọc tạp chí Toán Tuổi thơ bản diện tứ nhé. CÁC LỚP THCS CÁC LỚP 6 & 7 Bải 1(157). Nhà toán hoc De Morgan ( 1 806-1 871) khi được hoi tuổi đã trả lởi: Tôi X tuổi vào năm X 2 . Hoi năm X 2 đó ỏng bao nhiêu tuổi? vữ KIM THỦY Bài 2(1 57). Cho tam giác ABC với BÃC = 1 00° vả AB = AC. Trèn tia AB lấy điểm D sao cho AD - BC. Tính ÁDC. NGUYỄN BẢ ĐANG (Hà Nội) Bài 3(157). Giải hệ phương trình I X 3 + y 3 + 7(x + y) = 3(x 2 + xy + y 2 ) + 5 ■ Ỉ3 | Ỉ3 4 H + 1 \y + 1 Vx+ v 'y HOÀNG ĐỨC NGUYÊN (GV. trường THPT chuyên Đại học Sư phạm Hà Nội) Bài 4(157). Cho các sổ thực a, b, c vả d thỏa mãn: a > b > c > d; a + b + c + d = 9; a 2 + b 2 + c 2 4 d 2 = 21 . Chứng minh rằng ab - cd > 2. LÊ XUÂN ĐẠI (GV. THPtehạyền Vĩnh Phúc) Bài 5(157). Cho một bàng õ vuông kích thước 7x7 (gồm 49 ô vuông đơn vị). Đặt 22 đấu thủ vào bảng sao cho mỗi ô vuông đơn vị có không quá một đấu thủ. Hal đấu thủ được gọi lả tấn cóng lẫn nhau nểu họ cùng trên một hàng hoăc cùng trên một cột. Chứng minh rằng với mỏi cách đặt bất kì luỏn tổn tại ít nhất 4 đấu thủ không tấn công lấn nhau. HÀ VĂN NHÂN (GV. THCS Hoằng Xuân, Hoằhg Hóa, Thanh Hóa) Bài 5(157). Cho tam giác ABC nhọn, không cân tại A, nội tiếp đường tròn (O). Gọi M, N là hai điểm cố định thuộc cung nhả BC sao cho MN // BC và tia AM nằm giữa hai tia AB và AN. Gọi p là điểm nào đó trẽn đoạn thẩng AM. Đưàng thẳng đi qua p song song vãi BC cắt AC, AB lẩn lượt tại E, F. Đường tròn ngoại tiếp tam giác NEF cát đưàng tròn (O) tại Q khác N. Chứng minh rằng đường thẳng PQ luôn đi qua một điểm cố định khi điểm p di chuyển trên đoạn thẳng AM (P khác M). TRẤN QUÁNG HÙNG (GV. trường THPT chuyển Đại học Khoa học Tự nhiên Hà Nội) SOLUE UIA MAIL COMPETITION QUESTIOdS Transỉated by Nam Vũ Thành 1{157). Whẽn asked about his age, the mathematỉđan De Morgan (1800-1871) answered: |'m X years □ld ín the year X 2 . How old was he in the year X 2 ? 2(157). Given the triangle ABC with ZBAC = 100° and AB - AC. Let D be a point on the ray AB such that AD = BC. Find the measure of ZADC. U 3 +y 3 4 7(x + ỵ) = 3(x 2 + xy + y 2 ) + 5 3(157). Solve thefollowing sirnultaneous equations 3 x + 1 PHIẾU ĐÃNG Kí THAM Dự CUỘC THI GTQT NẦM HỌC 2015-2016 |y+i Vx+7Ỹ 4(157). Given the real numbers a, b, G, and d such that a > b > c > d, a í b ¥ ũ ¥ d = 9, and 3 2 \ b 2 + c 2 f <£ - 21 . Prove that ab - cd > 2. 5(157). Given a 7 X 7 square board which contairs 49 small squares. Place 22 players on the board such that each small square has at most one player. Two players are said to contend each other if they are either on the same column or on the sa me rank. Prove that for any arbitrary arrangement of the players, there exist at least 4 players who do not contend one another. 6(157). Given an acute triangle ABC which ỉs non-isosceles at A and inscribes a circle (O). Let M and N be two fi xed polnts on the minor arc BC such that MN iì BC and that the ray AM lies between the rays AB and AN. Let p be a point on the lỉne segment AM. The line passing through Pand parallel to BC intersects AC and AB at E and p, respectively. The circumcircle of the triangle NEF intersects the circle (O) at another point Q apart ỉrom the point
  • Prove that the line PQ always passes through a tixed point when p moves along the line segment AM (and p does not coincide with AJ). »1@g • Ngây 30.01. 201 6, Ths. Vũ Kim Thủy, Tổng biẽn tập Lạp chí Toán Tuổi thư và các cán bộ Tạp chí đã đến trao 1 1 suat quà cho các gia đình chính sách vả các hộ nghèo ờ phường Mộ Lao, quận Hã Đông, TP. Hã Nội. Tơi dự buổi Lễ trao quà có ông Nguyên Vãn Long, Bĩ thư Dảng ủy; õng Bạch Hồng Ilicu, Phó Chủ tịch UBNL) Phường; õng Bạch Hững Tiến, Chủ tịch Mặt trận tể quốc Việt Nam Phương Mộ Lao, cùng lãnh dạo các ban ngành, đoàn thc của phường và các gia đĩnh chính sách, các hộ nghèo. TbS. Vù Kim Thủy và ông Nguyền Văn Long trao quà cho các gia đình Gian hàng cùa Cóng ty cổ phần Ván hóa Giáo dục Long Minh • Tữ ngày 10.2 đốn ngày 15.2.2016, Phố sãch Xuân lan đầu tiên dã dưực tổ chữc Lại phố Lẻ Thạch, Hà Nội với hđn 20 gian hàng dưực tníng bày cõng phu. bắt mắt. Có nhíẻu loại sách được mang đến Hổi chợ như; sãch thiếu nhi, sách Văn học, sách Toán, sãch Kinh tế, sách Khoa học Kĩ thuật, ... Tạp chí Toán Tuổi thư dã tham gia glâl thiệu 10 dầu sách. • Ngày 4.3.2016, Tạp chí Toãn Tuổi thư dã gặp lành đạo Sd Giáo dục và Đào tạo Hòa Rình: ổng Đặng Quang Ngán, Phó Giám dốc sư Giáo đục và Dào tạo; õng tíùi Đức Ngọc, Trưởng phòng Giáo dục Tiểu học; ông Trương Trung Yên, chuyên viên 1'hõng Giảo dục Trung học. Trong buổi lãm việc Tạp chi đã gìởi thiệu về cuộc thi Câu lạc bộ Toán Tuổi lliơ toán quốc 2016 sẽ diễn ra vào tháng 6,2016 và cuộc thi AMC phối hợp VỚI AMT của Australla tố chức vào tháng 7.2016, Óng Dặng Quang Ngản đã cỏ các ý kiến ủng hộ các hoạt đũng cua Tạp chì vã cãc cuộc thi Tạp chi đang tể chửc. Cũng ngày Tạp chi đả gặp ông Nguyên Minh Tân, Giám dốc và ông Phạm Huy Đóng, Phó Giám dốc Cõng ty CP sách và TBTH Hòa Bình dể giới thiệu các ấn phẩm Toán Tuổi thơ đang phát hành. Phó Giâm dổc Sỗ GD - Đĩ Hóa Bình PV. Đảng Quang Ngàn và ThS. Vũ Kim Thúy Nơi Hoàng Thành xếp lớp các tầng vãn hóa các triều Lỷ, Trần, Lẽ. Hoàng Thành hỏm nay nằm giừa trái tim Thủ dô Hà Nội. Vẻ đẹp cổ kính ấy bừng sửc sổng mới vởi hoa sen ngãy mđi, vời đãn trẻ chủ nhân của tương lai. Kính dõ xưa, thủ d6 nay dẹp hảo sảng, vữa cổ kỉnh vừa trẻ trung. Bạn hãy viết bãi binh về bửc ảnh dẹp này. Chờ bải viết của bạn. VŨ SỜN NAM Ảnh: Phan Ngọc Quang CÁC HỌC SINH ĐƯỢC KHEN TRONG cuộc THI GIẢI TOÁN DÀNH CHO NỮ SINH Từ trá ị sang phát Đặng Thị Hoài Anh, Phan Huyẻn ngọc, riguyẻn Thj Thùy Trang. dÌÌHA 51 MCE [?57 2'nl^ frtiijỉn ỉỊiiVniị - ĩ'ÙịỉÌ ỉTf'fĩự Ịĩúì

    Công ty CF VPP Hồng Hà là nhà tài trợ cho 2 cuộc thiĩ và

    Giấy phép xuâ't bản: số 31/GP-BVHTT, cấp ngày 23/1/2003 của Bộ Văn hóa và Thông tin. Mã số: 8BTT157M16, In tại: Công ty cổ phần in Công Đoàn Việt Nam f 167 Tây Sơn, Đống Đa. Hả Nội. In xong vả nộp lưu chiểu tháng 03 năm 2016. NĂM THỨ MƯỜI BẢY ISSN 1859-2740 04/2016 Giá: lOOOOđ NHÃ XUĂT BAN GIÃO DỤC VIỆT NAM - BỘ GIÃO DỤC VÃ ĐÀO TẠO !0 NĂM CÁC LỘP TOÁN ĐẶC BIỆT ĐHSP VINH ẨU nam I HỌC BỔNG ASEAN te u Childrens Fun Maths tlournal NHÁ XUẤT BÀN GIÁO DỤC VIỆT NAM - BỘ GIÁO DỤC VÀ DÀO TẠO IB1SI Tổng biên tập: Th$. vũ K|M THÙY Thư kí tò a soạn : T rưỏng ba n biên íặ p: NGUYÊN NGỌC HÂN TRẤN THỊ KIM CƯƠNG CHỊU TRÁCH NHIỆM XUẤT BẢN Chù tịch HỘI dồng Thánh viên NXBGD Việt Nam; MẠC VÃN THIỆN ĩ ổng Giám đác NXBCỮ việt Nam; GS.1S.VỦ VẪN HÙNG Phó Tổng Giám đõ'c kiẽm Tổng biên lập NXBGO Việl Nam: Tỉ. PHAN XUÂN THÀNH TRONG SỐ NÀY NGND.VŨHỪU BÌNH TS- GIANG KHẮC BÌNH TS. TRẦN ĐÍNH CHÂU TS. VŨ ĐÌNH CHUẨN TS. NGUYỄN MINH ĐỨC ThS. NGUYỄN ANH DŨNG TS. NGUYÊN MINH HÀ PGS. TS. LÊ GUỐC HÁN PGS. TSKH. VŨ ĐÌNH HÒA TS. NGUYÊN €>ửc HOÀNG ThS. NGUYỄN vũ LOAN NGUYỀN ĐỨC TẤN PGS-TS-TÒN THĂN TRƯƠNG CÔNG THÀNH PHẠM VĂN TRỌNG ThS.HG QUANG VINH TÒA SOẠN Táng 5, số 3G1 đường Trưởng Chỉnh, quặn Thanh Xuản, Hã Nội Điện thoại (Tai): 04 356S2701 Điện sao (Fax): Ũ4. 35682702 Điện thư {Eniail); tữantuoithữ@vnn.vn Trang mạng (VVebSile). //www.loariluoitho.vn nm NGUYỄN VIẾT XUÃN 55/12 Trần Đinh Xu, p, cáu Kho, Q.1, TP, HCM ĐT: 08.66821199, DĐ. 0973 308199 Trị sự - Phát hành TRỊNH THỊ TUYẾT TRANG, VŨ ANH THƯ, NGUYỄN HUYỂN THANH Chế bản: Đỏ TRUNG KIẼN Mĩ thuật: TÚ An Dành cho học sinh lóp 6 & 7 Một sò dạng toán vể đa thức một biến Võ Xuân Minh Bạn đọc phát hiện Một bài toán cựctrị cố nhiều cách giải Nguyễn Duy Thái Đo trí thõng minh Số tiếp theo Nguyễn Đức Tấn Bạn muôn du học Dựthi học bổng Singapore Thủy Vũ Nhìn ra thế giói Đẽ chọn đội tuyển dựthi OlympicToán Quốc tế của Hổng Kông nãm2Q1Q (Vòng 1) Mai Vũ Com pa vui tính Không giải phương trình Phạm Tuấn Khải Phả án cùng thám tửsêlôccôc Món quà biến mất tê Ánh Tuyết Đến với tiếng Hán Bài 67. Tôitừthành phố Hổ Chí Minh tớì Nguyễn Vũ Loan Dành cho các nhà toán học nhỏ Bài toán dựng đường tròn Nguyễn Bá Đang Để thi các nước ÂMC 2Ũ1 5 - Upper Primary Dívision Australian school years 5 and 6 Đỗ Tmng Hiệu MỌT SO DẠNG TOAN VỀ DA THỨC MỘT BIẾN VÕ XUÂN MINH (GV. THCS Nguyễn Văn Trỗi, Cam Nghĩa, Cam Ranh, Khánh Hòa) Các dạng toàn về đa thức một biến khà phong phú. Sau đây là một sô' dạng toàn về đa thửc một biến thường gặp phù hợp với nội dụng kiến thức lởp 7,
  • Tinh giá trị của đa thức Ví dụ 1. a) Cho đa thức g{x) = 2x 2 + 3x - 9. Tính g(-3). b} Chứ f(x) = X 2 + bx + c- Biết f(2) = 7- Tính f(-1 ) + f{5). Lời giải, a) g(-3) = 2.(-3 ) 2 + 3. (-3) -9=0.
  • Ta có f(2) = 2 2 + 2b - c = 7 ò 2b 4 c = 3. f<-1> + f(5) = (1 - b + c) + {25 4 5b + c) = 26 + 2(2b c) = 26 + 2 3 = 32 = (a 4 b)[-b(a 4 b) - ab + b 2 ] = (a 4 b}.0 = 0. Vậy -a - b là một nghiệm của f(x).
  • Tìm nghiệm của da thức Ví dụ 4. a) Tim nghiệm của đa thức X 2 - 5x + 6 .
  • Tim một nghiệm của da thức A(x) - X 5 + ax 2 + 1 bx 4 c biết a - 2b + 4c = - 7 . 2 Lài giải, a) X 2 - 5x + 6 = 0 fc> X 2 - 2x - 3x 4 6 - 0 2 . Cộng, trừ, nhân đa thức VI dụ 2. Cho A = 2x a - 4x 4 3 và B = X 2 + 4x, Thực hiện phép tính A - 2B và A.B . LỜI giải. A - 2B - 2x 2 - 4x 4 3 - 2(x 2 4 4x) - -1 2x 4 3. A.B = (2x 2 - 4x + 3)(x 2 4 4x) = 2x 2 (x 2 + 4x)
  • 4x (x 2 + 4x) + 3(x 2 + 4x) = 2x 4 + 4x 3 - 1 3x 2 4 1 2x.
  • Chứng minh một sô' là nghiệm của da thức VI dụ 3. a) Cho f(x) - ax 2 + bx + c, chứng minh rằng nếu b = a + c thì f(x) có một nghiệm là - 1 .
  • Chứng minh rằng nếu X D là nghiệm của 1 f(x) = ax I b (a ỹt 0 , b 0 ) thì — - là nghiệm của 0 g(x} = bx -§- a.
  • Cho f(x) = X 3 + ax 2 - abx + ab 2 + b 3 . Chửng minh rằng -a - b là một nghiệm của f(x). Lài giải, a) f(-1) = a(-1 ) 2 4 b(-l) 4 õ-a-b 4 c -0 (vì b = a 4 c). Vậy X - -1 là nghiệm của f(x).
  • Ta có f(x ) = ax - b 55 0 = x ữ = a f 9 k -a + a = ũ. t Vậy — — là nghiệm của g(x). Xo
  • Ta có f(x) = x 2 (x 4 a) - abx 4 b 2 (a 4 b). => f(-a - b) - (-a - b) 2 (-b) - ab(-a - b) + b 2 (a 4 b)
  • -b(a 4 b) 2 4 ab(a 4 b) 4 b 2 (a 4 b) <=> x(x - 2} - 3{x - 2) - 0 <=> (X - 2}(x - 3) - 0 « x - 2 = 0 hoặc x-3 = 0<=;x = 2 hoặc X = 3. Vậy đa thức X 2 - 5x + 6 có các nghiệm là X = 2, X = 3.
  • Ta có 1 1 a - 2b + 4c = 4- a - 2b 4 4c - 4- = 0 2 2 í=> ị 1f 11 — a + — — l 2 ) 1 2 ) ■. 2, b + c=0=>A|-Ị|=0. i 2j Vậy là một nghiệm của A(x).
  • Chứng minh da thức không có nghiệm Ví dụ 5. Chứng minh rằng đa thức B{x) = -X 2 4 6 x -10 không có nghiệm. Lài giải. B(x) = -(x 2 - 6 x + 9) - 1 = -(x 2 - 3x - 3x
  • 9) - 1 = - [x(x - 3) - 3(x - 3)] - 1 = - (X - 3 ) 2 - 1 < 0 với mọi X. Vậy B(x) không có nghiệm.
  • Xác định da thức Ví dụ 6.1. Xác định đa thức bậc hai f(x) thỏa mãn f(-1) = 10, f( 0 ) = 5, f( 2 ) = 1 . Lòi giải. Vì đa thức cần tìm lè bậc 2 nên ta đặt f(x) = ax 2 4 bx 4 c (a 0). Theo bài ra ta có f{0) - c - 5. f(-1) - a - b 4 c - 10; f(2) - 4a 4 2b 4 c - 1. Suy ra được a = 1 , b = -4, c = 5. Vậy f(x) = X 2 - 4x 4 5. Ví dụ 6.2. Tìm đa thức g(x) với X ẽ N biết g(1) - 1 và g(x) = 2 g(x - 1 ) - 1 . Lời giải. Ta cố g(1) - 2g(0> - 1 => g(0) - 1. Giả sử g(n) = 1 thì g(n + 1) = 2g(n) - 1 = 2.1 - 1 = 1. Vậy theo nguyên lí quy nạp toán học g(x) = 1 vởi mọi XÊ N. Vi dụ 6.3. Xác định đa thức h{x) biết rằng vởi mọi X thì h(x+1) = 3x - 2. Lởi giải. Đặt y = X + 1 = X = y - 1 . Tử giả thiết suy ra h(y) = 3(y - 1 ) - 2 = 3y - 5 Vậy h(x) - 3x - 5.
  • Chứng minh tính chât của đa thức thỏa mán điều kiện cho trước Ví dụ 7.1. Chứhg minh rằng nếu t(x) - ax + b có hai nghiệm phân biệt X 1 và x 2 thì a - b - 0. Lời giải. Vì x 1( x 2 là nghiệm nên ax 1 + b = 0 và ax 2 + b = 0. Suy ra (ax 1 + b) - (ax 2 + b) = 0 => a(x 1 - X,) = 0 =» a - 0 (vì X 1 / x 2 ). Kết hợp với ax 1 + b - 0 =í b - 0. Ví dụ 7.2. Chứng minh rằng nếu đa thức p(x) - ax 4 b (a 4 0) có nghiệm thi p(x) - a(x - X.}. Lời giải. Vì X 1 là nghiệm của p(x) nên p(x.|) = ax 1 -I- b = 0. -K, ị-poo-.' + ỊÌ--c«-*o- a v a J
  • Tìm tham sô' thỏa mãn điểu kiện cho trưóc Ví dụ 8. Cho h(x} = X 2 - rnx + 3.
  • Tìm m để h(x) có nghiệm lả -1 .
  • Với m vừa tìm được, tìm nghiệm thứ hai khác -1 của h(x). Lời giải, a} Vì h(x) có nghiệm lả -1 nên ta có h(-1 ) = 0 1 + m + 3 = 0^m = -4.
  • Gọi a là nghiệm của h(x} (a -1) thì h(a) = 0 <=> a 2 + 4a + 3 = 0 <=> (a 2 + 3a} + (a + 3} = 0 « a(a + 3} + (a + 3) = 0 (a + 1 )(a + 3} = ũ => a + 3 = 0 (vì a -1) <=> a = - 3.
  • Chửng minh dẳng thức, bất đẳng thúc Ví dụ 9.1. Chúng minh rằng x 3 -8 = (x - 2)(x 2 4 2x 4 4). Lởi giải. Rút gọn vế phải của đẳng thức ta có (x - 2)(x 2 + 2x + 4) = x ă - 2x 2 + 2x 2 - 4x + 4x - 8 = X 3 - 3. Vi dụ 9.2. Chứng minh rằng 2x 2 - 8x 4 9 > 0 với mọi X. Lởi giải. 2x 2 - 8x 4 9 = 2(x 2 - 4x 4 4) 4 1 = 2(x 2 - 2x - 2x f 4} + 1 = 2[x(x - 2) - 2(x - 2}] + 1 = 2{x - 2) 2 4 1 > 1 > 0 với mọi X.
  • Tim giã trị nhỏ nhất lán nhất của da thức Ví dụ 1Ữ.1. Tìm giá trị nhả nhất của A = X 2 3x + 1.

    Lởi giải. Biến đổi A ta được

    ..2.3 3 9 5 A-x +7X--X + 7— 7 ” 2 2 4 4 3 y 2 , 5 4 3 3 Đẳng thức xảy ra khi x + ^- 0^x - Vậy MínA ■= khi X ™ — Ậ, 4 2 Vi dụ 10-2. Tim giá trị lớn nhất của B = -X 2 - 8x + 2. Lời giải. Biển đổi B ta đước B=-(x 2 -4x-4x+ 16) + 18
  • [x(x - 4) - 4(x - 4}] + 18 = - (x-4) z 4 IBS 18. Đẳng thức xảy ra khi X - 4 = 0 <=> X = 4. VậyMaxB -18 khi x - 4.
  • Tinh tổng các số tự nhiên Ví dụ 11. Cho đa thức f(x) = X 2 + k.
  • Tim đa thức f(x) - f(x - 1);
  • Áp dụng câu a hãy tính tdng s = 1 + 3 + 5 + ... + { 2 n - 1 ). Lởi giải, a) f(x) - f(x - 1) = (x 2 4 k) - [(x - I} 2 + k] = x z - (x - 1)(x - 1} = X 2 - x z 4 X + X - 1 = 2x - 1. o
  • Thay X lần lưọt bằng 1, 2, 3„.,n vào (} ta được

    f( 1 ) - f(0) = 1 ; f(2)-f(1) = 3; f(3) - f(2) = 5; f(n) - f(n - 1) = 2n -1 Cộng theo vế của các đẳng thức trèn ta được f(n) - f(0) - 1 4 3 4 5 4 ... 4 2n - 1 - s Suy ra s = (n 2 4 k) - (0 4 k} = n 2 . BẶT MUA TẠP CHÍ CẢ NĂM HỌC TẠI CẮC cơ sở Bơu ĐIỆN TRONG CẢ HƯỚC MA ẤN PHẨM: c 169.1 MỘT BÀỰOÁN cực TRỊ CÓ NHIỀU CÁCH GIẢI NGUYỀN DUY THÁI (GV. THCS Nam Hống , 7X Hống Lĩnh, Hà Tĩnh) Trong đề thi tuyển sinh vào íởp 10 nãm học 2015 - 2016 tỉnh Hà Tĩnh có bài toán tỉm cực trị, mà không có nhiều thí sinh giải được. Tôi xỉn nêu kĩ thuật phân tích để tìm nhiểu cách giải cho bài toán này. Bài toán. Cho các số thực a, b. c thỏa mãn Do đó F > -1 . a 2 4 C 2 = 0 =>a = c =:0 a 2 4 c 2 = 1 vâ F = 2ac. a 2 4 b 2 4 c 2 =1. Tìm giá trị nhỏ nhất cùa biểu thức F - ab ị bc f 2ac. Phân tích. Từ a 2 4 b 2 4 C 2 = 1 có -1 <a, b, e< 1. • Xét b = ±1 => b 2 = 1 =4> F = 0.

    Xét b = 0 Ta có 2ac > -{a 2 + c 2 ) a 2 4 c 2 1 =4 ac > — — = -— =? F > -1. 2 2 Dấu bầng xảy ra khi a - -c và a 2 + c 2 - 1 44 a = -c = ± ■ 2 Từ định hướng trên ta có các cách giải sau Cách 1. Vì a 2 + b 2 4 c 2 = 1 nên F = ab 4 bc + 2ac = a 2 4 b 2 4 c 2 - ab + bc 4 2ca - 1 7 » h (a + 2 ac + c ) + b(a + c)+-— 4
  • 3--1 4 (a ị ũ) 2 I b(a + c) 4'
  • 3.- — 1 4 a + c + - =4 F > -1 . 43- ■ 1>-1 Cách 3. Ta có (a + b 4 c ) 2 >0 =4 ab4bc4ca > - a 2 4b 2 4C 2 t — — T — = — T- (4) 2 2 _2 . _2 a +c b 2 -1 1 -^. (5) 2 {a 4 cr > 0 =4 ca > - 2 2 Cộng theo vế của (4) và (5) ta được F > -1 . Cách 4 . Ta có 1 = a 2 4 b 2 4 c 2 44 2 = 2(a 2 4 b 2 4 c 2 ) =4 2F 4 2 = 2{ab 4 bc 4 2ca) 4 2(a 2 4 b 2 4 c 2 }
  • (a 4 b I c ) 2 f (a I c ) 2 + b 2 > 0 =4 F >-1. Cách 5. Xét F 4 1 = ab 4 bc 4 2ac 4 a 2 4 b 2 4 c 2 44 F 4 1 = (a 4 c ) 2 4 b(a 4 c) 4 b 2 44 (a 4 c } 2 4 b(a 4 c) 4 b 2 - F - 1 = 0. ( 6 ) Ta coi ( 6 ) là phưong trình bậc hai có ẩn là ỉ = (a 4 c). Để phưdng trình ( 6 ) có nghiệm thì A = b 2 -4.1 (b 2 -F-1) >0 'F>-uị-b 2 >- 1 - 4 Cách 6 . Ta có F = ab 4 bc 4 2ac = a 2 4 b 2 4 c 2 4 ab 4 bc 4 2ca - 1 = [(a 2 4 2ca 4 c 2 ) 4 b(a 4 c) 4 b 2
  • 1 = (a 4 c ) 2 4 b(a 4 c) 4 b 2 - 1

    ■4Ĩ Dấu đẳng thức xảy ra khi b = 0 và a = -c = Vậy MinF = -1. 2 Cách 2. Ta cỏ (a 4 b 4 c ) 2 > 0. (1) (a 4 c ) 2 > 0. (2) và b 2 > 0. (3) Cộng vế theo vế (1 ), ( 2 ) và (3) ta có {a 4 h 4 c ) 2 4 (a 4 c ) 2 4 b 2 > 0 44 a 3 4 b 2 4 c 2 4 2(ab 4 bc 4 ca) 4 a 2 4 2ac 4 c 2 4b 2 > 0 44 2(a 2 4 b 2 4 c 2 ) 4 2(ab 4 bc 4 2ca) > 0 44 (a 2 4 b 2 4 c 2 ) 4 (ab 4 bc 4 2ca} > 0 44 (ab 4 bc 4 2ca) > -(a 2 - b 2 4 c 2 ) = -1 , = Ệ{a 4 b 4 c ) 2 4 - 7(3 - b + c ) 2 4 4 Trong các cách 2, 3 r 4, 5 và 6 dấu bằng xảy ra

    .72 chăng hạn khi b 0 , a - -c = 4 Y ■ Các bạn hãy tìm thêm các cách giải khác nhé. Bài tập vận dụng Bài 1. Cho các số thực a, b„ c thỏa màn a 2 4 b 2 4 c 2 = 1. Tìm giá tri lân nhất cùa biểu thức F = ab 4 bc 4 2ac. Bài 2. Cho các sô thực a, b, c thỏa mãn a 2 + b 2 c 2 = 2016. Tìm giá trị nhỏ nhất cũa biểu thức
  • p = ab 4 2bc 4 ac:
  • Q - 2ab 4 bc 4 ac;
  • R = 2ab - bc - ac.

    © ÌHĨ1TT ? SÔ TIẾP THEO Bài 1. Trong các hình sau. hình nào không phù hợp với các hình còn lại? Bài 2. Tìm số tiếp theo của dãy số 2; 3; 3; 63; 3968; ... NGUYỄN ĐỨC TẤN (TP. Hổ Chí Minh) 331® TIM Sỡ CỠN THIẼU (TTT2 số 156) Nhận xét. Ki này vần có hạn tìm sai quy luật. Quỹ luật. Bải 1. Dẳy số 2, 5, 11, 17,23, 29, ... là dãy các số nguyên tố HẾn tiếp mà chia cho 3 dư 2. (Dãy số này cớ tẻn là Dây sổ nguyên tố Eìsensteĩrí). Bài 2. ở mổì hình, số ở trong hình vuông bằng tổng các lạp phương của bỂn SỂ ở bốn đỉnh của hình vuông. Do vậy, số còn thiếu trong hình vuông cuối cùng là 2 3 T 1 + 2 3 + 1 o 3 = 201 6. . ứ? Xin trao thưỗng cho các bạn cố tời giải thích chính xãc, ngắn gọn: Nguyễn Hải Khoa, 6A, THCS Lý Tự Trọng. Bình Xuyên; Hoàng Thị Mỹ Duyên, 7D, THCS Vĩnh Tường, Vĩnh Tưởng, Vĩnh Phúc; Nghiêm Ngọc Phong, 8A1, THCS Yên Phong, Yên Phong, Bắc Ninh; Diêm Đàng Hoàng, 8A1, THCS Mai Son, thị trấn Hát Lót, Sơn La; Nguyễn Thị Hổng Minh, 7C. ~3[f51«|INy (TTTÍ ư 156) THỀ CỜ (Kì 79) 1.Sa8+ Ôxa8 2.ĩbc8 £ic7 3.&XC7 Ềa8 4.&xb6 và thắng [gỊSrrT Kì này có nhiều bạn tham gia giải thế cà, nhưtig hấu hết chưa chính xác. Chỉ có duy nhất bạn Hoàng Phúc, 7B, THCS Hoàng Xuân Hãn, Đức Thọ, Hà Tĩnh làm đúng, được nhận phẩn thưởng. LÊ THANH TÚ THCS Cao Xuân Huy, Diên Châu; Nguyên Đức Phủ, 8A1, THCS Nghi Hương, cửa Lò, Nghệ An. Các bạn và nhỏm bạn sau được tuyên dương: Lé Quang Dũng, 7A1 , THCS Yên Phong, Yên Phong, Bắc Ninh; Le Ánh Tuyết, 7E1 , THCS Vĩnh Tường, Vĩnh Tưởng, Vinh Phúc; Nhỏm bạn Như, Na, Mỹ Kim, Hoàng Anh, Dũng Vũ, 7D, THCS Xuân Diệu, thị trấn Nghèn, Can Lộc, Hà Tĩnh; Ngõ Thị Ngọc Hân, 7C, THCS Cao Xuân Huy, Diễn Châu; Nhóm bạn lôp 7C. THCS Bạch Liêu, Yẻn Thành, Nghệ An. NGUYỄN XUÂN BÌNH THÊ CỜ (Kì 81) Trẳng đi và chiểu hết sau 2 nước. a b c d e f g h a b c d e f g h LẺ THANH TÚ (Đại kiện tướng Quốc tế) ® Dự THI HỌC BỐNG SINGAPORE THỦY VŨ N gày 29.5.2016 Bộ Giáo dục Singapore và Bộ Giáo dục Việt Nam lại phối hỢp tổ chữc vòng Ị cuộc thi chọn học sinh đi học bằng học bổng ASEAN. Đây là lần thứ 21 cuộc thi được tổ chức tại Việt Nam dành cho học sinh đang học lớp 8, 9 giòi đều các môn Toán, Anh vàn và IQ. Bộ Giáo dục và Đào tạo phátíorm cho học sinh lửp 8 và 9 các tỉnh trong độ tuổi. Mối tỉnh có 2 thí sinh (giòi Toán, Khoa học và Anh) dự thi. Riêng Hà Nội số thí sinh đông hơn (Học bổng A*star thì do các trường của Singapore phátíorm trực tiếp về các trường của Việt Nam). Đây là học bổng chung cho học sinh giỏi của 9 nướcASEAN và Trung Quốc. Ấn Độ do Singapore cấp nhưng mang tên ASEAN, chỉ các nước mà học bổng hướng đến. Mặt bằng chất lượng là bằng nhau chứ không chia đều mỗi nưác được bao nhiêu suất. Do đó sỏ' lượng cũng biến động qua từng năm, tùy chất lượng học sinh. Tại Việt Nam, năm đầu tiên được 8 suất và nãm cao nhất có 25 học bổng được trao. Từ tháng 3 form được phát vể các trường. Cuối tháng 5 kì thi tổ chức tại Hã Nội và TP. Hổ Chí Minh vởi 3 bài thi viết. Khoảng 5 tuần sau là bàí phỏng vấn mỗi thi sinh 15 phút. Đầu tháng 9 kết quả được thông báo và học sinh sang từ tháng 10 hũặc 11 để năm học mới bắt đầu vào tháng 1 năm sau. Trong thỏi gian hai tháng đó học sinh được hướng dấn cách mở tài khoản, học truyền thổng và nội quy trường, học Tiếng Anh trực tiếp qua các tác phạm van học. Năm học Sec 3 bắt đẩu từ ngày 2.1 hảng nảm. Cứ sau 8 tuần lại có một kì nghỉ. Kì nghỉ sau 16 tuần học thì dài hơn và kỉ nghỉ cuối nãm kéo dài 2 tháng. Học sinh sã thi xong A level vào cuối nàm dương lịch. Sau đó học sinh nam ờ Singapore nhập ngũ 2 năm nghĩa vụ. Đại học bắt đầu nãm học từ tháng 7 năm sau đó. Như vậy do cẩu trúc năm học khác nhau về hìổc thời gian nên học sinh nước ta nếu sang học Sec 3 thì thưàng ra trường sau các bạn học ở cùng trong nước trước đày, Đó là hỏi về học bổng ASEAN. Còn học bổng A*Star thỉ việc tuyển tại Việt Nam chỉ tiến hành trong 3 ngày liền và thường biết kết quả sau không quá 10 ngày. Học bổng là 100% đủ chi cả học và án ở tại nước bạn theo lối sống sinh viên, tiết kiệm. Nỏ khoảng 25 000 đô la Singapore moi năm. Mỗi cấp học, học sinh nhận học bổng sẽ được 1 cặp vé khứ hồì. Lên đại học được cấp thêm tiền ở để sinh viên có thể thuê chỗ ở. Sinh viên đại học hoạt động xã hội tốt, có thảnh tích cao sẽ được ưu tièn ờ trong kí túc xá nhà trưỏng, Chi phí cho ăn hết khoảng 100 đô la (chi cho ãn trưa ở trường), đi lại 50 đô la (có hỗ trợ giá) và tiêu vặt chừng 100 đô la (sách vở, đồng phục, văn phòng phẩm Khi học phổ thông các học sinh ân ở cân tin nhà trường. Nhà trường quản khu nội trú theo kỉ luật rõ ràng. Lén đại học các sinh viên có thể thuê ra ở ngoài vả tự nấu cơm. Hết phể thông các em có thể đi nước khác mà không ràng buộc gì. Nhưng nếu học tiếp đại học tại Singapore thì hoc bổng thường có điều kiện ràng buộc và ra trướng làm cho các công ty cùa Singapore trong 3 nãm. Có thể vể làm tại Việt Nam nhưng là cho các công ty của Singapore. (Còn tiếp j cuộc THI GIẢI TOÁN DÀNH CHO NỮ SINH MÙA THỨ BA ử Đối tượng dự thí: Các bạn học sinh nữ đang học THCS. ứ Thể tbức gứi bài: Mỗi tháng, trên TT7 sẽ dăng 3 bài toán. Các bạn học sinh nữ có thể thom dự giãi từng bài trẽn mỗi số và trên nhiều số. Các bài giài dược viết bâng toy hoạc đánh máy, trình bàỵ liền nhau và dán một ánh thẻ 4 X 6 và ghi rõ: Tham gỉa cuộc thí gỉàl toán dành cho nữ sinh, họ và tên (chữ in hoa có dốul lớp , trường , huyện (quận); tỉnh (thành phối số điện thoại phụ huynh , gửi vê Tòa soạn tạp chí Toán Tuổi thơ, táng 5, số 361 , Trường chinh, Q. Thanh Xuân, Hà Nội.

    Cuộc thi diễn ra từ tháng 3.201 ó đến hết tháng 2.201 7.

    ỂP Thời gian nhộn bài: Trong vòng 30 ngày kể từ ngày Tạp chí phát hành (theo dốu bưư diện]. ứ Đáp án và danh sách các bạn được khen dược dăng ở sổ báo ra 2 tháng sau. tt Kết quà cuộc thi được dâng trên Tqp chí số tháng 4 nãm 201 7. ứ Trao thưởng dự kiễn vào tháng 6 nãm 2017. / DífflH SÁCH Hpc SIÍIH ĐO0Ĩ GIẢI cuộc THI GÈ TOÁn DHÍIH CHO nữ SIHH MÙA THỨ HAI Cuộc thi giải toán dành cho nữ sinh mùa thứ hai diễn ra từ tháng 3.2015 đến hết tháng 2.2016 đi đước sự hưởng ứng nhiệt tình của các thầy cô giáo và các em học sinh nữ. Có rất nhiều em học sinh nữtham gia giải bài, qua đó góp phần phát động phong trào học toán và giải toán trong cảc nữ sinh. Sau đày là danh sách các em học sinh đạt giải mùa thứ hai (Ban tổ chức dự kiến trao giải vào tháng 6 2016). Giải Nhất: Kim Thị Hồng Lĩnh' 9E1, THCS Vĩnh Tưởng, Vĩnh Tường, Vĩnh Phúc.

    Giải Nhì: Nguyễn Thảo Chì , 9A3, THCS

    Làm Thao, Lâm Thao, Phú Thọ; Trần Thị Thu Huyền, 9A3, THCS Lâm Thao, Lâm Thao, Phú Thọ; Bùi Thị Quỳnh, 8A3, THCS Hùng Vương, TX. Phủ Thọ, Phú Thọ; Đinh Thị Hổng Nhung , 9A1, THCS Lê Danh Phương, Hưng Hà, Thái Bình. Giải Ba: Nguyễn Thùy Dương, 8A3, THCS Hùng Vương, TX. Phú Thọ, Phú Thọ; Nguyễn Thu Hiền , 8A3, THCS Lâm Thao, Lâm Thao, Phú Thọ; Lê Nguyễn Quỳnh Trang , 9C, THCS Vãn Lang, TP. Việt Tri, Phú Thọ; Nguyễn Thị Thảo Vy, 9A, THCS Đặng Thai Mai, TP Vinh, Nghệ An; Trần Thị Diểm Quỳnh, 8G, THCS Đặng Thai Mai, TP. Vinh, Nghệ An.

    Giải Khuyến khích: Nguyễn Thị Ngọc

    Huyên, 9A, THCS Hùng Vương, TX Phú Thọ, Phú Thọ; Lê Thu Trang, 0D, THCS Lý Tự Trọng, Bình Xuyên, Vĩnh Phúc; Nguyễn Phương Thảo, 9A2, THCS Supe Lâm Thao, Lâm Thao, Phủ Thọ; Phan Huyền Ngạc, 9B, THCS Vĩnh Tường, Vĩnh Tường, Vĩnh Phúc; Nguyễn Thị Thủy Trang, 9A1, THCS Hồng Bàng, Hổng Bàng, Hải Phòng; Võ Nguyễn Đan Phương , 8A3, THCS Thị trấn Phù Mỹ, Phù Mỹ. Bình Định; Hổ Gia Bảo , 9A6, THCS Thốt Nốt, quận Thốt Nốt, TP. cần Thơ. Chúc mừng các bạn. © TTT ĐẼ CHỌN DŨI TUYẼN Dự THI OLYMPIC TOAN QUỐC TÍ CỦA HỔNG KÔNG NấM 2010 (VÒNG 1) Ngày thi 29.5.2010 MAI VŨ (Dịch vả giới thiệu ) Suu dếy chúng tỏi giời thiệu một sổ bài toán phù hợp vợi kiến thức ở THCS
  • Cho f(n) = 3n 2 - 3n + 1, Tìm bốn chữ sà tận cùng của f(1 } + f(2) + ... + f(20l 0).
  • Cho n là một số nguyên dương. Mếu n chia hè't cho 2010 và chỉ một trong các chữ sổ của n là số Chắn. Tìm giá trị nhò nhất có thể có của n.
  • Cho n là một số nguyên dương lớn hơn 1. Nếu n lớn hơn 1200 lần của một trong các thừa sô nguyên tố nào đó của n, tính giá trị nhỏ nhất có thể có của n.
  • Có 111 quả bóng trong một chiếc hộp, mỗi quả một màu: màu đỏ, màu xanh lá cây, màu trắng. Biết rằng nếu lấy ra 100 quả bóng ta cỏ thể đàm bảo có đủ bổn màu đó. Tìm số nguyên N nhỏ nhất sao cho nếu N số bóng được lấy ra, ta đảm bảo số bống được lấy ra có ít nhất ba màu khác nhau.
  • Cho các số nguyên dương a, b, c, d thỏa mãn a>b>c>d, a + b + c + d - 2010 và a 2 - b 2 + c 2
  • d 2 = 2010. Hồi có thể có bao nhiêu bộ số (a, b. c, d) khác nhau?
  • Cho đa thức bậc hai với hệ số thực sao cho p<11> = 181 và X 2 - 2x + 2 < p(x) < 2x 2 - 4x + 3 đúng vài bất kí số thực X. Tính p(21 ).
  • Tìm số nguyên lởn nhất không thể biểu diễn dưâi dạng tổng của một vài sô' trong các số sau: 135, 136, 137, .... 144 (mỗi số có thể xuất hiện nhiều lần trong tổng hoặc không xuất hiện một lẩn nào).
  • Cho n là một sổ nguyên dưong. Bằng việc bô đi 3 chữ số cuối của n, nó trở thành cãn bậc 3 của n. Tính giá trị có thể có của n.
  • Cho p. q là các số nguyên sao cho p + q =
  • Nếu tất cả các nghiệm của phương trình 10x 2 + px q = 0 là các số nguyên dương, tính tổng tất cả các giá trị có thể có của p.
  • Cho ABCD là hình vuồng với độ dài cạnh là 1- p và Q là 2 điểm trên mặt phầng sao cho Q là tâm đường tròn ngoại tiếp ABPC và D là tâm đường tròn ngoại tiếp APQA. Tính giá trị dương lớn nhất cỏ thể có của PQ 2 . Biểu diễn PQ 2 đó trong dạng a + Jb hoặc a - sfb, trong đó a, b là các số hữu tỉ.
  • Cho ABCD là mệt hình vuông với độ dài cạnh bằng 3. p là điểm trên mặt phẳng sao cho mồi góc ẤPB, BPC, CPÒ và DPA nhỏ nhất là 60°. Nếu mỗi vị trí có thể cùa p được tô màu đỏ. hãy tỉnh diện tích vùng màu đổ đó. không vượt quá 2010, tìm giá trị lớn nhất của f(m). 1 3. Cho một lục giác có các góc bằng nhau vối độ dài các cạnh là 6, 7, 8, 9, 10, 11 (không nhất thiết phải theo thử tự}. Nểu diện tích của lục giác là kv3, tính tổng tất cả các giá trị có thể cỏ của k.
  • Trén mặt phẳng có hai tam giác, độ dàì mỗi cạnh là 18, 24 và 30. Nếu hai tam giác không hoàn toàn trùng khít khi chồng lên nhau, nhưng có chung đường tròn ngoại tiếp và chưng đường tròn nội tiếp tam giác, tính diện tích phẩn chung của cả hai tam giác.
  • ABCD là hình chữ nhật, p và Q tương ứng là trung điểm của AB và CD. AQ và CP cắt nhau tại
  • Nểu AC = 6 và ẤRC=150°. tính diện tích ABCD.
  • Gọi p, q, r, s là bốn nghiệm của phương trình 2(10x + 13) 2 (5x + 8)(x + 1) = 1. Nếu pq + rs là số thực, hãy tính giá trị của sổ thực này. ..... , -Ja 2 + 3ab + b 2 -2a-2b + 4 ...
  • Neu a.b- — , tính ab + 4 ((-,,((20 10-2009). 2003}.. .2). 1).
  • Cho X là một số thực khác 0 sao cho \/x s + 20x - \^í 5 -20x_ Tính tích tất cả các giả trị có thể có cũa X. Kì 22 Hãy thay các chữ cáì bởi các chữ số. Các chữ khác nhau biểu diễn các chữ số khác nhau. Lởi giải cần có lập luận lôgic. SIX + SIX + SIX = NINE + NI NE TRƯƠNG CỐNG THÀNH (Sưti tầm) KÌ 21 (TTT2 số 156) Giả sử CÓ GRE E N GRANGE COLORS • Dế thấy 0>1vàdoG~R<8 + 9-17 nên c = 0 4 1. • Nếu E 4 G < 9 thì N + E có tận cùng là s = o (loại), vậy E + G > 10, íừ đó s 4 1 - o. • Chú ý: E 4 IM = s, E 4 G = 10 4 R. hoặc E + N = 10 + S<=>E + G = 9 + R (). Trong cả hai trường hợp thì G > R, • Sử dụng G + R = 10 4 o hoặc G 4 R = 9 + o 0*) để xét các trưởng hợp sau:
  • c - 9, o - 8, s - 7 thl G 4 R - 17 - 9 4 8 nên R = o (loại).
  • c = 8, o = 7, s = 6 thì G + R = 1 7 = 9 4 8 (R = C), hoặc G + R = 16 = 9 4 7 (R = 0).
  • c = 7, o = 6, s = 5 thì G 4 R = 16 = 9 4 7 (R = C) hoặc G 4 R = 15 = 9 4 6 = 8 4 7 (R = 0, hoặc R = C). Dưới đáy ta sẽ bỏ qua các sổ Q. R mà trùng với c hoặc o.
  • c = 6, o = 5, s = 4. Xét G + R = 15 = 8 + 7, theo () thì E = 9 và E N = 4 (loại),
  • c = 5, o = 4 , s = 3 thì G 4 R = 1 4 = 8 f 6 hoăc G + R = 13 = 746. Ta xét các trường hỌp
  • G = 8, R = 6, theo ( ) thì E = 8 và E 4 N = 3(loạì), hoặc E=7vàE + N = 14=»N = 7 = E (loại);
  • G = 7, R = 6, theo o thì E = 9 và E 4 N = 3 (loại), hoặc E=8vàE + N = 14=*N = 6 = R (loại).
  • c = 4, 0 = 3, s = 2. Theo n OỐ.E + N = 12 và E + G = 9 + R, mlG + R = 13 = 8 + 5s7 + 6 hoặc G + R=12 = 7 I 5. Ta xét các trường hỌp
  • G = 8, R = 5 thì á = 6 và N = 6 = E (loại);
  • G = 7, R = 6 thi E = 8 và N = 4 = c (loại);
  • G = 7, R = 5 thi E = 7 và N = 5 = R (loại).
  • c = 3, o = 2, s = 1- Theo () có E 4 N = 11 và E + G = 9 + R, mà G4R = 12 = 8 + 4 = 7 + 5 hoăcG + R=11 = 7 + 4 = 6 + 5. Ta xét các trường họp
  • G = e, R = 4 thl E = 5 và N = 6, dẳn đến 5 + A
  • L mà A, L chỉ là 7, 9 (loại);
  • G = 7, R = s thỉ E = 7 = G (loại);
  • G = 7, R = 4 thi E = 6 và N = 5, dẫn đến 5 + A = L mà A, L chỉ là 8, 9 (loại);
  • G = 6, R = 5 thì E = 8 và N = 3 = c (loại).
  • c = 2 t o = 1, s = 0. Theo () có E + N = 10 và E + G= 9 + R, mà G + R = 11 = 8 + 3 = 7 + 4 = 6 4 5 hoặc G4R = 10 = 7 + 3 = 6 I 4. Ta xét các trường họp
  • G = 8, R = 3 thì E = 4 và N =6, dẫn đến 4 4 A = L vứì A= 5, L = 9 (thỏa mãn);
  • G = 7, R = 4 thì È = 6vàN = 4 = R (loại);
  • G - 6, R = 5 thì E - 8 và N = 2 - c (loại);
  • G = 7, R = 3 thì E = 5 và N = 5 = E (loại);
  • G = 6, R = 4 thì E = 7 và N = 3, dẫn đến 5 + A L mà A, L chì là 5. 8, 9 (loại). Vậy bài toán có nghiệm duy nhất là 8 3 4 4 6
  • 1 3 56 8 4

    219130 , Nhận xét. Bạn Hoàng Thê" Sơn, 9A1 , sỄTiI- THCS Hống’ Bàng. Hồng Bàng, Hải Phòng xứng đảng nhận phần thướng vì đã giải đúng và lập luận đầy đủ. VIỆT HẢI ứãni dãn ụỉiỊ «Ẽ kl ínuc ĐỂ THI CHỌN HỌC SINH GIỎI MÔN TOÁN LỚP 7 TỈNH BẮC GIANG Năm học 2012-2013 (Để đăng trên TTT2 số 156) Cãu 1. '15 A 1 V ị 18

    i + J_ì , 1ữ o 1 H o 1 1,12' 12 12 1
  • p= |x- 20121 + |x- 2013|. • Nếu X = 2012 hoăc X = 2013 thì p = 1. • Nếu X > 2013 thì p = |x - 20121 + |x - 2013| 1 + |x -20131 > 1. • Nếu X < 2012 thì p = |x - 20121 + |x - 201 3| |x- 20121 + 1 >1. Do vậy Min p = 1 khi X = 2012 hoặc X = 2013. Câu 2.
  • Ta có 2 x+ 2 .3 x+1 ,5 x = 1 0300 2 ít .2 2 .3*3.5 3< = 10800 (2.3.S) X = 900 <= 30 x = 30 2 « X = 2. Vậy X = 2.
  • Gọi số viên bi của An, Bình, Cưỡng lần lượt là a, h, c (a, b, c t N). Theo bài ra ta có „ a b a b b c b c a + b + c - 74; — » — =} — -T~. — tÍt. 5 6 10 12 4 5 12 15 Từ đó ta có a b c a + b + c 74 2 ĩõ 12 ĩ? 10 + 12 + 15 ' 37 => 3 = 20; b- 24; c = 30. Vậy số viên bi của An, Bình, cường lần lượt có là 20; 24 và 30. Câu 3. 1 ) Vì p là sô’ nguyên tô’ lớn hơn 3 nền p có dạng p = 3k± 1 (ke N, k > 1). • Với p = 3k + 1 suy ra p 2 + 2012 = {3k + 1) 2 + 2012 = 9k 2 + 6k + 2013 =i(p 2 + 2012) : 3. • Với p = 3k - 1 suy ra p 2 + 2012 = (3k - I) 2 + 201 2 - 9k 2 - 6k + 201 3 => (p 2 + 2012) : 3. Vậy p 2 + 201 2 là hợp số.
  • Vì n là số có haì chữ số nèn 9 < n < 100 = 18 < 2n < 200. Mặt khâc 2n lả số chính phương chẩn nên 2n có thể nhận các giã trị; 36; 64; 100; 144; 196. Bằng cách thử thấy chỉ có vởi 2n = 64 n + 4 = 36 lè số chính phương. Vậy n - 32 là số cẩn tim. Câu 4. B H M c F N
  • Vì IẦB là góc ngoài của tam giác AHB nên IẤB = ÂBH + ÁHB = ÃBH + 90°. Lại có ẺBC = ẺBẦ , ABC = ẤBC + 90° □o vậy IẦB = ẾBC. Xét AABI và ABEC có AI = BC, IÃB = EBÒ, BE = BA (vì aABE vuông cân) Do đó AABI - ABEC (c.g.c). Vì AABI - ABEC nên ta có ÃĨB = BCE. Suy ra IBH + BCE = IBH-ỂỈH = 90°. Do vậy ta có CE vuông góc vối BI.
  • Ta có DM ± DN (tính chất đường phàn giác} Gọi F là trung điểm của MN. khi đó FM - FD - FN. Vì AFDM cân nên FDM -FMD. FMD - MBD + BDM =MBD +MDC. Lại có CDF + MDC - MDF - FMD. Suy ra MBD = CDF (1) Ta cỏ MCD = CDF í CFD. (2) Vì tam giấc ABC cân tại A nên MCD = 2MBD. (3) Từ (1), (2} và (3), suy ra MBD = C?D hay ABDF cân tại D. Vậy DB = DF=-JmN. ĐỂ THI TUYỂN SINH LỚP 10 TRUNG HỌC PHỔ THÔNG TP. Hồ CHÍ MINH Năm học: 2015 - 2016 Thòi gian tàm bài: 1 Sữ phút (không kể thời gian giao đề) Câu 1. ( 1,5 điểm) Cho hai số thực a, b thỏa mân điều kiện ab = Ị a + b ?fc 0. Tính giá trị của biểu thức r 1 f 1 I 1 V 3 f J_ + J_v 6 í 1 I 1 1 (a + b) 3 { a 3 + b 3 Ị + (a + b) 4 { a 2 + b 2 ) + (a b) 5 { a + b } Câu 2. (2,5 điểm)
  • Giải phương trình 2x 2 + X + 3 = 3 x-v/x + 3.
  • Chứng minh rằng abc(a 3 - b 3 )(b 3 - c 3 )(c 3 - a 3 ) : 7 vãi mọi số nguyên a, b, c. Câu 3. (2 điếm) Cho hình bình hành ABCD. Đương thẳng qua c vuông góc vãi CD cắt đường thẳng qua Avà vuông góc vơi BD tại F. Đương thẳng qua B vuông góc vội AB cắt dương trung trực của AC tại E- KE Hai đường thẳng BC và EF cắt nhau tại K. Tính ti sô 3 a g Cảu 4. (1 điểm) Cho hai số dương a, b thỏa mãn điéu kiện a + b< 1. Chửng minh rằng a z -r ■ 4a b 4 Câu 5. (2 điềm) Cho tam giác ABC có ba góc nhọn nội tiếp đường tròn {O). Gọi M là trung điểm của cạnh BC và N là điểm đối xứng của M qua o. Đương thằng qua A vuông góc vâi AN cắt đương thẳng qua B vuông gỏc với BC tại D. Kẻ đương kính AE. Chứng minh rằng
  • BA.BC = 2BO.BE;
  • CD đi qua trung điểm của đường cao AH của tam giác ABC. Câu 6. (1 điểm) Mười vận động viên tham gia cuộc thi đấu quần vợt. cứ hai người trong họ chơi với nhau đúng một trận. Người thứ nhất thắng X 1 trận vả thua y 1 trận, ngươi thứ hai thắng x 2 trận và thua y 2 trận, ... , người thứ 10 thắng x 10 trận và thua y 10 trận. Biết rằng trong một trận đấu quần vợt không có kết quả hòa. Chứng minh rằng + x| + ... + xfo =yf + y| +... -VrO’ Câu 5. Ta có p = 1 1 1007 100B 1 1
  • + - 2012 2013 1 1 1 •'.11 = 1 + 4 + 4 +—+ . . ! 2 3 1006 1007 1003
  • ...
  • 1 2012 2013 1 w, 1 1 1 1 + —T — + ...— 2 3 2006 ■ 11 = 1 1 + 4+4 2 3 1 7 -ị — “ + ... + 1 1 1 2012 2013 1006 + 1007 loos" 1 "”' 4111 1
  • 2] — + — + — + .,,+

    246 2012 1 1 -1-1 + -1— 1 +

    1 2 + 3 4 + "‘ 2012 2013 Dovậy(S-P) 2013 = 0. = s. Giải toán quã thu: Bải 1(150). Cho 2015 số nguyên dương a T , a ? . a 3 . .... a 2ữ16 thỏa mãn J ; J_ I J_ I I 1 a 1 ã 2 ã 3 a 2016 = 300. Chứng minh rằng tồn tại ít nhất hai số trong 2016 số đã cho bằng nhau. Lài giải. Giả sử trong 2016 sổ đã cho khủng có hai số nảo bằng nhau, không mất tính tổng quát ta giẻ sử a 1 < a 2 < a 3 < . . . < a 2ũ16 . Vì a r a 2 , a 2 a 2ũ16 đều là số nguyên dương nên ta suy ra a 1 > 1; a 2 > 2; a 3 > 3 a 20i6 -
  • Suy ra 1,1,1 , 1 .1.1 31 a 2 33 32016 2 3 4 { 1 iVfi,i,i, 1 Y 1^2 3j 1^4 5 6 7) 1 1 1 1 1024^1025 + 1026 + '" + 2016; < 1 * + TT- 2Z + 4- 2 3 + ... + -?ír.2 10 = 1 1 < 300 ọ r>2 «3 Pj1 0 2016 Mâu thuẫn với giả thiết. Dũ đó điểu giẳ sử là sai. Vậy trong 2016 sứ đã cho phải có ít nhất 2 số bằng nhau Nhận xét. Bài toán khá đẹp. phương pháp giải cũng không xa lạ vối học sinh giỏi toán nên có khả nhiều em giải và giải đúng. Tuy nhiên nhiều em còn làm vắn tắt hoặc đánh giá chưa khéo. Xin kê’ tên một sổ bạn có lởi giải tốt: Nguyễn Đữc Hiếu , 7C10, Vú Minh Bảo Khành, 7C1 0, Lẽ Quang Huy , 7C10, THCS Trần Phú, Lê Chân. Hải Phỏng; Ngõ Đứt Hoàng , Trển Nhật Hoa, Trần Phương Thao, Nguyễn Thị Thùy Trang, 6A, Thiều Thị Hạnh Nguyên, Lê Thị Phương Linh , 6B, THCS Xuân Diệu, Can Lộc, Hà Tĩnh; Nguyễn Hồng Khánh Lầm, 6L, THCS Hà Hụy Tập, TP. Vinh, Nghệ An. PHÙNG KIM DUNG Bải 2{1 56). Cho tam giác ABC cỗ Â - 40° , c - 30° Trên cạnh AC lấy điểm D Sáo cho CD = AB. Tính SỐ đo ÃBÒ. Lài giải. F A D c Trên nửa mặt phẳng bờ AC chứa B dựng tam giác đều AEC. Xét hai tam giác BAC và BEC có BC chung, EC = AC, BCA = BCE (= 30°) nên ABAC = aBEC (c.g.c) m AB = EB và ẤBC = ÉBC = 110° (theo giả thiết). Từ đó ÁBE = 360° -1 10° -1 10° -140° => BẦE = BẼA = 20°. Trén tia đối của tia AC lấy điểm F sao cho AF = AB. Xét hai tam giác càn AFB và BAE cố AB chung, ẼÂB = ÁBÉ = 140°, FA = EB AAFB = ABEA (c.g.c) => FB = EA = AC = F D Từ đó AFBD cân tại F, dẫn đển p-gp 1S0° -ẤFB 180 ũ -20° SQO 2 2 Ta cỏ ẤBÒ = FBÒ -FBẢ =30“ -20“ =60° Nhận xét. Xin nèu tèn một số bạn có lòi giải gọn hơn cả: Nguyễn Đức Tân, Tạ Hoàng Hải, Nguyễn Quốc Thứ, Nguyễn Trung Hiếu, Phạm Thùy Linh, Nguyễn Quang Huy, Vũ Ngọc Ảnh, Ngô Bình Minh , Triệu Hổng Ngọc , Nguyễn Thu Hương, Trần Tiến Đạt, Cao Bức Học, Nguyễn Việt Thu, 7A3, THCS Làm Thao, Lâm Thao, Phú Thọ; Lê Quang Huy, Nguyễn Đức Hiểu, Nguyễn Duy Phủc, 7C1 0, THCS Tran Phũ, Lê Chản, Hải Phòng: Nguyễn Tiến Phong , 7A1, THCS Yên Phong, Yên Phong, Bắc Ninh: Lê Xuân Hoàng, 6A, THCS Đặng Thai Mai, TP. Vinh, Nghệ An. HỔ quang vinh Bai 3(156). Cho các số nguyên dương a. b thủa min ab 4 1 là số chinh phương. Ghứbg minh rằng tổn tại số nguyên dương c sao cho ac 4 1 vả bc 4 1 đều là số chính phương. Lời giải. Giả sừ ab + 1 = n 2 (ne N). Chọn c = a + b + 2n. Ta có ac + 1 = a(a + b 4- 2n) + 1 = a 2 + 2na + ab + 1 = a 2 + 2na tn 2 -(a 4 li) 2 . bc + 1 = b(a f b ị 2n) + 1 = b 2 + 2nb + ab + 1 = b 2 4- 2nb + n 2 = (b + rt} 2 Vây ac + 1 và bc + 1 đều là số chính phương. Nhận xét. Có nhiều bạn hiểu để không đúng. Các bạn chọn c = b thỉ ac + 1 là số chính phương và chọn c = a thì bc 4 1 là số chính phương. Như vậy không đúng yêu cầu bài toán. Lưu ý rằng phải chỉ ra một giá trị cùa c, sao cho cả hai sổ ac- 1 và bc + 1 đều là số chính phương. Cách chọn c như trong bài giải có thể không phải là duy nhất. Các bạn sau đày cỏ bái glảl tốt: Đỗ Thúy Hồng, SAI, THCS Yên Phong, Yên Phong, Bắc Ninh; Trần Đình Hoàng, 6C, THCS Hoàng Xuân Hãn, Đức Thọ, Hà Tĩnh. NGUYỄN AIMH DŨNG Bài 4(156). Cho các số dương a. b, c thoa mãn abc = 1. Chứng minh rằng: a b c f- + - 4 — + b c a 3 b a — I — t {a c c 1 1 lì, !- + - + - . a b cj Lời giải. Áp dụng bất đẳng thức AM-GM ta có ,ặ=T(D c‘ 1 c 3 c Chứng minh tương tự ta cũng có
  • c. a 3 . b . b c 3

    b b c b a a b a f 4^ £33^ = 3?! c a Cộng theo vể của (1), (2), (3) ta được Jb a cV 1 1 ì') ... 1 ã + c + bj" l.ã*b + cj' Mặt khác, do abe = 1 nèn theo bất dẳng thức AM-GM a b c 1 1 1 ta có 7- + —+—+ —+—+— b c a a b c = í^+ab'| + | — + bc l + í — + ca ụ !V ) 1« 2 2a+2bt2c. (5) Tử (4} và (5) suy ra đpcm. Đầng thức xảy ra khi a - b - c = 1 . Nhận xét. Có nhiều bạn tham gia giải bài và có lời giải đủng. Các bạn sau đây có lời giải đúng và ngắn gọn: Lé Ngọc Hoa, Chu Vãn Việt, 8E1, Lê Anh Dũng, Kim Thị Hổng Lĩnh, Nguyễn Hoải Phương, Phùng Thị Xuân Thuỷ, 9E1, Nguyền Cõng Kiên, 9B, THCS Vĩnh Tường, vĩnh Tường, Nguyễn Kim Ngân, 9A1 r THCS và THPT Hai Bà Trưng, TX. Phúc Yên, Vĩnh Phúc; Đặng Thị Hoài Anh, 9B, THCS Nguyễn Thượng Hiển, ứng Hoà, Hà Nội; Trần Quốc Lập, Trần Thị Thu Huyền , 9A3, THCS Lâm Thao, Lâm Thao, Nguyễn Sơn Lâm, 9A4, THCS Giấy Phong Châu, Phù Ninh. Phạm Trần Khánh Linh, 8A, THCS Hùng VƯdng, Phú Thọ, Phú Thọ. CAO VĂN DŨNG Bài 5(156). Cố thể xếp 9 hình vuông gồm 4 0 vuông nhả {hình 1) và 7 hình thước thợ gồm 4 ô vuông nhẻ (hình 2) để phủ kin bàn cở s X 3 ỏ vuông nhả hay không? Ta tô màu các cột trắng, đen xen kẽ nhau như sau: các cột 1, 3, 5, 7 tô trắng; các cột 2, 4, 6, 8 tõ đen, Mỗi hình vuông kích thước 2x2 (hình 1) luôn phủ kín 4 ò vuông kích thước 1x1, trong đó cỏ 2 ô đen và 2 ô trắng. Mồi hình thưửc thợ (hình 2) luôn phủ kín 4 & vuông kích thưóc 1x1, trong đó có 3 ôđen và 1 ỏ trắng, hoặc 1 ô đen và 3 ô tráng, Như vậy, 9 hình vuông kích thưởc 2x2 {hình 1) và 7 hình thước thợ (hình 2) luôn phủ kín một sổ lẻ ô đen kích thước 1 X 1. Mà với cách tô màư như trẽn thl có có 32 ô vuông kích thưửc 1 X 1 được tó đen là một số chẵn. Vậy ta không thể dùng các hình đã cho để phủ kín bàn cà 8 X 8. Nhận xét. Hầu hết các bạn gửi bài về tỏa soạn giải đúng. Các bạn sau đây có lời giải tốt: Lê Xuân Hoàng, 6A, THCS Đặng Thai Mai, TP. Vinh, Nghệ An; Nguyễn Phương Anh , 9H, THCS cầu Giấy, Cầu Giấy, Hả Nội; Đỗ Thúy Hổng, SAI , THCS Yén Phong, Yên Phong, Bấc Ninh; /Vgụylrt Vẩn Huấn, Nguyễn Công Kiên, 9B; Nguyễn Hoài Phương, Kim Thị Hông Lĩnh , 9E1, THCS vĩnh Tường, Vĩnh Tường, Vĩnh Phúc. TRINH HOÀI DƯƠNG Bài 6(156). Chù tam giác ABC (Â > 90 c ) nội tiếp đưàng tròn (O). H là trực tám của tam giác ABC. M là điém chuyển động trên cung nhỏ BC, Gọi D là giao điểm của BM và CH, E lả giao điểm của CM và BH. Chứng minh rằng trung điểm của đoạn thảng DE nằm trẽn một đường thẳng cố định. Lồi giải. Gọi p, Q theo thứ tự là giao điểm của BH và CA. của CH và BA. Lấy K thuộc PQ sao cho DK // PE. Vì DK // PE nén. theo định lí Thales ta cỗ DK DQ Vì H là trực tâm của AABC và M thuộc BC của đường tròn (O) nên Lỗ Quang Huy , Nguyễn Đúc Hiếu .

    Jằẩ NGtu Nguyễn Duy Phúc, 7C10, THCS Trần Phú, Lê Chân. Hài Phàng; Trần Đinh Hoàng, 6C, THCS Hoàng Xuân Hãn, Đức Thọ, Hả Tĩnh; Lẽ Xuân Hoàng, 6A, TIHCS Đăng Thai Mai, TP. Vinh, Nghệ An; Trần Quốc Lập, Trần Thị Thu Huyền. 9A3, THCS Lâm Thao, ÒP'E= CPH = 90° =BQH =BQD; PCE -ACM- ÁBM - QB D. Do đó ACPH co ABQH; aCPE co aBQD. (2) Từ (1) và (2) suy ra DK DK DQ HP BQ CP BQ 1 PẼ DQ PẼ ĨHQ CP BQ CP Điều đó cỏ nghĩa là DK - PE. Vậy PEKD là hình bĩnh hành. Gọi I là trung điểm của DE thì I thuộc PK, mà K thuộc PQ, suy ra I thuộc đường thẳng cố định PQ. Nhện xét. Các bạn sau có lài giải đúng: Trần Quoc Lập, Trần Thị Thu Huyền, 9A3, THCS Lâm Thao, Lâm Thao, Phủ Thọ; Lê Ngọc Hoa, 8E1, THCS Vĩnh Tường, Vinh Phúc; Nguyễn Minh Nghĩa, 9B, THCS Nguyễn Thượng Hiễn, ứng Hoà, Ha Nội. NGUYỄN MINH HÀ Thi giải toán qua thư Lâm Thao, Phú Thọ; Đỗ Thúy Hóng, SAI, THCS Yên Phong, Yen Phong. Bắc Ninh; Lè Ngọc Hoa, SẸ1; Kim Thị Hổng Lĩnh, Nguyễn Hoài Phương, 9E1; Nguyễn Công Kiên , 9B, THCS Vĩnh Tưởng, Vĩnh Tường, Vĩnh Phúc; Nguyễn Minh Nghĩa, 9B, THCS Nguyễn Thượng Hiển, ứng Hòa, Hà Nội. Từ sổ tháng 9 nám 2015, Công ty cổ phẩn Dịch vụ Giảo dục Việt Nam sẽ tặng các khóa học trực tuyến trên web$ite: hocmaìvn cho các bạn học sinh được thưỏng trong các chuyên mục vá các bạn học sinh được khen trong chuyên mục Kết quả thi giải toán qua thư. Các bạn học sinh sau khi nhận được mã cung cả'p thì đãng kí tại địa chỉ: thcs.hocmai. ựn/toantuoìtho (Xin liên hệ SĐT 0966464644 để được giải đáp}. IMUĨ' KHÔNG GIẢI PHƯƠNG TRÌNH Bài toán. Cho hai phương trình X 2 +- 201 5x - 201 6 - 0 và y 2 + 201 5y- 2016 = 0. Không giải phương trình có cách nào tính được X + y, X - y hay không? Biết rằng X > y PHẠM TUẤN KHẢI (Hả Nội) CHIA TỈ LỆ ĐOẠN TRUNG TUYẾN (TTT2 số 156) Giả sử AB < AC, AB = rriAM và AC = nAN. Từ B kẻ BP /ỉ MN và cắt AD ở điểm p. Từ c kẻ CQ // MN và cắt AD ở điểm ũ. Ta chứng minh được ABDP = ACDQ (g.c.g) nên DP = DQ. Do BP ií ME nên r - -7-77 - m. AE AM Vì CQ // NE nên = n. AE AN Từ đó 2 AD - AP + PD + AQ - DQ = AP - AQ = mAE I nAE = (m ị n)AE. Suy ra AD m + n ÃẼ = 2
  • VớiAB SAM và AC = ^AN thì = 2 AE 4
  • Với AB = SAM và - Ị- thì ^4“ " suy ra AE 3 2 3 „ 5 . . AN n » — tức lá — — 3 AC 3 5' Nhận xét. Một sô bạn giải bài này bằng cách tính tỉ số diện tích các tam giác. Phẩn thưởng kì này dành cho các bạn: Bùi Xuân Dưỡng , 8A1, THCS Yên Phong, Yên Phong. Bẳc Ninh; Lẻ Ngọc Hoa, 8E1, Phùng Thị Xuân Thủy , 9E1, THCS Vinh Tường, Vĩnh Tường, Vĩnh Phúc; Nguyễn Minh Nghĩa, 9B, THCS Nguyên Thượng Hiền, ứng Hòa, Ha Nội. ANH COMPA Rh Ọnlin e Math 'Ltìfj rryơy iniỉìiỊ CKÌ cỉrti.m iỉi hựrt 'ỉĩ.íin iịiói ỉicn tít ficí tạn iíi Cữc tipn học dính (ErhrtiTp mọi rrtítrt Tử gíiòc 1 r í i -I • V V w rj pr ■ « ’ F r í r J 1 Taa jar> r a « à chù một tập đoàn lân, công việc hết sửc bận rộn nên hàng ngày ông Ninh thường vể nhả vào khoảng 9 giờ tối. Hôm đó, do kế hoạch tiếp khách thay đổi nên ỏng Ninh về sớm hdn thường lệ. Vào tỏi nhà là ỏng lẻn phòng riẽng luôn. Chưa kịp thay quẩn áo ông đâ hăm hở bước tối chiếc tù đặt ở góc phòng. Vửa mới mua một vièn kim cương quý giá nên òng Ninh rất háo hức ngắm nghía. Tuy nhiên, vừa mỏ ngăn kẻo tủ, mặt ông Ninh đã biến sắc. Viên kim cương không còn â đỏ nữa, Ông lục tung cả tủ lên cũng không thấy đâu cả. Là người điềm đạm, kín đáo nên óng Ninh không vội làm to chuyện. Òng lặng lẽ gọi cho thám tử sẻlôccôc, nhờ giúp đỡ. Một lúc sau, thám tử đã có mặt tại nhà ông Ninh.
  • Tôi vừa bi mật mua viên kim cương để tặng vợ nhân ngày sinh nhật cùa bà ấy. Không ai biết việc này cả- Cũng không ai biết là tôi để nó trong tủ- Thê mà nó lại biên mất.
  • Òng để viên kim cương vào tủ từ lúc nào? -Tối qua. Hóm qua về muộn quả nên tôi cất tạm vào tủ rồi tắm rửa, ngủ một mạch đến sáng.
  • Thế tức là viên kim cương đã bị mất trong ngày hôm nay. Có những ai đang ò nhà óng hôm nay?
  • Vợ con tỏi đang đi du lịch, nhà chỉ có bà giúp việc vói một đứa cháu, À, mà có óng em họ tôi mói tới ỏ nhờ để sáng sớm mại ra ga đi miền Nam,
  • Bây giờ dù đã muộn nhưng tôi buộc phải nói chuyện với họ. ông nhất trí chứ? -Tất nhiên rồi. Tôi sẽ gọi từng người. Đầu tiên là bà Mơ - người giúp việc:
  • Từ sáng đến giờ, chị đã làm gì, ở đâu? -Tôi vin chợ búa, cơm nước như mọi ngày. Sáng nay, trước khi đi làm, ông Ninh bảo là tối tiếp khách về muộn, không ân cơm nên tôi định không đi chợ, chỉ nấu đơn giản... Thế nhưng lúc gần trưa lại có ông Phong, em họ của ông Ninh từ quê lẽn, ghé vào ở nhà để sâm mai ra ga. Vì thế nên tôi lại đi chớ để cơm nước cho tươm tất mộ! chút. Tiếp theo là cậu Binh - châu õng Ninh.
  • Cả ngày hôm nay, cháư đã làm gì?
  • Dạ, sáng cháu đi học. Sau khi ăn cơm trưa, cháu chở chú Phong ra trung tâm thành phố để chú mua sẩm vài thứ.
  • Vậy là chiểu nay hai chú cháu đi cùng nhau suốt à?
  • Không ạ. Cháu chỉ chở chủ Phong ra trung tâm thôi roi đi học luôn. Chú Phong tự về ạ. Cuối cùng là ông Phong:
  • Ông tới đây từ sảng à?
  • Không gẩn trưa tôi mởi tới - Cơm nưỏc xong tỏi nhờ cháu Bình chở đi mua vài ỉhi'r đè’ mang vào Nam làm quà, Sáng mai tôi lên tàu sớm. “ Ông mua được những gì thế?
  • Thì cũng vài thứ đặc sản miền Bắc thôi. À, mà may quá thám tử ạ! Lúc dạo phố tình cờ tôi đã mua được một cưốn tài liệu quý cho đứa cháu. Mấy tháng vừa rồi tôi tìm khắp nơi không có, bỗng nhiên lại mua được. Mừhg ơi là mừng!
  • Tài liệu gì thế ông?
  • Cuốn Tổng tập Toán Tuổi thơ ạ.
  • Tô i có nghe nói đến Tạp chi Toán Tuổi thơ. Lại có cả Tổng tập nữa à?
  • Vâng. 24 số tạp chí của một năm được đóng thành một tập dày. Cháu tôi mê cuốn này lắm. Nam nào tỏi cũng mua cho cháu một tập. Trong đó có rất nhiểu bài toán hay dãnh cho học sinh từ lớp 1 đến lớp 9-
  • Thế thi vừa quỹ, vừa tiện nhỉ. Khi nào gặp chắc tôi cũng sẽ mua cho đứa cháu. Sau khi hòi chuyện cả ba người, thám tử nói với ỏng Ninh:
  • Tôi bắt đẩu nghi một trong ba người ở nhà ông rai. Rất bất ngờ nên ông Ninh không thể đoán được đó là ai. Các thám tử Tuổi Hồng hãy giúp ông Ninh nhé! xsrna CON RŨA VANG (TTT2 số 156) Rất nhiều bạn gủi bài tham gia nhưng rất tiếc, sô bạn làm đúng hoàn toàn thi lại không nhiều. Vì sao nhỉ? vẫn vì một lí do quen thuộc: Thấy để bài có vẻ dế nên không ít bạn đá chủ quan. Có 2 chi tìểt được JI gàì bẫy’ 1 , đó là khoảng thời gian bài thơ được sáng tác và tên đầy đủ của bài thơ. Hầu hết các bạn chỉ chủ ý đến chi tiết đấu (bài thơ được viết trong thời kì kháng chiến chống Mỹ chứ không phải khảng chiến chổng Pháp) mà bỏ qua chi tiết thứ hai {'‘Bài thơ về tiểu đội xe không kính” chứ không phải "Tiểu đội xe không kính”). Hãy ghi nhớ: Để bài càng dễ. chủng ta càng dễ nhầm. Công việc càng đơn giản, chủng ta càng dễ cầu thả, chủ quan. SẫnịĩõT? Phẩn thưởng sẽ được gửỉ tôi: — Nguyễn Thị Diễm Quỳnh, 6A, THCS Long Châu; Hoàng Đức Long, 9A1, THCS Thị trấn Chờ, Yên Phong, Bắc Ninh; Lẽ Ánh Tuyết, 7E1, THCS Vĩnh Tường, Vĩnh Tường, Vĩnh Phúc; Vũ Thái Thủy Linh, 8B, THCS Bạch Lièu, Yên Thành, Nghệ An; Phan Lê Vân Nhì, 7A, THCS Hoàng Xuân Hãn, Đức Thọ, Hà Tĩnh. Thám tử Sêlôccôc Bài 67: Tôi từ thành phố Hồ Chí Minh tớỉ ThS. NGUYỀN vũ LOAN LTS. Nếu biết tiếng Hán bạn sẽ: 1, Hiểu các tù' Hán Việt, sử dụng tốt hơn tiếng Việt của mình. Trang kho từ vựng tiếng Việt rát nhièu từ Hán Việt.
  • Đọc được sách cồ. văn bia bằng chữ Hán vá Hán Nóm, thêm hiểu vãn chương, lịch sử nước Nam mình.
  • Hiểu ngôn ngữ mà cử 5 người trên thể giới có

    hơn 1 người dùng. Dễ dàng hợp tác. làm ăn với các nước và vùng lãnh thổ Trung Quóc, Hồng Kông, Đài Loan, singapore và cá Nhật Đán, Hàn Quốc. NẾU blẻtcả tiếng Anh và tiếng Hán thì thật là tuyệt. Từ mới. yỊÀ.cỏng; [tòng] từ; khởi đầu từ (gi ới từ) ítỉíỉ xìngmíiig; [tính danh] họ và tên m Itguỏií: [quốc tịch] quốc tịch 0 S3àqĩ: [nhật kỳ] ngày ílt zhùzhĩ: [trú chỉ] địa chỉ nơi ờ

    -^Ẻ|ỉ {^dìàrưí yóujiàn: [điện từ bưu kiện] hộpthirđiệntứ [giàn lịch] tóm tắt li lịch jặỊj xingbíẻ: [tính biệt] giới tinh Hí Ế chũshẽng: [xuất sinh] ra đời JÉJ^dìdiãn: [địa điểm] dịađiém Mầu câu. I A: {ỹtip ! íỹ P l | ỈỶÁ ? (Nĩ hão! Nljiào shétime?) Xin chào! Bạn tên gì? B: wíữ ! % ? {Nìhăo [ W5jiào M íngmmg. Nĩ ne?) Xin chảo! Mình tên lả Minh Minh, còn bạn? AiỊtằPl] (WỏjiàoxiăoLíng.Nĩduõdả?) Mình tên là Tiểu Linh, bạn bao nhiêu tuỗi? B: Tl £ . (WốjTnnián shrèrsuì.) Năm nay mình l2tuoỉ. A:$è)tfô*-£JL, immvu (Wò bĩ nĩ dảýĩđiăn cr, wõ shísì sui.Nĩjiâzài nă'er?) Mình lớn hơn bạn một chút, minh 1 4 tuồi. Nhà bạn ử đâu? , Í&BS ? (W6jìãzải Hénẻi, nĩ ne?) Nhã mỉnh ở Hà Nội, cỏn bạn? (Wô cống Hùzhỉmíng shì lải, wỏjiazài Hủzhìmíng shỉ.) Minh đển từ thành phố Iỉồ Chỉ Minh, nhà mình ờ thảnh phổ I lả Chí Minh. 38526xx, -?È|Ỉ ling@yahoo.com, (Wfijiào xiăo Líng, wồ shỉ yĩgè nủ háizi. Wỗ shỉ Yuènán rén. Wỗ dc chũshcng rlqĩ shì ỏr llng líng sì nián shPèr yuò sì hảo, wỗjĩnniản shỉ'Ẻr sui. WÒ zải l lénèi chũshõng. Wó shlgè xucshcng, Xiànzài wÕjiãzải Húzhìmíng shì. Wă dc diànhuả shì 3H526XX, wõ dc diànxĩ yóụịìàn shỉ: lmg@yahoo.com.) Mình tên là Tiểu Linh, mình là một cô gái. Mình lả người VictNam. Mình sinh vào ngày 4 tháng 12 năm 2004, năm nay mình 12 tuổi. Mình sinh rao Hà Nội. Minh là một học sinh. Hiện nay nhà mình ờ thành phố Hồ Chí Minh, Điện thoại cùa mình là 38526xx, hộp thư diện (ừ cùa minh là: ling@yahoo.com. (Kỉ sau đãng tiếp) WNrr19 GAS LAW$ AND PARTICLES 0F MATTER (Tiếp theo kì trước) VŨ KIM THỦY Question 8. A student observes the Brovvnian motíon of smoke particles in air with a microseope. She sees moving points of light. These points of light come from
  • aír particles only moving randomly B air particles only vibrating
  • smoke partỉcles only moving randomly
  • smoke particles only vibrating
  • both smoke and air particles moving randomly Question 9. Some gas trapped in a cylinder is compressed at constant temperature by a piston. Which of the following will not change? A, density B. mass c. molecular spacing D. pressure E. Vũlume Physics ĩerms observe Brovvnian mọtion smoke particle aìr microscope light ran dom cyíinder prston change density mass molecular s pacirtg pressure Vữiume trapped compressed quan sát chuyển động Braond khói phân tử không khí kính hiển vi ảnh sáng ngẫu nhiên ổng hĩnh trụ pít tông thay đổi mệt độ, khôi lượng riêng khối lượng khoảng cách phân tử áp suất thể tích được giữ được nẻn Practice. Tòa soạn chờ bài làm của các bạn gửi về. Bài dịch tốt được nêu tên trên báo và có phần thưỏng. Thời gian nhận bài đến 15.5.2016 tính theo dấu bưu điện. UM ; GAS LAWS AND PARTICLES 0F MATTER {TTT2số156) Q5. A. ị litre. 3 Q6. c. heated then compressed. Q7. A. the aìr ỉn the bag has become less dense. Nhận xét. Có rất nhiều bạn tham ị-TTT ; gia giải bài, tòa soạn xin trao quà cho các bạn có lởi giải đúng là: Vủ Thảo Nhi, Nguyễn Thị Băng Băng, 7C, THCS Bạch Liêu, Yên Thành, Nghệ An; Đỗ Gia Nam , 7D, THCS Vĩnh Tường, Vĩnh Tưởng, Vĩnh Phúc. HÀ MAI 'sm-m NGHE VÀ NHÌN (TTT2 số 156) Mùa xuân đã đến! Ai cũng hân hoan, vui tươi khi được hòa mình vào tiết trời xuân này. Xuân mang đến sức sống cho mọỉ vật, cho cây cối đâm chổi nảy lộc, Đặc biệt hơn nữa, trang không khí rộn ràng này ta bất chợt được ngắm nhìn bức ảnh về mùa xuân của nhiếp ảnh gia Phan Ngọc Quang. Nổi bật trên tâ'm ảnh là màu xanh bát ngát của những chổi non lộc biếc đang cựa quậy vưtín mình. Cạnh đỏ là một khu vườn hoa đang đua hương khoe sắc với đất trời bao la. Giữa tiết xuân ấy. dường như con ngươi cũng trồ nên tươi tắn và xinh đẹp hơn, Các cõ gái trang sắc tà ảo dài truyền thống rực rở không kẻm gì nhữhg bông hoa kia. Đôi bàn tay khéo léo của các thiếu nữ lướt nhẹ trên dây đàn trông thật thanh thoát và yêu kiểu. Khuôn mặt thánh thiện cùng đôi môi cười duyên dáng của họ như phản chiếu ánh hào quang của trdì đất vậy. Chỉ cần ngắm nhìn thẩn thái của các cô, cũng đủ làm cho ta cảm nhận những giai điệu xuân rộn ràng lắm rồi! Phải nói đây là một bức tranh vể mùa xuân rất xuản bởi nó có đẩy đù về hình khối, màu sắc. có xa có gần, có tĩnh có động. Bức tranh giúp ta cảm nhận đầy đủ những cung bậc cảm xúc của mùa xuân. : ị T Nhận xét. cỏ rất nhiều bạn gửi bài, tòa soạn xin được trao quà cho bạn có lời văn hay và giàu cảm xúc là: Lỗ Ánh Tuyết, 7E1 , THCS Vĩnh Tường, vinh Tưởng, Vĩnh Phúc, HÀ MY 20 THÁCH ĐẤU! THÁCH ĐẤU ĐÂY! TRẬN DẤU THỨ MỘT TRẢM BA MƯƠI SÁU Người thách đấu: Tạ Minh Hiếu. GV THCS Yên Lạc, Yên Lạc, Vĩnh Phúc. Bài toán thách đâu: Cho các số thực không im a, b, c thỏa mãn a 4- b 4- c - 3. Tim giá trị nhỏ nhất cũa biểu thức A= a 2 4 b 2 + c 2 - 2ab - 6bc - 4ca. Xuất xứ: Sáng tác. Thởĩ hạn: Trưãc ngày 08.5.201 6 theo dấu bưu điện. TRẬN ĐẤU THỨ MỘT TRÂM BA MƯƠI TƯ (TTT2 số 156} r + a +b 4C BỂt đảng thức đã cho tương đường với a 2 b 2 , c 2 (a + b + c) 2 h an + b H 4C > — === =í == b + c C4a a+b 2 v t3(ab + bc ị ca) a(a ị b 4C) b{a +b I c} c(a 4b 4 c) (a +b 4C) 2 b-s-c c+a a+b 2ự3(ab + bc + ca} a b c a 4 b 4 c .... <=>—=— + — — > -ri. (1)

    b + c C4a a + b 2v3(ab 4 bc + ca) Theo bất đẳng thức Bunhiacopxki ta có |[a(b 4 c) + b(c 4 a) 4 c(a + b)l> (a 4-b + c) 2 I a I b I c b + c c I a aib a b c <=>

    . (a 4 b 4 c) 4 4 — ằ — — — . b + c c + a a I b 2(ab ! bc 4 ca} Ta sẽ chứng minh — — > — a b I c 1 . (2)

    2(ab + bc 4 ca) 2-j3(ab 4 bc 4 ca) Đât t - a + b + c v i'3(ab -I- bc 4 ca) 0, từ bết đẳng thức cd bản (a 4 b 4 c) 2 > 3(ab + bc 4 ca), ta nhận được t 2 > 1 , suy ra t > 1. Bất đẳng thức (2) viết lại thành 3t 2 t

    4 1 » (t - 1){3t + 2) > 0, luôn đúng.

    Suy ra (2) được chứng minh. Từ (1), (2) suy ra đpcm. Đầng thức xảy ra khi và chĩ khi a = b = c. .. ữ ; Nhận xét. Bạn Nguyễn Minh Đức, r - . 9HỈ, THCS Trưhg vương, Hoàn Kiểm. Hà Nội cố lãi giải gọn gàng nhất là người đáng quang trong trận đấư này. Các bạn sau có lời giải đúng được khen: Lê Ngọc Hoa . ÔE1, THCS Vĩnh Tường, Vĩnh Tướng. Vĩnh phủc; Vũ Hoàng Kiên. 8A, THCS Chu Vãn An, Nga Sơn, Thanh Hóa. LÊ Đửc THUẬN BÀI TOÁN DựNG ĐƯỜNG TRÒN NGUYỄN BÁ ĐANG (Hà Nội) Bài toán dựng hình kết hợp với các hài toán chứng minh, tinh toán , quỹ tích gìủp chúng to hình thành vá phót triển tư duy khao học tính chính xõc. Việc vệ hình dúng (có thể dùng phển mềm hỗ trộ vẽ hình) rất quon trọng dể tìm ro hưóng giải toán dựng hình, Bài viết này giới thiệu một sổ bài toán dựng hình như thế. Ví dụ 1. Cho đường thấng d, và độ dài hai đoạn và AD (D e BC) tiếp xúc vớí (1 1 ), (l 2 ) thỉ Ụ 2 II BC. thẳng r 1 , r 2 (r, > r 2 ). Dựng hai đường tròn có bán BÀNH CHS NKÁTO^N H0t Nịéim kính r,| và r 2 tiếp xúc ngoài với nhau và tiếp xúc với đưàng thẳng d.

    Phân tích: Giả sử hai đường tròn (Oỹ r.,) và (0 2 ; r 2 ) tiểp xúc ngoải tại A và tiểp xúc vởi đường thầng d thứ tự tại B và c Ta có 0.|0 2 s r 1 4 - r ?ỉ C^B 1 d, 0 2 C L d. Hạ 0 2 D 1 C^B tại D thì 0 2 D = BC 0.,Đ = r : - X T Theo định lí Pythagore ta có DOị 0 |0 2 — 0 <=>D0| 4rf 2 4 D 0 2 2 ^4^2
  • Cách dựng:
  • Dựng trên một đưởng thầng hai đoạn thẳng liên tiếp EF = r v FG = r 2 , dựhg nửa đưòng tròn đưởng kính EG;
  • Dựng đường thlng vuõng góc với EG cắt nửa đưàng tròn tạí H khi đó FH = X = 2 ";
  • Trên đường thẳng d dựng đoạn thẳng BC sao cho BC = 2x = 2.2 ;
  • Từ đố ta dựng được các điểm D, 0 2 , '0 1 . Chứng mình: Ảp dụng Định lí Pythagore có

    opl =0 1 D 2 +D0| = Oi -r 2 ) 2 + 4^2 = (r-| 4r 2 ) 2 . Ví dụ 2. Cho tam giác ABC. xác định điểm D trên BC sao cho đường tròn nội tiếp các tam giác ABD và ADC bằng nhau. Lỗi giải. Gọi I là tâm đưởng tròn nội tiểp AABC. Đặt BC = a, CA = b, AB = e, a + b 4 c = 2p r 2p 1 - c + BD 4 AD. 2p 2 = b 4 CD 4 AD thì P 1 - p 2 = p + AD.

    Phận tích: Giả sử l 1 và l 2 thứ tự là tàm đường Theo định lí Thales ta có ỈỊÌ 2 IE_r-k k BC IJ r r
  • HD + DK - P 1 - c -r p 2 - b - p -I- AD - b - c. (2) Mặt khác pr = S ABC = S ABD + S ADC = (p, 4 p 2 )k = (p + AD}k. (3) Từ (1), (2) và (3) ta cỏ p-b-c 4 AD _ 1 p a p- AD <=» AD ' ~ p <-> Apĩ = p{p -a). (4) a p 4 AD • Cách dụng:
  • Dựng như ví dụi được FH 2 = p(p - a);
  • Dựng đường tròn tâm A bán kính FH r cắt cạnh BC tại D. • Chúng minh: Dựng (Ụ và (l 2 ). Từ (4), (2). ( 3 ) có (1), suy ra l,l 2 // BC. Ví dụ 3. Cho hai đường tròn (C; I\|) và (0 £ ; r 2 ) tiếp xúc ngoài tại A vả tiếp xúc với đưởng thẳng d tại B và C tương ứng, Tính bản kính đường tròn tiếp xúc với BC và tiếp xúc ngoài vối (0 1 ) t (0 £ ). Nêu cách dựng đường tròn đó- Lòi giải. • Phân tích: Gọi (O, r) là đường tròn tiếp xúc với (0^ r n ) và (0 2 ; r 2 ) và tiếp xúc vâi đường thầng d tại E (giả sử r^ > r 2 ). tròn nội tiếp AABD và AADC có cùng bán kính k Ta có 0,|0 2 = 4 r 2 , 0,|D = r 1 - r 2 . Qua Q kè đường thẳng song song với BC cẳt C^B tại M, cắt 0 2 C tại N. Ta có 00 = r,| + r, 00 2 = r 2 + r, MN = 0 2 D = BC = OM + ON. Theo định lí Pythagore (tương tự ví dụ 1) thì OM - 2^jr, ON -2 'Jíỹ và O2D =2^42 = 7v + v' r 2 r “ 7^2" =■ i’ 7 -. 7 r i + 7 r 2 Bạn đọc tự nêu cách dựng hình và chứng minh. Đến đây nhiều người cho là dễ, nhưng thực ra để dựng r phải qua nhiều bài toán dựng cd bản, ehầng hạn:
  • r- Ĩ1 - t‘l +<2 + 2 7 r l r 2 r 2 r l + f 2 + 27Ỹ2 (áp dụng định lí Thales). Vr 7 7 r 2 r r i r 2 7 r i r 2 (dựng nghịch đảo hai đoạn thẳng)
  • (r 1 +r 2 + 2^r 1 r 2 )r = r 1 r 2 (dựng đường tròn đi qua ba điểm). Ví dụ 4. Dựng đưàng tròn liếp xúc vãi hai cạnh cùa tam giác ABC và tiếp xúc với đưdng tròn ngoại tiếp tam giác ABC. { dề thi Nữ sình tài năng Châu Âu 2012 và định li Lyness) Lời • Phân tích; Giả sử đường tròn (J) tiếp xúc với cạnh AB, AC thứ tự tại M, N và tiếp xúc với đường tròn (O) tại p, suy ra o, J, p thẳng hàng. Đường thẳng PNI cắt đường tròn 0 tại E. Ta có AOPE và aJPN là các tam giác càn = ÕPE = ÓEP = jRp OE//JN. Ta lạl có JN 1 AC => 0E1AC => ỄÃ Éc => BE là phân giác của góc ABC =» CPE = CBỀ = ẤCE => ACEN và APEC đồng dạng (g.g) => => EC 2 = EN.EP. (1) EP EC Giả sử đường thẳng BE cắt MN tại I. Ta có IMP = NMP = NPỹ = ÉPy = ẺBP. => IMP - IBP =$ tứ giác BMIP nội tiếp =>BMP = B ÌP (cùng chắn BP) Mặt khác MPx = PMB = MNP {cùng chắn MP) => BĨP = MNP => ỂÍP = ENi AENI co AEIP (g.g) El EN _/> => 7=7 = ^7 El = EN.EP- (2) EP El Từ (1) và (2) =» El = EC = EA. Ta có ICB = Ểic - 1 ẵc -90° -ịÂ -lẻ = -Ịc 2 2 2 = Cl là phân giác của 0 => I là tâm đường tròn nội tiếp AABC — > AI 1 MN, • Cách dựng:
  • Dựng điểm I là giao hai đường phân giác AI và BE của AABC, với E thuộc (O);
  • Qua I dựng đường thẳng vuông góc với AI cât AB, AC tại M và N theo thứ tự;
  • Từ N dựng đường vuông góc vói AC cắt AI tại J, kẻ JM 1 AB;
  • Dựng đường tròn tâm I bản kính JN cắt (O) tại p. • Chíỉng mình: - Dễ dàng thấy JM 1 AB, JN 1 ÂC và JM - JN nên (J) tiếp xúc với {o) tại M, N. 'Do ÀẺ = ÁC nên OE 1 AC. Suy ra APOE co APJN, do đỏ Jp = JN. Bạn đọc hãy khai thác thâm các tính chất của đường tròn (J). Bải tập vận dụng Bài 1. Cho đoạn thẳng a. Dựng đoạn thẳng ĩồỊĨ. Bài 2. Cho tam giác đểu cạnh a. Dụng ba đường tròn có bán kính bâng nhau nội tiếp trong tam giác và tiếp xúc vãi nhau từng đỏi một. Bài 3. Cho ba đoạn thẳng a, b, C- Dựng ba đưàng tròn tiếp xúc nhau từng đôi một cố các bán kính là a, b, c. X AUSTRALIAN MATHEMATICS co MPETITIO N AMC 2015 UPPER PRIMARY DIVISION AUSTRALIAN SCHOOL YEARS 5 AND 6 Time alíovved: 60 minutes PGS, TS, ĐỖ TRUNG HIỆU (Hà Nội) (Sưu tầm và giới thiệu) Qucstions 1 to í 0, 3 marks cach
  • What does the digit 1 in 2015 represent? (A) one (B) ten (C) one hundred (D) one thũusand (E) ten thousand
  • What is the value of 1 0 twenty-cent coins? (A) $1 (B) S2 <C) £5 (D) S20 (E) S50
  • What temperature does this thermometer show? ị °c :30 Ị-25 :2D r15 0 (A) 25° (B) 38° (C) 27° (D) 32° (E) 28°
  • Which number do you need in the box to make this number sentence true? 19 + 45= 20 + n (A) 34 (B) 44 (C) 46 (D) 64 (E) 84
  • Which number has the greatest value? (A) 1.3 (B) 1 303 (C) 1-31 (D) 1.301 (E) 1.131
  • The perimeter of a shape is the distance around the outside. Which of these shapes has the smallest perimeter? (C) <D) (E)
  • The class were shown this picture of many

    dìnosaurs. They were asked to work out how many there were in half of the picture. v> Vỉ Ví v> Vĩ Vĩ Ví Ví v> Vi v> V) v> V) v> v> v> v> v> Ví v> V) v> V“> v> Vì Vj Ví v> V) Ví Vĩ v> Vỉ Vỉ v> v> V) V? Vi v> V) vv> v> Vj V? v> VìVi Ví Ví Vi Ví Ví Ví v> Vi Ví Vi v> Vỉ v> Vj Vĩ V) v> Ví Vĩ Vj v> v> v> Vì Vỉ v> v> Vì V) v> v> Vì Vỉ Vĩ v> Vi V V)V>Vi V) V) v> Vi V V) Vĩ Vỉ ww Vi Vi Vĩ Vi Ví Ví ^ j J V ■ Simon wrote 6x10.

    Carrie wrote 5x12, Brian wrote 10 X 12 +2. Rérny wrote 10 + 2x12. Whũ was correct? (A) Alt four were correct (B) Only Simon (C) Only Carrie (D) Only Brian (E) Only Rẻmy
  • In the diagram, the numbers 1, 3, 5, 7 and 9 are placed in the squares so that the sum of the numbers in the row is the same as the sum of the numbers in the eolumn. The numbers 3 and 7 are placed as shown. What could be the sum of the row? (A) 14 {B)15 (C) 12 (D) 16 <E) 13
  • To which square should I add a counter so that no tvvo rows have the same number of counters, and no two columns have the same number of Cũunters? For whigh of the shapes is this not possible? A m m 8 m c m m D m m m E (A) A (B) B (D) D (E E
  • A half is one-third number? (A) three-quarters (C) one and a third (E) one and a half (C)C of a number. What is the one-sixth (D) five-sixths Quesborts 11 to 20, 4 marks each
  • The trỉangle shúwn is tolded in halí three times wíthout untolding, making another triangle each time. Which íigure shows what the triangle looks like when unídded?
  • lỉ L - 10D and M - ữ.l, which of these is largest? (A)L+M (B)LxM (C)L+M (E)L-M
  • You want to com bi ne each of the shapes (A) to (E) shovvn below separately with the shaded shape on the right to make a rectangle. You are only aílowed to turn and slide the shapes, not flip them over. The tìnished pieces will not overlap and wi(ỉ íorrn a rectangle wíth no holes.
  • Aplumber has 12 lengths oídrain pipe to load on his ute He knows that the pipes won’t come loose if he bundles them so that the rope around them ís as short as possĩble. How does he bundle them? (A) <B) (C)
  • The numbers 1 to 6 are placed in the cirdes so that each side of the trigngle has a sum of 10. If 1 is placed in the circle shown, which number is in the shaded cirde? {A} 57 (D) 81 (B) 63 (E) 84 (C) 75 (Kì sau đăng tiếp) Bài 4NS. Tìm sổ các sô nguyên dương n không lớn hơn 2015 thỏa mãn =4y+ (kí hiệụ [a] là Số nguyền lớn nhất không n"
  • n " n 2 3 4 vượt quá a). VÚ ĐỈNH HÒA (GV. trường Đại học Sư phạm Hà Nội) ... íx-y -z = 2(Jỹz 4 Jỹ + Vz - Vx) Bài 5NS. Giải hệ phương trình; . ' !_ p v ''yz = X -i/3z + 1. CAO NGỌC TOẢN {GV. trường THPT Tam Giang, Phong Điền, Thừa Thiên - Huế) Gài 6NS. Cho nửa đưởng Iròn (O) đường kính AB Gọi M là điểm đối xứng của o qua A. Đường thảng qua M cẩt nửa đưởng tròn (0} tại c và D (C nằm giữa M và D). Gọi E lá giao điểm của AD và BC. Chửng minh rãng — — = 3— — . AD BE NGUYỄN ĐỬC TẨN (TP Hồ Ch! Minh) (TTT2 số 1 56} Gài 28NS. Giả sửn 4 \ n 3 1 1 là số chính phương. Vì n 4 4 n 3 4 1 > (n 2 ) 2 nén n 4 4 n 3 4 1 = (n 2 4 k) 2 = n 4 4 2kn 2 + k 2 vâi k là sổ nguyên dương. Do đó n 2 (n - 2k) s k 2 - 1 k 2 - 1 in 2 mà k 2 - 1 ? 0 => k 2 - 1 = 0 hoặc k 2 - 1 2 n 2 . • Nếu k 2 - 1 = 0 thì k - 1. Khi đủ n 2 (n - 2) = 0 => n= 2.

    Nếu k 1 thì k 2 - 1 > n 2 k 2 > n 2 => k > n =s> n - 2k < 0 => n 3 (n - 2k) < 0 => k 2 - 1 <0 => không tồn tại k. Vậy n = 2. Nhận xét. Các bạn sau có lài giải đúng: Trần Thị Thu Huyền, 9A3, THCS Lâm Thao, Lâm Thao; Lê Nguyễn Quỳnh Trang, sc, THCS Văn Lang, TP. Việt Tri, Phú Thọ; Kim Thị Hổng Lĩnh, 9E1, THCS Vĩnh Tường, Vĩnh Tưởng, Vĩnh Phúc; Trần Diệu Linh, 9B. THCS Nguyễn Thượng Hiển, ửng Hòa, Hả Nội; Chu Thị Hằng, 9AỊ THCS Yên Phong, Yẻn Phong, Bắc Nính. Bài 29NS. Vì abc - 1 nén {a + bc)(b + ca)(c 4 ab)
  • a(a 4 bc)b{b + ca)c(c 4 ab) - (a 2 4 1)(b 2 4 1){c 2 4 1 ) (a + b)(b 4 c)(c 4 a} = (a 4 b 4 c)(ab j -bc+ ca) - abc. (1) Áp dụng bẩt đẳng thức AM-GM ta được abc < ^(a + b + c)(ab 4bc + ca). (2) Từ (1), (2) suy ra s {a 4 b)(b 4 c)(c 4 a) > -X(a 4 b 4 c)(ab 4 bc 4 ca). 9 3 1 Do đó p >4(a 4b 4c) 4 9 a I b ¥ c 7 , .(1, . 1 Y = pa+b+c) 4 4(a 4b tc) 4 >3. 9 1.9 a H-b + c } Vậy giá trị nhò nhất của p là 3 khi a = b = c = 1 , Nhận xét. Các bạn sau có lòi giài đúng: Lể Nguyễn Quỳnh Trang, 9C, THCS Vàn Lang, TP. Việt Trì; Trần Thị Thu Huyền, 9A3 r THCS Lâm Thao, Lâm Thao; Kim Thị Hóng Lĩnh, 9E1, THCS Vĩnh Tường. Vĩnh Tưởng, Vĩnh Phúc. Bái 30N$. Bạn đọc tự vã hình. Gọi I là trung điểm của OM thi I cđ định. Hạ IK 1 MB, IN X DE. Ta có IK = = . 2 2 Ta có ME lì AB => MÉC - BÁC (đồng vị}. Mà MÕC - BAC - -ịsdBC ị =? MEC - MQC. T 2 ) Do đó tứ giác MOEC nội tiếp đường tròn (1. IO) (vì MCO = 90°). Chứng minh tương tự ta có tứ giác MOBD nội tiếp đường tròn (l r 10). Suy ra M, E, B, D cùng thuộc đường tròn (I, 10). Do đô tứ giác MEBD nội tiếp, mà ME // BD => MEBD là hình thang cân =» DE - MB => IN - IK - ị mà IN X DE 2 TT> DE luồn tiếp xúc với đường tròn cố định Nhận xét. Không có bạn nào giải đúng bài này. Các bạn được thương ki này: Trần ■ ,:L Thị Thu Huyền, 9A3, THCS Lâm Thao, Lãm Thao; Lê Nguyễn Quỳnh Trang, 9C, THCS Vãn Lang, TP Viạt Trì, Phú Thọ; Kim Thị Hồng Lĩnh, 9E1 , THCS Vĩnh Tường, vĩnh Tường, Vĩnh Phúc; Trần Diệu Linh , 9B, THCS Nguyễn Thượng Hiển, ửng Hòa, Hà Nội; Chu Thị Hằng, 9A1, THCS Yên Phong, Yên Phong, Bắc Ninh, Ảnh các bạn được thưởng ỏ bìa 4. NGUYỄN HIỆP Tạp chí Toán Tuổi thơ Tự HÀO VÀ TIN YÊU Nhục iitiìvtỷ Ut ì ; ÙzứjLóiLÌẲỆL Ềỉiứắ£±ặ f KÌhA Tậnệ %/ỉ C.ỈIL Tứ *ổĩ TctSy tĩ-lổ ) •' H M Jj-Lj -/ Ệ V/(|/ £/ (níj rrì,ỆÌ> - uJr.ĩị_ í 1 p /' I lp J - 4 M u fd Tat .1 1 " Hi ỉĩỊLếi- Tươi' >ít^ 1 ' I , rĩ : ,/> .' Wẫ L= ■c^tí-L . $ac hííêt Kịg.rt fcití5 Yẹị? Khỉo Tch yẠt fi Ễ -fr-M=Ã 5 — =r= &- ữ <- - ẺíHt c J 1 f í- - 1 % 11 o ít-r Ậsp y v> — s I J 1 1 ã * ,/. /S Tt. ! p, V p . .. J — * y f i ĩ p 0 = g — ĩfi^Ậjzfz r j m I §

    T -t^: c/ii. t t-t-ô-i. ĩ\àí iLÍt^y Ùt/ỈA- w J í?t

    — f ' ú c i> i,ệ ^1 J f Ị J T ' 1 ệ r -fij tiio ĩhẽ, T «ìrt/t nítẨt. gte jv r 4 Ị p P i ■ ,■ - I 1 /T =Ị= "ÚT " 1 = _ 1 - ITĨ P i ■ ,■ - J gjg — = *• - rft^ic 4 — ể cT/ , fĩy c^L-0 ^tiê /i r-íVVi<y TStỵi c/í Títúí TĩưỈL i -. Ó-L- r, I r I 7*t-í? rnfí' Mủ«áfc s ễ-Cq y f':ic l/i’ếri 5*%^ c ũ/ J r a ỉ i í— J» j ft|fcáfc t' t iJ r T_ XeS' 1.' ■} /tr íií rJtí; . t.iíĩì-M. íỂĩn. íỉiVíia ỈSLC TRÒ CHƠI VỚI BẢN DỒ VŨ NAM TRỰC Bạn hãy quan sát bản đỗ 10 tỉnh, thành phố, đong bằng sòng Háng, sau đây sẽ gọi chung là các tĩnh: Vĩnh Phúc, Bắc Ninh, Hà Nội, Hưng Yén, Hải Dương, Hải Phòng, Thái Bình, Hà Nam, Nam Định, Ninh Bình. Trò chơi được đặt ra là bạn cần tô màu tẩm bản đổ này sao cho các tỉnh liền nhau không cùng một màu. Như vậy nhiều nhất, ta phải cần đến 10 màư ứng với 10 tỉnh. Nhưng người ta hay quan tâm đến vấn đề làm sao cho sđ màu phải dùng là ít nhất. Vậy bạn nên bắt đầu từ đâu? (Xem hình 1 ) Ta nên chú ý đến Hưng Yẻn và Hà Nam, nhìn trén tấm bản đổ này thì mỗi tỉnh này đều giáp vâi 5 tỉnh khác (ỏ bản dổ của cả nưãc thì Hà Nam còn giáp thêm vái tình Hòa Bình), Bắt đầu ta tỏ Hưng Yên màu số 1. Như vậy các tỉnh Hải Dương, Thái Bình, Hà Nam, Hà Nội, Bẳc Ninh đểu không thể tô màu sõ 1 . Khi Hài Dương tô màu số 2 thì Thái Bình phải màu khác với Hưng Yên và Hải Dương. Vậy ta phải dùng màu thử 3 khi tô màu chô tỉnh Thái Bình. Liệu 3 màu có phái là số màu cần dùng ít nhất? Dễ thấy Hải Phòng, Vĩnh Phúc, Nam Đình không giáp vói Hưng Yên nên có thể dùng màu số 1. Để ý đến Hà Nam, Nam Định, Ninh Bình sẽ thấy phải dùng 3 màu. Do đó Thái Bình và Ninh Bình nên cùng 1 màu. Ta gọi đó là màu số 2. Hà Nam sẽ là màu số 3, Hà Nội màu sổ 2. Vấn đề còn lại Hải Dương màu sô mấy? Vì các tỉnh quanh Hải Dương đều đã dùng các màu sô 1, 2, 3. Vậy Hải Dương phải tỏ màu số 4? Bài toán trỏ nên phức tạp, Đến đây ta thây Vĩnh Phúc chỉ giáp một tĩnh, Nính Bình và Hải Phòng giáp 2 tình nên trên bản đổ này màu cũa 3 tỉnh đó sẽ tô dễ dàng sau cùng. Vậy ta chỉ cẩn Ghú ý 7 tỉnh còn lại. Bãy giờ tô Hưng Yèn màu số 1 (hình 3), Hải Dương màu số 2 thì Thái Bình và Bác Ninh phãi màu số 3. Hà Nội không thể tò màu số 3 và 1 nèn Há Nội phải được tô màu số 2, Hà Nam cũng không thể tô màu số 1 và 3 nén Hà Nam phải được tô màu sò’ 2, trùng màu Hà Nội (I), Vậy ít nhất bản dô nàỵ cắn dùng 4 màu. Bạn thử tự tô nhẻ để thấy tấm bản đồ đơn giản này đã phải dùng tỏi 4 màu và 4 màu lả đủ, CLB6. Given the numbers X, y, and z not equal to 1. Find the value of the ... . . xy-2y +1 yz-2z+1 zx-2x+1 following expression M = — - \ — ) xy-x-y+1 yz -y —z +1 zx -z -X +1 CLB7. Find the nunnbers a, b, and c such that a 2 + b 2 + c 2 = b 2 -c 2 Jí 7 c -a a 2 -b 2 — ỉ + 1 ■ a 2 +30 b 2 +4 c 2 +1975 CLB8. a) Given the numbers a and b such that a ị b = 1. Prove that a 3ư + b 317 > 0
  • Let a, b. and c be the lengths of the sides of a triangle. Prove that 1 < ■ b , c
  • + ——< 2 . b i c c ị a a ■ b CLB9. Given twelve po si lì ve integers from 1 to 12. Is ìt possible to arrange these numbers in a cìrcle such that the sum of any two adjacent numbers i$ greater than 12 ? Explain why? CLB10. Given the parallelogram ABCD. Find the position of a point M inside the parallelogram such that MA 2 + M8 2 + MC 2 +• MD 2 attain its minimum value. NGUYỄN ĐỨC TẤN (TP. Hổ Chí Mình) 'mm* Góc OLYMPIC Bài 1. Ta có 243 342 = (3 5 ) 342 = 3 1710 = (3 2 ) 855 = g855 < 10865 SỐ 10 865 là số tự nhiên nhỏ nhất có 856 chữ SỐ. Vậy số 243 342 có ít hơn 856 chữ sô. Bài 2. Ta có 2ab = c 2 , ac = 4b 2 ^2b_c a 2b 3 _2b_c_a+2b + c c a2b c 2b c a 2b + c + a suy ra a = c = 2b. Do đó 5a + 4b + 3c 10b-h4b + 6b 20b 3a + 2b-hc ” 6b + 2b + 2b IDb Bải 3. Ta có M = 7|x - 4| + 2\x - 4| + |x - 1| + X 7.0 ị 2(4 - x) + (x - 1) + X = 7. Đẳng thút: xảy ra khi X - 4 = 0, X — 4<0.x — 1 >0 o X - 4. Vậy giá trị nhỏ nhất của M là 7 (đạt tại X = 4). Đài 4. Ta có a # 0 và 201 5 a < 1968g suy ra a = 1. Do đó 201 5 + bcdẽ.9 = 1 968g. Vì 2016 + bede.â chia hết cho 9 nễn 196&g + 1 chia hết cho 9^1+9 + 6 + 8 + g + 1=25-i-g chia hết cho 9 => g - 2. Từ đỏ ta được bõdẽ = (1 9682-2015) : 9 = 1 963. Vậy abc deg - 1 1 9632. Bài 5. Trên nửa mát phầng bờ AC có chứa B vẽ tam giác đểu ACD. Vi AABC cân tại Acó A - 100° nên ABC = ẨCB = 40° Suy ra ỎCB = 20°. Ta có AABD cân tại A, BẢD = 40° nên Kì 10 (TTT2 SỐ156} ẨBD = ÀDB = 70°. Suy ra ÓBC = 30°. A Vì ABMC và ABDG có MBC = DBC (= 30°), BC chung, MCB = DCB (= 20°) nên ABMC = ABDC (g.c.g) > MC = DC mà DC = AC (vì AACD đẻu) => MC - AC ACAM cân tại c. Ta lại có ACM = ÀCB - MCB = 40° - 20° = 20 ,} . Do đó ÁMC - MAC - (180° - ACM) : 2 - 80°. png— Nhận xét. Các bạn đươc thưởng kì này: Diêm Đăng Hoàng, 8A1, THCS Chất Lượng Cao Mai Sơn. Mai Sơn. Sơn La; Đinh Vù Tùng Lãm , 7A2, THCS cầu Giấy, Cầu Giấy, Hà Nội; Nguyễn Chi Còng, 7A3, THCS Lâm Thao, Lâm Thao. Phú Thợ: Tảng Vản Minh Hùng, 9A, THCS Lý Nhật Quang. Đô Lưdng, Nghệ An; Nghiêm Ngọc Phong, 8A1. THCS Yên Phong, Yên Phong, Bắc Ninh. NGUYẺN HIỆP NHỮNG NĂM LẺ 6 trong lịch sử Năm 226 Giao Châu gồm 4 quận; Hợp Phố, Giao Chỉ, CỬU Chân và Nhật Nam. Năm 866 Cao Biền đổi Giao Châu từ An Nam đó hộ phủ thành Tĩnh Hải quận tiết trấn. Năm 966 Loạn 12 sứ quân: Kiều Câng Tiễn. Kiều Thuận, Ngô Khoan, Ngô Nhật Khánh. Đồ cảnh Thạc, Lý Khuê, Lã Đường, Nguyễn Thủ Tiệp, Nguyên Siêu, Phạm Bạch Hổ, Trân Lâm, Ngô Xương Xf
  • 1.1226 Trần cảnh lén ngôi, bắt đầu nhà Trần. Năm 1686 Thương gia Véretcủa công ty Đỏng Ấn đề nghị vua Pháp đánh chiếm Côn Đảo của nước ta. Năm 1696 Triều đình Lê - Trịnh bắt ngoại kiều phải nhập tịch Việt Nam, nói tiếng Việt và theo phong tục Việt. Năm 1696 Trịnh Càn cấm truyền bá đạo Gia Tô. Năm 1746 Nghĩa quân Hoàng Công Chất bắt sống trấn thủ sơn Nam Hoàng Công Kỳ. Tháng 2.1776 Nguyễn Lữ đánh Gia Định. Nguyễn Phúc Thành bỏ chạy về Bả Rịa. Năm 1806 Cao Văn Dung và Nguyễn Tình khởi nghía ở Sơn Tây, Hâỉ Dương chống Triều đình nhà Nguyễn. Năm 1826 Nhà Nguyễn tiển đánh các vùng thuộc Bình Hòa, Bình Định chưa thuần phục chính quyền. Tháng 2.1856 Nhà Nguyễn bắt đầu viểt bộ sử Khâm định Việt sử thông giám cương mục. Năm 1876 Quàn viễn chinh Pháp chia Nam Kỳ thành 4 khu vực hành chinh: Sài Gòn, Mỹ Thù, Vĩnh Long, Bát sắc. Năm 1886 Thực dân Pháp cho làm đường qua đèo Hải Vân nối thông Đà Nắng vối Huế. 26.1.1886 Paul Bert làm Tổng Trú sứ Trung và Bẩc Kỳ, ngang với thống đốc Nam Kỳ, mở đẩu chế độ quan vân thay quan võ cai trị. Tháng 2.1886 Nghĩa quân Đinh cỏng Trảng xây dựng cân cứ Ba Đình, Nga Sơn, Thanh Hóa. 17.2.1996 Thành láp tỉnh Kiên An do đổi tên tinh Phú Liễn. BỈNH NAM HÀ
  • 2.1906 Thực dân Pháp khai thác tuyến đường sắt Hài Phòng - Lào Cai dài 390 km, Tháng 2.1906 Phan Chu Trinh đi Quảng Châu gặp Phan Bội Châu, cùng đi sang Đòng Kinh (Nhật Bản). II. 1.1916 Thực dân Pháp ra nghị định động viên quân dự bị Việt ở Nam Kỳ và bắt lính người Việt ở cả ba kì đưa sang Pháp trong Đại chiến thể giới lần thứ nhất. 15.2.1916 Biểu tình đánh phá Khám lớn Sài Gòn. Nãm 1926 Báo Việt Nam hổn ra số đâu tại Pháp. Năm 1926 Cóng nhân sở Bưu điện Sài Gòn bãi công dỏi táng thêm người làm. 31.1.1926 Biểu tình phản đối Pháp trục xuất người Bắc Kỳ, Trung Kỳ ra khỏi Nam Kỳ. Năm 1946 Thành lập Quân ủy Trung ương, Năm 1946 Chính phủ lâm thòi, cải tổ thành Chính phủ liên hiệp lãm thời, tồn tại đến 2,3.1946 là ngảy Quốc hội họp ki đẩu tiên. 6.1.1946 Tổng tuyển cử đầu tiẻn cũa nưâc Việt Nam Dân chù Cộng hòa, 24.1.1946 Chủ tịch Chính phù lảm thời VNDCCH kí sắc lệnh quy định các thành phố Nam Định, Vinh - Bển Thủy, Huê và Đà Nang tạm coi là thi xã cho đến khi có lệnh môi. 30.1.1946 Thành lập Nha Thể dục Trung ương thuộc Bộ Thanh nièn. 20.2.1946 Thành lập Nha Công an Việt Nam. Hỏi: Anh Phó ơi! Các bạn ở lớp dưới có thể vận dụng kiến thức của lởp trên để giải bài trong TTT không ạ? NGUYỄN THỊ BĂNG BÁNG (7C, THCS Bạch Liêu, Yên Thành, Nghệ An) Đáp: Kiến thức không giài hạn Làm mọi cách được thõi Nhưng thi thì cẩn thận Dùng kiến thức học rói Đừng lấy cách lớp trên Giải bài cấp học dưỡi Hỏi: Anh Phó ơi! Lớp em có một bạn học rất yếu nhưng lúc thi thì lại được điểm cao nhất lóp. Theo anh thì liệu có phải bạn ý đem tài liệu vào phòng thi không ạ? PHAN THÀNH CƯỜNG (Quên ghì địa chì) Đáp: Học tài thi phận là thường Học phận thi tài ấy mới lạ Muốn tìm nguyên nhân từ từ đà Đừng vội kết quy nhé bạn Cưòng. Đáp: Chắc là tại học chưa càng Cho nên thư giãn là không thây cắn Tháng sau bài vỏ tăng dẩn Sẽ vui cười sẽ vui cười liên miên. Hỏi: Tại sao gẩn đây TTT không có mục Vui cười ạ? HOÀNG THỊ MỸ DUYÊN (7D, THCS Vĩnh Tường, Vĩnh Tường, Vinh Phúc) ANH PHỐ Bạn hãy vào website; //olm.vn/hieu-sach-oiiline để đọc tạp chí Toán Tuổi thơ bản điện LỪ nhé. CÁC LỚP 6 4 7 Bài 1(1 58}. Tìm tất cả các số nguyên dường X, y thỏa min (x + y) 4 =40x + 41. NGUYÊN đừc tấn (TR Hổ Chí Minh) Bài 2(1 58). Cho tam giác ABC vỡi BẦC = 1 20° , AC = 2AB. Đường thẳng qua A và vuông góc với AC cắt đường trung trực của BC tại o. Chứng minh rằng QBC là tam giác đều. NGUYỄN KHÁNH NGUYÊN (SỐ 3/29E , đường Đà Năng, Hải Phòng) CÁC LỚP THCS Bải 3(1 58). Cho phương trinh 4x 2 - 4mx + 4m - 5 = 0 {m là tham số). Tìm m để phương trình có một nghiệm âm, nghiệm còn lại lớn hơn 1 nhưng nhỏ hơn giá trị tuyệt đối của nghiệm ảm. LẠI QUANG THỌ (Phòng Giáo dục và Đào tạo Tam Dương, Vĩnh Phúc) Bài 4(1 5B). Cho các sổ thực dương 3 , b, c thỏa mãn a + b + c = 3. a 2 (b + 1) b 2 (c+1) , c 2 (a +1) Q Chútig minh răng — 7 - +- — — — P— H 4 2. a + b + ab b + c+bc c+a+ca CAO MINH QUANG (GV. THPT chuyên Nguyễn Bính Khiêm, Vĩnh Long) Bài 5(1 53). Có 102 diễn viên nam và nữ xếp thành vòng tròn múa xòe. Cứ 2 người kề nhau thi nắm tay nhau. Hỏi sổ cái nắm tay của hai người cùng giới và sổ cải nắm tay của hai ngưòi khác giới có thể bằng nhau hay không? vì sao? VŨ KIM THỦY Bải 6(1 58). Cho hai đường tròn (O) và (O s ) tiếp xúc ngoài tạl T. Một đường thẳng tiếp xúc vài (Q f ) tại D và cắt (O) tại A và B (A nằm giữa B và D). Gọi c là điểm thuộc cung BT khỏng chứa A của (0) (C Ỷ B, c -t T). Vẽ tiếp tuyến CE của (O ) (E là tiếp điểm). Chứng minh rằng giao điểm thứ hai của DE với đưủng tròn ngoại tiếp ACTE là tâm đưởng tròn bàng tiếp trong góc ABC của AABC. THÁI NHẬT PHƯỢNG (GV. trưởng THCS Nguyễn Vãn Trỗi, Cam Nghĩa, Cam Ranh , Khánh Hòa) SOLUE UIA NIAIL COMPETITION QUESTIONS Transtãted by Nam Vũ Thành 1(153). Find all positive integers V and y such that (x + y) 4 = 40x + 41. 2(153). Given a triangle ABC having /BAC = 120°, and AC = 2AB. The line passing through A and perpendicular to AC intersects the perpendicular bìsector of BC at o. Prove that the triangle OBC is an equilateral triangle. 3(158). Given the equation 4x z - 4 mx + 4m - 5 = 0, where m ís a parameter. Find m such that the equation has two roots in which one root is greater than 1, and one root is negatìve with íts absolute value greater than the other root. 4(1 58). Let a, b, and c be positive reai numbers such that a + b + c = 3. Prove that + b 2 (c + 1) + c 2 (a+1) a + ỏ + ab b+c+bc c+a+ca 5(153). There are 102 male and female dancers íorming a circle to dance, everyone joining hands with the persons next to them, Can the number of hand joins between two people of the same sex be equal to the number of hand joins between people of opposite sexes? Explain why. 6(158). G iven two circles (O) and (ơ) externally tangent to each other at T. Alinetangent to the circle (O) at D Intersects the circle (O) at A and B (where A lies between B and D). Let c be a point on the arc sr of the circle (O) which does not contain A ịC * B, c 7). Let CE be the tangent to the circle (ơ) where E is the point of tangency. Prove that the second intersection of the line DE and the circumcircle of the triangle ACTE is the center of the escribed circle in the angle ABC of the triangle AABC. PHIẾU ĐĂNG Kí THAM Dự CUỘC THI GTQT NÀM HỌC 2015-2016 • Ngày 23.3-2016 tại TP. Quảng Ngải xinh đẹp, đoàn công tác của Lạp chi Toán Tuổi thơ dà gặp lãnh dạo sỏ Giáo dục và Đào tạo tinh Quảng Ngãi. Cũng lãm việc có Ths. Trần Hữu Tháp, Phó Giám đốc: ỏng Đinh Huy Quang, Trưởng phòng Giáo dục Trung học; ông Dạng Phiên, Trưởng phỏng Giáo dục Tiểu học; ông Võ Thành Phước, chuyên vỉẽn Văn phòng, ThS. Vũ Kim Thủy, Tổng biên tập lạp chí Toán Tuổi Lhơ đã giđi thiệu về Tạp chi và các cuộc thi mà Toán Tuổi thơ dang tổ chức: Cuộc thi liên câu lạc bộ Toán Tuổi thơ toàn quốc diẽn ra vào tháng 6.2016; Cuộc thi AMC 2016 phối hợp với AMT vảo tháng 7.2016. Phó Giâm dốc Trần Hữu Thãp dà gỉởĩ thiệu ve ngành Giáo dục Quảng Ngài và ngành Giáo dục Quảng Ngãi sẽ IhaiEL gia các cuộc thi mà Tạp chi đang tổ chức, đề nghị dưa Lổng tập Toán Tuổi thơ vào thư vtện các nhà trưởng dể các thầy cô giáo và các em học sinh có một nguỏn tài liệu tốt phục vụ cho hoạt động dạy vả học. Phó Giám đốc Trần Hữu Tháp (giữa) tại buấì tàm việc vói TTT Ống Đinh Huy Quang cho biết các cuộc thi giải toán tiếng Anh mả học sính Quảng Ngãi tham dự đã đạt thành tích cao. Cũng trong ngãy đoãn công tác của Tạp chí dã gặp ỏng Huỳnh Hoàng Phương, Giám dốc và ống Lẽ Như Thong, Phó Giám đốc Cõng ty cổ phán sách vả thiết bị trường học Quảng Ngãi để giởi thiệu một số ấn phẩm Tạp chỉ dang phãt hành, • Ngày 24.3.2016, Tạp chí Toán Tuổi thơ đã làm việc VỚI Sở Giáo dục vâ Đào tạo Lính Phũ Yên. Cung trao đổi có ông Nguyên Trọng Thiện, Phó phòng Giáo dục Tiểu học; ông Phạm Huy Văn. Chánh văn phòng; ... Tạp chí dã giới thiêu vẻ Toán Tuổi thơ vả các cuộc thi đang diên ra. Ông Nguyên Trọng Thiện đã nõi vẻ một sổ nẽt nổi bật. của ngành Giáo dục Phũ Yên. Ông đâ dẫn học sinh Phú Yẽn dự thỉ Olymplc Toán Tuổi tlití toàn quốc vớí thành tích tốt. Các cuộc thi Toán Tuổi thơ Lổ chức giúp các thầy cô giảo và các cm dạy tốt hơn và học tốt hơn môn toán. • Trưởc đỏ ngày 22.3.2016, đoàn công tác của Tạp chí dã gập tổ Phổ thông của Phỏng Giáo dục và Đào tạo TP, Quy Nhơn, Bình Định vả ông Phạm Dinh Thuấn, Giám đốc; ông Dỗ Hữu Long, Phó Giám đốc Công ty cổ phần sách và thiết bị trường học Bình Định. PV, Giua mùa xuần. đỏng bảng Bác bộ hoa Xoan lớp lớp rụng. Nếu như â Hà Nội giờ lã mùa hoa sưa tráng như bông tuyết bay trong trời xuân trời Nam, tiếp đó lả mùa hoa Bằng lãng tím phố phường thì mũa này Tây Bác rỢp trời hoa Ban. Hoa Ban trắng và hoa Ban hồng. Những cátih hoa rụng cũng lớp lớp vơi dảy gợi nhỡ thư Nguyên Binh. Bạn hãy viết bãi tả vẻ đcp hoa Ban vã bức tranh hoa mùa Xuân này nhé. MORIS VŨ .Anh: Phan Ngọc Quang CÁC HỌC SINH ĐƯỢC KHEN TRONG cuộc THI GIẢI TOÁN DÀNH CHO NỮ SINH Từ trái sang phái: Lê rtguyẻn Quýnh Trang. Rim Thị Hỏng Lính, Tràn Diệu Linh, 5INGE Ỉ9SĨ f~Jỉ CP VẦN p Hỏỉiũ míu HỔNG HẢ 4UẠị tmy/ềt iỈTÁrtij - cạỉỉì f ỉni Công ty CP VPP Hồng Hà là nhà tàỉ trợ cho 2 cuộc thi: và Giấy phép xuất bản: số 31/GP-BVHTT, Gấp ngày 23/1 /2003 cùa Bộ Văn hóa và Thông tin. Mã số: SBTT158M16, In tại: Công ty cổ phần in Công Đoản Việt Nam, 167 Tây Sơn, Đổng Đa, Hà Nội. In xong và nộp lưu chiểu tháng 04 nám 2016. NĂM THỨ MƯỜI BẢY ISSN 1859-2740 Ị 59+1 6(T 05+06/2016 Giá: 200005 TRUNG HỌC C0 sở NHÀ XUẤT BẢN GIÁO DỤC VIỆT NAM - BỘ GIÁO DỤC VÀ ĐÀO TẠO Chỉỉdren'5 Fim Maths Joumal NHÀ XUẤT BÀN GIÁO DỤC VIỆT NAM - BỘ GIÁO DỤC VÁ DÁO TẠO HỘI ĐỔNG BIÊN TẬP Tổng btẽn tập: ThS. vũ KIM THỦY Thư kí tòa. soạn: T rưởng ban biên tập: NGUYÊN NGỌC NÀN TRẤN THỊ KIM CƯỠNG NGND.VŨHỨU BÌNH TS. GIANG KHẮC BÌNH TS. TRẨN DÍNH CHÂU TS. VŨ ĐÌNH CHUẨN TS. NGUYỄN MINH ĐỬC ThS. NGUYỀN ANH DŨNG TS. NGUYẺN MINH HẢ PGS.TS. LẾQUÒC HÁN PGS. TSKH. VŨ DÌNH HÒA TS. NGUYỄN Đửc HOÀNG ThS. NGUYỄN vũ LOAN NGUYỄN ĐỨC TẤN PGS-TS- TÔN THÂN TRƯƠNG CÓNG THÀNH PHẠM VĂN TRỌNG ThS. Hổ QUANG VINH TÒA SOẠN Tắng 5, sổ 361 đưởng Trường Chinh, quặn Thanh Xuân, Hà Nội Điện ihoại (TelJ: D4.3S6tì2701 ĐiỀn sao (FaK): 04.356Ẽ2702 Dfện thư(Emaíl): toantuoFtho@vnn.vn Trang mạng (VVeòsite}: //www.loantuoitho.vn ma NGUYÊN VIẾT XU An 55/12 Trần Đĩnh Xu, p. cẩu Kho, Q.1, TP. HCM ĐT: 0B.66B21199, DĐ. 0973 30B199 Trị sự - Phát hành. TRỊNH THỊ TUYẾT TRANG, VŨ ANH THƯ, NGUYẺN HUYỂN THANH Chể bản: Đỏ TRUNG KIÊN Mĩ thuật: TÚ An CHỊU TRÁCH nhiệm XUẮT bản Chù lỊch HỘI aốno Thánh viên NXBGD việi Nam; MẠC VÃN THIỆN Tống Giâm (lóc NXBGD việt Nam: GS. IS.VŨ VAN HÙNG Phá Tong Glãm dửc Kiêm Tống hiên lặp NXBGĐ Vlậl Nam; IS. PHAN XUÂN THÀNH TRONG SỐ NÀY Danh sách học sinh đoạt giải Thi giải toán qua thư(Năm học 2015-2016) Dành cho học sinh lóp 6 & 7 Các dạng toán vể đoạn thẳng Lrong hình học lớp 6 Võ Xuân Minh Học ra sao? Giải toán Ihếnảo? Sừ dụng phương pháp đánh giá để giải hệ phương trinh Lê Đữc Thuận, Cao Văn Dũng Do trí thõng mình Đố bạn biết hình nào, số nào? Nguyễn Đức Tấn Nhìn ra thếgiới Lài giải đề thi chọn đội tuyển dựlhi OlympicToản Quốc tế của Hồng Kỏng nãm 2010 (Vỏng 1) Mai Vũ Saỉ ỏ dâu? Sửa cho dúng Phương trình có nghiệm không? Phan Đình Ành CỬasoAC mt Tr27 20 năm học bổng ASEAN của Singapore cho học sinh Iâp9 Việt Nam Vũ Kim Thủy Phá án cùng thám tửsêlôccóc Kẻ khả nghi Bùi Phương Thảo Đèn vối tiêng Hán Bài 67. Tôi từthánh phố Hổ Chí Minh tứi (Tiếp theo kìtrưóc) Nguyễn Vũ Loan In J TRONG SỐ NÀY ] Tr 33 Tr 34 Học Toán bằng tiếng Anh Geometry Vũ Đô Quan Thách đấul Thách đâu đảyl Trận đấu thứ một trâm ba mươi bày Trần Bá Duy Linh Bạn dọc phát hiện Tìm nhanh lởi giải nhờ phát hiện trên hình vẽ có hai tam giảe đồng dạng và trung điểm của cạnh Nguyễn Đức Tấn Dành cho các nhà toán học nhỡ Sử dụng bất dẳng thức quen thuộc để giải bài toản cực trị hình học Lê Quốc Hán Chuyện dạy và học toán Khơi dậy khả nâng tự học và phát huy tỉnh sáng tạo của học sinh qua hoạt động nhóm ngoải giờ hạc Nguyễn Thị Binh Cuộc thi Vui chảo hè 2016 Dể thi các nưđc AMC 2015 Đỗ Trung Hiệu Câu lạc bộ Toán Tuổi ỉhú Câu lạc bộ Toán Tuổi thơ toàn quốc 2016 TTT Tr 50 Bạn muốn du học Phỏng vấn Vũ Kim Thư Com pa vui tinh Aì đúng? Nguyễn Đừc Tẩn Tr 52 Câu lạc bộ Dể hay khó Thử trả lời Toán học là gì? Vũ Kim Thủy Giờ ra chơi Vuì cười ỡỗ Hống Thịnh Trang thơ Chữ và chữ só Kì 23 Tr 54 Tr 58 Trương Cõng Thành TrƯòng ũlympic Hướng tới 50 nãm RECSAM Binh Nam Hà Rubic Hỏi... Đáp Đề thi giải toán qua thư Tr 63 DANH SÁCH HỌC SINH DOẠT GIẢI THI GIẢI TOÁN QUA THƯ Năm học 2015 - 2016 Giải Vảng: Đặng Quăng Anh, 9A, THCS Nguyễn Chích, Đỏng Sơn, Thanh Hóa; Lê Ngọc Hoa, 8EỊ THCS Vĩnh Tưởng, Vĩnh Tưởng, Vĩnh Phúc; Tạ Nam Khảnh, 8E1, THCS Vĩnh Tường, Vĩnh Tường, Vĩnh Phúc. Giảí Sạc: Nguyễn Minh Nghĩa, 9B, THCS Nguyễn Thượng Hiền, ửng Hòa, Hả Nội; Nguyễn Văn Thanh Sơn, 8/1, THCS Nguyễn Khuyến, Đả Nắng; Trần Quốc Lập, 9A3, THCS Lâm Thao, Lâm Thao, Phú Thọ; Phạm Hiếu Ngàn, 7A, THCS Hoàng Xuân Hán, Đức Thọ, Hả Tĩnh; Bùi Thị Minh Thư, 7A, THCS Hoàng Xuân Hãn, Đức Thọ, Hả Tĩnh. Giải Đổng: Trần Thị Thu Huyền, 9A3, THCS Lâm Thao, Lâm Thao, Phú Thọ; Nguyển Hữu Trung Kiên, 8A3, THCS Lâm Thao, Lâm Thao, Phú Thọ; Tạ Kim Thanh Hiền, 7A4, THCS Yên Lạc, Yẻn Lạc. Vĩnh Phúc; Nguyễn An Na, 7A, THCS Hoàng Xuãn Hãn, Đức Thọ, Hà Tĩnh; Hoàng Mạnh Nghĩa, ?D, THCS Lý Nhặt Quang, Đô Lương, Nghệ An; Lẽ Đình Thành, 70, THCS Lý Nhật Quang, Đó Lương, Nghệ An; Cao Việt Hải Nam , 9E, THCS Đặng Thai Mai, TP. Vinh, Nghệ An; TừTẩn Dũng, 7D, THPT chuyên Hà Nội -Amsterdam, cấu Giấy, Hà Nội. Giải khuyến khích: Bùi Anh Vũ, 9B, THCS Vĩnh Tưòng, Vĩnh Tường, Vĩnh Phúc; Lê Thị Hằng Nhì, 7A, THCS Hoàng Xuân Hán, Đức Thọ, Hà Tĩnh; Nguyễn Ngọc Anh, 7A, THCS Hoàng Xuản Hẳn, Đức Thọ, Hả Tĩnh; Nguyễn Văn Cường, 8A, THCS Hợp Tiến, Nam Sách, Hải Dưđng, Nguyễn Sỹ Quyền, 7D, THCS Lý Nhật Quang, Đô Lương, Nghệ An; Nguyễn Sỹ Trọng, 7D, THCS Lý Nhật Quang. Đô Lương, Nghệ An; Phạm Hổ Thảo Nguyên, 908, THCS Nguyễn Nghiêm, TP. Quảng Ngãi, Quảng Ngãi; Nguyễn Minh Đửc, 8A1, THCS Nhân Chính,
  • Thanh Xuân, Hà Nội; Nguyễn Đức Hiếu, 7C1Ũ, THCS Trần Phú, Q. Lẽ Chân, Hải Phỏng; Lẽ Hồng Nhung, 7A. THCS Vĩnh Yên, TP Vĩnh Yên, Vĩnh Phúc; Đổ Thúy Hóng, SAI , THCS Yén Phong, Yẽn Phong. Bắc Ninh. TTT CÁC DẠNG TOÁN vi ĐOẠN THẮNG TRONG HÌNH HỌC LỚP 6 VỌ XUÂN MINH ịGV. THCS Nguyễn Ván Trỗi, Cam Nghĩa, Cam Ranh, Khảnh Hòa) Bài này giới thiệu một số dạng toàn ựề đoạn thẳng, tia. dường thẳng thưởng gặp trong chương trình iởp 6 dể học sình hiểu rõ hơn khái niệm đoạn thẳng và điểm nằm giữa hai diểm.
  • Xác định điểm nằm giữa hai điếm • Nếu AB + BC - AC thì B nằm giữa A và c • Trên tia Ojf nều OA < OB thì A nằm giữa o và B. Ví dụ 1. Vẽ hai tia đổi nhau OA vá OB sao cho OA < OB. Lấy M là trung dìểm của OA và N lè trung điểm của MB.
  • Hải trong 3 điểm o. N, B điểm nào nằm giữa hai diểm còn lại?
  • Lấy điểm c sao cho MB + BC = MC. Hỏỉ trong 3 điểm N, B, c điểm nào nằm giữa hai điểm còn lại? Lài giải, a) ví o nầm giữa M vá B nén BO + OM = BM mà OB > OA > OM nên BO > 2 A M 0 N B c VI N là trung điểm của BM -J BN BO >BN. 2 Trèn tia BA có BO > BN nên N nầm giữa 0 và B.
  • Vi MB + BC = MC nẻn B nằm giữa M và c mà M và N cũng phía dối vdi B => N và c khác phía đỗì vdì B hay B nàm giữa N và c.
  • Tính độ dài đoạn thẳng • Nếu B nàm giữa Avà c thì AB + BC - AC. Vì dụ 2. Vẽ AB = 1 1 cm. o là một đỉểm thuộc đoạn thầng AB sao cho OB = 7 cm. Lấy trung điểm M của NB đoạn OA, Trẽn đoạn MB lấy N sao cho MN = — . _ 2 Tính AN và ON. I 1 1 — I 1 A M o N B Lởi giải. Ta có AO =AB - OB = 11 - 7 = 4 (cm> AO =MA=MO=p = 2 (cm). MB = AB - AM = 11 -2 = 9 {em} Lại có MN + NB = MB và MN = nẻn 2 MN = = 3 {em}; AN = AM f MN = 2 + 3 = 5 (em). 3 Như vậy trẽn tia MB cỏ MO < MN nên o nằm giữa M và N suy ra ON = MN - MO = 3-2 = 1 (cm).
  • Tim SỂ đoạn thẳng của hĩnh đã cho trước Ví dụ 3. Trong hình bên có bao nhiêu đoạn thẳng? A Lởi giải. Ta đẽm được 9 đoạn thẳng mà mỗi đoạn thẳng có đúng 3 điểm. Như vặy r sồ đoạn thằng 3 2 được tạo thành từ 9 đoạn thẳng đó là --9 = 27 {đoạn thẳng). 2 Có 3 đoạn thẳng mà mồì đoạn thầng có s điểm nên SỂ đoạn thẳng được tạo thánh từ 3 đoạn thẳng 5.4 . này là —ị '3 - 30 (đoạn thâng), Vậy hĩnh đã cho có 27 - 30 57 {đoạn thẳng),
  • Chủng minh tỉnh chất vồ đoạn thẳng Ví dụ 4. Vẽ đoạn thẳng AB và M là trung điểm của AB Lạy điểm c nầm giữa A và M, lấy điểm D sao cho A nầm giữa c vè D, chứng minh rằng: . DAtDB 2
  • CM + DM - AD - BC. Lởi giải. I 1 1 1 1 D ACM B
  • Vi DM - DA-i- AM và DM - DB - MB nên suy ra DM + DM = DA + DB + M A - MB. <-> 2DM = DA I DB. Vậy DM = DA + DB 2
  • VÌCM-BC-MB vả DM-DA + AM nẽn suy ra CM + DM . = DA+ BC + MA- MB. VậyCM I DM = AD K BC.
  • So sánh đoạn thẳng Vỉ dụ 5. Trên đường thẳng a, lấy 4 diểm thea thứ tự A, B, c, D sao cho AB < CD. Gọi M là trung điểm của AB, N lả trung điểm cùa CD. Hãy so sánh MN, AC và BD. Lòi giải. M B N D , AB CD Ta cổ MA - MB - pp, NC - ND - -=f- 2 2 vả AB <CD nên MA =MB <NC =ND. Mà AC = AM + MC và MN = NC + MC dữ đó AC < MN. Ta có MN = MB + BN, BD = ND + BN và MB < ND nén MN < BD. Vậy AC < MN < BD.
  • Chứng minh một diểm là trung điểm của một đoạn thẳng Phương pháp. Để chứng minh I là trung điểm của đoạn thầng AB ta có các cách chứng minh sau: • Chứng minh I nằm giữa A và B và IA = IB. AB • Chứng minh IA * IB - VI dụ 6. Vẽ đoạn thẳng AB = 3 cm. Trẽn đoạn thẳng AB lấy các điểm c, D, E sao cho AC - 2 ciiri, BD = 3 cm, BE = 4.5 cm. Chứng minh rằng D là trung điểm của đoạn thẳng BC và E là trung điểm của đoạn thảng CD. Lởi giải. A c E D B Vì điểm c nằm giữa hai điểm A và B nên BC = AB -AC = B-2 = 6(cm). Trên tia BA có các điểm c. D r E thỏa mãn BD < BE < BC nẽn D nằm giữa B và E; D nằm giữa B và C; E nằm giữa B và C, suy ra E nằm giữa c và D BC • Vì D nằm giữa B và c và BD - nẽn D là trung điểm của BC. 2 • Ta có ED = BE - BD = 4,5 - 3 = 1.5 (cm); EC = BC - BE = 6 - 4,5 »1,5 (cm). Vì E nằm giữa c và D và EC = ED nên E lã trung điếm của đoạn thẳng CD.
  • Chững minh hai doạn thẳng có cũng trung điểm Phương pháp. Lấy trung điểm của một đoạn thẳng rói chửng minh điểm đó cũng là trung điểm của đoạn thẳng thử hai. Ví dụ 7. Vẽ hai doạn thẳng bằng nhau AB = CD sao cho c nằm giữa A vá B; B nằm giữa c vả D. Chứng minh rằng AD vã BC có trung điểm trùng nhau. Lởi giải. Ẳ 0 i B ò Vì c nằm giữa A và B nẽn AC - AB - CB. VI B nằm giữa c và D nên BD = CD - CB, Mà AB = CD nèn AC = BD. Gọi I là trung điểm của BC thi IC = IB. Lừ dó AC + IC = BD + IB. Suy ra IA = 10, từ đó I là trung điểm của đoạn thảng AD, Vệy AD vã BC cùng cố trung điểm lã I,
  • Dựng doạn thẳng Ví dụ 8. Chữ trước đoạn thầng AB cỏ đọ dài d cm. Hãy dựng một đoạn thẳng có độ dài bằng -ịd +■ 3 (cm), 4 Lởi giải. A c D B E • Dựhg trung điểm c cùa AB. • Dựng trung điểm D của CB. • Trên tia dổi cúa tia BA dựng diểm E sao cho BE = 3 cm thì DE = -7 AB + 3 cm. 4 Bài tập tự luyện Bài 1. Vẽ doạn thẳng AB. lấy điểm c trén doạn thẳng AB sao cho AC > 2CB, Gọi M là trung điểm của AC f N lã trung điểm của MB.
  • Hỏi trong 3 điểm M, N. c điểm nào nằm giửa hai điểm còn lại? Vì sao?
  • Chọ AB = 3 cm t CB = 2 cm. Tính NC. Bài 2. Vẽ đoạn AB - 12 cm và lẩy c trên đoạn thẳng AB sao cho AC - 4 cm. Gọi M là trung điểm của BC.
  • Chứng minh rằng c lã trung điểm của đoạn thảng AM.
  • Chứhg minh rang CM vả AB có trung điểm trùng nhau. Bái 3. Trán dường thẳng a lẩy theo thử tự 4 điểm A, B, C, D sao cho AB = CD. Gọi I và K lần lượt lá trung điểm của các đoạn thảng AB và CD,
  • So sánh độ dài các đoạn thầng AC, IK và BD.
  • Chửhg minh rằng BC r IK vả AD có trung điểm trùng nhau. Bài 4. a) Cho n điểm phản biệt (n là sổ nguyên lân hdn 1). Nối n điểm đó dõi một với nhau thì được bao nhiêu đoạn thẳng? tì) Muốn có 4350 đoạn thẳng thì n bằng bao nhiêu? Bài 5. Cho trước đoạn thẳng AB có độ dảl d cm (d > 8). Hãy dựng một đoạn thảng cỏ độ dài bằng ^d-2(cm). 5 sử DỤNG PHƯƠNG PHÁP ĐÁNH GIÁ DỂ GIẢI HỆ PHƯƠNG TRÌNH LẺ ĐỨC THUẬN, CAO VĂN DŨNG (GV. trường THPT chuyên Hà Nội - Amsterdam) Hệ phưững trình ià một dạng toàn thường gập trong CSC ki thi chọn học sinh giói, thi tuyển s inh vào THPT. Có nhiêu cách đề giải hệ phương trình khác nhau, trong bài viềt này, chúng tôi xin dưa ra một vài kiểu đành giá đẻ giải một sổ hệ phương trình . Ví dụ 1. Giải hệ phương trình jxy + V2(x 4 + 7 ) = 3 (D |x 5 y-xy s = 2 (2)

    Lởi giải. Từ (2) ỉa có 0 < 2 = x 5 y 4 xy 5 = xy(x 4 ì y 4 ) s> xy > D vã X 4 + y 4 =-?r p> xy Thay (3) vào (1) ta được xy + —p— = 3. v*y Theo bất đẳng thức AM-GM, ta cỏ 3 = xy + = xy 4 -= + -== ựxy yxy v*y 33|Xy,— = 3, V v*y v*y Từ đó hệ phương trình đi cho tương dương vối 1 1 xy =- Cộng theo vế hai phướng trình của hệ ta được:

    5-x 2 +^5 — J +Ịx+-l = y 2 -2y +7. (3) Theo bất đẳng thức Bunhiacốpxkỉ, ta có <(1 + 14-1) (5-x")+|5-— 1 ì 2' X +— ; ị = 36. x/xỹ -Jxy »x = y = ±1. Mặt khác y 2 - 2y I 7 = (y - I) 2 + 6 £ 6 Vậy hệ phương trình dã cho tương dương vâi Vs-X z = j5--l = x + 1 \ ! X 2 X tí x = y = 1. y 2 -2y + 7 = 6 Nhận xét. Học sinh có thế giải bằng cách binh phương hai vẽ của phương trình (1) rồi lẩy phương trình (2) thế vào nhưng tời giải sẽ dài vé phửc tạp. x 4 +y 4 =• 2 Nhặn xét. Học sinh cò thể đạt Vỉ dụ 3. Giải hệ phương trình ]3x 3 y-2x 3 =-6 [xy 3 +2x a = xy m 1 ! \b = i hệ phương trinh dã cho về dạng Lởi giải. 4 để đưa TH 1: X = 0, thay vào hệ phương trình đã cho thì không tổn tại y thỏa mãn, • TH 2: X 0, chia cả hai vê của hai phương trình X +y |a + b%/2=3 lời giải sẽ dài và phức tạp , Ví dụ 2. Giải hệ phương trinh 1 _ . - x + - = 4-2y X [ab 2 - 2 (1) m nhưng của hệ đã cho lần lượt cho X 3 và X ta dược ; 3y - 2 fíf ? +m Ệì Ị. ,1 «/ 1 l Cộng theo vế của hai phương trình trên, ta được 1 y 3 +3y = Lửl giải. DKXD -j= < |x| < S- 45 2 \ 3
  • ff) Xét hàm số ỉ(z) = z 3 + 3z. © Với a > b thì f(a) - f(b) = (a 3 + 3a) - (b 3 + 3b)
  • (a 3 - b 3 ) + 3(a - b) - (a - b)(a 2 + ab - b 2 + 3) > D. Suy ra nếu F(a) = f(b) thl a = b. Từ đỏ f{y) = f [ ) 2 y =~ 5 - X Ta có X 3 -——2 |=-0«x J +3x-4 =0 <=> X = -2 y = 1 X = 1 => y = -2. Ví dụ 4. Giải hệ phương trinh I y = -X 3 +3X + 3 I X * 2y 3 + 6y 2 -6. Lởi giải. Biển dổi hệ phương trinh đã cho về dạng Jy-1 = -£X-2)(X+1) 2 (1) |x-2 = 2(y-1)(y + 1) 2 (2). • THI . Xét X > 2, từ (1 ) suy ra y < 1 , từ {2) suy ra X <2 (võ lí) • TH2 Xét X < 2. từ (1 ) suy ra y > 1 , tứ (2) suy ra X > 2 (vỏ lí), • TH3. Xét X - 2, từ (1) suy ra ý = 1 , thay vào (2) thỏa mãn. Vậy hệ phương trình cỏ nghiệm duy nhất là (x; y) = ( 2 ; 1 ) Nhận xét. Dể tỉm được lờì giải bài toàn này cần nhẩm được nghiệm (x; y) (2; 1). Vi dụ 5. Giải hệ phương trình |x 3 +9y 2 =17 (1) |x 2 -3xy + 9y 2 -9y =0 (2) Vi dụ 6. Giải hệ phương trinh í 1 1 2 \t 2 -ĩ + 7 ± J = T7- í 1 ) ị 1 f X 2 1 I y 2 1 + x y ( 2 ). Lời giải. ĐKXĐ ;o < X <1 [0 < y < 1 • Bổ để. Với a, b là các số thực dương thuộc 1 12 [0; 11 thl -1-+-L-S-L-. 1 + a 2 1 + b 2 1 + ab Thật vậy. bất đằng thức tương dương vói (a-b) 2 (ab-1) 1
  • < 0 < i ■ <0 í+a z 1+b 2 1+ab (1+a z }(l+b 2 }(1 + ab} (luôn đúng với a, b c [0; 1]). Đảng thức xảy ra khi a = b, Vậy bổ để dược chứhg minh. Từ (1) và bồ đề ta có X = y, thay vào (2) ta được 2Vx(1-x) = 1ox=|y A Vậy hệ phương trình đã cho cố nghiệm duy nhất £x; y) = Lởi giải. Phương trinh (2) là phương trình bậc hai ẩn X có nghiệm khi A„ = 9y 2 - 36y 2 + 3Gy > 0 íii 0 < y < 3 Phương trình (2) là phương trình bặc hai ẩn y có nghiệm khi Ày = -27x 2 - 54x + 81 > 0 <=* -3 < X < 1 . Do đỏ X 3 + 9y 2 <1 + 9.= 17. 9 4 Dấu bẳng xảy ra khi X - 1; y - T.. 3 Vậy hệ phương trinh đã cho có nghiệm duy nhất Bải tập tự luyện Bài toán. Giải các hệ phương trinh sau jx 3 -6x 2 + 12X-7 = y Ị-x 3 + âx 2 -19x + 11 = y 3 ; x + y + z = 2016
  • ■ x 2 +y 2 +z 2 = 2016 2 X 3 +y 3 4Z* = 2016 3 ; x E +xy 4 = y 10 4y 6 \'4x 4 5 + 1 + 8=6: Í2y 3 +y + 2xVT : 7 = 3VĨ^X lV2y 2 + 1+y = 4+Vx + 4; Bái 7NS. Tim các số nguyên X, y thỏa mãn 2x fi - y 2 - 2x J y - 320. HÀ VÃN NHÂN (GV, THCS Hoàng Xuân, Hoàng Hóa. Thanh Hòa) Bải 8NS. Trẽn bảng viết đa thức f(x) = X 2 + 3x 4 2. Ta thực hiện trò chơi sau: NỂU trén bảng có đa thức p(x) thì ta xóa bỏ đa Lhửc P(x) và thay vão đó là đa thức Q(x) - x z p(— + 1). Hòi sau 2016 bước làm như vậy X thỉ trên bảng có thể có đa thức g(x) - X 2 IDx + 9 hay không? Vì sao? LẠI THỊ MINH HOA (GV. THPT Nam Dòng Quan, Đóng Hưng, Thái Bình) Bài 9NS. Từ điềm A cố định nẳm ngoái đường tròn {O; R) Vẽ tìểp tuyến AB của đường tròn (O) (B là tiếp điểm). Cát Luyến ACD với c. D thuộc đưởng tròn (O) và c nằm giữa A và D sao cho tia AO nầm giữa hai tia AB, AC. Tĩểp tuyến tại c của đường tròn (O) cắt AB tại E. Ttf Ẽ kẻ đường thẳng vuông góc với OA tại F. Chứng minh rằng đường thẳng CF luôn đi qua một điểm cố định khi cát tuyến ACD di động. NGUYỄN ĐỬC TẤN (TP. Hổ Chl Minh) CUỘC ĨHI GIẢI Tonn DÀÍIH CHO nữ sinH (TTT2 số 157} Bài 1NS. Ta có (x z 4 4y z + 28} z = 17(x 4 + y 4 + 14y 2 4 49) < > [x z 4 4(y 2 + 7)Ỹ = 1 7Ịx 4 + (y z + 7) z ] 1 6x 4 - 8x 2 (y 2 4 7) 4 (y 2 4 7) 2 = 0 <-> [4x 2 - (y z 4 7)] 2 = 0 <=ĩ 4x 2 - y 2 - 7 = 0 « (2x 4 y)(2x - y) = 7. Vi X, y nguyền dương nén 2x 4 y > 0 và 2x I y > 2x - y. Do dó 2x 4 y = 7 và 2x - y = 1 . Vậy X = 2, y - 3. Nhận xét. Các bạn Ihani gia giải bãi đều giải đúng. Các bạn sau có lời giải tốt: Chu Thị Thanh, 8E1, THCS Vĩnh Tường, Vĩnh Tưởng, Vĩnh Phủc; Lưu Thị Phí/ong. Phạm Thị Thùy Trang. Ngò Thị Thanh Trúc, 8A1 t THCS Từ Sơn, Từ Sơn, Bắc Ninh: Nguyễn Ngọc Huyền, 9A, THCS Hùng Vương. TX. Phú Thọ. Phú Thọ; Nguyễn Thị Thảo Vy, 9Ã, THCS Đặng' Thai MaL TP, Vinh, Nghệ An; Đặng Thị Hoải Anh, 9B, THCS Nguyễn Thượng Hiển, ừng Hòa; Bủì Hương Giang, 71, THCS Lé Quý Đòn, Cầu Giấy, Hà Nội; Hoảng Hả My, 8A r THCS Chu Vãn An, Nga SỚn. Thanh Hóa. Bãi 2NS. Đạt t = X 2 - 2x > -1 thì phương trinh đã cho trỏ thành t 2 - 201 3t 4 2015 - 0. (1) Phương trinh (1} có A - 20 1 3 2 - 4.1.201 5 > 0 nén có 2 nghiệm t 1 , t 2 vái (t - t 1 ){t - 1 = 0. Do t^ = 2015 > 0, 4 = 2013 > 0 nén 2 nghiệm phản biệt t r L, > 0, Từ cách đát t có x z 4 2x - 1, - 0. (2) X 2 4 2x - 1 2 = 0. (3) Các phương trình (2), (3) có A 2 > 0, ứ 3 > 0 nên © phương trinh (2) có 2 nghiệm x v Xjj, phương trình (3) có 2 nghiệm x 3 , x 4 , Do dó phương trinh đã cho có 4 nghiệm x v Xg, x 3 , x 4 - Áp dụng định lí Vi-ét vãi phương trình (2), ta được xj + x 2 = -2 r x 1 x 2 = -t 1 . Suy ra X.J 2 + x 2 2 - (Xf 4 2 f - 2x.|X 2 - 4 4 2t|. Tương tự, vởi (3) ta có Xg 2 4 x 4 2 = 4 \ 2t 2 . Do đó X,, 2 4 x/ 4 x 3 2 4 Xj| 2 - 8 4 2(1, 4 t 2 ). Áp dụng định lí Vi-ét vãi phương trình(l), ta được t 1 4 t 2 = 2013. Vặy X , 2 + X 2 Z 4 Xj 2 + X 4 Z = 4034. Nhận xét. Các bạn sau có lới giãi đúng: Chu Thị Thanh, 8E1, THCS Vĩnh Tưởng, Vĩnh Tường, Vĩnh Phúc; Lưu Thị Phương, 8A1, THCS Từ Sơn, Từ Sơn, Bắc Ninh; Đàng Thị Hoài Anh, 9B. THCS Nguyển Thượng Hiển, ứng Hòa, Hà Nội; Nguyễn 77 j/ Thảo Vy.SA, THCS Đặng Thai Maí, TP. Vinh, Nghệ An; Nguyễn Ngọc Huyền. SA, THCS Híing vữơrig, TX. Phu Thọ, Phú Thọ. Gọi M là giao điểm cùa BD và AC. Nối OB. BE. CẺ, CD, OE. Ta có HB = HC, AO BC, OB AB. Trong lam giác vuông ABO la có AB = AH-AQ. (1) Ta c6 AABD co AAEB (g.g) = = AE AB EB Suy ra AB 2 = AD AE. (2) AC BD (vì AB - AC). (3) AE BE Ta cố AACD co AAEC (q.g) => = (4) AE CE Từ (3) r (4) suy ra ~ = -. (5) bsh bt Vì ABCM có H là trung điểm của BC. DH li MC nén BD = DM. (6) Từ (5), (6) suy ra BE CE Mà MDC = ẺEC (vi lứ giác BDCE nộị liếp). Do đó ADMC 00 AEBC (c.g.c). Suy ra DCM = ẺCB, Kết họp với DCM = DBC có ẼCB = DBC, suy ra BD II CE. Suy ra tứ giác BDCE lè hĩnh thang. Mà tử giác BDCE nội Liếp nên tứ giác BDCE lá hình thang cân, suy ra BG - DE. (7) Từ (1), (2) suy ra AD.AE = AH.AO. Suy ra AD AO AH " AE với R là bán hình đường tròn ngoại tiếp AODIH. Từ (7), (8} suy ra BC < 2R. Mã BC = 2BH nẻn BH < R Nhận xét. Bài toán này khó, không có bạn nào giải được. ;-.| 0 „ Các bạn sau dược thướng kì này: Chíl Thị Thanh, 8E1, THCS Vĩnh Tường, Vĩnh Tường, Vĩnh Phúc: Lưu Thị Phương, Phạm Thị Thúy Trang. Ngõ Thị Thanh Trúc, SAI, THCS Tử Sơn, Từ Sơn, Bắc Ninh; Đặng Thị Hoài Anh, 9B, THCS Nguyễn Thượng Hiển, ứng Hòa; Búi Hương Giang, 71. THCS Lẽ Quỷ Đôn, cầu Giãy, Hà Nội: Ngưyễn Thị Thảo Vy, 9A, THCS Đặng Thai Mai, TP. Vinh, Nghệ An; Nguyên Ngọc Huyền, 9A, THCS Hùng Vương, TX. Phú Thọ, Phủ Thọ. Ảnh các bạn đưdc thương ở bia 2. NGUYỄN HIỆP Do đỏ AADH co AAOE (c.g.c). Suy ra ÂDH = ẤOE. Do đó tứ giác DEOH nội tiếp, từ đó DE < 2R. (8) . Các bạn dược thưởng ki nãy: Vu Hải l2ĩ£ Trúc. 6A1, THCS Nguyễn Đăng Dạo, TP. Bầc Ninh, Bắc Ninh; Đường Minh Quân, 7C, THCS Bạch Liêu, Yên Thành, Nghệ An; Trển Đan Trưởng, 7A, THCS LýTựTrọng, Bình Xuyên. VTnh Phúc; Lẽ Đức Nam , 7A2, THCS cầu Giấy, Cầu Giấy. Hà Nội; Bùi Đức Anh, 8C1, THCS Tỏ Hiệu, Lế Chán, Hải Phỏng LỀ THANH Tủ ỊEP (TTT2 SỐ 157) THỀ CỜ (Kì 80 ) 1.«xg7+ &xg7 2.if6+ ầm 3.ôh7# LỚP 6 DÊ KIỂM TRA HỌC KÌ II MÔN Thời gian làm bái; 90 phút (khùng kể thời gian giao đề)
  • Phẩn trẳc nghiệm khách quan (3 điểm, mỗi cáu 0,5 điềm) Hãy ũhọn chữ cái đủng trước cãu trả tài đúng tmng các câu sau: Câu 1. Sũ nghịch đào của -3- là 5 _5 t
  • — T B. c. -5-r D. -3. 17 13 2 1 3 Câu 2. Nếu — X t — X + 2 = 0 thì giá trị của X là 4 4
  • 1 B. -1 c. -2 D, -3. Cãu 3. ị của 3 thì bằng ĩ- của số 3 5
  • 3 B. 4 c. 5 D. 10 . 2 2 Câu 4. Mót hình chữ nhát có diện tích i- cm 2 vãi chiếu dái bằng — cm. Vậy chu vi của nó là 9 3 „1 1
  • 1 cm B. 2 cm c. -cm D. — cm. 2 3 Cẳu s. Tia Oz nằm giửa hal tla Ox và Oy nếu
  • xOy + yOz = xOz.
  • xOz - xQy - yOz.
  • Trên cùng một nừa mặt phằng bở chứa tia Ox có xOy > xOz.
  • Trén cùng một nửa mặt phầng bở chứa tia Oy có xOy < yOz. Câu 6. Cho tia Oy nằm giữa hai tla Ox và Oz biết xGy = 40°, xOz = 130° thì yOz lá
  • Góc nhọn B. Góc vuông c. Góc tù D. Góc bẹt. II. Phẩn tự luận (7 điểm) Bài 1. (1 điểm) Thực hiện phép tính A = -^- :1,5 + 1-1~“. Bài 2. (1 điểm) Tinh nhanh ữ = -rr--T- + Ạr‘Tz- + “J—. 13 11 11 13 11 Bái 3. fí r 5 điểm) Lớp 6A cố số hoc sinh nữ chiếm 62,5% sõ học sinh của cả lớp vã có 12 học sinh nam.
  • Tinh số học sinh của lửp 6A.
  • Tính tì sô phần trấm của học sinh nam đối với học sinh nừ. Bải 4. (2,5 điểm) Vẽ hai góc kề bù xOy và yOz sao cho xQy - 3Q“.
  • Tính ỹÒz .
  • Trên nửa mặt phầng bở xz chứa tia Oy vẽ tla Ot sao cho zÕt =120° . Hỏi Oy có là tia phân giác của xOt không? Tại sao?
  • Vẽ tia phản giác Om của zÕt. Tính mdy. Bài 5. (1 điểm) Tìm X biết — - J_ =-- + —5- + -- + ,,. + —^—. X 9999 1.3 3.5 5.7 97.99 THÁI NHẶT PHƯỢNG (GV. ĨHCS Nguyễn Vàn Trỗi, Cam Nghĩa, Cam Ranh, Khánh Hòa) ĐỀ KIỂM TRA HỌC KÌ II MÔN TOÁN LỚP 7 Thời gian làm bài: 90 phút (không kề thời gian giao dề) Câu 1. Hãy chọn cáu trả lời dùng ứhg vớĩ A, B, c hoặc □
  • 8 4 báng:
  • 2' 8 . 2 3
  • 2 13 1B ,„5 3 b, Nếu l x l+T“T thi X bằng: 6 4 -1 12 1 , — 1
  • -r hoặc —7 12 ■ 12 C- 12
  • 2
  • không tốn tại giá trị của X.
  • Tử tỉ lệ thức — = — (á. b, c, d $■ 0) la có thể suy ra tỉ lệ thức nào trong các tì lệ thức sau? b d a d
  • 77 = 7 b c a b B c d
  • Nếu a/ 2 x = 4 thì X 2 bằng:
  • 2 B. 9
  • Khấng định nào sau đảy là sai ? X 2
  • là một đon thức,
  • 0 là một đa thức a d C cb
  • 16 b c D ' ã~ d'
  • 64.
  • 2x 2 + 3x - 1 là một đa thừc
  • 1D 100 là một đon thửc bậc 100.
  • Cho hai đường thầng XX 1 và yy' vuông góc với nhau tại 0. Sô cặp góc vuông bằng nhau nhưng không đối đỉnh với nhau là:
  • 4 B. 3 c. 2 D. 1.
  • Cho hai đường thảng xx’ và yy’ cắt nhau tại o. sỏ' cặp góc đổi đỉnh lạo thành là: A.4 B. 3 c. 2 ‘ũ. 1. Câu 2. Điển dâu X vào ô thích họp: Câu Khẳng định Đủng Sai 3 Biết X và y là hai đại lưọng tỉ lệ nghịch và nếu cho X = -2 thỉ y = 6, Khi đỏ hệ SỂ tỉ lệ lã -3. b Bậc của đa thửc ỉ(x) = X 3 - 3x 2 - 1 + 2x - X 3 là 2. c Cho AABC - AA'B'C' biết AB - 3 cm, AC - 6 cm, B’C - 5 cm. Khi đó chu vi AABC bằng 9 Ctrl. Câu 3. Tinh giá tri các biểu thức sau
  • (|-j|(4 r 34+5 f 66) b) 1 ttírẪ c) Ệ- ■ I \ ■■ I 8 5 j 4 [2 ị) I 2 I Ys 1.3 2,4 3.5 96.100 Câu 4. Hai đội máy cày cỏ tọng sổ máy cày lã 27 chiếc (có cùng năng suất) lãm việc trẻn hai cảnh đồng có diện tích bằng nhau, Biết đội I hoàn thành cõng việc trong 4 ngáy, đội II trong 5 ngây. Tính sỏ máy cày của mỗi đội, Câu 5. Cho đa thức ỉ(x) = -X 4 - 3x 2 + 5x + 7 + X 3 + 2x 2 - 3x - 10.
  • Rút gọn f(x).
  • Chỉ ra bậc r hệ số tự do, hệ sổ cao nhất cùa f(x). Càu 6. Cho tam giác ABC cân tại A. Trên cạnh AB lấy điểm E. Trên tia đđi cùa tia CA lấy điểm F sao cho BE - CF. Nối E với F cát BC tại o, kẻ El song song với AF (I c BC), Chứng minh rằng:
  • Tam giác BEI cân tại E. b) OE = OF. LẠI QUANG THỌ (Phòng Giáo dục vã Đào tạo Tam Dương , Vĩnh Phúc) DỀ KIỂM TRA HỌC KÌ II MÔN TOÁN LỚP 8 Thòi gian làm bài: 90 phút (không kể thời gian giao đề)
  • Phẩn trác nghiệm (3 điểm) Cảu 1 . Chũ tam giác ABC vuông tại A, AC = 3 cm, 6C = 5 cm. Diện tích của tam gìàc ABC bằng: A- 6 cm 2 B 12 cm 3 C- 15 cm z D- 10 cm 2 Câu 2. iABC í> ADER biết Ả-80°;Ẻ -70°; F = 30° thì: Ạ. D = 120° B. D - 70° C-Ê-80° D- c - 30° Cãu 3. Cho AABC co AMNK theo tĩ số đống dạng là 2 và AMNK co AHEF theo ti sổ đống dạng là 3. Vạy AABC co AHEF theo tỉ sổ đổng dạng nào dưới đây;
  • 6 B, 5 c.ị D.ị 2 3 Câu 4. Nếu a < b và c < 0 thi:
  • ac < bc B. aobc c. ac>bc D. ac = bc

    Câu 5. Điểu kiện xác định của phưcíng trinh

    ỊỊ + = 0là: x-3 2 + x
  • X # -ĩ B. X 3
  • X ? -3 và X -2 D. X 3 và X —2 Câu 6. Cho tam giác ABC có AD là phân giác, khi đó ta có: DB AD A ' DC AC c DB AB ° DC AC Câu 7. Phương trinh ĐKXĐ là: B DB AC ' DC AB □ DB AD Ư ' DC AB X + 2 X 5 , 5 X - 3 x+7
  • X 3; X -7
  • X 3; X 7
  • X 5
  • X 5; X 3; X * - 7. Câu 8. Bất phương trình nào sau đây là bất phương trình bậc nhất một ẩn?
  • -X + 2S0 2
  • Qx + 5 >0
  • 2x 2 + 3 > 0 1
  • >0 2x + 1 Cảu 9. X = 2 là nghiệm của phương trình nào sau đây:
  • 3x - 6 - 0
  • x 2 = 2 c 3x = 6
  • 2x-2 = 0 Câu 10. Một hinh lệp phương có thể tích là 1 25 cm 3 . Diện tích xung quanh của hình lập phương đó lả;
  • TŨ0 cm 2 B, 20 cm 2 c. 25 cm 2 D. 150cm 2
  • Phẩn tự luận (7 diêm) Bải 1. Giải các phương trình sau: 5x-2 , - 5-3x.
  • + x -1 + — ; 3 2 x-1 x + 1 . 2 2 2x
  • — — +- 0, 2x-6 2x + 2 (x + 1).(3-x)
  • |2x + 1|-5x =x +2, Bải 2. Giải bất phương trinh sau vã biểu dién tập nghiệm trên trục so; . X + 2 . X, a — — — 1 > 2X-I--3-; 3 2 5x + 3 9x T 2 7 — 3x
  • —7 . 4 5 8 Bài 3. Hai xe cùng khôi hành mật lúc từ hai địa điểm A và B cách nhau 70 km, đi ngược chiểu nhau sau một giò thì hai xe gặp nhau. Tính vận tốc của mỗi xe. Biết rằng xe đi từ A có vặn lốc lơn han xe đĩ tư B lá 10 km/giờ. Bải 4. Cho tam giác ABC vuông tại A, biết AC - 4 cm, BC = 6 cm. Kè tia Cx vuông góc vãi BC (tia Cx vã điểm A khác phía so vơi dưàng thẳng BC). Lấy trên tiâ Cx điểm D sao cho BD - 9 om. chứhg minh rằng BD // AC. Bài 5. Hình bình hành ABCD có BC = 2AB. Gọi
  • N thứ lự lả trung điểm của BC, AD. Gọi p là giao điểm của AM vơi BN r ũ là giao điểm của Mũ với CN, K lá giao điểm của tia BN với tia CD.
  • Chứng minh tử giác MDKB là hĩnh thang.
  • Tứ glac PMQN là hình gì? Vi sao?
  • Hình bình hành ABCD phải có thèm điểu kiện gi để PMQN là hình vuông? LẠI QUANG THỌ (Phòng Giảo dục vã Dào tạo Tam Dương. Tam Dương. Vĩnh Phúc) ĐỀ KIỂM TRA HỌC KÌ II MÔN TOÁN LỚP 9 Thời gian lãm bài: 90 phút í không kề thài gian giao đề)
  • Phần trắc nghiệm khách quan (3 điếm) Hãy chọn câu trả tời đúng ímg với A B. c hoặc D trong mỏi câu sau. Câu 1. Phướng trình -3) 2 = 2 tương đưdng với phương trinh
  • X 2 - 6x + 5 = 0 B. X 2 + 6x + 5 = 0 c. X - 3 = -2 D. X - 3 = 2. —3 Câu 2. Phương trình 2x 2 - (m + 1)x + m - 3 = Q có tích hai nghiệm bằng -Ỵ thì tổng hai nghiệm bằng
  • 1
  • 0. Câu 3. Hàm số V = ax 2 cỏ đỗ thị đi qua điểm (-1; -3) thi hàm sổ đó
  • Đóng biến khi X > 0 và nghịch biến khi X < 0. B. Đổng biến trên R.
  • Đồng biến khi X < 0 và nghịch biến khi X > 0. D. Nghịch biến trên R. Câu 4. Cho đường tròn (O, R) với dây AB = R\l2. Điểm M thuộc cung nhỏ AB (M A, M B) thì ẤMB bằng
  • 45° B. 90° c. 120° D. 135° Câu 5. Một dưởng tròn có chu vi là 6n cm r khi đó diện tích hình tròn đó là
  • 9 TI cm 2 B. 7 Ĩ cm 2 c. 36ĩt cm 2 0- 6 jt cm 2 Càu 6. Một tam giác có độ dãi ba cạnh lẩn lướt là 6, 8. 10. Quay tam giác đố quanh cạnh độ dài 8 thì diện tích xung quanh của hình tạo thành là
  • 64ti B. 80íi c. 48ji D. Kết quả khác. II. Phẩn tự luận Ợ điểm) Bải 1 . (1 điểm) Giải hệ phương trình J Vic - VỸ = 1 1 xy = 36 Bài 2. (1,5 điểm) Cho phương trình X 2 - (m - 3)x - m + 2 = 0. ()
  • Chứng minh phương trình (} luôn GÓ nghiệm với mọi m.
  • Tim hệ thức giữa hai nghiệm X 1 và x 2 không phụ 'thuộc vào m.
  • Tìm m thỏa mãn +xjj =7. Bài 3. (1,5 diềm ) Cho (P): y - ^x 2 và (d): y - X - m.
  • Tim m để {P) và (d) tiếp xúc với nhau, khi đó háy tính tọa độ tiếp điểm A.
  • Xác định điểm B E (P) thỏa mãn OA 2 - AB Z = 4 . 4 Bài 4. (3 điểm) Cho M là một điểm trên đường tròn (O; R) đưởng kinh AB. Tiếp luyến qua A cắt tia BM tại c.
  • Chửng mình rằng BM BC không đổi khi M di dộng trên (O).
  • Gọi I là trung điểm của MB. Chứng minh rằng tứ giác ACIO nội tiếp và ÃIB - BÕC.
  • Tm vị trí của M trẽn (O) để MA - MB lân nhất.
  • Khí B = 30°, tính phần diện tích tam giảc ABC ò ngaàí đưàng tròn (O). THÁI NHẬT PHƯỢNG (GV. THCS Nguyễn Vân Trỗi, Cam Nghĩa, Cam Ranh. Khánh Hòa) ĐỀ THI HỌC SINH GIỎI MÔN TOÁN LỚP 9 TP. HÀ NỘI Năm học 201 5 - 2016 - Thài gian làm bài: 160 phũt Bài 1. (5 điềm)
  • Cho ba số nguyên a, b, c, d thỏa mãn a 3 - b 3 = 2(c 3 - 8d 3 ). Chứng minh (a + b - c + d) chia hết cho 3.
  • Tim tất cà các sổ nguyên tố X sao cho (2* + X 2 } là số nguyên tố. Bài 2. (5 điềm)
  • Giâi phương trình \/2x 2 + 11x + 19 + \í'2x 2 +5x +7 = 3(x +2).
  • Tim tất cả các bộ ba số (x, y. z) thỏa mãn X + y + z = 3 1111 —+—+—= - X y z 3 X 2 + y 2 +z z = 17. Bải 3. (3 điểm) Jg -s/s 3
  • Cho ba sô’ X, y. z thỏa mãn 0 < X < 2 ' ũ < ỵ < 2 ' 0 < z < 2 x y + y z+zx = Tim giảtiị nhỏ nhất cùa biểu thức p = — 3 -4x 2 3-4y 2
  • Cho a, b r c lã độ dài ba cạnh cùa một tam giác. ,2010 ,2016 Chứng minh b+c-a c+a-b a+b-c ,,2016 c ,> a 2016 + b 2O16 Bài 4. (6 điềm) Cho tam giác đểu ABC cạnh bằng a. Lấy điểm Q bất kì trên cạnh BC (Q khác B, Q khác C). Trên tia đối của tia BA lấy điểm p sao cho CQ.AP = a 2 . Gọi M là giao điểm của AQ vá CP-
  • Chứng minh bốn diểm A, B, M, C cùng thuộc một đường tròn.
  • Gọi I, J, K lần lượt là hình chiếu của M trẽn AB, BC, CA.
  • Xác dịnh vị tri cùa Q dể độ dái IK lứn nhất.
  • Chứng minh MI 2 + MJ 2 + MK 2 khăng đổi khi Q thay đổi trên cạnh BC. Bài s. (1 điểm) ChD bảng ò vuông kích IhƯủc 10 X 10 góm 100 6 vuông kích thước 1x1. Điền vào mồi ô vuông của bảng một số nguyên dương không vượt quá 10 sao cho hai số được điền ở hai ỏ vuông chung cạnh hoặc chung đình thi nguyên lố cùng nhau Chửng minh trong bảng ó vuông đã cho có mọt số xuất hiện ít nhất 1 7 lán. 42 3-4z z
  • c 2015

    ĐẶT MUA TẠP CHI CẢ NÃM HỌC TẠI CÁC cơ sở BƯU DIỆN TRONG CẢ NƯỚC IHÃ ẤN PHẨM: c 169.1 -4 1 7 8 ? NGUYỄN Đửc TẤN (Ti R Hỗ Chi Mình) thiếu <asm ĐO BẠN BIET HĨNH NAO, so NAO? Bài 1. Trong các hình sau. hình nào không phù hợp với các hình còn lại. SÔ TIÊP THEO {TTT2 số 157) Nhặn xẻt. Cả hai bài kì này đều không quá khó, nhưhg vin có bạn tìm sai quy luật, Quy luật. Bài 1. Xét dãy số 1: 2; 4; 7; 8; 11; 13; 14; 16; 17; ... Đây là dãy các số tự nhiên tăng dần mà các số hạng không chia hết cho 3 vả không chia hết cho 5. Vậy số hạng tlẽp theo của dãy là 19. Bài 2. Quan sát từ trái sang phải, khi chuyển sang hình vuông tiếp theo, mỗi sđ ở đình hình vuông dịch chuyển sang đỉnh liền kể Ihea chiều quay của kim đóng hó, đỏng thài nỏ được tâng thêm 3 đơn vị. Do vậy ? = 7 I 3 = 10. Một số bạn còn tim ra quy luật: Tổng các sấ ở hai đỉnh đbi diện của mỏi hình vuông hdn kém nhau 2 đdn vị, cũng đúng nhưng cấn nêu cụ thể hdn. ịữ Xin trao thưởng cho các bạn: Nguyễn Trung Thể, SAI , THCS Mai sốn, thị trốn Hát Lót, Mai Sdn, Sdn La; Lê Thị Thu Thài, 9E1, THCS Vĩnh Tưởng, Vĩnh Tưởng, Vĩnh Phúc; Nguyễn Thị Ngọc Tràm , 6A1 , THCS Hổng Bàng. Q. Hổng Bàng. Hải Phòng; Lẽ Xuàn Hoàng, 6A, THCS Dăng Thai Mai, TP. vinh; Nguyễn Thị Báng Băng, 7C, THCS Bạch Liêu, Yên Thành, Nghệ An. Các bạn sau dược tuyên dương: Phan Quang Huy, Diêm Đăng Hoàng, 8A1, THCS Mai Sdn. thị trấn Hát Lót, Mai Sdn, Sdn La: Nguyễn Hoàng Đạo, Đưòng Minh Quân. Chu Minh Hiếu, 7C, THCS Bạch Lièu, Yèn Thành, Nghệ An. NGUYỄN XUÂN BÌNH

    LỜI GIẢI ĐỀ THI CHỌN ĐỘI TUYÊN Dự THI OLYMPIC TOÁN QUỐC TẾ CỦA HỒNG KÔNG NÃM 2010 «.?' w (VÒNG 1) ÍĨÌTIĨIÍM
  • Chú ý rằng f(n) - n 3 - (n - 1) a . Điểu này dẫn đến f(1 ) + f{2) - ... + f(201 0) - (1 3 - o 3 ) + (2 3 - 1 3 )
  • ... + (201 Ò 3 - 2009 3 ) = 201 o 3 = 201 3 . 1000. Vì 20 1 3 có chữ so hàng đơn vị là 1. Vậy bốn chữ số tận cùng là 1 000.
  • Theo già thiéì n = 2010k vã có chữ sổ tận cùng lá 0 nên tất cả các chữ số của 201 k đều là lẻ. Ta phải có k > 150, nếu k < 100 thì khi đó chữ sổ hàng trăm của 201 k lả chẵn; nếu 100 < k c 150 thì chữ số hàng chục nghìn của 20 1k là 2, đó là số chẩn, Hdn nữa k cẩn phải lã số lẻ vr chử số hãng dơn vi của 201 k phải lá lẻ. Do đó ỉa thử 201.1 51 - 30351 . 201.153 = 30753 và 201.155 = 31155. Vậy n= 311550.
  • Ta xét các trường họp sau: • Nếu n có dạng 2 a , khi đó ta có n 5 2.1200 - 2400, và số nhỏ nhất n của dạng này lả 2 12 =
  • • Nếu n có dạng 3 b , khí đó ta có n > 3.1200 = 3600, và so nhỏ nhất n của dạng này là 3 tì = 6561. • Nếu n có dạng 2 a 3 b , khi đỏ ta cố n > 3.1200 =
  • sau một số phép thử ta tim dược sỏ nhỏ nhất n của dạng này là 2 4 .3 5 - 3383. • Nếu n chia hết cho một sổ nguyên tổ lớn hơn hoặc bằng 5, khi đó ta có n > 5.1200 = 6000. Kết hợp với các trưởng hợp chứng minh ở trên, đáp án lá n = 3888.
  • Do sơ suẩt, đế bãi thiếu quả bóng màu xanh lam. • Từ điều kiên "lấy ra 100 quả bóng bất kì trong 111 quả bóng thl đảm bảo có đủ 4 màu" o suy ra mỗi màu có ít nhất 12 quả bóng, Thực vậy, nếu một màu (chảng hạn màu đỏ) cỏ n quả, với n > 11 thì cỏn 111 - n > 100 quả vởi 3 máu trắng, xanh lá cay, xanh lam, ta chọn 100 quà đó sẽ chỉ có 3 màu (điều này trái vói giả thiết). • Số bóng cần tìm là N > 37. Chảng hạn nếu có 12 quà màu đỏ, 12 quả màu trắng, 12 quả màu xanh lam vả 75 quả màu xanh lá cây thl thỏa mãn đieu kiện () vì khi lấy 100 quả thì còn lại 11 quả. Trong 11 quả đó xét 1 màu (chầng hạn màu đỏ) MAI VŨ (Sưu tầm . dịch và giới thiệu) có n quả thi n < 1 1 , suy ra có 1 2 - n > 1 quả đã được lấy. Bảy gỉỡ lấy ra 87 = 75 i- 12 quả màu xanh lá cáy và màu đỏ thì trong số 37 quả đó không đủ 3 màu. • Ta chỉ ra rằng số bóng cẩn chọn ít nhất lả N -
  • Thật vậy, khi lấy ra 88 quả bóng thi còn lại 23 quả. Nếu thiếu 2 màu thì từ điều kiện [) phải còn lại ít nhất 2.12 = 24 quả bóng, trái vửi điều kiện còn 23 quả bóng Vậy đáp án là N = 83.
  • Ta có a - b > 1 và c - d > 1. De vậy 2010 = a 2
  • b 2 + c z - d 2 - (a + b)(a - b) + (c 4- d)(c - d) (a + b) + (c + d) - 2010, đẳng thức xảy ra khi ta cóa-b = c- d = 1. Điểu nãy dẫn đến (b + 1 ) + b I [d 1 1) í d = 2010, hoặc b 4 d = 1004, trong đó b va d lã các sô nguyên dưdng với b > d + 2. Do vậy ta có 501 cặp (b; d) là {1003; 1), (1002; 2), ... , (503; 501), và co 501 bọ 50 (a; b; c; d) là (1004; 1003; 2; 1), (1003; 1002; 3; 2) (504; 503; 502; 501).
  • Chú ỷ rẳng (x - I) 2 I 1 < P(x) < 2{x : - I) 2 + 1 đúng vãi bất ki sổ thực X. Do vậy ta cần phải có p{x) = B(x - 1) a + 1. 1 <a<2. TửP{11) = 181, ta có 181 = a(11 -I) 2 + 1 tức là a= g- Dẫn đến P(21)=|{21-1) 2 .
  • Giả sử n a 1 + a 2 + ... + a k (*) với aj là các sô' nguyên (i = 1 , 2 k) thỏa min 1 35 < ẫị < 1 44. Lúc đó 135k < n < 144k <-> — < k < 144 135 Tổn tạl sổ nguyên k > 1 khi n 4 , n i. n n , 135 144 135 144 <=> 9n> 135.144 »n> 135 16 = 2160. Từ đó la chỉ xét k < 15. • Vớt k = 15 thì n £ 135 15 = 2025. • Vối k = 14 thin £144.14 = 2016. Như vậy vởi 201 7 < n < 2024 thi n không biểu diễn được trong dạng (). Đáp án là n = 2024. 16 (Kì sau đăng tiếp) om PHƯƠNG TRÌNH có NGHIỆM KHÔNG? Bài toán. Giải phương trình: X 2 + 2^x 2 ỹ + y + 1 = 0 Một học sinh gì ái như sau: ĐKXĐ x 2 y > 0 y > 0. Phương trinh tương đương vâi <=> (Vx 2 + yfy) 2 + 1 = 0. Vì y > 0 nèn (ì/ X 2 + yỹ} 2 + 1 > 0. Do đó phương trình vô nghiệm. Theo bạn lởi giải trên đúng hay saí? PHAN ĐÌNH ẢNH (GV THCS Thạch Kim, Lộc Hà, Hà Tĩnh) SAI LẦM ở DÂU? (TTT2 số 1 57) Lòi giải đủng như sau: Tử giả thiết — + — = vả ac 0, a # b, b * c. a c b“C a-b e + a-b 11 1 1 a-b + e Suy ra — — — — = — — — - ■- (a-b)c a-b c b-c a (b-c)a , í 1 1 T Do đó ta có (c + a -b) - — — — — 1 = 0. I (a-b)c (b-c)a • Nếu - — — - — — — - — thì (a-b)c (b-c)a . . . ,, , a _ a b

    (a - b)c = (b - c)a w — = -. c b-c . a-b — c a • Nẽu c + a - b - 0 thì 7 = — (vì hai phân b — c a c sổ này khác 0 và khác dấu). Do đó f ~ b = — chỉ xảy rạ khỉ c + a - b / 0. b — c c Điều kiện này cần đưa vào giả thiết. Nhận xét. Một sổ bạn có cảg (rà lời đùng, nhưng khống thay c I a - b = 0 vào xem kết luận có thỏa mân hay không. Cách làm như vậy chưa đầy đủ vì có khi tích hai thứa sế bằng 0 và hai thừa số đó đống thãi bằng 0. Chẳng hạn. nếu cho ủ = 2c vả xét (c + a - b)(b - 3c) - 0.

    Các bạn sau có lời giải đủng: Nguyễn Chi ltf Cống , 7A3, THCS Lâm Thao. Lãm Thao, Phú Thọ: Lê Ngọc Hoa, SE1 , Lé Thị Thu Thái, 9E1, THCS Vinh Tương, Vĩnh Tưởng. Vĩnh Phúc: Nguyễn Quỹ Dương, SA2, THCS Nguyễn Đãng Đạo, TP. Bẳc Ninh; Trần Quang Tài, 7 AI. THCS Yén Phong, Yên Phong, Bẳc Nính; Lé Hồng Minh, 8A, THCS Bạch Liêu, Yên Thành; Võ Hùng Tuấn, 9A. TI ICS Đặng Thai Mai. TP. Vĩnh. Nghệ An. ANH KỈNH LÚP ĐỂ TH[ LỚP 6 CÂU LẠC BỘ TOÁN QUẬN HOÀN KIÊM, HÀ NỘI Năm học: 2015-2016 Thời gian làm bài: 30 phút Câu 1 .{5,0 điểm)
  • Tìm X nguyên biết rằng: 2 + {-3 - [-4 4 (-5 4 1*1)1} - 3 - [4 - (-5 4 0)J.
  • Tính giá trị cùa A biết rằng: A = 6 + (-16) 4 (-26) + 36 V 46 + (-56) + (-66) 4 ... + 1996 + 201 6 Câu 2. (5,0 ỡiểm)
  • Tìm số nguyên n thỏa mãn điều kiện (n 4 5) chia hết cho (2n - 1).
  • Tim X biết rằng 1 14— Ị- ị| 1 4— V ] 1 4——1.. { 1-3 A 2A k 35 / 14 - 1 2014.2016 J 1008 h Câu 3. (3,0 điểm) Hưởng ứng Giao lưu Câu lạc bộ môn học em yêu thích cấp Trung học cơ sở quận Hoàn Kiếm nãm học 2015 - 2016, lớp 6A của một trường trang quận có 23 học sinh tham gia mòn Toàn, 13 học sinh tham gìa môn Ngữ vãn, 20 học sinh tham gia mòn Tiểng Anh, 3 học sinh tham gia mỏn Toán và mòn Ngữ ván, 7 học sinh tham gia môn Toán và mõn Tiếng Anh, 5 học sinh tham gia môn Ngữ vãn và môn Tiếng Anh, 2 học sinh tham gia cả ba môn, 5 học sinh không tham gia môn nào. Tính số học sinh của lớp 6A. Cãu 4. (5,0 điểm) Cho đoạn thẳng AB = 5 cm. Lấy điểm c thuộc đường thẳng AB sao cho BC = 2 cm.
  • Tinh độ dài đcạn thẳng AC.
  • Lấy một điểm M bất kì không nằm trên đường thẳng AB. Nổi M với các điểm A, B, c. Hãy tinh ti số diện tích cùa hai tam giác MAB và MBC. Câu Cho
  • (2,0 điếm) .,123 4 4 2 4 3 4 2015 2016 4 2015 4 2ổie ' Chứng minh M < 77 , 25 ĐỂ THI LỚP 7 CÂU LẠC BỘ TOÁN QUẬN HOÀN KIÊM, HÀ NỘI Năm học: 2015-2016 Thài gian làm bài: 90 phút Bải 1. (5,0 điểm)
  • Tính giã tậ của M - í 1-1 -1-ị ; : -2 6 .3 3 .j 1-ị -1-ị ; . 2 3j 36 { 3 4 ; A
  • Tính giá trị của phản số — biết rằng B A = 1 + 2 4 + 2* +„.+ 2 2012 + 2 2016 vả B = 1 + 2 2 + 2 4 + ...+ 2 2016 + 2 2018 . Bãi 2. (5,0 điềm )
  • Tlm X, y, z biết rằng ^ = 'ị- = — và 2x 2 + 3y 2 - 4z z + 116 = 0. 2 3 4
  • Tim số thực X biết rằng „ 1 3 2x + -
  • 2x 2 2
  • 2 . Bải 3. (3.0 điểm) Mức nước sinh hoạt của nhà bạn Đức Anh được thống kê trong bảng sau đãy: Thời điểm Cuối tháng 9/2015 Cuối tháng 10/2015 Cuối tháng 11/2015 Cuối tháng 12/2015 Chỉ số đổng hổ đo nước (m 3 ) 154 170 137 202 Biết số tiền nưởc phải trả cho mỗi m 3 nưởc là không đổi và tổng số tiền nhà bạn Đức Anh phải trả trong quý IV năm 2015 là 430 000 đồng. Tính số tiền nưốc phải trà trong mỗi tháng 10, 11 và 12 năm 2015. Bài 4. (5,0 điểm) Cho tam giác ABC cân tại A. Trên cạnh AB lấy điểm M, trên tỉa đối của tia CA lấy điểm N sao chc BM = CN. Kẻ MH, NK cùng vuông góc vởi BC (H r K thuộc BC). Gọi I là giao điểm cửa MN vè BC-
  • Chửng minh AHBM - AKCN vá I lá trong điểm của MN.
  • Đường trung trực của MN cẳt tia phán giác Ax của góc BAC tại p. Chứng minh PMB = PNC.
  • Chứng minh khi M di động trên AB và N di động trẽn tia đối của tia CA thỏa mãn BM = CN thì p là một điểm cố định, Bải 5. (2.0 điểm) Tim tất cả các số tự nhièn n nhận giá trị từ 1 đến 2016 sao cho phân so n - — khòng tẵglản. a*- 2016 ĐỂ THỊI LỚP 8 CÂU LẠC BỘ TOÁN QUẬN HOÀN KIÊM, HÀ NỘI Năm học: 2015-2016 77jó/ gian làm bài: 1 20 phút Bải 1. (5,0 điểm) ' 1 111 111
  • Cho các sổ a, b, c thủa mãn — + — + — = -3 và — - = 7. a b c a 2 b z c 2 Tính giá tri của biểu thức A = — — . a + b + c
  • Tìm các sổ thực a, b để đa thức f{x) = X 3 +■ ax 2 + bx - 1 chia hết cho đa thửc X 2 - 3x + 2. Bài 2. (5,0 điếm)
  • Chứng minh (7 2n + 3.13" - 4 rt+1 ) chia hết cho 19 với mọi sđ tự nhiên n. b} Tim các căp số nguyên (x; y) thỏa mãn điều kiện X - 2y 2 4- xy + 2x -í- 4y - 5 0. Bài 3. (3,0 điểm) Cho ạ, b là các số thực dưetng thỏa mãn a + 2b > 5. 14 Tìm giá trị nhỏ nhất của biểu thức p = 2a-3b - — . a b Bài 4. (5 điểm) Cho hình vuông ABCD có cạnh AB = 6 cm, Trên cạnh BC lấy điểm E sac cho BE = 2 cm, AE cắt dường thẳng CD tạl G. a} Tính diện tích tam giác CEG. b} Gọi F là trung điểm của DG, BF cắt AG tại K, CK cắt AB tại H. Chứng minh AK vuông góc với CK.
  • Dựng hình chữ nhật BCEM và gọi I là giao điểm của HE vè AC. Chứng minh BI và CM song song vỏỉ nhau. Bài 5. (2 dìểm) a} Cho a > b > c. Chứng minh M = a 4 (b - c) + b 4 (c - a) + c 4 (a - b) luôn dương.
  • Chứng minh rằng trong nãm số nguyên tố bất ki lán han 3, luôn tồn tạí hai số có hiệu chia hết cho 12. DỀ THI HỌC SINH GIỎI MÔN TOÁN LỚP 9 HUYỆN YÊN LẠC TỈNH VĨNH PHÚC Năm học: 2015-2016 Thời gian làm bài: 150 phút Bải 1. (2 điềm)
  • Cho ba số dương X, y, z thỏa mãn điểu kiện xy + yz + zx = 201 5 và p_ x | {2015 + y 2 X2015 + z 2 ) .. 'V 2015 + x 2 Chứng minh rằng p không phải lả số chính phương.
  • Cho biểu thức v'x 2 - 6x + 1 1 - Vx 2 -6x + 1 0 = 2015. Chứng mình biểu thức Q = </x 2 -Sx \ 10 + ì/x 2 -6x I 1 1 không phụ thuộc vảo X. Bải 2. (1,5 điểm) Tim số tự nhiên n lớn nhất sao cho sổ 2015 bằng tổng của n số a v a 2 a n trong đó a ( <1=1,2. 3. ... , n) đều là hợp số Bài 3. (2 điểm)
  • Cho đõ thị M có số đỉnh V. số cạnh E và số miền R. Khi đó ta có công thức Euler như sau V - E
  • R = 2. Em hãy kiểm chứng cóng thức trên qua các đổ thị sau: BỄ Hình 2 Hình 3
  • Giải phương trinh sau X 2 - x-1 = \'8x + 1. Bài 4. (2.5 điểm)
  • Cho tam giác ABC và hai dường trung tuyến BN, CM vuông gốc với nhau. Chứrig minh rằng

    1 12

    t í — . tanB tanC 3
  • Cho nửa đường tròn (Q) đường kinh AB, M là một điểm trẽn nửa đường tròn (O). Lấy điểm c trẽn tia AM sao cho AC = BM. Chứng minh rằng đưàng thảng d vuông góc với AM tại c luôn đi qua một điểm cổ định khi M di động. Bài 5. (2,0 điểm )
  • Trong cuộc thi ve tranh chù đề vể đảo Trường Sa của học sỉnh giòi toán lớp 9 huyện Yên Lạc, tình Vĩnh Phúc có tổng số 100 học sinh tham gia, trong đó mỗi người quen ít nhất 66 người khác. Hỏi cỏ phải trong mọi trường hợp luòn tốn tại A ngưủi đôi một quen nhau không?
  • Cho cảc so a. b, c lả các số thực dương Ihỏa mãn a + b + c = 1. Hãy tìm giá trị nhỏ nhất của hiểu rrr a 3 b 3 c 3 thức p = — - — - ị- — — - í — - — — . (1-a) 2 (1“b) 2 (1-c) 2 Ị (2015 + x 2 )(2015 + z 2 ) [ 1 (2015 + x 2 )(2015 + y 2 ) \i 20 1 5 -í- ỵ 2 V 2015 + z 2 Câu 1. a) Ta có a 2 + 1 = a 2 + ab + bc + ca = (a f b)£a + c). b 2 + 1 = b 3 + ab + bc + ca = (b + a)(b + c). c z + 1 = c 2 4 ab + bc + ca = (c + a)(c + b). Thay vào ta tính được M - 0.
  • Đặt k.3 1,3 1,3 3 , 3 3 . 3 K K K am = bn - cp - k =s a - — =-;b « -T-;c = —r nr n 3 p 3 Do đó ¥ã + ặ'b + = k; ^am z + bn z + cp z - \/k 3 - k. Suy ra '-/ã + ặỉĩ + võ - ặam 2 + bn 2 + cp 2 . Câu 2. a) Ta có X 2 +3x + 1 = (x + 3 )a/x z +1 » (x 2 + 1) - x>/x z +1 - 3\/x z +1 + 3x = Q <=> (Vx 2 +1 -x)(Vx 2 + 1 -3) - 0 Vx 2 TĨ-x = 0 (1) «=> Ị— vx 2 +1-3-0 (2). (1) <=> Vx z +1 = X <“> jx >0 |x 2 +1 = X 2 (Võ nghiệm}. (2) <=> Vx 2 +1 =3ox 2 +1=9«X 2 =6 <=> X = ±2^2.
  • Ta cỏ x z - xy = 6x - 5y - 8 (X - 5)y = x z - 6x + 8 () • Vói X = 5 thi phương trinh (■) vỏ nghiệm • Vởì X ít 5 thì x 2 -6x + 8 „ 3 => y = — - X -1 + x-5 X -5 Suy ra X - 5 fc {± 3; ± 1}. Xél các trưởng hợp ta có các nghiệm cần tìm lá (.y); (6 ; 8); (4, 0); (8. 8); (2, 0)- Cáu 3. a) Với a, b > 0. Áp dụng bất đẳng thức A 1 11 1 AM-GM ta có - + -!- >2, -.-- a b \ a b Vì X, y > 0 nèn theo (*) ta cỏ — — 2v/ãb a ) b ■ n A = X 2 + y 2 xy 1_ + J_ x 2 +y 2 + 2xy
  • 4xy 5 +-—+ 4xy J - 4 - + - - +2 JỈ**y r 2 + y 2 + 2 xy (x + y ) 2 V 4 *y ■ — ~Ĩ + T~Ĩ (x + y) (x + y)
  • 2 = 4 + 5 + 2=11. Dấu đẳng Ihửc xảy ra khi X = y = Ỳ Vậy MinA= 11 khi X = y =1.
  • Vãi x r y, z > 0 theo bất đẳng thức Bunhiacõpxkĩ ta chúng minh được nf_ | n 2 | p 2 (m + ntp) 2 X y z x+y+z • Nếu ab + bc + ca £ a + b + c. Áp dụng (*) vá kết hơp với abc £ 1 ta có a b c _ a z c + b 2 a + c 2 b b c a abc a z b z c 2 (a -b + c) z T + T 4 T 111 — — — — + c a b c a ta

    abc abc (a + b + c} z

    ab + bc + ca ĩa ^ b + c (1) (abc) 2 • Nếu ab I bc + ca > a -I b ! c. Áp dụng (*) và kết hợp với abc £ 1 ta có a b c a z c + b ? a + c 2 b

    b c a abc a 2 c z b 2 a 2 0 2 b z (ab + bc + ca) z

    c a b > a + b -s- c abc abc ab + bc-ca > a +b + c. (2) Từ (ì) và (2) suy ra điểu phải chứng minh- Câu 4. a} A IM AM => AAMI co AINB => 777-- - -7-7-. BN IN Do đó AM.BN - IM.IN. Dễ dáng chứng minh được IM - IN => AM.BN - IM 2 - IN 2 ,
  • Đật AM = rm, BN = n, IM = IN = X. Vi AAMI và âAIB đống dạng nên AM Ạ| IA 2 m “ÃĨ = ÃB => "bc" b"

    Tương tự ta có . (2) ca a Vì tam giác MIC vuông tạl I nên IC 2 = CM 2 - IM 2 . Do IM 2 = mn và CM = CN nên IC 2 = (b - m)(a - n) - nrm = ab - bn - am.

    IC 2 n m Do đó = 1 ---—. {31 ab a b IA^ IB^ IC^ Tử(1), (2). (3) suy ra T^r+-1- bc ca ab Câu 5. a) Chứng minh bằng phương pháp phản chứhg. Giả sử vài mọi cách tò đếu không tổn tại hai điểm cùng màu mà có khoảng cách bằng 2016. Lẩy hai điểm M và N sao cho MN - 20 16 Vả thi tân tại các điểm p. Q thỏa mãn cảc tam giác MPQ. NPQ là các tam giác đều vởỉ độ dàl cạnh bằng
  • Khi đó r theo giả sử trẽn thì hai điểm củ khoảng cách bằng 2016 được tỏ bởi hai màu khác nhau, nén M. N phải được tô bdi cùng một màu, chẳng hạn M, N: Xanh, P; Đả, Q: váng (Hĩnh vẽ). (Đỏ) p (Vàng) Từ đó. nếu điểm M được tô màu Xanh, thì mọi điểm nẻm trên đưởng tròn têm M. bàn kinh 2016%/3 đểu được tò màu Xanh. Nhưng irẻn đưdng tràn này ta luân tìm được hai điểm có khoảng cách giửa chúng bằng 2016. Mâu thuẫn vối giả sử trên. Từ đó suy ra điều phải chũhg minh,
  • Ta sẽ chứng minh sinh víén mang sỏ’ 333 là "Sinh viên hay". Ta chia 997 sinh viên (tết cả các sinh viên ngoại trừ các sinh vìẽn mang số 333, 666 và 999) vào 498 tập hợp sau: {1, 2, 4, ...), (3,
  • 12. ...}, {5, 10, 20,"...},'... , (331, 662), {335, 670}, ... , {499, 998}, (501), (503} (995), {997}. Bây giờ ta xét một nhóm bất kì gồm 500 sinh viên có chửa sinh viên 333. Nếu một trong cãc sinh viên mang số 666 hoặc 999 thuộc nhóm đó thì nhóm dó là "nhóm hay”. Ngược lại ta có 499 sinh viên chia vào 498 tập hợp trên. Khí đó theo quy tắc Dírichlet thl tổn tại hai sinh viên thuộc váo một nhóm. Trong trưởng hợp có hai sinh viên thuộc cùng một nhóm thì một trong hai sinh vièn đỏ cỏ số chia hết cho số của sinh viên kia. Tử đó ta có nhóm này là "nhỏm hay" và sinh viên mang số 333 là "sinh viên hay". Cuổì cùng ta sẽ chứng minh các sinh viên mang các số từ 334 đến 1000 lá cảc "sinh viên tói™. Rõ ràng các sinh vién mang số tử 501 đẽn 1000 lập thành mội "nhóm toi" và do đó cáo sinh Viẽn mang SỐ từ 501 đến 1000 không phải là ‘sinh viên hay". Bây già ta xét nhóm gom 500 sinh viên sau đây: 334, 335 667, 669, 671, 673, ... , 997. 999. Nhỏm này bao gổm các sinh viên mang số 334 đến 667 và các sinh viên mang số lè từ 667 đến
  • Ta dể dàng thấy rằng nhóm này lả “nhóm tói" bởi vi với 334 < n < 500, sinh viên mang sô’ n thuộc nhóm đó và các sinh viên mang số 2n không thuộc nhóm đó cỏn số 3n đá vượt quà 1 000. Với n 501 thì 2n vượt quá 1000. Do đó ta không tìm thấy hai sinh viên nào mà sổ của sinh vién này chia hết cho số của sinh vièn kia. Điểu đó dẳn đến nhóm này lã "nhóm tồi" vá các sinh viên tư 334 đến 1 000 là các “sính viên lổi". Vậy đáp án cần tỉm lả 333 23 Giải toán qua thii Do đó EA = EC và EB = EF, dẫn đến BC = AF. Bãi 1(1 57). Nhà toán học De Morgan (1806 - 1871 ) khi được hỏi tuổi đà trả lói: Tôi X tuồi vảo nầm X 2 . Hỏi nãm X 2 đó ông bao nhiêu tuổi? Lỗi giải. Nhá toán học Do Morgan sinh váo năm 1 806, ông X tuổi váo nàm X 2 . Suy ra X 2 - X - 1806 (vì X F N*) Do đó x(x - 1) = 1806, vì 42.43 - 1806 nẻn X = 43 Suy ra X 2 = 1 849. Vậy vào nấm 1849 õng De Morgan 43 tuổi. Nhận xét. Bài toán này rất thú vị nên có nhiểu em tham gia giải và giải đúng Xin nêu tên mội số em trinh bây tốt: Nguyễn Minh Thủy. Nguyễn Công Hải. 6A3, THCS Lảm Thao, Lãm Thao, Phú Thọ; Nguyên Ngọc Mai. Nguyên Thị Việt Trà. Nguyễn Đấng Doanh, Lê Phan Thảo Vy, 6B, THCS Hoàng Xuân Hãn. Đửc Thọ, Hà Tĩnh; Nguyễn Thị Ngọc Mai, Nguyễn Mai Linh, 7A4, THCS Yên Lạc, Yên Lạc, Vĩnh Phúc; Đặng Hổng An. Phan Thị Quýnh Trang. 7C. THCS Bạch Liêu, Yên Thành, Nghệ An; Trương Thị Thu Lan. 7A1. THCS Yên Phong, Yẽn Phong, Bắc Ninh, PHÙNG KIM DUNG Bải 2(157). Cho tam gĩảc ABC vửi vá AB = AC. Trẽn tia AB lấy điểm D sao cho AD = BC. Tinh Trẽn cạnh BC lấy điểm E sao cho BAE = 60°. Qua B kẻ đường thẩng song song với AC cẩl AE tại F. Vì BÂC = 100° nên ABC = ÀCB = 40° và ÓÀF = BĂC - BÂÈ = 1 00° - 60° = 40°. Suy ra ẤFB =40 ữ (vì BF //AC) ÃCB = CBF = 40° (vì BF lì AC). Suy ra tam giác ADF đéu. Mặt khác AAEB =ACEF (c.g.c) nên CF = AB = CA. Do đó AADC - AFDC (c.c.c) Suy ra ÃDC = FDC = 30°, Nhận xẻt. Các bạn sau có lài giải gọn hơn cả và được khen: Tử Tẩn Dũng, 7D, THPT chuyên Hà Nội - Amsterdam, cáu Giấy, Hà NỘI; Nguyễn Lan Chi, Trương Thị Thu Lan. 7A1, THCS Yân Phọng, Yên Phong, Bắc Ninh: Nguyễn Thị Ngọc Ánh, 7E2, THCS Vĩnh Tường, Vĩnh Tưởng, Vỉnh Phúc; Trần Quang Vinh, 7A, THCS Hùng Vương, TX. Phú Thọ; Nguyễn Vũ Hà, Khổng Doãn Hưng, Nguyễn Công Hiếu » 7A3, THCS Lâm Thao, Lâm Thao, Phú Thọ; Hoàng Thị Phương Anh, 7C, THCS Nhữ Bã Sỹ, Hoằng Hóa, Thanh Hởa; Đường Hải Sáng, 7C, THCS Bạch Liêu, Yên Thành; Nguyễn Trình Tuấn Đạt, Lỗ Đình Tủ, Nguyễn Sĩ Quyền, 7D, THCS Ly Nhật Quang, Đò Lương. Nghệ An: Trần Đình Hoàng. 6C; Nguyễn An Na, Thái Thị Thu Sang L 7A, THCS Hoàng Xuân Hãn, Đửc Thọ; Nguyền Trúc Quỳnh, 7/1, THCS Lé Văn Thiênnì TP. Ha Tĩnh, Hà Tĩnh; Phan Thị Như Quýnh, 715, THCS Nguyễn Thị Minh Khai, Cam Phúc Bắc, Cam Ranh, Khánh Hòa. HỔ QUANG VINH Bài 3(157). Giải hệ phương Èrình Lài giải. Biến đổi phương trinh (1) như sau (x + y)(x 2 - xy + y 2 } + 7{x - y) - (x 2 - xy + y 2 + 7) = 2(x + y} 2 - 2 « (x + y - 1}(x 2 - xy + y 2 + 7)
  • 2(x + y - 1 )(x + y + 1 } = 0 (X + y - 1){x 2 - xy + y 2 - 2x - 2y + 5) = 0. (3) Đặt A = X 2 - xy + y 2 - 2x - 2y + 5 thì 2A = 2x 2 - 2xy + 2y z - 4x - 4y ( lũ = (x z - 2xy + y z ) + (X 2 - 4x + 4) + (y z - 4y + 4} + 2
  • (X - y) z + (X - 2) z + (y - 2) z + 2 > 0. 24 Suy ra A> 0. Từ đó và (3) suy ra X I y = t. (4) Từ (4) ta có (x + 1 )(y + 1 ) - scy + X + y + 1 - xy + 2 [z 1 9 ( 3 s4(+yr+2=T 4 'í 5 ) Vỡỉ điều kiện X > 0, y > 0 và (5), ta biến đổi phương trình (2) như sau \2 ì 3 16 (Vx +Vỹ ) 2 « C9 16 X + 1 V y + 1 3(Vy+i + Vx + ĩ) 2 <x+1)(y + 1) " X + y + 2 N ,xy 3{y-x + 2) + 6V(y r 1)(x^ĩ) 16 xy + 2 1 + 2^/xỹ 9 + 6^/xy + 2 16 xy + 2 1 + 2,/xỹ' Từ' đó kết hợp vởì (5) ta có xy + 2 9 + Biịxỹ +29 + 99 1 + 2i/xỹ - 16 16 - 8 Suy ra 8xy-18^xỹ + 7 < 0, hay (2^ỹ-1K4^ỹ-7)íO. (6) Do 2,/xỹ sx + y = 1 và 4-/xy < 2 < 7 nên (6) xảy ra cliĩ khi 2^xy = 1 = X + y , tức lả khi X = y = .

    Thừ lại thấy (x.y) = M lì 22 \ / Lhỏa mãn hệ phương trinh ban đầu và là nghiệm duy nhất. Nhận xét. Các bạn sau cố lỡi giải đủng: Nguyễn Ngọc Huyền, 9a! THCS Hùng Vương, TX Phú Thọ, Phủ Thọ; Tạ Nam Khánh, Lẻ Ngọc Hoa, Nguyễn Cóng Huấn, 8E1, THCS Vĩnh Tưãng. Vĩnh Tưàng, Vĩnh Phúc; Chu Thị Hằng. 9A1, THCS Yên Phong, Yên Phong, Bắc Ninh; Nguyễn Minh Nghĩa, 9B. THCS Nguyền Thượng Hiển, ững Hòa, Hả Nội. NGUYỄN VIỆT HẢI Bài 4(1 57). Cho cãc số thực a. b, c và d thỏa man; a > b > c > d; a + b + CH- d = 9; a 2 + b 2 + c 2 + d 2 = 21. Chửng minh rằng ab - cd > 2. Lởi giải. Ta có (ab + cd) + (ac + bd) + (ad + bc)

    { a + b + c + d) 2 -( a z +b 2 +c 2 +d 2 ) , 30 2 Từ giả thiết a > b > c > d, nên ab + cd > ae + bd > ad + bc, suy ra ab + cd 2 10. Đặt X = a + b, ta có X 2 + (9 - x) 2 = (a + b) 2 + (c + d} 2 = 21 + 2(ab + cd) > 41 Suy ra X 2 - 9x + 20 > 0 (x - 4)(x - 5) > 0. Mặt khác dox a-b>c+dvàa-b+c+d 9. suy ra X > 5. Do đó. 25 < 0 2 + b 2 + 2ab = 21 - (c 2 + d 2 ) + 2ab < 21 + 2(ab - cd), suy ra ab - cd > 2. Đẳng thức xảy ra khi a = 3, b = c = d = 2- Nhận xét. Đây là bài toán khó nên có rết ít bạn tham gla giải bảl. Các bạn sau đây có lời gỉải ĩỗt; Vù Mạnh Tùng, Lương Hoàng Tùng, Nguyễn Minh Đức, 9H1 , THCS Trưng Vương, Hoàn Kiếm, Hà Nội. CAO VĂN DŨNG Bàí 5(1 57). Cho một bảng õ vuông kích thước 7x7 (gồm 49 ỏ vuông đơn vị). Đạt 22 đấu thủ vào bảng saa cho mỗi ỏ đơn vị có không quá một đảu thủ. Hai đẩu thủ dược gọi là tẩn cõng lẳn nhau nẩu hg cùng trên một hồng hoặc cùng trên một cột. Chứng minh rằng với mồi cách đật bất kì luôn tồn tạỉ ít nhất 4 đầu thủ không tần công lẫn nhau. Lởi giải. Ta đánh số các ò vuông đớn vị bài cảc số từ 1 đến 7 sao cho mỗi số dùng đủng 7 lần, mỗi ô điển 1 sổ, càc Ẹố trên cùng một hàng và cùng một cột đõi một khác nhau như hình vẽ sau 1 2 3 4 5 6 7 2 3 4 5 6 7 1 3 4 5 6 7 1 2 4 5 6 7 1 2 3 5 6 7 1 2 3 4 6 7 1 2 3 4 5 7 1 2 3 4 5 6 Như vậy, 49 ô vuông dơn vị được chia làm 7 loại ó vuông, mỗi loại có 7 õ ghi cùng một số. Đật 22 đấu thủ vào 7 loại ố vuông, theo nguyên lí Dirichlel thì tồn tại ít nhất 22 7
  • 1 = 4 đấu thủ thuộc củng một loại ỗ vuông nào đó. các dấu thủ này đỗi một khàng tẩn cõng nhau. Ta có điểu phải chứhg minh. i I Được THƯỞNG Kl NÀY Gọi K lá giao điểm thử hai cùa PQ và đường tròn (O). Nhận xét. Các bạn sau đây có lòi giải tốt: Tạ Nam Khảnh. Lè Ngọc Hoa. 3E1, THCS Vĩnh Tưởng, Vĩnh Tưàng; Trẩn Minh Huy, 7A f THCS Lý Tự Trọng, Bình Xuyên, Vĩnh Phúc. TRỊNH HOÀI DƯƠNG Bài 6(157). Chữ lam giác ABC nhọn, không cân tạí A, nội tiếp đường tròn (O). Gọi M, N là hai điểm cổ định thuộc cung nhỏ BC sao cho MN // BC và tía AM nằm giữa hai tia AB vá AN. Gọi p lá điểm nào đó Irèn đoạn thang AM. Đường Ihẳng đi qua p song song vối GC cắt AC, AB lần lượt tại E, F. Đường tròn ngoại tiếp tam giác NEF cất đường trán (O) tại Q khác N, Chứng minh rằng đường thẳng PQ luốn đl qua một điểm cố định khi điểm p di chuyển trèn đoạn thẳng AM (p khác M). I rtĩ ỉilảl Vì MN ii BC nên MÃB = NÁC, (1 ) Do đá NÃB = MAC. Vi tử giác ABNC nội liếp và EP // BC nẽn ẤNB = ẨCB = ÃẼP- AP AE Từ đó suy ra AABN oo AAPE => -77- - T77- ( 2 ) AB AN Từ (1 ) và (2) ta có AABP trt AANE ĩ PBẦ = ÉNẦ. Kết hợp vãi các tử giác AQBN, ENQF nội tiếp, ta có PBQ = PBA I ẨBQ = ENA I ÀNQ ỒNQ = 180° -ẺFQ = 180° -PFQ tứ giác PBŨF nậi tiếp. ) vởi tứ giác AQBK ) - FBÒ = ẤBO = ẨKG nộỉ tiếp, suy ra FPQ = FBQ = ẤBQ = ÃKQ. Nói cách khác AK ì ì FE // BC. Do đỏ điểm Vậy đường thẳng PQ luôn đi qua điểm Nhận xét. Các bạn sau có lởi giải tốt: Tràn Lặp, Trần Thị Thu Huyền, 9A3, THCS Lâm Thao, Lâm Thao, Phủ Thọ: Nguyễn Minh Nghĩa, 9B, THCS Nguyễn Thượng Hiền, ừng Hòa, Hà Nội. NGUYỄN MINH HÀ Nguyên Minh Thùy, 6A3, THCS Lãm Thao, Lâm Thao, Phú Thọ: Nguyễn Trủc Quỳnh, 7/1, THCs' Lè Văn Thỉỗm, TP. Hà Tĩnh, Hả Tĩnh: Tạ Nam Khánh, Lê Ngọc Hoa, 8E1. THCS Vĩnh Tưởng, Vĩnh Tường, Vĩnh Phúc; Đường Hải Sáng. 7C, THCS Bạch Liéu, Yén Thành: Nguyễn Sĩ Quyền. 7D, THCS Lý Nhật Quang, Đó Lương, Nghệ An; Trương Thị Thu Lan, 7A1, THCS Yên Phong, Thi giải toán qua thư Yên Phong, Bắc Ninh; Từ Tẩn Dũng, 70, THPT chuyên Hà Nội -Amsterdam, Cẩu Giấy; Nguyễn Minh Đức, 9H1, THCS Trưng Vưđng, Hoàn Kỉếrrt; Nguyễn Minh Nghĩa, 9B. THCS Nguyễn Thượng Hiển, ứng Hòa, Hà Nộì; Hoàng Thị Phương Anh. 7C, THCS Nhữ Bá Sỹ, Hoằng Hóa, Thanh Hóa; Phan Thị Như Quỳnh, 7/5, THCS Nguyễn Thị Minh Khai, Cam Phúc Bấc, Cam Ranh, Khánh Hòa. Từ SỐ tháng 9 nảm 2015, Công ty cể phần Dịch vụ Giảo dục Việt Nam sẽ tăng các khóa học trực tuyến trèn vvebsite: hocmai.vn cho các bạn học sinh được thưởng trong các chuyên mục vá các bạn học sinh được khen trong chuyên mục Kết quả thl giải toán qua thư. Các bạn học sinh sau khi nhận đưdc mã cung cấp thi đăng kí tại địa chỉ: thcs.hocmai- vrựtoarìtuoìtho (Xln Hên hệ SĐT 0966464644 để được giải đáp}. 20 NĂM HỌC BỔNG ASEAN CỦA SINGAPORE CHO HỌC SINH LỚP 9 VIỆT NAM vo KIM THÚY AC là tử viết tẳt cửa Cộng đốngASEAN bằng tiếng Anh (ASEAN Community). Cộng dồng ASEAN thành lập chinh thức từ 31.12.2015. Năm 2016 này tạp chi Toán Tuổi thơ mở chuyên mục cửa sổ AC dể bạn dọc hiểu hơn ưể vùng đất con ngitòi của 10 quốc gia vời 625 triệu dàn. N ăm 1995 Việt Nam gia nhập ASEAN. Nắm 1996 Việt Nam bát đầu Lham gia thi học bổng ASEAN của Singapore cùng với các nước trong khối ASEAN. Năm đầu tiên ấy đả có 110 em dự thi tại 2 hội đóng thi ở Hà NỘI. Thành phố Hó Chi Minh và có 8 em giành õược học bổng. Đố là 4 em nam: Vũ Thành Nam. Mai Thanh Bình (Hà Nộỉ). Vũ Thanh Hài (Hải Phòng}, Lê Hồng Minh (Hà Bắc) và 4 em nữ: Dậu Phan Ngọc Thanh (Thừa Thiên - Huế) r Phạm Nguyền Anh Thư, Phạm Thị Thư Thư (TP. Hỗ Chí Minh), Lý Phương Hạnh (Tìển Giang), Cuộc thi gốm ba bài thi: một bài Tiếng Anh (thời gian thi 2 giờ), 1 bài toán 25 càu hòi bằng Tiếng Anh (thời gian thi 2 giỡ) và một bài trắc nghiệm thõng minh IQ gốm 60 câu (lâm trong 20 phút}, diền ra trong mật ngày. Sau một thăng gần một nửa SQ thí sinh dự thi được Dại sử quàn Singapore báo đỗ vòng 1 vá sau dó từng ngươi dược phỏng vấn. Một tháng sau s ngưài được bão đã qua cả 2 vòng. Ngày 13.11.1996, 8 học sinh Việt Nam vừa hết lóp 9 sang Singapore theo hạc bổng ASEAN. Các em bắt đầu vào lớp Sec 3 tức lớp thứ 3 trong bậc Trung học Phổ thông (Secondary). Sau đó là 2 nảm Tiển đại học (Preuniversity). Cu di 4 nàm phái thi A level tức lốt nghiệp Trung hoc Phổ thông và giành hạc bống vào Đại học. Bái thi được gửi sang Anh chấm, cả 3 học sinh đều theo học ở các trương Phổ thông vả Tiền đại học thuộc top 10 của Singapore. Các em đãng kí vào đại học và cản cứ vảo điểm thi BỄ được xếp vào các ngành có điểm cao hoặc phải xuống ngành điểm thấp hơn tùy kấl quả điểm sau đó. Trước đỏ, ki thì Olevel sau lớp Sec 4 cũng dã gửi sang Anh chấm. Khi đỏ cảc em cũng tự đàng kí vào các trường Ti én dại học trong top 10 trường tốt nhất, Tùy theo điểm sẽ quyết định các em được hay không vào các trướng đã đàng kí vả xuống trưởng hạng thấp hơn. Vào đại học đa số các em chọn các ngành vế Tin học, Viến thông, Kinh tế, Tài chính, Bất động sản để học liếp. Mai Thanh Binh nay điều hành một cõng ly con chuyên vé game Online ử Việt Nam, còng ty me ở Singapore. Lê Hồng Minh làm â vãn phòng luật tư vấn về đãng ki SCI hữu trí tuệ công nghiệp. Vú Thanh Hải dạy khoa Kinh tế cho đại học NUS, Singapore. Vũ Thành Nam làm kĩ sưà công ty Glữlbal Foundries tại Singapore. Nguyễn Anh Thư lãm tài chính cho tập đoàn VinGroup ỏ Việt Nam. Phan Thị ThưThư làm kĩ sưcỏng ty dấu khí. Dậu Phan Ngọc Thanh làm kì sư công ty dấu khi Schlumberger, Lý Phương Hạnh học bất động sản nay làm ở TP. Hỗ Chí Minh. Nắm thử hai. nám học 1998 - 1999 có 13 học sinh, nãm thứ ba có 21 học sinh. Nảm học 2006 - 2007 có 25 học sinh giành dược học bổng, là năm có so học sinh dỗ cao nhãì. Từ nam hục 2005 - 2006 có thêm học bổng A*Slar (The Agency for Science, Technology and Research) các trưởng trực tiểp làm việc với Việt Nam chọn học sinh giỏi và cấp học hổng. Tửnãm 2006 nuức bạn hắt đẩu tuyển học sinh lởp 6, 7 của Việt Nam sang học lốp dầu cấp Trung học (Sec 1). Các em nãy sẽ phải học 4 nàm Trung học, 2 năm Tiển dạt học và 3 hoăc 4 năm Đại học tùy ngành. Với cách ra đé llii gồm nhiều bải. tiếng Anh ở mức độ khó và thêm IQ đã làm cho các ki thi của nưủc bạn tổ chức luôn chọn dược những học sinh giỏi nhất. Các trường NTỦ, NUS, Ratlles, NJC, Hwa chong, Nanyarg, ACS, Victoria, Temasek,... của singapore đi tuyển đưdc hàng trăm học Sinh giỏi ở Việt Nam. Ờ Há Nội, đó là các trưởng Trưng vương, Hả Nội - Amsterdam. Khoa học Tự nhiên, Đại học Sư phạm. Chu Vãn An, Việt Đửc. Trần Phú, Kim Liên. Nguyễn Tãt Thành, Ngõ Sĩ Liên, Lương Thễ Vinh, Marie Curie, Nguyễn Trưởng Tộ. Lô-mỏ-nõ-xỂp, Giảng Võ... Ở Nam Định là các Lrưàng Trần Đăng Ninh, Lê Hóng Phong, ở Hài Phùng lả trưdng Trãn Phú. Ngoái ra còn có học sinh ở các tỉnh BỂC Ninh. Nghệ An, Vĩnh Phúc, TP Hó Chí Minh, Gia Lai, Hưng Yên, Hài Dương. Thái Nguyên, Thái Binh, Kiên Giang. Thửa Thiên - Huế,... Hai mươi nàm tròn tĩnh từ ngày 3 học sinh lãp 9 đầu tiên của Việt Nam sang Singapore học học bổng ASEAN và A*star sau đó. Sau này. hàng nghìn sinh viẽn Việt Nam cũng được tuyển sang Singapore bầng học bồng ASEAN. Rất nhiễu trong só' đó đã sang học tiếp ở Anh, Mỹ, và nhiều ngưửi đã trở thành thạc sĩ, tiến sĩ được đào tạo ở Singapore, Anh, Mý, ủc..„ mang về nhà. Lẽ ra cháu mang luôn đến cỏng ty, nhưng lúc đỏ cháu sực nhở phải qua nhà lấy vài thứ để buổi trưa mang vào bệnh viện cho mẹ. về tỏi nhà, cháu để cái túi đựng bọc tiến trẽn kệ TV roi chạy vào bếp ăn tạm miếng bánh. Đang àn thì cháu có điện thoại. Một người cùng phòng bệnh với mẹ cháu gọi, bảo cháu phải vào viện ngay. Hoảng quá. cháu vội vã khóa cửa rồi phóng xe đi mà chẳng nhớ gì tới bọc tiền. -Thế lúc não thì cháu phát hiện bọc tiền bị mất?
  • Sau khi xong việc ở bệnh viện thì cháu sực nhớ ra Lúc đó khoảng hai rưỡi, ba gid chiều ạ. Cháu vội phóng xe về nhà thì chẳng thấy bọc tiển đâu cả.
  • Từ lúc cháu để bọc tiền cho Lãi lúc phát hiện bị mất, trong nhà có những ai? ang trèn đường về nhà sau giở làm việc thì thám tử Sêlôccòc nhận đƯỢc cuộc gọi của Thu Minh - một cô gái trẻ mà thám tử đã tình cà quen biết trong một chuyến du lịch mấy năm vể trước.
  • Chú ơi! Chú giúp cháu với! Cháu vừa bị mất tiền... Tiền của công ty ạ.
  • Chết! Mất nhiều không?
  • Dạ, so với người khác thi chắc cũng không quá nhíểu. Nhưng với cháu thi quả lớn ạ. Mẹ cháu đang nằm viện, cháu thì vừa mởi được nhận vào công ty nên.,,
  • Thôi được rồi, chú sẽ tới nhà cháu bây giờ. Nhắn địa chỉ cho chú luôn đi. Hơn nửa tiếng sau, thảm tử Sêlôccỏc đã có mặt tại nhà Thu Minh.
  • Sáng nay, cháu rút tiền ỏ ngán hàng rồi Kẻ khả nghi BÙI PHƯƠNG THẢO (7B, THCS Phan Bội Châu, Tứ Kỳ, Hải Dương)
  • Chỉ có 2 đứa em họ của cháu thối ạ. Bác giúp việc thì vào bệnh viện vởi mẹ cháu từ sáng sỏm, tối mói vể.
  • Cháu noi rõ hơn vể 2 đứa em đi!
  • Đỏ là Hải vả Binh. Hảỉ là con cỏ cháu, đang là sinh viên nãm cuối. Hải ở nhà nhả cháu suốt từ hổi vào dạỉ học tãi giờ. Bình lã con cùa bác họ cháu. Binh vừa xin được việc, à tạm nhà cháu vài tuần, khi nào tìm được chỗ ưng ý để thuê thì sẽ chuyển đi. Hàng ngày Hải và Bình đi lừ sáng, buổi trưa thường về ăn ở nhá roi chiểu lại di r
  • Chú cần nói chuyện với hai cậu em của cháu.
  • Vâng. Chú có thể đợi một chút không ạ? Hai đứa chắc sắp về lởi nhà rồi. Khi Hải về lới nhà, Ihám tử hỏi ngay xem cậu ta đã làm gì trang buổi trưa hôm qua
  • Dạ, cháu vế sớm hơn mọi khi một chút để nấu cơm vì biết bảc giúp việc phải ở trong bệnh viện.
  • Thế Bỉnh có về ãn cùng không?
  • Có ạ. Trưa qua Bình lại rù một bạn nữa vế, thè là ba đứa bọn cháu cùng ãn cơm. Đúng lúc đó, Binh về tới cửa. Thảm tử sẻlóccỏc vào việc luôn. Bình kể:
  • Cháu vể thi anh Hải đang nấu. Cháu và Huy - bạn cháu - ngồi ồ phòng khách, sau đó thì vào bếp an cơm.
  • Các cháu đă làm gì lúc ỏ phòng khách?
  • Bạn Huy xem TV, còn cháu thì Chat trên FB với đứa bạn thân đang đl công tác xa ạ. Mải Chat nên anh Hải phải gọi mấy lần cháu mỏi đứng lên.
  • Vậy à? Chắc có chuyện gì thủ vị thi cháu mài Chat quên cả đòi chứ nhì?
  • Vẳng. Bạn cháu kể vể chuyển di tỏi một bản miền núi heo hút, chưa có điện. Thấy cảnh dân bản phải đi rất xa mỏi lấy được nước suối vế dùng, bạn cháu và nhỏm cõng tác đắ góp tiền mua tặng một chỉẽc máy bđm. " Thôi, cả hai cháu Hải và Bình có thể lên phòng. Chũ có chút việc trao đổi riêng với Thu Minh. Sau đó, thám lử nói với Thu Minh;
  • Cháu nên nói chuyện thêm với một trong hai

    ngưài em của cháu. Chú thây nghi ngà lẩm.

    Thu Minh chưa thể đoàn được người đó tà ai. Câc thám tử Tuổi Hông cò đoán ra không? Mất trộm vì ngủ quên (TTT2 số 157) Thám tử nghi ngờ ông David bởì ông này nói rằng thú mỏ vịt đẻ con. nuôi con bẳng sữa vã sống dưới nước. Trên thực tế, thú mỏ vịt là một trong số rất ít những loài thú đẻ trứng. Hơn nữa, thú mỏ vịt còn lả loài bán thủy sinh, tức là vừa sống trên cạn, vừa sống dưới nước. “T Phần thưởng kì này được gửi tới; ■ - — — Nguyên Hải Khoa, 6A, THCS Lý Tự Trọng, Bình Xuyên, Vĩnh Phúc; Lể Thị Thúy Hiền, 7B9, THCS Chu Vãn An, Ngô Quyến, Hải Phỏng; Nguyễn Minh Anh, 7A, THCS Hoàng Xuân Hãn, Đức Thọ, Hà Tĩnh; Nguyễn Xuân Thành Đạt, 70, THCS Thị trấn Gio Linh. Gio Linh, Quảng Trị; Ngõ Võ Hoàng Việt , 6A3, Trung học Thực hãnh Sài Gòn, P.4, Q.5, TP. Hổ Chí Minh. Thám tử Sêlôccôc Bài 67 : ặềAÍ&Mììĩ Tôi từ thành phố HỒ Chí Minh tới (Tiếp theo kì trước) Ths, NỠUYỄN vũ LOAN LTS. Nlếu biet liếng Hán bạn sẽ:
  • Hiểu các tử Hán Việt, sứ đụng tốt hon tiếng Việt của minh. Trong kho từ vựng tiéng ViệL rát nhiều từ Hán Việt.
  • Đọc dược sách cổ, văn bia bằng chữ Hãn vá Hán Nôm. thêm hiểu vàn chương, l[ch sử nước Nam mình.
  • Hiều ngôn ngữ mà cứ 5 người trên thế giới có hơn 1 người dùng. Dẻ dàng hợp tác. làm ần với các nước vả vũng lãnh thồ Trung Quốc, Hồng Kõng, Đài Loan, Singapore vả cả Nhật Bán, Hán Quốc. Nếu biết cả tiểng Anh và tiếng Hán thi thật là tưyệt. Míìu tóm lắt lí lìeh cả nhãn ■ (RtùmQingLíng- Nguyền Khánh Linh) &m. mm.mm 2004^12^040 nnnn (zhiyè- nghề nghiệp): ttht: 38526-xx, íi T^líllỉ # : ling@yahoo.com. Tập đục.
  • HẾI Mikc.Htíd — Nĩhầo, wòjiảoMike, w6shìyĩgèxucsheng.
  • iiíigA. mÈim lík, WỆtftM&®ỉặị, Wó shì Mẽiguỏ rén, wõ zài Mciguó chũshẽng, xiàrưải wỏjiãzảỉ Yuènán NVõdcshẽngrìshìèr líng língsì nián bãyucshtwũ hào, wõ iĩrinián sht'crsiiì Xiàiuải wỡ yốu yĩgcYucnán péngyỏu, lã cóng Hcnèi lủi, (ă shi yĩgc nản hâi/ì Bải tập. Đọc vá nối
  • 2 )
  • Sì JL?
  • E«JL ?
  • ttkAUL*?
  • Hề lim A,
  • fỄJbAífl^9lT|ĩ5fe.
  • &mt.
  • HcỂI-lỄTẫỄ38526xx
  • nim dj
  • HtN Ạĩệ.
  • GEOMETRY HÌNH HỌC (Tiếp theo kì trưởc) Polygons A polygon is a closed plane íigure formed by three or more line segments, called the sides úí the polygon. Each side intersects exactly two other sides at their endpoírts. The points of intersectíon of the sides are vertices The term polygon will be used to mean a convex polygon, a polygon in vvhich each interior angle has a measiire of less than ISO 0 . For example. are polygons. The following tĩgures are not polygons: A polygon with three sides is a triangle. A pỡlygon with four sides is a quadrilateral. VŨ ĐÔ QUAN Maths Tcrms poiygon plane figure li ne segment siơe intersect endpoint points of intersection vertex đa giác phẳng, mặt phẳng hỉnh, chử số đoạn thẳng cạnh cát đầu mút, điểm đẩu mủt các giao điểm đỉnh convex lổì interior trong measure sỏ' đo less than nhỏ hơn quadrĩlate rai tứ g i á c Practĩce. Bạn dùng các từ vựng đã được gợi ỷ để dịch bài khóa trên. Bài dịch tốt được chọn đãng, được nẻu tên và có phấn thưởng. Bạn cẩn gửi trưdc ngày 15,6.2016 mới được tham dự chọn trao thưởng. THÁCH ĐẤU! THÁCH ĐẤU ĐÂY! TRẬN ĐẤU THỨ MỘT TRĂM BA MƯƠI BẢY Ngưòi thách dấu: Trần Bá Duy Linh. 1 2T 1, THPT chuyên Nguyền Bỉnh Khiêm, Vĩnh Long. Bài toán thách dấu: Cho tứ giác ABC D ngoại tiếp đường trôn tâm I thỏa mãn (AI + DI) 2 -t- (BI + Cl} 2 = (AB CD} 2 . Chứng minh rằng ABCD lá hình thang cân. Xuã't xử: Sáng tác. Thỏi hạn: Trước ngày 08.06.2016 theo dâu bưu điện. TRẬN ĐẤU THỨ MỘT TRĂM BA MƯƠI LĂM (TTT2 số 157) BỔ để 1. Nếu liếp luyến tại A của đường tròn ngoại tiếp tam giác ABC cất BC tại M thi MA Z - MBMC. Bạn đọc tự chúng minh bổ đề này. Bổ dể 2. Cho haì đường tròn (0,|), (O^) tiếp xúc ngoài vối nhau tại T Các điểm A. B thuộc (0,j) t các điểm c. D thuộc (0 2 ) và M là giao điểm của AB và CD thỏa mãn MA.MB = MC.MD thì MT là tiếp tuyến chung của hai đưửng tròn (0 1 ) r (0 2 ). Chímg mình (hình 1) Qua M lần lượt kẻ các tiếp tuyến ME, MF đẽn (0,|), (0 2 ) . Trẽn tia đối của tia E0 1 lấy điểm K sao cho EK F0 2 - |y Hình 1 Theo bổ để 1, chú y ràng MA.MB - MC.MD, La có ME= vMA.MB = vMC.MD = MF. Kết hạp VỐI EK = F0 2 và MÈK = 9CT - MFÒ 2 , suy ra AMEK - AMF0 2 . Do đó MK - M0 2 . Từ dó, chú ý rằng o 1 k~o 1 e+ek-o 1 t+fo 2 -o 1 t+to ;ỉ o z o 1 . Suy ra AMO^K « AM0 1 Q 2 . Kết hợp vâi 0,E - C^T, suy ra AMO^E - AMO,,T Vậy MTQ-I =MEOi =90 1 ’. Điều đó có nghĩa là MT là tiếp tuyến chung của Trỏ lại giải bãi toán thách dấu • Phần thuận (hinh 2) Gọi s là trung điểm cung AB của (O). thuộc nửa mặt phẳng bở AB không chửa (I), (K); M, N tương ứng là tiếp điểm cùa (I), (K) vái AB, p, Q tương ứng là tiếp điểm của (I), (K) với (Q). Dễ thấy IM // os. Kết hợp vâi IP IM; OP os, suy ra IPM - ' - " - ÕPS- 2 2 Do đó s. M, p thẳng hảng. Điểu đá có nghĩa là PSA-ÃSM, 32 Hỉnh 2 Dể thấy SPÃ=|sđẽẦ-^sđSB = SÃB = SẦM. Vậy ASPA co ASAM. Do đó SM.SP = SA 2 . Tương tựS, N, Q thẳng hàng và SN.SQ = SB 2 = SA 2 . Vậy SM.SP = SN.SQ. Từ đó, chú ý rằng (I), (K) tiếp xúc với nhau tại c, theo bổ đề 2, suy ra sc lá tiếp tuyến chung của (I). (K). Vậy theo bổ đề 1, SC=%/SM.SP =SA. Từ đó, chú Ý rằng c nầm trong (O), suy ra c thuộc phần đưởng tròn tâm S r bán kinh SA, vẽ trong (O), kí hiệu là (W). • Phẩn dảũ Gọi s là trung điểm cung ẦB của đưửng tròn (O), thuộc nửa mặt phãng bở AB không chửa (W) Hình 3 Lấy điểm c bất kì trên (W). Gọi d là đường thẳng đi qua c và vuông góc với SC; T là giao điểm của sc và AB; I, K theo thứ tự là giao điểm của d với đường phân giác của các góc CTA, CTB; (I), (K) theo thứ tự là các đường tròn tâm I, K, đi qua c, Dẻ thấy (I), (K) tiếp xúc ngoài với nhau tại c, Vì ITC =ITA và (I) tiếp xúc vâl TC nén (I) tiếp xúc vâi TA. Gọi M là tiếp điểm của (I) và TA; p là giao điểm thứ hai của SM và (I) Vì sc tiếp xúc vãi (1} và c thuộc (W) nèn. theo bổ đe 1 ta cỏ SM.SP = sc 2 = SA 2 . Do đó SP SA SA SM Điẻu dỏ có nghĩa lâ ASPA co A$AM- Kểt hợp với SA= SB. Suy ra SPA - SẦM - SẦB -SBA, Do đó tứ giác PASB nội tiếp. Vậy p thuộc (O). Dế thấy IM // os. Kết hdp với IP - IM; OP - os. suy ra IPM = IMP = ỐSP = ỐPS, Điều đó cỏ nghĩa lá p, I, o thẳng hàng. Vậy (I) tiếp xúc trong vâi (O), Tương tự (K) tiếp xúc với TB vá tiếp xúc trong vâi (O). " • Kết kuận Quỹ tích của c là phẩn đường tròn tâm s, bân kinh SA. vẽ trong (O), Nhận xét. Bái loàn nãy khó nén không có võ sì nào dăng quang trong trặn đấu này. Phấn thưởng xin gác lại kl sau. NGUYỄN MINH HÀ nHỪ PHÁT Hiện TRÊyìnpẼ cổ IIAI ĨAIĨI GIÁC OỒÍIG DẠÍIG ufi ĨRUne ĐIỂÍD CÙA CẠAH NGUYỄN ĐỨC TẤN (TP. Hồ Chi Minh) Chủng ta bẩt đầu từ bài toán cơ bản sau: Gài toán. Chữ AM, A'M H lãn lượt là các đường trưng tuyển cùa các tam gìàc ABC, AB'C r . Chúhg mình rầng AABC co AA'B'C' khi vã chỉ khỉ AABM co AA'B'M'. Lỏi giải. AB _ BC ÍAB 2BM 2B'M r ả=B' 'B = B' A' AB BM A'B' B'M f B = B' <=> AABM co AA r B'M\ Bàl toán co bản trên giúp ta có được lòi giải nhiều bài toán khó. Sau day là càc bài toàn minh họa. Bài toán 1 . Cho tứ giácABCD nội tiếp đường tròn (O), H. K thử tự lả hỉnh chiếu của B lẽn AC, CD. Gọi M, N lần lượt là trung điểm của AD, HK. Chứng minh rằng MN vuông góc với NB. (Đề thì tuyền sinh vâũ lờp 10 chuyên toàn, trường Dại học Sư phạm TP. Hồ Chỉ Minh, năm học 2014 - 2075; 1 rll 4-11-31 B Dề chứng minh được tứ giác BHKC nội tiẽp, do vậy ta có BKH = BCH, BHK I BCD = 1 S0 Q . Do đỏ BÂb = BHK, BDÃ = BKH -> aBAD 00 ABHK (g.g) nẻn ABDM 00 ABKN =>DBM = KBN, = BK BN => MBN = DBK, — 77 = — BM BD Nhưvậy ABMN co ABDK (c.g.c) => BNM - BKD. Mà BKD - 90°. Do đó BNM = 90°. Vậy MN ± NB. Bãi toán 2. Cho tam giác nhọn ABC, các đường cao AD, BE, CF cắt nhau tại H. Các đưởng thảng qua A song song với BE, CF lần iưựl cắt CF. BE tại p, Q. Chứhg minh rằng PQ vuông góc với đường trung tuyến AM của tam giác ABC. (Thì giải toán qua thư- Tạp chí Toán Tuổi thơ 2 sô 4 tháng 6 nám 2003) LỞI giải. A Gọi o, K lần lượt lã giao điểm của PQ với AD, AM. Vi tứ gỉảc APHQ lá hình bình hành nên o là trung điểm của AH. Do AP n BE. BE 1 AC => AP 1 AC, Ta có PAD = BCÀ (cùng phụ vãi DAC} và ẤPH = BÃC (cùng phụ với PAB) nin AABC co APHA tg.g) -> AAMC to APOA => MAC - ÒPA Ta có ÕPA + PẴK =MÃC +PAK =90°. A Do đó AKP = 90° Vây PQ 1 AM Bài toán 3. Từ một điểm A bẽn ngoải đường Iròn (O), vẽ các tiếp tuyẽn AB, AC và cát tuyển AEF đến đường tròn (EF không qua o và B r c là tiếp điểm). Gọi D lá điểm đổi xứng của 5 qua o. Các tỉa DE, DF cắt AO theo thứ tự tại M. N. Chứng minh OM - ON. (Đê thi váo lớp 10 chuyên Toàn, THPT ũhuyèn TP. Hồ Chi Minh, nằm học 2013 - 2014) Lởi giai. A Vẽ OH 1 EF tại H => HE = HF. Ta có OA 1 BC, DC 1 BC nên OA u co => CDN + DNM = 180°. Mà CDN + CẼF = 1 80°. Do đó DNM - CEF. [1) Ta có ADNM co ACEF (g.g). Gọị Q là trung điểm của NM thi ADNQ <n ACEH DN QN 4 — = —. (2) CE HE Vi năm điểm O r H, B, A, c cùng thuộc một dường tròn đường kính OA nên CHA = CBA => CHA = BOẢ. Mà BOA = DON rên DON = CHỀ Từ đó và (1) có ADON í/5 ACHE <g.g> DN ON =»-r = 7^r(3) CE HE Từ (2) và (3) cỏ QN = ON =5 Q s 0 OM = ON. Bài toán 4. Từ mật điểm M â ngoài đường tròn (0) vẽ các tiếp tuyến MA, MB vóí (0). Gọi H là giao điểm của AB và OM, I là trung điẩm của MH. Đường thẳng AI cắt (O) tại K (K khác A). Chứng minh HK-L AI (Đề thi tuyền sinh vào lởp 10 chuyên Toàn. THPĨ chuyên TP. Hồ Chí Minh, năm học 2014 - 2015) Lời giải. Vẽ đưỡng kính AC cùa đường tròn (O). Ta có CBÀ = ẤHM ( 90°) và CAB = ẤMH- Suy ra ACBA co AAHM (g.g). Lại cỏ IM = IH, HA - HB nên ACHA co AAIM => HCA - IẤM mà KCA- IAM {cùng phụ vối CAI) => HCA = KCA. => Hai tia CH, CK trùng nhau do đó c, H, K thẳng hàng Mã ÁKC = 90° Vậy HK 1 AI. Bài tập vận dụng. Bài 1. (Lớp 8) Cho tam giác nhọn ABC {AB > AC), các dường cao BD, CE. Gọi F là giao điểm của BC và DE. Gọi M, N lần lượt lá trung điểm của các đoạn thẳng BD, CE, Chứng minh rằng hai gốc CFD và MFN cố chung tía phân giác. Bải 2. (Lớp 9) Cho Lử giác ABCD nội tiếp đưỏng trùn (O) (I khác O) vá AC cắt BD tại I. Đường thẳng vuửng góc VỚI 01 tại I cầt AD lại E- Gọi M, N lấn lượt là trung điểm của AD, BC. Chứng minh ràng ẺOI = BNI- Bâi 3. (Lởp 9) Cho Lam giác nhọn ABC (AB < AC) nội tiếp đường tròn (O). AD là đường phân giác của tam giác ABC. E chính là điểm chinh giữa cung lởn BC. Gọi M là giao điểm của ED và đưửng tròn (0). K, I lẩn lượt là hình chiếu của M trên AB, BC. Đường thẳng qua B song song vối KI cẩt Al, AC lần lượt tại N. F. Chứng minh ràng N là trung điểm của đoạn thẳng BF. sử DUNG BẤT ĐẲNG THỨC QUEN THUỘC DỂ GIẢI BÀI TOÁN cực TRỊ HÌNH HỌC PGS TS LẺ QUỐC HẢN (Khoa Toàn Oại học Vinh) T ' hi giải toán gĩài bài toán cực trị hình hgc r chúng ta thường sử dụng các bất dẳng thức đại số quen thuộc. Sau đây chúng tõí sẽ giới thiệu một số bài toán minh họa,
  • Bất đẳng thức AM-GM vài hai số
  • Cho a, b là hai số không âm thì a + k > ýab. Đẳng thức xảy ra khi và chỉ khi a - b. Bài toán 1. Cho đường tròn tâm o bản kính R và dãy cung BC (với BC < 2R}. Hây xấc định vị tri điểm A trên cung lớn BC và điểm Đ trên cung nhỏ 1 -| <1 BC dể tổng — — - + — — dạt giả tri nhỏ nhất. DA DB DC Lời giải. c Nhận xét. Vì AD < 2R nên với mối điểm D trẽn cung nhò BC luôn chọn được điểm A trẽn cung lởn 1 1 BC sao cho AD - 2R để — — “ ~r nhỏ nhất. Do dó

    AD 2R chỉ cẩn xác định vị trí của A, D sao choAD là đường ,1 1 kính của điiâng Lròr ■ và — h — — đạt giả trị nhỏ nhất. DB DC Lởi giải. Theo bất đẳng thức AM-GM ta có

    Ũ_l 22 QTĩ:. DB DC VDB DC Kẻ đưởng kỉnh EF của đưởng tròn (O) vuông góc với BC tại K. gọi H là chăn đưdng vuông góc hạ lử D xuống BC. Khi đó E, K, F cố định, Vẽ dường kính DA. Vi ÀBD =CHD = 90° , DẦB = DCB (cùng chắn cung DB) nên AABD cn ACHD (g.g). Từ đó DB. DC - 2R.DH .

    DA DQ Kẻ DJ X OE tại J thì DH = KJ < KE. DA DB DC 2R V2REK 2R BE (VI trong tam giác vuông EBF vửi dường cao BK có 2R.EK - EF.EK - BE 3 ). « . 1 1,1 .,12 Vậy giá tr nhỏ nhảt của là -—4 — — DA DB DC 2R BE đạt được khi và chĩ khi DA trùng với đưủng kỉnh EF, nghĩa là DA lả đường kính vuòng góc với BC.

    Bất dẳng thức quen thuộc. Với mọi X, y. z thì X 2 4- y 2 + z 2 > xy + yz T zx. (x + y + z) 2 > 3{xy + yz T zx).

    Đẳng thửc xảy ra khi và chỉ khi X = y = z. Bài toán 2. Cho tam giác đều ABC cạnh bằng a. M lá m&t điểm nào đỏ nằm trong lam giác. Đường thẳng qua M, song song với BC cát AB vã AC tương ứng tại E và D', Đường thẳng qua M. song song với AC cắt BC và AB tương ứng tại F và E'. Đường thẳng qua M r song song vái AB cắt AC vả BC tương ứng lại D và F\ Hãy xác định vị Lrí của điểm M dể diện tich tam giác DEF dạt giá trị lớn nhất và tim giá trị lốn nhất đó. Lởi giải. Đặt MD = X, ME = y, MF = z. Do tính chất tam giác đều ta có X 4 y 4 z = DD' 4 DA t D'C = AC = a.

    . Sade AE.AD AD.AE Ta lạ có = - — . S ABC AB.AC a 2 Tương tự ®BEF BE.BF CF.CD S ABC "" a 3 S ABC a 2 A Từ đó S DEF - S A6C“( S ADE +S BEF + S CFd) =,|2ỉl= 1- ÍAD.AE BE.BF 'ABC . CF.CD 1 a 2 -(x(a-X} + y(a-y) + z(a-z)) xy + xz + yz ^ 1 c „1 C v5 „2 V -r =* =>DEF - 7T a ABC - TT a {x t y + z) 12 V3. Vậy giá trị lởn nhất của S DEF bằng --a 2 , đạt được khi và chì khi x = y = z = l« M là tâm cùa lam 3 giác đều ABC, 2 . Bất đẳng thức AM-GM vỏi ba sô' ,t a+b + c ặabc. • Nếu a. b, c là ba số không àm thỉ ủ Đẳng thức xảy ra khi và chỉ khi a - b - c, • Nếu a, b, c là ba sõ dương thì (a + b + c)ị 1+1+1 >9. l a b C J Đẳng thức xảy ra khi và chì khi a = b = c, Bài toán 3. Cho tam giác ABC và một điểm M trang tam giác, Gọi A‘, B\ C' tương ứhg lá giao điểm của các đường thẳng AM, BM, CM với BC, CA. AB. Xác định vị tri M để
  • — — + ^7 + — — đạt giá trỊ nhỏ nhất. MA' MB’ Mơ ma mb mc t ,, u . UJÙ MA' MB r Mơ Lời giải.
  • Gọỉ H và K thử tự là chàn đường vuỏng góc hạ từ A và M xuống BC. , MA' MK SiưỊfu'. Theo đinh II Ta-lèt có -rrr - ^T 7 - ■ AA' AH S A gc Tương tự = Ị = faí&

    BB S ABC cc S ABC

    MBC 1 ^MAC + ^MAB = ®ABC MA' MB' , Mơ , nèn — - 7 +- 7 - 7 - + -T 77--3 1 - AA' BB' Cơ A Theo bất đảng thức AM-GM, ta có AA' BB' Cơ MA' + MB' + Mơ ị AA' BB' , CơYMA' , MB’ MC \ n +77777+777^7 ^TT+^r+^r >9, 1 MA' MB' Mơ 11 AA' BB' CC'Í , . t , . , . , A , AA' BB' x Cơ , _ Dù đó giã tr nhỏ nhát của — — — + — — băng 9, ' MA' MB' Mơ
  • , „„ . , . . . , , . . MA' MB' Mơ 1 đạt được khi và chí khi — = TỈ— = - 77 — = — « M AA' BB’ CC' 3 là trọng tàm cùa AABC.
  • Đặt AA' BB' CC' 7777 - X- 7777 - y, 777- MA' MB' Mơ Khi dó X, y, z > 1 và ma AA' „ .. MB .. . MC 7-777 = 7-77-7-1 = x-1; 7-7— = y - 1; 7 - 7 — = z - MA' MA' MB' MC' Ta lại có —+— +— =1=>xy + yz + zx = xyz. X y z Từ dủ MA MB MC (x-1)(y-1)(z-1> MA' MB' MC’ = xyz -(xy + yz + zx) + x + y + z-1 = x 1 y I z - 1 Y 1 1 1 \ = (x + y + z) - + - + - |-1>8. X y zj n ,, ,, . , , , . A LÍÍ1J MA MB MC Dọ đó giã hi nhò nhất của biểu thức 7777 7777- -7737 bằng 3, đạt được khỉ và chì khi X - y - z - 3 MA
  • = -7777- = 7777- = 77 fc=> M lã trọng tàm iABC- AA' BB' CC' 3
  • Bất dẳng thửc Bunhiacốpxki vãi hai cặp sỏ'

    • Với mọi a, b, X, y khác không thì (ax + by ) 2 < (a 2 + b 2 )(x 2 + y 2 ). Đẳng thức xảy ra khi và chỉ khi a ; X = b : y. Bải toán 4 . Cho góc vuông xAy. Tim gìá trị nhả nhất của biểu thức vâi các điểm B. c khác AB + ACv'3 A di dộng tràn cảc lla Ax, Ay lưđng ứng. Lài giải. Áp dụng bất đẳng thức Bunhiacốpxki ta có (AB - ACV 3} 2 s 4 (AB 2 + AC 2 ) = 4 BC 2 . Suy ra AB + ACv'3 < 2BC. Do đó Đẳng thức xảy ra khi vả chi khì AB Ac An ỉn — — — — =- <-4 AC — ABy3. ì & BC

    4 AB+ACV3 2 Vậy giá trị nhỏ nhất của biểu thức AB t AC>/3 bằng khi đủ AABC vuông tại Acó c = 30°.
  • Bất dẳng thức Bunhiacôpxki vái ba cặp sò' • Vối mọi a, b, c, X, y, z khác không thl (ax -ì- by + cz) z < (a 2 + b 2 + c 2 )(x 2 -í- y 2 + z 2 ). Đẩng thức xảy ra khi và chì khi a : b : c = X : y : z. • Đặc biệt khi X = y = z = 1 thì (a + b + c) z < 3(a z + b z + c 2 ). Dẳng thức xảy ra khi và chỉ khi a = b = c. Bái toán 5. Cho AABC có các cạnh lã BC - a, CA - b, AB - c và một điểm M nào đó nằm trong tam giác. Gọi X, y. z là các khoảng cách từ M đến các cạnh BC, CA r AB. Xác định vị trí của điểm M để biểu thức — đạl giá trị nhỏ nhất và tìm X y 2 ' giá trị nhỏ nhất đó. Ltri giải. Đặt s = s^c (không đổi). 1 Khi đó s = Syec + SyiAB + S M AC - 2 ( a + ky + cz). Suy ra ax - by + cz - 2S, Theo bất đầng thức Bunhiacốpxki ta có 2 .Jĩĩụ.iặ + jĩĩ.ị < - J í a b c ' ' a b c v 1 4- 1
  • 1 li iv: ữ [X y ZJ L* y 2 J (a + b + c) a b c (a+b+c) z X y z 2S a . b , c (a+b + c) 2 Vặy già trị nhó nhát cua h— bang - — X y z 2S đạt đưọc khi và chỉ khi X = y = z, hay M là tâm đường trôn nội tiếp AABC. Bài tập tự luyện Bài 1. Cho tam giác ABC và một điểm M nào đó nằm trong tam giác. Các đưàng thẵng AM, BM, CM cắt cảc cạnh BC, CA, AB tướng ửng tại D, E.
  • Tìm già trị nhỏ nhất của các biểu thức 4^' 2 T ísfjEf ; 'I,adJ (be ) T CF ) VMD Vme Vmf Bài 2. Cho tam giác ABC và một điểm M nào đó nằm trong giác. Các đường thẳng đi qua M song song với các cạnh BC. CA, AB tạo thánh ba tam giác nhỏ có diện tích 5^ S 2 , S 3 . Xác định vj tri điểm M để tổng S 1 + S 2 + Sg đạt giá trị nhỏ nhất. M OẠV KHR nỉK Tự HQC ui Pil HUV TÍnH ste TẠO cún HPC SMH QUA HŨ«T ĐỌnc nHúm HGOPl Glỉ HQC NGUYỄN THỊ BÌNH (707 tòa B, Mụtberry Lsne, Mỗ Lao, Hà Đông , Hà Nội) T ạl sao phải học Ngoài giờ học ? Thực tiễn trong các năm gần đây, chúng ta cúng đã áp dụng cách dạy: cho học sinh hoạt động nhóm trên lớp. Thực tẽ' cho thấy nhiều tình huống, nhiều bải toán đòi hỏi có sự họp lác của nhiều người mãi nghĩ được cách giải hoãc nghĩ được nhiều phương pháp giải khác nhau, thể nên hoạt động nhóm trên lửp đã phát huy dược ưu thế của ná. Tuy nhiên, cũng thực tế cho thấy lâp học của ta học sinh quá đông, lóp học thi không đũ tiện ích, và đặc biệt có nhiểu vấn để đòi hối phải có thời gian thu thập tài liệu, bàn bạc kĩ lưỡng, tổng kết một cách khoa học. Xuất phát lừ lẽ đó, chúng tôi đã chọn thèm mộl giải pháp nữa để tìết học được thêm hiệu quả: Hoạf động nhỏm ngoài giở học. Khi đưa ra và'n da này. chủng lòi có lường đến một số ráo cản, Thứ nhất, mặt số giáo viên nghĩ rằng học sinh bảy giỏ đã học quá nhiều : học trên lớp, học thêm, học tiếng Anh, học vi tính. ... dẫn đến không càn thởi gian để học nhóm; Thứ haì, gia đinh các em không muốn cho con em mình tụ tập khi không cỏ sự quản li của giáo víẽn. ... còn nhiều li do khác chưa kể đến. Nhung thực tể hoạt động nhóm mà chúng tói đưa ra ẻr đây khõng yêu cầu học sinh phải đến nhà nhau má mục dích giủp học sinh biết chủ động suy nghĩ, biết đoàn kết với nhau, biết cộng tác làm việc để đạt kết quẳ cao han mà lại không tổn nhiều thời gian như khi lảm việc một minh, điều mà người Việt ta còn chưa quen. Chủng tôi thực hiện như sau:
  • Chia lớp thành các nhóm khoảng 5 học sinh. mỗi nhóm chọn một học sính có khả năng làm nhóm trưởng, các thành viên côn lạl trong mồi nhóm phải có sự đồng đểu ịvê học ỈỊ/C và khả nàng sử dụng vi tính). Các nhóm này có thể duy trì trong cả một nãm học.
  • Khi được giáo víến phản công, nhóm trưâng phân vỉệc chi tiết cho moi thánh vièn Irong nhỏm. Việc này làm vào giở nghỉ giữa các tiết hoặc sau buổi học.
  • Các thành viên nghiên cứu để hoàn thành phần cõng việc của minh. Khi cần trao đổi trong nhóm, cúng tranh thủ gặp nhau trên lóp, hoặc nếu cần có thể gọi điện thoại, hoặc trao đổi qua email.
  • Nhóm trưởng Lập hợp, quyết định đáp án rồi trình bày thành bái.
  • Giảo viên lựa chạn các nhõm cỏ bải làm tết. cho đại diện nhóm trình bày trước lốp ở thài điểm thích hợp.
  • Các bài cùa các nhóm còn lại, cùng với những bài đã được trinh bày, cho dán vào Báo toán của lốp hàng ngày trẽn lâp để mọi ngưởỉ có thời gian tham khảo, học hỏi thêm. Nội dung này sẽ được gỡ bỏ khi có nội dung khác kế tlểp. • Như vậy, để có được bái tập giao cho các nhóm và các nhóm có thài gian chuẩn bị kĩ lưỡng, giáo vièn phải chuẩn bị vá giao nội dung bài tập cho học sinh trước khoảng bốn. nèm ngày đến một tuấn. • Phơững pháp hoạt động theo nhóm ngoài giỡ học này dạc biệt có hiệu quả cao đối với những bài ôn tập chương. Nội dung kiến thức và các dạng bàl tặp đã học trong cả một chương chỉ được ôn lại trong một tiết hoặc nhléu nhái là hai tiết (Chương I, Chương |t - Đại sổ lớp 8, Chương I, Chương II, Chương IV- Hlnh học lớp 8 đẽu chỉ có một tiết ôn tập). Hoạt động của các nhóm ngoài giờ sẽ giúp cho học sinh chủ động ôn tập trước một cách cố hệ thống và giúp giáo viên chuyển tải hết khối lượng kiến thức trong một thời lượng nhẫt định. Sau đáy tà một sổ vi dụ mà chủng tôi đã thực hiện. Ví dụ 1. Tiết 18. Ôn lập chương I Đại 50 8: Phép nhân và phép chia các đa thức. Hãy phân toại các dạng bài tập trong chương này. Yêu cẩu cụ thể khí trình bày;
  • Trong chương, có mấy dạng bài tập cơ bản, lá những dạng nào?
  • Mỗi dạng chọn một bài trong sách giáo khoa hoặc sách bãi tập (phần Bài tập ôn tập chướng I) để minh họa, trình bày chi tiết lài giải của bài tập nảy.
  • Những chú ý khi giải bài lập dạng nãy. “ Những sai lẩm thường mắc phài khi giải dạng bài tập này (học sinh sé nhà lại nhữhg lưu ỹ mà giáo viên khắc sáu trong từng bài học hoặc từ chính những cái sai của bàn thân hoệc cùa bạn bè đâ xảy ra trong quá trình học tập).
  • Tất cả nội dung trẽn đây, giáo viên photo và phát cho các nhóm. Khi nhận được, các nhóm họp nhanh: nhìn vào phần Bài tập ôn tập chương I của sách giáo khoa và sách bài tệp dể phán loại sơ bộ (ví dụ: 6 dạng bái tập chinh}, rói phàn cho mồi cá nhân phụ trãch việc chọn bài, giải bài, nêu chú ý cho mỗi loại bài tập, hẹn lịch họp lần 2. Trong khoảng thời gian từ lần 1 đẽn lần 2, các thành viên trong nhóm trao đổi kĩ hơn sau khi đã có thời gian nghiên cửu. Lần 2: các cả nhân tham khảo phấn việc của bạn trong nhóm, góp ỳ. sửa đổi (nếu cần) bổ sung, nhóm trưởng néu ý kiến tổng hợp vá quyết định, sau đỏ phàn cõng ngươi trinh bày thành văn bản. • Thực tê diễn ra là: vẫn cùng một nội dung bài tập này, nhưng cố nhóm phân làm 5 dạng, có nhóm phân thành 6 dạng, thậm chí có nhóm phân thành 10 dạng bài. Chỉ cần một con số phân loại như vậy. giáo viên cũng có thể thẳy được nhóm nào đã đẩu tư suy nghĩ nhiều hoặc nhóm nào đã biết khái quát hóa vãn để. Tất nhiên việc phân loại bài tập mang tính chất tưdng dối nhưng chủng ta có thể phân loại các dạng bài tập cơ bản của chương I thành 6 dạng bài tập như sau:
  • Dạng 1 . Thực hiện phép tính.
  • Dạng 2. Phàn tích đa thức thành phân tử.
  • Dạng 3. Tính nhanh.
  • Dạng 4. Rút gọn biểu thửc.
  • Dạng 5. Tìm X.
  • Dạng 6. Chứhg minh biểu thửc luòn dương, luôn âm hoệc tim giá trị lơn nhất, nhò nhất của một biểu thức, • vẫn giao bài tập cho nhóm như đă nêu ở trẽn, giáo viẻn cho học sinh chuẩn bị ôn tập tất cả mồi chương côn lại của chương trình đại số. Việc này giảo viên có thể thống bảo từ trước để các nhóm chủ động hdn VỂ mặt thời gian trong quá trinh học tập. Thực tế cho thấy: Ở chương I, bãi cùa các nhóm thực hiện cỏn có phần lúng lúng, nhưng đến chương III thi kết quả rất khả quan: Hầu hết các nhóm đều đã phân tích được 2 dạng chính trong chương Phương trinh bậc nhất một ẩn đó là:
  • Giải các loại phương trinh bậc nhất một ẩn và phương trình quy vể bậc nhất một ẩn,
  • Giải toán bằng cách lập phương trình.
  • Trong dạng 1 lại chia nhò thành: Phương trình bậc nhất một ẩn; phương trình tích; phương trinh chứa ẩn ủ mẫu.
  • Trong dạng 2 có mấy loại toán cơ bản: Toán chuyển động, toán nâng suất, toán phần trăm, toán quan hệ về số, toán lên quan đến hình học, toán phản chia đếu đặn. ... Từng chú ý khi giải moi dạng toán được các em nêu rất rõ ráng Như vậy 8D% mục đích dạy học của người thiy coi như dã đạt được. • MỘI vẩn dỂ dặl ra lá: ngoài nhptig bài ôn tạp Chương mà 'ta đã thấy rõ tác dụng của “Hoạt động nhỏm ngoài giơ học", thi những tiết học khác có áp dụng được không và tác dụng đen mức nào? Càu trả lời là: Tiéì Lý thuyết và tiết Luyện tệp đều áp dụng được. Ví dụ 2. Tiết 1 2. Bài hình bình hành (lớp B). Đây là một bài lý thuyết có rất nhiều, kỉến thửc: Định nghĩa, tính chất, dấu hiệu nhận biết hình binh hành, Nểu không khéo, người giáo viên rãt dễ đẩy học sinh vào hoàn cảnh tiếp nhận kiến thức một cách bị áp đảt, một điều nẽn trành đối với học sinh đã lương đối lớn lá học sinh lớp 8. Để trành được điều đố, giáo viên giao bài cho các nhóm như sau: ■H Hãy đọc trước bài Hình binh hành trong sách giáo khoa. 1 .. 1
  • số nhóm chứng minh tính chát 1 ; số nhóm 3 3 chứng minh tính chất 2 và số nhóm chứng minh tính chất 3.
  • Tường tự, mòi sâ nhỏm chứng minh dấu hiệu nhện biết dạng 1; 2; 3; 4; 5.
  • Hai nhóm chuẩn bị bảng phụ. Dưới sự dẫn dẳt khéo lẻo bằng hệ thống câu hỏi tích hợp. học sinh gần như củng nhau xây dựng và hiểu thấu đáo được bài học. Các chứng minh đó chính là nhũng bài toán giúp học sinh vận dụng kiến thức mà các em vừa tim dược, cộng với những hinh vẽ mà các em trình bày trên mảy tính sê giúp các em dễ nhở vả nhà rất lâu. Vỉ dụ 3. Tiết 12. Phân tích đa thức thành nhân tờ bàng cách phổi hợp nhiều phiỉơng pháp (lốp 8). Trong bài này, ngoải việc thông qua các ví dụ, giáo viên giúp học sinh rắm bắt được cách phôi hợp các phương pháp đã học: Đặt nhân tử chung, dùng hằng dẳng thức, nhóm các hạng tử dể đạt được mục đích là phân tích dược đa thức thánh tích, thl còn một phấn việc cũng cẩn thiết là giãi thiệu cho học sinh phưđng pháp lách hạng lử. Dày khòng phải lá trọng tám của tiết học nhưng hoại động nhóm ngoài giở học của trết nảy lại gây được hứng thù cho học sinh bài nó đùi hỏi sự sáng tạo cùa học sinh, học sinh phải hợp lực để nghĩ ra một cách giải không có trong sách giáo khoa. Bải tập phẩn nãy là: Đọc sách giảo khoa để hiểu 2 cách tách rói vận dụng làm câu b, c; moi nhỏm nghĩ thêm một cách khác. Kết quả là hầu hết các nhóm nghĩ ra cách thứ 3 theo yêu cẩu cùa giáo viên, mà lại là cách khác nhau. Điều này khiến các em rất thích thú! Tuy nhiên cũng có nhóm quan niệm "cách" chưa đúng: vẫn chỉ làm tương lự như cách 1 , cách 2 nhung có khác một chút má vẫn cho là cách khác. Thòng qua việc trình bày của các nhóm, các em chọn dưdc cho mình cách làm ngắn nhất, lạl dễ nhò bởi giáo viên có thể gọi đó là cách của nhóm bạn Hà hay bạn Nam. • Bài tập nhóm trên cũng có thể giao cho học sinh trang tiết luyện tập phân tích đa thửc thành nhân Lử tiếp sau đó. Trên đáy chỉ là 3 trong rấl nhíỂu ví dụ mà mồi giáo viên đểu có thể suy nghĩ để đưa ra cho hoại dộng nhóm. Kết quả thực hiện cho thấy học sinh dược khơi dậy sự say mè lự học, phát huy đưdc tính sáng tạo của học sinh. Hoạt động nhóm giúp phát huy được sức mạnh tổng hợp của các em, rèn luyện cho các em dám nói, dám trình bày một cách rõ ràng ý tưởng của minh trước đảm đóng. Các em thấy mình làm chủ kién thức, thấy được trán irọng bởi kếl quả của các em tìm ra còn được lưu trên "Báo toán" của lớp nữa. Dương như các em say sưa hạc hđn hẳn và chỉ mong dợi để dưdc cô giáo giao nhiệm vụ mỏi. Bản thân chủng tôi cũng thay rất tâm đác khi áp dụng phưdng pháp dạy học náy. cuộc THI "Vui &&Ầ0- "í¥è 20Ĩ6 Hè đã về, chúng ta lại cùng nhau tham dự Cuộc thi vui chào hè, nhằm giúp cảc bạn cùng bàn luận, chia sẻ và giải trí vãi những bài toán vui sau một năm học. Cuộc thi Vui chảo hè 2016 dành cho mọi độc giả của Toán Tuổi thd Các bạn học sinh, các bậc phụ huynh, các thầy cô giáo hay bất cứ độc già nào thấy vui và trà lời được câu hỏi là có thể tham dự. Đế bài sẽ được đãng trên Toán Tuổi thơ 2 các số 159+160 và 161+162 (ra tháng 5 và 8 nâm 2016), Các bài toán sẽ được chấm riêng vã cuối cuộc thỉ sẽ tinh tổng điểm của cả nhân, tập thể tham dự- Các cả nhân và tập thể có điểm cao nhất sẽ được nhận gìảì thưởng gổm: Quà tặng, giấy chứng nhận và tiền mặt. Đáp án và danh sách đoạt giải sẽ đăng trên Toán Tuổi thơ 2 số 164. Bài tham dự gửi về Tạp chi Toán Tuổi thơ, tẩng 5, sô' 361 Trường Chinh, Thanh Xuân, Hà Nội. Ngoài phong bì ghi rõ “Tham dự cuộc thi Vui chào hẻ 2016”. Thời hạn nhận bài giải: Hết ngày 20-9-2016 (theo dấu bưu điện). Sau đáy là nội dung câu hỏi của kì này: Bài 1. Cho các chừ số 0, 1, 2, 3, 4, 5, 6. Có bao nhiêu số có 6 chữ số khác nhau lập được từ các chữ số đã cho và các chữ số 2 và 3 đứhg cạnh nhau? CAO VÀN DŨNG Bài 2. Một chú chuột xuất phát tử đỉnh A của lưới ô vuông để đến lây miếng pho mát ỏ đỉnh B của lưới ô vuông đó, Biết rằng chú chuột chỉ được phép di chuyển trên các đưàng lưới. Hỏi có bao nhiêu đường đi ngắn nhất để chủ chuột lấy được miếng pho mát? Bàì 3. Cho bảng 7 X 5 ô vuông (hình vẽ). Haỉ bạn Nam và Hà chơi theo quy tắc sau: Tìửig người lần lượt đánh dâu X vào một 0 vuông, đến lượt người đi tiếp theo phải đánh dấu X vào õ có mộl cạnh chung vối õ người đi trước vừa đảnh dấu. Người nào không cỏ cách để đánh dấu nữa thi là ngươi thua cuộc. Hà di trưóc, hãy chỉ ra cảch đi để Hà chẩc chắn thắng Nam trong cuộc chơi này. NGUYỄN ĐỨC TẤN (TP. Hổ Chi Minh) Bài 4. Bạn Thân tách số 2016 thảnh tổng của 21 số nguyên liên tiếp từ 86 đến 106 để điển mỗi số vào một ô tròn trên "mạng nhện" sao cho các tổng ba số trên mỗi đưửng thẳng thi bầng nhau và cảc tổng bảy số trên đỉnh mỗi đa giác đều thi bằng nhau. Bạn Thán mới điển đưdc bảy số vã chở bạn điển tiếp xem được bao nhiêu cách đấy. (Há Nội) Bài 5. Cho hình lập phương ABCD.MNPQ như hình vẽ- Trên mỗi mặt cùa hình lập phương được viết một số nguyên dương. Tại mỗi đình cùa hình lập phương ta viết số bằng tổng của 3 số trôn 3 mặt chứa đỉnh đỏ. Giả sử các số tại các đỉnh A, B và p là 4, 5 và 6. D c
  • Tim số viết trên đỉnh Q;
  • Tìm số cách điển số vào Cấc măl của hình lập phương. hoang trong hảo (TP. Hồ Chí Minh) AUSTRALIAN MATHEMATICS co MPETITIO N AMC 2015 UPPẼR PRIMARY DIVISION AUSTRALIAN SCHOOL YEARS 5 AND € Time a(towed: eo minutes (Tiếp theo kì trước) PGS. TS. ĐỖ TRUNG HIỆU (Hà Nội) (Sưu tầm và giới thiệu)
  • A square piece of paper is íolded along the dashed lines shovvn and then the top is cul off. The paper is then unĩolded. Which shape shows (Aỉ (B) (C) (D) (E)
  • Sally, LI and Raheelah ha ve birthdays on differenl days in the week beginning Sunday 2 August. No two birthdays are on tollovving days and the gap between the first and second birthday is less than Ihe gap betvveen the second and third. Which day is deíinitely nol one of theìr birthdays? (A) Monday (B) Tuesday (C) VVednesday (D) Thursday (E) Friday
  • Asquare of side length 3 cm ỉ$ placed alũngside a square pf side 5 cm. What i$ the area, in square centiruetres, of the shaded part? (A) 22.5 {B) 23 (C) 23.5 (D) 24 (E) 24.5
  • A cube has tha letters A, c, M, T, H and s on its ẸÍX ĩaces. Here are two views of this cube. Which one of the following could be a tlìird view dĩ the same cube? <A) LvỊrd (B) fOi [Oi ịyrịị 1 Hpj (C) (D) <> <E> Qucstions 21 to 25, 5 marks each 21 . A teacher gives each oí three sLudents Asha, Betty and Cheng a card with a ‘secret 1 number on it. Each looks at her own number but does not know the other two numbers. Then the teacher gives them this intormation. Alt three numbers are different whole numbers and their sum is 13. The product of the numbers Is odd. Betty and Cheng now knaw what the numbers are PI1 the other two cards, but Asha does not have enough intormation. What number ìs an Asha's card? (A) 9 (B) 7 (C) 5 (D) 3 (E) 1
  • In this multiplication, L, M and N are different digits. What is the value of L + M + N? L L M _x M N M 5 M (B) 15 (C) 16 (E) 20 3 cm 5 cm (A> 13 (□) 17
  • A scientisl was lesíing a piece of metal which contains copper and zinc. He tound the ratìo of metals was 2 parts copper to 3 parts zinc. Then he melted this metal and added 120 g of copper and 40 g of zinc into il, torming a new píece of metal uvhích weighs 660 g. What is the ralto of coppẹr and zìne ỉn the new metal? (A) 1 part copper to 3 parts zinc (B) 2 parts copper to 3 parts z!nc (C) 16 parts copper lủ 17 parts zinc (D) 8 parts copper to 17 parta ánc (E) 8 parts copper to 33 parts zỉnc
  • Jason had betvveen 50 and 200 identical square cards. He tried to arrange them in rows of 4 bul had one leíl over. He tried rows oi 5 and then rows cf 6, but each time he had one card left over. Finally, he discovered that he could arrange them to form one large solid square. How many cards were on each side of this squara? (A) 8 (B) 9 (C) 10 (D) 11 (E) 12
  • Eve has S400 in Australlan notes !n her vvallet, in a mixture of 5, 10, 20 and 50 doUar notes. As a surprise, Viv opens Eve's wallet and replaces every nole with Ihe next larger note. So, each $5 note is replaced by a $10 note, each $10 note is replaced by a $20 nole, each $20 noto is replaced by a $50 note and each $50 note is replaced by a $100 note. Eve discovers that she now has $900. How mtich of this new total is in $50 notes? (A)$50 (B)$100 (C) $200 (D) $300 (E) $500 For questions 2Ỉ to 30, shade the answer as a whole number from 0 to 999 in the space provided ôn the ansvvẽr sheet. Queshon 26 is 6 marks, question 27 is 7 marks. question 28 is 8 marks, question 29 Í5 9 marks and question 30 is 10 marks.
  • Alex is desígning a square patio. paved by puLling bncks on edgẽ using the basketvveave pattorn shown. She has 939 bricks shq can USB, and designs her patio to be as large a equare as possible. How many bricks does she use?
  • There are many ways Ihal you can add three ditíerent posỉtive whole numbers to get a total QĨ 12, For instance, 1 + 5-“-6-12is one way but 2 + 2- 8=12 is not. since 2, 2 and 8 are nol all dilỉerent. If you mulliply Ihese Ihree numbers. you get a number called the product. Of aH the ways to do this, whal is the largest possible product?
  • I have 2 watches with a 12 hour cycle. One gains 2 minutes a day and the other loses 3 minules â day, lí I Sét them al Lhe correcL time, how many days will it be betore they next lògether tell the correct time?
  • A 3 X 2 flag is divided Irrto 5ÌX squares, as shown. Each square is to ba coloured graen or blue, so thai every square shares at least one edge wlth another square oĩ the same colour. In how many different ways can this be done?
  • The squares in a 25 X 25 grid are painted black or vvhile in a spiral paltern. starting wilh black at the centre □ and spiralling out. The diagram shows how this starts, How many squares are painted black? AUSTRALIAN MATHEMATICS COMPETITION AMC 2015 JUNI0R ĐIVISION AƯSTRALIAN SCHOOL YEARS 7 AND 8 Time allúMed: 75 minutes Questions 1 to 10, 3 rnarks each
  • 2015 - 201.5 equals (A) 2036.5 (B) 2116.5 (C) 2225.5 {D) 2216 5 (E) 2115.5
  • The value of X ìn the diagram is (A) 100° (B) 130° (C)110° (D) 120° (E) 90°
  • The tríp to school lakes 23 minutes. I need to be at school at 9:05 am. The latest I can leave home is (A) 8;46 am (B) 8:37 am (C) 8:52 am (D) 8:42 am (E) 8:48 am
  • WhaL IS Ihe value of 100 tvventy^cent coins? (A) $20 (B) $10 (C) $200 (D)$2 (E) $100
  • What is the area of this triangle in square centimetres? (DỊ 7 (E)6
  • When the bell rang, there were 3 teachers and 6 students in the classroom. Sev&ral students arrived atterthe bell Once everyone had arrived, lhere were 4 students for every leacher. How many studenls arrịved after thè bell? (A) 18 (3)12 (c) 6 (D)3 (E)9
  • A movie lasts for 2— hours. The movie is shown 3 ìn two equal gessions For how many minutes does each session last? (A) 85 (3) 70 (C) 80 (D) 65 {£) 75
  • Four unit squares are laid out in fĩve dífferent arrangements as shown below. Which one has Ihe largest perlmeter? (A) (B) (C) (D) (E)
  • Ari, Bryce, Cy and Eric are members of our school's baskelball leartv Ari is 136 cm lall, He is 14 cm taller than Bryce who in turn is 6 cm shorter than Cy. Eric is 11 cm tallerthan Cy. Eric's height is (A) 1 83 cm (B) 205 em (C) 178 cm (D) 189 cm (E) 177 em
  • Ana, Ben, Con, Dan and Eve are sitting around a table in that order, Ana calls out the number 1, then Ben calls oul the number 2, then Con calls out the number 3, and so on. After a person callg out a number, the next person arcund the table calls out the next number, Anyone who calls out a multiple of 7 must immediately leave Who is tha lasl person remaining at the table? (A) Ana (B) Ben (C) Con (D) Dan (E) Eve Questions 11 to 2Q, 4 marks eacli
  • -7 of 33 is equal to 19 (A)76 (B)19 (C)| (D) ỉị (E) 10
  • The diagram shows a cỉrcle and a squạre with the same centre. What ttaction of the circle (D) (E) I 11 3 13, In the addition below X, yand z represent three diỉỉerent digits. What is the value of X + y -ir z?
  • 4 X X 4 z y z (A) 9 (B) 8 (C) 10 (D) 7 (E) 6
  • A cube has the letỉers A r c, M r T, H and s on its sìx faces. Here are two views QĨ this cu be. Which one of the following could be a third view oí the same cube?
  • Fìve students are tú be photographed in a row with the tallest in the centre and the shortest two at the ends. If no two students are the same height, how many dilĩerent arrangemenls are possible? (A) 6 (B) 2 (C) 10 (D) 5 (E) 4 16 . Three boys and three girls all celebrate their birthday today, but they are each ditterent ages. The youngest ĩs 1 year Dld. The sum of the ages of the three girls is the same as the sum of the ages of the three boys, What Is the smaltest pos- sible total oỉ all six ages? (A) 22 (B) 24 (C) 28 (D) 21 (E) 26
  • Jenna measures three sides of a recíangle and gets a total of 80 cm. Dylan measures three sides of the same rectangle and gets a total of 88 cm, What is the perimeter of the rectangle? (A) 1 1 2 cm (B) 132 cm (C) 90 cm (ũ) 168 cm (E) 156 cm
  • Jim i$ running five laps ơttheschoal 3 oval, When he is of the way round his íourth lap r whal íractiữn oí his run has he completed? (A) I <B) I (C) I
  • I (E) I
  • How many two-digit numbers have the property that the sum of the dĩgits is a pertect square? (A) 15 (B) 18 (C) 13 (0) 19 (E) 17
  • On thĩs cube, opposite íaces add tD the same sum and all taces are príme numbers. (Note that 1 is not prime.) What ỉs the smalìest possible total of the íaces whích cannot be seen? (A) 41 {D) 47 (E) 37 (C)45 Questions 21 to 25, 5 marks each
  • A recipe requires 2 kg sugar, 4 kg butter, and 6 kg tlour to make 8 cakes. How many cakes can you make if you have 9 kg sugar, 17 kg butter and 28 kg flour? (A) 40 {8} 34 (C) 37 (D) 32 {E) 36
  • Two ordlnary dtce are rolled. The two resulting numbers are multiplied together to create a score. The probability of rolling a score that is a multiple of six ỉs < íc >7 < D) 3 . ‘ E >2
  • Jill and Jack are exercising at a beach. They both start from the car park at one end of the beach. Jill runs at a constant speed and Jack walks at a constant speed. When Jill turns at the end of the beach to run back, she notices that Jack is Lhen halfway along the beach. How far along the beach will Jack be when Jill next passes him? (A) Two-thirds of the way (B) Five-sixths of the way (C) Three-quarters of the way (D) Five-eighths of the way (E) Seven-eighths of the way
  • The country of Numismatica ha$ six coins of the following denominations: 1 cent, 2 cents, 4 eents.l 0 cents, 20 cents and 40 cents. Using the coins in my pocket, I can pay exactly for any amount up to and including 200 cents. Whal is ĩhe smallest number of eoins I eould have? (A) 12 (B) 10 £C) 11 (D)9 (E)B
  • In the diagram, PT = TS = SQ = QR r ZPQR - 90° and ZQPR - x° Q Then X is equal to (A) 20 (B) 25 (C) 27.5 (D) 22.5 (E) 30 For quostions 26 to 30, shads the ansvver as an integer from 0 to 390 in the space provided on the answer sheel. Question 26 is £ marks, question 27 is 7 marks, question 28 is 8 marks, question 29 is 9 marks and question 30 ìs 10 marks.
  • I write down three different po si ti ve whole numbers that add to 96. The sum of any two is divisible hy the third. What Ĩ5 the largest of these three numbers?
  • At a ỉootball match, one-third of spectators support the Reds and the rest supporl the Bkies. At halí-time 345 Blues supporters leave because Iheir te am is losing, and Ihẹ remaimng Blues supporters now make up one-third of the tolal, Hgw many Reds suppọrters are thẹre?
  • A 3 X 2 flag is divided into six squares, as shown. Each square is to be coloured green or blue, so that every square shares al least one edge with anolher square of the same coloưr. In how many differant ways can thls be done?
  • Zoltan has a list of whole numbers, all iarger than 0 but smaller than 1000. He notices that every number in his list ís either one-third of anolher number in the list or three tìmes anolher number in the li$t. What i$ the largest number o( dilĩerent vvhole numbers that can be an Zoltan's Hst?
  • In a stack of logs. each row has exactly one fewer log than the row below. With 9 logs, the tallest possible stack is shovvri. Wilh 2015 logs, how many rows are Ihere in the lallest possible Dành cho các thầy cô giáo CUỘC THI sấne TÁC CÕU HỎI uó BÀI TẬP PHÁT TRIỂn riíTítữ Lưc mùri T0fln CUR HOC SIHH BẠC THCS Nhằm tạo ra ngân hàng cảu hỏi giúp phát triển nãng lực của học sinh đồng thời động viên, khuyến khích các thầy cô giáo sảng tạo nhiều hơn nửa để có những giờ dạy hiệu quả cao, có thệ thổng cảu hỏi, bài tập cỏ chãt lượng tốt, tạp chí Toán Tuổi thơ tổ chức Cuộc thi sáng tác càu hỏi vả bải tập phát triển năng lực môn toán của học sinh bậc THCS. Đảy là một cuộc Ihi mới và là một cuộc thi lởn trong năm 2016 và nàm 2017 trên tạp chí.
  • Nội dung bài tập, câu hỏi. Các bài lập. câu hủi môn toán giúp phảt triển nâng lực học toán của học sinh. Ban tồ chức hoan nghênh các bài tập, càu hỏi có hình vẽ minh họa để các em học sinh thấy rằng mòn toán thật thú vị. Các bài tập, câu hỏi phải là các bài mái chưa xuất hiện trẽn bất kì sách, báo nào. Mỗi cá nhãn, tập thể gủl một lần hệ thống bài tập và cảu hòi gồm ít nhất 40 bài tập, câu hủi cho cả bốn lớp 6 r 7, 8 và 9 (mỗi bài tập, câu hỏi cần ghi rõ dành cho lớp mấy)
  • Đỏi tưdng dự thi. Các thầy, cố giáo, các cán bộ quản lí giáo dục.
  • Thời hạn nhận bải dự thi. Kể từ tháng 1.2016 đến hết tháng 12.2017. Các bài dự thi cấn viểt rõ trên phong bì: Tham dự Cuộc thi sáng tác cãu hỏi và bải tập phát triển nâng lực mòn toán của học sinh bậc THCS. Trong bài dự thi ghi rõ: Họ và tin, địa chỉ, số điện thoại, email và gùi ve: Tạp chí Toán Tuổi thơ, tầng 5, số 361 Trường Chinh. Thanh Xuãn. Hà Nội. Tạp chí Toán Tuổi thơ sẽ đăng các bài tệp, câu hỏi hay và các tác giả sẽ được nhận nhuận bút.
  • Tổng kết vả trao giải. Hết tháng 12.2017, tạp chí Toán Tuổi thơ sẽ tổng kết cuộc thỉ. Ban tổ chức sè chấm các bài thi dựa trên các tiêu chí: Số lượng bài tặp, cảu hỏi, chất lượng chuyên môn của từng bài, từng càu và sự đa dạng vé nội dung. Giải thưỏng gổm Giấy chứng nhện, tiến mặt và quà ỉậng. Theo dể nghị của bạn dọc, Tạp chí lùi thời hạn nhận bài dự (hi đến tháng 12.2017 BAN TỔ CHỨC CÁCH THỨC GỬI BÀI CHO TOÁN TUổI THƠ Vởi học sinh giải bài. MỄi bài toán trong mục thi Giải toán qua thư, mỗi bồi tham dự trên các chuyên mục khác, đều phải làm riêng trên một tò giấy và ghi rõ các thông tin cần thiết: Tham dự mục gì, số tạp chí, tên để bài, họ và tẽn học sinh, lớp. trưâng, quận (huyện), tỉnh (thành). Học sinh nên ghi thèm địa chỉ nhả, số điện thoại nhà riêng, số điện thoại phụ huynh (nếu có) đl Tòa soạn tiện gửi phần thưởng. Riêng cuộc thi dành nử sinh, mỗi số tham dự, thí sinh cẩn dán 1 ảnh 4 X 6 vảo bài giải, Với các cộng tác viên gửi bài. Các bài viết cùa tác glà gửi đến Toán Tuổi thơ chưa được đăng Ồ bất cứđău. Bãi đã gửi Tạp chi không gửi cho báo khác, Nếu là bãi sưu tẩm, tác giả cần viết rô vào bãi gửi lã sưu tầm vã dln nguổn sưu tầm Nếu là bài dịch, tác giả cần ghi rõ vào bãi gửi là dịch và gủí kèm bản gốc. Các cộng tác viên có thể gửi bài qua đường bưu điện đến Tòa soạn: sổ 361 Trường Chinh, Thanh Xuân, Hà Nội hoặc gửi email đến: toantuoitho@ vnn. vn CẬU LẠC Bộ TÕAHTUỈITHS CÂU LẠC BỘ TOÁN TUổl THƠ TOÀN QUỐC 2016 T ừ ngày 6.6.2016 đến ngây 8.6.2016 sẽ diễn ra Cảu lạc bộ Toán Tuổi thơ toàn quốc 2016 tại trưởng Ngòi Sao Hà Nội, Q. Thanh Xuân. Hà Nội. Tham dự là CÉC Câu lạc bộ Toán Tuổi thơ đến lừ mọi miền đất nước. Khác vởi ũlympie Toán Tuổi thd toàn quổc. đề của Câu lạc bộ Toán Tuổi thơ toàn quốc 2016 là các bài toán bằng tiêng Anh đề các em học sinh dược tiếp cận VỚI cách thi chuẩn Quốc tể. Vối các bạn học sinh bậc THCS thl cỏ thêm hai vòng thl là Tiếp sức toán và Du tịch Toán học. Thi cá nhãn
  • Đề thi gổm có 16 câu bằng tiếng Anh, 15 càu đẩu chì ghi đáp số (mỗi càu 5 điểm), cáu 16 (25 điểm) học sinh trình bày lời giải bằng tiếng Anh vào tử trả lài.
  • Thài gian làm bài là 30 phũl Thi liên CLB {Dành cho học sinh cáp THCS) Vòng 1. Tiếp sức toán
  • Sáu thí sinh của moi CLB lẩn lượt giải 6 bài toán chỉ ghi đáp sổ ("các thi sinh ngói theo thư tự sô" háo danh tảng dân). Thí sinh nào giải xong, nộp bài cho giảm khảo, thí sinh tiếp theo mởi được nhận đề để giải. Thời gian toi đa là 30 phút cho 6 bãi toán.
  • Moí bài giải dung dược 2 điểm, giải sai 0 diểm.
  • Các CLB giải bài nhanh nhất được cộng 1 điểm (cỏ không quả 10% tổng sổ các CLB tham gia thi đấu được cộng điểm).
  • Sau khi các CLB giải bài xong và nộp bài cho giám khảo thì giám khảo mở đáp án và chấm trực tiếp vào bãi vá cộng tổng điểm. Thư kí trường thi sẽ thu kết quả tạl bàn của giám khảo. “ Cỏ 6 đến 8 CLB điểm cao điểm nhất dược vào thi đấu vỏng 2. Du tịch Toản học để tranh giải Vàng, Bạc, Đồng (nếu có các CLB bằng điểm nhau thì dùng càu hỏi phụ để phản loại). Vòng 2. Du lịch Toan học
  • Có 6 thành pho Hà Nội, Nam Định. Thanh Hóa. Vinh, Huế, TP. Hồ Chi Minh cho các CLB đến tham quan. Đảy là 6 noi đặt trường thi xưa. Hai giám khảo sẽ lá chủ nhàn của bàn đạí diện mãi thành phố đô,
  • Sáu thí sinh của mổi CLB cùng giải 6 bái toàn. ■Đội trưởng đến thành phố thứ nhất để lấy để bài 1 (theo chỉ định của Ban tổ chức) sau đỏ các em học sinh của CLB củng glảl bài roi độỉ trưởng nộp kết quả cho giảm khảo ở thành phố thứ nhất, nếu kết quả chưa đủng thì giám khảo sẽ yèu cầu làm lại đến khi não CLB dỏ đưa ra được kết quả đúng bái 1 thl CLB đó mớì có được chữ kí cùa giám khảo ã thánh phổ thứ nhất để đến (hành phố thứ hai nhận dể bài 2 giải tiếp, cứ tiếp tục như thể cho đến bài 6. Tổng thời gian tối đa để làm cà 6 bải toán là 30 phút. Vòng 2 sẽ kết thúc khi hết giở hoặc đã có 2 CLB về đich (CLB nào đã về đích thì tinh nguyện viên phụ trách CLB dò phất cờ đề Ban tổ chửc biết). Sau khi cô trổng báo hiệu hết gíử thì Thư kí trường thi sẽ đến thu kết quả tại bán của các CLB. Moi bài giải đũng đưỌc 2 điểm.
  • Sau khi kết thúc ha! vòng thi, Ban td chức sẽ cộng tổng điểm cả hai vỏng thi và xếp giải. TTT CLB11. Finđ X given thai x-500 x-403 , x-71 „„ — 7T-— + - — — I' —7 = 10. 43 61 103 CLB12. Prove ttiat n ?Q - 11 is not divisible by 2016 for all positive integers n. CLB13. Given X > 0. Find the minimum value of the following expressicn. ( 1 \ ( 11 2 r lì ; x+— +4 X + — -3 1 H- X X 1 X \ / V / CLB14. Given the numbers a, b, c, and d such that a 2 + b 2 = c 2 + = 2016 and ac 4- bd - 0. Find the value of the following expressỉon M - {a + b) 2 + (c + d) 2 - CLB15. Let ABC be an equilateral triangle with its height equal to h. M is a point on the ínside of the angle BÃC. Let MD be a llne perpendỉcular to BC at D, ME perpendlcular to AC at E, and MF perpendicular to AB at F (MD - X. ME - y. and MF - z). Given that 2x = y + z, calculate X in terms of h. NGUYỄN DỬC TẤN (TP. Hồ Chi Minh) CÂU LẠC Bộ TOfin TUỔI THO CLBl.Ta có M = 1 + ■ 52 (ĐKXĐ X 1963). 1963 -X « Xét x> 1963 =>1963 - X < 0. Do đó M < 1. (1) • Xét X < 1 963 => X < 1 962 (vì X c Z) => 1963 -x>1. Do đó Mí 53. (2) Từ(1), (2) thì Mí 53, VX€ z. Đẳng thức xảy ra khi vả chỉ khi X = 1962. Vậy M có giá trị lớn nhất lá 53 khi X - 1962. CLB2. Đặt d = ƯCLN(a, b) thì a = dx, b = dy, với X, y nguyên dương và ƯCLN(x, y) - 1. Ta cỏ a 2 + b 2 = d 2 (x 2 4 y 2 ) chia hết cho ab = đ 2 xy Từ đỗ X 2 4 y 2 : xy => X 2 4 y 2 : X và x z 4 y 2 : y => y 2 : X và X 2 ■ y mà ƯCLN(x, y) = 1 nén X = y = 1. Do đó a - b. Vậy p = 2. CLB3. Ta có 1 4 a 2 b 2 = abc(a 4 b 4 c) + a 2 b 2 = abỊca 4 c(b ■ c) 4 ab] = ab[a(b 4 c) 4 c(b 4 c}] = ab(a 4 c)(b 4 c). Tương tự 1 4 b z c 2 = bc(a 4 b)(a i c) vả 1 4 c z a 2 = ca(a 4 b)(b 4 c). c 2 (a 4 b) 2 ab(a 4 c}[b 4 0} (TTT2 số 157) A Do đó Q =■ bc(a I b)(a I c)ca(a ị b)(b I c) = 1 . Từ(1), (2), (3) suy ra 1 1 S ABD + S A0N ( S Aflữ S ACẽ) + — S ADÉ

    1 1 = 2 ( S ABD + S ACE + S ADE ) = 2 S '

    Nhận xét. Các bạn sau cớ lởi giải tốt 8 được thưởng ki này: Lê Ngọc Hoa, Trần Bình Minh, Nguyễn Công Huấn, 3E1, THCS VTnh Tưởng, Vĩnh Tưởng. Vĩnh Phúc; Nguyền Khắc Thái Bình, 8B, THCS Nguyễn Thượng Hiền, ừng Hòa, Hà Nội; Nguyễn Tuệ An, 7C, THCS Bạch Liêu, Yên Thành, Nghệ An. NGUYỄN HIẺP CLB4. Ta có T = n 2 - 201 5n 4 8 là tổng các chữ số của sổ tự nhiẽn n nén 0 < T < n. Dề thảy n > 1 =-> T > 0 =>(n- 1 )(n - 2014) >T>0=>n> 2014.(1) Mật khác T < n => n 2 - 2016n 4 8 < 0 => n 2 - 2016n < 0 => n(n - 2016) < 0 => n < 2016. (2) Từ (1), (2) vả ne N^n=2015. Thử lại n - 201 5 cá tổng các chữ số là 2-041 4 5
  • 8 và n 2 - 2015n 4 8-8, thỏa mán. Vậy n- 2015, CLĐ5. Gọi K là trung điểm của AE. Ta có AARD = AACE (g.c.g) = S ABD = ^ACE ) Ta có AADN = AADK (c.g.c) S ADN = S ADK- Vì AK = ^AE=>S adk =^S aữe (3) 2 2 PHONG VAN VŨ KIM THƯ (Nghĩa Đõ, Hả Nội) N ếu bạn muốn được đi du học thi sau khi vượt qua các bải kiềm tra viết, bạn cền phải trải qua vòng thi phóng vấn. Thường các bài phỏng vẩn có nội dung xung quanh những càu hồi về bản thân, gia đình và trường hạc crta bạn. Dưởi đáy là một vi dụ VỂ bải phong vấn đó. Bạn hãy tự trà lởi và gửi cáu trả lời về ỉòa soạn. Ki sau chúng tồi sẽ đàng bài trả tòi.
  • Hello,
  • My name is . I am conducting your intervievv today.
  • Please sit down. i Please have a Seat.
  • Firsl of all, whal is your name? ì Are you Vu Thanh Nam? / You are Vu Thanh Nam, aren'1 you?
  • Are you a student? / Are you a pupĩl?
  • What grade 3 re you in? i Have you tìnished grade 9? I You've fìnished grade 9 haven't you?
  • When dìd you fínish grade 9? I How long ago did you íìnish grade 9? / Did you fìnish grade 9 thís year or lasl year? You íinished grade 9 Ihis year r didn't you ị
  • VVhere do you study? I What is your school? I What is the name of your school?
  • Where is It? / Where is your school located? I Is your sehool [n the City or in the suburb? / Is. your school in near the centre of the City?
  • Is it a Public school? I 1$ it a Government school? / Is it a privale school? I Is it a Public or a private school?
  • Is ỉt a good school? / Is it a good one?
  • Is the school far from your house? I How far Is il from your school to your house?
  • Haw do you often go to school? i Do you go to school by bicycle, by bu$ or by motorbike?
  • Is your school very big? i How big is your school? / How many students are there in your school? / How many teachers are there in your school?
  • Oh, thal's very big, ỉsn't it?
  • Do you love your school?
  • Why do you love your school?
  • How many days a week do you go to school? / On which days of the week do you go to school? ì Do you go to school every day of the week?
  • Are you going to school now? / Why? Ị Why not?
  • Do you go to school in the mornỉng or in the aílernoon?
  • How mạny lessDns / periods do you usually have every day?
  • What subjects do you study at school ? ! Can you tell me what subjects you study at school?
  • What subjects are you good at? / What subjects are you not very good at? Are you good at Mathematics? Physics?
  • Oh, you do study Engllsh, dqn't you? / Are you good at English? / How many years have you studied English? -Doyou like English?/ Mathematỉcs?/ Literalure? / Why? / Why not? Whal subjects do you like? / What subjects don't you like?
  • What are your marks Ị scores in Mathematics? Physics? / What is your average in Mathematics?
  • Do you receive / Have you received any awards for your study achìevements? ỉ What awards have you received íor your achievements?
  • How do you assess > judge your own sludy resulls? Are you a good sludent? / an excellent student? I an average student?
  • Do you brỉng your school records? í Can I see them? I Can I see your school records?
  • Tell me somethỉng about your family.
  • Is your family a big one? How many people are there in your íamily? Have you got any brothers? / Have you got any sisters?
  • Is she your elder or your younger síster? t How old is she? / How old are you?
  • What’s your father's job / occupation? What does your íather do?
  • Whal , s your mother's job ị occupation? What does your mother do?
  • Does your mother I íalher Ị Do your parents help you in your study? Do you ask your mother/ tather / parents to help when you have a difficult problems I queslìons?
  • Does your molher teach your class? Has your mother ever taught your class? / Does your mother teach your sister s class?
  • Is she a good teacher?
  • Do you think ìt's 3 good thing / an advantage to have your mother as a teacher in your school?
  • Do you take extra lessons out of class? 52 Gẫm AI ĐÚNG? Đài toán. Cho điểm A nằm ngoài đường trôn (O; R), vối OA = 2R. Gọi N, p thứ tự là giao điểm của OA vãi (O) sao cho p nằm giữa A và N. Vẽ các tiếp tuyến AB, AC của đưàng tròn (O) vỡl B, c lã các tiếp điểm. M lả điểm trẽn cung BC chứa p của đường tròn (O). Tiếp luyến Lạl M của đường tròn (0) cắt AB, AC lẩn lượt (ại D r E. Bạn Mùi nói rằng: "Sổ đo của góc DNE nhỏ hơn 30°"- Bạn Thân nói rầng: ‘Sô do của góc DNE lớn hơn 30°''- Bạn Dậu nối rằng; "Số do cùa góc DNE bầng 30 D ". Theo bạn thì ai nói đúng? NGUYỄN ĐỬC TẤN (TR Hồ Chi Minh) KHÔNG VẼ DƯỜNG TRÒN (TTT2 SỐ 157) • Già 5Ử tam giác nhọn ABC có trực tám H, nội tiếp đường tròn (ũ). Gọỉ J là trung diểm của AH thì J là tâm đưởng trùn đường kính AH. Cách 1. Dựng đưàng kính AP cùa đưửng tròn (O). Kẻ dương thảng PH cắt đưởng tròn (Q) tại điểm K thì ẤKP - 90 p nên ẢKH - 90° , do đỏ điểm K thuộc đường trân dường kính AH và K là giao điểm của (O) và (J). Cách 2. Kẻ đưửng thẳng AK 1 0J và cắt đường tròn (O) tại diểm K thì 0J là đưởng trung trực cùa AK, do đó JK = JA= JH, suy ra điểm K thuộc đường tròn đương kính AH và K lã giao điểm của (O) và ( J). Cách 3. Dựng trung điểm M của doạn thầng BC. Kè đường thẳng MH cắt đường tròn (O) tại điểm K. GọiAPIà đường kính cửa (Q) thì ẤBP = ẤCP =90° nèn BHCP là hình bình hãnh, do đó PH đi qua trung điểm M của BC, suy ra ẨKM = ẤKP = 90°. Vậy điểm K thuộc đưòng tròn dường kính AH vả K là giao điểm của (0) và (J). • Chú ý. - Trong trường hợp riêng khi AB - AC thi điểm K trùng vởi điểm A. 'Với tam giác ABC tù thi bốn điềm H, p, M, K vẫn thảng hàng, chửng minh lương tự. _ Nhận xét. Các bạn sau dược nhận phấn thưởng vi cè lời giải đúng và chứng minh đầy đủ: Lẽ Thí Thu Thái, 9E1, Lê Ngọc Hoa, 8E1, THCS Vĩnh Tưởng, Vĩnh Tưàng, Vĩnh Phúc; Nguyễn Vãn Cường, 8A, THCS Hợp Tiến, Nam Sách, Hảt Dương: Nguyễn Minh Tràng, 9A2, THCS Yẽn Phong. Yèn Phong, Bắc Ninh. ANH COMPA THỬ TRẢ LỜI TOÁN HỌỔ LÀ GÌ? VŨ KIM THỦY Thầy chủ nhiệm và ũác em học sinh ngồi quày quần kiểu hàn tròn. Học sinh 1: Cậu cỏ được vể quê nghỉ hè không? Học sinh 2: Không nhưng lở được đi nghỉ mát ở Thỉnh Long. Học sinh 3: Này, thôi đi. Bắl đầu sinh hoạt rối. Thưa (hầy, thẩy hưâng dln chúng em một chương trình gì thật thú vị chứ ạ? Thầy Thủy: Chương trình thì khác đấy, còn hay đến mửc nào để các em nhận xét. Chúc cáu lạc hộ dế hay khó cùa chúng La phát triển. Học sinh: Chúng em cũng chúc thấy mạnh khỏe và cỏ nhiều chương trình thú vị. Tháy Thủy: Hõm nay thầy trò mình cùng nhau trao đổi VẾ một để tài rất đặc hiệt nhẻ. Càu lạc bộ của chúng ta là câu lạc bộ vế những vấn để toàn. Vậy (ôi hỏi các em nhé: Toán học là gi nào? Học sinh 1: Thưa (hãy toán học lá một khoa học tự nhiên nghiên cứu các con sđ và các hình ạ. Học sinh, 2: Thưa (hẫy toán học là hình học. dại số và số học hợp thành ạ. Thẩy Thủy: Đáy là mộl cáu hẻi khó đấy. Nhà toán học Nga A. A. Markov đã trà lời: Toán học là cái gì mà c. F. Gauss, L. Chebysev, Líapunôp,
  • Chiècơlổp vá (ỏi nghiên cứu. J. H. Poỉncaré thl cho rằng: Toán học - Đó là nghệ thuật gán cho nhĩểu thứ khác nhau cùng mộl tên gọi. Học sinh 1: Thưa thầy thế thi làm sao mà hiểu dược toản học là gi ạ? Thầy Thủy: ứ, đinh nghĩa toán học rất khó thể hiện qua ít cáu chữ. Trước đây người ta cho rằng toán học là khoa học về quan hệ giữa số lượng và hình dạng không gian của thế giới hiện thực, Vi vậy người la xểp toán học vào hàng ngũ các khoa học tự nhiên. Gòn bây già, chủng ta Ihừ bẩt dầu từ dgi tượng nghiên cứu của toán học. Các em thử trả lõi cho tỏi xem các nhà hóa học nghiên cứu gi nào? Học sinh 3: Các nhà hỏa học nghiên cứu các chất và các phản ứng tương tác giữa chúng ạ. Thầy Thủy: Còn các nhà sinh vậl học nghiên cứu gì? Học sinh 2: Các nhà sinh vật học nghiên cứu về động vật, thực vật ạ. Thấy Thủy: Cảc nhà thiên vàn học nghiên cứu cái gi nào? Học sinh: Các nhà thiên vãn hoe nghiên cứu các thiên thể và toàn bộ vũ trụ ạ. Thầy Thủy: Các em trả lởi đủng lẩm. Đó chính là các hiện tượng tự nhiên. Vi vậy người ta xẽp Vật lí, Hóa học, Sinh vật học. Thiên vãn học, ... là các khoa học tự nhiên. Học sinh 2: Thua thấy mình đang nói chuyện về toán học cơ má ạ. Thầy Thủy: Em đìmg vội. Tôi muốn hồi các em bây giồ đáy. Toán học thì nghiên cứu cái gì nhỉ? Học sinh 1 : Thưa thầy toán học nghiên cứu điểm, dưàng thẳng, con số, các phương trình và các hình khác nhau ạ. Thẩy Thủy: Thế các con số cứ tón tạl Irong (ự nhiên không? Học sinh 1: Có ạ. Học sinh 2: Khàng ạ. Thầy Thủy: Em bảo có vi sao nào? Học sinh 1 : Thưa thầy em thấy con sô d máy điện thoại, ở quyển sách, ở tớ lịch và nhiều nơi khác ạ. Thầy Thủy: Thế trước khi có các nhà thiên vãn thì đã có cảc hành tỉnh chưa? Học sinh: Cố rói ạ. Thấy Thủy: Nếu không có các nhã khoa học thì động vặt, thực vật có tổn tại không? Học sinh: có ạ. Đáy là sự khác nhau giữa toán học và khoa học tự nhiên. Điểm, dưdng thẳng, hàm số, ánh xạ, (oán tử, ma trận, ... là cái mà con người nghĩ ra, những 54 nhà toán học nghĩ ra. Muđn nghiên cửu một đối tượng hay một hiện tượng nào đó bằng toán học thi phải gạt bô tất cả đàc điểm về tính chất của đối tưạng và hỉện tượng, chỉ giữ lại nhũfhg đặc trưhg cho số IƯựng vá hình dạng của chủng và trừư tượng dần dần lèn. ĐỐI tượng nghiên cứu của toán học là những cãi không phải sắn có trong tự nhiên. Đó là những gì rrâ các nhá loán học nghĩ ra. Vì vậy toán học không phải là khoa học tự nhiên. IMhiểu người nhầm lẫn điều này. Học sinh 3: Thưa (hẩy trong tự nhiên có những vật mà toán học nghiên cứu ạ. Thầy Thủy: Em thử ví dụ xem nào. Học sinh 3: Thưa thầy cái vành nón„ cái vành xe đạp, cải vành mâm lá nhữhg hình tròn ạ. Thẩy Thủy: Tất cả nhữhg cái đó đểu chỉ lả hình ảnh của hình tròn thôi. Từ thực liền cò các vật đó các nhà toán học mỏi nghĩ ra một hình hình học là hình Irỏn. Vì thế giống như nghệ thuật, toán học là phưong tiện độc đáo của nhận thức Giống như các hình tượng nghè thuật, các hình ảnh toán học lá mội hình thức đặc biệt cùa toán học dùng để phản ánh thực tiễn. Học sinh 2: Thưa thầy nhưng như thế mồi người trừu tượng theo một cách, tưởng tượng khác nhau (hì sẽ dẫn đến các khái niệm rất khác nhau thi sao ạ? Thầy Thủy: Mặc dù là lự do tưửng Lượng và trừu tượng hóa nhưng nó vẫn theo quy luật của tư duy. Ví dụ để định nghĩa hình vuông, ngươi ta phải dựa vào hỉnh chừnhệt hoặc dựa vào hlnih bình hành, muốn định nghĩa hình bình hành thi cần định nghĩa tứ giác, muốn dịnh nghĩa tứ giác thi cẩn định nghĩa đường gấp khúc ... Cử như thế mãi đến cuối cùng các nhà toán học đéu thong nhẩt là phải còng nhận một số khái niệm cơ bàn như điểm, đương thẳng, mặt phẳng, ... Do dó dù là ỏ châu Á, châu Phi, châu Mỹ hay ỏ bát kì đâư thì cử nói đến hình vuông người ta đều tưởng tưdng ra đó là hĩnh có 4 cạnh bằng nhau và có 4 góc vuông: đều tưâng tượng ra hai dường chéo của hình vuũng lã bằng nhau, bằng nhau má không thể có hai vật gì trong tự nhiên có thể bằng nhau như thế được. Điểu này cũng tương tự nhơ con rông tuy không có trong thực tế nhưng người Trung Hoa và người Việl Nam tưàng tượng khã giong nhau. Còn việc trừu tượng cỏ thể theo hướng này, hương khác sẽ nảy sinh gác khái niệm khác nhau và vl vậy mới làm cho toán học phát triển. Ngày nay toán học đi nghiên cứu các khải niệm trừu tượng hon rất nhiều so với các quan niệm về số và cóng thức. Các khái niệm đó sinh ra vó số vấn đề mà muốn glảl quyết chúng ta còn phải đưa ra những khải niệm khác còn trừu tượng hơn. Học sinh 1: Thưa thầy thế thi làm sao mã có Ihể ửng dụng vào cuộc sống được ạ. Thầy Thủy: Toán hgc có thể ứng dụng vào cuộc sống một cách trực tiếp nhờ giải các bài toán đặt ra từ thực tiễn. Song nhiễu hơn cả là hàng nãm, hàng chục nãm sau nó được ứng dụng rất bất ngỡ vào cuộc sống. Ví dụ lí thuyết nhóm chính thức ra đới vào giữa thế kỉ XIX mà mãi đến nãm 1890 nhà bác học Nga Phêdõrỏp mài dùng lí thuyết nhóm vào nghiên cứu tính thể học, giải quyết bãi toán phân loại các mạng tinh thể trong khủng gian. Trong Vật lí, li thuyết nhóm trử thành cóng cụ nghiên cứu có hiệu lực cửa cd học lượng Lử. Chính vì thế người ta gọi toán học là người đay tớ và là hoàng hậu của mọi khoa học. Để nghiên cứu toán học thì một nhà toãn học làm việc cần mẫn cũng chĩ có thể nắm được khoảng 5% toán hạc hiện đại. Do tỉnh chết hiện đại ấy mà toán học vừa là khoa học cổ nhất, vừa lá khoa học trẻ nhất, Toán học thế kí XXI là một cây đại thụ khổng lồ mà muôn cành nhánh của nó đàm chổi, nảy lộc trèn một gốc cây là toán học cổ truyền. Chỉ trong hơn 20 phút chúng La định trả ĩởi càu hỏi toàn học là gi thi thặt khó. Thôi cảc em hãy bằng lòng vđi những giải thích như vậy nhé. Dần dần chủng ta sẽ hiểu hơn nhất là với em nào trử thành nhà toán học. Vui cười □ 71; - Này cặul Bãy giở lớ không còn Ị à học sinh bét lởp nửa rối. Cõ giáo: - Náy Ti, em bỉếl ngủ gặt trong lớp là không tốt chứ? Tĩ: - Vãng. Ngủ gật thì sẽ bị mói cổ ạ. Tồ: - Vậy ã? Chức mửng cậu! Ti: - Có gì mà mừng! Tồ; - Sao lạ ỉ không mừng? Cậu tiến bộ thế kia mã! 71; - Tớ chuyển thảnh học sinh bét trường rồi. 71; - Tớ thấy sách báo nói rằng nước ngọt chinh !à nguồn sống của muôn loâi. Tè: - Đúng ròi. 7T ' Thế mà mẹ tớ cứ cấm uống nước ngạt, nhất là trước bữa cơm. Trong giờ địa li. Cõ giáo: - Vàng là một khoáng sản quý. Em cô biết vãng được phân bô' nhiêu ở đáu không? Tồ: - Thưa CÕ T ở trong các hiệu vàng ạ. ĐỖ HỒNG THỊNH (Xóm ì ỉ, Xuổn Thành, Xuân Trường, Nam Định) Trang tbơ Trang tbơ Trang tbcr íH- Trang tbử ĐẶNG TOÁN (An Dồng, Dông Giang. Dỏng Hưng. Thái Bình) Tâm Sự hạt nước Tói ỉà con của mẹ sóng Hay lẽn chơi tít tầng không cao ười Cùng mây giỏ đến mọi nơi Ở đáu khò khát ỉà tói tôi liền Khí nào bạn ihấy mưa tuôn Ấy là tôi nhô mẹ hiên dòng sông Ve íhâm cho thổa nỗi lòng Rồi mai íõi ỉạí ìhong dong giúp đời BÍNH NAM HÀ Thư Dầ truyện ngán Thơ là câu chuyện xảy ra hàng ngày Viết nên bằng tihạc điệu Truyện là bài thơ dài Mang giai điệu khác Rung lên nhiều rung động lòng ngi0i Củng như thơ gửi đến cuộc đời ước mơ xa và niềm hi vọng Tâm sự chày bỏng bộn bể trên uỏn vẹn trang thơ Hè 1985 Kì 2 3 Hây Ihay các chữ cải bởi các chữ số. Các chữ khác nhau biểu diễn các chữ số khác nhau. Lài giải cẩn có lập luận lôgic, FŨRTY
  • TEN TEN s I XTỸ TRƯƠNG CÔNG THÀNH (Sưu tắm) ặ ■ ta m m Mtmmm- ■ b ta ta m m 23ẵini HINH HỌC {TTT2 sổ 157)
  • Các đường thẳng vuông góc Một góc có số đo bằng 90° là mộl góc vuỏng. Nểu hai dưỏng thảng cắl nhau tại góc vuông thì hai đường thẳng đó là vuông góc. <1 90° d 2
  • và d 2 là hai đường thảng vuông góc, được kí hiệu là d 1 -L d 2 - Kí hiệu gốc vuông trong một góc ở nơi giao nhau cho biết rằng hai đường thẳng đó vuông góc.
  • Các đường thẳng song song Nêu hai dường thẳng nằm trong cùng một mặt phảng không cắt nhau thì haỉ đường thảng đó song song với nhau. Trong hình vẽ d i d 2 các đường d 1 và d 2 song song với nhau, kí hiệu là d 1 // d 2 . Nẻu hai đường thẳng song song được cẳt bỏi một đường thẳng thứ ba. xem hình dưới đây, khi đó số đo góc có mối quan hệ như đâ được chỉ ra, trong đó x° 4 y° Ị > "7 > Nhận xét. Tòa soạn rất vui khi — nhận dược một “núi" bài tập cùa càc bạn gửi đến. Đa số các bạn đều dịch đúng. Tạp chỉ ưu tiên gửi quà cho các bạn có bài dịch chính xác vã trinh bày cẩn thận là: Phan Thế Anh, THCS Trần Phu, TP. Phủ Lý, Hà Nam; Phạm Thị Minh Ánh , 7A2, THCS Trưng vương, Mé Linh, Hả Nội; Phạm Hồng Ngọc , 7B, THCS Nguyễn Trãi, Nam Sảch, Hải Dương; Nguyễn Thị Xuân Nhi , 6A1 , THCS Hồng Bàng, Hồng Báng, Hải Phòng; Nguyền Xuân Thành Đạt, THCS Thị trấn Gỉa Linh, Gia Linh, Quảng Trị. 58 ETERNA VŨ cuộc THI CÁC CLB TOÁN TUổl THƠ TP. SƠN LA N gày 24.4.2016, sân trường tiểu học Tô Hiệu, TP. Sơn La, tỉnh Sơn La tưng bừng cở hoa chào đón 16 Câu lạc bộ Toán Tuổi thơ xuất sẳc nhất đến từ 16 trường trẽn địa bàn thảnh phồi về dự Giao lưu các Câu lạc bộ Toán Tuổi thơ khối lớp 4 thành phố Sơn La năm học 2Q15-2016. Các Câu lạc bộ đã trải qua ba vòng thi nghiêm túc nhưng không kém phần sóí động: Thì cả nhản: Tiếp sức đồng đội; Du lịch Toán học theo mô hình của tạp chí TTT. Cô Đoàn Kim Xuyến, Trưởng phòng Gũ - ĐT cùng các Phó Trưởng phòng: Thầy Trần Quốc Bình, thầy Lò Ván Hưng, thầy Đào Vãn Quang; cỏ Lẽ Thị Hoài Thu, Chuyên viên; thầy Nguyễn Xuân Bắc, Phó Hiệu Thi cả nhé n trưởng trường TH Tô Hiệu; cô Hoàng Bun Luông, Phó Hiệu trưàng trường TH Chiếng Xôm đả chỉ đạo và tham gla tổ chức thành cõng buổi Giao lưu. 56 học sinh được trao Giải cá nhàn, gồm: 01 giải Nhất (Nguyền Trần Tuấn Kiệt, TH Chiềng sinh), 06 giải Nhĩ, 17 giải Ba. 32 giải Khuyển khích. Giải tập thể gồm: 01 giải Nhất (trường TH Quyết Tãm), 02 giải Nhì, 03 giải Ba vã 04 giải Khuyến khích. ThS. Vũ Kim Thủy, Tổng biên tập tạp chí Toán Tuổi thơ đã tới dự, tàng sách cho thư viện trường Tiểu học Tõ Hiệu, tặng quà cho toàn bõ học sinh tham dự và trao giải thưởng cho các thí sinh đoạt giải. pv Tiếp sức c lỗng đội ũu ỉịch Toàn học Sau mộí ựông thi ThS. Vũ Kim Thủy tặng sấch cho thư viện ỉ rường TH Tồ Hiệu Hướng tới 50 NĂM RECSAM N àm 1999, chủng lôi là ba người Việt Nam đến dự một khỏa học ở thành phổ Penang, Malaysia, thành phố lởn thứ hai của nưởc bạn, nằm trên đào, có cầu Penang dài 13 km nổi vãi đất liền. Tất cả có 37 người đèn từ 10 nước ASEAN gổm các giáo viên dạy từ dại học đến tiểu học, một sô' bỉên tãp viên và cán bộ quản li Giáo dục. Tèn khỏa học là Creative and Critical Thinkịng in Mathematìcs For Gifted Learners. Đây là một trung tâm bổi dưỡng vể phương pháp giảng dạy các môn Khoa học và Toán học cho cả Đông Nam Á, được lập ra từ năm 1967. SEAMEO tên đầy đủ là Southeast Asian Ministers of Education Organisation lức lả Tổ chức Bộ trưởng Giáo dục các nước Đông Nam Á. Còn SEAMEO RECSAM la Soưlheast Asian Minislers of Education Organisalion Regional Centre for Educalion in Science and Mathematics nghĩa là Trung tảm vùng vể Giảo dục Khoa học vả Toán cảc quốc gia Đông Nam Á (trong khuôn khổ SEÂMEO) Trụ sở SEAMEO RECSAM (từ đây gọi tắt là RECSAM) đặt tại Ja|an Sultan Azlan Shah, 11700 Galugor, Penang, Malaysĩa. Penang là một đảo nằm à Tây Bắc phẩn phía Tây (tức phần bán đảo} cùa Malaysia, có hơn 2 triệu dán, Riêng Georgetovvn City tức phần trung tâm của cả Penang có 300 000 dân (xấp xỉ thành phố Nam Định 300 000 dân trong tỉnh Nam Định 2 000 000 dán}. Đây là trung tâm lân thứ hai của Malaysia. Thành phố có sân bay Quốc tế, cũng trải Ihàm như sân bay Changi ở BÍNH NAM HÀ Singapore và Kuala Lumpur ỏ thủ đô Malaysia. Cảy cầu dài 1 3 km không thẳng mà uốn lượn nổi tiếng thế giới. Đường sá rất hiện đại vả ngay từ bấy giò đã có nhả cao 70 tầng. Thành phố lả nơi cư ngụ của các dần CƯ gọc Malay, Ấn Độ vả ngươi Hoa, đông nhất lá ngươi Hoa. RECSAM nằm trên con đương từ sân bay về trung tâm thành phố, giữa một vùng vốn toàn đổi núi- Tại đó, có hơn một chục tòa nhà xây dựa vào thể đồi cao, thấp, quấn quít lối đi lại rất thuận tiện. Nhìn be ngoài, khu nhà cũng không cỏ vẻ hiện đại. Ở đây bao gổm cả lớp học, ki túc xá, nhà ăn, hộí trường lớn, nhò, thư viện, nhà trẻ, ... chỉ thiếu các sân thể thao. Mỗi năm RECSAM mỏ từ 3 đến 4 khóa đào tạo chính, Mỗi nước trong vùng cử trung bình 3 học viên đến 3 lớp khác nhau theo các chuyên đề cùa tửng năm. Đây vốn là một trung tám được ngưdi Mỹ giúp xây dựng. Lúc đầu ỏ đây còn có các giảo viên phương Tây đến giảng dạy. Nay chủ yếu chỉ gổm các giảo viên tuyển trong vùng. Việt Nam có một giảng viên lã phó tiến sĩ Trần Vui của Đại học Huế đến dạy từ 1996. Kinh phí đảo lạo do các nước thành viên đóng góp. Tuy vậy số tiền đó chưa đủ để duy tri sự lổn tại của trung lãm. Chính vì thế, bèn cạnh các khỏa học chỉnh cho ngưởi của 10 nưởc trong vùng. Irung tâm cỏn mỏ các lớp bổi dưỡng cho giáo viên người Malaysia. Các bang và trưởng gửi giáo viên dén học phải đỏng tiền cho trung tâm. Các phòng học cỏn được huy động hết công suất vở] cảc lỡp ôn thi vào đại học. Ngôn ngữ sử dụng ỏ đảy là Tiếng Anh nhưng cách phát Ém đã bị “Đông Nam Á hóa' r . Philippin dạy bằng Tiếng Anh ngay từ lúc học sinh mới đi học nên các học viên nước này nói Tiếng Anh tốt hơn cả. Nhìểu nước khác dạy từ lớp 3, 4 vã giảng bài bằng Tiếng Anh vởì các mức độ khác nhau từ toàn bộ như Malaysia, đến một phần như Myanma, Thái Lan. Học viên Singapore. Brunây, Malaysia sử dụng Tiếng Anh thành thạo như tiếng mẹ đẻ- Học viên Việt Nam, Lão, Campuchia và Indonesia lúc đó còn gặp khó khăn hơn trong việc sử dụng Tiếng Anh. Các phòng học đểu được trang bị máy điểu hòa nhiệt độ, đèn chiếu. Nhiếu nhà còn có đầu vỉdeo, tivi. máy tính, mảy in, ... Nhìểu đồ dùng dạy học tiện dụng để ngay trong phông. Khi học sử dụng các chương trình của máy tỉnh hay máy tính cầm tay đểu có mảy để chiếu màn hình lên bảng cho học viên theo dõi lừng bước làm cùa giảng vién. Các học viên học rất nghiêm túc, luôn mang theo bút, bút chì, thước, lẩy, ê ke, ... như học sinh tiểu học. Bãi thực hành gáp máy bay giấy thả bay vả tính giở được các học viên nghiêm túc lảm theo cẩn thận. Người Việt Nam khi đến trung tâm này đểu có nhận xét chung rằng điéu kiện học tập ở đây rất tốt. sách VỂ Toán vá các môn Khoa học Tự nhiên nhiều và hầu hết là bằng Tiếng Anh. Tuy vậy, những diều mà Trung tâm đang theo đuổi và dạy cho các học viên cũng không xa lạ và mởi lấm đối vởi chúng ta. Điểu chủ yếu vẫn là ngoại ngữ; Tiếng Anh được dùng như tiếng bản địa à Trung tám nảy. Học viên Việt Nam hồi đó đảnh vật vâỉ Tiếng Anh trẽn láp khi nghe giảng, khi đọc tài liệu, dặc biệt là khi phải viết các tiểu luận vả lự trình bày vấn để. Rút ngắn khoảng cách về trĩnh độ ngoại ngữ chúng la hoàn toàn cố thể trao đổi bình dẳng với bạn bè về mọi vấn đề mà Trung tâm này đang lảm. Cũng đã đến lúc cẩn cỏ những trung tâm như thế cùa ASEAN đặt ở Việt Nam. Điểu ấn tượng nhất dể lại trong tôi lã Phòng Toán (Maths Lab) với rất nhiéu đổ chơi, học cụ, mô hình bảng biểu cho việc học toàn. Điều thú vị là có mô hình bài toàn Tháp Hà Nội. Điểu ấn tượng nửa là Thư viện với rất nhiểu sách từ các nước nói tiếng Anh. TỦ SÁCH TOÁN TUỔI THỔ CÁC DẠNG TOÀN CẠC CẬU ĐÓ OXP TIẾU HỌC số trang: 172, Khổ: 17 K 24 cm. Giá bia: 21 000 dâng. úurtvAI uẦầ/ầ lT4rVi.tM : 279 BAI toAnmInh ho c phăng OLumPit m a-ki HUM*

    m

    SỐ irang: 276: khổ: 17 K 24 cm. Giá bia: 45 000 đổng. Số trang: 133; Kho: 1/ X 24 cm. Giá bia: 21 OOŨ đong. Số trang: 216; Khổ: 17 X 24 cm. Giá bia- 35 000 dâng. ỊpĩDGp ỄƠR5® NTHỨC KĨ NĂNG TOÁN 5 y* ị < 0 — 0 /r r ■ |tr*L íM I IUU kíryive.rfr I sí trang: 216; Khố: 17 K 24 cm. Giã bia; 22 000 dóng. Đóng tặp tạp ctii cả năm 2010 Khổ: 19 X 27 cm Giá bia: 95 000 dâng. SỐ trang: 136. Khổ: 17 X 24 cm. Giá bìa; 23 000 đồng. Đóng lập tạp chi cả năm 2014 Khổ' 19 X 27 cm Giá bìa: 145 000 dổng. TốngĩẠp TOÁN TUỔI THƠ 201 1 (HSMIC) tt > Đóng tặp tạp chỉ cà nãm 2011 Khố- 19 X 27 cm Giá bìa: 104 000 dổng. nhanh nhất ạ? Có phải các bạn ở Hả Nội luôn nhận được sởm hơn các bạn ở tình khác không ạ? Nếu thế thì cơ hội được đăng tên của các bạn ở tỉnh xa bị giảm (So uníair!}. NGUYỄN MINH HIỂN (Võ Cưởng , TP Bắc Ninh, Bắc Ninh) Đáp: Em ra ngay bưu điện To nhất gần nhà mình Đặt cho cả nám học Gửi tặn nhà nhẹ tẽnh Thời gian bào nhận bài Nhìn theo dấu bưu điện Mà dài tận hai tháng vẫn không hể unỉair. Hỏi: Em có được trả lòi cho các câu hỏi của chuyên mục Rubìc hỏi... đáp không ạ? BBN Đáp: Hòi là quyền của em Lại trả lời luôn thể Thế thì anh bắc ghế Ngồi chơi uống nước trà Còn càu hỏi của bạn Gửì vể từ muôn nơi Em làm sao biết nhỉ? Để có cãu trả lờị. Hỏi: Anh Phó ơi! Khi em sáng tác truyện cưới thì cẩn phải sáng lác bao nhiêu truyện ạ? Có quy định số lượng không ạ? NGUYỄN NGỌC LINH (7 AI. THCS Làm Thao, Lâm Thao, Phú Thọ) Đáp: Truyện cười hay truyện không cười Lương không bắt buộc nếu người người khen Đáng lèn tất cả cùng xem Mọi người cười đau bụng là em đạt rối. (6D, THCS Vĩnh Tường, Vỉnh Tưởng, Vĩnh Phúc) ANH PHÒ Bạn hây vào VAibsite: h 1 1 p iỉloỉ m. VD/hỉeu-sacỉt-ODline đề đọc tạp chí Toán Tuồi thơ bán điện từ nhé. CÁC LỚP 6 & 7 Bải 1(155+160}. Cho đa thức f<x> = ax 2 - bx + c với a. b, c là các số nguyên và a khác ữ sao cho 1(9) chia hết cho 5 và f(5) chia hết cho 9. Chứng minh rằng ỉ(104) chia hết cho 45 LẠI QUANG THỌ (Phóng Gião dục và Dào tạo Tam Dương, Vĩnh Phúc) Bải 2(159+160). Cho tam giãc ABC vuông tại A. Kẻ AH vuông góc với BC tạí H. Trên tia đối của tia HA lấy điểm M sao cho AH = 3HM. Trên cạnh AC lẩy điểm N sao cho AC = 3AN. Tính số đo BMN. HUỲNH THANH TÂM (Bưu điện An Nhơn. Bình Định) CÁC LỚP THCS Bải 3(159+160). Tìm các số nguyên X, y thỏa mãn

    +■ >/29x + 3 + 1 = -y/4y 2 + 4y -1 + 2x. BÙI HẢI QUANG (GV. THCS Vãn Lang I, TP. Việt Trí, Phú Thọ) Bải 4(159+160). Tim giá trị nhủ nhất của biểu thức A = x 2 + y 2 + z 2 -{xy + yz + zx), vớì X. y, z là các số thực 3 thỏa mãn X > 3 và xyz = 1 . THẢI NHẬT PHƯỢNG (GV THCS Nguyễn Ván Trỗi. Cam Nghĩa, Cam Ranh , Khánh Hòa) Bải 5(159+160). Xam XÓI bản dồ N sau. Néu tẽn các vùng của N kề vâi vùng
  • W'
  • r 2 ; cí r e - r 8 VŨ KIM THỦY Bàl 6(159+160). Cho tam giác vuóng càn ABC (AB = AC). Đưởng phàn giảc của góc ABC cắt cạnh AC tại E. Gọi bán kinh dưòng tròn nội tiếp tam giác ABC là r. Chứhg minh rằng EC 2r. VŨ ĐÌNH HỎA (GV. trường Đại học sư phạm Hà Nội) SOLUE VIA MAIL COMPETIIIOH QUESTIOHS Translated by Nam Vũ Thành 1(159+160). Given the palynomial ỉịx) - ax 2 - bx I c, where á, b, and c are integers and a is not equal to 0, such that f(9) is divisible by 5 and /(5) is divisible by 9. Prove that /(104) !5 divisible by 45. 2(159+160). Given a right triangle ABC with the right angle al A. Let AH be perpendicular to sc al H. Let M be a point on the opposite ray of the ray HA such that AH = 3 HM. Let N be a point on AC such
  • that AC - 3 AN. Find the measure of ZBMN. 3(159+160). Find all inlegers X and y such LhaL --W29x + 3 +1 s y'4 y 2 5 PHIẾU ĐĂMG Kí THAM Dự CUỘC THI GTQT NÃM HỌC 2015-2016
  • 4ỵ-1 + 2x. 4(159+160). Find the minimum value of the expression where X, y and z are real numbers such A - x 2 + y z + z 2 -(xy + yz + zx) 3 that x> 3 and xyr= 1. 5(159+160). Consider the map N below. List the regions of N that are adjacent to regíon a)r 7
  • u c ) r Ê' 6(159+160). Given a right isosceles triangle ABC (where AB = AC). The angle bisector of the angle ABC intersects AC at E. Let r be the radius oỉ the incircle of the triangle ABC. Prove that EC = 2/. Muc tiêu của chúng tôi là lầm ra ĩihững sần phẩm bảo vệ sức khửo học đưởng và thân thiện với mỏi trường. FsceỊ?WPfc http; iJ#ĩfÀMrWSẫtm MMÍÌI HỔNG HÁ tiNCí NH JVm Pniqrv /Mbg 'V-iW í ^rpVppigr kUiT Trn td CKNH CHI NHÁNH TP.HCỊ Gf mTỒ ŨT*m MSt Hoa- m. «. ctỉíi h f£hnHiín GÕ sỉ II (ỉa Hể Sii li lỡxi sãi HN HMb 0 "|.H ĩrj Hti :«l: DJ 3â5S.íÌậr ■ FJ1. Ẻ4 3ĐU4S57 ũiụii In* W.Ĩ75B 2151 Ernair. HFạp!i)n Li lu 1 n FAh . la Sĩ -5 B- ăt » nvgo&na: rói vcbiwírj CS 9 I «1 CHI nhanh đa riAhllì z3»ít ủ™ạ Yli Thf
  • lũ Ặn. Q ãm LỆ. TP Rỉ uĩny EIẼĨ1 IPŨUI U5Ĩ!J*4HME fs: lữíiịấỊmt Biển xanh màu Lhỡi gian triệu năm sóng vò. Trời xanh mâu thanh thícn hỏa quyện một gam màu tử xiía. Xa xa lã những dãy núi xanh cây lá vởi một màu xanh của cong viên bãi biển. Giữa bửc Iranh mâu xanh ấy là vầng tròn đỏ của mặt trời trẻ trung lúc bĩnh minh. Bạn hãy tả vẻ dẹp của bức tranh biển nãy. Tỏa soạn chở hài viết lối của bạn để dâng trong sổ tháng 7+8.2016. MORIS VŨ Ảnh; VKT CÁC HỌC SINH ĐƯỢC KHEN TRONG cuộc THI GIẢI TOÁN DÀNH CHO NỮ SINH í 1 .. , Từ trái sang pnải: Chu Thị Thanh. Lưu Thi Phưung, Phạm Thị Thùy r H| Hl nnyr 1^4 B Cõng ty CF VPP Hồng Hà là nhà tài trợ cho 2 HONG HÀ cuôc thi: và iiNCEira ' u v t LLU

    ĩrttyrn tỉriủiịi - ỉịrVỈ*f*j ĩiài Giấy phép xuất bản: số 31/GP-BVHTT, cấp ngày 23/1/2003 cùa Bộ Vãn hóa và Thông tin Mã số: SBTT159M16, In tại: Công ty cổ phần in Công Đoàn Việt Nam. 167 Tây Sơn, Đống Đa. Hả Nội, In xong vả nộp lưu chiểu tháng 05 năm 2016. NHÀ XUẤT BẢN GIÁO DỤC VIỆT NAM - BỘ GIÁO DỤC VÀ DÀO TẠO Giá: 20000đ Cạn đọc yêu quý, Cả vùng Đồng Nam Á này chỉ có các tỉnh phía Bác Việt Nam mình cở iiượe mùa Thu. Mùa Thu trùi trong xanh như pha lẽ. Nắng vàng sắc mặt ong. Giỏ gựn sóng lăn Lăn hồ Gươm, lay khẽ lá cây lộc vừng vcn hồ huyền Ihoại. Dần xa cái nóng cháy mồ hối, hè đã qua rỗi, Chưa Ihấy cãi rét co ro, đông côn chưa LỜI. Hả Nội dang trong nhưng ngày dẹp nhất trong năm của mùa Thu thứ 1007 kố từ ngày mảnh dất giữa hỗ Tây và hồ Gươm trở thành dát Kinh kì, Đãy cũng là lúc tiếng trống trường rộn lẽn bắl đảu một nám học mới. Mũa thu cũng lã mũa sinh nhật của Toàn Tuổi thơ và dây lã múa thu thư 17 Lạp chi đèn vởi học đường. Mối nãm học mời, Tạp chi dcu muôn cõ nhửng bươc tiến mời, đổi thay mới dể làm bạn dồng hành tối nhất vớ ỉ cát: bạn trẻ yẽu toán. Tờ bảo bạn dang cầm trên tay là so 161+162 trong chuỗi gản 200 tờ báo của 16 năm qua, Tờ báo nảy bầl dầu một thời kì mới Toán Tuổi thơ song hãnh cùng các nhã trương lửng năm học. Tơ bão nãy lã sổ" 1 /9 của nãm học nãy. Sô tiếp theo sò là 2/9, Sđ cuối cũng của năm học ra dâu tháng năm sè là 9/9, Trong 9 số dó có 3 sõi dặc biệt: 1/9. 3/9 và 6/9 là các so gộp vđi dung lượng gấp dõi ra vào các dịp đầu nani học mứl, dỊp 20.11 và dịp Tết. Nội dung các số gộp đang kết quả cuộc thi CLB Loãn quốc, các hoạt dộng chuyên mòn dịp 20.11, các đề thi chuấn bị cho cuộc Lhị cấp tỉnh vã loàn quốc cũng các bãi bịch sử toàn, toán vuí nhàm dẫn dát độc giả hiểu hơn vê vân hỏa toán học. Các sỏ' gộp sẽ tăng số dẻ trong chuyên tnụe Đẻ ra kĩ này, vốn lả chuyên mục cốt lõi vả truyền thống của tạp chí. Như vậy học ki 1 bạn có 4 số lừ 1/9 đến 4/9 trong đõ cò 1/9 và 3/9 lã 2 số gộp. Kĩ 2 bạn có 5 sỏ’ từ 5/9 đến 9/9 trong dỗ 6/9 lã sô' gộp. Dây là thay đổi từ chinh dõi hỏi của độc giả, phù hợp vơi sự trơ ỉạỉ của mùa tựu trường và kì hè truyền thống. Đọc xong sổ 9/9 bạn sẽ nghỉ hè thực sự với các hoạt động thể thao, ầm nhạc, bơi, cắm trại, dã ngoại và háo hức đón chờ năm học mới vảo đầu tháng 9. Năm học mới đến. số 1/9 lại đến củng bạn. Các chuyên mục và cách tổ chức bài vở cũng diêu chỉnh chút Í1. cho phù hợp VỚI oách phát hành mởỉ vì lợi ích độc giả. Chí riêng de ra kì này số 9/9 bạn phải chờ đến số 1/9 năm học sau đọc ldi glái vả tìm tên mình. Bạn nhớ nhé. Tữ nay ngoài con sổ như 191 trong chuỗi đãnh số Lừ năm 2009. còn Lhêm số như 2/9 bên cạnh Lheo thứ Lự Lrong năm học. Bạn vãn cỏ Lhể đặL bão Lheo Lưng quỹ hoặc cả nãm dương lịch. Ví dụ quý rv này là các số 2/9, 3/9 và 4/9 còn cả nấm 2017 là các số lừ 5/9 dến 9/9 của năm học náy và các số 1/9 đếu 4/9 của nãrri học sau. Bạn dã yêu quỷ Loản. dã yêu quỹ Toãn Tuổi thơ. Bạn hãy Liếp lục cùng chúng lòi xây lâu dài Toán học, ươm vườn CLÍ3 Toán cho tương lai. Toán Tuổi thơ sẽ không phụ cõng mong chơ của các bạn. Chúc nâm học mơi thảnh công. TBT TẠP CHÍ TOÁN Tllổl THỐ TRONG SỐ NÀY Toán quanh ta Định lí Pytago Moris Vũ Nhìn ra thê' giỏi Lài giải để thi chọn đội t uyển dự thi Qlympic Toán Quốc tẻ của Hóng Kông nằm 2010 (Vòng 1 ) (Tiếp theo kì trước) Mai Vủ Bạn muốn du học Phỏng vấn (Tiếp theo kì trước) Vũ Kìm Thư Lịch sử Toán học Ai là ngưõi chứng minh hình học đẩu tiên? Lé Quốc Hán Compa vui tính Sổ diểm được tồ màu? Nguyễn Đúc Tấn Phá án cùng thám tử Sêlôccôc Chiếc nhẫn trang túi Nguyễn Thị Lan Đến với tiếng Hán Bài 68. Hà Nội có rất nhiều viện bảo tàng Nguyễn Vũ Loan Học Vật lĩ bằng tiêng Anh £29 Unit 20. Gas law and parlicles of malter theory section Binh Nam Hà Bạn dọc phát hiện Suy nghĩ để mỏ rộng mỗi bài toán Phan Duy Nghĩa Dành cho các nhà toán học nho Định lí stevvart và ứng dụng Thái Nhật Phượng Tr 58 Học ra sao? Giải toán thế nào? Quan hệ giữa hai bất đảng thứcAM-GM và Nesbit Ngõ Vãn Thải t)ế thi các nước AMC 2015 Senior Division (Tiếp theo kì trước) Đỗ Trung Hiệu Thách dấu! Thách dã'u dây! Trận đấu thứ một trăm ba mươi tám Bùi Hái Quang Gíờ ra choi Vui cười Nguyễn Thị Diệu Nga Đo trí thông minh Hình nào không thích hdp? Nguyễn Đửc Tấn Vào thăm vưởn Anh ở chữ Cnemical elements Nguyễn Ngọc Sơn Trường Olyinpic Nam Hà, Hà Nam nghĩa lá gì? Vũ Kim Thủy Tr 62 2 Phóng sự ánh ThS. Vũ Kim Thủy, Tổng biên tập. Trưởng ban ồng Trương Quang Luyến, Chủ tịch HĐQT. Tổng tổ chức Cáu iạc bộ TTT toàn quốc 2016 Giám ớổc Công ty cổ phần VPP Hổng Hà Trao giải cho các học sinh đoạt giải cuộc thi Giải ỉoán qua thư theo năm học 2015 - 2016 Đại diện công ty cổ phần Vãn phòng phẩm Hiệu trưởng trường TH Ngỗ! Sao Hà Nội Hổng Hà nhận quà lưu niệm của Ban tổ chức nhận quà lưu niệm của Ban tổ chức Ban tổ chức trao cờ cho các đoàn tham dự SSE(3âHIấS0WẲHỉ!Ì? Các vị đại biểu và Trưởng đoàn, Lanh đội, Giám khảo, Giám thị Các thl sinh háo hức và hồi hập chờ đến phần thi cá nhân Các thỉ sinh đang làm bài thi cá nhân Cắc đội trưởng bốc thãm xem đội mình mang tên nhà toán học nào Chờ vé đi tiếp thành phô' khác Cà đội cùng trao đổi để tìm kếỉ quả của bài toán Một trong hai đội vê đích đâu tiên Trao giải phần thì Tiếp sức toán và Du lịch toán học Trao giải cho các đội đoạt giải Vàng phần thi Tiếp sức toán và Du lịch toán học

    BÁI PHdĩ bé cùn TỒDG BIÊn Tập T8I IỄ HNRI nsc cuoc THI CLB TTT Tròn QUỐC £OIG N gày 25.10.2000 tà tạp chí Tũản Tuổi thơ đầu tiên ra mắt độc giả. Cuộc thi cho học sinh tiểu học hưởng ứng phong trầo đọc Toán Tuổi thơ đã dược khởi nguổn từ Mê Linh. Vĩnh Phúc (nay là huyện thuộc ngoại thảnh Hà Nội) nãm 2001. Ngày 9.9.2001, ba trưàng tiểu học Lưu Quý An. Phúc Yên A, Tiền Chảu A đẫ tham gia giải các bài toán vui. Dịp kì niệm 1 năm ra mẩt, ngày 25.10.2001 , Toán Tuổi thơ tổ chức mọt cuộc thi toán tại cung thiếu nhi Hà Nội, chúng tôi đã td chức Hội thi trung thu Toán Tuổi thơ với nhiều học sinh tham gia giải các bài toán vui. Khi đó Toán Tuổi thơ còn là nột Phụ trương cùa Toán học vá tuổi trẻ. Ngày 30.1.2002 tức là cách dảy gần 15 năm, đơn vị tạp chí Toán Tuổi thơ chính thức được thảnh lập. Olympic Toán Tuổi thơ là một sự kiện giảo dục được học sinh cả nước mong đợi vào mỗi dịp tháng 6 hảng nãm. Sau 10 kì tổ chức, Glympic Toán Tuổi thơ đă thu hút hàng nghìn bạn học sinh ở cấp Tiểu học và THCS đến từ 54 tình, thành trên cả nưóc tham dự, tạo hiệu ứng xã hội lích cực và được truyền thống đánh giá cao. Olympic Toán Tuổi tho đã truyền cảm hửng cho các thầy cỏ và các em học sinh, tạo nên một phong trào say mê học Toán, thi đua giải Toán từ cấp trường íởi quận, huyện, tỉnh, thành. 10 nãm. Olympic ghi nhận sự trưởng thành của nhiều thế hệ lài năng Toán học mà đại diện là em Nguyễn Thế Hoàn - HCV Olympic Toán Tuổi thd - HCV Olympic Toán Quốc tế năm 2014, 2015; Em Vũ Xuân Trung - HCV Olympic Toán Tuổi thơ - HCV OlympicToãn Quốc tế năm 2015 và là một trong 6 thí sinh Việt Nam tham dự Olympic Toán Quốc tế nàm 2016. Chúng tôi tự hào vì đâ tạo ra một sán chơi trí tuệ, lành mạnh bổ ích cho học sìrh, giáo viên và các nhà quản lí giáo dục. Tiếp nối thành cõng của Olymplc Toán Tuổi thơ, để phù hợp hơn với chương trình đoí mởi toàn diện ngành giáo dục, tạo cơ hội cho học sinh Việt Nam tiếo cận gần hơn với các kì thi Toán Quốc tế, :ừ năm 2016. Tạp chí Toán Tuổi thơ đổi mởi Olympìc Toán Tuổi thơ theo mô hình phù hợp hơn là Câu lạc bộ Toán Tuổi thơ. Tạp chí tổ chức sinh hoạt liên tình các Câu lạc bộ Toán Tuổi thơ cho Tiểu học vả Trung học Cơ sở thông qua các bài toán, cãu đố toán, IQ bằng Tiếng Anh. Câu lạc bộ TTT có những đ ểm mỗi so với Olympic Toán Tuổi thơ như sau: • Điểm mới thứ nhất của kì thi là các bạn nhỏ đọc đé trực tiếp bằng Tiếng Anh. Đáy là cuộc thi sẽ phát động phong trào học toán và học Tiếng Anh. • Điểm mới thứ hai là Vòng thi Du lịch Toán học. Các bạn nhỏ cần lưu ý nếu như Vòng thi Tiếp sức toán dù đúng dù sai bạn vẫn đến đ JỢc bài thứ 6 thì ở Du lịch Toán học cả đội phải chung sức giải dúng từng bài mới dến dược cả 6 thành phô. Cảc giám khảo nhớ kí vào tờ trả lời của các đội mang đến nếu kết quả đúng, coi đó là tấm vé để các bạn du lịch tới thành phố tiếp theo. Vòng thì thứ nhất được cộng điểm thời gian cho 3 đội nhanh nhất còn Vòng Du lịch Toán học sè dừng lại khi 2 đội đi được cả 6 thành phố hoặc hết giò quy định. Không được Ưnh cộng điểm thời gian ở võng thi nãy. Ngáy 19.1.2016 Tạp chí đã tổ chức thảnh cõng cuộc thi CLB Toán Tuổi thơ liên tỉnh gồm các tỉnh, thành tnam gia: Hà Nội, Nam Định, Hưng Yên, Quảng Ninh. Sơn La, Thái Bình, Vĩnh Phúc. Đây lả sự hội Lụ đáng yêu đầu tiên cho một đề mó với đẩy đủ các gương mật từ thảnh thị dến nông thôn, đổng bầng, trung du và miền núi. Rả't vuì mửng khi có tinh Sơn La tổ chức cho học sinh thi Câu lạc bộ theo cách thức như Câu lạc bộ Toán Tuổi thơ toàn quốc cho TP. Sơn La. Càu lạc bộ Toán Tuổi thơ toàn quốc năm 2016 được tổ chừc tạì Hà Nội, dành cho khối lớp 5 (Tiểu học) và lớp 8 (THC5). Các CLB THCS sẽ tham dự: Thi cá nhân, Tiếp sức Toán và Du L|ch toán học. Riêng cấc em bậc Tiểu học chỉ tham dự vòng thi cá nhản. Tâ't cả cảc đề thi toán đều bằng tiếng Anh. Chương trình diễn ra trong 3 ngày, từ 06/06/2016 đen 08/06/2016. LỄ Khai mạc và Bế mạc trao thưởng dược lổ chức tại Hộì trường Hoa Sen 1, khách sạn Kim Liên. Trường thi đặt lại trường Ngói Sao Hà Nội. Cóng ly cổ phần Vàn phòng phẩm Hổng Hà là nhà tàì trợ chính. Năm ray có 286 học sinh của 29 đoàn chia thảnh 23 đội Tiểu học và 20 đội THCS từ 24 tỉnh, thành trên cả nước tham dự. Đề thi lần đẩu tiên 5LÌ aụng ngôn ngữ Tiếng Anh thu nút mạnh mẽ cảc thành phố lớn và các tĩnh có phong trào mạnh như Hà Nội, TP. Hổ Chí Minh, Hải Phòng, Nam Định, vinh Phúc... nhưtig cũng không làm khó các tình mìển núi như Sơn La, Yên Bái, Lạng Sơn... hay các tình Tây Nguyên như Dẳk Lắk, Tây Nam Bộ như Kiên Giang, Long An... Tất nhiên không Ihể thiếu những ’.ỉnh có truyền thống hiếu họG nhưĩhải Bình, Thanh Hóa, Hà Tĩnh... Chúng ta chào mừng Quảng Ngãi, Hưng Yèn lần dầu dến với cuộc thi Toán Tuổi thơ toàn quốc. Qua cuộc thì này, các bạn được kiểm tra đánh giá:
  • Năng lực tư duy
  • Năng lực toán học
  • Vốn từ vựng Tiếng Anh -Tạo động lực, tạo dà cho bưỡc phát triển sau này. Bên cạnh các CLB, lần đầu tiên những bạn học sinh lớp 5. được nêu tên trẽn chuyên mục Thi giàỉ toán qua thư năm học 2015 - 2016, được mòi tham dự phần thi cá nhân cùng vâi cảc thi sinh dẩn từ các CLB xuất sắc nhất các tính thảnh. Hi vọng cùng với việc được gặp gỡ, học nỏi qua các kì thi mối, các thầy cô, bạn bè đến từ 24 tỉnh, thành cả nước, được khẳng định mình, các bạn nhỏ từ miển Nam ra cũng được tham quan thù đó Hà Nội, các thành phố Hải Phòng, Nam Định vá nhiêu danh lam thắng cảnh nổi tiếng khác. Các bạn từ miền núi phía Bắc về có dịp được đến các bâi biển phía Bắc như Bãi Cháy Đồ Sơn, Thịnh Long... Cuộc thi chắc chắn sê là kì niệm đẹp với tất câ mọi ngưài dặc biệt là các em học sinh. Đó chính là niềm vui, là thành còng chung của tất cả chúng ta. Thay mặt Ban lổ chức tói xin tuyên bố Khai mạc cuộc thi Câu lạc bộ Toán Tuổi thơ toàn quốc 2016. Chúc sức khỏe tẩt cả mọi ngươi. Xìn cảm ơn đã chủ ý lắng nghe. TOÍin CỈMH cuộc THI cfiu LỤC Bộ TORIi TUỔI THU ĨOÀn QUỐC 2016 (Tường thuật) C hiền 3.6.2016 ban tổ chúta sự kiện của tạp chi đã có mặt tại trường Ngôi Sao Hà Nội, nai diễn ra cuộc thi câu lạc bộ Toán Tuổi thơ toàn quốc 2016. Một lần nữa các cõng việc để phục vụ cuộc thi được rà soát lại. Sáng 6.6 Ban tổ chức họp vơi các đoán đến tơ 24 tình, thành tạì ưưãng Ngõì Sao Hà Nội. Các đoàn nhận quà tặng của công ty Cổ phần VPP Hổng Hà, Công ty cổ phần thiết bị Giảo dục Dả Nắng. Trưa Ể. 6. 201 6, lảnh đạo trưởng Ngôi Sao Hà Nội đã chiêu đãi các trưởng doán, lãnh dội vả cảc tình nguyện viên. Sau bữa trưa, CÉC đoản nhanh chóng di chuyển về Hội trưàng Hoa Sen 1, khách sạn Kim Liên để dự l_ễ khai mạc vảo lúc 14 h. Dự khai mạc có Óng Nguyễn Đức Hỡu, Phó Vụ trưởng Vụ Giảo dục Tiểu học, T$. Tạ Ngọc Tri, chuyên viên Vụ Giáo dục Tiểu học; TS. Nguyễn Thành Anh, Phó Tổng biên :ập NXBGD Vệt Nam, ThS. Vũ Kim Thủy, Tổng biên [ập Tạp chí Toán Tuổi tnơ. Trưởng han tổ chức Cuộc thì; Õng Trương Quang Luyến. Chủ tịch Hội dồng Quản tộ, Tổng giám dốc Công ty cổ phần VPP Hống Hà, cãc lãnh dạo Sở Giáo dục của một sổ tỉnh, thành, các đơn vị thành viên của NXB Giảo dục Việt Nam... phóng viển bảo Giảo dục và Thởi đại, báo Hi Nội mới, VTV2, báo Còng Lý,... Cuộc thi vinh dự dược Thủ tưởng chính phú Nguyễn Xuân Phúc gửi lẵng hoa chúc mửhg. Sau phần vãn nghệ, Tổng biên tập Tạp chí có bài phát biểu khai mạc. ỗng Trương Quang Luyến đại diện nhà tài trợ chinh lẻn phãt biểu. Tổng biên lập thay mặt ban tổ chức tặng quà lưu niệm cho nhả tài trỢ, lãnh dạo trường Ngói Sao Hà Nội và 2 dội Quảng Ngãi, Hưng Yẽn lẩn đấu tham dự cuộc thl toàn quốc do Tạp chí tổ chOc.Tlêp theo là phần trao giải cuộc thi Thi giải toán qua (hưỉhoo nám học 2015 - 2016. Ấh LíỢrg nhất lá màn diễu hành của các dội trẽn sân khấu, 286 gương mặt xuất sẳc của 43 đội đến từ 24 tình, thành trén cà nước, trải dãi từ Nam ra Bắc: Kién Giang, Tiến Giang, Thành pnõ’ Hồ Chí Vin, Long An, Phú Yên, Đẩk Lắk, Quảng Ngãi. Hà Tin. Nghệ An, Thanh Hóa, Nam Định, Ninh Bình. Thái Bình, Hải Phóng, Hung Yên, Hòa Bình, Hà Nội, Vĩnh Phúc, Bắc Giang, Phú Thọ, Sơn La, Yén Bải, Lạng Sdn. Lào Cai. Sáng hòm sau, tuy giở khai mạc cuộc thi lá 7hl5 nhưng cỏ rất nhiều đoàn đến từ rất sớm. 12 phòng thi chLẩn bị cho vòng thi Cá nhân và phông bại, bàn ghê' ngoài sân trưòng phục vụ cho 2 vòng thi Tiếp sức toán vả Du lịch toán học òã dược chuẩn bị chu dáo. Các bảng biểu đều được in 4 màu. Các thầy cô trưởng đoàn, lãnh dội tận tình dẫn các em trong dội của mình đến các phòng thi. Cùng lúc dó trong phòng hội đồng, ban tể chức có cuộc họp vởi các cản bộ coi thi. Không khí trước giờ thí rất háo hức. Kểt thúc vòrg thi cã nhản, Ban tổ chức và cán bộ coi thi, các em học sinh di chuyển ra ngoài sân trướng õể tiếp tục vòng thi Tiếp sức toán. Các tình nguyện viên và các thầy cô của trường Ngôi Sao Hà Nộì khẩn trương kè bản ghế theo sơ đó cuộc Ihỉ. Khi có kết quả của phần thi Tiếp sức toản, ban tổ chức chọn ra 8 đội có điểm cao nhất tiếp tục thi vòng thi Du lịch toán học: Vĩnh Phúc, Nam Định, Bắc Giang, Hải Phòng, TP. Hó Chí Minh 5, Đắk Lák, TP. Ho Chí Minh A, Thái Binh. Để phục vụ cho phần thi náy, tại sán trường kê 8 khu vực cho 8 đội thi và 6 bàn giám khảo tượng trưng cno 6 thành phô. cả 2 phần thi diễn ra rất sôi noi và hào hứng tạo nên sự thủ vị cho cuộc thi. Hai đội phất cở VỂ trước trong vòng Ihi Du lịch toán học là Vĩnh Phúc và Nam ĐỊnh. Chiêu ngày 7.6.2016 các doán cùng càc bạn tinh nguyện vỉèn của đoàn rĩính tham quan các điểm du lịch. Sảng 8.8.2016, HỘI trường Hoa Sen 1 khảcn sạn Kim Lién đón chảo mọi người về dự Lề bế mạc cuộc thi cáu lạc bộ Toán Tu6i Ihơ toàn quốc. Đến dự có GS. TS Vũ Vãn Hùng, Tổng Giám đốc NXB Giáo dục Việt Nam; ông Trương Quang Luyến, Chủ tịch Hội đồng Quản trị, Tong Giám đốc Cõng ty cổ phẩn VPP Hồng Hả, một số Phó Giám dốc sa Giảo dục - Đào tạo các tỉnh cũng đến dự' các phóng viên báo đài VTV2, bâo Hà Nội mới, ... Trưởng ban tố chức ThS. Vũ Kim Thủy phát biểu tổng kết cuộc thỉ. Mặc dù đây là nãm đầu tiên thi Toán bằng Tiếng Anh nhung có 1 bạn cấp Tiểj học đạt điểm tối đa 1ŨQ đ ểm, 2 bạn THCS đạt 95 điểm. Phấn thi Du lịch toán học các đội đều làm được it nhất 5 trong tổng số 6 bải th . Có 24 Huy chương Vàng, 48 Huy chương Bạc, 72 Huy chương Đổng. 64 giải Triển Vọng dược trao cho các em thi sinh. 3 Cup Vàng, 3 Cup Bạc và 2 Cup Đông trao cho 6 đội xuất sắc nhất thí Tiếp sức toán và Du lịch toán học. Nàm nay cò thèm nhiều giải khác: giải thí sinh giải đúng liên tục nhiều càu rhấL, giải thí sinh đạt điểm cao nhầt. Hội trưởng như vỡ òa khi tên các em được xướng lên. Các thầy cô giáo cũng tự hào, vui sưãng cùng các em học sinh thân yéu của mình. Cuộc thi Câu lạc 00 Toán Tuổi Lhơ khép lạ vởi nhiều cảm XÚC khó tả đan xen lẫn nhau. Tuy cuộc thi chỉ diễn ra trorg 3 ngày nhưng những cảm xúc, nhữhg kỉ niệm má nó để lại trong mồi thí sinh, mồi thầy cô giáo sẽ còn đọng lại. PV THƯ CẢM ƠN Câu lạc bộ Toa." Tuối thơ toàn quốc 2016 đã diễn ra trong 3 ngày từ 06/6/2016 đến 08/6/2016 tại Hà Nội, vói 29 đoàn gốm 23 đội Tiểu học và 20 đội THCS đến từ 24 tỉnh, thành từ Lào Cai. Lạng Sdn đến Kiên Giang, ủy viên Bộ Chính Trị. Thủ tướng Chính phủ Nguyền Xuân Phúc dành sự quan tàm và gửi lẵng hoa chúc mưng cuộc thi. Đến dụ Câu lạc bộ Toán Tuổi thơ toàn quốc 2016 có Lãnh đạo Vụ Giáo dục Tiểu học, Bộ Giáo cục và Đào tạo, õại diện Ban Dãn vận, Thành ủy Hà Nội, nhiễu Lãnh dạo sở Giáo dục -Đào tạo các tỉnh, thành vả nhiểu -Đại biểu auan tâm tới Cuộc thi. Trước ngày Khai mạc, NGUT, Ngô Trển Ái đã gửi thư động viên, khích lệ cuộc thi. GS. TS. Vũ Vàn Hùng, Tổng Giảm õôc Nhà xuêt bản Gáo dục Việt Nam dã tới dự Le bế mạc tổng kết và trao ihướhg cho các CLB đẹt giải. Nhà tài trợ chinh lã Cõng ty cổ phần Văn pnòhg phẩm Hóng Hà đã đắng hành cùng cáu lạc bộ Toán Tuổi thơ toàn quốc. Các đơn vị đã tàì trợ và giúp dữ cho cuộc thi: Nhà xuấ: bản Giáo óục Việt Nam; trưởng Ngôi Sao Hà Nội; các bạn tình nguyện viên; Xỉ nghiệp Bản đố 1 - Bộ Quốc phòng; Cõng ty cổ phần Đầu tư -Phát triển Giáo dục Đà Nắng; Công ty cổ phần Bản đổ và Tranh : ảnh Giảo dục; Sputnik Education: Tạp chí Toán học và Tuổi trẻ; Trường Tiểu học Ban Mai, Trường tiểu học Dàn lập Đoàn Thị Điểm, ... Đã có nhiều đài, báo đến dự. ghi hìrh, đưa tin về ; sự kiện này; Phòng Xã hội ■ VTV1 ; Phòng Khoa học Xã hội - VTV2, Đài Truyền hình Việt Nam; VŨV; Báo Hà Nội Mói; Bảo Giáo dục và Thòi đại; Báo Công lý; ... Các Sở Giáo dục - Đào tạo, các Phòng Giáo dục
  • Đào tạo ớ các dịa phương đã tạo diều kiện tốt cho các ém học sinh tham gia. Các thầy cô giảo, các bậc phụ huynh dã miệt tình, quan tân và : theo cùng cảc em trong nhữhg ngáy diễn ra Câu lạc bộ Toán Tuổi thơ toàn quổc 2016. Tạp chí Toán Tuổi Lhd chân thành cảm ơh các tổ chức, các cá nhán đã đỏng góp vão thảnh còng của Cáu lạc bộ Toàn Tuổi Ihơ toàn quốc. toan tuổi THƠ . BÙI PHÁT biểu cùn ràiGBiÊĩiĩỊỊp TẠI Lễ BÍ IM cuôc THI CIB ĨTT ĨOHTI cuốc 2016 Hõm nay chúng la lại tể tựu lại đày trong buổi Le bê rrạc sự kiện Cảu lạc bộ Toàn Tuổi thơ toàn quốc lấn đầu tiên. Thủ đó Hà Nội chứng kiến 286 học sinh, 286 gương mặt yêu toán thay mạt 29 đoàn gồm 23 đội tiểu học, 20 đội THCS từ 24 tình, thành phô' từ Lào Cai, Lạng Sơn đến Kiên Giang. Lẩn đầu tiẻn nhiều bạn biết Hồ Gươm. Hồ Tây, Văn Miếu - Quốc Tử Giám, và cũng lẩn đầu tiên nhiều bạn được tham gìa thi toàn quốc, thi toán bằng Tiếng Anh. Thật thú vị khi các bạn THCS được giải toán trong Vòng thỉ tiếp sức cẩn chiến thuật, chiến krợc và vào Vòng thi du lịch cần đoàn kết. nhanh, tỉnh tảo và thống nhất cao mơi đạt kết quả. Theo đánh giá của Ban tổ chức, đề thi vừa sức, phổ điểm trải từ 5 điểm đến 95 điểm ở THCS. Rất vui là điểm Tiểu học có điểm 100/100 dù đề được đảnh giả là khóng dễ. Phần thì cả nhán cho thấy các địa phương Hà Nội, Nam Định, Vĩnh Phúc, Thái Bình, LÉO Cai, thành phố Hổ Chí Minh, Thanh Hóa, Lạng sơn, Phú Yên có thành tích đổng đểu và khá cao. Phần thi tiếp sức và du lịch chữ 'hãy sự phối hợp khi làm bài tập thể của các đội Hà Nội chưa tối ưu như các đội được vào vòng 8 dệì. Phẩn thi du lịch diên ra hào hứng đầy mâi lạ. Chúng ta đâ có một kì thi thành công và 24 tỉnh thành có mặt ngày hôm ray là sự góp mặt của các sáng lập viên Câu lạc bộ Toán Tuổi thơ loàn quốc. Con số đó nói lên kết quả của công tác chãm lo đến học hành, đến ươm trồng nhữtig tài năng toán học tương lai. Có được kết quả đó, chủng La cảm ơn sự quan lảm của Bộ Giáo dục và Đào tạo, Vụ Tiểu học, lẳnh đạo Nhà xuất bản Giảo dục Việt Nam, cõng ty cổ phần Vãn phòng phẩm Hồng Hà, nhà tài trợ chính cho mọi cuộc thi của Tạp chí Tũár Tuổi thơ. trường Ngôi Sao Hà Nội, trường Ban Mai và phóng viêr VTV, các cơ quan báo chỉ và truyền thông, các Sở Giáo dục, Phòng Giảo dục, các thầy cò vá các bậc phụ huynh cùng 33 tình nguyện viên. Tất cả đă chung tay làm nên cuộc thí Câu lạc bộ Toán Tuổi thơ toàn quốc có ý nghĩa, trồ thành cuộc ‘hi uy tín. cảm ơn Thủ tương Chính phù Nguyến Xuân Phúc đâ gửi tặng lắng hoa chúc mừng, nhiều đơn vị đả tál trợ cho cuộc thi. Sự thành công của cuộc thi là kết quả của công tác chuẩn bị hết sức nghiêm túc, tỉ mi, là sự tham gia nhiệt tinh, chám lo chu đáo cho các em học sinh của Irưởng doàn, lãnh đạo. Câu lạc bộ Toán Tuổi thơ toàn quốc 2016 được diễn ra vói mong muốn tạo ra một hoạt dộng trí tuệ, vởi phương châm: học mà chơi, chơi mả học, là cấu nối kết nối các em, thầy cô đam mè toán học trẽn cả nước có cơ hộì được giao lưu học hỏi và cọ xát để từ đó tự đánh glá được kiên thức. Vì vậy hi vọng sau kẽt quả cuộc thì rày nhũng đội đoạt giải cao tiếp tục phát huy khả nâng toán học của mình, những em giải chưa cao cũng lấy dấu mốc đó để phấn đấu hơn nữa. Sau đây, sẽ là giây phút thầy cô, các bậc phụ huynh và các em học sình chờ đợi và hổi hộp nhất. Tôi xin nhương lời cho MC. Chúc sức khỏe mọi người VÉ xin cảm ơn đâ chú ý lắng nghe. 12 cuốc THI CÂU LAC Bố TOÁN TUỒI THƠ TOÀN QUỐC 2016 CHILDREN'S FUN MATHS JOURNAL NATIONAL COMPETITION 2016 ĐỀ THI CÁ NHĂN THCS SECONDARY SCHOOL INDIVIDUAL PAPER Thời gian: 30 phút (Durătion: 30 minutes) Problem 1. The íractions in the number pattern above are equally spaced. Find the miss ng ữactions X and y. Problem 2. Three frìends try tũ gưess the number of swee!s n a closed botỉltì. Each of Ihe three ca ve a guess oỉ 93, 137, and 164. Hovvever none of them guessed t correctly. One triends guess is o" by 1 7 sweets, one is ofĩ by 22 sweets and one is off by 44 sweets. How niany sweets are there in the bottle? Problem 3. A man hi 35 4 sons; each is 4 years alder thar :he yũLnger brolher after him, and the eldest brDther's age is 5 ỈÌTIBS Ihe youngest brothers age, Fínd the age ũf the eldest brDtherand the youngest brother. Problem 4. Each diagram in the sequence below consists of a number of dots. Diagranì 4 Diagran 1 1 DĨBgrarĩì 2 Diagratĩ j 3 Write down the nember of Ihe diagram that has 66 dcis. Problem s. Given a trapezium ABCD (where AB II Cũ), ZABC - 90°, BD 1 AD, BC - 5 om, and BD= 13 cm. Fird the area of the trapezium ABCD. The diagratìĩs are nat draự/n to scale Problem ỗ. Find the maximurn value ũf the express on p = 2x x 2 +l' Problem 7. Sữlve ĩhe equation |x - 1| +- |x - 2| + |x + 3| + ... ■+■ |x T 201 6 1 - 2017x. Problem 8. A right pentagonal prism has its siớes painlẼd with red, yellow and green painỉ such that each vel'tex does not have any :wo sides ct the SE me colnr. Pind the number of sides vvhich are painted red. Problcm 9. Does there exist a prÌTie numoer which car oe VDTỈtten as a 4 + 64 where a is an integer? Problem 10. Art applìance shop nrders 24 boxes of 5fnall electric fans (where all boxes contah Ihe sanie number ũf small fans) and 25 boxes ữF bigge r electric lans {vvhere all bũxes also conlaih Ihe sa me number of big fans). The totál number of fans ordered is 120D. How many fans are Ihere in each 0 OX that contains small fans? Problem 11. Given a quadhlateral ABCD having AC peroendicular to BD. AC = 6 cm, and Sũ = B cm. Lel M and N be the midpoinls dF AD and sc, respectively. Find the lenglh of MN. A Problem 12. Let ASC be a triangle having zB = 2ZC, AB = 60 cm ard AC = 60 cm. Find the length of the Intemal angle bìsec.or BD. c Problem 13. Compare A - 2 X 3 54 and B = 6 X 5 32 . Problem 14. Evaluate the follỡwir>g 201 6 2 - 4034 - 201 7 2 . Problem 15. A convex polygon has 14 dìagonals. Find Ihe sum of the measures of its niernal angles. Problem 16. (VVritten paper/T ự luận) Given a triangle ABC having 0 - 2ZC. Let AD 1 BC [De BC), and E be the midpoint of BC. Prove that AB = 2 DE. A The diagrams are not drawn ta scale cuốc THỈ CÂU LAC BÔ TOÁN Tltểl THƠ TOÀN QUỐC 2016 CHILDREN’5 FUN MATHS JOURNAL NATIONAL COMPETỈTION 2016 ĐỀ THI TRUNG HỌC cơ sở SECONDARY SCHỎOL PAPER VÒNG 1: TIẾP SỨC TOÁN ROUND 1: RELAY RACE Thời gian: 30 phút cho cá 6 câu hói (Duration: 30 mínutẼS for 6 problùtns) Problem 1 . A box con lai ns 70 balls. The ratio oỉ the numbe' of red balls to the nunnber of white balls is 2:3, and the r alio of the number of white balls to green balls is 3:5. Delermine the number of red balls and the nLimber of green balls. Problem 2. Given that 3*4-3 a 4 4 a -9-M6- 25, and 5 7 - 5 a 4 7 2 - 25 4 49 - 74. Find the integer X such thai 6 “ X - 45. Problem 3. A convex polygon has all of its oiagondls havíng the sa me length. How many s des can thi s polygon ha ve? 3x - 1 2 Problem 4. Find all integsh values of X (where X 0) such that the expression p = — — has a non-zero

    integer value. x Problem 5. Let X and ybe :wq positĩvereal numbe^s and given theexpression p ~y-2x. Determine the sign of the expression p. X + y +xy Problem 6. Gìvcn the following ĩigure. where AB and DE are both perpendicular to BD, Ac is perpEndìcular to CE. AB = 5 cm, BD 5= 12 cm, and DE = 7 crrì. Find Ihe length of BC. ( The diagratn is CUỐC THI CẦU LAC BO TOÁN TUỒI THƠ TOAN OUỒC 2016 CHILDREN'S FUN MATHS ỊQURNAL NATIONAL COMPETITION 2016 ĐỀ THI TRUNG HỌC cơ sở SECONDARY SCHỎOL PAPER VÒNG 2: DU LỊCH TOÁN HỌC ROUND 2: MATHEMATIC s TOUR Thời gian: 30 phút cho cà 6 câu hói (Durủtion: 30 minutes for £ problems) Problem 1. ir the sum of three consecutive integers is snialler thar 75, what is the maxỉmum possible value of the smallest number? Problem 2. Gìven 2016 real numbers a^, a 2 , ... , and a 2 r Jie . Given thai the sum of any 5 arbitrary numbers among the given numbe r s are equal lo 0, Fina a 2ữ16 . Problem 3. Find Ihe measure GÍ the smallest angle of an isosceles LrapezíuTi having one base equal tó its side and the ữLher ba se equal to its aiagonal. Problem 4. Fihd the mỉnimurn value of the expressỉon p = |x - 4| - |x - 7|. Problcm 5. Given a convex quadrilateral ABCD having z 'ABC = ZCDA = 90° and ZBCD > ZBAD. Compare the lwo diagonals AC and BD of the quadrilaleral. x + 4037 x + 6050 X 4 8061 X + 18122 Problem 6. Solve the followlng equation 2016 4 2015 2014 — 2013 — ' GLOSSARY VŨ THÀNH NAM ác từ tiếng Anh được sử dụng trong Cuộc Ihi Câu lạc bộ Toán Tuổi thơ toàn quốc ngày 07.06.2016 tại Hà Nội. íraction phản số number SỂ patíern quy luật guess đoán times lẳn diagram biểu đổ dot chấm trapezium hình thang area diện tích the maximum vaiue giá trị lán nhất expression biểu thức sotve giải equation phương trình riglìt pentagona! prism lãng trụ đứng ngũ giác side cạnh suũh that sao cho vprtex đỉnh exỉst ton tại príme nguyên tố integer số nguyên total tổng quadrilateral tứ giác perpendicular vuông góc midpoinỉ trung điểrn respectively tương ứng iength độ dài trĩangte tam giác inỉemal angle bisector phản giác trong compare so sánh evaluate tính con ve X polygon đa giác lồi diagonal đường chéo measure so đo intemal angie góc trong prove chứng mình ĐÁP ÁN ĐỀ THI CÁ NHÂN THCS ANSVVERS FOR SECONDARY SCHOOL INDIVIDUAL PAPER Problem 1. X = 6 8 (5 điềm) Problem 2. 120. (5 điếm) Problem 3. 15; 3. (5 điểm) Problem 4. 16. (5 điềm)

    r 1565 2 Problem 5. cm 24 (5 điểm) 5 (hoc sinh viết 65— crrr 24 vẫn cha điểm tối đa). Problem 6. 1. (5 điểm) Problem 7. 2Ũ33136. (5 diếm) Problem 8. 5. (5 điếm) Problem 9. No. (5 diểm) Problem 10. 25. (5 điểm ) Problem 11. 5 cnv (5 điểm) Problem 12. 35 cm. (5 điểm) Problem 13. A > B. (5 điếm) Problem 14. 1. (5 điếm) Problem 15. 900°. (5 điểm) Problem 16. Let F be the rridpoinl DÍ Ac and draw Ihe lines EF and DF, then EF is the midsegmeht of ĩhe trìangle ABC, Ihus EF lì AB and AB = 2EF(1). ị '5 điểm) A s nce DF ỉs the median of the right triangle ADC wĩth respec: to the hypotenuse, DF - FC -AF => ZCDF= zc. (5 điểm) s nce ẼF H AB => ZCEF = ZB = 2 zc (5 điểm) => /ŨFE = /CEF - / CDF = /c = /COF => ZDFE - ZEDF => DE - EF (2). (5 điềm) F rom ( 1 ), (2) => AB = IDE. (5 điềm) Học sinh làm cách khác đúng vẩn cho điềm tối đã. Lưu ỳ: Học sinh có thế xét thêm khi góc B tủ hoặc vuông. Biếu điếm không thay dôi. ĐÁP ÁN ĐỀ THI CÁ NHÂN THCS ANSVVERS FOR SECONDARY SCHOOL INDIVIDUAL PAPER VÒNG 1: TIẾP SỨC TOÁN ROUND lĩ RELAY RACE VÒNG 2: DU LỊCH TOÁN HỌC ROUND 2 : MATHIMATICS TOUR Mỗi ỷ đúng ơUỢc 1 diếtn, cả bài dũng dược 2 diểm . Problcm 1. Red: 14; Green: 35 (học sinh viết 'đỏ; 14, xanh: 35" vẫn cho điểm tối đa). Problem 2. 3; -3 [học sình viét ±3 cũng cho d ểtv toi đa). Problcm 3. 4; 5. Problem 4. 2; 6. Problem 5. p > 0; p = ó (học sinh viết p > 0 cũng cho điểm tối đa). Problem 6. 5 cm; 7 cm. Môi bài dúng dược 2 Problem 1. 23. Problcm 2. ũ. Problem 3. 72°. Problem 4. 11. Problem s. AC > BD. Problcm 6. -5. KẾT QUẢ GÂO LẠC BỘ TOÁN TOÓI THƠ TOÀN QOÔG 2016 - THCS GIẢI CÁ NHÃN STT Sỏ bao danh Ho va tèn Tình, Thanh phổ G ki 1 1 VP12 Lẻ Thanh Bình Vĩnh Phúc Vàng 2 VP11 Dương Quang Giang Vĩnh Phúc Vàng 3 VP09 Nguyeh Ván C^iỂn Vinh Phúc Vãng 4 TH07 Vũ Tuấn Kiệt Thanh Hóa Vảng 5 TB07 Vũ Hải Đàng Thải Binh Vàng 6 PY07 Nguyễh Đặng Hoàng Tin Phú Yẽn Vảng 7 PT10 Lẽ Vương Hưng Phủ Thọ Vãng 8 NĐ07 Nguyễn Phương Nam Nam Định Vàng 9 LSI 2 Trấn Hoàng Quoc Anh Lạng Sơn Vàng 10 HP07 Khuất Nguyên Cương Hảì Phóng Vãng 11 YB08 Trần Anh Đức Yên Báì Bạc 12 VP07 Tạ Nam Khánh Vĩnh Phúc Bạc 13 VP10 Phan Trung ŨChg Vĩnh Phũc Bạc 14 TH10 Lê Trung Kiên Thanh Hóa Bạc 15 TB12 Phạm Xuán Trưởng Thái Bình Bạc 16 QNIglO Nguyên Công Thảnh Quàng Ngãi Bạc 17 PT09 Nguyễn Huy Toàn Phú Thọ Bạc 18 PT11 Trần Thị Hiển Trarig Phí, Thọ Bạc 19 PT07 Nguyẳn Phúc Thánh Phú Thọ Bạc 20 ND11 Nguyền Đức Anh Nam Định Bạc 21 NĐ10 Nguyễn Huy Vũ Nam Đinh Bạc 22 NĐQ8 Đinh Xuân Hoàn Nam Định Bạc 23 LCaữS Hoàng Quang Thắng Lão Cai Bạc 24 HPD8 Nguyên Lân Cương Hải Phòng Bạc 25 HP12 Dố Tuấn Minh Hải Phòng Bạc 26 HN09C Lé Hoàng Minh Há Nội c Bạc 27 HN11A Trần Vlinh Dũng Hà Nội A Bạc 28 HN10A Tạ Sơn Bách Hà NõiA Bạc 29 BG07 Nguyên Thị Diễm Quỳnh Bắc Giang Bạc 30 BGỮ8 Phạm Minh Quân Bấc Giang Bạc 31 YB09 Nguyễn Trung Hiếu Yên Bái Đóng 32 YB12 Nguyên Vàn Tiến Yên Bái Đỗng 33 SG07B Nguyễn Thiện Nhán TP. Hố Chi Mình B Dóng 34 SGlOB Ngô Tấn Sang TP. Hổ Chi Minh B Đổng 35 SG12A Bùi Lãm Tiến TP. Hố Chi Mình A Đong 36 TGỮ8 Đinh Nhựt Tiến Tisn Giang Đổng 37 TG09 Nguyễn Lê Thanh Hương Tiến Giang Đỏng 38 TH09 Đinh Thanh Nga Thanh Hóa Đóng 39 TH11 Hoàng Khảnh Linh Thanh Hóa Đóng 40 TH08 Đàm Lê Tuấn Kiệt Thanh Hóa Đóng 41 TBOB Phạm Thị Thu Hà Thái Binh Đổng STT Sũ bao danh Ho và tên Tỉnh, Thành phũ Giải 42 TB11 Nguyễn Nhật Minh Thải Bình Đổng 43 PV10 Nguyễn Háng Ngọc Phú Yẻn Đổng 44 PY1 1 Lưu Thị Như Quýnh Phú Yên Đong 45 PY12 Võ Mạnh Quyển Phú Yên Đồng 46 PT08 Trổn Tìèn Long Phú Thọ Đong 47 NB12 Hoàng Lè Giang Ninh Bình Đóng 48 NB07 Trân Mạnh Trí Ninh Binh Đồng 49 LS09 Nguyên Tuấn Dũng Lạrg Sơn Đóng 50 LCa07 □ương Minh Quang Láo Cai Đổng 51 LCalD Phạm Đửc Thang Láo Cai Đóng 52 HB07 Nguyễn Khánh An Hòa Bình Đổng 53 HP09 Trần Thanh Hải Hải Phóng Đổng 54 HN07C Nguyên Vinh Khánh Hà Nội c Đống 55 HN10C Đỗ Thành Đạt Hà Nội c Đổng 56 HNOSC Nguyền Mình Thy Hà NỘI c Đóng 57 HN07A Nguyền Quốc Dũng Hà Nội A Đổng 58 BG12 Chu Bá Hiếu Bác Giang Đống 59 BG10 Diêm Thị Quyèn Bác Giang Đổng 60 ĐL07 Nguyễn Tân Dũng Đắk Lầk Đổng 61 VP08 Lẽ Ngọc Hoa Vĩnh Phúc Triển vọg

    62 SG08B Phạm Nguỵẻn Hoàng Phụng TP. HóChi Minh B Triển vọrg 63 SG09B Nguyễn Tnãnh Qua^g TP. HỂ Chi Minh B Triển vọng 64 5G11B Bùi Nguyễn Ngọc Thắng TP. Hổ Chí Minh B Triển vọng 65 SG07A Trần Lý Bằng TP. Hỗ Chi Minh A Triển vọn g 66 SG10A □ương Phúc Thịnh TP. Hồ Chi Minh A Triển vo^g 67 THI 2 Nguyễn Tnanh An Thanh Hóa Triển vọ’’g 68 TB09 Nguyên ũửc Hiếu Thai Bình Triển vo-g 69 PY09 Lẽ Như Hoài Thương Phú Yên Triển vọng 70 NDD9 Nguyễn Thành Duy Anh Nam Định Triển vcrg 71 LS07 Phàm Tuấn Kiên Lạng Sơn Triển vong 72 LS08 Lộc Tuấn Hùng Lạng Sơn Triển vọ”g 73 LS10 NguyỄn Khánh Lình Lạrg Sdn Triển vọ^g 74 LCall Bùi Phương Anh Láo Cai Triển vọ^g 75 HB09 Trần Anh Đửc Hòa Bình Triển vọ r g 76 HBOB Nguyên Gia Bách Hòa Bình Triển vọ n g 77 HBt2 Đinh Thị Ý Thơ Hòa Bình Triển vọng 78 HP11 Trần Khánh Huyền Hảl Phóng Triển vọng 79 HN12C Đặng Hùng Dương Hà Nội c Triển vọng 80 HN 1 ‘ c Nguyễn Tuân Kiệt Hã Nội c Triển vọ r, g 81 HN08A Nguyễn Tnãi Hùng Hà Nội A Triển vọ n g 82 HN12A Lê Thanh Tùng Hã Nội A Triển vọng 83 HNŨ9A Vũ Đức Quang Hà Nội A Triển vọ n g 84 BGŨ9 Trương Anh Huy Bắc Giarg Triển vọng 85 BG11 Trấn Tìén Đức Bấc Giarg Triển vọng 86 ĐL10 Nguyễn Tấn Tà 1 Đảk Lắk Triển vọ n g KẾT QUẢ CÂU LẠC BỘ TOÀN TOÕÌ THƠ TOÀN QUỐC 2016 - THCS GIẢI TIẾP SỨC TOÁN VÀ DU LỊCH TOÁN HỌC A f \ \ 1 r A % tể NHẠNXET BAI LAM CUA THI SINH TRONG cuọc THI CLB TTT TOAN QUOC 2016 Phần thi cá nhãn của bậc THCS cỏ 120 em ihi sình dựlhi. Phổ diểm trài từ 5 đến 90 điểm. Nnứhg bài câ tỉ lệ thí sinh làm đúng thấp chủ yếu rơi vảo các bài hình. Sau đây chúng tôi đưa một số nhận xét và hướng giải cho một sò Dài để bạn đọc tiện tham khảo.

    Các bài 3, 6, 9 r 14 là các bái củ LÌ lệ học sinh

    làm dũng khá cao chiêm Lử 73,2% đen 33,19% tổng sô Lhí sinh dự thi. Đầy là nhửhg bài toán dại số cơ bản thuộc dạng tìm giá tri nhỏ nhất, tính glá tr[ biểu thức... • nếp đến lả nhóm bài 13, 4, 1, 15 có lỉ lệ bải Lhí sình lãm đúng gần bằng nhau. Thật ra các bài này thuậc dạng Lính toán đơn giàr, Lưy nhiên để □ịch được đúng vá hiểu dể các bái nãy khá khá.

    Bà ' 13, đây là dạng toán so sárh 2 biểu thức A và B chứa lũy thừa. Để so sánh được ta cần phải đưa các lũy thừa dó về cĩrg cơ số hoặc cùng só mũ. Sau khì biến đổi, ta có A = 162. (243) 1 10 ;

    B = 150.(1 25) 10 . Vậy A > B , có 76,47% 1h sim cỏ đáp àn dũng. ★ Bái 4, để bái yêj cáu tìm hinn có 66 chấm nhò. Ta thấy rằng hình 1 gổm 6 chấm nhỏ, hình sau nhiều hơn hình liền trước 4 chấm. Đáp àn là hình thứ 16. Có 75,63% thí sinh làm đúng.

    Bài 1 , khi Thí sinh dịch và hiểu được để Ehi việc giải bải toàn trà nén rẩt đdn giản. Đày là dây số cách đểu nhau 1 khoảng là [JỊ|--lY 2 5 Ta 48 16 J 48 1 3 tính dược X =4, y = — . 6 3
  • Bài 15. để tính số đo các góc trong ta cẩn tính được so cạnh của đa giác rày, áp dụng còrg thức tĩhh số đưâng chéo của da giác n cạnh lã n n .

    tính được số cạnh bằng 14. Như vậy, tổng số đo các góc trong là (14 -2). 180° - 900°. Bãi 1 và bài 1 5 có 68,91 % thí sinh làm đúng.

    Bà I 7 là dạng giải phương trình chửa dấu giá trị

    tuyệt đối. Ta thấy vế phải của phương trình luôn không ãm hên X cũng không âm. Từđó, bỏ dấu già trị tuyật dối và thư gọn ta tìm được X = 2033136. Cỏ 63,87% thí Sinh cố đáp án đủng.

    Các bải 5, 11, 12, 16 có số thi sinh làm õúng dưới 50%. Đáy là các bài thuộc dạng toán hình học.
  • Bái 5, la kẻ DK i. AB, dẻ dáng lính được Dc = KB = 1 2 cm. Ảp oụng hệ thức lương trong tam giác 1 69 vuông ADB, ta cỏ AB = CIT1 - Vậy diện tích 'ỊỊ 0 hình thang lè cm 3 . Bài này không khó

    nhưng mất nhiểu thời gian vi lởi giải cần kè thêm hình, cẩn thận khi tinh các phân sổ. có 39.5% thí sinh có đáp án đứng.

    Bài 11. dể tính được bải này ta cũng cán phải lấy

    thÊm điểm. Gọi o là Lrung điểm của DC, khi đó MO vã ON lã đưdng trung bình trong AAŨC vá ABCD. Dũ vậy MO ti AC. NO ỉi BO > MO ĩ \IO. Áp tính Chat dường trung binh ta tính được MO - 3 cm, NO - 4 cm. Tứ đó, MN - 5 cm. có 29,41% thí sinh có đàp án đủng. 4- Tỉ lệ thí sính làrr đúng thấp nhất lá bái 12 (23,53%). Tuy khàng cần phải vẽ thèm hinh nhưng bàl này được cho là khã khó so vâi các bài hình khác trong đẾ. Trước hết ta áp dụng tính chất đưóng chân giác trong của AACB ta có AB AD 60 BO-BD 60 80 4— - 4— t: ì — — - t>— — = — — -1, (1) BỌ DC BC BD BC BD □ể dàng chứhg minh dược AABD oo AACB (g.g) AB AC 60 80 m => 4“ = 4“ í=> 44 = 44- (2) BD CB BD BC Từ (1 ) và (2) la tính được BD - 35 (cm).

    Bài 1 6, dãy là dạng bài toán hình yêu cẩu viết lờ giải oằng tiếng Anh. Có một số Lhỉ sinh có lởi giải diirg tuy nhiên trình bày bẳng liếng Việt ha ặc viết irộl số tư tiếng Anh chưa chính xác, ngứ nháp câu cnưa chuẩn. Bái này cố 29,41% thí sinh nằm Irong phổ diểm lừ 1 ũ đến 25 điểm. Bạn đọc có thể tham khảo lủi giải chi tiất phần đáp án đẻ thi. MAI VŨ ĐỔI MỚI DẠY VÀ HQC TOÁN HỌC MÀ VUI VỚI BÀI TẬP MỞ NGUYỄN THỊ BÍNH (707 tòa B, Mulberry Lane. Hà Đóng. Hà Nội) Việc dạy học hiện nay không còn đan thuần là truyền đạt kiến thức mà ngirởi giảo viên phải đóng vai írâ là người truyền cảm hứng học tập cho các em học sinh, giúp các em cô niềm dam mê học hói, tim tòi và phái triển hản thân hơn nữa. Đổi mới phương pháp dạy học là làm tích cực hòa hoại dộng học tập, tạo sự hứng thủ cho học sinh trong mẫi tiết học. Dưởi dày, chủng tói trinh bày phương pháp tổ chức giò học cho học sinh dưỡi dạng Bài tập mỏ vỡĩ hình thức Học mà vui- Phương phép này cò tác dụng rất lởn khi tiến hành trong các giờ ôn tập cuổi (nỗi phần, mỗi chương vá cuối học kì. 1 . Các dạng toản vế phương trinh dường thẳng Bài 1. Cho 3 điểm A, B và C(-3, 6) như hình vẽ. Sử dụng 3 điểm này, n ãy ra một bài Lập thể hiệr được các kiến Ihửc cơ bản vể các bại đưòng Trước khi dạy bài này, giáo viên đã yỗu cẩu học sinh chuẩn bị các dạng bài tập vé hàm sổ và đỗ thị trong chương lỉ Sau dở học sình suy nghĩ và dưa na các càu hỏi vê phán kiến thức này, chẳng hạn. 1 .1. Xác định hàm số y - ax + b biết đổ thị hàm số cắt trục tung tạì đìểrr, B và cắt trục hũành tạt điểm A? Càu hải này có tác dụng khắc sâu được kiến thứt về dó thị hàn - ' so bậc nhất. Dưởng thẳrg y - ax - D cắt trục tung tại điểm có tung độ bàng b và cắt trục , — h hoành tại điểm có hoành độ là — ■ Hơn ru7a nếu giả theo cách thứ hai: Đường thảng y - ax + b đi qua 2 điểm A(-3, 0) vả B(D, 3). Đáy cùng là cách để học sim tự tìm ra tọa độ A, B <h nhír trên hình vẽ má giảo viên khống cán ghi rõ :ọa độ. 1.2. Xác định hàm số y = ax + b bĩểt đổ thị của nó di qua 2 điểm B và C7 Giải càu nảy sẽ giúp nọc sinh thảnh thạo kĩ nâng giải hệ phương trinh. Đáp só'j y = -X + 3. 1.3. Xãc định hãm số y - ax + b biết đo thị của nó di qua c và song song với AB. Đoi với bá này giảo vièn cẩn nhấc học sinh dìểc kiện b 1 * b.,. 1 .4. Xác định hãm số y = ax + fa biết đó thị của nó đi qua A và vuông góc với AB. Điều kiện để nai đường thảng y - a r x + b 1 và y = a 7 x 4- b 2 vuông góc vâì nhau lá a 1 .a 2 = -1 . 1.5. Tìm giao điểm của naí đường thẳng tim được à 1.3 vả 1.4. Khi học sinh có ý định cật cảu hài này, giáo viên cô Ihể giúp học sinh đạt lẽn các đLlủhg thẳng cho dế gọi, ví dụ: (d,|); (d 2 ). <N đó, nọc sìrh cần phải giải hệ phương trinh I y = X + 9 y = -x-3 Nghiệm cùa hệ phương trinh hên là g ao điểm cần tìm. Do đó D(H3, 3). 1.6. Tứ giác ABCD lá hình gì? Câu hỏi nây rất hay vi học sinn dược ón tập cả đạ số và hĩnh hpc. 1.7. Tìm tọa độ lâm 1 của hình VL.õrig ABCD. Tâm l(X|, y|) là tr jng điểm của AC nin X| = -3 + (-3) 0 + s -3;y| - - 3. Vậy l(-3, 3). Từ đó ta có b = 3, a = 1. 2 1.8. Tim diện ưch hình vuông ABCD? Ta có S ABCD = AB 2 = OA 2 + OB 2 = 18. Với một bài loàn thi cũng dã có khá nhiều càu hòi. Nhưng nếu giáo viên khéo động viên học trò thi còn có thể ra được khá nhiều càu hủi mới lạ nữa. 1.9. Viết phương trinh cùa dường thảng 01? 1.10. Xấc đinh góc giữa đưùng thảng AB vởì tía Ox? Mặc dù càu 1.9 vã 1.10 đểu là câu hỏi phát triển của cảc cáu hỏi Irước nhưng dểu phải dùng dến kiến thức cơ bàn, trong câu 1.9 thi cắn dùng cạrìg đường thẳng y - ax, cảu 1.10 ôn lại cách tírh tỉ sõ lượng giác của một góc ri"ơn hoặc dùng tính chất tam giác cán. Lun ý. Giáo viên yêu cầu hạc sinh dưa ra càu hói sau phải khàc dạng càu hòi trước dế tránh trường hộp cò nhiều càu hỏi tương tự nhau.
  • Các dạng toán VỂ phương trình bậc hai Đây là nội dung cực kì quan trọng trong chương trĩnh Đại số lớp 9, đồng thời là nền tảng vững chắc để học sinn học liếp ò bậc Trung học pho thõng. Học tốt ve phương Lrlnh bậc hai, đáy sẽ lá cd sở dể giải quyết phương trình bậc cao (bặc 3, 4, 5, ...), vẽ dấu của tam thức bậc 2, vê vị trí tương dôi giữa đuỡng cong và đường thẳng ... Sau đấy tà một số bài tập mỏ và cãc càu hỏi của nô mà hoàn toàn học sinh có thề tự đặt ra và giải được. Bài 2. Cho phương trình X 2 - 2(m I 2)x I m I 1 = 0. Hãy ra cãc cảu hòi dể làm thònh để toán thể hiện các dạng bải ma em biết? 2.1. Giải phương trình khi m - 1 [hoặc bất cứ số _3 hừu tì nào, muốn khó hơn ta sẽ cho m =--). 2.2. Tim các giá trị của m để phương trinh có hai nghiệm phân biệt? Đối vởi bải này, la có A' = m 2 + 3m + 3. Để giải bất phương trinh A' > 0. học sinh lớp 9 phải lựa chọn một trong hai cách; phân tícn tnãnh tích rồi xét dấu hoạc biên đoi thành dạng A’ = (am + b ) 2 + c > D rối giải tiếp. □ ù học sinh có lự giải quyết được thi giáo viên cũng nén giảng cho học sinh, ở đây A m = 9 - 1 2 < □ nên tam thức TI 2 - 3m - 3 vô nghiệm, vậy õùtng tìm cách phân tích cho ton thời gian mà cẩn ôi biến đổi dể chứng tò biểu thức luôn Gương hoặc luôn âm. 2.3. Tm m dể phương trinh cỏ nghiệm kép? 2.4. Tim m để phương trình vô nghiệm? 2.5. Tm m õể phương trinh cố hai nghiệm trải dấu? 2.G. Tim rn để phưtìng trình có hai nghiệm cùng dấu? 2.7. Tìm m để phương trình có hai nghiệm ũhân biệt cùng dương? 2.G. Tìm m để phương trình có hai rgliiện’ cùng âm? Lun ý. Có thề ả càu hỏi 2.7 và 2.8 học sinh gặp khò khăn ỏ bước kết hợp nghiệm của các bấí phitơng trình. Lúc đó giảo viên cẳn giúp càc em nẳm được cảch kết hạp nghiệm. 2.9. Tinh p = X 2 - x 2 theo m. Luu ý. Dạng cãu hỏi này thì cứ phương trình bậc hai nào chứa tham sổ thi đéu hỏi được. Ta có í 9 , , 1 f b . c 1 p = +xị — (x 1 + x 2 r -2x.|X 2 = I -—-2— { a a J = [2[rrH-2ji] 2 -2(m + l) = 4m 2 + 14m + 14. [học sình pnảì nhđ rằng dể áp dụng được hệ thức v-ét thì trước đó cẩn phải thêm một bưởc ừả lởi cáu 2.2 phương trình luôn cố 2 nghiệm phân oiệt). 2.10. Tin giả trị nhỏ rhất của Q - X 2 + x§. GỢỈ ý trả lửl. Q = x? + x!=|2m + || a -i 7 7 Váy MinQ = — <=> m = -— . 4 4 2.11. Tìn một hệ thiVc liên hệ giữa x r x 2 ỡộc lập vâi m? Gảỉ câu hỏi nảy học sinh dược khác sèu kiến thức về hệ thức Vi-ẻt và vận dụng một cách linh hoạt. Theo hệ thức Vi-ét ta có b . x t + x 2 = — = 2m + 4 a ũ X|.Xj = — = m - 1 <=i 2x jX 2 = 2m + 2. a Trù' theo ve của hai đẳng thức trên ta õược
  • + x 2 - 2x 1 x 2 = 2 là một hằng Sũ, độc lập vói m. Trên dãy là một số câu hỏi thông thường nhất vể phương trình bậc hai cnửa tham số. Với nhiTng cáu hỏi đó hầu hết các kiến thức Cd bản về phương trình bậc 2 đâ đuợc khắc sâu. Vởi đỏi tượng giỏi hơn. giáo viên có thể ra Lhên câu hỏi sau đây, vừa lá để phong phủ thèm càu hoi, vứa là để tham gia cùng học sinh phát triển bài toán, gỉúp không khí lớơ nọc thèm sôi nổi. 2.12. Tìm giả trị của m để x.(1 - 2x^) + x 2 {1 - 2x,|} - m 2 . Một lẩn nừa giáũ vlèn nhấc lại vài hạc sinh râng tất cả các bài tập liên quan dến các nghiêm X 1 , x 2 của phương trình bậc 2, đểu phải sử dụng hệ thức V -át. Như vậy ta có phưdng trình X 1 f Xj - 4x^2 = m 2 « 2(m + 2) - 4(m + 1) m 2 t=> -2m - m 2 « m(m + 2)=0 í=hii=Ũ hoặc m = -2. Học sình dã phải giải mộL phương trình bậc ha khuyẽl má đủi khì do quá quen giải phương trinh bậc 2 dú các hệ sổ nên cũng không ít học sinh có chút lúng túng. Có thể trong khi đưa ra câu hỏi 2.12 thỉ giáo viên cũng đưa tỉìêm cáu hồi 2.13 vả yêu Gầu nhôm có học sinh giòi íàỉrt. 2.13. Tìm m để Xi(1-2x 2 ) + x z (1-2x 1 ) = 4Vm-1 -8. Vâi bài toán này buộc học sinh phải giải phướng trình VÈ tỉ -2m- 4ựm - ĩ - s « 2rn + Wm— 1 -8 = 0 <=> m + 2Vm-1 -4 0 « m-1 + 2i/m-ĩ -3 ũ 4=> X 2 + 2X - 3 - 0 (Đặt X - Vm-1) Ta có (1 + b + c = 0 X 1 = 1 ; X 2 = -3 rn = 2. Số nghiệm của phưđng Irhh bậc 2 cỏ mổi quan hệ vái vị trí tương dôì của một dường thẳng vá một parabol. I lọc sinh có thể dựa trên phương trìn n bậc hai ơã cho biến đổi thành X 2 - 2(m + 2)x - m
  • 1. Khi đó xét parabol y - X 2 (P) và đường thảng y = 2(m Ý 2)x - m - 1 (d) để ra các bài tặp sau đáy. 2.14. Vữi m = ', hãy tìm Lạa độ giao điểm của parabal (p) và đương thảng (d)? 2.15. Tìm m dể (d) cắt (p) lại hai diểm phán b ệt? 2.16. Tim m để (d) tiếp xúc vớì (p)? 2.17. Tìm m dể (d) khàng cắt (p)? 2.18. Tĩm m để (d) cắt (p) tại 2 diểm khác phía đo với trục tung? 2.19. Tirr m dể (dj cắt (P) tại 2 diểm cùng phía dõi với trục tung? 2.20. Tìm m để (d) cắt (P) tại 2 điểm nằm bẽn phả trục tung? 2.21 . Tìm m để (d) cắt (P) tại 2 điểm nằm bèn trá trục tung? Các bàì lặp mang ịỉnh tổng kết, tiểu kết như trên cỏ rấ: nhiều. Saư một vài bá lí thuyết hoặc kểl thúc một vẩn đề hoặc hết một chương, một phấn, giáo vièn cùng học sinh đúc kết lại các dạng bà tập cơ bản theo cách tự mình ra câu hỏi sẽ giúp học sinh nắm vững kiến thức hơn. Saư đây chúng tôi xin trinh bầy rõ hơn về cách tiến hành hay nôi cách khác tà phương pháp tổ chức lóp học dể thực hiện ý tưởng nêu trên. Tô’ chức lớp học: có thể tiền hành theo 2 phương án. Phương ãn 1. chia lửp thành 2 dội, tổ chửc Ih giứa 2 dội với nhau. Đội A ra câu hỏi cho đội B. Sau 3-4 phút doi B phải có lời giải trình bày cò thề nói hoặc bằrg đè" chiếu tùy theo điều kiện. Đội A cũng ohải cá đảp án. Như vậy 2 đội đểu phải giải bài toán đò. Tiếp theo lá đỏi B ra càu hải chữ đội A. Yêu cầu không lạp lạt dạng cáu hỏi dội A. Vá cứ tiếp tục như vậy. học sinh sẽ rất hào hííhg, tạo không <h’ SÕI động cho lứp học. Vai Irò của giáo viên là làm vọng tài chơ 2 đội để đánh giá câu hỏi của đội nào hay, thể hiện được kiển thức cơ bản; lài giải cùa dội nào đúng, hay, gọn và đội nào huy động được nhìẽu thành viên tham gia giải bãl hơn. Phương án 2. chia lởp thánh nhiẻu nhâm. cảữ" này phù hợp với bài toán dã có sần câu hữ . Như vạy việc ra các câu hòi chơ bài lập mở dược lảm vào đẩu giờ, trong khoảng 10 phút, cà lớp cùng ra đề nhưng chứa giải bài. Giáo viên ghí lại trả" bảng, cnầng hạn với bài toán mở vé phương trình bậc 2 nêu trẽn ta ghl 10 câu hỏi dấu. Sau dó phâ" nhỏm như sau: V dụ IỞD cỏ 50 học sinh. Mối em nhận được một mảnh giấy ghi các chữ cái kèm số như sau; A v A.Ị, a 3 , .. ■ f A 1ũ 3v B 2 - B 3 ■■
  • B 1C C 1- C, C 3 , . C 10 D r ữ l D j, . D 10 E r E Z' E 3 , .. .., E 1g Đầu tiên tất cả học sinh mang số 1 (5 em) lậD nành một nhóm, nhiệm vụ là cùng nhau gải cáu so 1. Các em mang sô 2 giải càu 2. Tương tự như thế vdì các em Sũ 3, 4 1 0. Năm em mâ chụm lại lám 1 câu nhỏ thì chắc chắn ráì nhanh. Cà giáo xem VÉ chỉnh lạ! lời giải. Tiếp Ihec, fâì cả các "Ọc sinh mang tên A quy về 1 nhóm m6i, cũng như vậy với các học sinh mang B, c, D, E. Lẩn lượt nỗ người trình bày câu mình đã biết cách làm cho toàn nhóm nái. Sau bưởc náy, mỗi em đã nắm được loàn bộ 10 cáu nòi của bàl loàn. Nhóm nào xong trước sẽ tháng cuộc. Vái phương pháp dạy và học này, g áo viên không vất vả mà hiệu quả cao vì mỗi học sin ì đểu phả nỗ lực làm việc. Nếu ở bước 1, học sinh yếu trả lỡ hoặc lười cỏ thể dựa váo bạn khác để cỏ lài giả thl đến bước 2 cũng không thể lưài dược nửa, v' học sirh này phải hiểu được lời giải dể trinh bày lạ cho nhóm 9 người nữa cùng nghe. Vậy mỗi "ỌC sinh đéu phải cô gắng để hiểu được lời gíảl. Như vậy trẽn đày là 2 phương pháp tổ chúc lửo học theo kiểu “Học mả vui, vui má học", dã áo dụng rất thành còng. Kêì quả là học sìrh rất th'c" vá hiểu Dài, nhớ bàl râì kĩ. Giáo vlần có thể cho học 5 nh thi ra dể vào tiết trưốc dó và thi giải bả vào tiết sau. Tùy theo câu hỏi có phụ thuộc nay dộc lập nhau để ta chạn cách to chức theo phương án 1 hoặc phương án 2 cho phù hạp. 24 Nhản kí niệm 40 năm lập quan hệ ngoại giao Việt Nam - Thái Lan (6.8.1976} VƯƠNG QUỐC THÁI LAN BÍNH NAM HÀ AC là từ viết lắt của Cộng đồng ASEAN bằng tiếng Anh (ASEAN Community). Cộng đồng ASEAN thảnh tập chinh thức từ 31.12.201 5. Nãm 2016 này tạp chi Toán Tuổi thơ mỏ chuyên mục Cửa SỔAC để bạn đọc hiểu hơn về vùng đất con người của 10 quốc gla VỞI 625 triệu dấn. giỡi thường xuyên đạt từ 5 õến 7 triệu T hái Lan là nước thuộc nhóm các nước Đông Nam Á bán đảo (Việt Nam, Lào, Campuchia, Myanmar và Thái Lan). Diện tích 513115 km 2 , chạy dãi 2500 km từ Bắc xuống Nam. Dân so 69 triệu người. Thải Lan giáp vớì các nước: Lào, Myanmar, Campuchia, Malaysia và vịnh Thải Lan (bờ biển dài 1840 km thuộc Thái Bỉnh Dương) VÉ bờ biển Ấn Độ Dương (dài 865 km). Thủ đô lã Bangkok. Tôn giáo chính lả Phặt giảo. Ngón ngữ chính là tiếng Thái. Binh quản GDP thu nhập dầu người/năm là 4000 USD. Thái Lan có 4 vùng: phía Bắc là núi, miền Trung là đổng bằng, Đông Bẳc là cao nguyên, phía Nam là dổỉ núi. Vùng dũng bằng dược col là vựa lũa của nưỏc náy. Vùng núi phía Nam nhiều khoáng sản, mững cánh rừng và nhìểu thắng cảnh đẹp có thể là địa chl du lịch của du khách. Cuối Lhế kl XVI cương vực địa lí Thải Lan vể cơ bản như ngày nay. Từ 24.6.1930 chế độ quân chù lập hiển được thành lập. Tèn nước được đổi thành Thái Lan từ 24.6.1939. Sự chênh lệch mức sống thành phố và nông thôn cùn khả rõ. Thái Lan khá chú trọng phát triển giáo dục và phát triển nguồn nhãn lực. Họ dầu tư xây dựng cơ sở vật chất cho các trường học. Cũng giếng như nhiều nước Đông Nam Á khác, Thái Lan gặp vấn đế vể ảch tắc giao thông ở thủ đô. Hiện Thải Lan là nước có quy mố kinh tế đứng thử tư Đòng Nam Á. Thái Lan lá nước xuất khẩu gạo số một thế tấn/năm. CQngu đậu nành, dẩu cọ, dứa, ... ngoái ra còn xuất khẩu nhiều gà, nuôi tôm, cá sấu, ... Công nghiệp nước này chú trọng về: thực phẩm đóng hộp, dệt may, da, đồ nhựa, giày thể Thao, đã quý, điện, điện tử, máy vi tính, õ tỏ, xe máy, ... Thái Lan có 4 cảng biển quốc tế, 5 sàn bay quốc tế và gẩn 30 sân bay địa phương. GỂn đây viễn thông là lĩnh vực phát triển mạnh của Thái Lan. Bên cạnh dó là các lĩnh vực ngân hàng, du lịch. Du lịch là ngành mà Thái Lan có thế mạnh với các thành phố Bangkok, Paỉtaya, Phu-kẹt, Chiềng Mai dã nổi tiểrg trong cảc dịa chí du lịch của the giâi. Du lịch nổi tiếng là nhờ cách phục vụ và các dịch vụ, các tiểl mục xiếc voi, xiếc cả sấu, ... Thải Lan cũng lả nước mà ngươi Việt Nam sang du lịch nhiều nhất. PHÁT HUY TÍNH SÁNG TẠO TỪ MỘT BÀI TOÁN ĐƠN GIẢN MAI VĂN NĂM (GV. THCS Khảnh Hẩng, Yên Khánh. Ninh Bình) Trong sách giào khoa Toàn 6, tập 2 cò bài toàn 83 trang 41. Tuy tè bài toán đơn giản nhưhg nếu biết cách khai thác sẽ tạo hửng thú hạc tập đăng thòi khai dậy khả năng ĩirduy sáng tạo cho các bạn học sinh thòng qua việc xét các bài toàn đảo và bài toán lương tự từ bài toàn gốc. Bải toán 1. Lúc 6 giở 50 phút bạn Việt đi xe đạo từ A đến B vái vận tốc 15 km/h. Lúc 7 giã 10 phút bạn Nam đì xe đạp từ B đến Avởì vận tổe 12 km/h. Hai bạn gặp nhau ở c lúc 7 già 30 phút. Tính quãng dường AB. Định hưởng 1 . Tỉtn nhiều cách giải khác nhau cho một bài toàn. Cách 1. A c B I 1 1 6h5G' 7h30‘ 7h'0 r 2 Thời gian bạn Việt di từA dên c lả giử. Thời giar 1 3 bạn Nam đi tù B đến c la -7 giở. 3 Quàng đường AB có độ dài là AB = AC -t CB = 15.| + 12.-1 = 14 (km). 3 3 v Cách 2. Ta thấy đảy lã bãi loàn ngược chếu, nhưng thờ' điểm xuấl p"át lè khác nhau. Vi vậy để đơn giảr ta ỔLte vế cùng thời điểm xuất phát. A D c B I 1 1 1 6h5D' 7h10' 7h30' 7h10 f Cách 2.1. Giả sử lúc 7 già 10 phút hai bạn củng xuất phảt. Bạn Nam xuấl phái tại B đi dến c, bạ* Việt đi từ D đến c. Quãng đường DB có độ dài là DB- 1(*2 + 15) = 9 (km). Quãng đường AD có cộ dài là AD = -1.15 = 5 (km). Vậy quãng đưởng AB = AD + DB = 14 (kmj. Cách 2.2. A c B E 6h50’ 7h3ữ’ 7h10' 6h5D’ Glả sử lúc 6 già 50 phút hai bạn cùng xuất phái. Bạn Nam xuất phát từ E dến C r bạn Việt xuất phát từ Ađẽn c. Giải bài toán tương lự như trên, quãng dường A3 dải 14 km. ■Định hưởng 2. Thay đổi dữ kiện từ bài toán gốc đê tạo ra bài toán mới. Bảl toán 2. LÚC 6 giờ 50 phút bạn Víặt đì xe đạo cừA cấn B vàl vận tốc 15 km/h. Lúc 7 giờ 10 phút bạn Nam <31 xe đạp từ B đển A. Hai bạn gặp nhau ở c lúc 7 gớ 30 phút. Biết quãng đường AB dà 14 km. Tính vận tốc cùa Nam. Lòi giải. Quãng dường bạn Việt đi lừ A đến chỗ gặp nhau c là AC = 15. = 10 (km). 3 Quãng dường Nam di lừ B đển c lá 3C = 14-10 = 4 (km). 1 Vận toc của bạn Nam là 4 : = 12 (km/h). 3 Bải toán 3. Lúc 6 già 50 phủi bạn Việt dí xe đạo từA đến B vâi vận tốc 15 km/h. Lúc 7 giờ 10 phút bạn Nam đi xe đạp tử B đèn A với vận tốc 12 km/h. 3 ẽt quãng dưỡng AB dài 14 km. Hỏi hai bạr gặp nhau lúc mấy giờ? Lòi giải. Quãng đường AD bạn Nam đl được Tl/ởg khi Việt khởi hành lá “5. 1- = 5 (km). Thời gian tơ 7 giò 1 0 phút đến khi gàp mau là 14-5 1 t = - 77 =4 (già) =20 phút. 12 + 15 3 Vậy Nam và Việt gặũ nhau lúc 7 giờ 30 phút. Bãi toán 4. Lúc 5 giờ 50 phút bạn Việt đi xe đạo ỈƠA đến B. Lúc 7 giã 10 phút bạn Nam di xe đạc từ B đén A. Hai bạn gặp nhau ở c lúc 7 gìà 30 phút. Biết qLing đường AB dàl 14 km. Tinh vận ĩđc của moi người biết vận tốc của Việt lơn hơ vặn tốc của Nam lã 3 km/h. Bài toán 5. Lúc 6 giờ 50 phút bạn Việt đĩ xe đạo từ A đến B. Lúc 7 già 10 phút bạn Nam đi xe đạo ĩừ B dấn Â. Hal bạn gập nhau ỏ c lúc 7 giò 30 phút. Biết qLãhg dưởng AB dài 14 km. Tinh vả 'ốc của mồi người bìểt vận tổc của Việt bằng 1- vận tốc của Nam.

    Các bạn hãy giải hai bài toàn trén nhé. TỦ SÁCH TOÁN TUỔI THƠ

    CAC DẠNG TOAN CAC CẠV ĐO CAP TIÍƯ Hoe f ' /■«. V

    Vv/ VÃ V
  • . . - số trang: 172; Khổ: J 7 X 24 cm. Giá bia: 21 000 đổng. số trang: 276; Khổ: 17 X 24 cm. Giá bla: 45 000 dóng. Sò’ trang: 216; Kho : '7 X 24 cm. Giá bra: 22 000 đổng Số trâng: 135; Khổ: 17 X 24 sm. Giá bìa 21 oòc dõng. Số trang: 213; Khổ: 17 X 24 sm. Giã bla 35 ooc dSrg, Đóng táp l-ỉí, chí Gà năm 2G1Ũ Khổ' 13 X 27 cm Giá bla 95 ooc dồng. Số trang: 136; Kho: 17 X 24 cm. Giá bìa: 23 DOD đổng. Dóng tập tạp chỉ cà riãm 2ŨI4 Khổ; 19 X 27 cm Gĩã bìa: 145 DDO dnng. BẠN ĐỌC CÓ THỂ ĐẶT MUA TẬP CHỈ CẢ nam học tại các CỚ Sỏ BƯU QIỆN TRONG CÀ NƯỞC VỚI MÃ ĐẶT CÁC ẨN PHÀM NHƯ SAU: Tạp chi Toán Tuổi Lhcr 1: C169; Tạp chí Toán Tuổi thơ 2: C169.1; Tổng tập Toán Tuổ thơ 1 năm 2015: C169.2; Tổng tập Toán Tuổi Ihđ 1 nám 2015: C169.3; Tẩng lập Toản Tuổi thơ 1 năm 2014: C 169.4: Tổrg tập Toản ~uổi thợ 2 nám 2014: c 169.5: Tuyển chọn 10 nám Toán Tuồi thơ
  • cảc chuyên đe toán chọn lọc THCS: c 169.7: 270 hài toán hình học phẳng Olympic các nước: C1G9.8 Bàl g ảrg sỏ học: C169.6. LỜI GIẢI ĐÃ ĐÚNG CHƯA? Bài ỉoản. Cho hai số ‘hực X, y thỏa màn X 2 y 2 = 4. . ■ T i r^ I V. . Do đó A - ■ xy -n/2-I. như sau: ■ h 2 Suy ra J*ys2 Ịx - y +■ 2 < 2>/z + 2 x + y + 2 2 V 2 + 2 Tim giá Irị lớn nhải cùa ■, , 1 - X y Dau bang xảy ra khi X = y = v2. x + y + 2 Vậy MaxA = / 2 -1 khi X = y = yỉĩ. Một học sinh có lời giãi Một học sinh khác phát hiện ra lời giải trên chiỉa đùng. Theo bạn lòi glẻi trên có chỏ nào chưa đùng? Bạn hãy giải bài toán trên. NGUYỄN HÀM THÀNH (GV. THCS Lỷ Nhật Quang, Đô Lương, Nghệ An) . 5 3 . (x + y) . X +y Vi X 2 + y 2 = 4 nén xy < 1 , < — -2— = 2 4 2 Kì 24 Hay ihay các chứ cái hâi các chữ sổ. Các chứ khác nhau biểu diễn căc chứ sô' khác nhau. Lòi giải cần có lập luận lògic. Lưu ỷ é cá hai phép tinh thí các chứ cái giống nhau biêu thị các chữ sô' giống nhăU - + ONE + FOUR
  • ON| + ONE T w o F I VE TRƯƠNG CÒNG THẢNH (Sưu lầm) :mgEH»Kl 22 (TTT2 số 158) Từ giả thiếl SIX + SIX + SIX = NINE + NINE, ta có 3 X SIX = 2 X NI NE. (1) □0 2 X NINẼ = 3 X SIX < 3000 nên NINE < 1500, má N > 0 r suy ra N = 1. Thay N = 1 vào (1} ta được 30QS + 3X - 2020 + 1701 + 2E. (2) Từ 300S > 2020 - 3X > 1990, suy ra s > 6. Xét 3 trường hợp sau. • THI .3=7, thay Vào (2) dưỌc 1 701 + 2E = ao + 3X <110 nên I - 0 và 2E > 80 (loại). • TH2. s = 8, thay vào (2) ta đưọc 379 < 1701 - 2E = 380 + 3X < 410 nèn 2 < I < 3 (loại). • TH3. s = 9, thay vào (2) ta được 679 < 17DI - 2E
  • 680 + 3X < 710 nên I = 4 vả 2E m 3X. • Nếu E = Q thi X = 0 (loại). • Nếu E = 3 thì X = 2 (thỏa mãn). • Nếu E = 6 thì X = 4, mã I = 4 (loại). • Nếu E - 9 (loại vi 5 -9). Vậy bải toàn có nghiệm duy nhất là 942 + 942 + 942 = 1413 + 1413. Nhận xét. MỘL số bạn khống sử dụng bấL đẳng Ehửc dể hạn chế giả Irị GỦa N và s nèn phàí IhC' nhiều I rường hđp. r- r — Tòa soạn hoan nghênh các bạn có lởi giải tốt dược thương ki này: Hoàng Thế Sơn, 9A1. THCS Hống Bàng, Hống Bàng, Hài Phòng; Nguyễn Thủy Dương, 8A3, THCS Lim Thao, Lãm Thao, Phú Thọ; Đàng Minh Hoàng, 9A, THCS Nguyễn H ển, Nam Trực, Nam Định; Trương Thị Ngọc, 7C, THCS Nguyễn Bá Sỹ, Hoang Hóa. Thanh Hóa: Nguyễn Đức Sơn, 8B. THCS Xuân Diệu, Can Lộc, Ha Tĩnh. Các bạn sau dược khen kl nãy: Lê Ảnh Tuyết, 7E1, Chu Thị Thanh, BE1, THCS Vĩnh Tutìng, Vnh Tutìng, Lê Đức Thài, 8A2, THCS Yèn Lạc, Yên Lạc, Vĩnh Phúc: Nguyên Đáng Vù, 8A, THCS Lẽ Vãn Thịnh, Gia Binh; Bùi Xuân Diídng, 8A1, THGS Yên Phong, Yên Phong, Bắc Ninh. VIỆT HÀI KẼT QUA THI OLYMPIC QUỒC TẼ NAM 2016 CAC MON TOÁN, VẬT LÍ, HÓA HỌC, SINH HỌC VÀ TIN HỌC CỦA VIỆT NAM Olympic Toán Quốc tể IMO (Internatianal Mathematicãl Qlympiad) nãir 2016 tổ chức tại Hong Kong tử ngày 6 đến 16/7 vói 6Ũ2 thí sinh đến lừ 1 09 quốc gia và vùng lãnh tho. Thí sinh thi trong 2 ngáy 11 và 12/7. Đoản Việt Nam giành 1 huy ctiưdng Vảhg r 4 huy chương Bạc và 1 huy chương Đổng, xếp thử 11 không chỉnh thức toàn đoản tham dự. Chủ nhân huy chương Váng là Vũ Xuản Trung, nọc sinh lớp 1 2 , trướng THPT chuyên Thái Binh, Thài Binh. Đảy là lần thứ hai lrung giành huy chương vàng cho đội tuyển Việt Nam. Đặc biệt, Trurg đã từng đoạt huy chương Vàng olỵmpic Toán Tuổi thơ toán quốc năm 2012 do Tạp chí Toán Tuổi thơ tổ chửc.Đào Vũ Quang, học sinh lởp 12, trưòng THPT chuyên Há Nộii - Amsterdam, Hà Nội; Phạm Nguyễn Mạnh, học sinh lởp 11 trưởng Phổ thông Năng Khiếu, Đại học Quốc gia TP. H6 Chí Minh; Lẽ Nhậl Hoàng, học sình lớp 12 THPT chuyên Lẽ Quỷ Đòn, Bình Định; Hoàng Anh Dũng, học sinh lớp 12, THPT chuyên Lam Sơn, Thanh Hóa đoạt huy chương Bạc. Vũ ĐửcTàì, học sinh lớp 12, THPT chuyên Lẽ Hồng Phong, Nam Định giành huy chương Đồng. olyrnnic Vậl r Quốc Ịế IPhO (International Physics Olympiad) nám 2016 do Thụy sĩ vá Uchtenstein cùng đ&ng cai, được tố chửc tại Thụy Sĩ. Cuộc thi diẻn ra từ ngày 10 đến ngày 18/7. với 398 học sinh đến lừ 87 q u ỏ'c gia và lãnh thổ. Thí sinh dự Ihi của dội tuyển quốc gia Việt Nam dều đoạt huy chương, gồm 2 Huy chương Vàng, 2 Huy chương Bạc vá 1 Huy chương Đcng. Cụ thể, các em Đinh Thị Hương Thảo, học sinh lớp 12, trương THPT chuyên Lẽ Hóng Phong, Nam Định; Nguyễn Thế Quỳnh, học sinh lốp 11, trường THPT chuyên Võ Nguyên Giáp. Quảng Bình đoạt Huy chương Vàng. Nguyên Quang Nam, học sinh lớp 12, trương THPT chuyên Đạì học Sư phạm Hà Nội; Phạm Quang Minh, học s nh lâp 12, Trưởng Trung học phổ thõng chuyên Hà Nội-Amsterdam, TP. Hà Nội đoạt Huy chương Bạc. Một huy chương Đồng Ihuộc vể Phạm Ngọc Nam, học sinh lớp 12. Trướng Trung học phổ thòng chuyên Lẽ Hổng Phang, Nam Định. Đặc biệt Đinh Thị Hương Thào được nhận giải đặc biệt "Nữ sinh cháu Ả đạt kết quả cao nhất” do Hội Vật I' Châu Ả Thái Bình Dương trao tăng. Olympic Hóa học Quốc tế IChO (International Chemistry Olympiad) lần thứ 48 nãm 2016 được tổ chửc tạì Tbllisl thủ dò Grjzia. có 75 quốc gia vả vùng lãnh thố tham dự với tổng so 280 thí sình. Có 3/4 thi sinh dự th của đội tuyển quốc gia Việt Nam đoạl huy chương, gom 2 huy chưdng Vàng và 1 huy chương Bạc. Điểm thi của các thí sinh đoạt huy cnưdng dểu xếp thứ hạng cao trong cuộc thi. Chù nhân huy chương Váng là Đinh Quang Hiểu, lão 11,THPT chuyên Khoa học Tự nhiên. Dại học Quốc gia Hả Nội vả Nguyễn Khánh Duy, lớp 12, THPT chuyên Lam Sơn, Thanh Hỏa. Nguyễn Thành Trung, lóp 12, THPT cnuyên Lê Hóng Phong, Nam Định đoạt huy chương Bạc. Cuộc thi olympìc sinh học Quốc tẽ IBO (International Biological Olympiad) lân thứ 27 nãm 2016 diễn ra tư ngày 17 dến 23/7 năm 2016 :ại trương Đại học sư phạn Hà Nội, Thành phố Hà Nội, Việt Nam. Đãy là lần đầu tiên Việt Nam dăng cai tổ chức kỉ thi này. Sau một tuần diẻr ra kì thi. đoàn Việt Nam đã Vinh dự giành đuạc kết quả đáy ân tương, cả 4/4 em tham dự đểu đoạt huy chương, gồm 1 huy chương Vàng, 1 huy chương Bạc và 2 huy chương Đống. Tấm huy chương Vàng duy nhất của đoán thuộc về có gãĩ Vú Thị Chinh, trưdng THPT chuyên ĐH Khoa học tự nhiên - ĐI I Quốc gla Hà NỘI. Huy chương Bạc thuộc ve Lê Thị Horig Hoa, 1 HPT chuyên Há Nội
  • Arrstersdam. Hai thí sinh khác cùa đoàn Việt Nam là Nguyễn Ngọc Minh Hảì, học sinh ưưòng THPT chuyền Hạ Long, Quảng Ninh và Nguyẻn Đắc Hiẽu, nọc sình trường THPT chuyên Lam Sơn, Thanh Hóa dều ƠDạl huy chương Đổng. Qlympic Tin học Quõc tể IOI (International □lymp ad in Iníormatics) lăn thứ 28 nàm 2016 tổ chức tại Cộng hòa Liên Bang Nga vơi tổng số 308 thí sinh đến tử 81 quốc gia và vùng lãnh thổ. 4 thỉ sinh của đoàn Việt Nam tham gia đềư đoạt huy chương, gồm 2 huy chương Vàng, 1 huy chương Bạc và 1 huy chương Đông, xếp thứ7 không chính thức toàn đoàn tham gia. Hai thí sinh đoạt huy chương Vàng là Phan Đửc Nhậl Minh, lớp 12 và Phạm Cao Nguyên, lớp 11, THPT chuyên Khoa học Tự nh ên, Đại học Quốc gia Há Nộí. Huy chương Bạc thuộc về Trần Tẩn Phát, lốp 12, Phổ thòng Nàng khiếu, Dại hạc Quốc gia TP. Hổ Chi Minh. Lẽ Quang Tuân, lớp 11, THPT chuyẻr Đại học Sư phạm Hà Nội giành huy chương Dồng. HA MY I hãng cố d[rih li nàa đtín giản hdn vả dễ nhâ •hơn địrh r nổi tiếng Pytago (Pythagorean Theorem): Trong một lam giác vuông, nếu b và c là các cạnh góc vuông và a lả độ dài cạnh huyền, thl ạ 2 = b 2 + c 2 .
  • ứng dụng từ hàrg nghi’' nãm nay chính lả việc sử dụng định I' dào của định lĩ này: Neu Lam giác ABC có B 2 = D 2 + c 2 thì gÓG A (dốì diện cạnh a) sẽ lã góc vuông. Người Ai cập co dai dã phãt híận ra tam giác 3 r 4, 5 gọi là tam giác Pytago, do 5 2 = 3 2 + 4 2 . Họ dùng các sợi dày, tạo thành các nút để có 12 đoạn dài bằng nhau. Từ đó tạo thành các góc vuông trẽn mặt đất noi mã nai đoạn dài 3 khoảng vá 4 khoảng gặp nhau (hình 1). Hỉnh 1
  • Vỉ dụ 2 là bà toãn vê cải cây đổ. Ngưài ta đo dược từ gổc đến điểm gãy dài 7 m, từ gốc đế" điểm ngọn cây chạm đất là 24 m. 24 m H Hỉnh 2 Vậy chiếu dà từ điểm gãy đến ngọn là cạnh huyền và bằng '/í4 2 + 7 2 = i/625 = 25 (m). Từ đó chiếu dài của cày khi chưa bị gãy lá 25 4 - 7 = 32 (m).
  • Một đơòng lãn dốc có mặt cắt tạo thành tarr

    giác vuông (hình 3). ĐỊNH Lí PYTAGO Mật người muốn sử dụng định lí pylago và đưỏng lăn dể tinh bár kinh của một quả cầu đặc biệt. Anh ấy đánh dấu d ểm A rổi õẩy quả cấu đi từ chân dường lãn c đen lúc A chạm đường lãr lẩn 2 thỉ đárh dấJ là B. Khoảng cách CH vả BH đếư dược báo tại rroì điểm dừng của quà cấu. Do đó cb=Vch 2 -bh 2 . Mã CB chỉnh lả chu V của Quả cầu nôn

    CB ì/cH 2 +BH 2 27Ĩ 2ĩĩ Có bạn sẽ thác mắc sao kháng đo chiều dải từ c dến B luồn mà phải tính toán làm gì? Thật ra õáy là một phẩn trong một bài toán vật lí tổng hợo nghiên cứu nhiểu kh a cạnh cùa chuyển động lãr và cách trẽn chỉ là 1 trong nhiều cảch tinh bá" kính vặl hình cẩu. Hình 4 3ạn chắc đã biết câu chuyên nổi tiêng vể Romeo. Bạn thử tưởrg lượng Romeo muốn dùng sợ aãy buộc dể nén’, một lá thư cho Juliet đang đứng trê" ban cõng. Romeo cao 1,80 m, đứng cách ba" cống 3 m. Ban cóng cách rrál đal 5 m, Julíet cao 1 ,G5 m thì ít nhất sdi dãy phải dãi bao nhiêu th' Romeo mỏi làm được việc gửi thư? 30 (Còn tiếp ) CUỘC THI SANG TAC CAU HOI VA BAI TẶP PHÁT TRIỂN NĂNG Lực MÔN TOÁN CỦA HỌC SINH BẬC THCS LỚP 6 MÂ: RTNLOOI Cầu 1. a) Viết sổ tự nHèn nhẻ nhấl có tổng các chữ sô bằng 41 .
  • Viết số tự nhiẻn nhỏ mất có các chữ sd khác nhau và tổng các chữ số bằng 41 . Câu 2. Có bao nhiêu phần tù thuộc tập họp các số có 3 chữ số chia hết cho 90, liệt kê các phần tử đó. Cảu 3. Cho a : b, tim ƯCLN(a + 1. b) vá BCNN(a r b, a + 1). Cáu 4. Cna 4 điểm trang dỏ không cỏ 3 điểm nàa thẳng nàng. Nối 4 điểm đó vởi nhau. Tính sô' gốc có các đỉnh là 4 điểm đó? Câu s. Tính giả tri của biểu thức 51 52 53 100 "TT'ĩd 198' Câu G. Chửng minhrâng sổ B = 4. ,.48. ..8 -13...3 + 1 (gồm 10 chữ 50 4 r 1 0 chí) số 6, 1 0 chữ số 3) là sấ chính phương. Cảu 7. sẳp xếp các phân sô’ sau theo thử tự từ .- 992 994 995 997 996 nhỏ dẽn lởn , _Y. ■ - 3003 3009 3012 3018 3021 1 1 1 1 . Cáu B. Cho s = 1 + -4--' — — + — — + ... H — . Cnứng 2 3 Z 4 3 lũ 9 minh rằng s khăng là só nguyên. Cảu 9. Tim BCNN(à, b) vâi a = 123456789 vả b- 987654321. Câu 10. Vẽ ninh trống 12 cày thành 7 hàng, mối háng 4 cây. Câu 11. Trên đường Ihẳng a lấy 4 điểm theo thứ tựA, B, c r □ sao cho AB > CD. Gọi E là trung điểm của AB. F là trung điểm của CD. Hãy so sánh EF, AC và BD. LỚP 7 MÃ: PTNL001 Cảu 1. Bậc của tồng hai đa thửc như thê nảo nêu
  • Hai da thức có cùng bậc
  • Hai da thức khác lỉậc. Cảu 2. Tong của hai 5Ò thập phân vó hạn tuần hoãn có thể lả sò dạng nàũ sau dây: sò nguyên, số thập phân hữu hạn, 5ố thập phân vô hạn tuấn hoàn? Cho ví dụ. Cảu 3. Tam giác ABC là tam giác gi nếu hai đường trung tuyến BM, CN và đưửng phản giác AD cùng đi qua một điểm? Câu 4. lylột tam giác dếu có cạnh yỈ3 cm. Tính khoảng cách từ trực tâm đến mẻi đỉnh. Câu 5. Tìm nghiệm của đa thức P(x) -X 3 + 5x - 7. ,41 Câu 6. Tìm so tự nhiên n sao cho — — o.abcabc... n
  • 0,(abc) là số thập phân vô hạn tuẩn hoàn chu kì abc vâl a. b, c là các chữ số khác nhau. Câu 7. Ba SỐ a, b, c có tổng là X. Chia a, b. c theo thứ tự tì lệ vổi 2. 3, 4 rổi chia theo thứ Lự tĩ lệ vãi 3, 5, 7 thi có một số giảm đl 1. Tlm X. Câu 8. Biết đa thức f(x) — -X 2 + bx + c có nghiệm là X = -2. Tinh f(1) + f(-5). Cáu 9. Cho ninh vẽ có AB = AC = BD vã DA = Dc. Tính các gốc trong của AABC. A Cảu 10. Cho iABC cỏ hai đường trung tuyến BD và CE cát nhau tạl G. Chửhg mirh rằng GD + GE < AD + AE. Câu 1 1. Cho AABC có AB = AC = 1 3 cni, BC = 1 0 cm. 1 ính độ dài đường trung luyến BM. LỜI GIẢI ĐỀ THI CHỌN DỘI TUYÊN Dự THI OLYMPIC TOÁN QUỐC TÍ CỦA HỒNG KÔNG NĂM 2010 (VÒNG 1) (Tiếp theo kì trước) MAI VŨ (Siíu tầm. dịch vá giới thiệu)
  • Đặt n - 100Ũ3 + b, ũ < 0 < 999. Khi bỏ đi 3 chữ số cuối của n ta có 3/iĩ = 3- Do đỏ a 3 = lOODa + b => b = a(a z - 1 000) > 0 a 2 > 1000 a > 32. Nếu a > 33 khi đó b > 33(33 2 - 100D} > 1000, điều nảy vô lỉ. Do vậy ta phải cho a = 32, suy ra b = 32(32 z - 1000) = 768. vậy n = 32766.
  • Gọi ư và f> là các nghiệm của phương trinh 10x 2 + px + q - 0. Khi đó ta có a + 6 - - -Ẹ- và 10 ct|ì = vậy nên 2010 =: p - q = (-10)(ct + il) +■ IOcííÌ ■ - p = 201 hoặc (a - 1)(ịỉ - 1) = 202. p > 0 nên 2 thừa số tì - 1 và p - 1 có thể lã 1 và 202 hoặc 2 vã 101. p CÓ thể là 2 và 203 hoặc 3 và 1 02, nê" -2050 vàp--105Q. Vậy cáu trả lòi là -3 1 00. A 8 Vi Q là lắm đường tròn ngoạ! tiếp iBPC nè" Q nằm hên dường trung trực của BC, và ta có PG = CQ. Vì D là tâm dường tròn ngoại tiếp của APQA => DQ - DA - 1. Vậy Q nằm trẽn dưửng tròn tâm D bán kỉnh 1 . Ta thấy có 2 giao dìểm giVa (D, 1) với dường trung trực của BC —> tân tại 2 vị trí của Q. ■Đe PQ (- CQ) đạt gíá trị lón nhất ta chọn õiển Q nằm ngoài AABC. Gọi E, F theo Ihứ tự là trung diem cửa AD vả BC. Ta dẻ dâng chứng minh dược aADQ lá tam g Éc đểu vỡi cạnh bàng 1. EQ = Vặ 2 Ád dụng định It Py-ta^gD trong ACFŨ, la có PQ 2 =CQ 2 = CF 2 +FQ*=fAÌ +ịĩ+ệ = 2 + 1 / 3 . 2 2 v ■ \ /
  • Troig hỉnh vi bén trái ỏ dưởì, lặp họp các diểm p thỏa mãn ÃPB > 60° là các điểm nằrr : r ong hai phản đoạn (đó lá cung lớn AB cửa đường trùn ngoại tiếp AA3P, với p được lấy là một trong hai vị trí để ABP là tam giác đểu). Do vậy phẩn tỏ màu R là phẩn giao nhau cùa 4 hình như vậy đổng dạng vứi nhau (kỉ hiệu vùng R là XYZW được tồ máu trang hinh gìũra bèn dưửi). Gọi o lé tàm cúa của đường cong lân AXWB. Do tính chất dối xứng, ta cỏ XBA = 45° vá XÒA = 90°. Tươrg lự như vậy, la có WÓB = 90°. D _ c . Ọ: í V X w % " Hon ntte, ÁÕB - 2AWB - 1 20° (góc ở tâm bằng 2 lần góc tại 1 điềm trên đường trán cùng chắn một cung). =* xồw = 360° - 90° - 1 20° - 9ũ n = 60°. Trong hĩnh vẽ bên phải dưởi dảy, biên của R gốm 4 cung bằng nhau XPW, XY, vz, wz và R chứa hình vuông XYZW, Do AB = 3 vả mồi cung có bár kinh bằng Vs nẽh SxYZW =3. mói hình viên phân có diện tích là 6 4 2 4 Vạy diện tích cắn tim là II 3i/Õ 1 2 3^ 3 + 4 = 2ji + 3 - 3>/3.
  • Giả si aang nhị phân cùa n lá a k a| ĩ ^a k _2...a2a 1 . với a ( bằng ũ hoặc ' (với 1 < i < k). Ta cò 2 \ a k a k-1 a k-ĩ" a 2 vàn-2 2 I=% Từ đò f( n ) = f( a k a k-1 a k-2— a 2 a l) = f( a k a k-1 a k-2"' a ỉ) a 1 ‘T( a k a k-1 a k-2 ■ -- a 3 ) +a 2 + a 1
  • ỉ(a k } + a k-1 + a k -2 + ■■■ + a 2 + a 1 = a k + a K _ 1 -a K _2 +... + 32 + a-|. f(n) bằng tổng các số 1 trong biểu diễn mị chân của n. Do Ũ < n < 201 ũ và 2 1ữ < 2010 < 2 11 nên dạng nhị phân của n cá nhiều nhất 10 chữ số Ị vì (11111111111 2) = 2047 >2010. Vậy nên già trị lớn nhát của f(n) là 10, ta cố thể kiểm tra lạif(1023) -f[2 1c - 1) - f(1111111111 2 ) - 10. Mở rộng lục giác có các góc bằng nhau (gọi là A) bằng cách kéo dài 3 cặp cạnh dổi diện ta được một tam giác đéu. Từ đây, ta thấy rằng tất cả các lục giác A đều dược tạo thành bằng cách bồ dl ba tam giác đều nhỏ ở ba dính của một tam giác đẽu Idn. ũề dàng thấy rằng, nếu độ dài các cạnh llén liếp của đa giác A là a r b r c, d, e, f (vứt a, c, e là các cạnh cửa ba tam giác đều nhỏ) thì khl dó ta có a + b + c = c + d + e = e + f + a nén a - d = e - b = c - f. /3 Hdn nữa, lam giác đếu cạnh X có diện tích — X, 4 và lục giác A hình thành bàng cách bỏ đì 3 tam giác dểu cạnh a. e, c của tam giác mả rộng cạnh n có diện tích là
  • ậ(n 2 -a 2 -ũ 2 -e 2 } vđi n = a t b ■> c. 4 Bây già ta chia các cạnh độ dà 6, 7, 8 r 9, 10, 11 thành 3 cãp có độ dài khác nhau. V) chu vi lục giác bằng 51, là sô lẻ nên độ lệch nhau trong mãi cặp phải lã 1 ({6, 7}, {8, 9), {10, 11}) noặc là 3 ({6, 9), {7,10} {8,11}). Trong Irưòng hợp thứ nhất, 2 cạnh oên giáp vãi cạnh 6 phải là 9 và 11 (đó là 2 số lớn hơn của 2 cặp côn lạt, 6 là sổ nhỏ hơn trong cặp của nó). Độ dài các cạnh lục giác đưdcxếp liên tiếp là 6, 9, 10, 7, 8. 11 hoặc là 9, 10, 7. 8. 11, 6. Diện tích lục gìácvỏi B = 6 là ỈẬ(25 2 — 6 2 -lo 2 — 8 2 ) 4 4 (Ta cũng có thể tính diện t ch lục giác v&i a - 9 là — f-(26 z -9 2 -7 2 -I I 2 }, cũng cho kéì quả như 4 trẽn). Tương tự, với trường hợp 2. I lai cạnh kẽ với cạnh 0 phẳi lả cạnh 10. 11 và độ dái các cạnh được sắp xếp liẽn tiếp như sau 6, 10, 8, 9, 7, 11. Diện tích lục giác là ậ(24 a - -S 2 -7 2 ) = 4 4 Kết hợp cả 2 trưởng hợp trẽn, càu trả lởi là 213, (Kỉ sau ổảng tiếp) PHONG VAN (Tiếp theo kì t rước) VŨ KIM THƯ (Nghĩa Đô, Hà Nội) rong số 159+160 ra thăng 4.2016 đã đăng cắc câu hói trong một bài phỏng vẩn du học. SỐ này chủng tôi đãng một bản trá tời cho các cãu hỏi đã đãng. Tuy nhiên tùy theo thực tế gia đĩnh vã trường tôp bạn đọc , hài trả iời Cí)a bạn cờ thể khác.
  • Good atìernoon.
  • My name is Nam. Nice to meet you.
  • Thank you.
  • My name is Nam. Yes, I am. Yes, thats right/ that J s correct.
  • Yes, I am.
  • I íinished grade 9 in May (a few months ago). i Yes.
  • I tinished gracie 9 this year.
  • I study at Trung Vuong school / lfs 3 public school.
  • It is dn Hang Bal Street, near the centre of the City.
  • It is a publìc schoal.
  • Yes , ít s .
  • Yes, jt is. 11 is about 6 kìlometres írom my school to my house.
  • I clten go to school by bicycle.
  • Yes, il is. “here are about 20Ũ0 students ano more than 100 teachers in my sữí’ool.
  • Yes , it s .
  • Yes, I do.
  • Becaiise Ihe teachers are very good and the students are triendly.
  • I go to scnool every day except Sun day. Ị I go to school on weekdays írom Monday to Friday.
  • I go tủ school 6 days a week from Monday to Saturday.
  • Nũ, I am not. I have a long summer holiday.
  • I go to school in :he morn ng.
  • I usually hava 4 or 5 periods every day. -At school, I study 11 subjects. They are Mathematics, Physics, Chemistry, Biology, Lite rai u re t Hi&tory, Geography, English, Iníomnatics, ...
  • I'm good at Mathematics. -Yes, I do, Not very gooo. I have studied Englis+I for seven years.
  • Yes, I do. I like all subjects out I li ke Mathematỉcs best.
  • My ave r age in Mathematics is 9.8. in Physics ỉs 9,1 .
  • I received awards for good students at Ihe ena of each school year.
  • Yes, I am a good student biii I Ihink I am not an excellent student.
  • Yes, I do. Yes, of courae. ì Yes, they are.
  • There are 4 people n my tamily; my father r my motner, my younger sister.
  • No, it isn t. I've gol a sister.
  • She is my younger sister. She is 12 years old. I am 15 yeare old.
  • My tather ís an edỉtor. He works at Mathemetlcs and Youth Magaĩine.
  • My molher ìs a ieacher. she leaches at Trung Vuong school.
  • Yes, he does. Yes, I da.
  • No, she doesn’t.
  • Yes, she is.
  • Yes, I do. Nhận xét. Toán “uổi Lhd khen và trao quà cho bạ" có các cáu trả lờì hay trong bài phòng vấn này lả Nguyễn Thái Hằng. 7/1. THCS LỄ Vãn Thièm, TP. Ha Tĩnh. Hả Tĩnh. CLB1 6. Factorise the íòllovving polynomìal A = X 2 + 4y 2 - 4xy + 3x - Gy — 4. CL817. Determine the positìve integer n such that B = ư 1 - n' 1 + 3n 2 - 2h + 2 is a prime number. CL818. Given real nunber X satistyng the conditior X 2 - 201 6x - 2 0. Find the value of the expression c - — — ~ x — X 2 CLB19. Determ re the integer n such Ihel r 2 — 4n - 3 is a squared number. CLB2Q. Given triangle ABC with meoiar AM. Let I be a point on segment BI AM such that AM = 4AI. BI intersects AC al M. Calculate :he ratiũ , BN NGUYỀN KHÁNH CHUNG (GV. Tnỉờng Lômônôxổp , Nam Từ Liêm, Hà Nội) xmm* CÂU LẠC BỘ TOÁN TUẩl THƠ (TTT2 số 1 58) CLBG. Ta có M _ y< — 1 >— (y— | Z(y-1)-(Z-1) | x(z-l)-(x-l)

    (x-1)(y-1) (y-i)(z-1) (z-1)(x-1) Tủ' f1 ), (2) và (3) suy ra , . a b c b 4 c c 4 3 a 4b y 1 z 1 X 1 ■ — + — • +■ — y-1 X — 1 z — 1 y-1 X — 1 z — 1 X 1 \ í y 1 1 / 'í 1

    < 1
  • ' % 1 '1 Z-1 Z-1 = 1 + 1 + 1 =3. CLB7. Ta có a^ + e- l, 2 _2 _i „z ,í _(.ỉ b -c c -a a -ũ — +— +— a z + 30 0 J -4 0*41975 . V / - - V / <=» a — \ a 3 -b '" ị b ! -c ! ' c 2 -1975 a s +30 2 C ĩ -a ỉ c — — — b 2 +4 = 0 3' á 4 1974a :i +b ? aV 4 29^4 0 . hV 4 3C 7 4 a* . ÍT> — I — — — — I = tl c ? 1-1975 a 7 + 30 b ? +4 < > aV 4 J 974a' - b 7 = aV 4 29b 5 4 0* = bV + 3c' 4 a' = 0 «a-b = c = ũ. CLB8. a) Ta có a + b - 1 > ũ -> a > -b =» a 317 > (-b) 317 = a 317 > -43 317 . Do đỏ a 317 + b 317 >0.
  • Ta có a, b, ũ lả đệ dái ba cạnh của mội tatr giác rên a, b, c > 0 vè 3 4 b>c, b 4 c >0, C4 3 >b (ba: dẳng hức tam giác). VI a > 0 và b + c > a nên a a 2a 2a — : — -< — -j . (1) a + b + c b + c 2[b-c] a + b-rG Tương tự b \ b 2b — f < — —— < — TT— ■ (2) a + b-c c + a a+b + c c c 2c . -í — — < — — — (3) a + b-c a+b a+b + c CLB9. Giả sử tổn tại một cách sẳp xếp 12 số nguyên dương từ 1 đến 12 trên một đường tròn sao cha nai số kề nhau bất kì có tong lớn hơn 12. Khi đó số 1 có hai 50 kể vửì nó và long của mõi số náy vài 1 đểu lớn hơn 12. Suy ra cả hai số này phải lán hơn 11, Mả trong các số đã cho chì có số 12 lân hơn 1 1. Do đó điều giả sử là sal. Vậy không thể sáp xếp 12 số nguyên dương từ 1 đến ' 2 trèn một đưỏng tròn saa cho hai số kể nhau bầt kl có tong lân hơn 12. CLB10. Ta CÒ MA 2 +MC 2 =»(MA + MC) 2 . Mặ; khác MA + MC > AC. Do dó MA 3 +MC 2 >AC 2 , (1) 2 Tương tự MB 3 +MD 3 >^BD 3 . (2) Từ (1) và (2) suy ra MA 2 + MC 2 + MB 2 + MD 2 > (AC 2 + BD 2 ). Đẳng Ihửc xảy ra khi và chỉ khi M là giao điểm cùa AC và BD. Vậy khi M là giao điểm của hai đưdng chéo AC và BD thì MA 2 4 MB 2 4 MC 2 + MD 2 đạt glá trị nhỏ nhất lã ^(AC 2 I BD 3 }. Nhản xét. Bạn cú lờ giải tốt cả 5 cái toản r được thưỏng kì náy: ' Tran Hổng Quỷ, 8E1, THCS Vĩrh Tuông, Vĩnh Tường, Vĩnh Phúc. NGUYỀN HIỆP . ■ : ■ “ĩ — n ? 5 — B ■ ĩ — ĩ AI LÀ NGƯỜI CHỨNG MINH HÌNH HOC BẦU TIÊN? Đ Ó là một câu hỏi rất khó t r ả lởi chính xác. Dựa vào các tài liệu còn IiAi giữ lại được, ta hãy đi ngược dòng lịch sử cùa nhân loại để làm sáng lỏ vãn đề này.
  • Các nền toán học Phương Đõng cổ dại Cảc nén vãn minh xa xưa của nhà" loại được hình ttiãnh vè phát triển dọc theo các con sông lởn của Châu Ph và Châu Á: sông Nil ở Châu Phi sinh ra nển vãn m nh Ai Cập, sông Tigris và sông Euphrates à Tày Á sirh ra nền vãn minh Đabylon, sóng Indus (Ấn Hà) và sõng Ganges (Hằng Hà) sinh ra nến vàn m nh Ấn Độ. sóng Hoàng Hà và sõng Dưdng Tử (Trưdng Giang) sinh , a nền vá 4 ? minh Trung Quổc cổ dại. Các công trình toán học của ngutìi Babylor được ghi trên các bản đất sét nung, còn người Ai Cập lh‘ ghì lại trên đá và giấy cò là những chất liệu giử dược lâu bén nèn ngày nay các còng trình của họ dần dấn được biết dển. Trong khi dó ngưâi Trung Quốc và người Ân Độ lại dũng những phương tiện rất dễ hư như vỏ cây hoặc tre nên các thành tựu của họ rất khó khôi phục và dần bì lãng quên theo thời gian. Tuy nhién tất cả cảc thành tựu toán học má ngLíở Ai Cập («hoảng 400D nã ni dến 1ŨŨ0 nărr trưửc Cõng nguyên) vá ngươi Gabylon (khoảng 2100 năm đén 600 nàm trước Công nguyên) dã đạt đitợc đểu rút ra từ kinh nghiệm thi/c tiền mà không thấy xuất hiện một phép chứng minh nào. Vi vậy bén cạnh nhiững thành tựu rực rã họ cũng có nhĩíng saỉ lắm. Chảng hạn ngươi Ai Cập cho rầng cõng thức tính diện tích của mộl tử giác vâỉ dộ dà các cạnh a. b, c, d cho bởi 5 = — - . Thực 4 ra công thức này chì đúng Khi tử giác đã cho là hình chữ nhặt.
  • Sự ra dời của toán học chửng minh Khi các nền vân minh Ai cập và Babylon suy tàn, một nen vãn minh mỏi xuất hiện trong cảc thành phô' chạy dọc theo bớ biển của Tiểu Á và sau này nằm trẽn lãnh thổ cùa Hy Lạp, trên các vùng bờ biển Sicỉl và Italia. Cải nhìn tĩnh tại cùa Phương Đông cổ đại đã trử Ihànli không thể chấp nhặn ỔLfỢc, vả trong một báu không khí phát triển cùa PGS. TS. LÊ QUỐC HÁN (Khoa Toán Đại học Vinh) chủ nghía ouy lí, ngươi la bắt đầu hủi: tại sao vá như thề nào? Thalès (khoảng thể kỷ thừ VI trước Công nguyên) được xem là rgiíởi chứng min n . những kết quả hình học đẩu tiên, õng sình â vùng VlilẼtus. một đỏ thị thương mại nằm trên vừng bà biển ph ÍB lảy T ểu Á. Ỏng đi du lịch nhiều nơi , từng sinh sống ở Aì cập irong một thủi gian dai vè thu lượm được nhiểu tri thức toán học ghi trẽn các Kim Tự Tháp. T r ong hình học óng được công nhặn là dã đưa ra nhữhg Kết quả sau dãy.
  • Vật vòng tròn được phân đôi bởi bất kì đường kính nào.
  • Các góc ỏ đáy của một tam giác cân thì Dằng nhau.
  • Hai góc đoi đỉnh thi bằng nhau.
  • Hai tam giác bằng nhau nếu chủrg có hai góc vá mọt cạnh tương ứng bang nhau (Thalès đã dùng kết quả này ỡể xác định khoảng cách từ bờ bìểr đến một chiếc thuyền).
  • Góc rội tiếp chắn nửa đường tròr bằng một vuông. Giá trị của nhùng kềt quả nả/ không chí bái chinh nội dung định t! mà quan trọng hơn là ĩhalès đã dùng càc lập luận tôgĩc dể sưy ra chúng (thay v' dựa vào trực giác và kinh nghiệm như các tiỂ r ’ nhàn trưức đó). Nhá toán học pythagore sinh sau Thalès khoảng 50 năm (năm 572 trước cỏng nguyêr) trên hòn đảo Aege cùa Samos và từng sống ở thánh phố Uiletus gần nhà Thalès. Người ta cho rằng Pythagore đã học hỏi ỔLrợc nhiếu ở ỏng già thõng íhảỉ ấy. Sau khi Samos bị đế quốc Ba Tư xàrr chiếm, Pithagore dì tir đến cảng biển Cmtona của Hy Lạp và lập nên trương phai triết học gọi là Trường phái Pythagore. Trường phái triết học "ày dựa trẽn một sự thừa nhặn rằng số nguyên là nguồn gốc của các thuộc tính của mỗì nguỡi và các vặt chất. pylhagore và các môn smh của óng đã chứng minh định lí mang lén ông: írong mộí (am gìảc vuông, bình phương cạnh huyền băng tổng binh phương hai cạnh góc vuông. Thực ra. người Trưng Quốc. Ai Cập và người Babylon dã biết dược kẽt quả này từ rất sởm, nhưng họ cril đưa ra các trưững hợp cụ thể mã khòrg tìm thay dấu véì g' ĐỀ THI TUYỂN SINH LỚP 10 THPT CHUYÊN TP. HÀ NỘI Nãm học: 2016-2017 Thỏi gian làm bài: 150 phút (không kề thài gian phát đề) Bài I. (2,0 điểm)
  • Giải phương tr'nh X 4 -2x a nt -Ậịx 2 -x) =0. X 2 + 2v — 4x = 0
  • Giải hệ phương trình ị -ĩ A 1 4x 2 - 4xy 2 I y' 1 - 2y I 4 = D. Bài IL (2,0 điểm)
  • Cho các số thực a, b, c đò một khác nhau thỏa mán a 3 - b 3 + c 3 = 3abc và abc 5t ũ. ab 2 bc 2 ca 2 Tính p = - I - i - — — . a 2 rb 2 -c 2 b a + c 2 -a 2 c 2 + a 2 -b 2
  • Tim tất cả các cặp số lự nhiên (x; y) thỏa mãn 2.x 2 - 9y a 4- 6y + 16. Bài III. (2.0 điểm)
  • Cho các số thực dương a, b, c thủa mân a 2 + b 2 c 2 = 3. Chứng minh 2a' 2b 2 2c‘ a~b 2 b”C c-a
  • > a + b + c.
  • Cho SỂ nguyên dương n thoa mãn 2-2\Zl2n 2 +1 là số nguyên. Chửng minh 2 + 2\/l2n 2 +1 là sò chinh phương. Bài IV. (3.0 điểm) Cho tam giác nhọn ABC có AB < AC vã nội tiếp đường tròn (O). Các đưửng cao BB\ CC’ cắt nhau tại điểir H. Gọi M là trung điểm của BC. Tia MH cắt đường tròn (0) ;ại điểm 3 .
  • Chứtig mình hal tam giâc BPC và CPB' dáng dạng.
  • Cấc đương phán giác cùa các góc Bpc, CPB' lấn lượt câl AB, AC tạ các điểm E và F. Gọi o' lá tâm đường tròn ngoại tiếp tam giác AEF; K là giao điểm cửa I IM và AO'.
  • ctií/ng minh tứ giác PEKF nội tiếp.
  • Chứng mình các liếp tuyến tại E và F của đường Iròn [O ) cắt nhaL tại một điểm nằm Lrêr đường tràn (O). Bài V. (1,0 điểm) Cho 2017 số hữu ứ dương được viết trên một đường tròn. Cnứlig minh tân tạ hai sò' được viết cạnh nhau trên dường tròn sao cho khi bỏ hai sổ đó thì 2015 số còn lại khống thể chia tnành hai nhóm mà tổng các sò" ử moi n^óm bằng nhau, lưu lại chứng tò họ dã chứng minh được õinh li ấy trong trường hợp tổng quát. Theo phỏng đoán, cách chứng minh định lí trên của trường phái pythagore dựa vào việc ghép hình và lính diện tích các hình phảng. Vi phép cnứhg mình nãy dõi hòi phải biết mội 50 tính chất của các dương thẳng song song nên ngươi ta cho rang các môn sình của Pylhagộre là những người góp ohần phái triển II thuyết các đường Ihảng song song,
  • Lởi kết Nhà triết học Hy Lạp vì đại Aristữte (khoảng năm 3" 0-290 trước công nguyên) được xem là cha dẻ của phuong pháp tam đoạn luận: vả nhà toán học Hy Lạp Euclide trong tác phẩm bấl hủ Cơ sớ của mình đã dựa trẽn phép tam đoạn luận để trình bày các kết quả hình học. Các lập luận càa Ẽucllde trong Cơ sỏ dã được xem là mẫu mực của các phep chứng m hh 'rong suốt hai nghìn nãm qua. Ngày nay, một học sinh trong trưởng Trung học cơ sở có thể cnứlìg minh được nhiều kết quả toán học, nhưng loài rgưởi đã phải mất hàng nghìn nãm mãi đạt dược thảnh tựu ấy. cây 1 . la cỏ ab s 1 nên ỊÍ4 (a+b) 3 a 3 p = 1 ' 3 1 f 1 1 ì 1 b 3 ; (a +b} 4 ^2 .2 v a b ) 6 (a I b) _M n a a +b s 3(a z +b 2 ì 6(a +b) 5 l a b J (a I b } 3 (a +b) (a +b ) 5 . 3 3 6 6
  • 1 — H 7 "I 7 (a + b) 2 (a-1-b) 2 {a+b) 4 (a -b) 4 Câu 2. a) DKXD: X 4 3 > 0. 2X* 4 X + 3 - 3 x^xT 3 <=» 2x(x - \lx + 3) - Vx - 3(x — v'x 4 3) - 0 Vậy F c <=> (x- <-> X - Vx + 3 - Q 2x-Jx+ĩ-Q í-4 X = X 1. 1 + JĨ3
  • Đề giải bài toán đã cho ta cẩn xét bài toán phụ sau: Cho m Ễ ~£, rn không chia hết cho 7. Chủng rnlnh rằng m 3 chia cho 7 dư 1 hoặc 6. Thật vậy, dặt m = 7k h r (k t Z; r e {1 ; 2; 3: 4; S; 6}). Ta có m s = (7k + r) 3 = 7(49k 3 + 21k 2 r 4 Skr 2 ) 4 r 3 , vở r 3 e {1; 3: 27; 64; 125; 216}. Má 1: 3; 64 chia hết cho 7 dư 1. Và 27; 125; 216 chia cho 7 dư 6. Vậy m 3 chìa cho 7 dư 1 hoặc dư 6. Trở lại bài toán đã cho. • Trong ba số a, b. c có một số chia hết cno 7 lh' abc : 7. Do đó abc(a 3 - b 3 )(b 3 - c 3 )(c 3 - a 3 ) : 7. • Cả 3 số a, b r c đều không chia hết cho 7. Thao bái loàn phụ trẽn íhì a 3 , h 3 , c 3 chia cho 7 dư 1 hoặc dư 6. Do đó tân tại 2 trong 3 số a\ b 3 . c 3 có cùng số dư khi chia cho 7, nên (a 3 - b 3 }(b 3 - c 3 ){e 3 - a 3 } ; 7. Vậy abc(a 3 - b 3 )(b 3 - c 3 )(c 3 - a 3 } : 7 với mọi só' nguyên a r b. c. Câu 3. Gọi o là giao điểm của AC và BD, gọi M là trung điểm cừa AF, Ta có OM là đường trung bình cũa lam giác ACF 1 nên suy ra OM CF và OM =-CF. 2 _ Ta chứtìg minh được OM II FC iì SE, SM I AO nên BM II EO suy ra tứ giác BMOE là hình bình hành. KẼ BẼ 1 Xét AKCF có CF ì: BE =* 7 - - ■

    KF CF 2 Câu 4. Áo dụng bất đẳng thức AM-GM cho hai số dương ta có: 9 , 1 9 4-7- 4a ĩf

    4a

    Ị 4 n.1 11 1 4 ) 4s 4 4a 4 4 và 4b - 7- > 2 |4b -7- = 4. 4 b V b V’ a I b < 1 nén 4 > 4a I 4b vá — > 4a. b Do đỏ ta cỏ -3a 2 — < -Ạ và 4a 2 4 < ũ. 4a 4 b Cộng Iheo vế cùa 2 bất đẳng thức trên ta suy ra điỂL phải chưng mình. Câu 5. aj Gọi F lá giaD điểm của EM vã AD. Ta có AN I/ ME (do AMEN là hình binh hành) Mà AN 1 AD (g.t) nân ME AD tại F => Á?E = 90°.

    Lại có ÁBE = 90 & do vậy BEAF lá lử giác hội tiếo -í DAB = MEB. Ta có DBA - ỂBM {vì cùng phụ với ÁBC ) Vậy ABEM </> ABAD (g.g) BE BM

    =, EA.BM = BD.BE => BA.BC = 2BD.3E. AB BD
  • Gọi K là giao điểm của Bũ và AC. Ta cỏ ẤBK = ÉBC, BAK = BEC (vì củng hù vâl BAC). FỉK RA Suy ra iBAK co A3EC (g.g) =t = 7 ^-.

    BC BE ... BA BD Mã -77 = 777-7 BE BM 2BD BC nên BK = 2BD => DK = BD. Gọi I là g ao điểm của CD và AH. Vì AI íì DK nên AI DK ÍH VI IH ĩỉ DB nên 7777 - BD Cl CD'

    CỊ CD' Từ trẽn suy ra AI - IM. vậy CD đi qua trung điểm I của đường cao AH, Cảu 6. Vi mỗi ngưiởí đỂu chơi 9 trậr vâi 0 ngưàĩ khác và không có trận hỏa nên Xỵ + = Xg + y 2 = ■■■= X 1( J + y 1[} " 9- Vì trong một cuộc thi mà không có trận hòa íhi tổng số trận thẩng luôn bàng lổng số trận thua nêrt X, + xj I ... + x 10 = y 1 < y 2 I ... I y 1D . Tacó(x? I x| I I x^oMy? I yf + ■■■ Wto) = {xi 2 -y 2 ) + (x 2 -y 2 )+*-{x?& -y 2 g) = 9(X1 -yi) 4 9(x z -y 2 ) +... I 9(x-0 -y 1Đ ) = 9f(x, + x z +... + x 1ũ )-(y 1 +y 2 +y u .ì 1=0 Vậy x? + x| -‘-■4 -xĨd = yỉ + y| +...+yitf nr CUỘC THI 6IRI TOÁn DÀÍ 1 H CHD nũ sinn (Tiếp theo trang 56) Nguyễn Ngọc Huyền. 9A. THCS Hùng Ĩ0S vvơng, TX. Phú Thọ, Phú Thọ; Tạ Xét AACE và ABDE có CAE -DBE (cùng chắn cung CD) t ẢEC = BEÓ (đối đinh) Suy ra AACE co aBDE (g.g) -■ậl'. (3) Bũ BE Rn AF Từ (1), (2) Và (3) suy ra -- = 3.. AD BE Cách khác: Đưởng tròn (M; MA) cắt DA tại Avà F. Chứng minh AACE co ABDE thì lừ (1) suy ra (3). Nhận xét. Cảc bạr có lời giải đũng: ĩrẩn Thị Thu Huyền, 9A3, THCS Lâm Thao, Lâm Thao; Thúy Tiên , 9A4 r THCS Yên Lạc, Yên Lạc, Vĩnh Phúc. Các bạn được thưởng ki này: Nguyễn Thúy Dương, Bùi Thị Quỳnh, Nguyễn Thu Hiên, 8A3, THCS Lârn Thao, Lâm Thao, Phú Thọ; Chu Thị Thanh, 6E* t Bạch Bùi Nguyệt Anh, 6D, THCS Vĩnh Tưởng, Vĩnh Tuủng, Vĩnh Phúc. NGUYỄN HIỆP CÁC BẠN ĐƯỢC THƯỞNG Từ trái sang phải: Nguyễn Thùy Dương, Bùi Thị Quỹm. Nguyễn Thu Hiền, Cnu Thi Thanh, Bạch Bùi Nguyệt Anh. 39 J Giải toán qua thu Bãi 1(158). Tìm tất cả các Sũ nguyên dương X, y thoa mãn (x - y) 4 = 40X + 41. Lời giải. Do x r y là số nguyên dương nên4Qx<4lx; 41 < 41 y f khi đó ta có (x + y) 4 = 40x + 41 <41x + 4 1 y = 41(x + y). Suy ra (x + y) 4 < 41 (x + y) « (x + y) a < 41 < 64 = 4 3 => X4 y <A . (1) Ta thấy X lá số nguyên dương nên 40x + 41 >40.1 +41 - 81 = (X + y) 4 > 81 =*x + y>3.<2) Từ (1) và (2) suy ra 3 É X I y < 4. Mà (x + y í w) => X + y - 3. Suy r a (x ; y) - (1 ; 2); (2 ; 1) (do X. y là số nguyê" dương). Thừ lại chì có X = 1 ; y = 2 Ihỏa mãn. Vậy X = 1 ; y = 2. Nhận xét. Háu hết các bạ" tham gia giãi oà đểu làm đúng, tuy nhiên một số bạn còn biấn đổi dà dòng mãi đi đến kết quả. Tòa soạn xìn kề tên các bạn có lài giải tổl: Hoàng Thị Phương Anh , 7C, THC5 Nhữ Bá sỹ, thị trấn Bút Sơn, Hoang Hỏa, Thanh Hóa; Trần Dinh Hoàng', Bùi Hồng Quên, Nguyền cẩm Vĩ, 6C, THCS Hoàng Xuân Mãn, Đức Thọ, Hà Tình. Lê Minh Việt Anh, 7A, THCS Vĩnh Yên, TP. Vĩnh Yẽn; Lé Thị Thanh Hương, 7D, THCS Vĩnh Tưởng, Vĩnh Tưởng, Vinh Phúc; ĩrắrt Đức An, Lê Thùy Linh, Lê Văn Quang Trung, 6B, THCS Lý Nhậl Qjahg, ũô Lương, Nghệ An: Trần Quang Tai, 7A^ 1 THCS Yẻn Phong, YẾn Phong, Bắc Ninh. PHÙNG KIM D JNG Bãi 2(158). Cho tam giảc ABC vãi 1 . ' , AC = 2AB. Đưàng thẳng CỊU3 A vuông góc với AC cắt đitóng Imng trực của BC tại o. Chứng minh rằng OBC lả tam giác đều. Lãi giải. Dựng tam giác đểu ABD sao cho D và c cùng thuộc nửa mặt phẳng có bờ là đường thẳng AB. Tam giảcADC có DAC = 60 c ; AD = AB = -AC nên AADC vuông tại D. Từ đó 3DC = ÃDC +ẤÕB =90 ° -60 ũ =1 50 D => ÕẢB = 360° -ÕẢC -BÁC = 360° -90° -120° =150° -i ỎÂB = ÓAD( = 150°). Do đó AOAB = AQAD (c.g.c) Suy ra OB = OD = oc. (1) Ta lại có BOC - BOD +DOC
  • (1BŨ° -2BDO) +(180° -2CDO) = 360° -2BDC =360° -2.150° =60°. (2) Từ (1) vã (2) suy ra AQBC là lam giác đều. Nhặn xét. Các bạn saư có lởi giải tốt: Lẻ Minh Việt Anh, 7A, THCS Vĩnh Yên, TP. Vĩnn Yên, Vĩnh Phúc: Nguyên Thị Mai Linh. Nguyên Trọng Thuận, Nguyễn Hĩru Quyên, 7C, THC5 Nhứ Bá Sỹ r Hoang Hóa, Thanh Hóa: - ập thể lởp 7D, THCS Xuân Diệu, Can Lộc, Hả Tính. HỒ QUANG VINH Bải 3(158). Cho phương trinh 4x 2 - 4mx + 4m - 5 = 0 (n lá tham sổ). Tim m để phương trình có mội nghiệm ầm, nghiệm còn lại lớn hơn 1 nhưng nhỏ hơn già trị tuyệt đối của nghiệm ảm. Lài giảỉ. Ta có A* = 4m 2 - 16m + 20 = 4(m - 2) z - 4 > 0. Do đỏ phương trình luôn có hai nghiệm phân biệt m-Vm 2 -4m + 5 n + Vn 2 -4m +5 *1 S

    với mọi m. Ta phải tìrr m để có X, < 0; x z > 1; X 1 4 x 2 < 0. (vì điều kiện x 2 < |x. 1 1 > X 1 + x 2 < Q). Ta có X*! 4 x 2 - m < 0.

    ■,. . TI- -y/nn 2 -4m + 5 . Với nr < 0 suy ra Xi = < 0 . Ta lại có m <■ Vm 2 -4m ‘ 5 , x 2 = — — -> 1 2 t ■:> Vm 2 - 4m + 5 > 2 - m > 0 <-4 m 2 -4m + 5 > m 2 -4m + 4 (luôn dùng) Suy ra m < ũ thỏa mân tất cả các yèu cẩu của bài toán. Vậy m < 0. Nhặn xét. Đày là bái Toán không khỏ vể phương trình bậc nai có tham số nèn hẩu hết các bạn tham gia gửi bái làm đúng vá cỏ cách giắt: như Lẻn. Các bạn sau có bài giải tốt: Lẽ Việi Hùng, 9/1, THCS Nguyễn An Ninh. TP. Vũng Tàu, Bà Rịa - Vũng Tàu: Đặng Quang Anh, 9A, THCS Nguyễn Chích, Đỏng Sơn. Thanh Hóa: Lẽ Đức Thải, 8A2, THCS Yén Lạc, Yên Lạc, Vĩnh Phúc; Ngưyễn Văn Cường, BA, THCS Hợp Tiến, Nam sách, Hàì Dương; Dương Vãn Mình, 9A, THCS Đặng Thai Mai, ĨP. Vinh, Nghệ An: Trần Hổng Quý, Chu Vãn Việt, Chu Thị Thanh, 0E1, THCS Vinh Tưởng, Vĩnh Tường, Vĩnh Phức: Đặng Minh Hoàng, 9A, THCS Nguyên Hiến, Nam Trực, Nam Định. NGUYỄN ANH DŨNG Bái 4(158). Cho các sô' thực dương a, b, c thỏa mãn a - b - c = 3. Chứng minh rãng Lời giải. Bầt đảng thức cần chứng minh tương đương với a - . a 2 (b li) |b bVi-D | e cVn) a + b + àb b4C-t-hc c + a + ca ab bc ca <1. a + b 4- ab b 4 c 4 bc c 4 a 4 ca Mật khác, áp dụng bất đẳng thức AM-GM ta có âb ab
  • — < 3 a ■ b i ab Tương tự ca 3i/ãv bc — < b 4 c + bc , c+a -1 ậ'ai> 1 1 a+b +1_a +b+1 3 b + C + 1 và Do đò ab bc ca . — 1 7 — - — + a + b- 1 -ab b 4 c T bc c + a + ca a 4 b 4 1 b - c 4 1 c + a-1 < — -4 + 4

    9 9 9

    2(a — b + c) 4 3 I = 9 = Đẳng thức xảy ra khi a = b = c = 1 . Nhận xét. Có rất nhiều bạn tham gia giải bài và có lời giải đúng, một số bạn biến đoi dál mói đi đến kểL quả. Các bạn sau đay có lời giải đủng và ngắn gọn: Nguyễn Như Cường, 8A, THC5 Bạch Liêu, Yén Thảnh, Nghệ An; Nguyẳn Thủy Dương, Nguyễn Thu Hiền , Bùỉ Trọng Vịnh, 8A3, THCS Lâm Thao. Lâm Thao; Nguyễn Minh Anh. 9A Z . THCS Giấy Phong Chàu, Phù Ninh, Phũ Thọ; Trần Sỹ Hoàng, 8C, THCS Hoàng Xuàn Hân, Đức Thọ, Hà Tĩnh; Tạ Thủy Tiên, 9A4, Tạ Kim Thanh Hiển, 7A4. THCS Yên Lạc, Ýẽn Lạc. vĩnh Phúc; Bùi Xuân Dưõng, 8A1, THCS Yên Phong, Yên Phong, Bắc Ninh; Hoàng Thê Sơn, 9A1, THCS Hồng Bàng. Hồng Bàng, Hải Phỏng; Nguyễn Vãn Cưỡng, 8 A, THCS Hợd Tiến, Nam Sách, Hải Dường; Võ Nguyễn tìan Phương, 9A4, iHCS Phù Mỹ, Phù Mỹ, Binh Dịnh: Lê Việt Hùng, 9/1, THCS Nguyễn An Ninh, Vũng Táu, Sà Rịa - Vũng Táu; Tạ Lê Ngọc Sàng, 9A, THPT chuyên Hà Nội Amsterdam, cấu Giấy, Hà Nội. CAO VẢN Dũng Bài 5(158). Có 102 diẻn viên nam và nữ xếp thành vòng tròr, mủa xòe. Cử 2 người kể nhau :hi nắm tay nhau. Hỏi số cái nắm tay của hai người cùng giới và số cái nắm tay của hai người khấc giái Cu thể bâng nhau hay không? Vì sao? Lời giải. Ta đánh số 102 ngườ theo vòng tròn tương ứng lả 102 số; x r x 2 , x 3 X 1Ữ1 vả x 102 . □iẽn viên nam □‘ánh số là 4l . dièn Vién nữ đánh sõ là-— 1. Kh đò Xj (i = 1, 2, 3, 102) chỉ nhận giã trị -t-1 hoặc -1; cặp số Xj , Xj 1 Piểu thị hai điẻn viên nắm tay nhau và tích X|X, t nhận glá trì 4l nếu hai diễn viên cùng giỏi vá bang -1 nếu hai diễn viên khác giới. Ta có (x 1 x 2 ).( X 2 X 3 >.(5(3X4 ) ... (x ipi x 1ũ2 ).(x 102 x r ) - 1 nên số càc tích (XịXị! bầng —1 là số cnấh. Glả sử sõí cái nẳm tay của hai người cùng giỗi và sô cái nếm tay của hai ngươi khác giãi bằng nhau. Khi đô x*x 2 + + X3X4 I ... + x 101 x 1ũ2 + = 0. Do đó có 51 tích X ( X| 1 1 bằng 41 và có 51 tích XX| +1

    bằng -1. Suy ra ổiểu giả sử lá sai. Vậy sõ cái nắm Tay cùa hai ngươi cùng giở và so c + a + ca 9 cái nắm tay của hai người khác giới không thể bằng nhau. Nhặn xét. Đày lã bài tũản lương đối khó. Các bạn sau có lởi giải đúng: Đặng Quang Anh. 9A, THCS Nguyến Chích, Đòng Sơn, Thanh Hóa; LỂ Việt Hưng, 9.1 , THCS Nguyền An Ninh, TP. Vũng Tàu, Bà Rịa - Vũng Tàu; Đặng Minh Hoàng, 9A, THCS Nguyền Hiền, Nam TrựG, Nam Đình. TRỊNH HOÀI DƯƠNG Bải 6(153). Cho hai đường tron (O) và (O') tiếp xúc ngoài tại T. Một đường thẳng tiếp xúc vâi (O') tại D và cắt (O) tại A và B (A nằm giữa 8 và □), gọ c lả điểm thuộc Cung BT không chứa A của (O). Vẽ tiếp tuyến CE của (0 ) (E là liếp điểm). Chứng minh râng giao đ.ểm thứ hai của ĐE vãi dương tróh ngcại tiếp /\CTF lá tám dường hỏn bàng tiếp trong gốc ABC của aABC. Lởi giải. Ta cấn có bổ để. • Bổ đế. Cho tam giác ABC nội liếp dường trùn (0). M là Irụng diểm cung AB chứa c của đường tròn (O). Nêu điểm K thuộc tia MC và thuộc nửa mặt phàng bờ AC không chừa B sao cho MK = MA thì K là tám đưàng tràn bàng tiếp đối diện đinh B của tam giác ABC. Pháp chửhg minh bổ dề trẽn đơn giàn, dựa váo KCÀ = MBA =MÀB =MCB và kÕA +CKA -NĨẦB +MÃK. Trờ lại giải hài toán, Gọi M la giao điểm thứ hal của DT và (O). TừAB tiếp xúc vài dương iròn (O’), sjy ra Mo II Do vá 00 I AB nên Mũ I AB. Do đó M là trung điểm cung AB chứa c cùa đường trôn (O). Xét c thuộc cung MT không chứa A. Gọi K lá giao điểm của tia MC vá tia DE. Vì các lam giác OTM, 0'TD theo thứ tự cân tại 0, O' và tử giác ATCM nội liếp nên IP ĐƯỢC THƯỜNG Kì NÀY Đặng Quang Anh, 9A THCS Nguyễn Chích, Dâng Sơn, Thanh Hóa: Trẩtĩ Sỹ Hoàng, ac, THCS Hoàng Xuãn Hãn. Đức Thọ, Hà Tĩnh; Lê Minh Việt Anh, 7A, THCS Vĩnh Yên, TP. Vinh Yén, Vĩnh Phúc; Nguyễn Như Cưởng, 8A, THCS Bạch Liêu, Yên Thành, Nghệ An; Lê Việt Hùng, 9/1, THCS Nguyễn An Ninh.TP. Vũng Tàu, Bả Rịa - Vũng Tàu; Nguyễn Vàn Ctlởng. BA, THCS Hợp TÌỂn. Nam Sách, Hàl kÕT - MẤT - -Ị-MOT - 2 J80°-2DT01 ũBt dÌt kỀt 2 2 Từ (1) suy ra tứ giác TCEK nội tiếp. <ếthợp vói CE tiếp xúc với đương tròn (0 ), suy ra MKT -CKT-CÈT-ÍDE - MDK. Do đố các lam giác MCT, MTK đóng dạng. Vậy MK 2 = MT.MD. (2) Từ (1), chủ ỷ rằng AD tiếp xúc với (0’), suy ra DỀT-TDA- MDA. Do đó các lam giác MAT, MDA đổng dạng. Vậy MA 2 = MT.MD. (3) Từ (2) và (3), suy ra MK - MA. Từ đó, chú ý rằng K thuộc tia MC và thuộc nửa mặt phẫrg bở AC không chửa B, theo bổ đẽ trên, suy ra K lã tám dương tròn bàng tiếp trong góc ÀBC của lam giác ABC. Nhận xét. Chứng minh tương tự khi điển’ c thuộc cung BM hoặc c trùng với M. Bái toán này khó, chì bạn Lỗ Việt Hùng, 9/1 , THCS Nguyễn An N nh, TP. Vũng Tàu, Bà Rịa-Vũng Tàu có lời giải tốt. NGUYỄN MINH HÀ Thi giải toán qua thư Dương; Đặng Minh Hoàng, 9A THCS Nguyễn Hiển, Nam Trục, Nam Dịnh; Nguyễn Minh Anh, 9A4, THCS Giấy Phong Cháu, Phù Ninh, Phủ Thọ; Bùi Xuân Duông, BAI, THCS v Ến Phong, Yên rJ hong, Bắc Nính; Hoàng Thế Sơn, 9A1, THCS Hổng Bàng, Hổng Bàng. Hải Phòng; l/õ Nguyễn Dan Phương, 9A4, THCS Phù Mỹ, Phù My, Bình Định; Tạ Lẽ Ngọc Sáng, 9A, THPT chuyên Hà Nội Amsterdam, cầu Giẫy, Hà Nội. OEm SỐ ĐIẾM Dược TÔ MÀU? Bải toán. Trong mặt phảng lấy 2016 điểm nào đó A v A 2 , Aj r .... A 2ũ1 g. Người la lõ đà tất cả các trung điểm của các đaạn thẳng nôi các cảp điểm trong 50 2016 điếm này. Gọi T là số các đ ểm được tò đỏ. Tìm giá trị lớn nhất, giả trị nhỏ nhất của T. NGUYỄN ĐỨC TẤN (TP. Hồ chỉ Minh) KHÔNG GIẢI PHƯƠNG TRÌNH (TTT2 sổ 158) Ta có hai phương trình: X 2 + 2015X- 2016 -0.(1) y 2 + 2015y- 2016 = 0. (2) Trừ theo từng vế của (1) cho (2) ta được X 2 -y 2 + 20i5x-2015y = 0 <=> (x - y)(x + y) + 201 5(x - y) = 0 í=> (x-y}(x + y + 2015) -0. Theo giả thiết X > y nên X + y + 2015 = 0 <=í X + y = -2015. (3) Thay (3) vào (1) ta được X 2 - (X -y}iỉ = 2016 o -xy = 2016. Ta có (X - y} 2 - (X + y ) 2 - 4xy - 2015 2 + 4.2016 - 20 1 5 2 -r 2.2.201 5 + 4 = (201 5 + 2) 2 = 20 1 7 2 . Mà X > y nèn X - y = 201 7. Mhận xẽt. NỂU sửdụng định li Vì-él dào thì La thấy ngay hai nghiệm X, y khác nhau cCa phương trình (1) thỏa mãn X + y — 2015 và -xy - 2016. tuy nhiên anh Compa yêu cấu khỗng giải hai phương trinh (1), [2) cùng có ngh"a là không dùng ôịnh II trẽn. p . Các bạn có lời giải dũng là: Bùi Thị

    L Quỳnh, Nguyễn Thu Hiền, Nguyên Thủy Dương, BA3, THC5 Lãm Thao. Làm Thao, Phú Thọ: chu Vàn Việt, 8E1; Lé Ánh Tuyết. 7E1, THCS Vỉnh Tưởng, Vnh Tưởng; Lê Đức Thài, 8A2, THCS Yèn Lạc, YÊn Lạc, Vĩnh Phúc; Trần Quang Tài, 7A1, THCS Yân Phang, Yên Phong, Bắc Ninh; Phan Thế Anh ẫ 7A2, THCS Trân Phú, TP Phủ Lý, Hà Nam; Dặng Minh Hoàng, 9A r THCS Nguyền Hiền, Nam Trực, Nam Định: Hổ Gia Bão. 9A6, THCS Thốt Nốt, Q. Thốt Not, TP. Cẩn Thơ. ANH COMPA Từ só' tháng 9 nãm 2015, Córg ty cổ phần Dịch vụ Giáo dục Việt Nam sẽ :ặng các khóa học trực tuyến trên websit@: hocmsLvn cho các bạn học sình được thưởng trong các chuyên mục và các bạn học sinh được khen trong chuyên mục Kểl quà thì giải toãr qua thư. Các bạn học sinh sau khì ihặn được mã cung cấp thì đăng ki tại địa chỉ: thcs.h ocma i. vn/to an tuo itha (Xin liên hệ SĐT 0966464644 õể dược giải đáp). CHIEC NHAN trong túi NGUYÊN THỊ LAN (9A2, THCS Yên Phong ị Yên Phong, Bẳc Ninh) S ảng chủ nhật, sau bữa sáng, vợ thám tử Sêlôccôc nói với chổng:
  • Lát nữa bà Mai bạn em đen gặp anh một chút. Bà â'y gọi hỏi em từ chiều qua nhưng bây giờ em mởi nói với anh vì muốn đêm qua anh ngủ ngon. Thám tử cười:
  • Chắc bà Mai lại gặp chuyện gi à? Không sao, anh sẽ hết lòng vì bạn của em. Một lúc sau T bà Mai tới. Thám tử von vã mỏi vào nhà và tươi cười trò chuyện để cố xua tan vẻ lo lắng trên gương mặt bà Mai. Roi bà Mai bắt dấu câu chuyện: -Tôi mới mua chiếc nhẫn kim cương, định để tặng con gái nhân sinh nhật sắp tới. Nào ngờ, vừa mang vể nhà chưa bao láu thì đã bị mất rỗi.
  • Bá cứ bình tình kể lại mọì chuyện cho tòi. Càng chi tiết càng tốt.
  • Tối hâm kia tôi dí mua chiếc nhẫn. Lúc vể tởi nhà thì đã hơn 9 giờ rái. Tôi để tú ì xảch của mình trẻn chiếc bàn trong phòng ngủ rổi đi tắm. Sau đó, tói vừa nằm vừa xem TV rồi thiếp đì, chầng nhớ gì đến việc cất nhẫn nữa.
  • Bà phát hiện nhẫn bị mất khì nào?
  • Chiều qua. Ngay lúc đó tõì đã gọi cho ba xã ông đấy. -Tức là sảng hòm qua, nhẫn vẵn còn? -Tôi khống biết nữa. Chỉ biết là khoảng 2 giờ chiều, toi sực nhồ ra là chưa cất nhẫn vào két nên đi mở túi xách và phát hiện bỉ mất. -Từ sáng cho tâi lúc đó, bả để chiếc túi ở nhà ả? -Vâng. Th' tôi ở nhà suốt, có đi đàu đâu mà mang túi.
  • Thế bả làm gì trong sáng hôm qua?
  • Thì cứ loanh quanh hết trong nhà rồi lạl ra vườn ngắm rau, ngắm hoa...
  • Có những ai ỏ nhà bá trong sáng hôm qua? Mi rJTfrfĩĩ;.ĩfll ỉblErh 1
  • Như mọi ngây, có bà giúp việc, con trai tỏi và con gái bà giúp việc mới tới chơi nữa.
  • Bây giở bà gọi cho từng người để tôi hỏi qua điện thoại cúng được. Nếu cẩn thì ta si gặp trực tiếp sau. Mà bà đi nói vãi ai về việc này chưa?
  • Chưa. Cả lúc mua lẫn lúc mất tôi đều chưa nói gì. Dầu liên, thám tử hỏi cậu Minh, con Irai bà Mai xem cậu đã làm gì r cr đâu trong thòi gian từ sáng tới 2 giờ chiều hòm qua.
  • Cháu ngủ SUỐI ạ. Gẩn trưa cháu mới dậy.
  • Ngày nào cháu cũng dậy muôn thể ư?
  • Không, thỉnh Ihoảng thôi ạ. Đém hôm trước cháu thức khuya xem phim trên mạng...
  • Sau khi dậy, cháu lãm gl?
  • Cháu ra vươn chạy vài vòng, vào tắm rồi ằn cdm trưa luôn ạ. Hóm qua bác giÚD việc nấu ngon quá nên cháu chén rất khỏe... Tiếp theo là bá Vãn giúp việc:
  • Sảng qua lõi vẵn làm việc như mọi ngày. Dọn dẹp, đi chợ, cơm nước.
  • Bà đã nấu món gì trưa qua?
  • Món cá ông ạ. Cậu Minh chỉ thích ãn cá sông nên lúc đi chd, gập mở cả sông tươi, tôi mua rõ nhiều , về chế biến vài mon. Nhân thể con gái tôi đang lên chơi cũng rất thích cá sông.
  • Ồ, nghe bà kể tôi cũng thấy thèm rổi. Mà bà mua cá gì vậy?
  • Cả nục ông ạ. Cuối cùng là cô Hoa, con gái bà Mai:
  • Cháu đã làm gì từ sáng tâỉ trưa hôm qua?
  • Cháu giúp mẹ dọn dẹp nhà cửa một lúc, khoảng lũ giở thì cháu di siêu thị mua sẩm vãi thứ linh tinh. Sau đó, thám tử nói với bà Mai: -Tôi bẩt đấu nghi một người rổì. Bày già ta cùng tới nhà bả để tõi fm hiểu thêm rồi mớ kết luận được. Bà Mai rất ngạc nhiên, không hiểu sao thán 1 tử có thể tìm ra manh mối nhanh thể. Các thám tử Tuổi Hổng hãy giải thích cho bà hiểu: Thảm tử đã nghi ngà ai vá căn cừ váo đâu mà ông Ịạì nghi người ổó? ™ MÓN QUÀ BIẾN MẤT (TTT2 số 158) Tất cả các bạn gửi bài kì này đều phá: hiện điểm vô lí trong 2 câu mà ông Phong nói với thám tử: “24 sô' tạp chí của một năm đuợc đóng thành một tập dảy"; "Trong dó có rất nhiều baì toán hay dành cho học sinh từ lỏp một đến lóp chín''. Không những thế, các thám từTuổi Hống còn nêu rỗ: Tổng tập Toán Tuổi thơ bao gổm 12 số tạp chí phát hành trong 1 năm, được đỏng tập thành một cuốn, Tổng tập TTT có 2 loại, một dành cho học sinh Tiểu học và một dành cho học sinh THCS. £ Phần thưởng được gửi tới: Nguyẻn Quang Thành, 6A, THCS Thị tràn Cao Thượng, Tân Yên, Bắc Giang; Trần Hải Nam, 7A3, THCS Lâm Thao, Lâm Thao, Phú Thọ; Phan Thế Anh, 7A2, THCS Trần Phủ, TP. Phủ Lý, Hà Nam; Vũ Thủy Dương, 7B, THCS Hoằng Trung, Hoằng Hóa. Thanh Hóa; Trần Đạt Vỹ, 6/3, THCS Phạm Hồng Thải, TP. Pleiku, Gia Lai. Thám ỉử Sèlồccóc Bài 68: tsj|*] Hả Nội có rất nhiều viện bảo tàng Ths, NGUYÊN vũ LOAN LTS. Nếu biết tiếng Mán bạn sẽ:
  • Hiều các lừ Hãn Vìệl, sử dụng ỉỏl hơn tiếng Việt cùa mình. Trong kho từ vựng tẻng Việt rất nhiều từ Hán Việt.
  • Đọc được sách cả r ván bia bằng chữ Hán vá Hán Nôm. thêm hều văn chưong. lịch sừ nước Nam rrinh.
  • H ẻL ngôn ngử mà cứ 5 ngưò i trẽn thẻ giớ có hơn 1 người □ ủng. Dễ dáng hợp tác. làm ãn vớícác rước vả vùng lãnh thè Trung Quốc, Hong Kõhg, Đài Loan, Srgapore vả că Nhật Bàn, Hãn Quóc. Nếu biết cả tiếng Anh và tiếng Hán thì thật là tuyệt. Từ múi. chéngshl: [thành thì] thành pho ilỉ 4 nảmc: [nả mã] nhu vậy, như thề /hánlăn: [tri én lãm] triền lãm, trưng bảy ị5||Ịj J ịỆỊ,$}ì$ Hénèi bówùguàn: Viện báo làng Hà Nội (SU hủ: [hồ] hè, hồ mróc Mẫu tàu. lễSpiányi: [tiện nghi] rè -$đuũ: [tỉa' nhiều ■ỊỆíbí^bỏuủguitn: [bác vật quán] viện bâu tảng ĩỉj -í lí lls hỉ bó wùguăn : V iệr. bào làng Lịch sừ s i'i hưátt j iãn hú : [hu ân ki Ẻ m hồ J 1 1 ồ Gưtrm 1 . A: ik ii iĩ? 1*1 ? (Nĩqù gu 0 H énèi ma?) Bạn đã từng & Ến H j N ộ] chưa I 13 : ậẾ ■£ ìỉ '|T 1*] . (W 5 q ừguó Hénèi.) Minh tùng đến H à Nội rôi A:ỉiỉrt : (HẻnẻidAma'?) HiiNỹi có to không? H; i5) $ íẫẨ , íítw -5) F*Ị SỊỈ Ấ A. (HẺnừi hẽn dồ, wỏmen dc ehiíngshi mẺiyủu hỏnèi nà nu- dà.) Hà Nội rất to. thinh phá cùa chúng minh không to nhu Há NỘI. A: íBỊfà ỊtẸ-® Ị2 7 I ] lénèi yõu bóvvùguăn ha?j 1 1 à Nội cỏ viện bán tàng chú? R: jS] 1*1 £ = + % 'Mặ íís tÈ . ÍÈ È ít õi £ 1 $ $Ị ÍẼ . c [ lủnủí yõư sãnshi duõ jiè húvvÙỊiuãn. w6 qùguù llshĩ bỏvvủguán .) Hù Nội cỏ hơn 3 0 viện bảo táng, minh lủng đốn viện bào tàng T Ịch sử, A: 1^'ltì.íE'È Í5JI*I , (Wà yèxiảngqù [ìẺuèi,} Mình cungmuóađểnHàNỘL 2- »ỄWl*l, ÀÍ8£ mm. ẽffniir+6. Ị® snRK «[. (Zhc shi Hcnci, Hcnà bcndà, rcnhcnduõ. Wõehũshcng dc chcngshì mciyóu Hdncinảmcđà. Hònciyóuyĩgc hon dà de hù. zhègehú ĩiãohiián ĩiãiỉhủ, zái ] ĩénèi shì zhõngxĩn. Ilénẻĩ hái ybu hỄnduõbóuủguăn, bótvủguăn dẹzhânlăn hc-n yứuyisi- Wõ xìhuãn pn l>ãwùguân.) Dáy tú Hà Nội, Hà Nội rât to, người rát đông, Thành phò nơi tủi sinh ra khống lo như Hà Nội. Hà Nội có rât nhiêu kõ. dày lá hồ Gươm, năm ớ trung tâm thảnh pho. Hà Nội còn có rất nhiêu viện bão tàng, các triên làm trong bão tàng rất cô ý nghĩa Tôi thích dj viện bão tâng, Thay the theuinẫii'
  • Wế tó 1 mĩm 2> Mrt w =+ mặn 5+ mnm 0+ ỈẨ^H&ĩtr — +
  • t-m ± » ầ /Jv M írks Àm ĨẼ Lís (Kì yau dũng tiếp) UNIX 20. GAS LAW AND PARTICLES 0F MATTER THEORY SECTION BÍNH NAM HÀ Question 1. A fixed mass of 3 Ìr occupìes 9.0 litres of air at a temperature of 300 K and a pressure of 1.2 atmospheres. The volume is reduced to 5.0 litres by increasing the pressure lo 2.3 atmospheres.
  • Assumìng Lhaỉ the air behaves as an ìdeal gas f calcưlate the temperature of the air after the reduction in volunne.
  • Give one reason why the actual temperature may be different from what you have calculated. Question 2. Explain brìetly, using the molecular theory, why:
  • liquids evaporate more quickly in ã draught.
  • rapid evaporatìon from a liqưid cools the liquid đown. Physics terms draught rapid escape retumìng carrìed íixed mass occupies (emperaỉure pressure behaves gió lùa nhanh thoát, thoảt ra quay trỏ lại được mang đì khối lượng cố đính chiếm nhiệt độ ảp suất cư xử, thể hiện Practice. Các từ vụtig dã cung cấp t r Ến sẽ giúp bạn dịch dược 2 báì thuộc phần lí thuyết vể gas law. DỊch được bạn sẽ học thêm dược các khái niệm mỏi vả ứng ciụng. Bạn sẽ hiểu hơn tại sao quần áo phđi gần cửa sò’ lại chóng khô hơn trong phóng không có cửa. Bạn hãy dịch vã gửi bài dịch về tòa soạn. Bài dịch lốt được chọn đãng và có phần Ihưỏng. SUY NGHĨ ĐÊ MỞ RỘNG MỖI BÀI TOÁN PHAN DUY NGHĨA (Phòng G ĩào dục tiểu học, sờ Giáo dục và Đáo tạo Hà Tinh) K hi học toàn nếu chững ta giải toán mà chỉ cán tim ra đáp số, tìm ra cách giải và dừng tại ỏ đò thi chưa thật hiệu quả. cùn sau khi tim cĩược đép sổ. tỉm ra cách giải bâị toàn rối chứng ta lại tiếp tục suy nghĩ trên mẫi bài toán đõ, tìm thêm cách giải, khai thác thêm những ỳ mời của bài toán thi đù là con (tưòng tổt dề khăm phã môn toán. Sau dãy chúng ta dua ra mật vỉ dụ. Bài toán 1. Cha lam giác ABC. Gọi I lá một đ Ểrr nằm trong tam giác. Từ I lãn lượt kẻ các đưàng song song vâi các cạnh BC. AB. AC. Các đướng thẳng đó thú tự cắt các cạnh BA f AC. BC tại E, F,
  • Nổi GE f BF, AK r chủng cắl nhau tại các điểm M. N, p (như hỉnh vẽ). Chửng mình rằng S MNP = S NE p + S MAF + Sp CK . A Lòi giải. Nối AI. BI, Cl. Ta CÓ S| BC = S EBC (vĩ chung dãy BC và IE // BC nén chiều caa hạ từ I vả E xuống đáy BC bằng nhaL). (1) Tương tự S| A3 = S FAB ( 2 ): S |AC = S KAC . (3) Cộng theo vé của (1), (2), (3) ta có S IBC + S IAB S IAC = S EBC + S FAB + S KAC' Stiy ra S ABC S gsc + S FA3 + S KAC S MNP = S NEB + S MAF + S PCK' Nhận xét. Nếu thay tam giác ABC bởi hĩnh chữ nhật ABCD thi ta QÓ bài toán mới. Bài toán 2. cho hình chữ nhặl ARCD. Gọi I lá điểm nằm trong hình chữ nhật. Qua I kẻ các đường song song với các cạnh AB BC lẩn lượt cắt các cạnhCB, BA,AD, DC tại các điểm M, N, p, Q. Nối ŨM t CN. BP vá AQ chúng cắl nhau lạ các diểrti E, G, H, K (nhưhlnh vè). Chứng mirh rằng S EGHií = S ECM + S GNB + S HAP + S KQD' Lòi giải. Nổi IA, IB. IC và ID. Ta có S| A3 = Sp AB (vì chung đáyAB và IP//AB 00" chiều cao hạ từ I và p xuống AB bằng nhau). (1) Tươrg lự S IAD = S OAD S IDC ” S MDC S IBC ” S NBC' Cộng theo vê của (1), (2). [3) vả (4j ta có S IA6 + S IAD + S IDC + S IBC ~ SpAB + Sqaũ + ®MDC - NBC' Suy ra S ABcD = Sp AB + S QA p + S MDC + S NBc . Do đó S EGHK - S ECM + S GKQ + S HA p T S KŨD . Các bạn hãy thay hình chữ nhật ABCD bởi hlnh thang ABCD hoặc tử giác ABCD và chửng mình kết quả tương tự nhé. ĐỊNH LI STEWART VÀ ỨNG DỤNG THẢI NHẶT PHƯỢNG (GV THCS Nguyễn Vằn Trỗi, Cam Nghĩa, Cam Ranh Khánh Hòa) ste wart (1717- 1785) ià nhà toán học vả thiên vãn học người Scotlanờ. Bài viết này sẽ tìm hiểu định tỉ stewart, các hệ quả và ứng dụng của định ỉỉ này.
  • Định lí Sỉevvart Cho tam giác ABC. Gọi D là mộl điểm tùy ỷ trên cạnh BC thì ta có hệ thức AB Z .CD I AC 2 .BD - AD 2 .BC = BC.CD.BD. 0) A B H D c Chứng minh
  • Nếu D = B hoặc D = c thì ta cố đpcm. • Nếu D nằm giữa B và c. Kẻ AH ± BC. G ả sử H nằm g ữa B và D (chứng minh tương tự Khi I- nằm ở vị trí khác). Ta cá AB 2 .CD - AC 2 .BD - AD Z .BC = (AH Z + BH Z )CD t (AH Z + CH Z )BD - ÍAH 2 - DH 2 )BC = BH Z .CD t CH Z .BD I AH Z (CD t BD - BC) - DH Z .BC = (BD - DH) Z .CD I (CŨ ị DH) 2 .BD - DH Z .BC = BD Z .CD + CD Z .BD + DH Z (CD 4 BD - BC)
  • BD.GD.(BD 4 cò) - BD.CD.BC.
  • Các hệ quả Đặt a = BC, b = CA, c = AB.
  • Nếu AD = m lã đường trung tuyến của AABC thì BD =DC=. 2 Hệ thức (1) trử thành _2 . ,2 ,,„2 a „2 -c a 2 2 4
  • Nếu AD = d a là đường phân giác cùa AABC thỉ BD CO a c b c 4b Suy ra BD - CD - b + c h 4C Hệ thửc (1) trở thành bc 2 fa z c ,2 a 2 bc . ,2 a z bc ■ + — — -di = „ , bc-dg = b-c b+c <=> dị - bc (bic) \2 1 - b -c (b + c) z bc(b 4 c + a)fb + c-a) (b + c) 2 '
  • Nếu AD s h s lá chiếu cao của AABC thì (1) trở thành c 2 1 /b z ~-h 2 +b 2 a-/b 2 -h z -ah z *=. a/b 2 - hẩ (a - i/b 2 -hẩ ) «-> 2a Jb 2 -n 2 =ạ a I h 3 -c 2 hay 2a.CH =a z +b 2 -c z (2) 4=>b 2 -tiị ( a c \ 4 a 2 z (2ab) z -(a 2 -b 2 -c 2 ) 2 ” "a - 2 4a <=> h 2 - (a - b + c)(a + b-c){D +c -a)(c +a -b) . (4a 2 ). (3) 4 . Đặt p = (a b i c) : 2 thì a I b — c = 2(p - c): b
  • c - a = 2(p - a); c 4 a - b = 2(p - b).

    Từ (3) suy ra a 2 hẩ :4 = Ptp -a)(p-b)[p-ũ)

    s = ah s = V°tP - a)(p - b)tp - c).
  • Nếu c < 90°. Từ (2) ta có 2a.CH - a 2 4 b 2 - c 2 <-> c 2 - a 2 4 b 2 - 2a.CH. « Nếu c > 9Ỡ°. T ương tự từ (2) ta dưdc c 2 = a z 4 b 2 - 2a.CH. • Nếu c 9Ữ° thỉ từ (2) có c 2 a 2 - b 2 .
  • Xét C <90° thi CH = b.cosC. và lừ (2) có c 2 = a 2 4 b 2 - 2ab.cosC Xé: c > 90° , chứhg minh tuơng tự ta dược e 2 - a 2 4 b z 4 2ab CỒS(1S0° -C). Nhận xét. Tử hệ thức (1) ta suy ra được nhiều hệ thức quen thuộc mà chủng ta thường xuyên sử dụng. Sau đãy tà một sổ ứng dụng của định li Stsw&it
  • Bải tập áp đụng Bãl toán 1. Cho 3 điểm A. B, c cố đinh vãi B rẳrr giữa A vá C r hai điểm D vả E không thuộc đưàhg thẳng AC thỏa mãn BD = BE và CE CD. AD 2 - AE 2
  • Chúng mình rằng — — không đổi. ce 2 -cd 2
  • Chííng minh rằng AD ■* AE «> CD > CE. Lỏi giải. Q
  • Ảp dụig định lí StewarL với AADC và AALC ta có da 2 .bc + dc 2 .ba-ob 2 .ac -ac.ba.bc EA 2 .BC + EC 2 .BA- EB 2 .AC = AC.BA.BC Kết hợp VỚI BD BE, suy ra DA 2 .BC + DC 2 .BA = EA 2 .BC + EC 2 .BA « (DA 2 - EA 2 ).BC = (EC 2 - DC 2 ),BA DA 2 -EA 2 BA . .... < > — rìr = ( khõn 3 dổi). EC 2 -DC 2 bc
  • Từ (DA 2 - EA 2 ),BC = (EO 2 - DC 2 ).BA, suy ra DA cẺA^ECc De, Bãi toán 2. cho AABC (AB AC) có H lá Irực tâm, o là tám đường trân nội tiếp. Chửng minh rằng HA 2 + HB 2 + HC 2 = -(a 2 + b 2 + c 2 + 4H0 2 ). 3 Lài giải.

    Gọi M là trung điểm của BC r G là trọrg tám của AABC.Theo định líEuler thi H, G r o thẳng hàng và HG - 2GO. Theo định lí Slewart cho AAHM ta có HA 2 .GM + hM 2 .GA -HG 2 .AM - AM.GA.GM « HA 2 + 2HM 1 - 3HG 2 = 4aM 2 3 u . 2 . _í HB 2 +HC 2 BC 2 <=> HA +2 — — i 2 4 ( 2 Ý ■ 3 f H0 b 2 + c 2 a

    à 2 > 1 <=> HA 2 +HB 2 -t-HC 2 -ị{3 2 +b 2 + c 2 +4H0 2 ). 3 HAI BÀI HÌnH HQC PHHÍ1G TR0Ỉ1G kì ĨHI IIỈ10 2015 NGUYỂN BA ĐANG (Hả Nội) Bàì viết này chủng tõi xin giới thiệu lởi giải hai bài hĩnh học phểng trong ki thi 1MO 2015 chỉ dũng kiến thức ở Trung học cơ sỏ. Bãi toán 1. Cho tam giác ABC nội tiếp đưúng trò" (O), đường tròn (A) cắt cạrìh BC tại □ vã E (B f D, E, c :heo thử tự ơó trẽn đường thẳng BC) vá cẩl đường tròn (Q) tại F và G (F, B, C r G theo lhJf tự đỏ trẽn đường trôn). Đường tròn ngoại tiếp aFBD cắt cạnh AB tại K. đường tròn ngoại tiẽp AECG cắt cạnh AC tại L. Các đường thẳng FK vả GL cắt nhaL lại X. Chứng minh rằng X nằm trẽn đường thẳng AO. (Hy Lạp đẳ nghị) Lãi giải. Ta có tứ giác FDEG nội tiếp và AF - AG. Suy ra GẼC = 180° -g!d = G?D. Gọi I là giao diểm của FG vè AC. Xét đường tròn (G) ta có ÁĨF = l(sđẤF+sđGC) =-l(sđÃG +sđGC) = -lsđẤC = ÃBC.(1) Vì các lứ giác BFKD và ECGL nội tiẽ'p nén ABC = KFD; GLC = GEC. Từ ( 1 1 ta có LGF = LG i = ŨF - GLC
  • ẤBC - GÈC = ABC- GFĐ => LGF = ABC - (KFD-KFG) = ẤBC - KBD + k?G = KFG. Suy ra aXFG cản tạỉ X, mà aAFG cân tại A. Do đó A, X, o thẳng hàng. Vậy X thuộc đường thẳng AO. Bình luận. Bãi nảy là hài dễ ctĩa ngày thi thứ hai, chỉ cẩn sủ dụng kiến thức về góc nội tiếp và từ giác nội tiếp, không cắn vẽ thêm các đường phụ Bài toán 2. Chó tam giác nhọn ABC (AB > AC). Gọi (O) là đường tràn ngoại tiếp tam giác, H là trực tâm tam giác. F là hình ch ếu vuông góc của A trên BC t M là trurg điểm BC, Q và K trên (O) thủa mãn HQA = 90” và HKQ = 90” (A r 3, c, K. Q trẽn (o) theo thứ tự đó). Chửhg minh rằng đưởng tròn (HKQ) và đường tròn (MFK) tiếp xúc vối nhau. (Ukraine đề nghị) Kéo dải AO cắt dường tròn (O) tạì D, AH kẻo dãi cảt õường tròn (0) tại E. Suy ra AD là đuàng kính của đưủhg tròn (Q), từ dó ÁQŨ = 9D U . Mà ÀQH 90°. Suy ra D, H, Q thẳng hàng. Gọi G là giao điểm của QD và BC, vì AABC nhọn nên H và E đối xủng vãi nhau qua BC. Do đò FH = FE. từ đó HC = CE. Ta có AF L BC và DE J AE nẽn DE II BC, từ đó BD = CE= CH, suy ra AGBD = AGCH. Vì GB = GC nên G - M, suy ra Q. H, M, D thẳng háng, Kẻ đường -tính QP của đường tròn (O), ta có PKQ = 90°. Mặt khảc HKQ = 90°. Suy ra p r H, K thẳng háng. Xé: đưửng tròn (O) la có PKIE = PQE = ÒQE = 90° -ịũOE =90° -QBE. 2 QMC = EMC = (sđ QC +sđBĨ3) = 4(sđQC+sdCE) =^sđQầ =QBE 2 2 Mặt khảc MHE 90° - QMC 90 a -ÓBE=»PKỀ MHE. Suy ra DQ lã tiếp tuyến của đường tròn rgoại tiếp á KHE tại H. Gọi I lả gỉao điểm của đưâng Lhẳng vuông góc vái HQ tạl H và đưòng thẳng Bc. Vi các đưòrg trung trực của HK vã HE cắt nhau tại I trên BC nên IK = IH = IE. lừ đó I là tám dường tràn ngoại tiép AKHE. Ta có IF.IM = IH 2 = IK 2 (vì IH ± HG và HF ± IG), suy ra IK lả tiếp tuyến đường tròn ngoại tiếp AVFK. Mặt khác Ihl là tiếp tuyến cùa đơờng tròn ngoại tiếp AHKQ có đường khh QH, suy ra IK là tiếp luyen của đường tròn ngoại tiếp AHKŨ. Vậy đưàng tràn (HKQ) và đưỏng tròn (MFK) tiếp xúc nhau. Binh luận. Bảl này được ban tổ chức xếp là bài khò của ngày thi thừ nhất, phổ điểm cho thấy các thi sinh đạt điềm rất thấp ả bài này. ĐỂ giải quyết bài toán này ta tìm một dường thẩng tiếp xủc vài cả hai dường tròn làm cẩu nổi dể chứng minh hai dường rròn tiếp xúc vdl nhau. QUAN HỆ GIỮA HAI BẤT ĐẮNG THỨC Ali/I-GM VÀ NESBIT NGÔ VĂN THÁI (GV. THPT Phạm Quang Thẩm, Vũ Thư, Thải Binh) B ái ơẳng thức AM-GM vá bấỉ dẳng thức Nesbit ba biến là một cập bất đẳng thức đẹp. cõ nhiều ứng dụng khi giải các bài ioân vế bãi đẳng thức. Mỗi bất đẳng thức đều cố cách Ghiíng mình độc lập với nhau. Trong bài viết này chùng ta sẽ sử dụng bầt dẳng thức này dề chimg minh bất I lẳng thức kia và ngược lại. đóng thời cũng đưa ra một số vi dụ, bài tập hay và khó được chùng minh nhờ àp dụng hai bất đẩng thúc này. • Bất dẳng thức AM-GM ba biến Vái a, b, c lá ba sổ [hực không ãm thi a 3 + b 3 +- c 3 > 3abc. • Bất dẳng thửc Nesbit ba biến Vởi a. b, c là ba sô' thực dương Ihì . . a , b , c 3 b-í-c c-a a i-b 2
  • Sừ dụng bất dắng thức AM-GM dể chứng minh bất dắng thức Nesbit ủặt p = — i- + -+- 3

    b-t-c c + a a + o „ c a b Q = - — - — + + - b+c c+a a+b ŨB thấy p + Q = 3. Theo bấl dẳng thức AM-GVi ta có

    a-b.b + cc + a b I c c I a a ' b ssttr— -7— — — - 3. (1) \ (b + c){c + a)(a + b) , . , a+c . b + a . c + b b-c c + a a + b S33j (a ' cj<b ' a>cc ' b) =3.(Z) và bất đẳng thức Nesbit ta cổ b 3 +c 3 a 2abc a z b 2 +c 2 2bc u 2 7 7 c * 2oa 2ab c z lế
  • — - — — > — 2 2 a 2 +b « a 3 +b 3 +c 5 >3abc.
  • Một số ví dụ Ví dụ 1 . Gọi a, b, c là độ dài ba cạnh của một tam giác. Chứng rrinn rằng (a + b)(b + c)(c + a) > 8(a + b - c)(b + c - a)[c - a - b). Lời giải. Theo bất đảng thửc Nesbit ta cỏ a b c <-) b + c 3 2 2 . 3 2 c T a .4 a-b 2 t ■ (- Ị b c a \ b f c J c + a f a - L b oia j I a I b / ■b + c-a c + a-b a*b~y ...
  • — — — + — — : — . ( 1 ) b + c Q-Jr g 3 +- b 2 ũ +c c*a a + b j V a, b r c là độ dải ba cạnh của một tam giác nê 1 ' a + b- c>0 r b + c- a>0,e + a- b>D. Ád dụng bất dẳng thức AM-GM la cá í (b - c}[c + a}(a +b) Từ (1) và (2) suy ra 3 Mh-P + M + Q>6» 2 M > 3 => M > 4. 2 Đảng thức xảy ra khi và chỉ khi a = b = c.
  • sử dựng bát dẳng thức Nesbit để chứng minh bât dẳng thức AM-GM ba biên • Nếu trong ba sô a, b. c có ít nhất một sô bằng 0, ta có đpcn. • Nếu a, b r c > 0, áp dụng bâì dẳng thửc AM-GM b + c-a c+a-b a+b-c
  • + — + - b + c c + a a + b ; J [b + c-a)tc ta-b)[a + b-c] í (b T c)(c + a)(a+b) ' 1 Từ (1 ) và (2) suy ra đpcm. Vi dụ 2. Cho a, b, c là ba số thực dương. Chửng minh rằng A = D 2 + bc + c 2 c 2 +ca a 2 a z -ab + b 2 1 . Lòi giải. Ảp dụng bất dang thức AM-GM vả bất đẳng thức Nesb l ta có k>- 1 1 \2 h í,*i ,2 0 +c tr 4 — — 1 { 1 e z +er 2 \ — — — + a 2 3 + & -2 a 1 - — — 1 ỉr 1 — -ỳ 11 1 1 1
  • 11 ajq 2 2 cj 1 1 c ■ J a b Ị a " ° C’ >

    b 2 + c z c 2 + a 2 a 2 +h 2 3 2 V J □ẳng thức xảy ra khi và chì khi a = b = c. Vi dụ 3. Cho a, b r c là ba Bố thực dương. Chứng minh rằng

    a 2 + (b + c) z b z +(c + a) z c z + {a + b) z . 15

    a[b+c) b(c + a) c(a+b) 2' Lởi giải. Áp dụng bất đảng thức AM-GM và bất đẳng thức Nesbl ta cỏ 4!=1. 3 2 B - Í a b c [ b + c c + a a+b f a b c 5 I ■— 4- + — — I + Ịb+C c-a a -b

    b+cc+a a + b a b c J V / ( 2Sc 2^fcã . 2i/ãb' 2 2 □ấu bằng xảy ra khi và chì khi a = b = c. Vi dụ 4. Cho a, b, c la ba SD thực dương thỏa nin abc = 1 . Chứhg minh rằng " c z 3
  • b t c c I a a I b 2 Lởi giải. Theo bất đẳng thửc Nesbil la cô (a + b-cj[ — 4— ĩỉ— 4— í— |>(a-h-c)

    b+c c+a a+b 2

    Í9 4b-c) 2 = + a + + b + — ' — c >— (a + b + c c + 9 a 4- b 2 V ] \ ) l a a b 3 c 2 a I b I c 3

    => ì + — — - > — - — — > — = — . b + c c í a a ? b 2 2 2 Đảng thức xảy ra khi và chi khi a = b = c = 1 . Vi dụ 5. Cho a r b r c > 0 thỏa mãn abc = 1 . Chứng mình rằng

    1,1,13

    c — "T + ^ — — . a 3 (b + c} b 3 (c+a) c 3 (a + b) 2 Lời giải. V a, b, c > ũ, abc = 1 nên s2 r*\2 c=- f ềị \2 JL

    í 1 . 1 a — + — 1 • J 11 — ỉ cl — — +— Đảng thức xảy ra khi và chì khi a b c 1 . Ví dụ 6. Cho ba số :hực dương a, b, c. Chứng mình rằng «3 1. , 1 k 3 Ik ì 3 D = 2a + 2 b L 2b + 2 c“ — _ — - 2c + 2 3 a + b “2 b+c c + a a + b 2 Lửi giài. Theo bất đẳng thức AM-GM ba biến ta có a 3 + 1 + 1 > 3a « a 3 -2a + 2> a 1 2a + 2 > a b + c b ị c Chứng m nh tương lự ta có 2b + 2 2c + 2 c + a Vạy D > c + a a b
  • a -b c a-b

    (theo bất đẳng b+c c + a a + b 2 thức Nesbít). Đảng thức xảy ra khi và chì khi a - b - c - 1 . Bầi tập vận dụng Bài 1. Cho a r b, c lá ba so thực dương. Chứng mình rằng E = ã 2 b 2 c 2 3
  • + l3 - - , a-7-

    (b ■ c) 2 (c 4 a) 2 {a I- b} 2 4 Bài 2. Gợi a, b r c là độ dải ba cạnh cùa một tam giác. Chííng minh rằng

    b + c-a + ^c + a-b ìl' a *b-c
  • Bải 3. Cho a, b, c lá ba số thực dương thỏa nan abc = 1. Chứng minh rằng c . a . b . — + f- + — > 2 a b c 1 . 1.1 — + — + — a 2 (b 2 + fj b ? (c 2 +1) c 2 (a 2 +1)
  • ĐỌC LẠI CHO ĐÚNG Trang số bảo 159+160, chuyên mục Học ra sao? Giải Toán thè nào? củ hải viet Sử dung phương pháp đánh giả để giải hệ phương trinh của hai tác giả: Lẽ Đủc Thuận (Pho Trưâng phòng Giáo dục và Đào tạc Hoàn KiêYn, Hă NỘI) và Cao Vãn Dũng (GV. THPT Tây Hổ, a Tảy Hổ, HÈ NỘI). Thành Lhật xin loi hạn đọc. AUSTRALIAN MATHEMATICS COMPETITION AMC 2015 SENIOR DIVISION AUSTRALIAN SCHOOL YEARS 9 AND 10 Time allovved: 75 minutes PGS. TS. ĐỖ TRUNG HIỆU (Hà Nội) (StAỉ tầm vá giới thiệu) Questions 1 to 10 r 3 marks each
  • What is the value of 21 X 2015? (A) 45231 (B) 54321 (C) 42315 (D) 14325 (E) 2351 4
  • II K = L + R 2 . L = 4, K = B5 and R Is positive, then R equals (A)5 (B)7 (C)9 (D)4 (E) 8
  • On their school nnlidays, Xenia and NgDG warked for a tarmer picking íruìt. Xenia WDrked for 5 days and Ngoe for 3 days. The íarmer paid them S100Q, which they shared In the same ratio as the days they workea. Ngocs share was (A) S325 (B) 5300 (C) $250 (D) 5375 (E) 3500
  • An equilaleral triangle. a sau 3 re and regular pen:agon are joined as in the diagram. What is the SĨZ0 of angle X? (A) 10B° (B) 105° (C) 90 D (D) 1 20° (E) 102 a
  • 3 2 -2 3 eouals 1 1 17 (A) -1 (B) 0 c 72 p — 72 E — 72
  • Janna measures three sides of a reclangle and gels a total ũf 80 cni. Dylan Tneasures three 5ides of the same '■ectangle and gels a total of B8 cm. What is the perìmeter of the rectangle? (A) 112 cm (B) 132 cm (C) 96 crr (D) 168 crn (E) 156 cm
  • Two ordinary dice are rolled. The lwo resulting numbers are mulliplicd togeiher Lo create a scnre. The prohaoillly oí rolling a SCO re Ihat is a multiple of six is ÍA) n 6 (B)T 12 «ỉ (0)1 •ỉ
  • Two congruent equilateral trangles overiap to make a concave hexagon as shown. Each triangle has a vertex on the others centre, What traction ũf the exagons area is shaded? (A) ị 9 <
  • The diííerence between two numbers is 20. When 4 is aoded lũ each numhe r Lhe la r ger is three times the smaller. What is the la r ger of the two original numbers? (A) 26 (B) 40 (C) 38 [0) 22 ỈE) 32
  • A ba r -tailed godvvit \was recordea by satelllte tag in 2007 to have flown 11 500 km in eght days. On average approxirnately how many kilometres per hour is tnat? (A) 120 (B) 6 (C) 1 (D)24 {E) 60 Questions 11 to 20, 4 marks each
  • Let A be the set (0, 1, 2^. Let B be the set (3, 6, y), where X Is an integer. I multiply each number in the first set by each number in the second set, Let c be the set of all Ihe numbers which are the results of Ihese multipl catiũns. What cDLild X be 5 Lich that c has exaclly 5 distincl elements? (A) 12 (B) 4 (C) 24 (0) D (E) 6
  • What is the value of thia expression? 1 1 1
  • -

    1 1 1 1 2 3 4 5 (A)A 16 45 0 TTT

    154 (B)A ' 14 £E) 66 (C)|ị 77
  • The diagram indicates a pattern of paving blocks that are laid diagonally across a rectangular floor measudng L cm X 2Ũ0 cm. Each block measures 8 cm X 4 cm, and any two bloc<s tliat toueh overlap by 5 cm. 5 cm k — •! 200 cm What is the value of L? (A) 253 (B) 155 (C) 400 (DÍ 250 £E) 158
  • In a class of 25 students, 11 students a r e fif:een years old and the rest are Bixteen years old. There are 15 boys. T"ere are twice as many sỉxteen-yea^old boys as fifleen-year-old boys. What ừactÍDn of the class are sixteen-year-ola glrls? (A) ả < B) f < c >á (0 > 5 <E) ề
  • Peter was cutting a pipe wìth an outside diameter oí 20 cm, When the cut was just thrpugh the wall o( Ihe Dipe, it Vivas 10 cm in lenglh. How thick was the wall af the pipe in centimetres? (A) 5 [8] 5^3-5 (C) 10-5^2 (D)4-7ĨÕ (E) 5(2— 73)
  • Thnee dìĩeren. dinosaur íossils have been found in the kĩngdom oỉ Mathernania. Euleraptar Ỉ5 twice as old as Gaussasaums, though when Gaussasaurus was alive Euleraptor was twice as old as Eermatops. The SLm of the ages of the Lhree íossils is 360 million years. How many million years a'lè' í Fermataps dd Gaussasaưrus íve? (A) 120 {B} 40 (C) 80 (D) 60 (E) 20
  • Al Gunaroo High School, a iwo-weekly (10-day) t metable 5 used, with 5 periods in each day. Mon Tue Wed Thu Fri Students ta ke se ven subịerts, English, Maths. Science, Geography, Hĩstory, PE and Ais, in a rotating sequence in the order gi ven, staiing wth Erglish on Day 1, perod 1. This takes up 49 of the 50 available periods. The one remaining period is an assembly which is held on ibe lìrst period of one of the days of the second week. Whlch lesson car never be held during period 1 of one of the other days of the second week? (A) Engl sh (8) Maths (C) Geography (D) PE {£) Arts (Ki sau dàng tiấp) Bãi 10NS. Tìm các sỏ nguyên diíơng a, b, c thỏa mãn a 2 - 2(b + c) và a 3 - b 3 - c 3 - 3abc. LƯU LÝ TƯỞNG (GV. THCS Ván Lang, TP Việt Tri, Phú Thọ) Bài 1 1NS. Tính p = a + h, biết rằng a 3 + b 3 4- 3ab = 1 . PHỪNG VĂN LONG (GV. THCS Vĩnh Tường, Vĩnh Tưởng. Vĩnh Phúc) Bài 12NS. Cho tứ giác ABCD vãi B = D = 90°. Trẽn tia DA lấy địểm E sao cho DE = DB. Đường tháng vuông góc vửì ED tại E cắ' tía CB tạ
  • Chứng minh rầng AACK cân.

    NGUYỄN DỬC TẤN (ĨP. Hồ Chí Minh) CUỘC THI GIRI Tonn DÀnH CHO nữ sinH (TTT2 số 157+158) Bài 4NS. Ta cố n n n n n n

    13n

    2 +■ 3 +■ 4

    2 + 3 Ã ~\2 Vi n n n 2
  • 3 4 4 1 3n lả số nguyên nén - là sổ 12 nguyên. Mà (13, 12) = 1. Do đó n là bội của 12. Ngược lại nếu n là bội của 12 thì thỏa mãn đảng thửc. Vậy có 2015 12 = 167 Sũ nguyên dương n k n ỏng lốn hơn 2015 thỏa mãn đẳng thức đâ cho. Nhận xét. Cãc bạn có IỜ! giải dửng: Nguyẻn Thúy Dương, Bùi Thị Quỳnh, Nguyễn Thư Hiền, 3A3; Trần Thị Thu Huyền, 9A3, BÙ! Thủy Linh, 8A1, THCS Lãm Thao, Lảm Thao; Nguyễn Ngọc Huyền, 9A r THCS Hùng vương, TX. Phú Thọ, Phú Thọ; Chu Thị Thanh, 8E1. Bạch Bùi Nguyệt Anh, 6D, THCS Vĩnh Tưởng, Vĩnh Tường, Vĩnh Phúc. Bài 5NS. Ta có X - y - z - 2(Vyz + Vỹ + Vz -Vx) (1) 3Vyz-x-V3z+1 (2) ĐKXĐ X > 0,y > 0,z > ũ. Ta có (1) <-> X +2-1 = y +z +1 +2sỊyz +ĩjỹ +2-VT « (Vx +1) -(Vỹ + Vz + 1 ) 2 4-> Vx +1 - Vy" + <jz +1 (vì Ví +1 vả Vỹ+Vĩ-S-I cùng dương). Vx = VỸ - Vz X - y + z - 2 VỹzV (3) Thay (3) vào (2) ta được y + z— Vỹz - v5z + 1-0 <-> 2 3 4 3 Thay giá trị y và 2 vãc (3) La dược X = 3. Hệ phirơng trình cò nghiệm (x; y; z) = Nhận xét. Các bạn có lòi giải đúng: Trần Thị Thu Huyền, 9A3; Nguyễn Thủy Dương, Bùi Thị Quỳnh, Nguyễn Thu Hiền, BÁ3, THCS Lãm Thac, Lârr Thao; Nguyễn Ngọc Huyền. 9A, THCS Hùng Vương, TX. Phú Thọ, Phú Thọ; Chu Thị Thanh, 8E1, THCS Vĩnh Tường, Vĩnh Tưàng; Tạ Thủy Tiên. 9A4, THCS Yẽn Lạc, Yên Lạc, Vinh Phúc. Bàl 6NS. Q Nói AC, BD. Dễ thấy AMBC ưì aMDA (g.g) BC MB "dã mõ' Vlả MẼ = 3MA. Do đỏ Ệ£ = 3 ỊÍẶ. (1) AD MD Xét AMAC và AMDB có M chung, MAC = MDB (cùng bủ BAC do tứ giác ABDC nội liếp) MA AC Suy ra AMAC to AMDB (g.g) => ^ = GCL. (2) y MD BD (Xem tiếp trang 39) cuọc THI *l/(6i (Ịs&àO' 20 ĩ 6 (Tiếp theo số 159+160) Bài 6. Có bao nhiêu 50 lự nhiẻn có 6 chữ số khác nhau, trang dó mói số có tdng ba chữ số dầu nhỏ hơn tổng ba chừ số cuối rrệt đan vị? CAO VĂM DŨNG (GV. THPT Tây HÔ. Q. Tây Hổ, Hà Nội) Bài 7. Xung quam mộl cái hồ hình tròn, người ta trổng 15 cây dừa. Người ta nhận thấy rằng các độ chênh lệch chiều cao của hai cảy dừa liên tiếp (kể nhau) luổn bằng nhau. Chúng minh rằng 15 cảy dừa dó có chiêu cao bằng nhau. nguyễn tiển lảm (GV. THPT chuyên Khoa học Tự Nhiên Hả Nội) Bài 8. Trên bảng ghi 20 câu khẳng định trong đó Cỏ một SỐ câu đúng và một số câu Sái. Ban tổ chức dưa cho bạn Việt các phiếu sau;
  • Trẽn bảng có ít nhất 1 khảng dính sai. 2 . Trẽn bảng có ít nhất 2 khăng đình sai.
  • Trên bảng có ít nhất 3 khầng đính sai.
  • Trẽn bảng có ít nhất 20 khẳng định sai. Việt cỏ thể giữ nguyền Lhứ tự các phiếu trên hoặc đổ! chẽ các phiẾu dó vãi điếu kiện: NỂU câu m đủng thl dưđc nhận thưởng 200000 đổng X m tư Ban tổ chức. Bạn hãy giúp bạn Việt sắp xếp hdp li các câu trên đề bạn Việt (TƯỢC nhận vế số tiển lãn nhất từ Ban tổ chức. NGUYỄN ĐỨC TÁN (TR H5 Chí Minh) Bài 9. IVỘt bảng điện tử ghi HÉ 201(5 gồm bảy hhh, mũi hình biểu thị một chữ cái, hoặc dấu huyển, hoạc một chữ sổ. Mỗi hình gâm các õoạn thẳng (độ dài như nhau) đèn ống tuýp nối vâi nhau, chẳng hạn chữ E cỏ 5 đoạn ống tuýp. Hai bạn thay phiên nhau bật đèn thao quy tấc sau:
  • Mỗi lần dược chọn một hình và bật dèn một so tùy ý doạn ống tuýp của hĩnh dó, hoặc được chọn hai hình và bật đèn sao cho sô ống tuýp sáng của hĩnh nảy bấng số ong tuýp sáng của hình kla.
  • Ban dầu bảng diện chưa sảng dàn. Ai là ngưởi bật cuối cùng để toàn bộ hình HÈ 2016 trên bảng điện sáng lên là tháng. Nếu bạn là người đĩ trước thì chọn cách bật đèn như thế nào dể chắc chắn thắng? NGUYỀN VIỆT HẢI (Hà NỘI) Bài 10 . Cho bảng hình vuông 5x5 như hình vẽ dưới. Hãy đìỂn mải chữ số 2. 0, 1 . 6 váo một õ vuỏng sao cho trẽn moi háng hay mỗi CỘL phải có đủng 4 chứ 50 vã mỗi chứ số chỉ được xuất hiện mật lần. Các chữ số nầm ở các ô vuông liền nhau tạo Ihành một sô (từ trải sang phải hoặc từ trên xuống dưới). B ết rằng các sô' bên ngoài hảng bằng tổng của eảc SỐ õược viết trong mỗi hảng hay mỗi cột 2016 603 207 621 621 BI HOÀNG TRỌNG HÀO (TP. Hồ Chí Minh) 2 1 G 1 6 0 1 6 2 126 63 162 36 63 36 THÁCH ĐÁU! THÁCH ĐẤU ĐÀY! TRẬN DẤU THỨ MỘT TRĂM BA MƯƠI TÁM Người thách dấu: Bùi Hầi Quang, GV. THCS Vãn Lang, TP. Việt Tri, Phú Thọ. Bài toán thảch dâu: Cho các số thực X, y, z thủa mãn X, y, z > 1 và X + y z = 5. Tìm gíá trị lỏn nhất của biểu thức „ 1-2x 1 - 2y 1-2z p ® T — T — — 3 — 7 — _ 7 + _3 . ' 7 X +7x-y-z+1 y +7y-z-x+1 z +7z-x-y + 1 Xuất xủ: Sáng tác. Thòi hạn: Trước ngày 08.10.2016 theo dấu bưu điện. TRẬN DẤU THỨ MỘT TRĂM BA MƯỬI SÁU (TTT2 SỐ 158) Vì a r b, c > 0 và a + b + c = 3 nên ta có A = a 2 + b 2 + G 2 - 2ab - 6bc - 4ca = (a + b) 2 - c 2 - 4ab - 4ca - 6bc = (a + b) 2 + g 2 - 4a(b + c) - 6bc = (3 - c) 2 + c 2 - 4a(3 - a) - 6c(3 - c - a) = 4a 2 I 8c z ! 6ca - 1 2a - 24c I 9 4a 9c 2 \
  • 9 + 6ca-9c-12a !
  • 23

    c T 2a + —c - 2 15c 225 ì 23 ' 529 I ) \2 ỉ , ■3 I +23 225 23

    15 ’ 23 225 . 225 23 23 M ... . Ị2. k 27..» Đãng thức xảy ra khi a - — ; b i-;c . 23 23 23 ....... 225 12 . 27. 30

    Vậy M'nA — — a - — — ; b - — T-;c - — . 23 23 23 23 Họ vầ lên Lởp: Trương: ..... CÙNG BẠN ĐỌC Tạp ch ĩ Toảr TụổUhữ đã XLiăỉ bản những cuốn sách chuyên de tập hợp từ cảc chuyên mục hay, nhiễu độc gả de nghị. Tạp cnỉ dur đirn biẾn soan 3 cuỏn sảcb mới. Ban cho :òa soạn biết muốn có cuốn sách nào (đánh dấu X tfàa ó tưtíng ứrg vã gửi vể địa chỉ 361 Trường Chỉnh, Hả Nôi):

    IQ - Đo trí thông minh. Phá àn cùng thảm tử Sẽlõccõc. 1 45 đế olympic và Câu lạc bộ Toán Tuổi thtí (bằng liếng Anh dự kiến giá bìa 70 ĐDŨ đong) , cảm ơn các bạn. wỊỊm ơ™- Nhận xẻt. Ban Nguyễn Sún Lảm, 9A4,

    THCS Giẩy Phong Chàu, Phù Ninh, Phù Thọ cỏ lửi giải gọr gàng nhất là người đàng quarg ỉrong trận đấu này. Các bạn sau có lời giải đúng được khen: Bùi Anh Vũ, 9B, THCS Vĩnh Tường, Vĩnh Tiiàng, Vĩnh Phúc: Đặng Quang Anh t 9A r THCS Nguyên Chích, Đong Sơn, Thanh Hóa: Nguyễn Thị Minh Thu t 0A2, THCS Yên Phong Yèn Phang/Bắc Ninh. LÊ Đừc THUẬN CÁCH THỨC GỬI BÀI CHO TOÁN TUỔI THƠ Vòi học sinh giải bài. u Mỗi bái toán irong mục th GTQT, rnỗì bài tham dự trèn các chuyên mục khác, đều phải lãm riêng trẽn một tở giấy và gni r ẫ cảc thông tin cần thiết; Tham dự mục gì, số tạp chí. tẽn để bài, họ và tên học sinh, lâp, trưởng, huyện (ũuận), tỉnh (thành). □ Học sỉrh hèn ghl thêm đ[a chỉ nhã, số diện thcại nhà, 50 điện thoại phụ huynh (nếu cỏ) dể Tòa soạn tiện gùi phầr tnưửng.

    Vãi các cộng tác viên gửi bài. Các Dài vỉẻT của lác giả gửi TTT chưa được đãng ở bất cứ đâu. Bài đã gùi TTT không gửi cho bảo khác. Nếu là bái SƯU tầm, tác giả cần viết rõ vào bài gủi là sưu tẩm và dẫn nguổti SƯU tấm Nếu là bài dịch, tác giả cần ghi rõ vào bãl gửi la dịch vã gủl kèm Dẩn gốc. ik Các cộng tâc vlèr có thể gửi bái qua đưòng DUU điện đến Tòa soạn: Tầng 5, số 361 Truủng Chinh, Q. Tharh Xuân, Hà Nội hoặc gửi enaíl đến: toan tuoi tho @vm.vn TTT Vui cười ũ Thấy Toãn cữ lúi híũ mãi bén máv J Tuổi nói với Thơ:
  • Hõm nay, cô bạn học giỏi nhà'l lớp đã thừa nhạn lớ chinh lá đối Ihủ nặng ki của cõ ấy.
  • Thế ả? Cớ ấy nói lức nào vậy?
  • Lúc chúng tớ đang căn kiểm tra sức khỏe. rj 1 « /|\ ■fị' J T'- 'P' 'pii JF|ỉ □ Thư bâo Tuổi:
  • Chiều nay tớ sẽ khao cặn chẽ. kem, cánh gà chiên đế ãn mừng thành tích cùa tớ.
  • Tuyột quái Thành tích gì vậy?
  • T('í vừa giảm dược nửa cân. J Tuổi hỏi Thơ:
  • Cậu đã lãm bãi loàn thảy giao hôm qua chưa?
  • Bải nào nhỉ?
  • Bài "Mộl người đi bộ với vận tốc 5 km/giờ. Hỏi người dó cần bao lâu để đi hểt quãng dưởng 7 km”.
  • À, bãi đâ lờ dang chừ...
  • Chở gi? Sao lại phải chở?
  • Mẹ lớ dang đi bộ Sáng nhà bác lớ, cách dãy 7 km, Sang tứl nơi mẹ sẽ gọi diện về cho tớ. Lính, mẹ hỏi:
  • Con viết email cho ai à?
  • Vãng. Con viết cho bà ngoại ạ.
  • Sao con gõ chậm thế? Mãi không xong bức thư.,.
  • Thì mắt bà kém. bà đọc chậm mà mẹ! Thấy Thơ ôm má kcu dau răng, mẹ bảo: Tại con ăn nhiêu kọo quá đáy.
  • Không phải tạj ãn kẹo dâu ạ... Ăn

    keo Llii cá hai hãm ràng đcu dinh ngọL, thế nhưng lạì chí cõ mãi một càì răng bị dau thôi. Chửng tỏ không phải tại kẹo. Ỷ - ‘H k- ■ afc atp. -ĩ*

    iT* rĩ 7T 1 r- 'T ^T' ! Trong giờ toán, cỏ giáo viết dẻ bãi lên bảng rồi nói: -Các số liệu của dề bải dã đảy đủ hếl rồi. Em não có thể dặt câu hỏi về ữiởi gian? Cả lớp còn đang im lặng suy nghĩ Lhì Thơ giơ tay:
  • Thưa có. bảy giờ lả mấy giờ ạ? NGUYỀN THỊ DIỆU NGA (Số 80 dường Xuân 68. TP. Huế, Thừa Thiên Huế) NGUYỄN ĐỨC TẨN (TP. Hổ Chi Mình ) tmm HÌNH NÀO KHÔNG THÍCH HỢP? Bái 1. Tim sổ tiếp theo của dãy số: 9; SI; 961; 9301; ... Bài 2. Hãy Ihay X, y bài các số tních hợp : HÌNH NÀO KHÔNG THÍCH HỢP? (TTT2 số 158) Nhận xét. Qưy luật của cả hai bài Kì này đều dễ phái hiện. Vởì bài 2, các hạn nén cụ :hể hóa quy luật qua các sổ đã cho. Quy luật. Bải 1. Vửì những cách nh‘n k^ác nhau la có thể tìm được quy luật tưtìng ửhg. • Cãch 1 (cất gnép hình); Các hình A, B. D được chia tnành (hoặc ghép bãi) 4 hình bằng nhau, còr hình c dưọc chia thánh (hoậc ghép bởi) 5 hình bằng nhau. Vậy hình c Không thích hợp với các hình còn lại. • Cảch 2 (gọi tên hình) : Các hinh B, c, D được ghép bởi các hình lam giác, còn hình A không được ghép bởi các hình tam giác. Vậy hình A không [hích hợp. • Càch 3 (rinh đổi xúng); Các hình B. c, D có hữu hạn Irục đắi xứng (tương ủng là 2, 5, 3 Irục đố xửtig), còn h'nh A cỏ vỏ số trục đổi xứng. Ngoà ra các bạn còn có thể nhin các hinh theo sô’ nét vẽ hoặc theo khía cạnh đa giác. Bài 2. Trorg dãy số, kể từ SỂ hạng thử hai, moi số bằng bìrh phương của sổ kể trư&c trừ đi ' . Vậy số tiếp theo của dãy là 3968 2 - 1 = 15745023. Tòa soạn xin trao thương cho các bạn có Idl giải chính xác, phát hiện duợc nhiều quy luật: Ma/ Thanh Tãm. 7A, THCS Lý Tự Trọng, Binh Xuyên; Nguyễn Thùy Mai, 7E1, THCS Vĩnh Tướng, VTn n Tưởrg, Vĩnh Phúc; Nguyễn Khánh Huyền, 7B, THCS Bạch Liêu, Yên Thành, Nghệ An: Vũ Thủy Dương, 7B, THCS Hoấng Hoa. Hoẳng Hóa, Thanh Hóa; Trán Đỉnh Hoàng, 6C, THCS Hoàng Xuân Hãn, Đửc Thọ, Hà Tĩnh. Các bạn sau được tuyên dương: Nguyễn Minh Hiển, 6A1, THCS Nguyễn Đãng Đạo. TP. Bắc Ninh, Bắc Ninh; Nguyễn Thúy Dưang, 8A3, THCS Lâm Thao. Lâm Thao, Phú Thọ; Vú Tníờng An, 7B, THCS Bạch Uéu, Yên Tnành, Nghệ An: Nguyễn Hưng Phàt, 6B; Chu Quyết Tiến, 8C; THCS Hoàng Xuàn Hân, ĐiYc Thọ, Phan Quốc Bảo, 7/1, THCS Lè Ván Thiẽm, TP. Hà Tĩnh, Hà Tĩnh; Trần Đạt Vỹ, 8/9, THC5 Pnạm Hổng T^ãi, TP. Pleiku, Gia Lai. NGUYẺN XƯÀh BÌNH CHEMICAL ELEMENTS (TTT2 sô' 158) THỀ CỜ (Kì 81) 1.@g2 %4 [1...àe4 2.®h3#; ădl 2.®gè#] 2.®c2# Esẵímiĩ Các bạn được thuồng kì rây: Lê Ngọc r " ,= " — Hoa, 8E1, THCS Vinh Tường, Vĩnh Tưởng. Vĩnh Phúc; Bùi Đức Anh, Hiệu, Lê Chân, Hải Phong, 8B r THCS Xuân Diệu, thị trấn Nghèn, Can Lộc, Hà Tĩnh; Di/Ong Lảm Anh, 9A1, THCS Yên Phong. Yên Sách, 5G, Th LÊ ThANn t u THÊ CỜ (Ki 83) L Ẽ THANH TU (Đai triển tưởng Quổc rê ì Trong ô chữ này là tên tiếng Anh của 10 nguyên tố hóa học. Bạn sẽ tìm ra chứ? NGUYỄN NGỌC SƠN (9B, THCS Nguyễn Thượng Hiền, ứng Hòa, Hà Nội) Nam Hà, Hà Nam ng hĩa là gì? vũ KIM THỦY
  • Nam Hã là Dàng Trong Nám 1527 mà Mạc cưốp ngủi vua lẽ. 1543 Nguyễn Kim phủ nhà Lẻ đảnh chiếm Tầy Bồ (Thanh Hỏa). 1545 Nguyễn Kim mãi. Trịnh Kiểm lù con rể Nguyẻn Kim lên nắm quyền, lìm cách loại phe cánh họ Nguyễn. Nguyễn Hũàrg, cor trai Nguyễn Kim xin vào trấn thủ Thuận Hóa nằm 1555. Nãm 1509 Nguyên Hoàng yết kiến vua Lè Anh Tỏng, Trịnh Kiểm. Trnh Kiểm pnong cho Nguyễn Hoàng trấn giữ cả Quảng Nam. Nám 1620 chúa Nguyễn Phưởc Nguyên ngửng nộp thuế cho chính quyỂn Lẽ - Trịnh dàng Ngoài. Tháng 3.1627 chúa Trịnh mang quân đi đánh họ Nguyên. Cuộc chiến kẻo dãi 45 nãm đến 1672. Ti: dây sõng Gianh làm giới tuyến. Mién Bắc sông Gianh lả Dàng Ngoài cùn gọi lả Bắc Hả. Miền Nan’, sõng Gianh là Đáng Trong, càn gọi lả Nam Hà- Danh xưng này Lón tại đến 1736 khỉ Tảy sơn đánh duổỉ nhà Nguyễn 1774 vả diệt nhà Trịnh 1786. Mãi 1 BŨ2 Nguyễn Ảnh mớ thu đuợc haì miền về một mối và đặt nỉèn hiệu Gia Long (là tẻn ghép lừ Gia Định VÓI Tháng Long}. Vậy Nam Hả chỉ Đàng Trong lả phẩn nước Việt từ sóng Gianh trả váo được dùng trong giai đoạn từ 1672 dền 1786.
  • Trường thỉ Hà Nam Dưới Triều Nguyễn dầu tiên cô 6 Irường Ihi, Truồng Hà Nôi thi đến nãm 1879 vả dóng cửa 1882. Mắm 1886 vua Dũng Khảnh cho hợp thí trưởng Nam Dinh vdi trưàng Hà Noi, thi tại Nam Định gọ] là trường Hà Nam. Từđó hai '.rường Hà NỘI Nam Định thành 1 trường thi cho ổến nằm 1915 lả khoá thi Hán hạc cuối củng. Chú ý, lừ 1&Ê6 cả nước cờ 5 trưởng thi; Thừa Thiên, Hà Nam (Nam Đinh), Thanh Hóa, Nghè An và Giá Định. Sĩ tử trưởng Hả trước đây gđm: Hà Nội. Sơn Tày, Tuyên Quang, Hưng Hỏa, Thải Nguyên, Lạng Sdri, Cao Bằng và Bắc Ninh. Sĩ tử trưởng Nam trước đảy gổm; Kam Đ[nh, Hưrg Yên, Quảng Yẽn, Hảì Dưang. (Lưu ý bạn đọc lủc đó tình Hà Nam chưa Cũ mã còn thuộc Hà Nội và Nam Đinh, "hải Sình chưa cõ mà côn thuộc Nam Định, Hái Phòng chưa có mà cồn thuộc Hải Dương). Vậy Hà Nam chì trưũng thí dành cho cả Bẩc Kỷ và thỉtạí Nam Dinh tử 1886 đen 1915.
  • Tình Hả Nam Nãm 1890 một phẩn đít phía nam của lỉnh Hà Nộỉ vả mót phẩn đất phia bắc cùa tinh Nam Đinh đươccắl rã để lệp tỉnh Hà Nam, một tình múi (Chứ Hà là Hà NỘL chữ Nam lả Nam Dinh). Vặy Hả Nam chi tĩnh Hả Nam từ 1 590 dõn 1964 vã tử 1996 đến nay.
  • Nam Hà lả tên một tỉnh trước dãy Nãm 1965 tỉnh Nam Hà đưọc Ihành lập dũ sáp nhập Nam Dịrh và Hà Nam, tĩnh lị là thành phố Nam Định. Nãm 1975 sảp nhập Nam Hả và Ninh Binh thảm lỉnh Hà Nam Ninh, tình lị là Ihảnh phổ Nam Định. Nấm 1991 tỉnh Nam Hà tái lốp do tách Hà Nam Ninh thảnh hai tỉnh Nam Hà, Ninh Bình. Năm 1996 tình Nam Hà tách thành hai tỉnh Nam Định và Hà Nam như cũ- Vậy :ình Nam Hà có hai giai đoạn ton tại lả 1965 - 1975 Vả 1991 - 1996 tổng cộng lá chỉ CÓ 15 năm. Thòi kì đất nưỡc chia cát,, mỗi tĩnh miền sác kếl nghĩa với một tinh miến Nam. VI thế Nam Dinh nay có chợ Mỹ Thù, pnỂ Vỹ Thủ. Hà Nam kết nghĩa vối Biên Hòa nen có trương THPT 8ỉên Hòa và à Bìán Hùa nay có '.rường THPT Nam Hà. Ngảy nay Cỏn Gỏng ty cổ phần Dược Nam Hà thuộc tĩnh Nam Định, trụ sử 415 phố Hàn Thuyên. Nhiều ngưài nhẩm vối Cârg ty cổ phần Dược hà Nam, von iảch ra từ Công ty Dưyc Nam Hà.
  • Hả Nam lả tén dảo thuộc thị xả Quàng Yên, Quảng Nính Dân cư à đây cỏ gốc gác lử Klm Liên, Hà NÔI di khai hoarg lập ẩp lừ đỏi vua Lê ĩ bải Tong 1434. Cá í lẽn Hà Nam Cũ lẽ mang tên Ihơo nghĩa phía Nam của sõng gợi nhô gốc tích vùng quẽ Nam sórg Hong.
  • Ngày nay chỉ càn tên Hà Nam, Nam Hà như sau: Tình Hà Nam (lỉnh lị lả Phủ Lý) Đảo Hà Nam ihuộc thi xã Quảng Yên, Quảng Ninh Cõng ty Dược Nam Hả (thuộc tinh Nam Đinh) Cong ty ũưực Hã Nam (tỉnh Hà Nam) Trường THPT Nam Hà (Biên Hòa, Đong Nai) Đội bóng nữ Phùng Phủ, Hà Nam Ngoài ra còn: Trường THCS Nann Hà, Kiến Ar, Hảl Phòng Như vậy Lỉnh Nam Hả Êhỉ có Irong 15 nám, lừ 1965 đến 1975 và 1991 đến 1996 tĩnh lị lả Nam-Đinh. Bạn đừng nhẩm tỉnh Hà Nam ngảy nay với tỉnh Nam Hà ngày xưa. Câu hỏi; Cầu thơ Nhá nước ba năm mỏ một khoa / Trường Nam thi lẳn vòi trường Hà là cua ai, viết nàn rảo? Đáp: Tự tin I /ổn gốc tự tì Thu mình nhả tại bê đi Trước dám dõng nói li nhi Thầy hỏi như không biết gi Bây giờ mạnh dạn lên đi Thả sâu rỗi nòi to lẽn ngại gì N lắn cái tự ti Biết đâu thành cái tự tĩN bất ngờ Hỏi: Anh Phó ơi! Nẽu giải bài mà được dàng tén trên TTT thì có được nhặn quà khỏng ạ? Mấy dứa bạn em dược dăng tên nhưng không được nhận gì cả. Nhóc Con tò mó (THCS Yèn Phong. Yên Phong. Bắc Ninh ) Đáp: ĐưỢc khen thì ơã khen rối Nếu ghi duợc nhận thì ngồi chò thôi Quà đang mang đến tận nơi Đúng tên đủng tuổi đủng người được trao Hỏi: Em luôn tự ti trong học tập, luôn không chắc chán về những đáp án mình đưa ra. Anh có cách gì giúp em không ạ? HOÀNG TRUNG NGUYỀN (7 A, THCS Đóng Lãm , Tiền Hải, Thải Binh) Hỏi: Ngói cạnh em có một bạn rất hay nói chuyện ríèng. Làm cách nào để bạn ấy đừhg nói chuyện nữa mà chủ ý vào học hơn ạ? NGUYẾN THỊ BẢNG BĂNG (7C, THCS Bạch Liêu. Yên Thành. Nghệ An) Đáp: Giờ ra chơi gặp bạn Nói thật nhiều chuyện di Dốc xong bầu tàm sự Vào giờ chắc bớt đi Nếu vấn còn nói nữa Chắc cũng chỉ thầm thì ANH PHÒ Bạn háy vào website: .7ulm.vnj'hitư-Siich-uiiliiit dể dọc tạp chí Tbán Tuổi thơ bán diện lừ Ikhú , CÁC LỚP 6 & 7 Bải 1(161+162). .111 1 2 3 4 2016 (có 2015 số hạng). Chứng mirh rằng 11 NGUYẺN NGỌC HỦNG (CV. THCS Hoàng Xuân Hãn, Dữc Thọ , Hà Tĩnh) Bài 2(161+162). Cho tam giác ABC vởi 3Â = 6B = 10C. Trẻn cạnh AC lẩy điểm □ sao Cho AD = AB. Chứhg minh rằng BD - AC. THÂN van chương (GV. THCS VÔ Như Hưng, Điện Bàn, Quàng Nam) CÁC LỚP THCS Bầi 3(161+162). Cho p(x) là da thức bậc 4 với hệ số bậc cao nhất bằng 1. Biết rằng P(2013) = 2014, P(2014) = 2015 và P[2015) = 2016. Tinh P(2Q12) + P(2016). TRÁN Vân Hưng (GV. THCS Yên Thanh, Can Lộc, Hả lĩnh) Bài 4(161+1 62). Cho các SỂ thực a, b, c thỏa màn á 1 + b z = 2. Tim gỉả tri lớn nhát, giá trị nhò nhất của biểu thút M-a + b+ c- abc. NGUYỄN ĐỨC TẤN (TP. Hổ Chí Minh) Bải 5(161+162). Đặi ũ 7tỉ = (1:2: S: 7: 10: 14; 35; 70} (tập họp các ưởc của 70). Ta vẽ biểu đâ cho D 7o như hình bén, Tim quy luật cùa biển đó trẽn. Hãy xác định các phần tù của D 21u rồi vẽ biỂL do cho D 21ũ . Bài 6(161+162). Cho lam giác ABC vãi phân giác AD. Gọi p, Q là hai điểm thuộc đoạn thẳng AD sao cho ÀBÌP - CBQ. Gọ E, F thứ tự là hình chiếu vuông góc của p lèn AC, AB. gọi H là hình chiếu vuống gác cùa Q lèn BC và K là hình chiếu vuông gòc của H lêh EF. Chứng minh rang KH là phân giác của BKC. TRẤN QUANG HƯNG (GV. trường THPT chuyên Khoa học Tự nhiên Hà Nội) VŨ KIM THỦY SOLVE VIA MAIL COMPETITION QUESTIONS Transỉated by Nam Vú Thảnh 1(161+162). Let A = -ị I -Ị- 1 4 t ■■■ -I — r— (which canlans 2015 Lerrns). Prove Ihat 2 3 4 2016 11 2(1 61+162). Given a triangle ABC having 3/A - 6/B - 1 0/C. Let D be a point on the side AC such tlìat AD = AB. Prove that BD = AC. 3(161+162). Let P(x) be a 4 [h degree polyromial having a leading coe^cient of 1. Giventhat P(2013)
  • =2014, P(2014) = 2015, and >(2015) = 2016. Find P(2012) + P(2016). 4(161+162). Lel a, b, and c be real numbers such thai ữ 7 I b 2 i G 2 = 2. Find L PHIÊU ĐẤNG Kf THAM Dự CUỘC THI GTQT NÂM học 2016-2017 70 ĩ 14 10 y 14 V 35 i>< ><f 2 5 7 the maximum vglue and the Tiirimum value of the expression M - a + b - c - abc. 5(161+162). Let D ?ũ = (1; 2; 5; 7; 10; 14; 35; 70} (which is a set of Ihe tactors of 70). A graph for D 7C is given beside. Find Ihe pattern of the aữove graph. Determine the elements of ữ 21 g and draw the graph for O 210 - 6(161+162). Given a triangle ABC and its ang le bisectorAD. Let Pand Q be Lhe poinls on AD such thai ZABP = ZCBQ. Let E and F be the orlhogonal projection of p ortũ AC ard AB, respectively. Let H be the orihogongl projection of Q onto BC and K be the o^ogonal projectíon of H onto EF. Prove that KH is the angle bi se c to r of ZBKC. TIN TỨC - HOẠT THĂN TRẠ! THƯỜNG BINH THUẬN THÀNH, BẮC NINH Ngây 15,7-20 ỉ R, Irung dịp kì uiỌin num ngáy Thương bĩnh Liệt sĩ. đoàn cựu chiến binh NXR Giáo dục Việt Nam vả Hội cựu chiến bính quặn Hoãn Kirm. Hã Nội đã dem Trụi thương bLiilì nặng Thuận Thũnh, Bác Ninh, ỏng Nguyền Minh Khung, phụ trách cơ quan văn phông; õng Nguyên Quý Thao, nguyên TBT NXBGDVN: õng Hà Sỹ Chuẩn, Chủ tịch Hột CCD NXB: ủng Vũ Kim Thủy, w RC1Ĩ Tĩặl vũ cáu ihãnh vicn Irrmg đoãn ciã ihỉìni hữỉ. lặng quà các thương binh, Hội CCB cuận đã cỏ nhiitìg Liết mục vãn nghệ lặng các thương hỉnh. FV. TTT GUỘC THỈ AMt LẨN ĐẨU TIÊM Tổ CHÚC TAI VIỆT NAM Sãng ugảy 28*7.2016 lần dâu tlẽn Guộc thỉ Toán học úc (AusLruhĩin Maíhemaỉlcs Conipetthon, AMC) dược tể chức ở Việt Nam tại trường Ngỏi Sao Hà NỘI, Q. Thanh Xuân, Uã Nội. Cuộc thl nãy lán dẩn íitni dược tẩ crhửc uầm 1978- Từ 1 lủ 111 2004, cuộc Ibí dù mỏ rộng cho cả dổi tượng học sỉnh Tỉểu học. Đến nỄLm 2015 đã có hơn 14 triệu lượt thi sinh của hơn 40 quốc gia trẽn thế gỉửi tham gia dự thi. Cuộc thi năm nay dược tổ cMc ữ Việt Nam vơi 5 cấp dộ: cạp đậ 1 (]ơp 3 Vỉi 4). cấp dộ 2 (lớp 5 vá 6J, cẩp liọ 3 lịớp 7 và b)p cấp dộ 4 Pửp 9 vã 10), cấp độ 5 (lẽrp II vá 12). Bài thi của cãc thi sinh SẼ dược chuyến sang ũc đtí chấm bằng máy, Nãm nay rỏ hơn TŨOO Lhi sính tử 30 tỉnh thầnh trong cầ nước dăng kí tham gỉa dự Lhi. Đạl ĐỘNG - GẶP GỠ sử quán ữc tại Hả Nộỉ dã hỗ LrỢ Ban tổ chức Lại Việt Nam trung quã trinh tổ chửc cuộc thỉ. Do bận cũng Lãc tái Ihãnh phố ĩ lổ Chĩ Minh, Ngãi Dại sử ũe tạỉ Việi. Nam dã cữ bù Kim ClcaTỴp Tham tán Giáo dục Klioa hoe của Sử quán đển tham dự Lẽ khai mạc vả phát biểu chão mừng, Cuộc thí Toán học ũc dược tổ chức tại ViẸI. Num sc dộng vịẽn phong trảo học toán tiếng <\nh trong các nhà trường. Đãy cũng lả mọL kẽnli thống tin để các cấp quản lí giáo dục, giáũ vlẽn, các vị phụ huynh, cãc i:m học sinh cũ thể lim hiếu VỀ phương phiip dạy vầ học Toán của ủc. LỂ RA MẮT SÀN PHẨM BIGSCHOOL Chiẻư ngây 1.8.2016. Lẽ ra mắt sán phẩm DỉgSchool dược tổ chức tại trdùng THPT chuyên Hả Nội - Amsterdam, Q- cầu Giấy, Hả Nội. Đến dự cỏ õng Phạn-. Tất Thắng, Phố Chu nhiệm ủy ban Vãn hỏa, Giáo dục. Thanh niên, Thiếu nlèn vả Khí dỏng của Quốc hộ í; TS. Nguyên Vinh Hiển. Thử (rương Bộ Ciịúù dục vủ Đào lạo; õng Nguyền Minh Hồng. Thử trưởng Bộ thõng tin vã Truyền thõng; õng Nguyên Long Hải, Bỉ thư Ban chấp hành Trung Ương Doãn, Chủ tỊcli HỎI dồng Dộỉ Tning Ương; ThS. Vũ Kìm Thúy. Tổng bicn Lặp Eụp chí Toán Tuổi thơ; õng Nguyễn Ngọc Thủy, Chủ tịch HĐQT Cõng ty Co phản Tặp doàn Giảo dục Ẹgome tEgroup); TS_ Lẽ Thống Nhất, Tổng Giảm dốc cỏng ty Cọ phần Trương học lởn Việt Nam, ... BigScliool giiỉp kết nõì gifc\o Yỉẽn và học síiứi trong vã ngoài tLưừc, dỏng LhỡJ dáp ứng nhu cảu của phụ huynh học sinh trong việc Lhco đỏi toãii hạ quá Irĩnh học lập của con cm 151 inh- RigSchaol lã một trường học không hạn chế số lưqng giáo viên vã học sinh. Vởl phương chãm: Dọc ngây ngây, học hảng say, hỏi bUTt ngay, chơỉ rất hay, thi tíếíi bộ, mgng xà hpi học tập l>lgschoo]vn tiứíá hụn lã một dĩa chi dể các em học sinh cố ihể học mà chơi, chơi mã học. Bạn cũng có thể dọc tạp chi Toán Tuổi thơ dỉện tử vã nhlẻu tãi lỉệti bổ ịch khác lrè:n blgschooi.vn Toán Tuổi Ihơ Ị ũ một trong mười lãm đốỉ tác đầu tiẽn của bigschool.vLL TTT Giấy phép xuâì bản: sô 31/GP-BVHTT, cấp ngày 23/1 /2003 của Bộ v&n hỏa và Thõng tin. Mã số: 8BTT161M16. In tại: Công ty cổ phẩn in Công Doãn Việí Nam, 167 Tây Sơn, Đống Đa, Hà NỘI. In xong và rộp lưu chiểu tháng 09 nám 2016. Giá: lOOOOđ NĂM THỨ MƯỜI BẢY ISSN 1859-2740 NÃM HỌC 2016 - 2017 NHÀ XUẤT BÀN GIÁO DỤC VIỆT NAM - BỘ GIÁO DỤC VÀ ĐÀO TẠO 1 V ặ' ỳi ịị^K '>«4 SỆ aií DỊGIl tii ụ» mmn eạ mt YfHn T 1 L • ’ĩ vr Tk. in^K H ri 'ẾPi |AuV f 1 / / Childrens Fun Maths ơournaỉ NHÀ XUẤT BÀN GIÁO DỤC VIỆT NAM - BỘ GIÁO DỤC VÀ DÀO TẠO m Tổng biên tập: ThS. vũ K|M THỦY CHỊU TRÁCH NHIỆM XUẤT BẢN Chù tịch HỘI dâng Thánh viên NXBGĐ Vlệl Nam; MẠC VÃN THIỆN Tầng Giám đến NXBGữ việt Nam: GS.TS.VÙ VẪN HÙNG Phô Tống Giám đoc hỉẽm Tồng bỉẽn lập NXBGO Vlệl Nam: TS.PHAN XUÂN THÀNH Thư kí tà a soạn : T rưỏng ba n biên tậ ọ: NGUYÊN NGỌC HÀN TRẤN THỊ KIM CƯƠNG ỦY VIÊN NGND.VŨHỪU BÌNH TS- GIANG KHẨC BÌNH TS. TRẦN ĐÍNH CHÂU TS. VŨ ĐÌNH CHUẨN TS. NGUYỄN MINH ĐỨC ThS. NGUYỄN ANH DŨNG TS. NGUYÊN MINH HÀ PGS. TS. LỄ QUỐC HÁN PGS. TSKH. VŨ ĐÍNH HÒA TS. NGUYỄN €>ửc HOÀNG ThS. NGUYỄN vũ LOAN NGUYỄN Đừc TẤN PGS-TS-TÔN THĂN TRƯƠNG CÒNG THÀNH PHẠM VÃN TRỌNG ThS. HỒ QUANG VINH TÒA SOẠN Tâng 5, số 3G1 đường Trưàng Chỉnh, quặn Thanh Xuân, Hã Nội Điện thoại (TẻI): 04 356S2701 ũiận sao (Fax): Ũ4.356827Ữ2 Diện thư (Email)] toantuoitho@vnn.vn Trang mạng (VVebSile). //www.loariluoitho.vn NGUYỄN VIẾT XUÃN 391/150 Trấn Hưng Đạo. p. cầu Kho. Q.1, TP. HCM ĐT: 08.66821199, DĐ. 0973 303199 Trị sự - Phát hành TRỊNH THỊ TUYẾT TRANG, VŨ ANH THƯ, NGUYỄN HUYỂN THANH Chế bản: Đố TRUNG KIẼN Mĩ thuật: Họa sĩ TÚ An TRONG SỐ NÀY Dành chõ học sinh lớp 6 & 7 Tổng các chữ sốcủa một số tự nhiên Thái Nhật Phượng Học ra sao? Giải toán thế nào? Bài toán tìm giá trị của biến để giá trị của biểu thức là sốnguyên Trịnh Phong Quang Đo trí thõng minh Sôi nào nhĩ? Nguyễn Ngọc Hùng CửasổAC Myanmar gần và xa Vu Kim Th ủy Nhìn rathếgìổi Để thi Toản và Khoa học Quốc tế IMSQ năm 2Ũ1 5 Trịnh Hoài Dương Com pa vui tính Chì dùng thước Nguyễn Xuân Bình Phããn cùng thám tửsêlõccôc Tờ giấy bí ẩn Đặng Hữu Hoàng Quân Bạn đọc phá! hiện Thèm một bất đẳng thức hình học Tạ Thập Dành cho các nhà toán học nhỏ Sô điểu hòa Kiều Đình Minh Để thi cãc nước AMC2015 Senior Division (Tiếp theo kì trưóc) Đỗ Trung Hiệu TỔNG CÁC CHỮ SỐ CỦA MỘT SỐ Tự NHIÊN THÁI NHẬT PHƯỢNG (GV, THCS Nguyễn Vàn Trồi, Cam Nghĩa, Cam Ranh, Khènh Hòa) Ta kí hiệu S(n) là tổng các chữ số của Sũ tự nhiên n khác 0,
  • Tinh chất với n 5t 0 thì S(n) có một số tính chất sau đây;
  • S(n) = S(10 .rí) (k £ N*). 2 . n = S{n) (mod 3) hay n - S(n) : 3; n = S(n) (rrtod 9) hay n - S(n) : 9.
  • Nếu s(m) s S(n) thì m = n (mod 9) hay m - n : 9.
  • 0 < S(n) < n.
  • n có k chữ số thì 1 0 k_1 < n < 1 0 k và S(n) < 9k.
  • Nếu n < ạa^Ị— a k thì S(n) < {a - 1) + 9k khi âa 1 a 2 ...a k cỏ ít nhất rrìột chữ số khác 9 và S(n) < a + 9k khí aa^— â k =99.. .9. II. Bài tập áp dụng Sải 1. Viết các số tự nhiên liên tiếp từ 1 đến 100 liến nhau ta được số A = 12345. ..99100. Nếu đặt dấu 'V vào giữa hai chữ $ố nào đó của A thì đuạc tong của hai số B và c. Hỏi B + c có chia hết cho 2015 không? Lởi giải. Ta cá S(1 2.. .9) = 45, S(1 01 1 .,,19) = 1 0.1 4 45 S(2021... 29) -10.2+45 S(9091...99) = 10.9 + 45, S{100) =1. Do đó S(A) B S(B + c) = 45.10 + (1 + 2 + ...+ 9).10 + 1
  • 901 (mod 3) B + c không chia hết cho 3,

    Vậy B + c không chia hết cho 2015. Bải 2. Hai số khác nhau đều có 1 00 chữ Số, trong đó cố 40 chữ số 1. 30 chữ sổ 2, 20 chữ Số 3. 10 chữ số 4. Hỏi sô này có thể chia hết cho sô kia không? Lởi giải. Gọi hai sỏ đó là m và n (m > n). Ta co S(m) = S(n) = 40.1 4 30.2 I 20.3 + 10.4 = 200-2 (mod 9). (1) 4 mvàn chia cho 9 đểu dư 2 nên n - 9a + 2 (a € 1*3). Giả sử m ; n =? m = kn (k nguyên và 1 < k < 4) => m = 9ka + 2k. Với 1 < k < 4 thì 2k bằng 4, 6 hoặc 8 =» m chia cho 9 dư 4, 6, hoặc 8 (mâu thuẫn với (m Do vậy m không chia hết cho n nên không thể có số này chia hết cho sổ kia. Bài 3. Xét sổ tự nhièn X. đổi chỗ tùy ý cảc chữ số của X ta được số y. Giả sử |x — y| = 22—2 (n chữ số 2, n Ễ N*}, Tìm giá trị nhỏ nhất của n vả chì ra một cáp số tự nhiên X, y để nhận giá trị đó. Ldi giải. Vì S(x) = S(y) nén |x - y| : 9 => 22—2 : 9 => 2rt i 9 => n i 9. Vậy giá trị nhò nhất của n là 9. Một cặp số tự nhiên (x; y) là (9012345678; 8790123456). Bài 4. Tìm sỏ chính phường n có 4 chữ số, biết n : 7 và S(n) = S(5n). Lài giải. Ta cà n - S(n) : 9 và 5n - S(5n) : 9. suy ra 5n - n - S(5n) + S(n) : 9 => 4n : 9 (vì S(n) = S(5n)). Nên n : 9 mà n : 7 => n : 63 => n = 63k = m 2 => 3 2 {7k) = m 2 => k = 7h 2 , từ đó n = 441h 2 . Mà 1000 < n < 1ODO0 nên 1000<44lh 2 < 10000 => h e (2; 3: 4) =>ne {1764, 3969, 7056}. Thử lại đúng. Bài 5. Cho SỂ nguyên tố p > 3. Biết rằng có số tự nhiên n sao cho p n có đúng 20 chữ số, Chứng minh rằng trong 20 chữ số này có ít nhất 3 chữ số giống nhau. Lài giải. Giả sử trong 20 chữ sổ của p n không cỏ 3 chữ số nào giống nhau, suy ra mỗi một chữ số 0, 1, 2, 3,..., 9 đều xuất hiện đúng 2 lần, Do đó S(p n ) = 2(0 + 1 + 2 + ...+ 9} = 90 I 3 =3- p n : 3 (vô lí). Vậy ta có điều phải chửng minh.

    Bài 6. Có hay không một sô chính phương n mà S(n) = 2004. Lởi giải. Giả sử tồn tại số chính phương n mà S(n) = 2004 sa> S(n) không chia hết cho 9 và S(n) chia hết cho 3 nèn n : 3- vi n là số chính phương nén n : 9 => S(n) : 9 (mâu thuẫn). Vậy không tồn tại số chính phương n thỏa mãn đề bãi. Bài 7. Chứng minh rằng A = 5555- 55 27 1 4n :9. Lời giải. Ta có n chữ số 5 A - 5( Ị 1 1 1 ...1 Ị oo' -n) +9(n +3) n chữ số 1 Vì S(1..,100) =n nên 1 ... 1 00 -n 19, mà 9(n +3) :9. Do đố A i 9. Bài 8. Cho A =9.. .9 (n chữ 50 9). Hãy so sánh S(A) và S{A 2 ). Lời giải. Ta có A + 1 = 10 n A 2 = (A -1)(A t i) +1 =(9l9 -1)(9l9 +1) +1 = 9— 980... 01 (n -1 chừ số 9 và n -1 chữ 50 0) Vậy S(n 2 ) = 9n = S(n). Bài 9. Tim n F N biết
  • n + S(n) - 2000.
  • n + 2S{n) = 2000. cj n + S(n) + S(S{n)) = 60. Lời giải, a) Từ đề bài ta suy ra n s 1999 nên S(n)< 1+9.3 28, do đó n > 2000 - 28 1972. Đặt n = 19ab thì 19ab + 1 + 9 + a + b = 2000 = lia + 2b = 90 => a chắn và do 7 < a < 9 nên a = 8 =$ b = 1 . Vậy n = 1 981 . Thử lại đúng.
  • Vi n - S(n) : 3 nên n + 2S(n) = n - S(n) + 3S(n) : 3. Mà 2000 không chia hết cho 3. Vậy khống có giá trị n nào thỏa mãn yẽu cẩu để bải.
  • Từ đề bải suy ra n < 60 = S(n) <5 + 9=14 nèn S(S(n}} < 9. Từ đó S(n) + S(S(n)) < 23 => 60 < n + 23. Do đó 37 < n < 60. Vì n = S(n) ■ S(S(n)) (mod 9) và 60 = 6 (mod 9) nên 3n - 6 (mod 9) => n = 2; 5; 8 (mod 9). Do đó n Ể {38, 41 . 44, 47, 50, 53, 56, 59}. Thử lại chỉ có 44. 47, 50 thỏa mãn. Vậy n € {44. 47, 50}. Bài 10. Tìm n e N biết S(n) = n 2 - 2016n + 9. Lởi giải. Vì S(n} > 0 => n 2 - 2016n + 9 > 0 => n 2 - 9 > 2016n = n- — > 2016 => n > 2016. n Vì S(n) £n=>n 2 +9 <2017n =>n + -<2017 n n < 2017. Do đó n = 2016. Thử lại thỏa mãn. Vậy n - 2016. Bài 11. Cho a = (2 9 ) 1945 Đặt S(a) = b, S(b) = C- Tính S(c). Lởi giải. Ta có a = (2 3 ) 5335 < 1 0 &835 =>b = S(a)< 9.5835 = 5251 5 => c = S(b) < 4 + 4.9 = 40. Suy ra S(c) <3 + 9 = 12. Ta cỏ S(c) = c = b s (2 3 ) 5835 = -1 {mod 9) => S(c) - 8. Các bạn hãy áp dụng các tính chất trên đề giải các bài tập dưới đáy nhé. Bải 1. Cho S(n) = S(2009n). Chứng minh rằng n : 9. Bài 2. Chứng minh rằng $(999n} = 27 với n = 1 , 2, 3 999. Bài 3. Chứng minh rằng nểu n : 999 thỉ S(n) > 18. Bải 4. Tìm n e N biết S(n) = n 2 - 1 988n + 26. Bài 5. Tim n e M biết n + S(n) + S(S(n)) = 90. Bải 6. Cho a - 4444 4444 . Đăt S(a) - b. S(b) - c. Tính S(c). Bải 7. Chứng minh rằng S(n 2 ) có dạng 9k hoặc 9k + 1 , hoặc 9k + 4 hoặc 9k + 7 {ke N). Bải 8. Cho số’ tự nhièn a sao cho nếu đũi chẻ các chữ số cùa a thì được số b gấp 3 lần số a. Chứng minh rằng b : 27 Bài 9. Chứhg minh rằng trong 1900 số tự nhiên liên tiếp cố một số có tổng các chữ sô chia hết cho 27. Bầi 10. Từ các chử số 1, 2, 3, 4, 5. 6, 7 lập tất cả các số tự nhiên có 7 chữ số, trong đỏ mỗi chữ số đẽu có mặt một lần ò mỗi số. Hòi có tón tại hai sổ mà số này chia hết cho số kia không? BÀI TOÁN TÌMJGIÁ TRỊ CỦẠ BIÊN DỂ GIÁ TRỊ CỦA BIÊU THỨC LÀ số NGUYÊN TRỊNH PHONG QUANG (GV. THCS Quảng Lạc, Nho Quan, Ninh Bình) Trong các kì thi chọn hạc sinh giỏi lớp 9, thi tuyền sình vào THPT, chúng ta thường gặp bài toàn rút gọn biểu thức chứa biến dưởi dấu càn bậc hai và tim các gia trị nguyên của biển (hoặc tìm cãc giá trị của biến) dể giá tặ cỏa biểu thức là sổ nguyên. Bài viết này chúng ta sẽ xét một số dạng toàn đó qua các vi dụ minh họa. • Nhận xét: vội n là số nguyên dưdng và n không là sỏ chính phương thì v'n lả số vô tỉ. Chíừỉg minh. Giả sử %/rĩ là số hữu tỉ. Đặt <Jn = ^-1 a,b G N; (a r b) -1;b *0. b Suy ra a 2 = rvb 2 . Vì n lã số nguyên dương, n không là số chính phương nên tón tại một sổ nguyên tố p nào đó là ưởc của n và không là ước của n 2 . Suy ra a 2 : p => a : p => a 2 : p 2 => b : p (mâu thuẫn với giả thiết {a, b)= 1). Do đó điều giả sử là sai. Vậy y'n là số vô tì. VI dụ 1. Cho biểu thức £ 2Vx -9 sfx + 3 2-v X + 1 x-5yx- 1-6 'Ix-2 3-Vx với X > 0; X 4; X 9.
  • Rút gọn biểu thức E.
  • Tìm các giá trị nguyên của X để gié trị của biểu thức E nguyên. v'v-1 Lởi giải, a) Biển đổi ta được E - ■ Vx -3
  • Ta có = Vx + 1 VĨ-3 = 1 + 4 Vx -3' • Nếu X không là số chính phương thì t/x là số vô tỉ. Suy ra E là sỏ' vô tỉ (loại). • Nếu X là sổ chính phương thì Vx là $ổ nguyên nên để E có giá trị nguyên thì 4i(Vx -3), Mà Vx - 3 >-3 nẻn (Vx -3)e{-2; -1; 1;2;4) => Vx f= {1; 2; 4; 5: 7} => X E {1; 4; 1 6; 25; 49). Kết hợp với DKXĐ ta được X - 1; 16: 25; 49. Ví dụ 2. Cho biểu thức A = 6x + 4 , 3V3x 3 -8 J 3x 3x + 2ý3x + 4 Y Ả 1 + 3ỹ'3x 3 I+Vãx \ J
  • Rút gọn biểu thức A.
  • Tìm các giá trị nguyên của X để biểu thức A nhận giá trị nguyên. (Đế thi chọn học sinh giỏi lớp 9, ThCỉa Thiên - Huế năm hạc 20Ũ6 - 2007) Lài giải. ĐKXĐ X >0; 3 . , 3x-2v'3x+1 3x-3 a A - — — --2. V3X-2 Vãx - 2
  • • Nêu 3x = 3 hay X = 1 thì A = -2. • Nếu X 1 và 3x không là sồ’ chinh phương thì i/3x là số vô ti, Suy ra A là số vỏ tỉ (loại), • Nếu 3x là sđ chính phương thì V3x lả số nguyên nên để E có giá trị nguyên thỉ {3x-3)i(^3x -2) =>({>ỗx -2)(V3x +2) +1) ỊẬx -2) => 1!(Jãx -2) =>(V3x -2) e{ -1; 1} => n/3x E (1; 3) => 3x € {1; 9} =>x eA 9). 3 Vì X là sô nguyên và so với ĐKXĐ ta được X 9. Vậy X E ( 1 ; 9 } Ví dụ 3. Cho biểu thức = Wx 2Vx-3 Vx+1 x + 3Vx-4 Vx + 4 ớx-1
  • Rút gọn p.
  • Tìm các giá trị cũa X để p nhận giá trị nguyên. Lài giải. ĐKXĐ X > 0 ; X * 1 .
  • p _7-3 v'x + 4
  • 19-3(^41 = 19 3> 3

    -Jx+4 Vx+4 Vx + 4

    _ 19 19 Ta có x>0=í ýx+4>4=> —ị — — <-)- Vx+4 4 „ 19 7 „ - , 7 ->p <-3 - —r=-. Suy ra -3<p < -Ẹ.

    4 4 4 Vì p là sỗ nguyên nên p e {-2; -1; 0; 1). I3T Gam số NÀO NHỈ? Bải 1. Hãy tìm quy luật và viết tiếp một số hạng của dãy số sau: 1.11,1, ĩẽs : 144 1 Ĩ26 ’ Ũ2 ' ' Bài 2. Cho dãy số 70; 1 61 ; 184: ... trong đó mỗi số hạng của dảy số bằng tổng cảc chữ số của số hạng đứng trưâe số đó nhân với 23. Hỏi số hạng thứ 2016 bằng bao nhiêu? NGUYỄN NGỌC HỪNG (GV. THCS Hoàng Xuân Hãn, Đức Thọ, Hà Tĩnh) IMrm ĐÔ BẠN BIẾT HÌNH NÀO, sô NÀO? {TTT2 số 159+160) Nhận xét. Bài 1 chì cần để ý đến số ngôi sao trong mỗi hình. Một số bạn nêu quy luật tĩ số giữa sổ ngôi sao và số chấm tròn, cũng đủng nhưng không phải dấu hiệu đặc trưng. Bài 2 nhiều bạn diễn dạt chưa rõ khi gọi tên các số ở đỉnh của tam giác. Quy luật. Bàí 1. Số các ngói sao trong mồi hình A, c, D là số chẵn, còn số ngòi sao trong hình B là so lẻ. Vậy hỉnh B không phù hdp vãi các hình còn lại. Bài 2. Ở mỗi hình, số nằm trong tam giác bằng tích của số ở đỉnh phía trên và số ả đỉnh phía dưởi bèn trái; số ở đĩnh phía dưởi bên phải bằng tổng của số bén trong tam giác và sổ ở đĩnh phía dưới bên trái- Theo quy luật đó, dấu ? bên trong tam giác là số 24 (= 3-8), dẩu ? ở đỉnh phía dưới bẽn phải là sổ 32 {= 24 + 8). Xin trao thưởng cho các bạn giải tết ỊỊ hi ; cả hai bài: Nguyễn Tiến Phong, 7A1,

    THCS Yên Phong, Yên Phong, Bắc Ninh; Mai Thanh Tâm, Đàữ Ngọc Hải Đăng, 7A, THCS Lý TỰTrọng, Bình Xuyên, Vĩnh Phúc; Triệu Hồng Ngọc, 7A3, THCS Lâm Thao. Lâm Thao. Phủ Thọ; Nguyễn Vãn Thanh Sơn, 8/1, THCS Nguyễn Khuyến, Đả Nắng. Các bạn SAU dược tuyên dương: Bạch Bùi Nguyệt Anh. 6D. Vũ Ngọc Ánh Tuyết, 6B, THCS Vĩnh Tường, Vĩnh Tường, Vĩnh Phúc; Dương Thị Hổng . 7A3, THCS Yên Phong, Yên Phong, Bắc Ninh: Trần Sỹ Hoàng, 8C, THCS Hoàng Xuân Hãn, Đức Thọ, Hả Tĩnh; Thần Hùài Thương, 717, THCS Võ Như Hưng. Điện Bàn, Quảng Nam. NGUYỄN XUÂN BÌNH Ị 121 49 9 1 , 11^ Suy ra xe 225;-;-;-^ {thỏa màn ĐKXĐ). 4 9 1 6 J Bài tập Bải 1 . Cho biểu thức sau với X > 0. X 4: ( 2 | 3 5n/x- 7 \ 2vx*3 "[ì/x-2 2x-3x/x-2
  • Rút gọn biểu thức A.
  • Tim X để A có giá trị là một số nguyên. (Đề thi tuyển sinh lớp 10 THPT chuyên Thái Bình Bải 2. Cho biểu thức D x 2 -v'x 2x Wx 2(x -1) , B = — — p- -L= — +—7- — — với X > 0, X 5* 1 . x + Vx+1 Vx vx -1
  • Rút gọn B. 2v'x 5x -10>/x k) Tl m x ^ kiểu D = Tlhận giá trĩ nguyên. Bài 3. Cho biểu thức c = vx , — với X > 0. x-Vx +1 nảm học 2014 - 2015) Tìm tất cả các giá trị của X để c nhận giá trị nguyên. ® m m. LỜI GIẢI có ĐẸP KHÔNG? Bài toán. Cho tam giác ABC vuông tại A có đường cao AH. So sảnh 5AB + 3AC và 5AH + 3BC. Một học sinh dã giải như sau: Ta có AB AC = AH.BC (= 2S ABC ). 3AC - 5AH AC BÕ „ AC AH Suy ra = — ỉr BC AB lJA AC . 3AC Mà <1 nen BC 3BC 3BC-5AB 5AH 5AB Suy ra 3AC - 5AH < 3BC - 5AB. Do đó 5AB 4 3AC < 5AH 4 3BC. Các bạn thấy lời giải trèn có đẹp không? NGUYỄN ĐỨC TẤN ( TP. Hồ Chí Minh) mmm PHƯƠNG TRÌNH có NGHIÊM KHÔNG? (TTT2 sô 159+160} Lời giải sai ở suy luận x 2 y > 0 <=> y > ũ, vi khi X = 0 thỉ bất đẳng thức x 2 y > ũ xảy ra với số y tùy ỷ. Lời giải đúng cển xét hai trường hỢp. • THI: X = 0 thi phương trình có dạng y f 1 = 0, suy ra y = - 1 . • TH2: X 0 mà x 2 y > 0 thi hoác y = 0, hoặc y > 0. Với y = 0 thì phương trình cò dạng X 2 4 1 = 0 nên

    phương trình X 2 + 2y'x 2 y 4 y 4 1 - 0 vó nghiệm.

    Vói y>0thìy + 1>0 nên phương trình X 2 4 2i/x 2 y + y + 1 = D vò nghiệm. Cũng có thể glảl như đế ra khi X khác ũ. Phương trinh cớ nghiệm duy nhất là (x; y) = (0: -1 ). Nhện xét. Cỏ bạn sai khỉ viết X 2 4 2v'x 2 y 4 y = (X + Vỹ) z với y > 0. ... t Các bạn sau được thưởng: Lê Thị Phương Linh, 7B. THCS Vĩnh Tường! Vĩnh Tường. Vĩnh Phúc; Nguyễn Tiến Phợng, 7A1, THCS Yên Phong, Yên Phong, Bẳc Nính; Lương Thiện Quang, 9A2, THCS Lè Danh Phương, Hưng Hà, Thái Bình: Dương Văn Minh, 9A, THCS Đặng Thai Mai, TP. Vinh, Nghệ An. Các bạn sau dược khen: Lé Đức Thái , 3A2, Chu Thị Thanh, Chu Văn Việt, Trần Hống Quý, Lê Ngọc Hoa, Phùng Thị Khảnh Linh, SE1, THCS Vĩnh Tường, Vĩnh Tường, Vĩnh Phúc; LƯU Thị Phương, 8A1, THCS Từ Sơn, TX. Từ Sơn, Bẩc Ninh; Đổ Minh Hiếu, 9A2, THCS Lẻ Danh Phương, Hưng Hà, Thái Binh. ANH KÍNH LÚP CÁC BẠN ĐƯỢC THƯỞNG Từ trài sang phải: Lé Thị Xuân Thu, Chu Thị Thanh, Lưu Thị Phương, Mai Ánh Quýnh. CỬA SỔ MYANMAR GẦN VÀ XA VŨ KIM THỦY AC ià từ viết tắt của Cộng đồng ASEAN bằng tiếng Anh (ASEAN Commưnity). Cộng đống ASEAN thành iệp chính thức từ 31, 12.2015. Năm 2016 này tạp chi Tùán Tuổi thơ mỏ chuyên mục Cửù sổ AC để bạn đọc hiểu hơn vê vùng đẩt, con ngƯÒ! của 10 quổc gia với 625 triệu dãn.
  • Gần Myanmar rất gần với chúng ta bởi chỉ có 3 giờ bay là tối. Từ Hà Nộỉ, máy bay qua bầu trài Laos và Thailand rối hạ cánh ở Yangon, thủ đô cũ của Myanmar, thành phố cảng. Khi chúng tôi đến là vừa hết mùa khô nóng. Khí hậu Myanmar đẩu tháng 7 không khác lắm vỏi Hà Nội. Những cơn mưa đến bất chợt rồi tạnh nhưthời tiết phương Nam nước mình. Từ Bắc xuống Nam, Myanmar ờ gẩn như tương đương vĩ độ với nước ta. Thủ đô cũ Yangon ngang vĩ độ VỚI tình Quảng Tr[ cửa nước mình. Một bất ngd lớn với đoàn chúng toi là các món ăn ỏ nưởc bạn gần giống với các món của Việt Nam và rất ngon. Tuấn, ngưài hướng dấn đoàn cho biết thêm số đông các đoàn khách nước mình sang đéu khen món ăn ở nưởc bạn là ngon. Phô xá của Myanmar khá giống với khu phố Tây có từ Pháp của Hà Nội, Sài Gòn. Đường phố nhiều cây xanh như đường phố quận Ba Đình. Xe khách gợi nhớ thửí chúng ta đang ở thời kì bao Cấp. Xe cũ, chở đông ngưởi chen chúc, không điều hòa. Phụ xe dứng ở cửa mở san sàng làm dâu hiệu cho xe dừng, đón khách. Myanmar còn gần vỡi Việt Nam ủ văn hóa với rất nhiều chùa thở Phật. Gần vói Nepal và India, Myanmar mới thực là đất nước chùa tháp. Vô vàn tháp Vàng chĩa lẻn trời xanh. Một nét gần nữa là giữa trung tâm Yangon, một khu siêu thị, ngân hàng, văn phòng và nhà ở của Hoàng Anh Gia Lai mới mọc lên. Nóc tòa nhà có chữ VIỆT NAM màu đỏ nổi bật. Nhiểu chiếc xe lam chở khách trên đường phố gợi lại hình ảnh Sài Gòn ngày mới giải phóng
  • Xa Xa đáy không phải xa vể địa lí mà vì còn ít ngưài biết về Myanmar. Rất nhiều người Việt mình đã đi Singapore, Thailand, Campuchia, Laos... Cùng ở Đỏng Nam Á nhưng Myanmar giờ mới thành điểm đến của người Việt, đến tham quan vá đến đầu tư Người Myanmar trông hiền hành nhưng vẫn có nét đặc truhg khác lạ khi đàn ông có trang phục như mặc váy, đi dép xỏ ngón và bỏm bẻm nhai trẩu. Đàn ông Myanmar hầu như khống hút thuốc lả như ở nước ta. Lạ là họ nhổ nước trầu đỏ quạch ra vỉa hè và cả ra phố khi đang lái xe. Trầu ăn không có vỏ và vò cau, chì có lõi cau và vòi cùng lá trầu nên không có bã trẩu. Lại nói chuyện đi lại. Tuy đường sả Myanmar còn lạc hậu, xe lam, xe khách cũ kĩ nhưng thủ đỏ Yangon không cho xe máy hoạt động. Ra ngoại thành và nông thôn xe máy mới xuất hiện Lạ nhất là xe đi theo luật chạy bên phải nhưng đa số xe lại có tay lái đặt bẽn phải (ta gọi là tay lái nghịch). Vậy nên khách xuống xe phải đi vé bên giữa đường, Thưởng có phụ xe mà cửa, đứng giữa đường cho khách được an toàn. (Còn tiếp) LỜI GIẢI ĐỀ THI CHỌN ĐỘI TUYÊN HỒNG KÔNG NAM 2010 Dự THI OLYMPIC TOÁN QUỐC TÊ' (VÒNG 1} (Tiếp theo kì trước) MAI VŨ (St/U tầm, dịch và giới thiệu) Ta thây tam giác đã cho có các cạnh 1 8, 24, 30 là tam giác vuông. Do vậy trung điểm o của cạnh huyền mỗi tam giác là tâm đưàng tròn ngoại tiếp của các tam giác đó và bán kính đường tròn nội tiếp là (18 ị 24- 30) 2 = 6 Ta gọi tam giác đã cho là ABC vối Ắ < B < c = 90°, AB = d (hình vẽ). Đường kính DE của đường tròn đi qua trung điểm của AC tại K. Do vậy, OK là đường trung binh của AABC suy ra CK = 12, mà đường kính dường tròn nội tiếp tam giác ABC bằng 12 nên OK là tiếp tuyến của đường tròn nội tiếp. Từ đó BC // DE. T ương tự cỏ FE // BA. Tam giác thứ hai là DEF. Dễ dàng thấy nhũng phần thuộc 2 tam giác (là 6 tam giác được tô màu) đóng dạng với iABC. □iện tích AABC la 18 X 24 ; 2 - 216. Ta tính diện 3 vùng tô màu tại A, B, c. Tại Atam giác KOA cạnh 9, 12, 15 có diện tích bằng 216 : 4 = 54. Tại B tam giác BMN cạnh 6, 8, 10 cỏ diện tích là 6 X 8 : 2 = 24 (vì BM = BO - MO = 1 5 - 9 = 6). T ương tự như vậy, tại c là tam giác có canh 3, 4, 5, có s = 3 X 4 : 2 6 (vì CP = CK - PK - CK - MN = 12-8 = 4). Vậy phần diện tích cẩn tìm là 216 - 54 - 24 - 6 = 132. 1 5. Gọi s là trung điểm của AC. Khỉ đó R e BS với BR : RS = 2 ; 1 (vì R lã trọng tâm của AABC), Ta có BS = 3 (S là trung điểm của AC và là tâm đưàng tròn ngoại tiếp AABC) ra RS = 1. Dặt AR = X, CR = y A p B Áp dụng định lí Cosin trong iARC, AARS, ACRS ta có 36 = X 2 4 y 2 - 2xy cos 1 50° = {3 2 4l 2 -2.3.1.COSRSÂ) 4 (3 2 4 1 2 - 2. 3. 1 . cos RSC) 4 v&xy = 20 4 %/3xy (vl cos R$Ầ = —CCS RSC) =>xy %■ Vậy Sabcd 2 ~Ỉ3
  • Biến đổi biểu thức vế trải của phương trinh: 2(1 00x 2 + 260x I 169)(5x 2 + 13x + 8) =1. Đặt u = 5x 2 + 13x 4 8, khi đố phương trình trở thành 2{20u 4 9)u = 1 =>■ u = -Ị- hoặc u = — ị 5x z 4l3x 4^- =0 (1) 20 2 20 hoặc 5x 2 4l3x 4^ =0. (2) _ ừ (1) có pq từ (?) có rs 100 10 Như vậy pq 4 rs là số thực. " 159 17 329 Đáp án là = — r— . 100 10 100 ĐỀ THI TOÁN VÀ KHOẠ HỌC QUỐC TẾ IMSO NĂM 2015 PHẦN CÂU HỎI CÓ CÂU TRẢ LÒI NGẮN TRỊNH HOÀI DƯƠNG (GI/ THCS Giảng Võ, Bù Đinh. Hà Nội) Sưu tầm và giói thiệu MAI VŨ (dịch)
  • Anne hỏi tuổi thầy giáo của mình. Thầy trả lời "Bây giờ tuổi thầy là một số chính phương nhưng sau ngáy sinh nhật của tòi sẽ là một số nguyên tố* Giả sử, tuổi của thẩy nhỏ hơn 65 và lớn hơn 20 thì bây giờ thầy bao nhiêu tuổi?
  • Cho biết 240384x234 = 56090256. Tính giá trị cùa a + b.
  • Nếu ta đạt cả 3 phép toán +. X vãi các cách khác nhau vào chồ giữa các số của biểu thức 5 — 4 — 6 — 3, mãi chỗ có gạch nối ghi một phép toán. Sau mỗi cách đặt biểu thức mang một giá trị, hây tính giá trị lân nhất trong các giá trị đó.
  • Trong hình vẽ dưói đây, hỉnh bát giác đều ABCDEFGH và hình lục giác đểu UKLMN có cùng tâm 0 sao cho AB ti IJ. Biết CBJ 56°, tính BJK, theo đơn vị đô. B A
  • Một bé trai có một tách trà và một bé gái cỏ một cốc thủy tinh rống cùng thể tích vửi tách trà đỏ. 1 Bước đẩu tiên, cậu bé rót nước trà từ tách sang cốc. Bước thử hai, cô bé rót nước trà từ cốc sang tách. Bước thứ ba, cậu bẻ rót — nước trà từ tách 4 sang cốc. Bước thứ tư, cô bé rót -ị: nưỏc trà từ cốc sang tách. Tiếp tục rát như vậy, sao cho sau mối bước, mẫu số tàng thêm 1. Tính phần nưốctrà còn lại trong tách sau bước thứ 13. Ệ. Một hội đổng gom 5 thành viên, người chủ tọa ngồi cố định một ghế của bàn tròn. Cố bao nhiêu cách để 4 người còn lạì cùng ngồi bàn đó nếu ở đó chì có đúng 8 ghế? (Kì sau đảng tiếp) E5T 17. Theo giả thiết Va 2 + 3ab + b 2 -2a-2b + 4 a b = — . ab + 4 Biến đổi a 2 4 3ab + b 2 - 2a - 2b + 4 = (a - 1 ) 2 + {b - 1 Ỹ 1 3ab + 2 > ữ với mọi a, b. Ta thấy a*b xác định và dương khi a, b dương. Khi a là sổ dương, ta có . ýa 2 + 6a + 4-2a-4 + 4 y'(a 4 2) 2 _ 1 “ 2a + 4 2(3 + 2) - 2
  • Đặt z = vx 3 + 20x = 3/x 5 -2Qx. Khi đó ta có z 5 = X 3 4 2ŨX và z 3 = X 5 - 20x. Cộng theo vế ta được z 5 + z 3 = X 6 + X 3 =» z 5 - X 5 4 z 3 - X 3 = 0. Sau khí biển đổi ta được 0 - ịz - x)(z 4 +Z 3 X + Z 2 X 2 - zx 3 + X 4 + 7 + zx + X 2 ). {)

    Vĩ X Ỷ- 0 nên z ¥= 0. Ta có

    2 2 z +zx + >r T zj và z 4 4 Z 3 X + Z 2 X 2 + zx 3 + x^ xf 2Í „ 4 1 2 2 „ rt z + - 4 rr X i 9 ■ 4 ívr > 0. V 2 ) l 2 ) 2
  • z Từ đỏ suy ra thừa sổ thứ hai trong vế phải của () luôn dương, dơ đó z = X nên X fx 3 +2ŨX => x(x 2 -4)(x 2 -5} Q=>x ±siẽ. Vậy tích các giá trị của X lã -5. ĐỂ THI CHỌN HỌC SINH GIỎI MÔN TOÁN LỚP 7, QUẬN 9, TP. HỒ CHÍ MINH Năm học: 2015- 2016 Thời gian làm bài: 120 phút (không kể thòi gian giao đề) 2 2 y + z Gài 1. (4 điểm) 2 2 2
  • Tìm X, y, z biết rằng \ + \ + 5
  • Cho A = 333 444 và B - 444 333 . Hảy so sánh A và B. Bải 2 . (6 điểm) — 2xy
  • Cho hai sô X và y khác 0 thỏa mãn 3x - y - 3z và 2x H- y - 7z. Tính N = — . X + y
  • Tim X, y, z biết rằng X và y tỉ lệ nghịch với 3 và 2; y vả z tỉ lệ nghịch với 4 vả 5 và 3x 2 -y 2 -t-z 2 = 1971 . b-c + 1 a + c + 2 ạ + b - 3 Bải 3. (4 điểm) 1 . Tim ba số a t b, c biết rằng a b c a + b + c
  • Một lâp học được gắn 10 bóng đèn, mỗi bóng đèn cố còng suất định mức 40 w và hai quạt trần, mói quạt trần có công suất định mức 100 w. Biết rằng 1 kW = 1000 w và mỗi tháng lởp đỏ chì học 26 ngày.
  • Nếu lỏp học đó sử dụng tất cả các thiết bị trên trong 8 già mẽi ngày. Hỏi mỗi tháng lớp học đỏ tiêu thụ hết bao nhiêu kW.h điện?
  • Để tiết kiệm điện, lởp học chỉ sử dụng bỏng đèn trong 3 giờ và quạt trần trong 8 giờ mồi ngày. Hỏi mỗi tháng lâp đó đã tiết kiệnn được bao nhiêu kW.h điện? Bài 4. (5 điểm) Cho tam giác ABC có ba góc nhọn (AB <AC). Kẻ BE 1 AC tại E và CF 1 AB tai F (E £ AC, F Ẻ AB), BE cắt CF tại H,
  • Chứng minh AEF = ABC.
  • Chứng minh HA + HB + HC < ^-(AB - BC CA), V Bài 5. (1 điểm) Có 4 đồng tiền với 3 đổng tiển thật có khối lượng như nhau vả 1 đổng tiền giả có khối lượng khác. Hãy nêu ra cách tìm đồng tiền giả chỉ với hai lần cân? (cân đĩa và không có quả cân). Hãy giải thích? HẼtạng dận liTậi dỄíkiitruớữ LỜI GIẢI DỂ THI LỚP 7 CÂU LẠC BỘ TOÁN QUẬN HOÀN KIẾM, HÀ NỘI Năm học: 2015-2016 Bài 1 . a) Ta cớ < 1 Ý 1 ._■»/' 1 Ỷ M= 4 :-4-12 3 -4 ị =1-1 = 0. ỊeJ 36 ÍJ2j
  • Sau khi biến đổi B ta được B =(1-h2 2 )[l + 2 4 +2 fl +... + 2 2016 ]=5.A. A B 2 5' Bàí 2. a) Để áp dụng tính chẩt dăy tì số bằng nhau ta đặt X = 2k y =3k z - 4k Do đó 8k 2 + 27k 2 - 64k 2 - -116 <= -29k 2 - -116 <íij k 2 = 4 k = ±2. ỉ=ỉ=ị=k: 2 3 4 • Với k = 2 thì X - 4, y = 6. z - e. • Với k - -2 thì X - -4, y - -6, z - -8.
  • Nhận xét |a + b| > |a 4 b|. o Đẳng thức xảy ra « ab > 0. Áp dụng () ta có 2 = 2x+- +- — — 2x

2X + -4--2X 2 2 2 2 Đầng thức xảy ra khi và chỉ khi Bài 3. Gọi X (đổng) là sốtiển phải trả cho 1 m^nưâc (x>0). Theo bài ra ta có (170 - 154)x + (187 - 170)x + (202 - 187)x = 480000 => X = 10000. Từ đó ta tính được $ổ tiền phải trả trong tháng 10 là 160000 (đổng), tháng 11 lá 170000 (đong), tháng 12 là 150000 (đồng). Bải 4.

  1. Dế dàng chứng minh được AHBM - ÀKCN (cạnh huyền - góc nhon). Suy ra HM - KN. Từ đó ÃHMI = AKNI (cạnh góc vuông - góc nhọn). => IM - IN. Vậy I là trung điểm của MN, A
  2. Vì Pl là đưàng trung trực của MN nên MP = NP. Ta có AP là dường phân giác của góc BAC trong tam giác cên ABC nên AP là đường trung trực của cạnh BC => BP - CP. Vậy ABMP = ACNP (c.c.c). Suy ra PMB = PNC.
  3. Dễ dàng chửng minh được AABP = AACP, suy ra ÃBP = ÃCP, Từ ABMP - ACNP suy ra MBP - NCP. Từ trẽn suy ra NCP = ẤCP = 90° (2 góc kề bù bầng nhau) => MBP = 90°. Từ đó PB 1 AB. suy ra p cố định. Bài 5. Đặt p = — . n 2 -2016 Nhận xét: Để P chưa tối giản thì — chưa tối giản. p 1 Để phân số — chưa tối giản thi n + 43 vả 167 phải p có ước chung d 1. Sổ 1 67 là số nguyên tố Vặy các số tự nhiên n cấn tìm có dạng n - 1 67k - 43 với ke (1; 2; 3;... ; 12}. Giải toán quã thu: Bái 1(159+160). Cho da thức í(x} = ạx 2 - bx + c với a, b r c là các sế nguyên và a khác 0 sao cho f(9) chia hết cho 5 và f(5) chia hết cho 9. Chứng minh rằng f(104) chia hết cho 45. Lởi giải. Ta cò f(x) = ax 2 - bx + c nên f(5) = 25a - 5b + é; f{9) = S1a -9b + c; f{104) - 10816a-104b + c. Via, be Znênf(104)-f(5) = 1D791a-99b : 9. (1) Mà f(5) ; 9. (2) Từ (1) và (2), suy ra f(1C4) i 9. (3} Xét f ( 1 04) - f(9} = 10735a - 95b : 5. (4) Mà f(9) : 5. (5) Từ (4) và (5), suy ra f(104) : 5. (6} Mà ƯCLN(5, 9) = 1. (7) Từ (3), (6) và (7) suy ra f(1ũ4) ; 45. Nhận xét. Đây là bài toán về đa thức rất cơ bân, nhiều em tham gia giải và giải đúng. Xin nẽu tên một số em trình bày rõ ràng và đẹp hơn: Đào Vàn Chiến, Phạm Minh Hải, Vũ Minh Khái, Trần Anh Tú, ÊA3, Nguyễn Thu Hương, Nguyễn Đức Tẳn, 7A3, THCS Lâm Thao, Lâm Thao, Phú Thọ; Tn/ơng Thị Thu Lan, 7A1, THCS Yên Phong, Yên Phong, Đắc Ninh; Hoàng Thị Phương Anh, Lê Hoảng Quỳnh Dương, Nguyễn Minh Tú, 7C, THCS Nhữ Bá Sỹ, Bút Sơn, Hoầng Hỏa, Thanh Hỏa. PHÙNG KIM DUNG Sái 2(159+160). Cho tam giác ABC vuông tại A. Kẻ AH vuông góc với BC tại H. Trên tia đối của tia HA lấy điểm M sao cho AH = 3HM. Trên cạnh AC lấy điểm N sao cho AC = SAN. Tính số đõ . Lởi giải. Từ giả thiết AC - 3AN. suy ra S AHC - 3S AHN; . () Kẻ NP 1 AH (p e AH) thì từ () ta có CH - 3NP (theo công thức tính diện tích tam giác), Âp dụng định lí py-ta-go vào các tam giác vuông AHC va APN ta co AH 2 = AC 2 - HC 2 = (3AN) 2 - (3NP) 2 = 9(AN 2 - NP 2 ) = 9AP 2 . Suy ra AH = 3AP. Ta lại CÓAH - 3HM nên HM - AP, từđóAH - PM. Âp dụng định lí Py-ta-go ta có BN 2 = AB 2 + AN 2 = (HB 2 + AH 2 ) + (AP 2 + NP 2 ) = (HB 2 + PM 2 ) + (HM 2 + NP 2 ) = (HB 2 + HM 2 )
  4. (PM 2 + NP 2 ) = BM 2 + MN 2 . Thèo định lí py-ta-go đào thì ABMN vuông tại M, hay BMN = 90°. Nhận xét. Các bạn sau có lời giải tốt: Trần Hải Nam, Khổng Doãn Hưng, Phạm Thùy Linh, Triệu Hồng Ngọc, Lê Thị Phương Lan, Lê Hổng Anh, Nguyễn Thu Hương, Vũ Ngọc Ảnh, Triệu Thị Hóng Ánh, Nguyễn Vũ Hà, Nguyễn Đức Tản, 7A3, THCS Lâm Thao, Làm Thao, Phú Thọ; Lẽ Hồng Nhung, 7A, THCS Vĩnh Yên, TP. vĩnh Yẽn, Vĩnh Phúc; Vũ Khảnh Huyền, 7A, THCS Vĩnh Tân, Vĩnh Lộc, Thanh Hóa; Trịnh Hoàng Anh, 7D, THCS Xuân Diệu, Can Lộc, Hà Tĩnh: Trương Nhật Nam, 7/1, THCS Kim Đổng, Hội An, Quảng Nam. Đài 3(159+160). Tìm các số nguyên X, y thỏa mãn Lỏi giải. ĐKXĐ: 29x + 3 > 0, 4y 2 + 4y - 1 > 0. • Chủ ý: Một số chính phương không có dạng 4k+ 3 (ke N). Vì 4y^ + 4y- 1 =4(y 2 + y- 1) + 3 nên 4y 2 + 4y- 1 khủng là số chính phương vãi mọi sổ nguyên y, tức là 4y 2 +4y-1 là số vô tỉ. Từ giả thiết suy ra ý29x+3 =ý'4y 2 +4y-1+2x--T (1)

    5 Bình phương hai vẽ của (1) ta được 29x + 3 = 4y 2 + 4y-1 + 2| 2X-Ạ-1 X

    4y 2 +4y-1+Ị2x-~1j .(2) Với X, y là sồ nguyễn thi 4y 2 +4y-1 là số vỏ , i V, * r . I Ị . L nên 2x- -1 = 0. (3) ■ tì và tất cả các số hạng còn lạl trong (2) là số hỉ/u tỉ .21 5 Thay vào (1 ), suy ra 29x - 3 <-> 4y 2 + 4y - 29x - 4 - 0. (4) 7y + 5 4y 2 + 4y - 1 Từ (3), ta cố X = ■ 10 Thay vào (4) vả biến đổi, ta được (y - 5){40y + 37) = 0, 7 5 + 5 Vt y là sổ nguyên nèn y = 5, từ đó X = = 4. Thử lại thì (x r y) = (4, 5) thổa mãn ĐKXĐ và thỏa mãn phương trinh đã cho. Vậy (X, y) = (4, 5). Nhận xét. Các bạn sau đây có bài giải tốt: Nguyễn Thu Hiền, 8A3, THCS Lâm Thao, Lâm Thao, Phú Thọ; Trần Hống Quý. Tạ Nam Khánh, Chu Vấn Việt. Lê Ngọc Hoa, Phạm Thành Dũng, BE1, THCS Vtnh Tương, Vĩnh Tương, Vĩnh Phúc. NGUYỄN ANH DŨNG Bài 4(1 59+1 60). Tìm giá trị nhỏ nhất của biểu thức A - + y 2 - 7 2 -(xy +ỵz +zx). vói X, y, z là các số thực thỏa mân X > 3 vã xyz = 1 . Lơi giải. Ta có 2 0 A = x 2 + (y + z) 2 -x(y + z)-3yz 3 = — X Ễ +(y + z-0 3 u 2' 4 _ , , , x,2 = (y+Z“> X 3 - 27 9x I 3 X 1 1x 2 ~w Vậy A đạt giá trị nhỏ nhất là , đẳng thức xảy ra khl x = 3 X 3 y + z = — é*

    2 2 1 1 yz - — - -7 X 3 X =3 9±V33 y = z = 12 9 + ^33

    Ĩ2 Nhận xét. Đây là bài toán hay và khó nên có rất H bạn tham gia giải bài. Các bạn sau đảy có lơi giải tốt: Phạm Thành Dũng, Tạ Nam Khánh, Trần Hổng Quý, Lê Ngọc Hoa, Chu Vãn Việt, 8E1, THCS Vĩnh Tường, vĩnh Tường, Vĩnh Phúc; Nguyễn Khánh Hưng, 9A16, THCS Giảng Võ, Ba Đinh, Hà Nội. CAO VĂN DŨNG Bài 5(159+160). Xem xét bản đồ N sau. Nêu íẽn các vừng của N kể VỚ!
  5. r 7 ;
  6. r 2 ; c ) r 6- Lởi giải, a) Các vùng kề vơi vùng r 7 là f Ậ , r s và r a .
  7. Các vùng kề vói vùng r 2 là r t và r 4 (không tỉnh r 6 ).
  8. Các vùng kể với vùng r 6 là r 4 (không tính r 2 ). Nhận xét. Có rất đông các bạn gửi bài đến tòa soạn, tuy nhiên nhiều bạn không hiểu rô khái niệm hai vùng kề nhau nên trả lời sai. Các bạn sau có lời giải tốt: Hoàng Thị Thu Huệ, 8A1, ĨHCS Nghi Hương, TX. cửa Lò, Nghệ An; ĩrẳn Hỗng Quy, Chu Vàn Việt , Lê Ngọc Hoa, Tạ Nam Khanh, Chu Thị Thanh, Nguyễn Thúy Mai, Phạm Thành Dũng, 8E1, THCS vĩnh T Ưởng , Vĩnh Tường; Lê Đức Thải, SA2, THCS Yên Lạc, Yên Lạc. Vĩnh Phúc; BÙI Thị Quỳnh, 8A3, THCS Lâm Thao, Lâm Thao, Phú Thọ. ‘trịnh hoài Dương Bài 6(159+160). Cho tam giác vuông cán ABC (AB = AC). Dường phân giác của góc ABC cắt cạnh AC tại E. Gọi bán kính đường tròn nội tiếp tam giác ABC là r. Chứng minh rằng EC = 2r. Lòi giải. Gọi H là hình chiếu của A trẽn ẼC. I là giao điểm eũa BE và AH thì I là tàm đưòng tròn nội tiếp AABC. K là giao điểm cùa BE và đường thẳng qua H song song với AC. A Vì tam giác ABC cân tại A, từ đó AH là đưởng cao thì cũng là đường trung trực. Ta có KĨH =BĨH =IÃB +IBẦ =45° +-45° 2 -ẤCB+IBC -KHB +KBH -IKH. Nguyễn Thu Hương, Nguyễn Đức igH Tấn, 7A3. THCS Lám Thao, Lâm Thao, Phú Thp; Trương Thị Thu Lan, 7A1 , THCS Yên Phong, Yên Phong, Bắc Nỉnh; Vũ Khánh Huyền, 7A, THCS Vĩnh Tân, Vĩnh Lộc, Thanh Hóa; Trịnh Hoàng Anh, 7D, THCS Xuân Diệu, Can Lộc, Hà Tĩnh: Trương Nhật Nam, 7/1, Do đó AHIK cân tại H, suy ra KH - IH - r. Vì KH là đường trung bình của ABEC nên EC = 2KH. Vậy EC = 2r. Nhận xét. Bài toán này không quả khó nhưhg nhiều bạn phải tính toán rất vất vả mới có được lòi giải. Các bạn sau có lời giải thuần túy hình học: Tạ Nam Khánh, Lê Thị Xuằn Thu, Chu Văn Việt, Lê Ngọc Hoa. Chu Thị Thanh, Phạm Thành Dũng, Trấn Hồng Quỳ, 8E1, THCS Vĩnh Tưòng, Vĩnh Tường, Vĩnh Phúc; Lê Thị Bảo Anh, 9A1 , THCS Nghỉ Hương, TX. cửa Lò, Nghệ An, NGUYỄN MINH HÀ Thi giải toán qua thư THCS Kim Đóng, Hội An, Quảng Nam; Phạm Thành Dũng, Tạ Nam Khánh, Trần Hổng Quỷ. Lê Ngọc Hoa, Chu Văn Việt, 8E1, THCS Vĩnh Tướng, Vĩnh Tưởng, vinh Phúc; Hoàng Thị Thu Huệ, 8A1, THCS Nghi Hương. TX. cửa Lò, Nghệ An; Nguyễn Khánh Hưng, 9A16, THCS Giảng Võ, Q, Ba Đình, Hà Nội. Từ SỐ tháng 9 nám 2015, Công ty cổ phần Dịch vụ Giảo dục Việt Nam sẽ tặng các khóa học trực tuyến trên website: hơcmai.vn cho các bạn học sinh được thưỏng trong các chuyên mục vả các bạn học sinh được khen trong chuyên mục Kết quả thi giải toán qua thư. Các bạn học sinh sau khi nhận được mã cung cẩp thì đâng kí tại địa chỉ: thcs.hocmai.vnAoantuoitho (Xin liên hệ SĐT 0966464644 để được giải đáp}. UI m CHỈ DÙNG THƯỚC Bài toán. Cho đưàng tròn đưâng kính AB vả điểm M thuộc đoạn AB. Chỉ dùng thước thẳng hãy dựng một đường thẳng đi qua M và vuông góc vái AB. NGUYỄN XUÂN BÌNH (Hà Nội) AI NÓI ĐÚNG? (TTT2 số 159+160) Tam giác vuông AOB có cạnh huyền OA = 2BO nên PO = PA=PB = BO. Suy ra ÀOB = 60° , tương tự cố Áoc = 60°. Do đó BOC = ÀOB + Ấdc - 1 20°. Theo tính chất tiếp tuyến cắt nhau thì DỎM = DOB; ỄQM = Ẻõc => DÒE = DÕM+- ẼÕM - Ị (BOM + COM) 1 -±BÓC 2 60°. (TTT2 số 159+1 B0) Do D và E nằm ngoài đường tròn nên ND và NE cắt đường tròn tương ứng tại H, K. Tia OH nẳm giữa hai tia ON, QD nên POH >POD , tia OK nằm giữa hai tia ON, OE nẻn PÕK >POÈ. Ta có PÔH + PÕK >PÕD +PÕỀ =DŨÉ =60° (1) Mặt khốc POH -hPOK -2.ÓNH +2.ÕNK = 2.(ÕNH I ỐNK) = 2.HNK = 2.DNÈ. (2) Từ (1) và (2) suy ra DNE > 30° Vậy bạn Thân n6i đủng. . Nhận xét. Bạn hãy xét trưdng hợp OA khác 2R xem DNỀ có lởn hơn 30° hay không. Các bạn sau được nhận phần thưdng vì có lòi giải đúng: Chu Thị Thanh, 8E1, THCS Vĩnh Tường, vĩnh Tường. Vĩnh Phúc: Nguyễn Vãn Cường, 8A S THCS Hợp Tiến, Nam Sách. Hải Dương. ANH COMPA THẾ CỐ (Kì 84) Trẳng đi trước chiếu hết sau 2 nước. THỀ CỜ (Kì 82) 1 ,®d5+ Bxd5 2.Sc8# Các bạn được thưởng kì này: Nguyễn Đảng Vũ, 8A, THCS Lẻ Vin Thịnh, Gia Binh, Bắc Ninh; Đỗ Minh Hiếu, 9A2, THCS Lẽ Danh Phương, thị trấn Hưng Hà, Hưng Hà, Thái Bình; Nguyền Đức Tân, 7A3, THCS Lâm Thao, Phú Thọ; Mai Anh Quỳnh, 8A, THCS Chư Vãn An, Nga Sơn, Thanh Hóa. LÊ THANH TÚ LỂ THANH TÚ (Đại kiên tướng Quốc tế) H ài Nam - con Irai lớn của em gái thám tử Sêlôccôc - vừa hoàn thành xuã't sắc khóa học “Thám tử trẻ". Hôm đó, một ngày đẹp trời, Hải Nam phấn khởi tới nhà thám tử Sêlôccôc để khoe với bác về khóa học và kết quả mà cậu vừa đạt được. Mâi bước vào sân, Hải Nam đã thấy xe cảnh sát. Cậu càng ngạc nhiên hơn khi thấy trong nhà không có bác mình mả chĩ có hai chú cảnh sát. Hải Nam cất tiếng chào rối lo lắng hối:
  9. Có chuyện gỉ với bác của cháu phải không ạ? Bác cháu đáu rồi ạ?
  10. Cháu cứ bình tĩnh. Cháu là ngưởi quen cùa thám tử ư?
  11. Cháu là cháu ruột cùa thám tử ạ. Mẹ cháu là em gái của thám tử
  12. Cháu có mang theo giây tà tùy thản không?
  13. Dạ có. Cháu có thẻ sinh viên và giấy phép lái xe đây ạ. Sau khi kiểm tra giấy tờ của Hải Nam, một chú cảnh sát nói:
  14. Rất buon phải báo cho cháu biết là thám tử vừa bị bắt cóc. Các chú đã có một số chứng cứ để kết luận đây là vụ bắt cóc Giờ chỉ còn tìm hưởng để giải cửu ...
  15. Bác cháu bị bắt cóc lảu chưa ạ?
  16. Mới thôi. Các chú nhận được tin báo là đến ngay đáy,
  17. Các chú đã tim được manh mối nào chưa ạ?
  18. Chưa. Rất tiếc là chưa.
  19. Cháu có thể lên phòng cùa bác cháu chứ ạ? 16
  20. Tất nhiên. Hải Nam vội vã lẽn tầng hai. Khi vào phòng của thảm tử, cậu phát hiện một tà giấy ở mảy in. Trẽn tờ giấy cỏ những kí hiệu sau: )& !( t A !) @$ @@ )tf ! A !! @ A ! A !) )4 )! “Tờ giấy bí ẩn thật!' 1 - Hải Nam nghĩ thầm. Rồi cậu tất tả cầm xuống tầng dưới.
  21. Chú di! Cháu thấy tờ giấy này ở máy in. Liệu có liên quan gì tới vụ bắt cóc không ạ? Một chú cảnh sát bảo:
  22. Rất CÓ thể. Cháu thử tìm cách gỉải mã xem! Các chú đi kiểm tra thêm khu vực xung quanh, lát nữa sẽ quay lại. Còn lại một mình, Hải Nam chau mày suy nghĩ. Bốn dòng kí hiệu trên tờ giấy đều đƯỢc đánh máy. Hải Nam chdt nghĩ đến bàn phím mảy tính. Khi ấn giữ nút Shìft, go số 1 ta có dấu !, gõ số 2 ta có @ r gõ số 3 ta cố.., Rồi Hải Nam nhớ đến bảng chữ cải Tiếng Việt. Cậu lấy giấy bút viết viết cái gì đó. '‘ơ-rê-ca!" - cậu mừng rỡ reo lên. Đúng lúc đó hai chủ cảnh sát cũng vừa vể tới noi. Hải Nam nói ngay:
  23. Có lẻ cháu đã tìm được manh mối rói ạ. Chú cháu minh cẩn đến ngay đường Trần Hưhg Đạo, đổng thời cấp báo cho công an ở khu vực đó để phối hdp ạ. Hai chú cảnh sát tưdi cười nói với Hải Nam:
  24. Cháu khá lẩm! Bây giờ thì các chú có thể

    nói thật với cháu rồi. Thực ra là không có vụ bắt cóc nào cả. Biết cháu vừa hoàn thành xuất sắc khóa học "Thám tử trẻ” nên thám tử Sêlôccôc và các chú quyết định thử tài cháu thôi. Việc cháu nhanh chóng tìm ra manh mối qua tò giấy bí ẩn đó đã nói lên khả năng phán đoán và óc quan sát nhanh nhạy của cháu rổi- Chắc chẩn sau này cháu sẽ trà thành một thám tử tài giỏi.

    Đố các bạn biết căn cứ vào đâu mà Hải Nam iạì bảo các chủ cảnh sát tởi ngay đường Trần Hưng Đạo? XEES KẺ KHẢ NGHI (TTT2 sô' 159+160) Thám tử Sêlôccôc đắ nghi Bình, bởi Bình kể rằng nhóm bạn của cậu ấy đã góp tiền mua máy bom nước để tặng cho dân bản ở một vùng hẻo lánh chưa có điện. Kì này, tất cả các bạn đểu trả lời chính xác. Đặc biệt, có một thám tử mới học lởp 4 cũng đã tham gìa làm bài và làm rát đúng. Đó là Nguyễn Huy Phước, 4A (nâm học 2015 - 2016), TH Vĩnh Tuy, Bỉnh Giang, Hải Dương. Xin chúc mừng! Phần thưàng sẽ đưdc gửi tới bạn Phưốc và những bạn sau: Mầu Văn Tủ, thôn Phú Thượng B, Thượng Trưng, Vĩnh Tưửng: Tống Phú Làm , 6A {nãm học 2015- 2016), THCS Lý Tự Trọng, Bình Xuyên, Vĩnh Ph úc; Nhóm bạn Vu, Kim , Hoàng Anh, Châu, Như, Dung, Na , 7A {nãm học 2015 - 2016), THGS Xuân Diệu, Can Lộc, Hà Tĩnh; Ngô Võ Hoàng Việt, số 29 đường 10, p. Tân Phú,
  25. 7, TP. Hổ Chí Minh. Thám tử Sèlôecôc THÊM MỘT BẤT ĐANG thức hình học TẠ THẬP (TP. Hổ Chi Minh) Bài viết này chùng tôi xin giới thiệu cùng các hạn thèm một bẩt đẳng thức hình học. Đây là một bài toán hay và khò và chì dùng kiến thức ả THCS ổề giải. Bài toán 1. Cho tam giác ABC nhọn. Chứhg minh rằng cotgA +cotgB -cotgC > Trước hết ta chứng minh hai bổ đề. • BỔ để 1. vởi a, b. c dương thì a + b + c > ^3(ab4-bc + ca). Chứng minh a I b + c > '3(ab + bc \ ca) « (a + b + c) 2 > 3(ab + bc + ca} » a 2 +b 2 + c 2 + 2(ab + bc + ca) >3{ab4bc4Câ) <= a 2 4b 2 4C 2 -ab-bc-cs> 0 <=> (a - b) 2 + (b - d) 2 4 (c - a) 2 > 0 {Bất đẳng thức luòn đủng). Bõ để được chứng minh. • Bổ để 2. Nếu tam giác ABC nhọn thì cotgA.cotgB + cotgB.cotgC + cotgC.cotgA= 1. Chứng mình Vẽ các đường cao AD, BE, CF của tam giác ABC. Gọi H là trực tâm của tam giác. Ta có ẤHF - ẨBD (cùng phụ vứi HAF ) AF Đo đó C 0 tgA.C 0 tgB = cotg A.cotg AHF = — p CF HF.AB = jj^ 2 S HA B CF CF.AB S ABC 2 T ương tự cotgB.cotgC = HBC ; cotgC.cotgA = HCA . S ABC S ABC Suy ra cotgA.cotgB - cotgB.cotgC 4 cotgC.cotgA = 1 . Quay lại bài toán ban đẩu. Áp dụng bổ để 1 ta có cotgA -cotgB 4C0tgC ^3{cotg A.cotg B Tcotg B. cotg c +cotg C.cotg A) \S. Ta còn chũbg minh được BD CD BC BC 2 BC 2 CbtgB + CotgC - 777 = Tương tự AD AD AD AD.BC 2S ABC ' cotgC ỶOũtgA = AC' 23 ABC Do đó cotgA + cotgB tcotgC = ; cotgA +cotgB = AB 2S abc ab 2 +ac 2 +bc 2 2S 'ABC DO dó AB 2 + AC + BC 2 45 ABC /3 <=> S ABC £-{AB 2 4AC 2 +BC 2 ). Chúng ta có thêm mật bất đảng thức sau. Bài toán 2. Cho tam giác ABC nhọn. Chứng minh rằng S ABC <(AB 2 4 AC 2 +BC 2 ). 12 Sau dây là bài tập íự luyện Bài tập. Cho tam giác ABC nhọn. Chứng minh rằng: . . A . B . c 1
  26. sin-— .sin— .sin— <, — ; 2 2 2 8
  27. cosA + cosB 4CQSC í 2
  28. tg A 4 - tg B -4- tgC > 3-s/s . 40 SOLVING LINEAR EQUATIONS IịlSì WITH ONE UNKNOVVN GIẢI PHƯƠNG TRÌNH BẬC NHẤT MỘT Ẩn VŨ NAM ĐỊNH To solve a linear equatìon with one unknown f the unknown should be isolaled on one side of the equation. This can be done by períorming the sa me mathematical operations on both sides of the equation. Remember that if the same number is added to or subtracted from both sides of the equatỉon, thỉs does not change the equality, likevvìse, multiplying or dividing both sides by the same nonzero number does not change the equality. For 6x -5 example, to solve the equation = 2 for X, the variable X can be isolated using the following steps: 6x - 5 = 8 (multiplying by 4) 6x = 13 {adding 5) 13 X = --- (dividing by 6) 6 13 . Theretore, X = -- is the solution.

    6 Math terms linear equâtion unknown solve isolaỉe side phương trình tuyến tỉnh, phương trình bậc nhất

    Ã* jỊ r an, an so giải cô lập, tách biệt, riêng biệt vế, phía operatìon phép toán equaliỉy đẳng thức Practice Bạn hãy dịch đoạn bài khóa trên với các từ đã cho. Bài dịch tốt, gửí sớm {tính theo dấu bưu điện trên phong bì) sẽ được chọn đăng và được quà tặng, Trang tbơ Trang tbcr # Trang tbơ Trang tbcỉ VŨ KIM THỦY Quảng Ngãi Bén dòng sông Trà Khúc Nhìn Thiên Ấn vạn nãm Một thành phổ xừ\h xắn Quảng Ngãi gần mả xa vượt đường dài Hỉnh Định Ghé Dồ Bàn hoang ULI Năm thế ki xưa cũ Lổng lòng khách lãng du Trầm mạc íháp Cánh Tỉẽn l\tỢng voi chầu thành cổ Theo ta về Sa Huỳnh Diển Mỹ Khê* lộng gió Gặp rửng dừa Quàng Ngãi Ngỡ dang còn Tam Qĩưm

    Mía ngon đường cát trắng Dọc đường dài Khu Nãm 23.3.2016 Từ Bình Định đốn Quảng Ngãi

    Mỹ Khẽ nãy thuộc Quảng Ngãi không phài Mỹ Khê, Đã Nang * Tam Quan thuộc Binh Dinh NGUYỄN TRỌNG DỎNG (TI ỈCS Phú Lộc, Krông Năng, Dẩk Lẩk) Hoa Pbượog Tháng năm uể ki niệm hồng hoa phượng Tiếng ve ngàn như tiếng lá rận ràng Hè lại đến nhuộm vàng cửa lớp Khóa trời xanh trong sắc lá mênh mang Rộng săn trường thưa váng hước chăn em Hoa phượng cháy những nỗi niềm dang dở Từng di qua xạc xào ngọn gió Vần íhay lòng thao thức ƯỚĨ bảng đen Mùa hạ săn tniờng như ừò chơi trốn tìm Chỉ hoa phũỢng ỉim dim đdi má í đỏ vdn dõi bóng em con dường sách uở Dế mùa í hu không bỡ ngỡ hạt sương mai De săn trường chãy đố sáng nay Như ki niệm cứ hồng qua nuỉa náng Cho ta sổng tháng ngày thanh thản Bạn bè ơỉỉ Hoa Phượng mãi trang tim! THÁCH ĐẨU! THÁCH ĐẤU ĐÀY! TRẬN DẤU THỨ MỘT TRĂM BA MƯƠI CHÍN m p Người thách đấu: Trần Quang Hùng , GV. trường THPT chuyên Khoa học Tự nhiên Hà Nội, Bài toàn thách dẩu: Cho tam giác ABC nộí tiếp đưỏng tròn (O), đưỏng cao AH, trung tuyến AM. Gọi p, Q là hai điểm thuộc cung BC không chứa A sao cho PQ I! BC vá tia AP nằm giữa hai tia Aũ và AB. Gọi K, L thứ tự là hình chiếu vuông gôc của B, c lên AP, AQ.
  29. Chứng minh rằng H. M. K, L cùng thuộc một đường tròn, gọi đường tròn đó là (I).
  30. Gọi giao điểm khác K của AP và đường tròn (I) là N , Chứng minh rằng NL luôn đi qua một điểm

    cố định khi p, Q di chuyển. Xuất xứ: Sáng tác. Thời hạn: Trưãc ngáy 03,11.2016 thao dấu bưu điện. TRẬN ĐẤU THỨ MỘT TRĂM BA MƯOt BẢY 9 ■ (TTT2 SỐ 159+160}

    Có hai võ sĩ nhận lời thách đấu nhưng

    chỉ có một võ sĩ có lời giải đúng là Lê NgạcHoa, 8E1, THCS Vĩnh Tường, Vĩnh Tưỡng, Vĩnh Phúc. Sau đây là lài giải cũa võ sĩ Hoa. Trên nừa mệt phảng bờ AB không chứa I dựng điểm p sao cho AAPB co ADIC, Ta có IẤP +IBP =IÃB +BÃP -IBA -ẤBP -- IAB + IDC -rÍBA +ÍCD = ỊỏÃB + ỊcDA + Ị ABC + Ịbc'D 2 2 2 2 = 1(DÃB + CDẦ +ẤBC +BCD} = 1.360 ũ =180 0 Do đó tứ giác AIBP nội tiếp. (1) Từ (1} vá chú ý rằng AAPB co ADIG. Suy ra ÃPÌ - ẤBÌ - ỈBC; ÃÍP =ẤBP -lõb =ICB; BPÌ = BÂi =IẤD; BÌP =BÂP =IDC =IDA. Do đó APIA co ABCI và APIB co AADI

    AP IP AI , BP IA ...

    Suy ra — — = - 7 — Si -77 và 777 - 77- (2)

    BI BC Cl BI ID Theo định li Ptolemy ta có AB.IP = BI.AP + AI.BP. Từ đó và (2) có AB.BC = AB.IP. AP BP 'ìBC Br .-77 + AIB 1.-7 1 BI BI IP = DLrtr +■ nLDr. BI^+AIEL^ BC DI JlP = BI 2 + BL^=B. 2 +*Ua DI IP DI , AI Do đó AB.BC-BI 2 -—ị.BLQI . DI Tương tự CD.BC - Cl 2 +|,CI.BI. AI Cộng vế với vế hai đẳng thức trên và áp dụng bất đẳng thức AM-GM, ta cỏ Tương tự AD(AB + CD) > (AI + DI) 2 . (4} Từ (3), (4) và AD BC - AB + CD suy ra (AB + CD} 2 >(AI +DI) 2 -(BI +CI) 2 (5) Kết hợp với giả thiết (AB + CD) 2 = (AI ■ DI) 2 + (BI + Cl} 2 suy ra đẳng thức ò (5) xảy ra. Do đó đẳng thức xảy ra ở (3} và (4). SuyralA ID;IB IC. Do đó BAD =2.lẤD =2.1 DA -CDẦ; ÃBC = 2.IBC = 2.ICB = DCB. Suy ra DAB +ẤBC = Ị(BÃb +CDA +ÃBC +DCB)
  31. 1 - 360 ° - 180 °, 2 Suy ra AD ìi BC. Từ đó và ÁBC = DCB thì ABCD là hình thang cân. Nhận xẻt. Đương nhiên võ sĩ Lẽ Ngọc Hoa là người đâng quang trong trận đấu này. NGUYỄN MINH HÀ BÃhlH «H0 ú&mụmt Húi NHÃ lUtìl số D9ỂU HÒA ThS. KIỀU ĐÌNH MINH (GV. THPT Chuyên Hùng Vương , Phũ Thọ) 111 Vài mỗi sổ nguyên ơương n, tổng H n =1+4 + 4 + „. + — được gọi tà số điểu 2 3 n hòa thứn. Dãy (H dược gọi tà dãy số diều hòa (hay dãy diều hòa). Bài viết này chủng tôi sẽ trình bày một sổ kết quả cơ bần về số đíéu hòa cũng như các bài tũán liên quan tnà chứng ta thường gặp trong các kì thi Oiytnpic. Sài toán 1 . (Đống nhất thức Cataian ) Chứng minh rầng vởi mọi n F N. thi

    ,1.11 1 1 11 234 2n n + 1 n + 2 2n Lãi giải. Biến đổi vế trái ta cỏ ,11 1 1 2 3 4 2.n

    (. ,1 , 1 ,1 — Ị 1 + —+— + .,.+ V 2 3 2n 21^ + 1 + ... + -

    2 4 2n

    |4 11 1 'ì í' 11 1 = 1 + — + — + ... + — - 1 + — +—+... + — 2 3 2n 2 3 11 1 n + 1 n + 2 2n' 1 = (n+1) -11+...+ 11 -7+4+...+- ]-n 1 2 n = {n - 1}H n - n = (n + 1)H n+1 - (n + 1). Ti T, T„ Suy ra UL = -T- + -T- + ...+ 2 3 n+1 = (H 2 -1) + {H 3 -1} + + <H n+1 -1) = H 2 t H 3 + ... + H n+1 " = - H 1 + T n + H n + 1 = (n + 2)H n+1 -(2n + 2). Bải toán 3. (Canada MO 1998, Albania BMO TST 2014} Cho ne N, n > 2. Chứng mỉnh rằng 1 4,1 3 2n-1 n + 1 1 V ư 1 1 1
  32. 7-+4+...+T— n 2 4 2n 11+ ^ o £11— ị J I Ị 1 £ H £11 . Lài giải. Bất đẳng thức cần chửng minh tượng đương với 1+4+...+——— l>(n+1) 3 2n - 1 Ị v i 1 , 1 . — + — + ... + —— •{ ) 2 4 2n Nhận xét. Đây là đổng nhất thúc khá đơn giản, tuy nhiên lại cố nhiều ứng dụng khi giải toán. Chúng ta sẽ jt>ẩí gặp điều này trong phần sau. 8ài toán 2. Vối n e BT, cho H n =1 + | + l + ... + -l;T n =H 1+ H 2 + ... + H n ; , I Ti T? T n u„ = -4+ -4- + — 4-. 2 3 n + 1 Chửng minh rằng T n = {n ■ 1)H n+1 - (n + 1); U n = ir + 2)H n+1 -(2n + 2). Lài giải. Tệ cỗ T - .4 . 11,4 . 1 . IV ,4.1.1, 11 1 2 ụ 2 3J l ỉ 3 nj n n-1 . n-2 . . 1 — — — — — I — - — +.., + — 12 3 r ín-1 ,1 Yn+1 4l 'n + 1 4 Ta sẽ chứng minh () bẳng quy nạp. 8 0 Thật vậy với n - 2 thì () trồ thành 4> 4 (đúng). 3 Giả sử () đúng vởi n - k > 2, tức là .41 1 k 1+4+... + — — - { 3 2k - 1 Vởi k > 2. ta có 3 4 1 = k £ 2, tức là (k + 1)í 1+1 + ...+-L' [2 4 2kj 4 . 1 . 1 V k + 1 1+77+...+— — " 1+— — 7 3 2k-lJ 2k + 1 fl 1 , 11,1, =1 4+4+-+ — +4 + [23 2k-1 ỉ 2 M 1 1 1 1 k {2 4 2k I { 1 1 1 1 1 4+4+— +4- ụ 4 2k J M 1 1 1 k + 1 2k + 1 k ị 1
  33. — + 2 2k + 1 k + 1 1 I — + — + ...+ 12 4 2k Do đỏ 2k + 2 2k + 1 k-2 2k + 2' (k + 1}Ị 1 + 1- + ... + — !— + — !— 1 3 2k-1 2k + 1 . kl1+ < 3 2k-1 ‘H k| 1+4+...+ — 1 3 2k-1 ,1 . , 1
  34. 1+4+ ■ + — 3 2k — 1 .11 1
  35. 1 - + - + .•- + — 1 + 2 4 2k k + 1
  36. 2k + 1 k + 2 2k - 2 M -<k- 2 ) '11 1 — + — + — — 2 4 2k .MI 1
  37. — + — + ... + ■

    2 4 2 k k + 2 2 k + 2 1 , 1

    + — ■ 2 4 1 2 k + 2 . £ Theo nguyên lí quy nạp thì (*) đúng với mọi nE N, n > 2 . Bài toán 4. (Brazil MO 1983) 11 1 Chứng minh rằng H n =1 + - + - + ... + — không là 2 3 n số nguyẻn với mọi ne M, n > 1 . Lời giải. Gọi k là số tự nhièn thỏa mãn 2 h < n < 2 k ' 1 và M là tích tất cả các số lẻ khàng vượt quá n. Ta có , . „11 1 1 H n = 1+ — + — +... +-T- + ... + - 2 3 n M 2 k 1 .M.H n = 2 k_1 M +2 k_2 M +2 M . — + n 3 M . , + — + 2 2 k-1 M M , , . Ể z (vì Y khủng nguyên). 11 1 vệy H n = 1 ... + — khởng phải sổ nguyẻn. 2 3 n Bài toán s. (IMO 1979) Cho p, q Ẽ N sao cho 11 1 2 3 4 1 1 -+- -= 1 - q 2 3 4 1318 1319 Chứng minh rằng p chia hểt cho 1979. Lởi giải. Áp dụng đồng nhất thức Catalan ta có p = J_ 1 11 q 660 661 + " + 1318 1319 1 1 1 — •— — +
  38. 660 661 1319 660 1 . 1 -+■ +... ị - 1 1 1 1319 ( 1 ,660 1319 1979 w 1 1 ) 71 1 V Ị , 661 1318 Ị ụ 31 9 660 1979 . 1979 1 A — + =1979.;—. B 1319.660 660.1319 661.1313 Trong đó B là tích của các số nguyên không vượt quá 1319. Mà 1979 là số nguyên tố. Vậy p chia hết cho 1979. Bài tập vận dụng Bải 1. (IMO SL 1989) Chứng minh rằng 1 2 | 1 | 1 2 1 | 1 2 2 3 4 5 6 + "' + 478 %79 480 1S9 «22 641 k =o(ĩ61 + k)(480-k) Bài 2. (IMO SL 1970) Với n nguyên dương chãn chứng minh rằng ,111 1 1- — + — — 7 + ... — - 2 3 4 n J 1 . 1 , 1 — 2 ; — — + + + ... +— — . Ị n + 2 n + 4 n + 6 2 n J Bài 3. /Canada MO 1973) Chứng minh rằng n + Ht + H 2 +H 3 + ... + H n 1 = nH rt ,Vn > 2. Bải 4. (Rom Matb Magazine, July 1998) 1 1 1 1 Cho A = 4- , 1.2 3.4 5.6 2011.2012 B - 1 1 1 1007.2012 1008.2011 1009.2010 1 2012.1007 Tính ậ. B Bải 5. Chímg minh rằng 1 1 1 . 1 1 2 m + 1 2+2 2 m +3 2 m+1 2
  39. 1+-+-+ .. - + -T- 1+\vke N 2 3 2 k 2 11 12
  40. 1 + -T + -7 + ,.., + - 7 - > „ ,Vke N 2 3 2k 3
  41. -<1+-ị+^+... + — < k,Vke N 2 2 3 2 k - 1 . 1 . 1 , 1 . 1 13 k + 1 k + 2 k + 3 2 k 24 „ 1 1 1 1 f ) - — — - ■+ - — — — + - — — — + ... 4 —7— — 7 > 1 r Vk e rí k + 1 k + 2 k-3 3k+1 .1 1 , 1 1 _ L 1 2 , 2 3k + 1 3k +2 3k + 3 5k +1 3 Bải 6 . (APMO 1997) Cho s = n — 4 — — — + . . . + 1 1 , 1,1 1 ,1.1 1 + — 1 + — + 77 1 + — + — + ... + - 3 3 6 3 6 1993006 ở đây các mẫu số chứa tổng riêng của dãy các nghịch đảo của £ố tam giác. Chứng minh rằng s > 1001. Bải 7. (ÍTOT, JuniorA - Level, Fatl 2013) Số 1--ị + -Ị---ị + ... 1 --7- được biểu diễn 2 3 4 2n - 1 2n dưởi dạng một phán số tối giản. Giả sử 3n ị 1 là một sô' nguyên tố- Chứng minh rằng tử $ố của phân số này lả bội của 3n + 1. AUSTRALIAN MATHEMATICS COMPETITION AMC 2015 SENIOR DIVISION AUSTRALIAN SCHOOL YEARS 9 AND 10 Time allowed: 75 minutes (Tiếp theo kì trườc) PGS. TS. ĐỔ TRUNG HIỆU (Hà Nội) (Sưu tầm t/à giôi thiệu)
  42. Given an integer N greaterthan 1, the sum of N and the second largest ĩactor of N can be íound. For example, with N = 55, the suim is 55 + 11 = 66. For how many integers is this sum equal to 42? (A) 3 (B) 4 (C) 1 <D) 0 (E) 2
  43. Unequal squares PQRS and STUV are aligned with the straight line PST. XQ and YU are perpendicular to XY. The length XQ + YU is the same as Questíons 21 to 25, 5 rnarks each
  44. The diagram shows a regular hexagon with sides of length 1 inside a square. Four vertices of the hexagon lie on sides of the square; the other two vertices lia on a diaganal of the square. What is the side length of the square? (A}73 (B) 6(^3 - J2) (C)I(V3W2) (D)Ujẽ+j2) 2 2 (E}2
  45. This list 3 was made by Norm and Zoltan, starting with 3 and 5 and then taking turn$ to add one number to the list. Norm went tirst. VVhenever it is his tum, Nornn s new nimnber {shown círcled) is the product of the last two nunnbers in the list, VVhenever it is his turn, Zoltan's rew number (shown boxed) is found by di viding the second-last number by the last number. What is the 201 5th number in this list? (A) -L (B) 3 (C) 5 :
  46. Points p, Q, R and s lie on a circle and ZSRQ = 60° |f RS = 4, RQ = 6, $p = X and PQ - y, then a possible solution for X and y is p (A) X - 4 and y - 2 (B) X - y = 3 (C) x = y = 4 (D)x = 4andy = 3 (E) X = 5 and y = 2
  47. How many sequences oỉ se ven positive integers a 1 < a 2 < a 3 < a 4 < a 5 < a e < a 7 are there such that each is less than 1 00 and each number apart from the last is a íactor of the next number in the sequence? (A) 3 (B) 7 (C) 4 (D) 6 (E) 1
  48. Two spheres, one of radius 2 and the other of radius 4. have the same centre. What is the edge size of the largest cu be that fits between them? (A)§ (B)Vi9-1) b ó (C) (Đ)|(V22-2) (E) -Ệ
  49. How many positive integers n less than 2015 11 have the property that — + - can be simplitied to 3 n a traction vvith denominator less than n?
  50. A 100 X 100 X 100 cube PQRSTUVVV is made oỉ 1 X 1 X 1 non-overlapping cubes. z is a poirìt on PS such that PZ = 33. Through how many of these 1x1x1 cubes does vz pass? V u
  51. At Berracan $tation, northbound train$ arrive every three minutes starting atnoon and tinishing at midníght, while southbound trains amve every fìve minutes starting at noon and íìnishing at midnight. Each day. I walk to Berracan station at a random time in the aíternoon and wait for the íirst train in either directíon. On average, how many seconds $hould I expect to wait?
  52. In a 38 X 32 rectangle ABCD, points P.Q.R and s are chosen, one on each side OỈABCD as pictured. The lengths AP r PB, BQ, QC. CR r RD. DS and SAare all positive integers and PQRS is a rectangle. A p B For questions 26 to 30, shade the answer as an integer from 0 to 999 in the space provided on the ansvver sheet. Question 26 is 6 marks, question 27 is 7 marks, question 23 is 8 marks, question 29 ìs 9 marks and questicm 30 is 10 marks. What ỈS the largest pos$ible area that PQRS could have?
  53. The set s consists of distinct integers such that the simallest is 0 and the largest is 2015. What ỉs the mìnimum possible average value of the nụmbers in s? Bài 12N5. Cho tam giác ABC Bài 10NS. Tim các sổ nguyên X, y, z thỏa mãn |x - 2y| - z - 2x 2 + 1; |y - 2z| - X = 4y + 2; |z - 2x| - y = 6z 3 - 4. LẠI QUANG THỌ (Phóng Giáo dục vá Đào tạo Tam Dương, Vĩnh Phúc) Bài 11NS. Cho a, b, c là độ dài ba cạnh của tam giác. Chứng minh rầng a + b.a + c 2a — 1 " — — 4 " ■— < 2 . 3a + c 3a + b 2a + b + c TRƯƠNG QUANG AN

    (GV. THCS Nghĩa Thắng, Tư Nghĩa, Quảng Ngãi ) nhọn có các đường cao AD, BE S CF,

    1 Biết rằng A = 60° , BC = a. S ỮEF = -S ABC . Tính diện tích tam giác ABC theo a. TẠ THẬP (TP. HỒ Chí Minh) cuDc THI GIÚI ĩoAr dAik CHO no sinH (TTT2 sò' 159+160) Bải 7NS. Ta có 2x 6 ị y 2 - 2x 3 y - 320 <=> X 6 4 (X 6 - 2x 3 y 4 y 2 ) = 320 «4 X B + (x 3 - y) 2 = 320. Ta có x; y e z nên (x 3 - y) 2 là số chính phương. Vì {x 3 - y) 2 > 0 =ì> X 6 < 320. Mà X 6 z nên |x| < 2. • Nếu X e {0; 1; -1} thì (x 3 - y) 2 E {320; 319) {loại). • Nếu x = 2 thì (8 - y) 2 = 256 8 - y e {16; -16) {-8; 24). • Nếu X = -2 thì (-8 - y) 2 = 256 « -8 - y ẽ {1 6; -1 6) ey Ẹ {-24; 8). Vậy (x; y) e {(2; -8); (2; 24); (-2; -24); (-2; 8)}. Nhận xẻt. Các bạn cỏ lời giải đúng: Bùi Thủy Linh, 8Â1 , Triệu Hổng Ngọc, 7A3, Nguyễn Thu Hiền, ỖA3, THCS Lâm Thao, Làm Thao, Phú Thọ; Lê Hổng Nhung, 7A, THGS vĩnh Yên, TP. vĩnh Yên, Đổ Thị Minh Hải, 8A, THCS Lý Tự Trọng, Bình Xuyên, Chu Thị Thanh, 8E1, THCS Vĩnh Tường, Vĩnh Tương, Lẽ Thị Xuân Thu, 8A2. THCS Yên Lạc, Yẽn Lạc, Vĩnh Phúc; Lưu Thị Phương, 8A1 , THCS Từ Sơn, TX Từ Sơn. Bắc Ninh; Mai Ánh Quỳnh , Hoàng Hà My, 8A, THCS Chu Văn An, Nga Sơn, Thanh Hóa. Bài 8NS. Xét P(x) = ax 2 + bx + 0 . ta có áp = b 2 - 4ac. Ta có Q(x) =x 2 = (a + b + c)x 2 +{2a +b)x +a. Do đố Aq = (2a + b) 2 - 4(a + b - c)a = b 2 - 4ac = Ap- Như vậy trong trò chơi trẽn, đa thức ban đầu và đa thức thay thế đểu có dạng ax 2 + bx + c và có giá trị của Ạ bằng nhau. Kiểm tra thấy các đa thức f(x) và g(x) có A khác nhau. Do đó sau 2016 bước, trên bảng không thể có đa thức g(x}. Nhận xét. Cách khác: Nếu b, c > 0 thì a + b 4- c > a. Do đó đa thút mới phải có hệ sđ của X 2 lân hơn 1. Vậy không thể viết được đa thức g(x). Các bạn có lơi giải đúng: Chu Thị Thanh, 8E1 , THCS Vĩnh Tường, Vĩnh Tường, vinh Phúc; Bùi ThùyLinh, 8A1 , THCS Lâm Thao, Lâm Thao, Phủ Thọ. Bài 9NS. Bạn đọc tự vẽ hình. Vẽ AM là tiếp tuyển của đường tròn (ũ) (M là tiếp điềm khác B). Ta có M cố định. Nối MC, MF, OB, OM, BM. Vi OB = OM, AB = AM OA là đường trung trực của BM, Mà F € OA— > FB - FM — } AFBM cân tại F => BMF = MBF. (1) Vì BM 1 OA, EF 1 OA => BM // EF => IỸĨBF = BFE. (2) Vì ỐBE = ÕCE > ỐFE 90° nên B, c. F, 0, E cùng thuộc đưàng tròn đường kính OE => BFE ■ BCE. (3) Ta có BCE - BMC. (4) Từ (1), (2), (3) và (4) suy ra BMP BMC- Do đỏ hai tia MF và MC trùng nhau. Vậy đường thẳng CF luôn đi qua điểm cố định M. Nhận xèt. Không có bạn nào giải được bài toán này. Các bạn dược thương ki này: Bùi Thủy Linh, 8A1, THCS Lảm Thao, Lâm Thao, Phú Thọ; Lê Thị Xuân Thu. 8A2, THCS Yên Lạc, Yên Lạc; Lẻ Hồng Nhung, 7A, THCS Vĩnh Ỳen. TP. Vinh Yên; Chu Thị Thanh, 8E1, THCS Vĩnh Tường, Vĩnh Tường, Vinh Phúc; Lưu Thị Phương, 8A1 , THCS Từ Sơn, TX. Từ Sơn, Bắc Ninh; Mai Ảnh Quỳnh, 8A, THCS Chu Vân An, Nga Sơn, Thanh Hóa. Anh các bạn dược thương ả trang 6. NGUYỄN HIỆP vũ KIM THỦV
  54. Con số theo con ngưởi từ khi mãi sinh ra. Bạn có một con số đeo hoặc viết ở tay cùng con số của mẹ để hộ lí, bác sĩ khỏi nhầm. Bạn sẽ có ngày sinh, có số ghi trên giấy khai sinh. Lân lèn một chút bạn thấy sổ nhà cùa mình, số trên td lịch hàng ngáy vã sđ trẽn cái đổng hổ ngảy đém chạy cẩn mẫn. Bạn tập nhớ số điển thoại của ba, của mẹ. Bạn biết minh còn có số đo chiều cao, số đo cân nặng, số đo huyết áp, số đo nhịp tim. Lớn hon chút nữa bạn quan tâm xem nhà minh rộng bao nhiêu m 2 , sổ tiền lương của ba. mẹ nhận được mỗí tháng là số có bao nhiêu chữ số, Rồi bạn có số chứng minh thư, số hộ chiếu, số tài khoản, ...
  55. Bạn hăy tưởng tượng một ngày các con số đột nhiền bị biến mất. Điểu gì sẽ xảy ra nhĩ? Bạn sẽ bắt đầu đi làm vào lúc nào vì các đóng hó không còn con sá? Ra bển xe buýt bạn biết đi tuyến nào để đến địa điểm cẩn đến vì số tuyến xe cũng không còn. Gay go nhất là các số điện thoại không có, câng việc và mọi mối liên hệ sẽ ra sao?
  56. Bởi con sô quan trọng như vậy nên từ mấy nghìn nâm trước các con sô đã ra đời â Ân Độ, Ả Rập. Hy Lạp, Trung Quốc, ... Tập hợp số đếm ra đời trưỏc hết: 1, 2, 3, ... Ngày nay tập hợp này ở nhiều nưốc coi là tập sổ tự nhiên {Việt Nam coi tập số tự nhiên là: 0, 1,2, 3, ...).
  57. Bây gìở chúng ta hãy cùng xem bảng già xe chạy như sau: Bèn Thdi gian Giáp Bát 13 05 13 30 13 55 14 05 14 30 14 55 15 10 Thướng Tín 13 25 13 50 14 15 14 25 14 50 15 15 15 30 Đồng Vàn 13 50 14 15 14 40 14 50 15 15 15 40 15 55 Cầu Họ 14 15 14 40 15 05 15 15 1540 16 05 16 20 Cổng Hậu 14 25 14 50 15 15 15 25 15 50 16 15 16 30 NÚI GỎI 14 40 15 30 1540 16 05 16 45 ' Phủ Giầy 14 43 15 05 1543 16 30 Bảng cho 7 bến xe trên tuyến Hà Nội - Nam Định với 7 chuyển xe mỗi ngày của một hãng. Bạn hãy nhìn vào bảng để trả lòi các câu hỏi sau:
  58. Có bao nhiêu xe dừng ờ bên Núi Gôi?
  59. Cố bao nhiêu xe dửng ở cà bến Núi Gôi và bến Phủ Giầy?
  60. Mất bao nhiêu thòi gian để xe khởi hành 13 55 từ bến Giáp Bát đến bến cổng Hậu?
  61. Thài gian dài nhất để đi từ bến Giáp Bát đến bến Núi Gôi là bao lâu?
  62. Một ngưài đến bến Đồng Vãn lúc 14h10 và đi chuyến ngay sau đó thì sẽ đến bến cổng Hậu lúc nào nếu xe chạy đúng giờ? Bạn thấy không, các con sô’ liên tục tham gia vào cuộc song của bạn.
  63. Bạn cùng xem một ví dụ thứ hai về các con số nhé. Đây là kẻt quả hai bảng đấu giải Quốc gia. BẢNG B Hà Nội Hảì Phòng Nam Định Quầng Ninh Thừa Thiên - Huế BẢNG A Hà Nội thắng 3-1 Nam Định thắng 3- 3 - D Hã Nội thấng 1 -ờ Nam Dinh thang Đà Nắng Long An TP. Hũ Chí Minh Bình Định Cấn Thơ □ 2 - 1 Đà Nang thắng 2-1 Long An tháng Bình Định thẩng Đà Nắng tháng 3-2 1 -0 Bạn hãy nhìn vào 2 biểu đá để trả lời các cảu hải sau:
  64. Nêu tên các đội đứng đẩu bảng và vào vòng trong.
  65. Mỗi đội đầu bảng đả đả mấy trận?
  66. Tính trung bình sô' bàn thắng mồi trận của mỗi dội đẩu bảng. Đây là ta mái nêu 2 vỉ dụ đầu tiên vễ các bài toán liên quan đến 5Ố tự nhiên. Hẹn gặp các bạn ỏ các bài viết sau. QUY CHÊ CUỘC THI CÂU LẠC BỘ TOÁN TUỔI THƠ TRÊN TẠP CHÍ Cuộc thi Câu lạc bộ Toán Tuổi thơ toàn quốc dự kiến sẽ được tổ chức vào nửa đẩu tháng 6.2Q17. Bên cạnh cuộc thi đó để các em học sinh được rèn luyện và tham khảo các để toán tiếng Anh, từ nàm học 2016-2017, Tạp chí sê tổ chức thêm Cuộc thi giữa các Câu lạc bộ Toán Tuổi thơ trên tạp chí,
  67. Nội dung dể thi. Hàng tháng trên Tạp chi sê đãng đề thỉ gồm 5 bài toán bằng tiếng Anh.
  68. Đoi tượng dự thi. Mửì trường THCS có 1 Câu lạc bộ Toán Tuổi thơ lớp 8 được tham gia dự thi. Các cãu lạc bộ tham gia dự thi là các Câu lạc bộ của trưởng dã dăng kí với tạp chí Toán Tuổi thơ (các trường tải Quy chế Câu lạc bộ Toán Tuổi thơ, mầu dâng kí và theo dõi danh sách các Cáu lạc bộ trẽn vvebsite: toantuoitho.vn}.
  69. Thời gian nhận bài giải. Sau 1 tháng kể từ ngày tạp chí phát hành (theo dấu bưu điện). Các bài tham gia dự thi trình bày các câu liẺn nhau trên một tờ giấy bằng tiếng Việt hoặc tiếng Anh (trình bày bằng tiếng Anh được ưu tiên), trên bài thỉ ghi rõ câu lạc bộ Toán Tuổi thd của trường, huyện (quận), tỉnh (thành) và ghi ngoài phong bì; Tham dự Cuộc thi Cảu lạc bộ Toán Tuổi thd và gửi về tòa soạn; tầng 5, số 361 Trường Chinh, Q. Thanh Xuân, Hà Nội.
  70. Tổng kết và trao giải. Tòa soạn sẽ tiến hành chấm và đãng danh sách 5 Câu lạc bộ có bài giải tốt nhất trên tạp chí ở mỗi số và danh sách các câu lạc bộ giải đủng trên website: toantuoitho.vn sau 2 sỏ’ báo kẩ từ sổ đăng đề bài. Cuối năm học tòa soạn sã tổng kết và trao giải. Giâi thưởng gốm giấy chửng nhận, quà tặng và tiển thưởng. CLB1, Find the polynomial p{x) where its degree smaller than 3 and satisfying the condition 2P(x) - P(1 - x) - X - 1 for all value of X, CLB2. A man hás 4 Sons, each õf whom 1$ 4 years older than his next younger brother; and the eldest is fìve tìmes as old as the youngest. What is the age of each? CLB3. Give positive real numbers X, y such that xy = 1. x^ y 2 1 Find the smallest value of E — + — + — 1 — . y X X + y CLB4. Let M = 4 + -4- + -4- + — + — r- Compare M to 2. 2 2 2 2 3 2 " CLB5. Given trapezoid ABCD where AB and CD are base sides, CD = 3AB. Let E be a point on AD such that DE = 3AE. A line goes through point E, parallel wìth CD g and meets BC at F. Calculate ^ ASFE ■ $CDEF NGUYÊN KHÁNH CHUNG (GV. trường Lô-mó-nô-xốp, Q. Nam Từ Liêm, Hà Nội) TIN TỬC - HOẠT ĐỘNG - GẶP GỠ KI thi IMG lần thứ 19 do uỷ ban giáo dục cơ bân, Bộ giảo dục Thái Lan đàng cai tổ chức tại thành phô' Chiang Mai từ ngày 14.8 đến ngày 20.3.2016. Cuộc thi Toán học trẻ quốc tế IMC (ỉntemational Mathematics Cùmpetìtion) là cuộc thi Quốc tế được tổ chức dành cho học sinh trên toàn thể giãi với 2 độ tuổi dưới 14 { EM 10) và dưới 17 (IWYMIC) từ năm 1999. Kĩ thi năm nay cố 622 học sinh dự thi đến từ 31 quốc gia và vùng lãnh thổ. Được sự cho phép của Bộ Giáo dục và Đào tạo. đoàn Việt Nam gồm 24 học sinh của trường THPT chuyên Hà Nội - Amsterdam được chia thành 6 đội gồm 4 đội Junior (độ tuổi dưới 14) và 2 đội Senior (độ tuổi dưới 17). Mồi đội tham gia gồm tối đa 6 thành viên: Trưởng nhóm, Phó nhóm và 4 học sinh. Mỗi khối thi phải thực hiện hai phần thi bao gồm: Phần thi cá nhân và Phần thi đổng đội, ngoài các phần thi trên, các đội sẽ được tham gia các hoạt động tham quan dã ngoại, đèm giao lưu văn hỏa, ... Đoán học sinh Việt Nam giành được 8 cúp đóng đội trong đó có 2 cúp vô địch, 23 giải cả nhân trong đó có: 1 HCV: 4 HCB; 12 HCD và 6 giải Khuyến khích. Ngày 14.9.2016, Hội Toán học Hà Nội dả tổ chức Semina các chuyên đề toán bối dương học sinh giỏi tại trướng THPT Nguyễn Siêu, Khoái Châu, Hưng Yên. Trong Semìna, NGND. GS. TSKH. Nguyền Ván Mậu, Chủ tịch Hội Toán học Hà Nộỉ đà cỏ bài phát biểu nêu bật tác dụng của việc tổ chửc các buổi Semina để góp phần nâng cao chất lượng đội ngũ giáo viên bồi dương học sinh giỏi. Toán Tuổi thơ đã tặng tạp chí cho đại biểu. TTT AilLUL» CÂU LẠC BỘ TTT (TTT2 số 159+160) ■ ■ Do đó ab + cd = 0. CLB11. Ta có x-500 X -403 X -71
  71. I — — I — — — =10 CLB12. • Nếu n chân thì n 20 - 11 là sổ lẻ mà 2016 là số chân nên n 2 ' 1 - 11 không chia hết cho 2016. • Nếu n lẻ thì n - 2k - 1 (vâi k là số tự nhiên) nên n 20 -11 = (2k + I) 20 - 11 = {4k 2 + 4k + 1) 10 -11 chia cho 4 dư 2, Mà 201 6 chia hết cho 4, Do đó n 20 - 11 không chia hết cho 2016. Vậy n 20 - 1 1 không chia hết cho 201 6 với mọi n nguyên dương. CLB13. Đặt t = X + — . X Ảp dụng bất đẳng thức AM-GM ta được t > 2. Do đóM = t 3 + 4t 2 -3t + 10 = (t 3 - 2t 2 ) + (6t 2 - 1 2t) + (9t - 18) + 28 = t^t - 2) + 6t(t - 2) + 9(t - 2) + 28 > 28. Đảng thức xảy ra khi và chỉ khi t = 2 {=> X = 1 . Vậy MinM = 28 (đạt được khi X = 1). CLB14. Ta cô 2016(ab + cd) = 2016ab + 201 Gcd = (c 2 + d 2 )ab -I- (a 2 + b 2 )cd (abc 2 + b 2 cd) + (a 2 cd + abd 2 ) = bc(ac + bd) + ad(ac + bd) = 0 (vì ac + bd = 0). Vậy M = (a 2 + b 2 ) + (c 2 + d 2 ) - 2(ab + cd) = 4032. CLB15. Bạn đọc tự vê hình. Đặt AB = Ảc = BC = a. xảy ra hai trưởng hợp: • THI: M nằm trong aABC- Ta có S MAC + S MAB + S MBC = S ABC => ịAC.y ị ị AB.Z I ịec.x = -Ị-BC.h 2 2 2 2 1,1 1 " 1 =— a.y + — a.z + — a.x — a.h

    2 2 2 2 => y + z + x = h, Mà 2x = y Ỷ z. Do đỏ X =Ạh. 3

    TH2: M nằm ngoài AABC và nằm trong BÂC. Ta cô Sịyiac +S M ab -S-mbc -Sabc => -rAC.y + -Ị-AB.z 4bC.x =BC.h 2 2 2 2 11 1 1 = — a y + — a.z a.x = — a.h 2 2 2 2 =»y + z- x=h. Mà 2x = y + z. Do đỏ x = h. I . o-. Nhận xét. Các bạn được thưởng ki này: Trần Hổng Quỷ, 8E1. THCS Vĩnh Tưỡng. “ Vĩnh Tường Lẽ Hổng Nhung. 7A, THCS Vĩnh Yên, TP. Vĩnh Yên, Vĩnh Phúc; Nguyễn Đức Phú. 3A1 . THCS Nghi Hương, TX. cửa Lò, Nghệ An; Nguyễn Khắc Thái Bình, 8B, THCS Nguyễn Thượng Hiền, ứng Hòa, Hả Nội; Nguyễn Chi Công, 8A3, THCS Lâm Thao, Lảm Thao, Phú Thọ. NGÙYẾN HIỆP Nhãn 790 nãm vương triê ự Trân 1226 - 2016 ĨHlên ĨRlilG 10.1.1226 Trần cảnh lên ngôi Vua. Bắt đẳu vương triều Trần. Hương Tức Mạc lã quê hương nhà Trần được xây cất nhiều công trình. Trấn Thủ Độ làm Quốc thượng phụ. 1239 Vua Trần Thái Tỏng lệnh cho Phùng Tá Chu lá Nhập nối Thái phó dựhg hành cung â đây. Cung Trùng Quang vã cung Trùng Hoa được xây dựng. Cung Trùng Quang dành cho Thượng hoàng, cưng Trùng Hoa dành cho Vua đương nhiệm. 1262 Hương Tire Mạc được thẳng là Phủ Thiên Trương vai trò như kinh đỏ thứ 2 của nhả Trần. 1232 Năm sinh Trần Quốc Tuấn {sau là Trần Hưng Đạo) 1258 Quân doanh Vị Hoàng ra đói. 1258 - 1285 Sông Vị Hoàng được đào. 1258, 1285, 1288 Ba lển chiến thẳng quân Nguyên Mỏng. 1293 Trần Nhán Tỏng nhường ngôi, ổng lập Thiển phải Trúc Lám Yên Tử. 1300 Trần Hưng Đạo mất. 1305 Tháp chùa Phổ Minh được xây dựng. Chùa xây nãm 1108. 1428 Lẽ Thái Tổ chìa nước ta thảnh 5 đạo. Nam Định lúc đó thuộc Nam Đạo. 1446 Nam Định thuộc trấn Sơn Nam thượng. 1466 Lộ Thiên Trường đổi lảm Thiên Trương thừa tuyên. 1469 Thiên Trường thừa tuyên dổi thành Sơn Nam thừa tuyên. 1471 Đắp đè sòng Hồng. 1741 Nam Định là ly sở của Sơn Nam Hạ. 1786 Nguyễn Huệ ra Vị Hoàng. 1788 Nam Định là phủ lị củá trấn sơn Nam Hạ. 1802 Nhá Nguyền cho xây thảnh Nam Định, 1804 thành được xây xong. 1810 Thành Nam Định cố 34000 người. 1812 Nhà Nguyễn cho xảy cột cờ Nam ĐỊnlì {1 trong 3 cột cồ chính: Huế, Hà Nội, Nam Định). 1813 Bắt đẩu thi Hương ở Nam Định. 1820 Nam Định lá 1 trong 7 trường thi cả nƯƠC- 1822 Sơn Nam Hạ đổi thành trấn Nam Định, Tên gọi Nam Định chính thức ra đời. 1831 Trấn Nam Định đổi thảnh tỉnh Nam Định. 1832 Đáo sõng mơi, gọi lã sông Đáo. 1833 Xáy lại thành Nam Định. 1846 Xảy chùa Vọng Cung. 1850 Hình thành chợ Vị Hoàng và chợ Rong. 1853 Cột cờ thảnh Nam Định được xây dựng quy mô
  72. U! HORÍỈG - Mi ĐỊÍÌH BỈNH NAM HÀ như ngày nay. 1865 TựĐírc đổi láng Vị Hoàng thành Vị Xuyên. 10.12.1873 Pháp đánh thành Nam Định lắn thứ nhất. 1874 Pháp trả lại Nam Định cho triều đình Huế. 27.3.1883 Pháp dành thánh Nam Định lẩn thứ hai. 1885 Trương thỉ Hà Nội nhập vãũ Nam Dịnh. Xây dựng Nhà máy làm chất anbumin tại Nam Định. 1887 Còng sứ Nam Định thảnh Tổng đốc cai quàn tình Nam Định và Ninh Bình gọi lả Tổng đốc Định Ninh. Xây dựng Nhà thờ Lớn tại Nam Định. 1888 Khánh thành trường đua ngựa Nam Định. 1889 Thành lập phàn xưởng kèo sdi của Bá Chín Hội, 100 công nhân. 1890 Cắt một phấn đất tỉnh Nam Định lập tình Thái Binh, một phần đất Hà Nội và một phẩn Nam Định lập tình Há Nam. 1894 Lập nhà máy Rượu. 1B95 Lập nhà máy Chai. Nam Định được chia làm 10 phố, 10 phô chia làm 40 đường. Nay còn đường Hà Nội, đường Bắc Ninh, đường Ninh Bình, đường Hưng Yên. đường VỊ Xuyên, đương Phù Long, đường Nâng Tỉnh, Hàng Đồng, Hàng sắt, Bến Gỗ, Đống An, Cột cơ giữtẻn cũ. 1897 Nam Định có 30000 dãn. Hà Nội có 30000 dân, Hải Phòng 15000 dàn, Sài Gòn 30000 dân vá Chợ Lởn 100000 dân. Nam Định là thành phố lón thứ 4. Lễ xương danh Khoa thi cọ 30000 ngươi dự. 1898 Toàn quyển Đông Dương quyết định xây diíng đường sất Hà Nội - Nam Định. 1900 Dựng xưởng sợi A vá xưỗng cơ khí do Dupré hùn vốn với Bá Chín Hội. Đây là tiễn thân Nhà máy Dệt Nam Định- 1902 Dựng xưởng bông sợi thứ 3, 8.1.1903 Khánh thánh dường sắt Hà Nội - Nam Định. 1905 Khánh thành đương sắt Nam Định - Vinh. Nam Định cùng Hà Nội, Huế, Gia Định được mồ trương Sư phạm. 9.12.1908 Sáp nhập các trường Thông ngôn Hà Nội, sư phạm Hà Nội, Jules Ferry Nam Định {tút Thành Chung) thành trưởng Thành Chung Bảo hộ, Đây chính là trương Bưởi - Chu Văn An. 1913 Sòng Vị Hoàng bắt đẩu bị lấp. 1915 Khoa thi Hương cuối cùng ở miễn Bắc tại Nam Dịnh. 1917 Hoàn thánh 1 động cơ Nhà máy Điện. 1919 Nam Định nhập với Hà Nam thành tỉnh Nam Định. Đáp: Hỏi: Anh Phó ơi, tại sao khi tham gia chuyên mục cuộc thi giải toán qua thư theo năm học thì mỗi bài toán phải làm vào một tờ giây riêng và ghi đẩy đủ họ và tên, lớp trường, huyện, quận, tỉnh, thành ạ? ĐỖ THANH THANH TÚ (9A10, THCS Giảng Võ, Ba Đinh, Hà Nội) Đáp: Chuyện này anh nói mãi rồi Mỗi bài lại chuyển cho người khác nhau Anh thi có chấm được đấu Gửỉ cho anh tất mọi cấu thì phiên. Hỏì: Em thấy tạp chí có chuyên mục cuộc thi giải toán dành cho nữ sinh, không biết tòa soạn có mỏ chuyên mục cuộc thi giải toán dành cho các bạn nam không ạ? ĐỖ THU HÀ (6A8, THCS Thành Công, Ba Đinh , Hà Nội) ANH PHÒ Sau này nếu anh có tiền Sẽ ra tờ báo dành riêng Các bạn nữ làm tất cả Ra đề cho bạn nam nhi Thế rồi tất cả ổự thi Gửi bài về cho nữ chấm. Hỏi: Trong lởp em có một bạn học rất giỏi nhưng bạn ấy hơi nhút nhát, rất ít giơ tay phát biểu, Em phải làm thế nào để động viên bạn ấy ạ? NGUYỄN NGỌC MINH (THCS Vĩnh Tường, Vĩnh Tường, Vĩnh Phúc) Đáp: Học giỏi tà tốt rồi Ra đời thêm vốn sống Rèn luyện nói chỗ đông Mỗi ngày thêm tẩn tới Giò thì em chờ đợi Rạn hãy vào website: //olm.vn/hicu-sach -Online để dọc tạp chí Toán Tuổi thơ bán điện tử nhé. CÁC LỚP 6 & 7 Bài 1(163). Tìm các số tự nhiên a r b f c sao cho a nhỏ nhất thỏa mãn 7a 2 - 9b 2 + 29 = 0 và 9b 2 - llc 2 - 25 = 0. CAO NGỌC TOẢN (GV. THPT Tam Giang, Phong Điền, Thừa Thiên - Huế) Bải 2(163). Cho tam giác ABC vuông tạì A vãi đưàng cao AH (AB < AC). Trên tia AH lấy điểm D sao cho AD - BC. So sánh AB CD vàAC BD. NGUYỄN KHÁNH NGUYÊN ('Só'3/29E, đường Đà Nẵng, Hải Phòng) CÁC LỚP THCS Bài 3(163). Giải phương trình Jx- — + 5.Ỉ1-— + 2= 3x -I- — - V X V X X NGUYỄN ĐỨC TẤN (TP. Hồ Chí Minh) Bài 4(163). Tìm giá trị lởn nhất của biểu thức 13b p - a 2 +b 2 " 1,1 a b 4a— +4, trong đó a, b là các sô \ - - ỉ 4 thực thỏa mân 1 <a s2; 1 <b<2. NGUYẾN VĂN NHO (GV, THPT Nguyễn Duy Trình, Nghi Lộc, Nghệ An) Bài 5(163). Cho bản đổ M trong hình vẽ. Mỗi cách tô màu đòi hỏi hai vùng kể nhau khống cùng màu.
  73. Tìm một cách tô bằng 4 màu cho bản đó đề.
  74. M có tô được bằng 3 màu không? Tại sao? VŨ KIM THỦY Bài 6(163). Dựhg ra phía ngoài tam giác ABC đã cho các tam giác đéu ABE và ACF. Gọi M, p thứ tự là trung điểm của BC. EF. Gọi H là hình chiếu vuông góc của Atrẽn EF. Chứng minh rằng MP = MH. NGUYẺN MINH HÀ (GV. trường THPT chuyên Đại học Sư phạm Hà Nội) SOLUE Ultì MAIL C0IY1PETITI0IỈ DUESTIONS Translated by Nam Vũ Thành 1(163). Find thẽ natural numbers a, b, and c su ch that â is minimum and that 7â z - 9 í) 2 + 29 = 0 and 9ÍJ Z - llc 2 - 25 = 0. 2(163). Gi ven a triangle ABC with the ríght angle at A , AB < AC, and its height AH. Let D be a point on the ray AH such that AD = BC. Compare AB CD and AC-BD. PHIẾU ĐĂNG Kí THAM Dự CUỘC THI GĨTQT NĂM HỌC 2016-2017 3(163). Solvethe following equation Jx- — + 5.Í1-— 1-2 - 3x +— . V X V X X 4(163). Given the real numbers a and b such that 1 < a < 2 and 1 < b < 2. Find the maximum value ofthe expression p - a 2 I b 2 -| — 1 |-4a 1 4. V 5(163). Given a map M as in the following tigure. The map is to be colored in such a way that any two adjacent regions are not of the same color.
  75. Fínd a way to colorthe map using 4 colors.
  76. Can the map M be colored using 3 colors? Why? 6(163). On the outside of a given triargle ABC , construct equilateral trĩangles ABE and ACF. Let M and p be the midpoints of BC and EF, respectively. Let H be the orthogonal projection ũf A onto EF. Prove that MP = MH. Hộ ì đồng $ư phạm nhà trường Trường THCS Vinh Tường được đâL tại trung Lâm thị trấn Vinh Tường, LrƯơc mặt lã mội hỏ nưửc trong xanh, Lhơ mộng, Trong 40 năm xây đựng t đổi mơi vã phát triển, càc thê hệ thảy vã trò trường THCS Vinh Tưởng đã không ngừng nỗ lực vun đầp để ngôi trường mãi lã điểm sáng của giáo dục tỉnh Vĩnh Phúc. Bước vào thơi ki đổỉ mửỉ, bên cạnh việc giữ vững nẻ í lốp truyền thống, THCS Vĩnh Tưởng đã tạo dựng cho minh một diện mạo mớỉ. Hoạt động dạy và học cửa nhủ trưởng đẻu mang thõng diẽp Tắm nhìn, Tăm huyết, Trí tuệ” đảo tạo nguồn nhãn lực chất lượng cao cho tỉnh vinh PMc trong sự nghiệp cõng nghiệp hóa, hiện dạl hỏa đất nước. Đội ngũ giáo vĩẽn của trường ngày càng vũng tay nghề, nhiều thầy cô giáo dã trở thành giáo viên dạy giỏi cấp huyện, cấp tinh. Vớt những nồ lực của thầy và trò nhà trường, củng sự lãnh dạo thống nhất và đúng đán của chi bộ, ban giám hiệu, chất lượng giáo dục của nhả trường luôn dược duy tri vả tiến bộ. Học sinh xếp loại văn hoả Giỏi chiếm từ 75% dến 80%, số học sình xếp loại Trung bình chỉ còn dưới 0 n 5%- Cô 100% học sinh xếp loại hạnh klem Khá và TỐL, không cò học sinh vi phạm tệ nạn xả hội. Hảng năm có 50% số học sinh iởp 9 thí dỗ vảo trường THPT chuyên Vĩnh Phũc; tử 10 đến 20 học sinh dỗ vảo trường TIIPT chuyên của các trường Dại học, Chất lương học sinh gioi hàng năm của trương được duy trì và khống ngưng phất triển* Từ nãm học 1997-1998 dến nay, nhả trương dả cỏ: 1 huy chương Đồng các môn Khoa học thế glởỉ (Ld.S.O): 2 HCV, 1 HCR Olympic: toán Singapore má rộng: 1 giai Nhất Lhi Luyẽn truyền gỉửi thiệu sả ch Loàn quốc; 132 giải Khu vực, Quoe gia: Olympic toán Hã Nội mở rộng, thi giải toán qua mạng, thi Tiếng Anh qua mạng, thi giai toán trên máy tinh Casio; thi Thể dục Thể thao (Trong dỏ tổng cộng cò 46 giai Nhất). Thi học sirih giối cấp tỉnh lớp 9 cỏ 1951 giải các môn vịn hỏa (Trong dỏ cũ 126 glảỉ Nhất). Tạp chí Toán Tuổi thơ được học sinh nhà trường rất yêu thích, mỗl năm cỏ tữ 3 đến 5 em đoạt giải Cuộc thi giải toán qua thư theo Iiãm học. Từ năm học 2008-2009 đốn nay, mõ ĩ nãm nhà trường có ĩt nhất 2 học sinh dự thỉ Olỵmpic Toán Tuổi thơ toàn quốc; Cuộc thi Cầu lạc bõ Toán Tuổi thơ toản quốc vả đã có: 10 HCV t 9 HCB, 3 HCĐ và 2 giải Khuyến Khích, Đặc biệt nám học 2008-2009 nhà trường có 5 học sinh (cả tỉnh Vinh Phúc có 6 học sính) tham dự và cầ 5 em doạt giải VỚI 2 ITCV, 3 TTCR. Ghi nhặn những kết quả dã dạt dƯỢc, nhiều năm URND tỉnh VTnh Phúc công nhận trương THCS vinh Tường lả dơn vị Lá cờ dầu bậc THCS; năm học 2012-2013 dưdc Thủ tướng Chinh phu tang cơ Lhỉ dua đơn vị xuất, sắc vả tháng 11 nãm 201 5 r nhã trương vinh dự được Chủ tích nưửc tặng I- hưởng Huân chương Lao dộng hạng Nhất, Giãy phép xuấì bản: sổ 31/GP-BVHTT cấp ngày 23/1 /2003 của Bộ Vãn hóa và Thông tín. Mã số: 8BTT163M16. In tại: Công ty cổ phẩn in Công Đoản Việt Nam, 1 67 Tây Sơn, Đống Đa. Hả Nội. In xong và nộp lưu chiểu tháng 10 nãm 2016. Lễ khai giảng năm học 2016-2017 Trường THGS Giáng Vò, quận Ba Đinh, Hả Nội lã trường THCS lớn hàng đảu loản quốc, vờỉ gẩn 200 cán bộ, giáo vlõn, cồng nhân viên vả gần 4000 học sinh, Nhả trưởng 1 nõn là ddn vị có [.hành tĩch cao cua thành phố Hà Nội về chất lượng dạy vã học. Năm học 2Ũ 15 2016, trường THCS Giảng Võ tự hầo là một điểm sáng của ngành Giáo dục quặn Ba Dĩnh vả của TP. Hả Nội, là năm học mã nhả trường cỏ thảnh ti ch vượt trội nhất trong suốt 26 năm qua. Thi giáo viên giỏi cấp quận, nhà trường củ s giáo viên dạt giải với 1 giải Xuất sắc [cỏ giảo Lê Thị Loan), 3 giái Nhất, 4 giải Nhi. Thi đo dùng dạy học tự lãm cấp quận: có 7 giải, trong dó 1 Nhất. 3 Nhì, 3 Ba, Thi giáo vlẽn giỏi cấp thảnh phố có 1 giáo viên doạt. giải Nhất mon Vật lí. Số học sinh đoạt Học bổng loại I là 246 em, loại II 264 em. Kết quả thi học sinh giỗi cấp quận: Học sính giói các môn văn hõa lởp 0; ũlympic liếng Anh 6. 7, tì' Violympic toán qua Iiiạng; tiêng Anh qua mạng (IDE); ũlymplc toán 8 có tổng cộng 432 giải, trong đó: 54 Nhất, 104 Nhì, 141 Ba É 133 giải Khuyến Khích. Kết quả thi hục sinh giỏi cấp thành phơ: Học sính giỏi các mõn vãn hóa lớp 9; ũlympíc tiếng Anh qua mạng [ĩ OE lớp 9); Vỉolympìc toàn qua mạng lởp 9; Olympic tiếng Anh thành phố [Languagelink) lửp 9; Olympỉc tu tin Hà Nội mô rộng có tổng cộng 142 giải, trong đố cõ: ỉ I Nhất, 43 Nhì, 63 Ra, 25 giảỉ Khuyến Khich. Thỉ học sinh giỏi cấp Quốc gia - Khu vực, có 26 giả ỉ, trong đó Vialympic t.oãn Quốc gia: 4 TICĐ; thi APMOPS (thi Loàn châu Á - Thải Binh Dựơng): 1 Bạch kim, 1 HCV, 4 HCĐ: cuộc thi Câu lạc hộ Toán Tuổi thớ toàn quốc 2016: 1 TICR, 3 HCĐ ? 2 giải Triển vọng; Tiếng Anh qua mạng (IŨE) lớp 9 cấp Quốc gia: 2 HCV, 3 HCB, 3 HCĐ, 1 giải Khuyến Khích; Qlympio Tài nàng liếng Anh toán quốc lản thử 111: 1 HCB. Thi học sinh gíỏỉ Quốc tế, tổng cộng có 108 giải. Nhà trưởng vinh dự được Bộ Giáo dục vả Đạo tạo, sỡ Giáo dục vả Đão tạo Hà Nội giao nhiệm vụ thành lập đoàn học sinh tham dự cuộc thl tại Thãi Lan và Trung Quốc. Thi Toán - Khoa học Quốc tế tại Thải Lan cỏ 12 học sinh tham gia vả doạt 12 Huy chương gồm 2 HCV, 6 HCB ? 4 HCĐ. Thỉ Vở dịch đội tuyển to ân tại Trung Quốc: 4 HCB, 4 ĨĨCÍ1. Tbl toán Quốc tế giữa các thành phố (IT0T-2015] có 5 Nhất, 7 Nhì, 17 Ra. Thi Toãn AMC 8 [Cuộc thi toàn Hoa Kỹ) có 3 HCV, 6 HCB, 5 HCĐ. Thl toàn Quốc lẽ giữa các thành phố (ỈT0T-2016) có 6 HCV, 6 HCB, 13 HCĐ. Thỉ tìm kiếm Lải nãng Toán học trẾ lần thử II, 2016 (MYT51 cũ 4 HCV, 7 HCB, 9 HCĐ. Các cuộc thl phong trảo nhà trưửng luỏn đạt thành tích cao cấp quặn, cấp thành phố vã cấp quốc gía, vớt tổng cộng 18 giải Nhất, 13 giải Nhì, 15 gỉái Ba. Thỉ thể thao cấp quận cõ 4 HCV, 3 HCB, 16 HCĐ; cấp thảnh phố củ 19 IICV, 16 HCB. 17 HCD, 1 giải dồng đội: cấp quốc gia có 1 1 HCV; 4 HCR- Cuối năm học 2015-2016, nhã trường cỏ 464 lượt học sính đỏ vào các trường THÍT chuyên vớí nhiều em dạt số dicm Thủ khoa. Nhà trưdng đà vinh dự dược tặng nhiêu danh hiệu vã phần thưởng cao quỷ của Nhã nưởc, Chĩnh phủ, của tliãnÌL phố Hà Nội và của ngành Giáo dục: Bằng khen của Chính phủ, Bàng khen của Bộ Giáo dục vầ Đào tạo; Huân chương Lao dộng hạng Ba năm 1997, hạng Nhi nãm 2004, hạng Nhất năm 2009: Huân chương Dộc lặp hạng Ba năm 2014. TRUNG HỌC Cơ SỞ NĂM THỨ MƯỜI BÂY ISSN 1859-2740 NÃM HỌC 2016 - 2017 Gỉá: 20000đ Childrens Fun Maths ơoumal NHÀ XUẤT BÀN GIÁO DỤC VIỆT NAM - BỘ GIÁO DỤC VÀ DÀO TẠO E rổng biên tập: Th$. vũ K|M THÚY CHỊU TRÁCH NHIỆM XUẤT BẢN Chù lịch Hội dông Thạnh vlẽn NXBGĐ Vlệl Nam; MẠC VÃN THIỆN Tầng Giám đỗc NXBGữ việt Ham: GS.TS.VÙ VẪN HÙNG Phô Tổng Giám đoc hỉẽm Tồng biên Ịập NXBGD Vlệl Nam: T5.PHAN XUÂN THÁNH Thư kí tò a soạn : T rưỏng ba n biên lậ ọ: NGUYÊN NGỌC HÀN TRẤN THỊ KIM CƯƠNG ỦY VIÊN NGND. vO HỮU BÍNH TS. GIANG KHẮC BĨNH TS. TRẦN ĐÌNH CHÂU T3. VŨ ĐÌNH CHUẨN TS. NGUYỄN MINH ĐỬC ThS. NGUYỄN ANH DŨNG TS. NGUYÊN MINH HÀ PGS. TS. LẾ QUỐC HÂN PGS. TSKH. VŨ ĐÌNH HÒA TS. NGUYẾNĐỬC HOÀNG ThS. NGUYỄN vũ LOAN NGUYÊN ĐỨC TẤN PGS.TS. TÔN THÂN TRƯỜNG CÒNG THÀNH PHẠM van trọng ThS. HỒ QUANG VINH TÒA SOẠN Tắng 5. sỏ 361 đường Trướng Chinh, quận Thanh Xuân, Hà NỘI Điện thoại (Tel)- 04.35GS2701 Điện sao (Fax): Ữ4. 35682702 Điện thư (Emaỉl): tapchitũanmứithũ@gmail.dam toantụQith o@vn n . vn Trang mạng (VVebsite)' //www.toantuoitho.vn NGUYÊN VIẾT XUÂN 391/150 Trấn Hung Đạo, p. cầu Kho. 0.1. Tp. HCM Đĩ: 08.66821199, DĐ: 0973 303199 Trị sự - Phát hành TRỊNH THỊ TUYỂT TRANG, VŨ ANH THƯ, NGUYỄN HUYỂN THANH Chê bản: Đố TRUNG KIẼN Mĩ thuật: Họa sĩ TÚ An TRONG SỐ NÀY Dành chõ học sinh lớp 6 & 7 Bài toán chứng minh chia hết có điều kiện Nguyễn Anh Tuân Vận dụng bất đẳng thức tam giác để có lòi giải của bài toán NguyỄn Đức Tấn Cửa SỔAC Myanmar gần và xa (Tiếp theo kì trước) Vũ Kim Thủy Nhìn ra thế giói Để thi Toán và Khoa học Quốc tế IMSO năm 2015 (Tiếp theo kì trước) Trịnh Hoài Dttơng Thách dả'u! Thách đã'u đãy! ]. Tai»l Trận đấu thứ một trăm bốn mưoĩ Nguyễn Duy Liên Cuộc thi tư duy thuật toán và tính toán Australia mở rộng Tạ Ngọc Trí Đến vãi tiếng Hán Bài 68. Hà Nội có rất nhiều viện bảo tảng (Tiếp theo kì trước) Nguyễn Vũ Loan Học Vật lí bằng tiếng Anh Unit21 . The Earths magnetism Mũrís Vũ Dành cho các nhà toán học nhỏ Suy nghĩ vể một dây bất đẳng thức Cao Minh Quang Một số bất đẳng thức trong tam giác và tứ giác Nguyễn Văn Ngạc Tr 11 Tr 19 Tr 55 [trong số này] Học ra sao? Giải toán thế nào? Một số kĩ thuật tính giá trị biểu thức Nguyễn Khánh Chung Các tính chất và đôi điểu thú vị vể tam vuông Nguyễn Đức Hảo Để thi các nước AMC 2014 Senior Division Đẽ Trung Kiên Kết quả Thi giải toán qua thư Bạn muốn du học Dự thi học bổng Singapore Thủy Vũ Compa vui tính Có khẳng định được không? Tạ Thập Phá án cùng thám tử âêlõccõc Cuộc gpi Quốc tế ỡổ Thị Hiền Anh Lịch sử Toán học Năm ngôi sao trong chòm sao đại số thuyết sô Lẻ Quốc Hán Bạn đọc phát hiện Suy luận và phát triển từ một bài toán Vương Thị Hải Chữ vả chữ sỏ' Kì 25 Trương Cõng Thành Toán tiêu dùng Thuê 6 tồ Vũ Mulberry giác Tr 34 Tr42 Tr 44 và lí Tr 4fi Tr 56 Đo trí thông minh lẽn so Nguyễn Đức Tấn Sai ở dâu? sửa cho đúng Bạn có ý kiến gi khấc? Nguyễn Ngọc Hùng Chuyện dạy và học toán Kinh nghiệm giảng dạy các bài toán quỹ tích Nguyễn Thị Bính Giở ra chđi Vui cười ĐỖ Hồng Thịnh Trường Olympic Tuổi của một số thiết bị, vật dụng và cóng nghệ Bính Nom Hà Rubic hỏi... đáp Để thi giải toán qua thư Tr Ê4 Tf 62 BÀI TOÁN CHỨNG MINH CHIA HẾT CÓ ĐIỀU KIỆN NGUYỄN ANH TUẤN (GV. THCS Hòa Hiếu 2, TX. Thải Hòa, Nghệ An) T rong quá trình giải toán về phép chia các sổ nguyên chúng ta thưởng gặp dạng toán sau: Cho bĩểtA : m chứng minh rằng B : m, hoặc Chứng minh rằng A m B : m. Ỷ tưởng chính để giải dạng toán này là chứng minh {kAi tB) : m trong đó k, t e N* và (k, m) = 1, (t, m) = 1 , từ đó ta sẽ suy ra đước điều phải chửng minh. Trong bài viết nảy, chúng ta thường sử dụng các kiến thức cơ bàn sau:
  77. Nếu tổng (hiệu) của hai số chia hết cho m và một trong hai số đó chia hết cho m thì số còn lại chìa hết chù m.
  78. Mọi số tự nhiên đều viết được dưới dạng tổng các chữ sô của nó cộng vối một số chia hết cho 9.
  79. Nếu tích a.b chia hết cho c mà (b, c) = 1 thi a chia hết cho c. Chúng ta cùng xét một sỏ ví dụ sau: Ví dụ 1. Cho a, be z, chứng minh rằng 2a + 5b ! 11 <=»a+8b : 11. Ldi giải. Cách 1. Ta có 5(2a + 5b) + (a + 8b) = 11(a + 3b) : 11. Do đó a 4- 3b i 11 É 5(2a + 5b) i 11

    c->2a + 5b : 11 (vì (5, 11) = 1). Cách 2. Ta có 6(2a + 5b) - (a 4- Sb) = Il(a 4- 2b) í 11 . Do đó a + 3b : 11 « 6(2a + 5b) : 11

    2a + 5b i 11 (vì (6, 11) = 1). Các bạn giải tương tự như trên trong cách 3, 4, 5 dưới đây vá hẫy tìm thêm các hệ thức khác như thế. Cách 3. (2a I 5b) + 9(a I 8b) = 11(a I 7b) : 11. Cách 4. 13(a+ Sb) - (2a + 5b)= 11 (a + 9b) : 11. Cách 5. 2{a +8b) - (2a +5b) = 11b ; 11. Ví dụ 2. Cho A = 7 n í 3n-1 và B = 7 n+1 + 3(n + 1)
  80. 1 (với n e N). Chứng minh rằng A : 9 B ; 9. Lời giải. Ta cỏ 7A - B = 7(7 n + 3n - 1) - (7 n + 3(n + 1) - 1) = 18n - 9 = 9(2n - 1) i 9. Do đó A ; 9 « 7A ! 9 ư B : 9 (vì (7, 9) = 1). Ví dụ 3. a) Chửng minh rằng abc + deg:37 <-> abcdeg:37.
  81. Chứng minh rằng abc-degl7 <= abcdeg'7.
  82. Chứng minh rằng abcdeí : 1 1 <=> ab +cd +ef ‘11. Lờí giải, a) Ta cố abcdeg -(abc 4deg) = (1Ũ00.abc +deg) -(abc +deg) = 999.ãbc=37.27.ãbẽ:37. Do đó abc + deg:37 <-> abcdeg:37.
  83. Ta có abcdeg + (abc-deg) = (1000.abc I deg)’t (abc-deg) = 1001 ãbẽ:7 Do đó abc -deg:7 » abcdegl?.
  84. Ta có abcdeí - (ab 4- cd 4- ef) = (10C00ab + 100cd + ef)-{ab + cd -4-eí) = 1 1(909ab + 9cd) 111. Từ đó suy ra abcdet :1 1 <=> (ab + cd + ef} Í1 1 . Vi dụ 4. Cho một số tự nhiên cớ ba chữ số chia hết cho 37. Chứng minh rằng bằng cách hoán vị vòng quanh các chữ số của số đó, ta được hai số nữa cũng chia hết cho 37. 3 Ldi giải. Giả sử abC’37, khi đô ta cẩn chứng rnính beai37 và cab:37. Thật vậy, ta có lO.abc -bca = (1000a +1ũ.bc) -{10. bc +a} = 999 a - 37.27,a :37. Suy ra bcai37 {vì abc:37 nên lOâbc :37 ). lo.cab-abc =(1000c +10ab) -{10ab +c} = 999c = 37.27.c :37. Suy ra 10.cab:37 (vì abc:37). Vậy cãb:37 (vì (10, 37) = 1). Ví dụ 5. Cho một số tự nhiên chia hết cho 7 góm sáu chữ số. Chứng minh rằng nếu chuyển chữ số đầu xuống cuối cùng, ta vẫn được một số chia hết cho 7. Lòi giải. Giả sử abedẽfị7, ta cá 1Ũ s .bcdefa -abedeí = I0 5 (1ũ.bcdef +a) -(10 s .a ibcdeí) = 999999 bcdef =7.142857.bcdef 7 Vi abcdeĩ 17 nên 10 5 .bcdefa :7 => bcdifã'7 (vì(10 5 , 7) =1). Ví dụ 6. Cho một số tự nhiên chia hết cho 7 có ba chữ số, trong đố chữ sô hàng chục bằng chữ số hàng đđn vị. Chứhg minh rằng tổng các chữ số của nỏ chia hết cho 7. Lởi giải. Giả sử abb!7, ta có 100(a+b+b)-ãbb = 100(a + 2b) -{100a f10b - b) =189b7. Suy ra 100(a + 2b) : 7=> a + 2b ; 7 (vi (100, 7) = 1). Ví dụ 7. Chứng minh rằng 5a 2 + 15ab - b 2 : 49 3a + b : 7 (vãi a, b F z). Lài giải. Nếu 5a 2 + 1 5ab - b 2 : 49 => 5a 2 - 15ab- b 2 ; 7. (1) Mặt khác lại có (5a z + 1 Sab - b 2 ) + (3a + b) 2 = 7a(2a + 3b) : 7. (2) Từ (1) và (2) suy ra (3a + b) 2 • 7 =ỉ> 3a + b : 7 (vì 7 lâ số nguyên tố), • Nếu 3a + b : 7, ta cỏ (3a + b) + 2(2a + 3b) = 7(a + b) : 7 => 2(2a + 3b) i 7 => 2a + 3b i 7 (vì (2, 7) - 1). Suy ra (5a 2 + ISab - b 2 ) + (3a + b) 2 = 7a(2a + 3b) : 49, (3) Vi 3a 4- b i 7 nẻn (3a 4- b) 2 i 49. (4) Từ (3) và (4) suy ra 5a 2 + 15ab - b 2 : 49, Vậy 5a 2 + 15ab - b 2 ị 49 í=> 3a + b i 7. Nhận xét. Chủng tạ có thể vận dụng ý tưởng trên vào bài toán chứng minh chia hết khổng cù diều kiện, tức là muốn chứng minh A : m ta sẽ chíỉng minh (A ± km) : m trong đó k e N. Đài tập tự luyện Bài 1. Chứhg minh rằng nêu g 4- 3e 4- 2d - c - 3b
  85. 2a : 7 thì abcdegi7. Bài 2. Chứng minh rằng:
  86. abcd:29 <=> (a ị 30 4 9c I 27d):29.
  87. abc ! 23 <-> 30a + 3b — 2c ! 23 Bài 3. Cho X, y e N. Chửng minh rằng 2x4- 3y : 17 <=» 9x + 5y : 17. Bài 4. a) Chứng minh rằng 10 n 4- 72n - 1 : 81 (n Ể N)
  88. Chứng minh rằng số gom 27 nhóm chữ số 10 thì chia hết cho 27,
  89. Hai số tự nhiên a và 4â có tổng các chữ số bằng nhau, Chứng minh rằng a chia hết cho 3. Bải 5. Cho X, y e z, chứng minh rằng
  90. X 2 + 20xy + 269y 2 : 169 <y.ì 2x + 7y : 13-
  91. 3x+2y : 11 «x 2 + 16xy-57y 2 ; 121, Bài 6. Chứng minh rẳng với mọi ne N thì
  92. 9 2n + 2009 ; 10;
  93. 9,1 0 n + 72 ; 81. Bài 7. a) Hai số tự nhiên a và 2a đểu có tổng các chữ số bàng k. Chứng minh rằng a chia hết cho 9-
  94. Chứng minh rằng 1 0 n I 1 Bn - 1 chìa hết cho 27.

    c} Chứng minh rằng số gồm 81 chữ số 1 thì chia hết cho 81. 4 2 VẬN DỤNG BẤT ĐẲNG THỨC TAM GIÁC DỂ CÓ LỜI GIẢI CỦA BÀI TOÁN V ận dụng bất đẳng thức vể cạnh trong tam giác không chỉ giúp ta có được lời giải bài tũàn mà đôi khi còn cho chúng ta tời giải đẹp. Sau đây, chúng tõi xin gĩởi thiệu đến cãc bạn một sổ hài toán minh họa về phương pháp này.

    Bất đẳng thức về các cạnh trong tam giác: Trong một tam giác có độ dài ba cạnh là a, b, c thì |b - c| < a < b + c.
  95. Bài toán 1. Cho a. b ; c là độ dài các cạnh của một 3 h c tam giác. Chứhg minh rằng — — — H — - — + — — < 2. b + c c + a a-t-b Lài giải. Theo bất đầng thức tam giác, ta có b + c > a.
  96. A , a 2a 2a

    Do đó = < ; . b + c b-c + b7rc a+b + c T ương tự, ta có b 2b , c 2c — ' — < — r — và — — — < — ; c + a a + b-c a-rb a + b + c . „ a b c 2a -4 2b I 2c „ Vậy + + — — < ; = 2. b + c c + a a rb a + b + c 8ài toán 2. Cho X, y là hai số dương. Chứng minh rằng + V y > V x + ỹ- Ldi giải. A Xét AABC vuông tại A có AB = v'x, AC = ýỹ. Vì AABC vuông tại A nên ta có BC 2 = AB 2 + AC 2 (theo định lí Pythagoras). Do đó BC 2 = X + y (BC >0) ;=> BC = yỊx + y. NGUYỄN ĐỨC TẤN ỢR Hổ Chi Minh) Mà trong AABC cố AB + AC > BC (bất đầng thức tam giác). Vậy Vx > ^x+ y. Bài toán 3. Cho X, y là hai số dương vả X > y. Chứng minh rằng Vx - ýỹ < -Jx - y , Lòi giải. A B c Xét aABC vuỏng tại A cỏ BC = >/x, AB = ựỹ. Vì AABC vuông tại A nên AB 2 + AC 2 = BC 2 (theo định lí pythagoras). Do đó y + AC 2 = X => AC - ýx - ý (vì X > y). Lại có BC - AB < AC (bất đẳng thức tam giác), Vậy y/x - ựy < ựx - y. 5 Bài toán 4. Cho tam giác ABC có G là trọng tâm. Chứng minh rầng GA 2 4 - ŨB 2 4 GC 2 < 2(GA.GB 4 GB.GC 4 GC GA) Lài giải. A D Gọi M là giáo điểm của AG và BC. Trên tia đối của tia MG lẩy điểm D sao cho MD - MG. 1 _ Ta có MG = AG (vi G là trọng tảm của AABC) và MB B MC. Mà GD = MG + MD và MD = MG (giả thiết) nên GD = GA Xét aMGB và aMDC có MG = MD GMB - DMC (hai góc đối đỉnh) MB - MC. Vậy AMGB - AMDC (c.g.c) => GB ■ CD- Trong AGCD có GD < GC 4 CD. CD < GC 4 GD, GC < GD 4 CD (bất đẳng thức tam giác). Do đó GA< GC + GB, GB < GC ■ GA, GC < 6A+ GB =>GA 2 <GA.GC I GA.GB, GB 2 <GB.GC + GA.GB. GC 2 < GC.GA + GB.GC. Suy ra GA 2 4 GB 2 4 GC 2 < 2{GA.GB 4 GB.GC 4 GC.GA). Đài toán 5. Cho hai điểm A. B nằm ngoài đường thẳng d và nằm cùng phía đối với đường thẳng d. Xác định vị trí điểm M trên đường thẳng d để MA 4 MB đạt giá trị nhỏ nhất. Lòi c Vẽ AH 1 d tại H. Trên tia đối của tia HA lấy điểm c sao cho HC - HA. Ta có d là đường trung trực của đoạn thẳng AC. Ggi N là giao điểm của BC và đường thẳng d. Vì M, N e d nên NA = NC, MA - MC.

    Nếu M = N thi MA+ MB = NC 4 NB a BC. Nếu M khàng trùng với N thì trong AMBC có

    MC 4 MB > BC (bất đẩng thửc tam giác). Vậy khi điểm M trùng vâi điểm N thì MA I MB đạt giá trị nhỏ nhất.

    Cách dựng điểm M: Vẽ AH 1 d tại H. Trên tia đối của tia HA lấy c sao cho HC = HA. Khi đó M là giao điểm cùa BC và d. ® 1 MYANMAR GAN VA XA (Tiếp theo kì trưôc) VŨ KIM THỦY AC là từ viết tắt của Cộng đồng ASEAN bằng tiếng Anh (ASEAN Communitỵ). Cộng đổng ASEAN thành lập chính thức từ 31.12.2015. Năm 2016 này tạp Ghi Toán Tuổi thơ mở chuyên mục cửa sổAC đề bạn đọc hiểu hơn về vùng đất, con người của 10 quốc gia vòi 625 tríệu dân. ói tiếp về những khác lạ mà chung ta thấy khi đến nước bạn. Khu vực quảng trường trung tâm khá giống các thành phố châu Âu. Điếu này vẫn tương đồng với khu vực Nhà hảt Lớn, Hà Nội. Nhưng cái khác ở đây là kiến trúc mang dáng dấp khác, không giống Pháp mà như kiểu Anh. Ai đã đến Malaysia thì thấy khu vực quảng trường trung tâm â hai nước khá giống nhau. Một cây tháp cao với kiến trúc đơn giản vươn cao giữa quảng trưửng giống tháp Độc lập ở Indonesia nhưng ở đây nhỏ hơn. Nhân tiện kể về kiến trúc nói thêm về nhà ở của dán Yàngon. Nhíểu ngòi nhà cồ ở những khu pho cũ chắc có tuổi gần tám, chín thập niên. Nhìn qua nó có dáng dấp khu nhà tập thể. Chiều chưa muộn nhưng nhìn vào cầu thang đã thấy tối. Cầu thang đi ngay từ hè phố. lẽn thẳng một mạch đến tầng trên chứ không rẽ ngoặt có chiếu nghỉ. Lên trên hai bên là hai nhả. Vậy là cầu thang thi chung nhưng hai nhá hai bền khá độc lập. Điều lạ nhất là nhiều nhà 6, 7, 3 tầng ở mặt phố lân không có thang máy. Mỗi gia đình ở các tầng trên thả sãn một sợi dây, đẩu có treo các hộp hoặc túi. Các túi đó treo cả ngày trước cửa nhà tầng 1, ngay tẩm mắt người đi đường, nhìn khá lạ mắt. Chẳng ai lấy thế làm điéu. Khi chủ nhà cần mua (gia vị. quà sáng, nhu yếu phẩm .) họ gọi cho nhà hàng. Người mang hàng đến chỉ việc bỏ vào túi có sắn ghi địa chỉ phòng, chủ nhà kéo lén rổi thả tiền vào hạ xuống trả cho nhà hàng. Có khi túi treo từ sáng đến trưa vì chủ nhà đi vẩng. Một điều đáng nhó là các nét vãn hóa Phật giáo mà bạn được trải nghiệm. Các khu chùa đều rộng lân. Chùa Vàng ở Yangon có cả khu thang mảy đưa khách tham quan lên khu vực sảnh chính. Trong toàn bộ các khu chùa khách tham quan đểu đi chân đất (thưởng là chân trần, không tất). Ở chùa Vàng tại trung tâm thì có giá cho khách để dép, khi ra được phát giấy ướt lau chân, ở các khu chùa bình thường khác giày dép để ngoài cổng trước khi vào sân. Vậy nên sang Myanma đi dép xỏ ngón vừa giong người sở tại vừa tiện khi vào chùa không phải tháo giầy, tất. Trang phục ở nhiều ngôi chùa quy định là quần phải tới mắt cá chân. Bạn được hướng dẫn mua trang phục để phù hợp nếu lỡ mặc kiểu quần dã ngoại chưa tói đầu gối. Điểu khó quên nhâ't là 11 km lên chùa đá vàng hay chùa Kyaikhtiyo. Đường dốc ngoằn nghoèo như lên Tam Đảo. Xe chuyên dụng là xe tải như trong quân đội. Có 6 hàng ghế inox bắc ngang, mỗi hàng ghế 6 người, vừa tối đa số khách một xe chỏ được vừa không làm người ngồi bị xô qua xô lại khi xe vào cua và lên, xuống dốc, Rất nhiều ngưòi sợ khi đi chuyến xe này vì đường dốc và hẹp, xe mui trần. Nhưng may là xe chỉ đi một chiều, có chặng nghỉ, chờ xe đi ngược lại qua hết. Lên đến nơi là cả một không gian kĩ thú ngút tầm mắt. Xa xa là ngôi chùa vàng xây trên tảng đá. (Còn tiếp) ® ĐỂ THI TOÁN VÀ KHOA HỌC QUỐC TẾ IMSO NĂM 2015 PHẦN CÂU HỎI CÓ CÂU TRẢ LỜI NGẮN (Tiếp theo kỉ truởc) TRỊNH HOÀI DƯƠNG (GV. THCS Giảng Võ, Be Đình, Hà Nội) Sưu tắm và giới thiệu MAI VŨ (dịch)
  97. Trong cách sắp xếp à hình dưối đây, mỗi sò' là hiệu số không âm của 2 số ở phía trên liền vối nó. Tính số trung binh cộng của 8 gíả trị có thể được của 2. 0
  98. Có bao nhiêu sổ nguyên dương từ 1 đến 2015 khỏng chia hếl cho một trong các số sau: 2, 20, 201 và 2015?
  99. Trong một cuộc khảo sát 100 sinh viên, có 84 sinh viên khỏng thích chơi quần vợt, 74 sinh viên không thích chơi trượt tuyết, 62 sinh viên không thích chơi cả quẩn vợt và trượt tuyết, vậy có bao nhiêu sinh viên thích chơi cả quấn vợt vá trượt tuyết?
  100. Biết rằng 0, 2, 4, 6, 8 là các chửsố chẵn, cần bao nhiêu chữ số chẩn để viết các số từ 1 đến 100?
  101. Hai số được gọi là "cặp số gương" nếu một trong hai số đó được tạo thành bằng việc đảo ngược thử tự các chữ số của số kia. Ví dụ, 123 và 321 . Nếu tích của một căp số gương là 146047, thì lổng của cặp số gương này li bao nhiêu?
  102. Trong hình lục giác dưới đây. số 1 được đật vào tam giác cao nhất. Cố bao nhiêu cách khác nhau đặt các số 2, 3, 4, 5 và 6 vào các tam giác trống còn lại sao cho tổng các số trong các cặp tam giác đối nhau qua tàm lục giác là 5, 7, 9?
  103. Có 6 hình chóp giống nhau vởi đáy là hình vuông được xếp thành một hình lập phương có thể tích 2744 cm 3 (Xem hình 2). Tính độ dài đường cao OH từ đỉnh o đến đáy hình vuông ABCD của mồi hình chóp, theo cm (Xem hình 1).
  104. Có bao nhiéu số trong các số 1 X 2, 2 X 3, 3x4, 29 X 30 chia hết cho 3 hoặc 5?
  105. Một con cáo thấy có một con thỏ à phía trước cách nó 21 m. Trong một giây, con thỏ chạy 5 bưác, cáo chì chạy được 3 bước. Biết rằng khoảng cách 4 bưỏc của cáo bằng 9 bưủc của thỏ. Nếu khoảng cách mỗi bước chạy của thò là 0,6 m thi mất bao nhiêu giây để cáo bất được thỏ? 8
  106. Xét tất cả các số có 3 chữ sô sao cho các chữ sổ đó khác nhau và đều khác ũ. Tỉnh tổng tất cả các số có 3 chữ sô đó.
  107. Cho tam giác ABC vuông vởi € = 90°. Tia phản giác của góc A cắt CB tại Đ. Nếu CD = 3 em, BD — 5 em, tính đệ dài AB, theo em. A
  108. Hình vẽ dưới đây gổm 15 ò vuông đơn vị. ĐƯỬng AB chia hình đó thành 2 phần có diện tích bằng nhau. Tính giá tri của 5., • BN M A
  109. Vẽ hình vuông ABCD với các điểm E. F, G, H là trung điểm của các cạnh như hình vẽ. NẾU tổng diện tích phần được tó đậm là 15 em 2 , hãy tính diện tích hình vuông ABCD, thao em 2 . D G c A E B
  110. Các kí hiệu I, M, s, o, 1 và 5 được viết thành một hàng ngang theo thứ tự nào đó. (1) M có thể là kí hiệu đầu tiên hoặc cuối cùng từ bên trái. (2) s là ki hiệu thứ tư tính từ bẽn trái. (3) s â phía trái o và không nhất thiết ở cạnh o. (4) I ở bên phải M và không nhất thiết Cf cạnh M. (5) o và 1 đật cạnh nhau. (6) Chỉ cò đúng một kí hiệu nằm giữa I vả 1. Vậy kí hiệu thứ hai tính từ trái là kí hiệu nào?
  111. Cho ABCD là hình chữ nhật với điểm E thuộc CD và F là điểm thuộc BC sao cho ÁEF =90°, AF = 25 em. Độ dài cùa DE, EC, CF, FB, AE và EF là các sổ nguyên dương. Tính diện tích hình chữ nhật ABCD, theo em 2 . D E c
  112. Cho các sõ nguyên dưdng khác nhau có trung bình cộng là 38 và 52 là một số trong các số nguyên đó. Nẽu bỏ đi số 52, trung bình cộng của các số nguyên còn lại là 37. Tìm số nguyên dương lỏn nhất có thể có trong các số nguyên này.
  113. Tổng của 47 số nguyên dương khác nhau là 2015, Có nhiều nhất bao nhiêu số trong các số nguyên dương đó lả số lẻ?
  114. Ta gọi "phin $ô' đơn vị” là phin sổ trong đó có tử sô lả 1 và mẫu sô là một số nguyên dương nào đó. Ta biểu diễn số 1 bằng tổng của 7 phản số đơn vị khác nhau, biết 5 trong 7 phân số đó là 111 1 1 7 , 7 -, 77 -, 77 -. Tỉnh tích của 2 phân 50 đơn vị 3 5 9 15 30 còn lại.
  115. Từ 99 số nguyên dương nhò hơn 100. ta chọn

    nhiều số khác nhau sao cho khỏng có tập con nào của các số được chọn có tổng bằng 1 00. Nếu như tổng các số được chọn là lớn nhất, hãy tìm sổ nhò nhât trong các số đã chọn. THÁCH ĐẤU í THÁCH ĐẤU ĐÂY! TRẬN DẤU THỨ MỘT TRĂM BỐN MƯƠI ■ m Người thách đấu: Nguyễn Duy Liên, GV. THPT chuyên Vĩnh Phúc.

    ã 2b 5c

    Bài toán thách đấu: Tìm giá trị nhỏ nhất của biểu thức p = -— + — + - 4 , trong đó a, b, c lả các sổ thực dương thỏa mãn a z + b 2 + c 2 = 6. tỉC ca Xuất xử: Sáng tác. Thòi hạn: Trước ngày 08.12.2016 theo dấu bưu điện. TRẬN ĐẤU THỨ MỘT TRĂM BA MƯƠI TÁM (TTT2 số 161+162}

    , 1 — 2x , 1-2y 1-2z Ta có p = — — — - — 7+ , — - 4 — - — - + , ' — - — 7 x 3 +7x-(5-x)+1 y 3 +7y-(5-y)+1 z 3 +7z -{5 -z) +1 1-2x 1-2y , 1-2z x 3 +8x-4 y 3 +8y-4 z 3 -8z-4 Ta sẽ chứng minh 1-2x „ 1 1 «

    , ^x~4 (1) X 3 + 8x -4 20 4 Vì X > 1 nên (1) -» 1 - 2x X - 5 X 3 + 8x - 4 20

    20(1 - 2x) < (x 3 I ỗx - 4)(x - 5) <-> 20 - 40x < X 4 - 5x 3 + 8x 2 - 44x 4 20 ệỉX 4 - 5x 3 + 8x 2 - 4x > 0++ x(x 3 - 5x z + 8x - 4) ?- 0 <=> x(x - 1)(x 2 - 4x + 4) > 0 o x(x - 1)(x — 2) 2 > 0 (luôn đúng), Vậy (1 ) đúng, o x(x - 1)(x — 2f > 0 (luôn đúng), V 1— 2y 1 1 T ương tự ta có — r- — — — < -4y “4 y 3 J -8y-4 20* 4 Từ (1), (2), (3} suy ra p 0++ x(x 3 - 5x 2 + 8x - 4) 5: 0 <=> x(x - 1)(x 2 uôn đúng), Vậy (1) đúng, 1 1 t 1-2z 1 1 — <^y-j(2); £ <-z--.(3) -4 20 4 z 3 + 8z - 4 20 4 J 1 . 1 V ( 1 1 V ( 1 lì 1 , Ị 20 4 J 20 4 J 1 20 4 20 |{x-1)(x-2) 2 =0
  116. ,{y-1)(y-2} 2 =0 ' ' , „ li < v ' <-> y = 1,x = z = 2, {z~1)(z -2) 2 = 0 z=u = y = 2 1 , - 3 5 3 1 = -4(x + y + z}-4 = 44 . = -4. 20 4 20 4 2 |{x-1}(x-2) =0 Dấu đẳng thức xảy ra khi = ị<z-1){z-2> 2 'x + y + z = 5 Vậy maxP =!»- khi chẳng hạn (x, y, z) = (1, 2, 3), Nhận xét. Ta phát hiện bất đầng thức (1 ) nhờ dự đoán 1 -2x X 3 + 8x -4 < ax + b. () với V X > 1 và tìm a, b bằng cách lần lượt thay X - 1 , X - 2 vào hai vẽ' của () ta có 1 - 2.1 ! I 3 +8.1-4 1-2 2 2 3 i 8.2-4 = a.1 + b = a.2+b 2a-tb = ^ 20 b 1 £* Nhận xét. Đăng quang trong trận đấu này lã vô sĩ Tạ Nam Khánh, 9E1 , THCS Vĩnh Tường, 4 - 4 . Vĩnh Tưởng, Vĩnh Phúc. Vì bạn Khánh đả có lời giải ngắn gọn và sớm nhất. Bạn Phạm Thành Dũng, 9E1, THCS Vĩnh Tường, Vĩnh Tường, Vĩnh Phúc cũng có lời glảl đúng và được khen. LE ĐỨC 'thuận cuộc THI Tư DUY ĨHUẬĨ Toún uù TínH Tũtìn nusĩnnuí) mờ RỌÍ1G COMPUTATIONAL AND ALGORITHMIC THINKING (CAT) TS. TẠ NGỌC TRÍ (Vụ Giào dục Tiểu học, Bộ Giàũ dục và Đào tạc) hông giống như bất cứ một thòi điểm nào khác trang lịch sử phát triển của thế giới, tư duy thuật toán đang trở nén là một phần của thế giới ngày nay của chúng ta. Tin học nghiên cứu về cấu trúc và sự biển đổi của các hệ thống tự nhiên và nhân tạo mà chúng tạo ra, xử lí, cất giữ và giao tiếp thông tin. Trẽn nhiều lĩnh vực khác nhau từ nghiên cứư y sinh đẩn việc hẹn hò, và sử dụng sức mạnh và khả năng của công nghệ, Tin học chuyển các dữ liệu thành các thòng tín mà con người có các giải pháp xử lí hàng ngày. Tin học là một mỏn học đang phát triển rất nhanh chóng và cố cơ sở là Toán học. Học sinh học Tĩn học là học các thuật toán đơn giản, các cấu trúc dử liệu và các kĩ thuật tính tữản có nển tảng lả thông tin và giao tiếp, và thể hiên quá trình học tập thông qua các nhiệm vụ vể lập trình cho máy tính. Trong chương trình giáo dục phổ thông của Các nước thì Tin học cũng được xảc định là một môn học quan trọng để phát triển cho học sinh nhiều năng lực chung, nhự năng lực tính toán, nàng lực công nghệ thông tin và truyln thông. Cuộc thi tư duy thuật toán và tính toán Australía mở rộng (CAT) là cuộc thi giải toán, kéo dài trang 60 phút. Cuộc thi này nhằm xác định tiềm năng của học sinh về tư duy lập trình, những tiềm nâng mà học sinh thường không có cơ hội để thể hiện. Tuy vậy CAT không phải là cuộc thi lập trình và CAT không đòi hỏi học sinh có kiến thức vể lập trình! Kết quả của bãi thi CAT có thể giúp phát hiện ra các tài năng trong tương lai vể lập trình mà các hoạt động thống thưủng trong học tập không làm được. Một sỗ câu hỏi trong bài thì kiểm tra khả nãng thể hiện của học sinh về tư duy mạch lạc (tư duy câu lệnh); những câu hòi khác đùi hải suy nghĩ lôgic, trong khì đỏ còn cỏ những câu hòi yéu cầu cao hơn đòi hỗỉ phải xác định và úng dụng của tu duy thuạt toán. CAT cỏ 4 bài thi bao gỗm: bài thi cho khối 5-6 (UP), bài thỉ cho khỗị 7-8 (J), bài thì cho khối 9-10 (I), và bài thi cho khối 11-12 (J). Mỗi bài thi gồm sáu câu hỏi trắc nghiệm, tiếp theo đó là chín câu hỏi yêu cáu cao hơn với câu trả lời cố kết quả là một số nguyên. Ví dụ về một câu hỏi trong bài thi CAT: The snĩtĩer dog at the airporì. stũps beside 3 trũiley piled high with 60 suitcases. One of the sưitcases contains contraband peanụts. The dog can tetl vvhether peanuts aie hidden in any one of the group ữf suitcases, but it gets tired if it has to do tao much snìtting. What is the smaitest number oỉgroups otsuitcases it musí ăniff in arder to isũlate the suitcase with the peanuts? • Đề bài dịch sang tiếng việt: Một chú chó đánh hơi ở sân bay đang đứng bên cạnh một xe chó 60 vati được chất cao. Một trong những vali có chửa củ tạc bị cấm nhập cảnh''. Chú chó có thề phát hiên trong một chóng nào đó có vati chứa củ tạc hay không , nhưng chú chó sẽ bị mệt nếu phải làm Việc quà nhiều. Hôi số ít nhất các chóng vali mà chủ chò phải ngủi dể có thể chỉ ra dược vati chứa củ tạc? CAT được tổ chức và quản lý bồi Quỹ ủy thác Toán học Australía (Australian Matheniatics Trust) phối hợp vứi ủy ban Olympic Tin học của Australia (Australian Iníomnatics Olympiad Committee). Quỹ ùy thác Toán học Australia Trust là một tổ chức phi lợi nhuận quốc gia. Mục đích các hoạt động cùa AMT là: • Lằm phong phú cho hoạt động giảng dạy và học tập môn Toán và môn Tin học của học sinh tất cà các cấp học. • Tổ chức các cuộc thi phát hiện tìm kiếm tài nâng Toán học và Tìn học. • Tổ chức các hoạt động, hội thảo vể học tập, giảng dạy cho học sinh vá giáo viên. • Xuất bản sách để quảng bá cho việc học Toán. Tin học cho học sinh Australia vả Quốc tể. ủy ban Olympic Tin học của Australia cũng như AMT được hỗ trợ bởi một mạng lưỡi rộng lớn của các tinh nguyện viên nhiệt tình từ Australia và trên toàn thê' giới, bao gôm các học giả, các thành viên Hội Toán học. Tin học. các nhà giáo dục toán học, tin học và các cựu thành viên cùa các đội tuyển thi Toán hạc, Tin học cùa Australia. Thòng tin thêm về AMT xin truy cặp tại đường dẫn http : tt www.a mt.edu.au/ Thông tin thêm về CAT và danh sách các học sinh trèn thế giới đạt kết quả cao nhất trong cuộc thi CAT 2015 cỏ thể truy cập tại đường dẩn sau http : //www.3 mt . ed u -3 u/ín fọ rm atics/ cat/ w Lí/ật nhập cảnh của Australia cẵm không cho mang vào các toại hạt như tạc, chú thích người dịch. 11 DỀ THŨ CHỌN HỌC SINH GIỎI MÔN TOÁN LỚP 6 HUYỆN HOANG HÓA, TỈNH THANH HÓA Nám học: 2015 - 2016 Thời gian làm bải: 120 phút ị không kể thòi gian giao đê) Bải 1 . (4,5 điểm) Tính giá trị các biểu thức sau
  117. A-| + |:5--^.{-3) 2 . 3 6 18 b} B - 3 <5.[{5 2 + 2 3 ) : 11)- 16} + 201 s.
  118. c = 1 4 - ——— 1.3 Ỵ 1+ J4 1+ J_j Ẳ 2- 4 Ầ 3 - 5 J .. 1 + - 1 2014.2016 Sài 2. (4,0 điểm)
  119. Tìm sổ tự nhiên X biết 8.6 + 288 : (x - 3) 2 = 50. b} Tìm các chữ sô' X. y để A = xi 83y chia chó 2, 5 vã 9 đều dư 1 .
  120. Chửng tỏ rằng nếu p là số nguyên tố lởn han 3 thì p 2 - 1 chia hết cho 3. Bài 3. (4,5 điểm) 5
  121. Cho biểu thức B - — — — (n Ẹ z, n ft 3). n- 3 Tìm tất cả các glá trị nguyên của n để B là số nguyên
  122. Tìm các sô nguyên tố X, y sao cho X 2 +■ 1 17 = y 2 .
  123. Số 2 1W viểt trong hệ thập phân có bao nhiêu chữ số? Sải 4. (5,0 điểm) Cho góc xBy = 55°. Trên các tia Bx, By lẩn lưạt lấy các điểm A và c sao cho A B, c B. Trên đoạn thẳng AC lấy điểm D sao cho ÁBD = 30°.
  124. Tính đọ dài AC, biết AD - 4 cm, CD ■ 3 cm.
  125. Tính sõ đo của DSC.
  126. Từ B vẽ tia Bz sao cho DBz = 90°. Tính số đo ÀBz. Sài 5. (2,0 điểm)
  127. Tìm các chữ số a, b. c khác 0 thỏa mãn abbc = ab X ac X 7.
  128. Cho A = 1 7 -3 |. Chửng minh A là số tự nhiên chia hét cho 5. ĐỀ THO CHỌN HỌC SINH GIỎI MÔN TOÁN LỚP 7 TRƯỜNG TRUNG HỌC THựC HÀNH SÀI GÒN, TP. Hồ CHÍ MINH Nam học: 2015-2016 Thời gian làm bài: 120 phút (không kể thời gian giao đề) Bài 1. (3 điểm) Tính giá trị biểu thức M = 201 6 10 + 2016 11 201 6 10 - 2016 11 ' Bài 2. (2 điểm) Tinh nhanh N = - 1 - - — 1 . 1000 1000.999 999.998 990.997 3.2 2.1 Bài 3. (4 điểm) Tim X, y biết
  129. 5 2ỉí_1 = 5 2ỉt_s + 1 25.24.
  130. X - y = xy = X : y (y 4 0). Bài 4. (2 điểm) L bz-cy cx-az ay-bx J Biẽt rang — = — — = — — . Chứng minh X : y : z = a : b : c. a b c Bài 5. (2 điểm) Cho A = |2x 4 + 3x 2 + 1| - !-2x 4 - X 2 - 1 1 . Chủng to rằng giá trị biểu thức A luôn không âm vãi mọi giá trị của X. Bài 6. (3 điểm) Cho tam giác ABC, kẻ đường cao BD vuông góc với AC (D <= AC). Chứng minh rằng nếu 3BD 2 + 2AD 2 + CD 2 = AB 2 + BC 2 + CA 2 thi AABC cân. Bài 7. (4 điềm) Cho AABC cỏ ba góc đều nhọn. Gọi M, N lẩn lượt là trung điểm của các đoạn thẳng AB, AC. Kẻ AH ± MN tại H và CK vuông góc với đường thầng MN tại K. Chứng minh rằng
  131. AH = CK.
  132. BC= 2MN. LTS. Thật thú vị khi so sánh đề thi bãy giờ với các đề thi trưởc đây để hiểu thêm độ khó, dễ của thì cử xưa và nay. TTT giới thiệu đổ thi học sinh giỏi môn Toán lôp 7 tỉnh Hà Nam Ninh nám 1977 (là lớp cuối cấp 2, tương đương vòi lỏp 9 bày giờ). Bạn sẽ thấy không phầi bảy giờ đề thi khó hơn. DỀ THI CHỌN HỌC SINH GIỎI MÔN TOÁN LỚP 7 TỈNH HÀ NAM NINH Năm họe: 1976-1977 Thời gian làm bài; 24Ữ phút (không kế thời gian giao dề) Gài 1. (3,0 điểm) Cỏ SO viên bi trũng đỏ cỏ một vỉẻn khống đạt tièu chuẩn nên nhẹ hon các viên khác, Chỉ cần cản 4 lấn là có thể lấy ra được vièn bi không đạt tiêu chuẩn. Cân thê' nào? Bài 2. (4,0 điểm) Biểt rằng a, b, c là những sô hữu tỉ khác không, từng đỏi khác nhau và a + b + c = 0, hãy tính Ị-b-£ + -£ + ạ-bỴ^_ + _b £_j [ a b c H b - c c - a a - b J Bài 3. (3,0 điểm) Chứng minh biểu thửc X 12 vrSx 9 + X 4 - X + 1 dương với mọi X. Bài 4. (3,0 điểm) Cho tứ giác lồi ABCD. Hãy tìm quỹ tích điểm 0 trang tứ giác đó sao cho khi nối OB, OD thì tứ giác đã cho bị chia thành 2 phẩn (OBAD và OBCD) có diện tích bằng nhau. Bài 5 ' (7,0 điểm) Cho tam giác ABC cán tại đỉnh A và o là điểm giữa đường cao AH. Đường thẳng BO cắt AC tại E, đường thẳng CO cắt AB tại F. Biết diện tích tứ giác AEOF lã s.
  133. Tính diện tích của tam giác ABC.
  134. Tìm vị trí của điểm E trên đoạn AC-
  135. Đường thẳng song song với BC kẻ qua ỡ cắt AB tạl M và cắt AC tại N. Tính diện tích của hình ENMF. líạngiỊỊãiiẠtì ĐỀ THI TUYỂN SINH LỚP 10 CHUYÊN TP. HÀ NỘI, NĂM HỌC 2016 - 2017 (Để đãng trên TTT 2 sô 162 + 162 ) Bài 1. 1 . Điều kiện X > 1 hoâc X < 0. Biến đổi ta được X 4 - 2x 3 X 2 - X 2 t X - 2(x 2 - X) = 0. (1)

    Đặt t = ^2(x z -x) > 0.

    t 4 t 2 , Từ (1) = t = 0 hoặc l = 2. 4 2 • THI. t Ũ <=> X 0 hoặc X 1 (thỏa màn). • TH2. t = 2 Cĩ i/2(x 2 -x) = 2=*x 2 -x-2 = 0 CT 5 X = -1 hoăc X = 2 (thỏa mân). Vậy phương trình có tập nghiệm là {-1; 0; 1 : 2).
  136. Cộng theo vế của 2 phương trình ta có <=> (2x -y 2 ) 2 + (x -2) 2 =0 I X = 2 ị'x =2 <É=> i í=> 1 2x -y 2 =0 ly- ±2. Thử lại ta được X - y - 2. Vậy nghiệm (x; y) của hệ phương trình là (2; 2). Bái 2.
  137. Đẳng thức đã cho tương đương với

    (a I b I c)[(a - b) 2 ■ (b - c) 2 ■ (c - a) 2 ] = ũ

    >a + b + c- 0 hoặc a - b - c - ũ (loại). Thay vào p được p
  138. Ta có bc 2 -2bc ca = — I{a4b4c)= 0. 2ca 2 v ' 2*. X 2 = 9y 2 + 6y + 16 « 2 ỈĨ .X 2 = (3y + I} 2 + 15. • THI. X là số lẻ =!» X = 2k + 1 =>2 2k<1 x 2 = 2.4 k x 2 . Vi 4 k chia 3 dư 1 và X 2 chia 3 dưũ hoặc 1 nên 2.4 k X 2 chia cho 3 dư 0 hoặc 2- Mà (3y 4 1) 2 4 15 chia cho 3 dư 1- Vậy trường hợp X lè không thỏa mãn. • TH2. Nêu X chẵn => X = 2k =» 2 2k .x 2 - (3y 4 1 Ý = 15 — > (2 k .x - 3y- 1)(2 k .x ■ 3y M) - 15. Vì X, y e N => (2 k .x 4 3y 4 1) > (2 k .x - 3y - 1} khí đó xảy ra 2 trường hựp sau j2 k -X-3y-1 = 1 Ị2 k .x+3y + 1-16 hũặc 2\x-3y -1 = 3 2 k .x43y4l = 5. Từ trên tính được X = 2 vả y = ữ. Bài 3.
  139. Biến đổi vế trái của bất đẳng thức ta có VT = 2(a + b I c) ■ í 2ab 2 2bc 2 a ■ b 2 b + c 2 2ca 2 c I a (1) Mặt khác a 4 b 2 > 2bVã: b 4 c 2 > 2cy'b; c ị a 2 > 2a>/c. Suy ra 2ab 2 2 bc ; 4- a 4 b 2 b 4 c 2 ■cVb 4avc) ^b4ab 0 4bc a 4ca -——4 — — 4 — T-— ( 2 ). V (a 4 b 4 c) 2 < 3(a 2 4 b z + c 2 ) - 9 a 4 b 4 c > ab 4 bc T ca. (3) a 4 b 4 c < 3 Từ (1); (2) và (3) ta có 2a 2 2b 2 2c 2
  140. ■ + -—7 4 — — r- a I b b + c c 4 a a + b I c.
  141. Đặt A - 2 + 2\'í 2n 2 +1 = A -2 -2\ ,: i2n 2 41 A 2 -4A=48n 2 .(1) Từ (1) ta có A 2 : 4 => A : 2 => A = 2k (k G N). Thay vào (1) ta cố k 2 - 2k = 12n 2 . (2) Từ (2) ta có k : 2 — > k - 2m. Thay vào (2) ta có m(m - 1) = 3n 2 . V (m, m - 1 ) - 1 nên m : 3 hoặc (m - 1) : 3. THI. m : 3 -> m - 3a 2 và m - 1 - b 2 (a, b E= N) 3a 2 - b 2 = 1 => b 2 4 1 = 3a 2 : 3 (vỏ lí). THÍ. m - 1 :3=>m = a 2 vàm-1= 3b 2 (a, b £ N) A = 4m = 4a 2 = (2a) 2 là số chính phương. Bải 4.
  142. Gọi L lè điểm đối xứng của A qua o, khi đó BHCL là hình bình hành => p, H, L, M thẳng hàng. Suy ra ẨPH = 90°. Vậy A, p, c 1 . H, B' cùng thuộc đường tròn đường kính AH. Ta có ẤB'P = ẤC’P => PC'B = PẼTC và PBC' = PBÀ = PCẦ = PCB‘. Vậy ABPC’ co ACPB‘. (1) D
  143. a) Chứng minh tứ giác PEKF nội tiếp. Vì PE là phản giác cùa góc BPC’ EB PB FC PC T ương tự ta có 7— — — FB’ PB' ... PB _ PC .... Mà AA- =— — (theo (1)) PC' PB' ,, eb fc Từ đó suy ra — — = (2) EC' FB' 1 Mệt khác ABHC' co ACHB r . (3) Từ (2) và (3) ta có E r H, F thằng hảng và theo (1) BC' HE CB' HF PH là phân giác của EPF. Vậy PH đi qua điểm chính giữa của cung EF khỏng chứa A. Lại có AAEF cân tại A — > Phân giác AO' đi qua điểm chính giữa của cung EF không chứa A. Như vậy K là giao điểm của AO’ và PH thuộc đưởng tròn ngoại tiếp AAEF. Vậy PEKE là tứ giác nội tiếp,
  144. Gọi D lá điểm chính giữa cung BC không chứa A cùa đường tràn ngoại tiếp AABC, Q là điềm đối xứng của D qua BC nên BHQC là tứ giác nội tiếp. Suy ra HBQ = HCQ. Gọi E' là giao điểm của HQ và AB. khi đó BHE' - QCB. Ta lại có QHC = QBC, QHC = E 7 HC\ QBC = QCB => BHÉ 1 ' = C^HE' = HE' là phân giác của BHC' =>E' = E. Vậy Q £ EF. Ta có DQ là phân giác của BDC nẻn q5b = 180 °- ấ = Áef. 2 Từ đó tứ giác BŨŨE nội tiếp. Ta cố bTQ = BQD = 18D °- Ả = 90«-4 2 2 => ÁÊO ■ + BÊD = 90° => o 1 E ± DE. Vậy DE là tiếp tuyến của (O r ). Chứng minh tương tự, DF là tiếp tuyến của (0‘), Vậy nên các tiếp tuyến của {O') tại E và F cắt nhau tại D nằm trên đường tròn ngoại tiểp AABC Bải 5. Không mất tính tổng quát già sử các sổ đã cho đều là các số nguyên dương không cùng chắn. Gọi s là tổng của 201 7 sỏ đã cho. Khi đó xảy ra 2 trưởng hợp sau: • TH 1 . Nếu s là số chắn Ta cỏ các số không cùng lẻ. suy ra có ít nhất một sô chẵn nén tõn tại 2 sd chan lẻ cạnh nhau, vì vậy, khi bò đi 2 sô này thì 2015 số còn lại không thể chia thành hai nhóm có tổng bằng nhau. • TH 2. Nếu s là số lẻ Tốn tại hai sô' cạnh nhau có cùng tính chần lẻ (vi nếu không chúng sẽ chẵn lè luân phiên, khống thể xảy ra điều này vì 2017 là số lẻ), bỏ đi hal sổ này ta thây các số cùn lại không thể chia thành hai nhóm mà tổng các số ở hai nhóm bằng nhau. Vậy từ hai trường hợp trẽn suy ra tồn tại hai số thỏa mãn yêu cẩu. <B IQ trong vườn Anh Cảc từ sau đưọc sắp xếp theo một quy luật nhất định: AUSTRALIA RAISIN CREASE PATOIS VIRTUOSO V - J Bạn hãy chọn 1 từ trong số 4 từ dưới đây để điển vào vị trí tiếp theo của dãy từ trên: MAIN; SCHISM; GRANITOID; GENEROUS, ĐỨC NGUYỄN (st) ô chữ CHEMICAL ELEMENTS {TTT2 số 161+162} Hầu hết các bạn đều điền như sau: Hàng ngang (từ trên xuống): CARBON; SULẼUR; IODINE; ZINC; PHOSPHORUS: LEAD. Cột dọc (Từ trái sang phải): IRON; OXYGEN; ALUMINIUM; NEON. Cũng có không ít bạn chọn SILVER (vào chỗ SULPUR), do đó ALUMINIUM sẽ phải thay bằng AMERICIUM. íỉurTnr cá hai cách úng những bạn sau may mắn được nhận quà: Hạ Hiền Lương, 7A3 : THCS Lâm Thao, Lâm Thao, Phú Thọ; Đỗ Tiến Dủng, 7A1, THCS Vĩnh Yên, Vĩnh Yên, Vĩnh Phúc; Hoàng Như Diện, 8A2, THCS Yên Phong, Yên Phong, Bắc Ninh; Nguyễn Là Bình, 8A1, THCS Giảng Võ, Ba Đình, Hả Nội; Ngõ Hoàng Anh, 6D, THCS Hoa Quảng, Diễn Chảu, Nghệ An. Chủ Vườn Bài Hà Nội có rất nhiều viện bảo tàng (Tiếp theo kì trước) ThS. NGUYỀN vũ LOAN LTS. Nếu biết tiếng Hán bạn sẽ: 1- Hiểu các từ Hán Việt, sử dụng tối hơn tiếng Việt cùa mình. Trong kho từ vựng tiếng Việt rất nhièutừ Hán Việt.
  145. Đọc được sách cồ, vản bia bằng chữ Hán và Hán Nỏm, thêm hiéu văn chương, lịch sử nước Nam mình.
  146. Hiểu ngôn ngữ mà củ 5 người trên thế giới có hơn 1 người dùng. Dễ dàng hợp tác, làm ăn với các nước và vùng linh thả Trung Quốc. Hàng Kỏng, Đài Loan, Singapore và cả Nhật Bản, Hàn Quốc. Nếu biết cà tiếng Anh và tiếng Hán thì thật lá tuyệt. Tập đọc, L i íl í pj [) tĩl $5 ĩf W6 qùguỏ Hcnci bỏwùguăn. Jĩntiãn đc Tuỏyè mciyồu Tuótiãn dc Tuòyẽ nàmc nán, zuótiãn de 7UÒVC nảndé bùdélíão, Xiàngăngde dõngtiãn méiyõu Hénèi nàme leng, Hénèi de đõngtiãntài lcnglc, W6menxuéxiảo yõu yĩgè hẽn dà de túshũ guăn, túshũ guăn lì yõu hẽnduõ shũ, wồ chángcháng qùnàlìkànshEL Bài tập, 1 . Chọn đúng v/ hoặc sai X cho các câu sau:
  147. Síi*iaW*tlỉ4W«ilrỉ»Ằ.
  148. m®ýt, Am.
  149. a

    i

    i£ĩiĩa+.e?.

  150. Đọc vả nồi
  151. 7
  152. fôPtH+á£7 3HaĩÃWHA?
  153. fô£Ị|ỤLỉli ÍÈ ■?
  154. M£B &ỈLM ỈL*1
  155. ím8?ẪJM 8 ) (X) ( ) t ) ( ) ( ) ( )
  156. WẰiẵĩtm tir*.
  157. SẼ.
  158. fÊfòfcìá£3S52óxx
  159. THE EARTH’S MAGNETISM MORIS VŨ A magnetic íield is a region around a magnet in vvhich objects are affected by the magnetic force. The Earth has a magnetíc field vvhich acts as though there were a giant bar magret in its centre, lined up approximately betvveen its geographic north and South poles, although the angle is constantly changing. The north pole of a compass points tovvards a point called magnetic north, its South pole to magnetic South. Physics terms magnetic magnetism magnet magneíĩc fieíd magnetic force region affect từ, từ tính từ học, hiện tượng từ nam châm từ trường từ lực vùng, miền tác động Earth act giant bar geographic north South poỉe permanent magnet magnetịsation Trải đất tác dụng, ảnh hưởng khổng lồ thanh (thuộc) địa lí bác nam cực nam châm vĩnh cửu từ hóa, nam châm hóa Practice. Bạn hãy dịch bài khóa trên nói về từ trường của Trái đất dựa vào các từ vựng đã cho. Tỏa soạn chở bài viết (bài dịch) tốt của bạn. Bài dịch tốt và gủl sâm (theo dấu bưư điện) sẽ được chọn dâng và trao thưởng. SUY NGHĨ VẼ MỘT DÃY BẤT ĐẲNG THỨC CAO MINH QUANG (GV. THPT chuyên Nguyễn Bỉnh Khiêm, Vĩnh Long) Nhận xét. Với hai sổ thực dương X, y ta cớ (x 2 4y 2 ) 3 X 3 + y 3 (x +y)(x 2 +xy +y 2 ) (x + y} 3 2 f X + y xy(x + y) l 2 j 2 m i„k T, /„ , ,.\2 Chửng minh. Từ (x + yỴ > 2xy, suy ra x + y Ỹ xy(x + y) X 2 ) 2 Vi (x - y) 2 £ 0 và (x + y)(x - y) 2 £ 0, suy ra X 3 + y 3 {x + y)(3x 2 -3xy~3y 2 ) 2 6 _ (x H- y)(2(x - y) 2 + {x 2 + xy + y 2 )) 6

    (x » y)(x 2 4 xy I y 2 ) 6 (x + y)(x 2 + xy + y 2 ) (x + y)(4x 2 + 4xy ÝẨy 2 } 6 ” 24 a 3 +b 3 +c 3 „ fa + b + cì 3 ( 2 ) Ldỉ giải. Áp dụng bẩt đẳng thức (1) với số thực dương d ta có a 3 + b 3 ( a + b '; 3 c 3 + d 3 . f ’ c - d ■* 2 r V 2 , 2 { 2 ) 2 Cộng theo vế các bất đẳng thức trên và áp dụng (1) ta có a 3 + b 3 + c 3 + d 3 2 1 ( a + b
  160. c+ứ )\ 2 2 V 2 jj Hyĩ frj — lỉ - ^-(a + b + c + d) 3 Với d - , từ bất đẳng thức trên ta có 3 ,3 3 ( a + b + c a +b +c 3 + — 7 3 = (x + y)(3(x + y) 2 + {x - y) 2 ) > fx-yf 6 { 2 J Áp dụng bất đẳng thức AM-GM ta có ,,a . „3 (x + y) 4 (4x 2 -4xy +4y 2 )

    4<x + y) 3 c , [<x + y) 2 + (x +y) 2 +(4x z -4xy +4y 2 )! 3 27 4(x +y) 3

    2{x 2 + y 2 } 3 (x + y) 3 Kết hợp các bất đẳng thức trên ta đirợc dãy bất đẳng thức (1 ). Các đẳng thức ả (1) xảy ra khi X = y. Sau đây chúng ta sê xét một số bài toán áp dụng dãy bất đẳng thửc {1 ). Sài toán 1. Cho a, b, c là các số thực dương. Chững minh rằng ( a + b + c [ a + b - c ' ■ a + b-c + — — 1 = 2 — — — 3 í \ / L 3 , a 3 +b 3 +c 3 { a+b + cỸ 3 I, 3 Đẳng thức xảy ra khi a - b - c. Bài toán 2. Cho a, b, c là các sô thực dương. Chứng minh rằng 8abc . „ a 3 + b 3 + c 3 3abc {a + b)(b + c)(c + a) Lởi giải. Áp dụng bât đẩng thức AM-GM và (2} ta có a 3 + b 3 + c 3 > |' a + b + c ì 3 Ị (a + b) +{b +c} +(c +a) 7 Z 7^ỉ T 6 J i.( at b)(b-cK a 3 4-b 3 +c 3 8abc => — — +- — — —— £ 2. 3abc (a +b)(b +c)(c +a) Đẳng thức xảy ra khi a = b = c. Bài toán 3. Cho a, b, c > 0. Chứng mình rằng b ị c c + a ã + 3 '4(b 3 +c 3 } b + \/4(c 3 Ta 3 )
  161. _ỈỊb <2. t bh Lòi giải. Theo bất đẳng thức (1) la có $4{b 3 + c 3 } > b + c. Do đó a + 3|l 4(b 3 + c 3 } > a + b + c b \ c b + c ^a + ^4(b 3 +c 3 ) Satbtc ' T ưong tự ta có c + a b + ậ'4(c 3 +a 3 ) a + b c-r a a ■ b \ c’ a + b c + ^4(a 3 +b 3 ) a + b + c Cộng theo vế các bất đẳng thức trên, ta có điểu phải chứng minh. Đẳng thức xảy ra khi a b c. Bài toán 4. Cho a r b, c > ũ. Chứng minh rằng 1 1.11 a 3 +b 3 +abc b 3 +c 3 + abc c 3 +a 3 +abc a ^c Lời giải. Theo bất đẳng thức (1}, ta có a 3 + b 3 ă ab(a + b). Suy ra a 3 + b 3 + abc ằab{a + b + c). 1 1 Do đố — £ < — — — -. a 3 + b 3 + abc ab(a + b + c) Tương tự ta có 1 , 1 — <- — - — b 3 +c 3 + abc bc(a + b + c) 1 „ 1 — 2 £ — . c 3 4- a 3 + abc ca(a + b-c) Cộng theo vế của các bất đẳng thức trên ta có điểu phãi chứng minh. Đẳng thức xảy ra khi a = b - c. Bài toán 5. Cho X, y, z > 0 và xyz = 1 . Chửng minh rằng 0 9 X + y 9 _9 y + z x 6 + x 3 y a +y B y e +y 3 z 3 +z 6 z 9 + X 9 z 6 +Z 3 X 3 + X 6 2 . Lời giải. Đặt a = X 3 , b = y 3 , c = z 3 thì abc = 1 . Bất đẳng thức trở thành a 3 + b 3 b 3 +c 3 c 3 + a 3 — - + — - + — > 2. n ộ M J n n ■ — a +ab + b b +bc + c c +ca + a Theo bất đẳng thửc (1), ta có a 3 +b 3 a + b — — — — — > — - — . a * 2 -t ab \ b 2 3 Tương tự ta cỏ b 3 + e 3 b + c . c 3 + a 3 c + a b 2 + bc + e 2 3 c 2 + ca + a 2 3 Cộng theo vế các bất đẳng thức trẽn ta được a 3 +b 3 b 3 +c 3 c 3 +a 3 a 2 + ab+b 2 b 2 + bc + c 2 c 2 + ca4-a 2 a + b . b + c c + a 2 , . , — — 4- — — +— — = --(a4b+c) 3 3 3 3 |-3^ãbc=2. 3 Suy ra đpcm. Đẳng thức xảy ra khi a = b = c hay X = y = z = 1 . Bài toán 6 . Cho a, b, c r d > 0 thỏa mãn 1111 — + — + -7 - 4. Chứhg minh rằng a b c d V 2 + v 2 + v 2 1 2 < 2(a+b + c + d) -4. Lài giài. Theo bất đẳng thửc (1) ta có a 3 + b 3 a 2 + b 2 V 2 a + b Dù dó Ja 3 +ĨÕ 3 Jb 3 + c 3 Jc 3 + d 3 Jd 3 + a 3 \'^r- + T2 + f 2 ■ ! ỉ a 2 + b 2 b 2 +c 2 c 2 ~d 2 d 2 + a 2 a + b b + c c + d d + a Ta sẽ chứng minh a 2 +b 2 b 2 +c 2 c 2 +d 2 d 2 + a 2 aib b + c c I d d I a <2(a + b + c + d)-4.(3) Ta lại có a + b- a 2 +b 2 2 ab 2 a + b a + b 1 |A' a b (x + y- y-rZ + z + t + ỉ + x) ^ 1 1 1 1 ' ■ + — — + -
  162. + 7 + 7 x+y y+z z + t t+x 4 . J Do đó (4) đúng. Suy ra đpcm. Dang thức xảy ra khi X = y = z = t = 1 hay a = b 55 c = d= 1 Bải tập vện dụng Bài 1. (Thái Nhật Phượng) Chữ X, y, z > 0 thỏa mãn xyz = 1 . Chửng minh rằng
  163. x + y + y 3 z y + z I Z 3 X Z + X + X“y Bài 2 . (Nguyễn Bá Nam) Cho a, b, 0 0. Chửng minh rằng b+c c+aa+b — - — + — - — + „ 1 1 1 1 Do đó nêu đặt X = — ; y = z = — ; t =--- và chú ý a h c d 4a z 4b 2 rằng X + y + z + 1 = 4 bât đẳng thúte (3) viết lạt thành “ a . b ! b I c a b c Bải 3. Cho a, b, c> 0. Chứng minh rằng a 3 b 3 c 3 a 2 b 2 c 2 b 2 + ”2 + q 2 - b c a ■ Bải 4. Cho a. b, c > 0 . Chứng minh rằng yỊÃs ịa 3 +4b 3 + \''4b 3 + 4c 3 + \Uc 3 +4a 3 4e z c + a 1 1 1 1 — — + ——>2. (4) x+y y+z z+t t+x Theo bất đảng thức Bunhiacốpxki ta có (TTT2 sỏ 161+162) THẾ CỜ (Kì 83) 1 ,®xg6+ hxg6 2.Sh8# Các bạn được thưâng kì này: Nguyễn ±L:L Đăng Duy, 8A1. THCS Yên Phóng. Yên Phong. Bắc Ninh; Nguyễn Thanh Tùng. 8 D, THCS vĩnh Tưởng, Vinh Tường, Vĩnh Phúc; Trần Lăm, 6 D, THCS Lê Hồng Phong, Đức Thọ, Hà Tĩnh; Nguyễn Vàn Hạo, 9/1, THCS Nguyễn Du, Xã Điện Phương, Điện Bàn. Quảng Nam; Phan An Khánh, 9A2, Trường THCS Giảng Võ, Hà Nôi. LÊ THANH Tứ THẾ CỜ (Kì 85) Trắng di trước chiếu hết sau 2 nước. á b c d e f g h LẾ THANH TÚ (Đại kiện tướng Quốc tế) MỘT SỐ BẤT ĐẲNG THỨC TRONG TAM GIÁC VÀ TỨ GIÁC TS. NGUYỄN VẢN NGỌC (GV. Đại học Thăng Long) Có nhiều bài toán bất đẳng thức hình học mà khi giẳi chỉ cân sử dụng kiến thức về bất đẳng thức các cạnh trong tam giác. Sau đây chủng ta sẽ xét một số bài toán hình học ỏ lởp 1 và lớp 8 như thế. • Bất đẳng thức vể các cạnh trong tam giác: Trong một tam giác cố độ dài ba cạnh là a, b, c thì Ịb - c| < a < b + c. Bài toán 1 . Gọi M là điểm nằm bẽn trong tam giác ABC. Chứng minh rằng MB + MC < AB +ÃC. Lài giải. A Gọi N là giao điểm cùa BM và AC. Áp dụng bất đảng thức tam giác ta có BM + MC < BM + MN + NC = BN + NIC < AB + AN
  164. NC = AB + AC.

    Bãi toán 2. Gọi Q là điểm nằm trong tam giác ABC có nửa chu vi là p. Chứng minh rằng p < OA + OB + oc < 2p. Lài giải. A Theo bất đẳng thức tam giác ta có AB < OA + OB; BC < OB + OC; CA í oc + OA. Cộng theo vế các bất đẳng thức trẻn ta cỏ AB + BC 4 - CA < 2{OA + OB + 00). (1) Áp dụng bài toán 1 ta cố OB + oc < AB + AC; oc + 0A< BC + BA; OA + OB<CẠ + CB. Cộng theo vế của các bất đẳng thức trên và nít gọn ta cỏ OA \ OB + oc < AB 1 BC + CA. (2) Từ (1) và (2) suy ra đpcm. Bàì toán 3. Cho hai tam giác ABC và A'B’C' có AB - A'B’ và AC - A'C'. Khi đó BẦC < B7VC’ <=> BC < BX'. Lài giải. 0 B 8’ • Nếu BAC<B’A'C' ta phải chứng minh BC < B’C\ Trên nửa mặt phẳng bờ AC chứa điểm B lấy điểm D sao cho DÃC =B'Ả'C' vàAD = AB = A'B'. Dể dàng chứng minh aADC = AA'B'C' (c.g.c). Suy ra CD = B'C- Đường phân giác của DAB cắt cạnh CD tại H. Ta chứng mỉnh được ADAH = ABAH (c.g.c). Suy ra DH = BH. Từ đó BC < BH + HC = DH + HC = CD = B’C. 23

    Nếu BC < B'C' ta phải chứng minh

    BAC < B r b'Q\

    Nếu BẰC = B’ A’C’ thì AABC = AA'B’C' nên BC

    = B’C' (mâu thuẫn với BC < B'C).

    Nếu BẰC > b”Ầ'C' thì theo chứng minh trên ta

    có BC > B‘C' (nnảu thuẫn với BC < B'C’) Vậy ẺẢC < ẽTẦ ' c Bài toán được chứng minh. Bài toán 4. Cho tứ giác lồi ABCD. Chứng minh rằng AB + BC 4 CD 4 DA < 2(AC + BD). Lài giẳì. Q Gọi o là giao điểm của AC và BD. Theo bất đằng thức tam giấc ta cỏ AB < OA + OB: BC < OB + OC; CD < oc - OD; DA < OD 4 OA. Cộng theo vế các bất đẳng thức trên ta được AB 4 BC + CD 4 DA < 2(OA 4 OB 4 oc 4 QD> = 2(AC + BD). Bãi toán 5. Cho tứ giác lổi ABCD. Gọi M, N là căc điểm nằm trong tứ giác đó sao cho DMNC là tứ giác lồi. Chứng minh rằng DM 4 MN 4 NC < DA 4- AB + BC Lài giải. • THI . Đường thẳng MN cắt cạnh AD tại p và cắt cạnh BC tại Q. Theo bẩt đẳng thức tam giác ta có DM + MN +- NC < (DP + PM) 4 MN + {NQ + QC) = DP + {PM + MN 4 NQ} 4 QC = DP 4 PQ 4 QC < DP + {PB + BQ) 4 QC = DP 4 PB 4 BQ 4 QC < DP 4 PA 4 AB 4 BC — DA "ỉ" AB 4 BC. B • TH 2. Đường thẳng MN cắt cạnh AD tại p vả cắt Theo bất đẳng thức tam giác ta có DM + MN 4 NC < {DP + PM) + MN 4 (NO + QC) = DP 4 (PM 4 MN 4 NQ) 4 Qc = DP 4 PQ 4 QC <DP4(PA4AQ)4ŨB4BC = (DP 4 PA) 4 (AQ 4 QB) 4 BC = DA 4 AB 4 BC.

    TH 3. Đường thẳng MN cát cạnh AB tại p và cắt cạnh BC tại Q chúng minh tưđng tựTH2 Đài toản 6. Cho điểm M nằm trong tử giác ABC D có nửa chu vi là p.
  165. Chứng minh rầng MD 4 MC < DA 4 AB 4 BC. 3
  166. Chứng minh rằng p < MA 4 MB 4 MC 4 MD < ^p; c} Tim vị trí của M để MA -í- MB - MC 4 MD nhò
  167. Bạn đọc tự chứng minh tương tự các bài trẽn.
  168. Theo bất đẳng thức tam giác ta có AB < MA + MB; BC < MB ị MC; CD < MC + MD; DA < MD + MA. Cộng theo vế các bất đẳng thức trên và nút gọn ta được P<MA+ MB 4 MC + MD. (1) Áp dụng phẩn 3 ta có MD + MC<DA + AB + BC; MC + MB <CD T DA + AB; MB+ MA< BC + CD + DA; MA - MD < AB + BC + CD. Cộng theo vế các bất đẳng thức trên và rút gọn ta được 2(MA + MB + MC 4 - MD) < 3p. {2} Từ (1) và (2) suy ra đpcm,
  169. Gọi o là giao điểm của AC và BD. Ta có

    MA ■ MC k AC; MB 4 MD k BD. Cộng theo vê cảc bất đẫng thức trẻn ta được MA - MC - MB T MD £ AC - BD. Dấu đẳng thức xảy ra khí M ■ o. Vậy MA ị- MC + MB ■ MD nhỏ nhất khi M ■ 0. Bài toán 7. Cho đường tròn (O) bán kính 1 và 2016 điểm A.p A 2 , A 3 , A 2ũl(j . Chứng minh rằng tồn tại một điểm M thuộc đưàng tràn (O) sao cho Gọi M 1 và M ? là hai điểm trên đưdng tròn đổi xúng vởi nhau qua tâm o. Ta có A 1 M 1 1 A,M 2 ^M 1 M 2 = 2. A 2 M 1 tA 2 M 2 ỉ M 1 M 2 = 2. AgM,

    2016^1 + 2016^2 1:^2 = Cộng theo vế của 201 6 bất đẳng thức trên ta có (M 1 A 1 + M^A 2 +■ M 1 A a + ... + + (M 2 A + M^Aj + + ... + 2^2016 2.201 6. Do đó tồn tại điểm M trùng với M 1 hoác M 2 thỏa màn MA 1 ■ MAj + MA 3 + ... MA 2016 ỈÍ 2016. Nhặn xét. Bài toán trên có thể thay số 2016 bàng một sổ' nguyên dương tán hơn 1 bất kì. Bải tập vận dụng Bài 1. Cho tam giác ABC. Gọi M r N là hai điểm nẳm trong tam giác ABC sao cho BMNC là tứ giác lồi. Chứng minh rằng BM + MN + NC <AB+ AC. Bài 2. Cho tam giác ABC. Gọi N, p, Q thứ tự là trung điểm cùa AB, BC, CA. M là điểm nằm trong tam giác NPQ. Chứng minh rằng MA I MB i MC <ị{AB I BC I CA). Bài 3. Cho hình thang ABCD có AB ì! CD và AB < CD. Gọi p, H, Q thứ tự là trung điểm của DA, AB, BC. Gọi M, N là các điểm nằm trên PQ sao cho PM = MN = NQ. Chứng minh rằng DM + MN + NC < DH + HC. Bài 4. Cho hình bình hành ABCD có Â<90°. Chứng minh rằng BD <AC. Bài 5. Cho tam giác ABC có AB = c, BC = a, CA= b. Độ dài các đường trung tuyến ứng vớì các cạnh a, b, c lần lượt là m a . m b , m . Chứng minh rằng 2(m a - m b - m c ) < 3{a - b + c) < 4(m a - m b + m c ). Bài 6. Cho tam giác ABC có AB - c, BC - 3, CA - b- Goi r là bán kính đường tròn nội tiếp tam giác. Chứng minh rằng a + b-c 2r 1 1 1 A sin — 2 B sin — 2 : c

    sin — 2 Bài 7. Cho tam giác ABC cố diện tích s. Gọi M là điểm nằm trong tam giác. Chứng minh rằng 4S í AM.BC + BM.CA í CM.AB. Bài 13NS. Tìm các số nguyên X sao cho 3072 chia hết cho (x+ 2015)2 4 |x + 201 6|. TẠ THẬP (TR Hổ Chí Minh) Bài 14NS. Tìm các số thực không âm X, y, z, t thỏa mãn Vx +jyz = 2 + t;x 2 + y 2 4Z 2 + t 2 =3. NGUYỄN KHÁNH TOÀN (GV THCS Bắc Hải, Tiền Hải, Thái Bình) Bài 15NS. Cho tam giác ABC có các đường trung tuyến BE, CF cắt nhau tại G, vâi BF 4 FG - CE 4 EG, Chứng minh rằng ABC lã tam giác cân. NGUYÊN KHÁNH NGUYÊN (Số 3/29, đường Đà Năng, Hổi Phòng) (TTT2 sỏ 161+162) Bài 1ÚNS. Ta có a 3 - b 3 - c 3 = 3abc =>a 3 = b 3 + c 3 4 3abc -> a > b và a > c (vị 9 , b, c e N*) 2a > b ị c=>4a>2(b t c). Mà a 2 = 2(b + G). (1) =>■ 4a > a 2 => 0 < a < 4 (vì a € N*}, (2) Từ (1) suy ra a : 2 (vì b, c e N*). (3) Từ (2) và (3) suy ra a - 2. Thay a = 2 vào (1) ta đưọc b + c = 2 kết hợp vâi b, c e N\ ta có b = c = 1 . Vậy a = 2; b = 1 ; c = 1 . Nhận xét. Các bạn sau cỏ lời giải đúng: Nguyễn Thủy Dương, Vũ Linh Chi, Nguyễn Thu Hiền, Bủi Thị Quỳnh, 9A3, Phạm Thị Minh Ngọc, 8D, THCS Lâm Thao, Lâm Thao, Phú Thọ; Mai Thanh Tàm, SA, THCS Lý Tự Trọng. Bình Xuyên; Khổng Thị Thu Thủy, 9B, THCS Vĩnh Tường, Vĩnh Tưởng! Vĩnh Phúc; Nguyễn Thị Quỳnh Anh, 9A1, THCS Thị trấn Quán Hành, Nghi Lộc; Lè Ánh Minh, SD. THCS Đặng Thai Mai, TP. Vinh, Nghệ An; Nguyễn An Na, Bùĩ Thị Minh Thư, Nguyền Hẳĩ Ly, 8A, THCS Hoàng Xuân Hãn, Đức Thọ, Hà Tĩnh. Bải 11NS. Ta có a 3 4 b 3 + 3ab = 1 <=> (a t b) 3 - 3ab{a 4 b) + 3ab -1=0 » [(a + b) 3 - 1 3 ] - [3ab(a + b) - 3ab] = 0 o (a -L b - 1 )[(a + b) 2 T (a + b) 1 1 - 3ab] = 0 "a I- ta — 1 = 0

    => ,,

    (a + b) 2 + (a ị b) +1-3ab = 0 a 4 b = 1 (1) _a 2 + b 2 4 1 - ab 4 a 4 b = 0 (2) Ta có (2) 2a 2 4 2b 2 + 2 - 2ab 4 2a 4 2b = 0 (a - b) 2 + (a 4 I) 2 4 (b 4 I) 2 0 <=> a b !-1. Suy ra a + b = -2. (3) Từ (1) và (3) ta có a + b = 1 hoặc a + b = -2, Nhận xét. Bài toán trên xuất phét từ hầng đẳng thức 2(x 3 4 y 3 4 z 3 - 3xyz) = (x 4 y 4 z)[{x - y} 2 4 (X - 2)2 4 (y - 2)2]. Các bạn sau có lời giải đúng; Nguyễn Thùy Dương, Vũ Linh Chi, Nguyễn Thu Hiền, Bùi Thị Quỳnh, Bùi Thúy Linh, 9A3, Phạm Thị Minh Ngọc, 8D, THCS Lâm Thao, Lâm Thao! Phú Thọ; Khổng Thị Thu Thủy, 9B, Hoàng Thị Ngọc Diệp, 8E1 , THCS Vĩnh Tưởng, Vĩnh Tường! Vĩnh Phuc; Nguyễn Thị Quỳnh Anh, 9A1, THCS Thị trấn Quán Hành, Nghi Lộc, Nghệ An; Nguyễn An Na, Bùi Thị Mình Thư, Nguyễn Hải Ly, 8A r THCS Hoàng Xuân Hãn, Đức Thọ, Hà Tĩnh. Bài 12NS. Bạn đọc tự vẽ hinh.

    THI. E nằm giữa A và D

    Vi tứ giác ABCD nội tiếp nên ÁCK - BDÈ (cùng chắn cung AB). Vi tứ giác AKBE nội tiếp nên ẤKC = BED (cùng bù vỡi AEB ). Xét AACK và ABDE cỏ ÃCK = BDỀ; ÂKC =BẼD. Suy ra AACK 00 ABDE (g.g) kết hợp với ABDE cân tại D (vi DE = DB), suy ra aACK cân tại c.

    TH2- Trưởng hợp A nầm giữa E và D.

    Chứng minh tương tự.

    TH3. E =A. Chứng minh tương tự. Nhận xét. Các bạn cò lài giải đúng; Nguyễn Thủy Dương, 9A3. THCS Lãm Thao, Lâm Thao, Trằn Yến Linh, 9A4, THCS Giây Phong Châu, Phù Ninh. Phú Thọ! Khắng Thị Thu Thủy, 9B, THCS vĩnh Tường. Vĩnh Tường. Vĩnh Phúc; Nguyễn Thị Quỳnh Anh, 9A1 , THCS Thị trấn Quán Hành, Nghi Lộc, Nghệ An. Các bạn được thưởng kì này: K St Nguyễn' Thủy Dương, 9A3, THCS Lâm Thao, Làm Thao, Phú Thọ; Khổng Thị Thu Thủy, 9B, THCS vĩnh Tường, Vĩnh Tưởng, Vĩnh Phuc; Nguyễn Thị Quỳnh Anh, 9A1, THCS Thị trấn Quán Hành, Nghĩ Lộc, Nghệ An; Nguyên An Na, Bùi Thị Minh Thư, 8A, THCS Hoàng Xuân Hãn, Đức Thọ! Hà Tĩnh. Ảnh các bạn dược thưởng ồ bia 4. NGUYỄN HIỆP ĐE ĐE XUAT cuọc THI CÂU LẠC BỘ TOÁN TUổl THƠ TOÀN QUỐC
  170. 6 digits 0. 1,2, 3, 4, 5 are N$ed to tornn 6-digit numbers which have distihctdigits and are divisible by 15 and not divisibla by 10. How many auch numbers can be íormed?
  171. Calculate 1234 2 + 625 2 + 359(859 - 1250)
  172. 1234(1250- 1718).
  173. Fìnd ab such that 98ab:36,
  174. Solvethe following equation. (2x- I) 3 -6(2x- I) 2 + 12(2x- 1) = 8.
  175. Factorize the following polynomial. x 2 {x 2 + 2x+ 1) - 2x 2 - 2x 4 1.
  176. Solve the following equation. (X 4 2) 4 (2x 4 2 2 ) 4 (3x 4 2 3 ) 4 ... 4 (9x 4 2 9 ) = 10x 4 2 . 1 13
  177. Given that x + — X 6 Evaluate the folỈQwing expression. (x-1) 2 {x 2 4X41) 1? '
  178. Given a right-angle triangle ABC with the right angle at A, its median AM, and ĩts internal angle bisector AD. Find the area of triangle ADM given that AB = 21 cm and AC = 28 cm.
  179. Solve the following inequation. 4 1 Given the quadrilateral ABCD having ZBAD = = 90°, AD = 1 2 cm, and BC = 15 cm. Let M and N be the midpoints of BD and AC respectively. Find the length of MN,
  180. Find a natural number n such that 1 1 1 1 11 1.2.3 * 2.3.4 + 3.4.5 rt(rt4 1)(n 4 2) _ 45 '
  181. Find X and y such that 2x 2 i y 2 - 2xy - 2y + 2 = 0.
  182. Gi ven a triangle ABC with a fixed sỉde BC and a moving vertex A. Let M and N be the midpoints of AB and AC, respectively. Let D and E, be the orthogonal projection of the points M and N onto BC. respectively. On which line does the point A move SLich that DMNE is a square?
  183. Calculate 2017 X 20152015 - 2015 X 20162016,
  184. A natural nurriber gives remainders of 1 and 3 when divided by 4 and 5 respectively. What is the remainder when that number is divided by 40? TNP CÂU LẠC BỘ TOÁN TUổl THƠ NGUYỄN KHÁNH CHUNG (GV, trường Lỏ-mô-nô-xõp, Q. Nam Từ Liêm, Hà Nội) CL326. Determine the value of m such that the following inequation 3mx > X + 2 is Satisíied for all value of X > 1 . CLB27. Given integers X, y such that 3x - 2y + 1 =0. Find X, ỵsuch that the value of p - |x| + |y| is smallest. CLB28. Let ABC be an equilateral triangle with AB = 4. Let M, N, p be points lie on sides AB. BC, CD respeetively and satisty the condition AM = BN = CP = X (0 < X < 4). Find the value of X such that triangle MNP’s area holds the smallest value. CLB29. Determine value of X satisíying the following equation: (x 2 + I) 4 - (2x 2 - 2x + I) 4 = X 2 - 2x. CLĐ3Q. Given triangle ABC and D is a point ũn side BC. Draw parallelogram AEDF where E and F are on AB, AC respectively. Calculate the square of parallelogram AEDF know that triangle BDE ! S area is 3 and triangle CDF’s area is 12. GÀU ục Bộ TÕAKTĨÍÚiTHQ /H® CÂU LẠC BỘ TOÁN TUẩl THƠ (TTT2SÍ 161.162) CLB16. Ta có A = X 2 + 4y 2 - 4xy + 3x - 6y - 4 = (x - 2y) 2 + 3(x - 2y) - 4 = (X - 2y) 2 - (X - 2y) + 4(x - 2y) - 4 = (X - 2y)(x - 2y - 1) + 4{x - 2y - 1} " ( K - 2y - 1)(x - 2y + 4). CL817. Ta có B= n 4 -n 3 t 3n 2 - 2n +2
  185. n 4 - n 3 + n 2 + 2n 2 - 2n + 2
  186. n 2 (n 2 - h + 1) + 2(n z - n + 1) = (n 2 - n + 1}( n 2 + 2). Vì n 2 4 2 > 2 nên để B lè số nguyên tô' thì n 2 - n + 1 1 =í n 1 (vì n là sổ nguyên dương), Vãi n = 1 thì B = 3 là số nguyên tổ. Vậy n = 1 , CLB1B. Ta cỏ X 2 -2016x-2 = 0<> x-2016-- = 0 X í=> X-— = 2016. X Do dó „ X 4 + x 3 +x 2 -2x-4 „2 .... 2 4 c - 7 - -X + X+1--T— - { 0 0 = X-- + x-- + 5 = 2016 2 - 2016 +5 =4066277. \ X J X CLB19. Đật n 2 - 4n - 3 = k 2 (k G N) « (n - 2) 2 - k 2 = 1 <=> (n + k - 2)(n - k - 2) = 1. ■f n+k _2=1 ịn-k-2=1 fn+k-2 = -1 ịn-k-2 -í -1 n = 3 k = 0 : Vậy số nguyên n phải tìm là n = 1 hoác n = 3. CLB20. Kẻ ME // BN, E thuộc AC, Ta cỏ ME là đưửng trung bình của ABCN, suy ra BN =2ME. Xét AAMEcó IN iỉ ME, suy ra IN AI 1 ME AM - 4 ... BI 7 Vậy — 7 = -r. BN 3 IN 2 BN ~ 8' Nhận xét. Các bạn được thưởng ki này: Nguyễn Thùy Dương í. Nguyễn Thu Hiển, 9A3, Nguyễn Đức Tản, 8A3, THCS Lâm Thao, Lâm Thao, Phú Thọ; Phùng Hoài Thương, 9A1, THCS Nghi Hương, TX. cửa Lò; Trần The Trung, 9A, THCS Đặng Thai Mai, TP. Vinh, Nghệ An. NGUYỄN HIỆP Oữtnsa o „ - ■ - *L J MỘT SÔ KĨ THUẬT TÍNH GIÁ TRỊ BIÊU THỨC NGUYỄN KHÁNH CHUNG T rong càc đề thi học sinh giỏi cấp tỉnh, thành phổ hoặc các để thi vào THPT chuyên, ta thường gặp bài toán tinh giá trị biểu thức với già trị của ẩn đã cho trưởc. Sự thủ vị ỗ càc bài toàn này nằm ồ chỗ biểu thức cần tinh cô lũy thừa bậc cao, phức tạp; giá trị cho trước của ẩn thường chứa căn và rất khò khăn khi thay trực tiếp. Kĩ thuật thưởng sử dụng đối với các bài toán này thay vì thay số trực tiếp, ta biến đổi tìm các biều thức trung gian của ẩn rổi thể vào biểu thức cẩn tính giã trị cũng dược biến dổi rút gọn. Ví dụ 1 . Tính giá trị biểu thức A - X 5 + 2x 3 - X + 1 vdĩ X = Vv' 2—1, Lời giẳì. Ta Cú X = Vvfĩ => X 2 = v'2 - 1 => (x z + 1) z =2. Dù đó X 4 + 2x 2 - 1 = 0. Biến đổi biểu thức Ata được A = x{x 4 + 2x 2 -1) + 1= 0 + 1 = 1. 4(x + 1)x 2x 2 + 3x x 20ì2 {4x 2 + 4x - 2) + 2x + 1 2x + 1 <2x 2 + 2x) + x = 2 - X + 1 2x + 2 Thay 2x = V3 - 1 vào biểu thức biểu thức A đã rút Ví du 2. Cho X = 2 Vã - 2 2<Vã + 1) Tính giá trị của biểu thức A 4(x + 1)x 2013 -2x 2ũ12 -h2x+1 2x 2 +3x (Đề thi HSG tỉnh Thái Bình năm 2013) Lài giải. m Biến đổi X ta có gọn có A = 2-- = 3 -Vã. X - 1 Í 2 V 3-2 2(Vã+i) Ỉ2V3 +2-6V3 + 6 |' (V3 - 1 ) 2 _ V3 -1 8 V 4 =>2x = V3-1<=>2x + 1 = V3. <=> 4x z t 4x - 2 = 0 <=> 2x z + 2x - 1 = 0. • Rút gọn biểu thửc A V 3+1 Ví dụ 3. Chứng minh rằng X - V2V7 -1 + 3|I 2V7 + 1 là một nghiệm của phương trình x e - 18X 4 + 81X 2 - 112 = 0. Nhận xét. Đây là dạng toán chứng minh một sở đã xác định nhưng ỗ dạng phửc tạp là nghiêm của phương trình cho trước. Lỏi giải. Trong bài toán này, ta biên đổi giẻ thiết để dẫn tỏi phương trình dã cho. Nâng lên lũy thừa bậc ba của X và áp dụng hằng dẳng thức (a + b) 3 = a 3 + b 3 + 3ab(a + b) f ta có X 3 = 2V7 - 1 + 2V7 + 1 + 3V2V7 -I.V2 V 7 + 1. $ 2 V 7 - 1 + 2 V 7 + 1 ). =>■ X 3 =4V7 + 9x Lời giải. Ta thây a -I- b 8, ab 11. thay vào biểu thức p, ta có p = [a 2013 - (a + b)a 2012 + ab.a 2011 ]
  187. [b 2tl13 - (a + b)b 2012 + ab.b 2011 ] -0 + 0 = 0. Bải tập vận dụng Bải 1. Cho X = v'2 +1. Tính giả trị của biểu thức A = (x 4 - 4x 3 + 4x 2 - 2) 5 + (x 3 - 2x 2 - X -1 ) e . Bài 2. Cho f(x) = (2x 3 - 21 X - 29) 2012 . Tính giá trị của f(x) khi X = ị[ĩ + + 3|7 - (Đề thi HSG lớp 9 TR Hà Nội năm 2012) 2 Bài 3. Cho X = - . v'72-1-1 v'72+Ĩ + 1 Tính giá trị của biểu thức A=(x 4 -x 3 -x 2 + 2x- I) 2012 Bài 4. Cho X, y thoà mãn x = -ựy 2 + 1+^y + Vy 2 + 1. =>x 3 -9x = w?. => (X 3 -9x) 2 =112 =>x e -18x 4 -31x 2 -112 =0. Điều này chửng tỏ X là nghiệm của phương trình X 6 - 18X 4 + 81X 2 - 112 - 0. Sau dãy sẽ lê 2 ví dụ rút gọn hoặc tính giã trị biểu thức cẳn sử dụng hệ tlúíù liên hệ của 2 ẩn. Ví dụ 4. Rút gọn biểu thức A = <V-VSÕ - Vx+V5ÕWx + \/x 2 -50 , với X > -J50. Lài giải. Đặt a = Vx - \'50; b = 'Ịx. + V50 , suy ra a 2 t b 2 = 2x, ab = ýx' - 50. Thay vào A ta có A t ,, Ịa 2 1 - b 2 A -(a-b).i 1 — + ab -(3-bJ (a,t i bĩĩ - a2 7 b2 =4° =5. ( } í 2 4 ề Ví dụ 5. Cho biểu thửc p = {a 2013 - 8a 2012 r lia 2011
  188. (b 20t3 - 8b 2012 I 11 b 2011 ). Tính giá trị của p biết a = 4 + >/5; b = 4 - n/s. Tính giả trị của biểu thức A = X 4 + x 3 y + 3x 2 + xy - 2y 2 + 1 . Bài 5. Chứng mình rằng a = + V2 + y3 + y6 + V3 + V2 — — 'Jq là một nghiệm cùa phương trinh X 5 - 4x 4 + 3x 3 - 14x + 8 = 0. (Đề thi HSG lớp 9 tình Thái Bình năm 2014) CAC TINH CHAT VA ĐOI ĐŨEU THÚ VỊ VỀ TAM GIÁC VUÔNG NGUYỄN ĐỨC HẢO (GV. THC$ Lam Sơn, Q. 6, TP, Hồ Chí Minh) T am giác vuông có các tính chất đặc biệt nào? Để trả lời câu hỗi này, học sinh THCS cần phải hệ thống và tóm tắt lại kiến thức hình học và khi cần sẽ vận dụng trong việc giẻi các bài toán hình học về tam giác vuông vá một số bài toán liên quan, Trong cuốn sách giáo khoa THCS không có bài học riêng biệt vé điều này Sau đẫy là bèi viết tổng hợp về các tinh chất đó ớể bạn đọc dễ dàng ôn tập và vận dụng.
  189. Các tính chất cơ bản của tam giác vuông 1.1. AABC vuông tại A B + C = 90°. 1 .2. aABC vuông tại A thi cạnh huyển BC lân nhất. 1.3. Định lí Pythagoras (thuận và đào) AABQ vuỏng tại A BC 2 = AB 2 + AC 2 . 1 .4. AABC vuông tại A vối đường cao AH thì 3 tam giác ABC. HBAvà HAC đồng dạng với nhau. 1 .5. Các hệ thức vể cạnh và đường cao trong tam giác vuông ABC có đường cao AH: AB 2 = BH.BC; AC 2 = CH.CB: AH 2 = HB.HC; AH.BC - AB.AC; + AH Z AB 2 AC 2 1 .Ê. a) Tam giác vuông ABC cỏ trung tuyến AM thì AM = MB = MC = . 2 BC
  190. Nếu AABC có trung tuyên AM = —

    thì ABC là tam giác vuông, 1.7. # Nếu tam giác vuông ABC nội tiếp đường tròn thì cạnh huyền là đường kính của đường tròn đó. • Tam giác nội tiếp được đường tròn có cạnh là đường kính của đưàng trùn ngoại tiếp thì tam giác này vuông với cạnh huyển là đường kính. 1.8. Các dạng đặc biệt cùa tam giác vuông

    Tam giác ABC là tam giác vuông cân tại A khi

    và chỉ khi thỏa mãn một trong ba điểu kiện sau;

    Ằ = 90° và AB = AC. AB = AC; BC = ABv'^2. Â = 90°;B = 45° hoặc c =45°. AABC lả nửa tam giác đểu khi và chỉ khi thỏa

    mãn một trong các điều kiện sau:

    Â = 90°; B = 60°{hoậc c = 60°). AC = AB%/3 =

    2

    Â 90°; BC 2AB hay AC ABv5. 1.9. Diện tích tam giác ABC vuông tại A có đường caoAH là 0 AB.AC AH.BC ■Q — ■ — “ . 2 2
  191. Đôi diều thú vị vế tạm giác vuông Từ nhữhg kiến thức cơ bản trên suy ra các tính chất thú vị dưới đây. 2.1. Tính chất 1. Cho AABC vuông tại A, đường caoAH. A • Nếu AP là đưòng phân giác của BÃH (P e BH), thì ACAP cân tại c và phân giác của ACB vuông góc vỏi AP tại trung điểm K cũa AP. • Nếu AQ là đường phân giác cùa CAH (Qf HC) thì ABAQ cân tại B, và phân giác cùa ABC vuông góc với AQ tại trung điểm J của AQ. (Hai tính chất này bạn đọc tự chứiig minh). 2.2. Tính chất 2 Hai tia phân giác của B và c cắt nhau tại I thì khi đó • I là tâm của đưởng tròn nội tiếp tam giác vuồng ABC. • I là trực tâm AADE (với D và E là tâm của 2 đường tròn nội tiếp AAHB và AAHC). • I là tâm đưởrig tròn ngoại tiếp AAPQ. • AABC đổng dạng với AHDE. Từ AHDÂ ưì iHEC và AHBA co AHAC suy ra HD HA AB HE HC AC 2.3. Tính chất 3 Nếu AABC có đường cao AH và đường trung tuyến AM chia góc BAC thành 3 gốc bằng nhau thì AABC là nửa tam giác đều. Chứng minh. Ta dễ dàng chứng minh được AABM cán tại A. Mặt khác tam giác cân ABM có AH là đường phân . " uo .... MB MC giác nên suy ra HB = HM = —— = Dựng MK 1 AC tại K, Ta chứng minh được AAMK - AAMH =íMK =MH^MK =Ẹ-. 2 Như vậy tam giác vuông MKC có c = 30° 7* HẦC = 60° = BÂC =90°. Vậy AABC là nửa tam giác đểu. 2.4. Tính chất 4. Cho AABC (AB < AC) có đường cao AH r đưàng phản giác AD của BAC, trung tuyến AM. Biết rằng AD nằm giữa AH vá AM. Chứng minh rằng AD là phản giác của HÃM khí và chỉ khi BẰC - 90°. Chứng minh. • Nếu AABC có BẢC = 90°, ta chứhg minh được BẦH - ÁCH - MAC, do đó HẦD = MAD Vậy AD là phân giác của HÃM. • Nếu AD là phản giác của HÃM, ta cần chứng minh BẦC = 90°. Gọi N là trung điểm của AB. Vì MN II AC nên ÁMN MAC (so le trong). (1) Vì HN là trung tuyến cùa tam giác vuông AHB nên HN = NA = ANHA cân tại N. Suy ra ẤHN = BAH. (2) A Vì AD là phân giác của HAM và BAC nèn BAH - MAC (3) TÙ (1), (2), (3) suy ra ẤMN - ÃHN . Gọi I là giao điểm của AH và MN, ta chứng minh được AlMA 00 AIHN => AINA co AIHM. = INÀ = 90°. Vì MN // CA nên BÃC - 90°. 2.5. Tính chát 5. Cho AABC có đường phân giác AD. Chứng minh rằng BÃC = 90° + = AB AC AD Chứng minh. Ta kẻ DE li AC; DF II AB với E c AB và F c AC, từ đó tứgiácAEDF là hình thoi, suy ra DE= DF =AE = AF. Do DE // AC và DF // AB nén DE BD DF _ CD Ãc BC va ÃB CB

    DE DF BD+DC BC 1 ^AC + AB" BC BC 1 1 1 AB AC DE

    Giả sừcó BẤC = 90° thì AEDF là hình vuông. =>AD = DE>/2=-l- + -l 7 = |. AB AC AD • Từ -ệvà () ta có AD = DE1/2 AB AC AD = AD 2 = AE 2 +ED 2 - Từ đó AAED vuông tại E (định lí Pythagoras đảo). Suy ra hình thoi AEDF là hỉnh vuông. Vậy BAC = 90°. 2.6. Tính chất 6. Cho AABC có đường cao AH, trung tuyển BM và phán giác CP đổng quy tại một điểm.
  192. Chứng minh BẤC - 90° <=> HB - AC. b} Tính AB và AC theo BC, vởi BC = a và Â = 90 ữ . Chứng minh. Ạ
  193. Theo định lí Ceva ta có PA HB MC 1 PA HC

    PB HC MÃ PB _ HB (do MC = MA). CA HC Ị PA CA 'ị

    CB HB Vl PB CB Nếu HB = AC • Nếu BAC - 90° và AH 1 BC. Ta cỏ aCAB co ACHA = CB CA Kết hợp với {) suy ra HB = AC. CA CH CB - ÕÃ Lại có góc c chung nên AABC co AHAC =J. BAC = 90°. Vậy ta có điều phải chúTig minh.
  194. Gọi a, b, c là độ dài của BC. AC, AB tương ứng. Do à = 90° nên HB = b (theo cầu trên). Ta có b 2 4 c 2 - a 2 và c 2 - ab ^ b 2 4 ab - a 2 . ■ b 2 4 ab-a' (. af = 0 I b-Ị-— I = 5a 2 ữịỵỉs - 1) ... . Vs-1 AC = -Air — L và AB = a 4 2.7. Tính chất 7. Cho AABC vuông tại A (AB < AC), các đưởng phán giác AD và CE. Chủhg minh CE - 2AD AABC là nửa tam giác đều Chủng minh. Đặt BC = a, AC - b, AB - c. Áp đụng tính chất 5 ta có 1 1 Vĩ b c>/2 = A-T- => AO = 7 ■ b c AD b-c A Theo tính chất đường phân giác của tam giác ta có AE BE AE + BE AB c AC BC AC + BC AC + BC a + b' CA bc à + b Áp dụng định lí Pythagoras vào tam giác vuông AEC ta có EC 2 =EA 2 + AC 2 b 2 c 2 | b 2 _b 2 (2a 2 + 2ab) 2ab 2 (a I b) 2 (a I b) 2 a + b Ta cố CE = 2AD o CE 2 = 4AD 2 2ab 2 BbV

    i— — = — a + b (b + c) 2 « a(b + c) 2 - 4c 2 {a + b) = 0 â(â 2 4 2bc) -4c 2 (a 4 b) = 0 c-> (a - 2c)(a 2 4 2ac 4 2bc) = 0 » a = 2c

    BC = 2AB. () Kết hợp () với giả thiết BẤC =90° ta suy ra được CE = 2AD « AABC là nửa tam giác đểu. Vậy ta có điều phải chứng minh. Bài tập vận dụng Đài 1. Cho aABC vuông tại A, đường cao AH (HB < HC). Cho biết HC - HB =AB, Tỉnh góc B và góc c. Đài 2. Chứng minh rằng nếu AABC có c = 2Â và AC = 2BC thì AABC là tam giác vuông. Bài 3. Cho AABC vuông tại A, phân giác AD. Đường thẳng vuông góc với BC tại D cắt AC tại M. Chứng minh rèng DM DB. 2 AUSTRALIAN MATHEMATICS COMPETITION AMC 2014 SENIOR DIVISION AUSTRALIAN SCHOOL YEARS 9 AND 10 Time allowed: 75 minutes Đỗ TRUNG KIÊN (Sưu tầm vá giời thiệu) Questions 1 to 10, 3 marks each
  195. The expression that has the same meaning as 9x -3 is (C) 1 9x 3
  196. The value of — is 0.04 (A) 15 (B) 20 (C) 25 (D) 40 (E) 60
  197. If p - 9 and q - -3 then ụ 2 - q 2 is equal to (A) 64 (B) 72 (C) 84 (D) 90 (E) 96
  198. A circle has circumíerence n units. In square units, its area is (A) £ (B) I (C) n (D) 2 n (E) 4 n
  199. If K - and La 4 and K = 7, then R èquals (A) -18 (B) 1 (C) 12 (D) 8 (E) 2 6 . If X, X 2 and X 3 lie on a number line in the order shotvn below, which of the fùllowing could be the value of X? < > X 3 X 2 x (A) -2 (B) -1 (C) i (D) 1 (E) I
  200. A 2 metre broom is leaning against a wall, with the bottom of the brooni niaking an ang le of 45° with the ground, The broom slewly slides dovun thẹ wall until the bottom ef the brOQm makes an engle ũf 30° wíth the greund. Hew far, ìn metres, has the top of the broom slid down the wall? (A)72-1 (B)2-y/3 tC) Vã -1 (D) -73-/2 (E)2-V2
  201. The base of a triangle is increased by 25% and its height is increased by 50%. Its area has increased by (A) 25% (B) 50% (C) 66.6% (D) 75% (E) 87.5%
  202. On a section of the nurmber line five intervals are marked as shown. A c E <— ► -■ ♦ « ♦ — > 0 1 2 3 4 5 6 ' V A V ; B D lf a number X íalls in the interval A and a number y falls in the interval D r then the number 1 A(x + ỵ + 1) must falf ín which ìnterval? (A) A (D)D
  203. If (B)B (H)E £C)C — í— = 3 then — equals p - 2q q (A) -3 (B) 3 (C) 1 (D) I (E) 2 Questions 11 to 20, 4 marks each 11 . In a car park there are 3 Fords, 4 Holdens and 2 Hondas. If a parking inspeetor chooses 2 cars at random, the probability that bũth are Holdens is ... 1 4 1 (A) 4 (B) 44 (C) 4 4 27 6 < D > ĩ < E > s
  204. In this tigure, p and Q are the centres of two circles. Each circle has an area of 10 m 2 . The area, in square metres, of the rectangle is ii Tc (D, ẹ <E> f TE 71
  205. The value of >/1 + 2 + 3 1-4 í ... + 99 -hi 00 lies betvựeen (A) 14 and 15 (B) 25 and 26 (C) 30 and 31 {D) 71 and 72 (£} 100 and 101
  206. if - — 4 " - — - and a b, what ỈE the value of X?

    X - b X - a (A) a -b

    2 (B) (E) a 2 +b 2 a + b a + b (C) a 2 + b 2 2(a + b)
  207. In the diâgram, PS = 5. PQ = 3, aPQS ia right-angled at Q. ZQSR = 30° and QR - RS. The length of RS is (A) <B)JI <C)2
  208. ìệ. (E)4
  209. BÌNy. a seasonal vvorker in the tũwn of Covưra, colleeted an even number ũf buckets ùf eherrìes en his fírst day. Eaeh day aíter that he increased the number of buekets he pieked by 2 buekets per day. In the íirst 50 days he collected 3250 buckets. The number of buckets Billy collected on the 50th day was (A) 66 (B) 110 (C) 114 (D) 116 (E) 120
  210. Aíarmer walks 1 kmeastacross his paddock. then 1 km north-eastand then another 1 km east. Find the distance, in kìlometres, betvveen the farmer’s initial position and his final position. (A) 45 + ĩ 'lĩ (B) 4ĨÕ (C) 45 (D) a + b 2 (Kì sau đãng tiếp) Giải toán qua thiỉ Bài 1(161+162). Cho — + — 2 3 . Ị {có 2015 số hạng). Chứng mĩnh rằng Lời giải. Ta cử , 1 ( 1 1 ) . (ì 111) 2 [3 4} [5 6 7 8 J Mi 1 1 1.17 18 2015 2016 1 .71 1 w 1 . 1 1.1) 2 ', 4 4) { 8 8 6 8 J /1,1, 11) (w 16 16 16 Ị .MI. . 1 1
  211. ...+ — — + —— + ...+ + 17 18 2015 2016 1 , M 1 1 „ 21 4 4-2, 4 4-4. -7 4-8. -77 = 2 2 4 8 16 11 Nhận xét. Cố nhiều bạn tham gia giải bài, có bạn còn giải bằng hai cách. Các bạn sau có lởi giải tốt: Nguyễn Công Duy Khánh, 7A6, THCS cẩu Giấy r Cẩu Giấy, Hà Nội; Nguyễn Tuấn Dương, 6A5, THCS Chu Văn An, Ngô Quyền, Hải Phòng; Phạm Minh Đãng, 7A, Nguyễn Hữu Thắng, Nguyễn Minh Tiến. 7B, THCS Lỷ Tự Trọng, Bình Xuyên; Đõ Trị Dũng. 6A1, THCS và THPT Hai Bà Trưng, TX. Phúc Yèn, Vĩnh Phúc; Đào Văn Chiến, Kiểu Mạnh Tiến, Dào Thanh Xuân, 7A3, THCS Lâm Thao, Lârri Thao; Phùng Đàng Dương, 6C, THCS Vãn Lang, TP, Việt Trì, Phủ Thọ: Lé Xuân Hoàng, 7A, THCS Đặng Thai Mai, TP. Vinh, Nghệ An. PHỪNG KIM DUNG Bài 2(161+162). Cho tam giác ABC với ■TA 68 • 10" Trên cạnh AC lẩy điểm D sao cho AD = AB. Chứng minh rằng BD = AG. Lòi giải. Từ giả thiết 3A = 6B = 10C và Â 4-Ẻ4-C = 180°. Suy ra + ^4-0 = 180° => 6 = 30° 3 3 Kẻ AH 1 BC, AI 1 BD (H e BC, I e BD). Do AAHC vu&ng tại H có ẢCH - 30° nên AH = .(1) 2 w Mặt khác AABD cân tại A nên I là trung điểm của SŨ, suy ra 81 = (2) Mà BÃI = ịẽẦC = 50° => BÃI = ẤBH. 2 Do đó AAIB - ABHA (cạnh huyền - góc nhọn). Suy ra AH = BI, (3) Từ (1), (2) và (3) suy ra BD - AC. Nhặn xét- Các bạn sau có lởi giải tốt: Đồ Phúc Xuện, THCS Văn Lang. TP. Việt Trì; Kiểu Mạnh Tiến, Vũ Minh Khẳi, 7A3, THCS Lâm Thao, Lâm Thao, Phú Thọ; Lê Nguyễn Gia Huy, Trần Đức Tùng, Phạm Anh Tài, Cao Thị Thùy Dung, Nguyễn Đảng Doanh, Nguyễn Trung Kiên, Nguyễn Thị Ngọc Trâm, 7B, Trần Anh Tuấn. Bùi Hổng Quân, 7C. THCS Hoàng Xuàn Hãn, Đức Thọ; Phan Thị Hải Yến, 6C, THCS Xuân Diệu, Can Lộc, Hà Tĩnh. Hỏ QUANG VINH Bãi 3(161+162). Cho p(x) là đa thức bậc 4 với hệ số bậc cao nhất bẳng 1. Biết rầng p(20l 3) = 2014, P{2Q14) - 201 5 và P(201 5) = 201 6. Tinh P{2012) + P(2016). Lài giải. Xét đa thức Q(x) = p(x) - X - 1 . Theo giả thiết ta cỏ Q(2013) = P{201 3) - 2014-0 Q(2014)- P{2014)- 2015 = 0 Q(2015) = P{2015) -2016 = 0 Như vậy Q(x) là đa thức bậc 4 vối hệ sổ bậc cao nhất bằng 1 vả có các nghiệm là 2013; 2014 và 201 5 nên Q(x) - (X - 2013)(x - 2014){x - 201 5)(x - a), (a e R). Do đố p(x) = (x - 201 3}(x - 2014){x - 2015)(x - a) + X + 1. Tháy X bằng 2012 và 2016. ta được P(20I2) - -10059 + 6a; P(2016) = 14113 -6a. Suy ra P(2012} + P(2016) = (-10059 + 6a) + (14113 -èa) = 4054. Nhận xét- Đây là bài toán hay vè không quá khó. Hẩu hết các bạn gửi bài giải làm theo cách trên. Các bạn sau đây có bài giải tốt: Nguyễn Ván Thanh Sơn , 9/1 , THCS Nguyễn Khuyến. Đà Nắng; Nguyễn Thị Khánh Ly, 3A4, Dương Tiến Đạt, Phạtĩì Thị Kiều Trang, 9A2, THCS Yên Lạc, Yèn Lạc: Tạ Nam Khành, 9E1 , Phùng Quốc Lăm, 3E1, THCS vĩnh Tường, vĩnh Tương, Vĩnh Phúc; Nguyễn Hưng Phát, 7B, Trần Sỹ Hoàng, 9C, THCS Hoàng Xuân Hãn, Đức Thọ, Hà Tĩnh; Nguyễn Văn Cường, 9A, THCS Hợp Tiến, Nam Sách, Hải Dương: Trần Anh Tuấn, 7C, THCS Hoàng Xuân Hãn, Đức Thọ, Hả Tĩnh; Nguyễn Hữu Nghĩa, SD, THCS Lâm Thao; Nguyện Hà Phương, 8D, THCS Cao Mại, Lâm Thao, Phủ Thọ. NGUYỄN ANH DŨNG Bài 4(161+162). Cho cáo sô thực a, b, c thỏa mãn a 2 + b 2 + c 2 = 2, Tim giá trị lủn nhất, giá trị nhỏ nhất của biểu thức M = a y b I c - abc. Lởi giải. Ta có 2 u2 (a -b) 2 > 0 => ab < a l k < 2 a 2 + b 2 + e 3 2 Suy ra ab - 1 < 0. Do đó M 2 - Ị(a + b).1 + c.(1 - ab)f <[(a +b) 2 + c 2 ][1 2 + (1- ab) 2 ] = (2ab + 2)(aV - 2ab + 2). Suy ra M 2 = 2a 3 b s - 2a 2 b 2 + 4 = 2a 2 b 2 (ab 4 < 4 (vỉ a 2 b 2 > 0, ab r 1 < 0). „ (M > -2 (1) Ị M < 2 (2) Dấu bằng ở (1) xảy ra chẳng hạn khi (a, b, c) = (-1, -1, 0). Dấu bẳng ở (2) xảy ra chẳng hạn khi (a, b, c) = (1 , 1 . 0). Vậy MinM - -2 chẳng hạn khi
  212. 1 ) + (a, b, c) = (-1 -1, 0). MaxM = 2 chẳng hạn khi (a, b, c) = (1, 1, 0). Nhận xét. Dáy là bài toán hay và khỏ, một số bạn có lởi giải hay. Các bạn sau đây có lời giải tốt: Tạ Nam Khánh , Lẽ Ngọc Hoa, 9E1, THCS Vĩnh Tường, Vĩnh Tường, Vĩnh Phúc; Nguyên Hữu Trung Kiên, Bùi Thủy Linh Vù Lình Chi, 9A3, THCS Lâm Thao, Lâm Thao, Phú Thọ; Tnrơng Thị Thu Lan, 8A2, THCS Yên Phong, Yên Phong, Bắc Ninh; Nguyễn Vãn Thanh sờn, 9/1, THCS Nguyễn Khuyến, Đả Nắng. CAO VĂN DŨNG Bái 5(161+1 62). Đặt D ?0 - {1; 2; 5; 7; 10; 14; 35; 70} (tập hợp các ưởc của 70). Ta vẽ biểu đõ cho D 7ũ như sau. 70 10 14 ỊX 2 5 7 1 Tim quy luật của biểu đố trên. Hãy xác định các phần tử cùa D 21C rồi vẽ biểu đồ cho D 21ũ . Lời giải. Phân tích so 210 ra thừa số nguyền tố, ta có 210 - 2.3. 5.7. sổ lượng các ưóc dương của số 210 là (1 + 1)(1 + 1}{1 + 1)(1 + 1) = 16. Tập hợp các ước của 210 là D 21Ũ = (1; 2: 3; 5: 6; 7: 10; 14; 15; 21; 30; 35; 42; 70; 105; 210}. Để vẽ biểu đồ, ta sắp xếp các hàng kể từ dưới lên như sau: Hàng dưới cùng là 1; hàng thứ hai từ dưãi lên lả 2, 3. 5, 7 (một ước số nguyên tố của 210); hàng thứ ba từ dưỏi lèn lả 6, 10. 14, 15, 21, 35 (tích hai ước sổ nguyên tố); hàng thứ tư là 30, 42. 70, 105 (tích ba ưâc số nguyên tố) hàng trẽn cùng là 210. 2 ; 0 6 1 Nhận xét. Đây là bài toán hay và không quả khó. Có ba bạn sắp xếp các ước dương chưa hợp lí nèn vẽ biểu đổ sai. Các bạn sau đây có lài giải tốt: Nguyễn Văn Thanh Sơn, 9/1, THCS Nguyễn Khuyến. Đà Nắng; Từ Tấn Dũng, 8A, THPT chuyên Hà Nội - Amsterdam, cầy Giây, Hà Nội; Tạ Nam Khánh , 9E1, THCS vĩnh Tường, vĩnh Tường; Hoàng Doãn Hà Trang, 8A4, THCS Yên Lạc, Yên Lạc, Vĩnh Phủc. TRỊNH HOÀI DƯƠNG Sài 6(161+162). Cho tam giác ABC vôi phân giác AD. Gọi p, Q là hai điểm thuộc đoạn thẳng AD sao cho ‘,BF C3Q Gọi É, F thứ tự là hình chiếu vuông góc của p lên AC, AB, gọi H là hình chiếu vuông góc của Q lẻn ESC và K là hình chiếu vuông góc của H lêti EF. Chửng minh rằng KH lả phân giác củ á BKC. Ldi giải. Không mất tính tổng quát, giả sử AP < AQ. BỔ để. Cho tam giác ABC với phân giác AD. Gọi p, Q là hai điểm thuộc AD sao cho AP < AQ và ÁBP-Cẽa Khi đỏ ẨCP=BCQ Chứng minh, (hỉnh 1) Gọi M là điểm đối xứng của Q qua BC; N, R theo thứ tự là điểm đối xứng của p qua AB, AC. Ta có APQN = APQR. (1) Ta chứng minh được PBM -PBQ + QBMI= PBQ + 2CBQ = PBO + 2ẤBP = PBQ + PBN = QBN; BP = BN; BM = BQ. Do đỏ ABPM = ABNQ (2) Từ (1) và (2) suy ra PM = QN = QR. Từ đó, chú ý rẳng CM = CQ, CP = CR. suy ra ACMP = ACQR. Suy ra ẨCP ml RCP = ị (RCQ - PCQ = 1 (pCM-PC"q)= jMCQ = BCQ, Trở lại giải bài toán (hình 2) Gọi Y, z theo thử tự là hình chiếu của B, c trẻn EF. Ta cỏ AF = AE. Do đó Ỹ?B = ẨFE=ẤÈF= z Èc. Suy ra ABFY co ACEZ = {1} CE cz Dề dàng chửng minh PF = PE và ABHQ co ABFP, ACHQ co ACER BF PF PE CE BF BH /m Do đó - 7 - = Ạ-ị- = ỊJz. = -Nịl =$ ZỊ- = -111. (2) BH QH QH CH CE CH Từ (1 ) vả (2), chú ý rằng BY II cz // H K theo định llT , , , , 8 Y BH KY cz CH KZ Do đó ABYK co ACZK. Vậy BKY CKZ. Kết hợp với KH : YZ, suy ra KH là phân giác của góc BKC. 7 B H D c Hình 2 Nhận xét. Các bạn sau cá lời giải tốt: Tạ Nam Khanh, 9E1 . THCS vĩnh Tường, vĩnh Tưởng, Vĩnh Phúc; Nguyễn Văn Thanh Sơn , 9/1, THCS Nguyễn Khuyến, Đà Nẵng. NGUYỀN MINH HÀ Nguyễn Cõng Duy Khánh, 7A6, THCS du Giấy; Từ Tấn Dũng, SA, THPT chuyên Hà Nội Amstendam, Cầu Giấy, Hà Nội; Nguyễn Tưấn Dương , 6A5, THCS Chu Vàn An, Ngô Quyền, Hải Phòng; Tạ Nam Khánh, 9E1, THCS Vinh Tuông, Vĩnh Tưởng, Vĩnh Phúc; Kiều Mạnh Tiến, 7A3, THCS Lâm Thao. Lâm Thao, Phú Thi giải toán qua thư Thọ: Lê Xuãn Hoàng, 7A, THCS Đặng Thai Mai, TP. Vinh, Nghệ An; Trần Anh Tuẩn, 7C. THCS Hoàng Xuản Hãn, Đứt Thọ, Hả Tĩnh; Nguyễn Văn Thanh Sơn, 9/1, THCS Nguyễn Khuyên, Đà Nắng; Nguyễn Vãn Cường , 9A, THCS Hợp Tiến, Nam Sách. Hải Dương; Trương Thị Thu Lan, ÔA2, THCS Yên Phong, Yên Phong, Bắc Ninh, Dự THI HỌC BỔNG SINGAPORE THỦY vũ (Xem từ sổ ĩ 58 ra 4.2016) Học sinh vừa hết lớp 9 hoặc đã vào lớp 10 của Việt Nam đều được thi học bổng ASEAN hoậc A*S1ar của Singapore. Học bổng ASEAN thi tuyển tất cả các tỉnh thành và do Bộ Giáo dục và Đào tạo (Việt Nam) đứng ra phát form cho các địa phương. Mỗi tỉnh thành có 2 mẫu. Học bổng A*Star thì do các trưởng à Singapore sang tuyển trực tiếp tại Việt Nam qua một kì thi chung cho mỗi đợt dành cho một số trưởng, Các trường ỏ Singapore là các trường hàng đầu như chuyên NTU, chuyên Nưs (hai trường đại học lớn nhất, danh tiếng của Singapore), trường Ratỉles (trướng THPT nổi nhất như Hà Nội - Amsterdarn của nước ta, nhưng đây là trường tư) r trường NJC, Hoa Chông, Nanyang, Vichtoria, Temasek, ACS, ... Tại Hà Nội, một số trường nổi tiếng thường có đông học sinh giành được học bông này: THCS Trưng Vưong, Hà Nội - Amsterdam, chuyẻn ĐH Quốc Gia, chuyên ĐH Sư phạm, Chu Vãn An, Ngô ST Liên, Giảng Võ, Nguyễn Tất Thành, Lômônôxôp, Nguyễn Trường Tộ, THPT Kim Liên, Việt Đức, Trần Phú ... ơ các địa phương khác cũng là các trưởng truyền thông như THCS Trãn Đàng Ninh, THPT Lê Hổng Phong (Nam Định), Trán Phú (Hải Phòng), Phan Bội Châu (Nghệ An), Trần Đại Nghĩa (TP. Hỗ Chí Minh) ... Học bổng ASEAN và A^star đều cò ho sơ giống nhau: 1 đơn (form) có mẫu của nuúc bạn cung cấp, dược điển đầy đủ thông tin, hợp cách; bảng điểm từ lớp 6 đến lớp 9 (hoặc lớp 10), bản sao giấy khai sinh, bằng tốt nghiệp THCS (nếu là lởp 10). Các giấy tờ đều cỏ bản tiếng Anh, có công chứng hoặc xác nhận của hiệu trưởng. Thời điểm làm hổ sơ là tháng 3 và 4 nêu đi học THPT còn đi đại học thường làm tháng 11, Ngoài ra học bơng A*star còn rải rác làm vào Kì thi viết diễn ra vào cuối tháng 5. Học sinh phải làm bài thi toán bẳng tiếng Anh từ 30 đến 35 câu trong 120 phút theo hình thức tự luận, không sử dụng máy tính. Để thi gổm tất cả các vấn đề nước bạn chú ý dạy mà chương trình chúng ta ít chú ý: tài chính trong gia đình, thể tích các hình khối phức tạp, thống kê, xác suất, đồ thị chẩm, dồ thị cảnh - lá ... Bài phỏng vấn được tiên hành sau 5 tuần nêu kết quả bài thỉ toán khoảng được 80%, bài IQ được khoảng gần 90% và bài Anh vãn được khoảng 65%. Đây là học bổng ASEAN. Còn học bổng A*star thì phỏng vấn chì sau một vài ngày, Đây là cuộc nói chuyện 15 phút về đé tài bàn thân, gia đình và nhà trướng của học sinh. 39 (Còn tiếp) TraDg tbo’ Trang tbử Trang tbo’ # Trang tbo’ L J NGUYỄN TRỌNG DỎNG (GV, THCS Phũ Lộc, Krông Năng, Đấk Lắk) Bạỉpbấi) Ai chăng đi về mái trường nhữ nhỏ Phấn trắng bảng đen lời giáng của thầy Bao lứa học trò tưng tăng sách vở Mỗi buổi tới trường /à một niềm L!tií Kí ức cuộc đời trong ta uđn thế Như niềm tin đẩu sông cạn dá mòn Ta lớn bởi trang sách và bè bạn Bởi tóc thầy sợi tráng cứ nhiều thêm! Bụi phấn rơi rơi tóc. thầy hóa nắng Ngờ hạt sương mai lấp lánh đất người Đây tình nghĩa mái trường hồng ngọn gió Cho em hay biển học vốn vô bờ Què hương cho ta dáng vóc CUỘC đời Bạo nghĩa íám người trả vay , vay trả Với máí trường nguyện lăm con én nhỏ Khi mùa thu về bụi phấn bay bay... BÍNH NAM HÀ MYANMAR Myanmar gần Myanmar xa Đây là Miến Điện Nghe lâu rồi mà. Đất Phật nhiều chùa Rững nhiêu đá quý . Con người hiền hòa Nói CL/Ời rủ rí Đàn ông diện váy Lưng giẩt Jphone. Bỏm bẻm nhai trầu Dép ỉ.ẽ xỏ ngón. Có gi vội dãu. Cuộc sống chỉ cầu Ăn ngày vài bữa. Còn đâu củng chùa Dát vàng ngọn tháp. Xe dò cũ íã Thếm xe đạp lai. Nhà tầng thd dây Kéo hàng môi buổi. Nhà ngoại thành đấy Lúp xăp tre pheo Săn vườn không thấy Cồ mọc giăng đầy. Cây thốt nốt xa, Con bò gầy trắng GỢi miền xa váng Chiều Campuchia . Xa rồi vần thấy Yangon miền xanh. 22 25.7.2016 ISMim có KHẮNG ĐỊNH Được KHÔNG? Sài toản. Một số cung của một đường tròn được tô bởi hai màu xanh và đỏ, tổng số đo độ dài của các cung được tô đỏ bằng -ị độ dài đường tròn và tổng số đo độ 4 dải cùa các cung được tỏ xanh bằng j độ dài đường tròn. Bạn Toán nói với bạn 5 Tho: "Trên đường tròn luòn tìm được 4 điểm A, B, c, D sao cho tứ giác ABCD là hình chữ nhật và các điểm A, B, c, D không bị tỏ màu’ 1 . Bạn có biết tại sao bạn Toán lại khẳng định được như vậy không? TẠ THẬP (TP. Hổ Chí Minh) rai SỐ ĐIỂM DƯỢC TÔ MÀU (TTT2 số 161+162} I Gọí T là sô' các trung điểm được tò đỏ của các đoạn thẳng tạo bối 2016 điểm phân biệt A v A 2 , A A ”3’ 1 rt 2Dl6‘
  213. Xét giả trị lớn nhất của T. Nối mồi điểm với 201 5 điểm còn lại ta được 2015 đoạn thẳng. Vỉ có 2016 điểm nên sau khi nối như thế cố 2016.2015 lẩn nối và mỗi đoạn thẳng được nối 2 lần, do đó số đoạn thẳng được nối là 1008.2015 = 2031120. Nếu các trung điểm đểu khác nhau thì có tất cả 2031120 trung điểm và giá trị lớn nhất của T bằng 2031120. Điều này có được khi thực hiện thuật toán vẽ hình như sau. Đẩu tiên lầy 3 điểm không thầng hàng A 1f A 2> A 3 và tô màu các trung điểm của chúng, do A r A 2 , A a là 3 đỉnh một tam giác nên 3 trung điểm của chúng phân biệt nhau. Vẽ một đường thẳng d 1 qua A, mà không đi qua A 2 , A 3 và khàng đi qua các trung điểm đã có. Vẽ đường thẳng d 2 qua A^ mà không đi qua A r A 3 và không đi qua các trung điểm đà có, gọi giao điểm của d 1 và d 2 là A 4 . Nối giao điểm mói A 4 vái cốc điểm đã có và tô màu các trung điểm của chúng, nẾu có 2 trung điểm nào {cũ và mởi) trùng nhau thì vi lại d 2 sao cho đạt điều kiện là có điểm A 4 mà tất cả các trung điểm đểu khác nhau. Cứ tiếp tục làm như thế để có các điểm Ag. Aẹ A 201 0. do sổ các điểm và trung điểm là hữu hạn nên luôn vẽ được các điểm với điều kiện trên. Vậy giá trị lớn nhất của T bằng
  214. Xét giá trị nhỏ nhất của T. So sánh độ dài cùa tất cả 2031120 đoạn thẳng của 2016 điểm khác nhau tùy ý thì tổn tại độ dài lớn nhất là r với hai đẩu mút là A 1 , A 2 (nếu khác thì đồi lại). Hai đưàng tròn tâm A 1 và A 2 với cùng bán kính r cát nhau tại ... , 2016 nèn tất cả các điểm đa vẽ đều hầm trong hình tròn tém A t (kể cả đường biên). Tương tự, do A 2 A k < A 2 A 1 với k bằng 3, 4, ... ,2016 nên tất cả các điểm đã vẽ đểu nằm trong hình tròn tâm A 2 (kể cả đường biên), suy ra tất cả các điểm đã vẽ đểu nằm trong s là phần giao của 2 hình tròn với đường biên của s là hai cung tròn BAjC và BA 2 C , Dễ thấy với 2016 điểm khác nhau, khi xét 2015 đoạn thẫng A,Aj vói j bằng 2, 3 2016 thì có 2015 trung điềm khác nhau và khi xét 2014 đoạn thẳng A 2 A k vối k bằng 3. 4 2016 thì có 2014 trung điểm khác nhau. Giả sử 2 đoạn thẳng A,|Aj và A 2 A k có trung điểm M trùng nhau vái Aj là một điểm nằm trong s. Nếu A nầm trèn A q A 2 thì A 2 A k = 2A 2 M = 2(A 2 A j + AjM) = 2A 2 A ] + 2A.M = 2A 2 A J + A 1 Aj =A 1 A 2 + AjA 2 > A 1 A 2 . Nếu Aj không nèm trên A 1 A 2 lúc đó cỏ hình bình hành A 1 A 2 A.A k vãi góc à 1 à 2 A j là góc nhọn, suy ra góc à 2 A 1 A| Í là góc tù, do đó điểm A k nằm ngoài s, điều này khống xảy ra. Như thế 2 đoạn thảng bất kì A,|Aj và A 2 A k có trung điểm khác nhau. (Xem tiếp trang 54) Đỗ THỊ HIỂN ANH {9 AI, THCS Yên Phong , Yên Phong , Bắc Ninh) C hiều muộn hôm đó, đang khóa cửa văn phòng để vể nhà thì thám tử Sêlỏccõc nhận được điện thoại của ông Long, ống Long cùng vợ là bà Trang là những ngưòi bạn thân từ thời phổ thông của thám tử.
  215. Lảl nữa đến nhà tỏi ăn tối nhé. Hôm nay, vơ tôi tự tay nấu nưàng đấy. Bà ý vẫn nhở những món mà ngày xưa chúng ta đều thích.
  216. Hay quá! cảm ơn õng bả. Tôi đến luôn đây. Khoảng nửa giờ sau thám tửSêlôccòc đã có mặt â nhà ồng Long. Lúc đó là gần 7 giở tối. Sau vài câu trò chuyện, ông Long mdi thám tử ngồi vào bàn ăn. Chà! Mâm cơm gia đình thơm ngon quá! Ông Long rót bia, còn bà Trang thì niểm nò giới thiệu món này món kia. Đang ăn uống vui vẻ, chợt bà Trang như sực nhớ ra điẽu gì, vội xin phép chạy lên gác. Rồi thám tử và ông Long nghe tiêng "Ối! Trài ai!" vọng xuống, cả hai ông vội chạy lên.
  217. Sao thế mình? - ông Long hốt hoảng hỏi.
  218. Lúc làm cơm tôi tháo cái nhấn kim cương ra cho tiện... Tôi đặt ở bàn phấn trong phòng, định bụng sẽ câ't ngay... Nhưng lúc đó có điện thoại nên tôi quên biến đi mất. Giở sực nhớ ra thì không thẫy đâu nữa.
  219. Bà cử bình tĩnh! - thám tử động viên - Hãy cho tôi biết lúc bà nấu ăn thì trong nhà có những ai. Một lát sau, khi đả đỡ hốt hoảng, bà Trang kể:
  220. Lúc đó có cô Hoa giúp việc, ông Minh vả con gái tôi.
  221. Ồng Minh có phải là người mà lúc tôi vừa đến bà giới thiệu là anh họ của ồng Long?
  222. Đúng rồi. Ông ấy đên Ci nhà tôi từ hôm qua. Vì có chút việc riêng nên ồng ấy không ngói ăn cùng chúng ta.
  223. Hiện giờ ông ấy vẫn đang ở đây?
  224. Vâng. Đang ở trong phòng trẽn tầng ba.
  225. Tôi có thể nói chuyện riêng vởi từng người chứ?
  226. Tất nhiên.
  227. À, mả lúc bà bắt đầu tháo nhẫn để nấu àn là khoảng mấy già nhỉ?
  228. Khoảng 5 rưỡi, gần 6 giờ, ông ạ. Đầu tiên, thám tử Sêlôccôc gặp cỏ Hoa:
  229. Từ lúc bà Trang nâu ăn đến giờ, cô đi làm gi, ở đâu?
  230. Tôi cùng nấu nướng với bà Trang. Hai người ở suốt trong bếp. Bà Trang cầu kì lắm nên tôi phải phụ giúp rất nhiều- Tiêp theo là ông Minh:
  231. Ông có thể cho tôi biết ông đã làm gì, ỏ đâu trong suốt thời gian bà Trang ỏ trong bếp nấu nưởng?
  232. Tôi ở trong phòng để giải quyết một số việc qua email. Tôi cũng có 1 cuộc gọi điện thoại sang Hàn Quốc cho ngưài bạn. ông có thể kiểm tra thông tin này.
  233. Thật ư? Tức là Lói có thể gọi cho bạn của ông để kiểm tra về cuộc gọi?
  234. Tất nhiên. Đảy là số của anh áy. ổng gọi đi! Thám tử gọi. Bạn ông Minh xác nhận là có nhận 1 cuộc gọi của ông Minh, gọi từ Việt Nam. Ông này cũng cho biết lúc đó là khoảng 6 rưỡi chiều. Cuối cùng, thảm tử Sêlôccốc gặp có Mai, con gãi bà Trang.
  235. Lúc mẹ cháu cdm nước trong bếp, cháu có phụ giúp không?
  236. Không ạ. Lúc đố cháu lau dọn phòng khách, quét sân quẻt cổng để đón bác đến chdi mà. Cháu còn cắm hoa nữa. Lọ hoa tươi trong phòng khách mà lúc nãy bác khen đó ạ.
  237. Cháu gái đảm đang quá! Sau đó, thám tử Sêlôccôc nói nhò với ông Long:
  238. Tôi đã tìm ra manh mối đáng ngờ rổi. Đây là việc rất tế nhị nén tốt nhất ông hãy tự mình nói chuyện với ngưởi mà tôi nghi vấn nhé.
  239. Ông Long nghĩ mãi nhưng vẫn chưa đoán ra người mà thám tử Sêlôccôc nghi ngờ, Các thám tử Tuổi Hống có giúp ông Long được không? J2Mm CHIẾC NHẪN TRONG TÚI (TTT2 số 161+162) Bạn nào cũng phát hiện chính xác điểm vô lí trong lời kể của bà giúp việc: Cả nục là cá biển chứ không phải cá sòng. Bà giúp việc tên Vân đã để lộ sơ hở này khi nóỉ chuyện với thám tử. Các thám tử ‘Tuổi Hổng 1 ’ cũng nhanh chóng phát hiện điểm đáng nghi ngờ đó. '• — “ Các bạn sau si được nhận quà: Ma/ Từng Dương ', 6A, THCS Lý Tự Trọng, Bình Xuyên, Vinh Phúc; Nguyễn Công Duy Khánh, 7A6, THCS Cãu Giấy, Cẩu Giấy, Hà Nội; Nguyễn Thị Minh Phương, 6G, THCS Đệng Thai Mai, TP. Vinh, Nghệ An; Trần Lâm, 6D, THCS Lê Hồng Phong; NguyỄn Trung Kiên, 7B, THCS Hoàng Xuân Han, Đức Thọ, Hà Tĩnh. Thám tử Sẽlòccôc NĂM NGÔI SAO TRONG CHÒM SAO ĐẠI $Ô VÀ LÍ THUYẾT SỐ PGS. TS. LÊ QUỐC HÁN (GV. Khoa toán, trường Đại học Vinh, Nghệ An) Trong lịch $ử trụng cận đại nhiêu nhà toàn học đã cỏ nhiều cống hiến cho nền toán học của nhãn toại. Sau đảy chủng tỗì xin giởi thiệu năm nhà toàn học tiêu biểu trong lĩnh vực đại số và lí thuyết số. Diophante (Khoảng thẻ' kỉ thứ III}. Cho đến nay. người ta chưa xác định được năm sinh và quốc tịch của nhà toán học Hy Lạp Diophante, chỉ biết rằng ông sống vào thế kỉ thứ IIL cùng thài vói nhà toán học Héron. Ngoài sự kiện ông đã thành đạt ở Alexandrie, người ta không biết đUỢc gì hdn vể ỏng mặc dù có một bài trào phúng trong Hợp tuyển Hy Lạp có ỷ dịnh dưa ra mật sô' chi tiết về cuộc đời ông (trên một mộ chi): Thời thơ ấu của Diơphante đà trôi qua trong một phần sáu cuộc đòi ông, một phần mười hai lúc là thanh niên, một phẩn bảy nữa khi còn là độc thân. Sau khi xây dựng gia đỉnh đt/ơc năm năm vợ óng sinh được một con trai song lại qua đời trước bổ bổn nãm, dũng bằng nửa sổ tuổi cha minh hưởng thọ. Hỡi khách qua đường, tính xem Diophante thọ bao nhiêu tuổi? Diophante viết ba công trình: ArithméUque, đó là còng trinh quan trọng nhất cùa ông và hiện còn giữ được 6 trong 1 3 quyển; về các đa giác, chỉ còn giữ lại vài đoạn; Porísms, hiện bị thất lạc. Arithmétique là một tác phẩm lớn phân tích sâu sắc các vấn đề lí thuyết đại sổ và lí thuyết số và cho thấy tác giả là mót thiên tài trong lĩnh vực này. Phần còn lưu gìữ được cũa câng trình này nói về cách giai khoảng 130 bãi toán rất khác nhau dến đến các phương trình bậc nhất và bậc hai. Một số phương trình bậc ba đặc biệt đã dược giải. Quyển thứ nhất nói vể các phương trình xảc định một ẩn vá các quyển còn lại bàn về các phương trình bất định bậc hai hoặc cao hơn vói hai hoặc ba ẩn. Có những kết quả sâu sắc vể số được phát biểu trong Arithmẻtique. Chẳng hạn: hiệu hai lập phương só hữu tỉ cũng bằng tổng của hai lập phương số hữu tỉ, mệt vấn để mà sau này đi được Viète, Bachet và Fermat nghiên cứu tới. cỏ nhiều mệnh đé nói về cách biểu thị các số dưới dạng tổng cua hai, ba hay bốn binh phương, một lĩnh vực mà sau 1500 nãm sau mới được hoàn thiện bởi Fermat, Euler và Lagrange. Trong tác phẩm Arithmétique, Diophante đã đưa ra cách viết tát cho các ẩn số, lên đến lũy thừa bậc
  240. phép trừ, đẳng thức và phép nghịch đảo. Ngưài ta xem ông là người tiên phong trong việc sử dụng kỉ hiệu đại số để trinh bày các lập luận của mình.
  241. Fermat (1601 - 1665). Nhà toán học Pierre de Fermat (1601 - 1665} sinh ồ Tarn-et-Garonne gần thành phố Toulouse nước Pháp trong một gia dinh kinh doanh da thuộc. Ồng vốn là một luật sư nhưng lại rất đam mê nghiện cứu toán học và được xem như nhã toán học Pháp vĩ đại nhất cùa thế kỉ XVII. Trong những đóng góp đa dạng của Fermat cho toán học thì nổi bật nhất là việc đặt nền móng cho lí thuyết số. Trong lĩnh vực náy, Fermat tò ra có một trực giác và một khả năng suy doán khác thưởng, cảm hứng của ông cố lẽ được khơi dậy bởi tác phẩm Arithmétique của Diophante thông qua bản dịch Latin cùa Bachet de Méziriac dược công bố nãm
  242. Nhiều đóng góp của õng về Lí thuyết số lả những ỷ kiến ghi ngoài lề trong bản sao của ông về công trình cùa Bachet. Nãm 1670, khi ông qua đời được năm năm, nhữhg ghi chép này đả được con trai cũa ông là Clément-Samuel thu thập lại đua vào lần xuất bản mối cuốn Arithmétique. Hai kết quả sau đây gắn với tên tuổi ỏng. Định li nhỏ Fermat. Nếu p lã số nguyên tổ và (a, p) = 1 thì a p - 1 - 1 chia hết cho p. Định lí lốn Fermat. với mọi số tự nhiên n > 3. không tồn tại bộ ba sổ nguyên dương khác không X, y, z thỏa mãn đẳng thức x n + y" = z n . Ông củng đưa ra phỏng đoán F n = 2 2 +1 là số nguyên tố với mọi n là sổ tự nhièn. Sổ F n = 2 2 1-1 được gọi là số Fermat. Sau này Euler đã chỉ ra F 5 là hợp số chứhg tỏ phỏng đoán này sai, í. Nevvton (1643 - 1727). Isaac Newton sinh ở xã VVoolsthorpe quận Lilcolnshire nước Anh vào ngày Thiên Chúa giáng sinh (ngày 25.12.1642 theo lịch Julius và là ngày 4.1.1643 theo lịch Gregory được sừdụng ngày nay). Cha ông vốn là một nông dân vá đã mất trước khi Newton chảo đài. Thủa nhò Newton gầy yếu sống â nõng thôn với mẹ và cha dượng, ông học ở trường làng. Năm 18 tuổi Newton mới được vào học trường Trinity College tại Cambridge. Tại đây ông đọc Nguyên lí cùa Euclide và thấy mọi vân để được trình bày trong đố qưả hiển nhiên. Sau đó ống đọc La géométne của Descartes thì thấy có cải gì đó khó hiểu. Ông còn đọc Clavìs cùa Oughtred, các công trình của Kepler và Viète, Arìthmetica intìni- torum của VVallis, Từ việc đọc toán học ông chuyển sang tỉm tòi nó và lúc 23 tuổi, ông chứng minh thảnh công công thức nhị thức tổng quát và phát minh ra phép toán vi phân, ông còn quan tâm đến nhiều vấn để VỂ vật lí học, quang học và phát minh ra các nguyên lí co bản của lí thuyểt động lực. Do nhửng thành kiến của giãi khoa học quỷ tộc Anh, nhiều còng trình quan trong của ỏng chl được công bố nhiều nãm sau khi tác giả khám phá ra chủng. Đỏ là một thiệt thòi không những chữ bản thân ông mà còn cho nền toán học Anh đương thời. Tuy nhiên óng cũng được đặt vào những vị trí xứng đáng với tàì nâng và sự cống hiến không biết mệt mòi- Nãm 1696, ủng được chỉ định làm Hiệu trưởng trường đại học Mint và năm 1699 được phong danh hiệu Master (bậc thầy) cùa trường đại học này. Nãm 1703 ông được bấu làm Chủ tịch Hội (Khoa học) Hoàng gia (Anh) và giữ cương vị đó cho tới lúc qua đởỉ. Năm 1705 òng được phong bá tước. Khi mất thi hài ông được mai tảng tại nhà thờ VVestminaster.
  243. Euler (1707 - 1783). Nhà toán học Leonhard Euler sinh năm 1707 ở Basel, Thụy Sĩ là con của một mục sư. ỡng tốt nghiệp và giảng dạy tại trưởng Đại học Basel. Nãm 1727 ông được nữ hoàng Nga Ekaterina I mồi làm Giáo sư đại học vễ toán tại Viện Hàn lãm Saint-Pétersbourg do Nga hoàng Pierre Đại đế sáng lập. Mưủi bốn năm sau, ông chấp nhận lời mời của vua Phổ Friedrich II đến Berlin giữ cương vị Viện sĩ trong Viện Hàn lảm Phổ. Hai mươi lãm nãm sau ông quay vế Viện Hàn lâm st, Pétersbourg và làm việc d đó đến lúc qua đời vào nãm 1783, thọ 76 tuổi. Euler viết rất nhiều công trình toán học; tên tuổi ông gắn vâi nhiểu ngành nghiên cứu toán học, nhiều kết quả sông mãi vói thòi gian: Phương pháp Euier trong giải phương trình bậc 4, Đường thẳng Eulạr và Đường tròn chín điểm Euler trong tam giác, Công thức Euter Đ - c + M 2 về đa diện (trong đó Đ, c, M tương ứng là số đỉnh, cạnh và mặt cũa một đa diện lồi), Định lỷ Euìer và hàm Euler trong Lí thuyết số, Cõng thức Euler Ẽ ix - cosx + isinx trong số phức ... ỏng còn dành nhiếu bài viết vể giải trí toán học như các đố thị (graphe) liên thông (được gợi ý bải 7 chiếc cầu của Konigsberg), đường đi cùa quân mã trên bàn cà. Ông cũng cóng bố nhiểu công trình trong lĩnh vực toán học ửng dụng, đặt biệt trong lí thuyết về chuyển động cùa mặt trâng, bài toán ba vật thể của cơ học thiên thể, sức hút của các vật thể, thủy lực học, chê tạo tàu thuyền, pháo binh và lí thuyết âm nhạc.
  244. F. Gauss (1777 - 1855). Nhà toán học vĩ đại nhất thế kỉ XIX và thưàng được xếp ngang hàng với Archimèdes và Isaac Nevvton là Carl Friedrich Gauss, một trong ba nhà toán học vĩ đại nhất mọi thòi đại. Gauss là thần đóng nổi tiếng từ thài niẻn thiếu. Người ta kể vể một câu chuyện lạ thường là năm mới lên ba tuổi ông đả khảm phá ra trong việc kế toán của cha có nhữhg chq sai. Năm 1796, khi 19 tuổi õng đã phát triển một lí thuyết cho biết rằng: Một đa giác đều dựng được bằng thước thẳng vá compa khi vá chỉ khi sô cạnh có dạng 2 r ’.Fj...F, trong đó F n = 2 2 ti là số nguyên tố Fermat. Như vậy các đa giác đều có số cạnh là 17, 257 và 65537 đểu có thể dựhg được bằng thước thẳng và compa, điểu này các nhà toán học từ thời Hy Lạp cổ đại đến thời của ông chưa giải được. Trong luận án tiến sĩ cùa mình viết lúc mái 20 tuổi ông đã trình bày một phép chứng minh đầu tiên và hoàn toàn thỏa đáng về Định li ũơ hản của đại số học (nói rằng một phương trinh đa thúc bậc n với hệ số phức CÓ ft nhất một nghiệm phức). Gauss có nhiều đóng góp trong thiên vân học, trắc địa, điện học, hình học ví phân và phương pháp bình phương tối thiểu. Năm 1821, trong một luận văn về chuỗi siêu bộl, Gauss đã nghlèn cửu có hệ thống lần đầu tiên về sự hội tụ của một chuỗi. Ông cũng là một trong ba nhà toán học phát hiện ra Hình học phi Eucliđ, tiếc rằng khi còn sống vì một lí do nào đỏ ông đã không dám công bố các kết quả cùa minh về lĩnh vực này. Ông khẳng đinh: "Toán học là nữ hoàng của các khoa học và li thuyết số là nữ hoàng của toán học 1 ', Bản thân Gauss được người đời sau mô tả là ‘"một người khổng lổ của toán học, với chiều cao của mình ông đã nhìn thấy trởi cao và biển thẳm 1 '. Từ nám 1807, ông giử cương vị Giám đốc đài quan sát thiên vãn và Giảo sư toán học tại Đại học Gottingen cho dến lúc qua đời. SUY LUẬN VÀ PHÁT TRIỂN TỪ MỘT BÀI TOÁN VƯƠNG THỊ HẢI (GV. THCS Thái Sơn, Đò Lương. Nghê An) Nhiều bài toán toán khó được mỏ rộng từ những bài toàn đơn giản, Vì vậy đế học giỏi môn Toán thì không nhCíng bạn phải nắm chắc kiến thức cơ bản, biết vặn dụng cấc bài toán cơ bẳn mà còn phải biết phát triển, mỏ rộng các bài toán đó theo các hướng khãc nhau. Trong bài viết này chúng tôi sẽ mở rộng, khai thác một bài toán cơ bản trong sách giáo khoa lâp 8. Bài toán 1. Chữ tứ giác ABCD có E, F, G, H, M và N thứ tự là trung điểm của AB. BC. CD. DA, AC và BD.
  245. Chứng minh EFGH là hình bình hành.
  246. Chứng minh EG, FH và MN đổng quy. Lởi giải. D G c
  247. Ta có EH, FG thứ tự là đường trung bình của AABD và ABDC nên BD „ BD EH//BD, EH -—7-; FG//BD, FG 2 2 =>EH//FG,EH=FG. Vậy tứ giác EFGH là hình bình hành.
  248. Ta cỏ EM, NG thứ tự là đường trung bình của AABC và ABDC nên EM II BC, EM - NG//BC, NG = 2 2 'Si EM II NG, EM = NG. Dũ đỏ tứ giác EMGN là hình bình hành. Suy ra EG và MN cẳt nhau tại trung điểm của mỗi đường. Mạt khác EFGFI là hình binh hành nên EG và FH cắt nhau tại trung điểm cùa mỗi đường, Vậy EG r HF và MN đổng quy, Nhận xét. ở bài toán 1 các tứ giác EFGH vá EMGN là hình binh hành. Cảu hỏi xuất hiện lè tứ giác ABCD cén có điều kiện gi để các tứ giác trên là hình chữ nhật, hình thoi, hình vuông? Ta có bài toán sau. Bải toán 2. Cho tứ giác ABCD có E, F, G, H , M và N thứ tự là trung điểm củaAB, BC, CD, DA.AC và BD.
  249. Tứ giác ABCD cần có điểu kiện gì để EFGH là hình chữ nhật, hình thoi, hình vuông?
  250. Tứ giác ABCD cẩn có điểu kiện gì dể EMGN là hình chữ nhật r hình thoi, hình vuông? Lờì giải. Xem hình vè bàl toán 1.
  251. Ta có EF là đưàng trung bình của AABC nèn EFÍ/AC,EF = ^. 2 Để EFGH là hình chữ nhật thì EH ± EF, mà EF ,'/AC vả EH i! BD nên phải có AC 1 BD. Để hình bình hành EFGH là hình thoi thì EH - EF. mà 2EH = BD và 2EF - AC nên phải có AC - BD. Từ đó để hình bình hành EFGH là hình vuông thì phài có AC I BD, AC = BD.
  252. Ta có EN lâ đưởng trung binh của AABD nẻn AD EN// AD f EN = -7-, 2 Để hình binh hành EMGN là hình chữ nhật thi EIM 1 EM, mà EN H AD và EM ìi BC nên phải có AD 1 BC. Từ đổ để hinh hình hành EMGN lè hỉnh thoi thi EN = EM, mà 2EN = AD và 2EM = BC nẻn phài có AD = BC. Từ đó để hình bình hành EFGH là hình vuông thì phải có AD -L BC, AD = BC. Nhận xét. Từ bài toán 1 nếu ta thay giả thiết như sau: E, F ; G, H thứ tự thuộc các cạnh AB, BC, CD. DA nhưng không là trung điểm mà vần thỏa mãn EH // BD // FG và EH = FG thì các giả thiết của bải toán 1 cần thay đổi như thế nào? Ta cỗ bài toán saư. Gọi p, Q thứ tự là giao điểm của BM và EF, của DM và HG, Do EF // ÂC nén theo hệ quả của định lí Thales ta EP BP PF ÃM " BM " MC Mà AM = MC nén EP > PF. Tương tự GH = QG- VI 0 là tâm đổi xứng của hình bình hành EFGH nên o là trung điểm của PQ. Vì PQ // BD và NB - ND nèn làm tương tự như trẽn ta được MN đi qua trung điểm o của PQ, tức là N, o, M thảng. Vậy EG, HF và MN đồng quy tạl Q. Bãi toán 2. Cho tứ giác ABCD, trên cảc cạnh AB, BC, CD, DA theo thứ tự lấy các điểm E, Ẹ G, H AE CF CG AH 2 - . , $ao cho — = — — = — — - = . Gội M, N thứtư AB CB CD AD 3 là trung điểm của AC và BD.
  253. Chứng minh rằng EFGH là hình bình hành. D G c AE AM
  254. Xét tam giác ABD có theo đinh lí AB AD Thales đảo thi HE//BD. (1) Xét tam giác GBD có = ậậ, theo định líThales CB CD đảo thì GF // BD. (2) Từ (1). (2) suy ra EH // FG. Theo hệ quà của định líThales thì f£ = 4§.f = £ = E| ,eh=fg BD AB 3 CB BD Vậy tứ giác EFGH là hình bình hành. b} Ta có EFGH là hình bình hảnh nên EG, HF cắt nhau tại trung điểm o của mẻi đường. Bài tệp vận dụng Bài toán. Cho tứ giác ABCD có diện tích s. Trên các cạnh AB, BC, CD, DA theo thứ tự lây các điểm E, F, G, H sao cho = - = 1 = ^1 = t(t>0). Gọi AB CB CD AD M, N theo thứ tự là trung điểm của AC, BD. Gọi o là giao điểm của EG và FH.
  255. Chứng minh tứ giác EFGH là hình bình hành.
  256. Tính diện tích tứ giác EFGFI theo s. c} Tìm t để diện tích tứ giác EFGH lân nhất.
  257. Chúng minh rằng 3 điểm M, o, N thẳng hàng. 2
  258. Tim t để S EEGH -^S ABCD .

    Kì 25 Hãy thay các chữ cái bởi các chữ số. Các chữ khác nhau biểu diễn các chữ sổ khác nhau. Lời giải cần có lập luận lôgic. SIIMK X THEM = DEEPDEEP TRƯƠNG CÒNG THÀNH (Sưu tầm) ■ 331® DE» Ki 24 (TTT2 số 161 + 162) ONE _F o u R ONE ONE TWO F I VE

    Với R < 1 0 thì R + E = E nên R = 0. Ta có I = 2.0 khỉ u + N = V (la) hoác I = 2.0 + 1 khi u + N = 10 + V (1b). Từ 2.0 T (2a) hoặc 2.0 + 1 T (2b) r suy ra

    0 < o <4. Từ 2E = o (3a) hoặc 2E = 10 + 0 (3b), suy ra o chẵn, do đủ o = 2 hoặc 0 = 4. Chú ý rằng khống đông thài xảy ra (la) và (2a), cũng như (1b) và (2b). Xét 2 trường hợp sau:

    THI. Với 0 = 4, có (la) thì xảy ra (2b) nên 1 =2.0 = 3 và T = 2.0 +1=9, hoặc có (1b) thì xảy ra (2a) nên I = 2.0 f 1 = 9 và I = 2 0 = 8. Từ (30) có E = 2 hoặc từ (3b) có E = 7.
  259. Xét I ~ 8 và T = 9 khi có (ia) và (2b). Vái E = 2 và có (2b) thì 2N = 10 + w nên N > 7

    (vì w chẵn, khác 2), do có (la) thì V = u + N > Ù + 7 mà v<7 (loại).

    Với E = 7 và có (2b) thì 2N + 1 = 10 + w, nên N > 5, do cố (la) thì V = u + N > u + 5 mà V < 6 do đó V = 6, N = 5, u = 1 = w {loại}.
  260. Xét I = 9 và T = 8 khi có (1b) va (2a). Vãi E = 2 vả câ (2a) thì 2N = w chắn nền chỉ xảy

    ra w = 6 và N = 3. Do có (1b) thìlG + V - u + N £ 7 + 3 = 10 (loại).

    Với E - 7 và có (2a) thì 2N + 1 = w < 6, nen

    N < 2, do có (1 b) thì 10 + V = u + N < 6 + 2 = 8 (loại).

    TH2. Vãí o = 2, có (la) thì xảy ra (2b) nên I s 4 và T = 5, hoặc có (1b) thi xảy ra (2a) nèn I = 5 và T = 4. Từ (3a) có E = 1 hoặc từ (3b) cố E = 6.
  261. xẻt I = 4 và T = 5 khi có (1 â) và (2b). Vdi E = 1 và có (2b) th) 2N = 10 + w. Do w chẵn và w > 6 thì N > 8, do có (la) thi V = U + N>U + 8 nên u = 1 = E (loại). ' Với E = 6 và cỏ (2b) thì 2N + 1 = 10 + w nên N > 8 (nếu N = 7 thì w = 5 = T), do cỏ (la) thi V = u + N > u + 8 nên u = 1, V = 9, N = 8 và w - 7, còn F = 3 (thỏa mãn).
  262. Xét I = 5 và T = 4 khi có (1 b) và (2a).

    Với E = 1 và có (2a) thì 2N = w chẵn, chỉ xảy ra N = 3, w = 6. dù có (íb) thì 10í V = u í N = u i 3, hay là V + 7 = u. Mà u>3vì E- 1, Q = 2 (loại).

    Với E = 6 và có (2a) thì 2N + 1 = w.
  263. Nếu N = 1 thì w = 3, do có (1b) thì 10 + V = u I N = u I 1 r hay là V I 9 = u (loại).
  264. Nếu N = 3 thì w = 7, dù có (ìb) thì 10 + V = u + N = u - 3, hay là 7 + V = u nên Ư = 8. V = 1 , còn F = 9 (thỏa mãn), Bài toán có 2 nghiệm như sau: 286 + 286 = 572, 321 0 + 286 = 3496; 236 + 236 = 472, 9280 + 236 - 9516. lVr . 1 , 1 Nhận xét. Một số bạn không chú ý rằng trong 2 phép toán ỏ để bài các chữ số giỏng nhau thì biểu diễn cùng một số, do đỏ cho các đáp số khác với 2 nghiệm trên. Các bạn giải đúng đưạc thưởng kì này là: Trương Văn Quốc, ac, THCS Lê Vãn Thịnh Gia Bình; Bùi Xuân Dưỡng, 9A1, THCS Yên Phong. Yên Phong. Bắc Ninh; Lê Đức Thái 9A2, THCS Yên Lạc, Yên Lạc, Vĩnh Phúc. 48 ĐAN QUỲNH THUÊ ô TÔ VŨ MULBERRY M ột số người chọn thuê õ tỏ khi đi chơi hay làm việc khỉ phải đí xa. Tiển thué thường gồm tiền trả theo ngày thuê và theo kilomet (hoặc dặm) đã đi. Thưởng là thêm một khoản đặt cọc. Ví dụ 1. Một xe ô tó cho thuê với giá $42.50 một ngày cùng với $0.45 mỗi dặm đường đã đi. Ma ri thuê xe 3 ngày và quãng đường đi là 270 dặm. Tìm số tiền Mari phải trả, chưa tính tiền đặt cọc. Lởi giải. Số tiền thuê 3 ngày 3 X $42.50 = $127.50 Số tiền trả theo quãng đường 270 X $0.45 = $121 .50 Tổng số tiền phải trả 1 27 . 50 + 1 2 1 .50 = $249.00 (Bạn hãy đổi kết quả trên ra tiến đồng của Việt Nam (VND) để hình dung rõ hơn về giá cả. Tỉ giả là SI = 22300 VND) Ví dụ 2. Bạn sang nước ngoài và thuê xe. Bạn hãy tính tổng số tiến cho hai hành trình sau: Cost per Day Number of Days Cost per Mile Number of Miles Rental Cost
  265. $49.25 5 $0.50 325
  266. S38.75 7 $0.45 435 Ví dụ 3. Moris thuê xe của cõng ti du lịch. Giả thuê theo ngày là $47.85 mỗi ngày và S0.35 cho mỗi dặm. Moris đã đi 5 ngày và chạng đường 200 dặm. Hãng giảm giá cho 15% của tổng số tiền. Hỏi Morìs phải trả bao nhiêu tiền cho chuyến đi? Lởi giải. Số tiền trả thuê theo ngày Sỏ tiền trả theo quãng đưởng Tổng số tiền chưa giảm giá Sau giảm giá, số tiền phải trả là 5 X $47.85 = $239.25 200 X $0.35 = $70 239.25 + 70 = $309.25 0.85 X 309.25 = $262.86 Ví dụ 4. Bạn đang ở nước ngoài và thuê xe, Bạn hãy tính số tiền cho hai hânh trình sau: Cost per Day Number of Days Cost per Mile Number of Miles Discount Rental Cost
  267. $38.50 5 $0.36 380 1Q0freemiles
  268. $44.75 7 $0.45 270 17%

    Bài dành cho bạn. Bạn hãy hoàn thành Ví dụ 2 và V7 dụ 4 rồi gửi kết quả về tòa soạn. Bài làm đúng, gủl sớm CÓ quà tặng. cuộc THI SÁNG TÁC CÂU HỎI VÀ BÀI TẬP PHÁT TRIỂN NĂNG Lực MÔN TOÁN CỦA HOC SINH BẬC THCS LỚP 8 MÃ: PTNL001 Câu 1. Cho ví dụ một phân thức mà sau khi rút gọn thì số hạng tử của mẫu nhiều hơn số hạng tử của mấu của phần thức ban đầu. Câu 2. Khi thực hiện phép trừ haì phân thức có mẫu khác nhau, có khi nào không cẩn rút gọn một phẩn thức (có thể rút gọn được) trưâc khi quy đổng không? Cho ví dụ. Câu 3. Tứ giác ABCD có BÃD - ÃDC, ABC - BCÒ. Hãy so sánh CÃD và CBD. Câu 4. TứgiácABCD là hỉnh gi nếu AC cắt BD tại o thỏa mãn S AŨB = S CŨD và S AŨỮ = ^BOC' Câu 5. Phân tích thành nhân tử i/64 -Jĩ + V15.

    — 4x^ T 4x — 3 Câu 6. Tìm sỏ' nguyên X để A = — — — - X 3 +3x 2 -3x + 4 là số nguyên. Cãu 7. Giàì phương trình 1 1 I 1 - 3 x 2 + 6x + 5 X 2 + 14X + 45 X 2 +22X + 117 - 3x-S Cãu E. Cho tam giác nhọn ABC. hai đường cao BH và CK. Chứng minh rằng BC 4 < (AB 2 +AC 2 )(BK 2 + CH 2 ). Câu 9. Cho tam giác nhọn ABC cố H là trực tâm. AB HC Chứng minh rằng nếu — — ■ = — — ' thì aABC cân. AC HB Câu 10. Cho tứ giác ABCD có ABC = 90° và AB - AD. Kẻ BH 1 AC tại H. Chứng minh rằng DH.H8.BC.CA- HC.CD.DA.AB. LỚP 9 MÃ: PTNL001 Câu 1. Cho (d): y - ax + b và (d'J: y “ a'x 4- b’. Tìm điều kiện của a, a’, b ; b’ để d và d' vuông góc vâi nhau tại một điểm trên trục hoành. Cãu 2. Cho hàm số y = f(x) = {-m 2 + 5m - 7)x 2 . Hấy so sánh f(4 - 2\/S) và f(7“3\/ẽ). Câu 3. AABC nội tiếp đường tròn (0) có AB - 26 cm, BC = 24 cm, CA = 10 cm. Tính tổng khoảng cách từ tâm o đến 3 cạnh của AABC. Câu 4. Hình thang ABCD vuông tại A và D có AB = 3 cm, AD = 8 cm, CD = cm. Hỏi BC có là tiếp 3 tuyến của đưòng tròn đường kính AD không? Câu 5. Giải phương trình X 2 + 6x - 30 = 14t/2x -3. Càu 6. Lập phương trình bậc hai có hai nghiêm x q vá x 2 thỏa mãn X 1 ị x 2 = 1 và (x 2 + x|){x 3 4 xjị) = 925. X 3 Cầu 7. Cho (d): y = -mx + m và {d'): y = — - 4 — m m (m 0). Chứng minh rằng giao điểm của d và d'

    luôn nằm trên một đường cố định, với mọi m 0, Câu 8. Cho AABC vuông tại A, đường cao AH, Kẻ HM 1 AB tại M và HN 1 AC tại N. Chứng minh rằng HnVch I HMỨBĨH = AH>/bc. câu 9. Cho aABC có ẢCB = 135°. Trẻn cạnh ẰB lây điểm D sao cho BCD = 90°. Chứng minh rằng

    Ị 1 _2 AD 2 AB 2 AC 2 ’ Câu 1ứ. Cho hình thang ABCD vuông tại A và B. Gọi M là trung điểm của AB. Chứng minh rằng: đường thẳng qua A vuông góc với MD, đường thẳng qua B vuông góc với MC và dường thẳng qua M vuông góc vói CD thì đổng quy. DANH SÁCH CÁC CÁ NHÂN ĐOẠT GIẢI cuộc THI VUI CHÀO HÈ 2016 BẬC THCS Nguyễn Văn Thanh Sơn, 9/1, THCS Nguyễn Khuyến, Đà Nang. í:- Giái Nhỉ: Vũ Mình Khẳi, 6A3, THCS Lâm Thao, Lâm Thao, Phú Thọ; Đàm Ngọc Hiếu , SH, THCS Trần Hưng Đạo, Đỏng Hòa, Phú Yên. £ Giải Ba: Cao Ngọc Toản f GV, THPT Tam Giang, Phong Điền, Thừa Thiên - Huế; Nguyễn Đãng VŨ, 8A f THCS Lê Vàn Thịnh, Gia Bình, Bắc Ninh!
  269. Giải Khuyến khích: Từ Tấn Dũng , 7D, THPT chuyên Hà Nội - Amsterdam, cẩu Giây, Hà Nòi; Lê Hải Phong , 6B, THCS Lý Nhật Quang, Đô Lương, Nghệ An. lEXm cuộc THI VUI CHÀO HÈ 2016 (Đề đáng trên TTT2 sổ 159+160 và 161+162) Bài 1. Kí hiệu a = 23, hoặc b = 32. Ta cần lập các số dạng cdegh gom 6 chữ số, trong đó có a và 4 chữ số trong tập các chữ số s = {0, 1 , 4, 5, 6}. Chữsồ c khác ũ nèn có 5 cách chọn (từa và 1,4, 5, 6), chữ sd d có 5 cách chọn (từ a và 0, 1,4, 5, 6 không kể c), chữ sô' e có 4 cách chọn (tư a và Q, 1 , 4, 5, 6 không kể c, d), chữ số g có 3 cách chọn (từ a và 0, 1 , 4, 5, 6 không kể c, d. e), chữ sỗ h có 2 cách chọn (từ a và 0, 1, 4, 5, 6 không kể c, d,
  270. g)- Như thế cỗ 5!. 5 cách chọn cỏ a. Nếu thay a bài b thì có 5!-5 cách chọn có b- Vậy tổng số có 5E.ỈŨ - 1200 số. Bài 2. Chú chuột đi theo đường ngắn nhất nên chỉ có thể đĩ từ trái sang phải, hoặc từ trên xuống dưởi theo đưủng lưới. Số đường đi như thể từ điểm A đến điểm X bằng tong của sô’ ở bên trái và sô ở phía trên của X vả được ghi trên sơ đồ tại phía dưới bên phải điểm X như dưói đây. 1 1 1 1 1 1 1 2 3 4 5 6 7 7 1 3 6 to 15 21 28 35 1 4 10 20 35 56 84 119 1 5 15 35 70 126 210 329 5 20 55 Ĩ25 251 461 790 35 ỉ 54 483 1273 20 75 200 451 912 1702 B 2975 Bài 3. Đánh sô các ô như hình dưới đây. 1b 2a 2b 3a 3b 4a 4b 5a 6b 7a Sb 9a lOb lia 5b 6 a 7b Sa 9b lOa llb I2a I3b 14a I5b I6a 1 7b 18a 12b I3a 14b 15a 16b 17a 1 8b Cách đí để Hà thắng Nam như sau. Hà đi trước, đảnh dấu X vào ô (1b) phía trên bén trái. Chú ý rằng kể vối mỗi õ (b) chỉ là ỏ (a) nèn Nam chỉ được đảnh dẩu vào ô (a), nếu Nam đánh dấu vào ồ (ka) với số k nào đó từ 2 đến 18 thì Hà luôn đánh dấu được váo ô (kb) kề vói ô (ka), như thế Hà là người đánh dấu cuối cùng nên thắng cuộc. Đài 4. Kí hiệu các số ghi tại các ô tròn như ở hình có ba đa giác 7 cạnh duứi đây Để dễ tính toán ta giảm đi 86 ở mèi số cần ghi và chuyển về bài toán điền 21 sá nguyên liên tiếp từ 0 đến 20 vào các ô tròn vởi tổng các số là 0 4 - 1 + 2 + ... + 20 - 21 0. Tổng ba số trên mỗi nửa đường thẳng Aj + B| + Cj = 210 ; 7 = 30, tổng 7 số theo mỗi hàng A 1 + A 2 + ... + A 7 = B 1 -Ị- B 2 + ... + 6 7 = C 1 +c 2 + ... + C 7 là210: 3 = 70. Theo giâ thiết có bảng sau, trong đó ba số ghi trên mồi nửa đường thẳng ghi theo cột, bảy số trên mỗi đa giác ghi theo hàng ngang. 1(30) 2(30) 3(30) 4(30) 5(30) 6(30) 7(30) A(70) 0 7 BỰC) 13 20 3 c<70) 17 10 Từđỏ à cột 1 là + c, = 30 - 0 = 30. T ương tự ở cột 2 là A 2 + B 2 = 30 - 17 = 13. ở Cật 3 là A 3 + c 3 = 30 - 13 = 1 7. Ở cột 4 lảA 4 + 0= 30- 20 = 10. ở cột 5 là Bg + Cg - 30 - 7 - 23. Ở cột 6 là Ag + Cg = 30 - 3 - 27. Ở cột 7 là Ă 7 + B 7 = 30 - 10 = 20. Mặt khác ở hàng A cỏ A a + A 3 + A 4 + + Ay — 70 7 — 63. ớ hàng B có
  271. B 2 + B 5 + B 7 = 70 - (1 3 + 20 I 3) = 34. Ở hàng c có C 1 + C 3 +C 4 + C 5 + C 6 = 70 -(17 + 10) = 43. Diển các số vào bảng sao cho sổ ò các ô đều khác nhau ta được kết quả sau 1(30) 2(30) 3(30) 4(30) 5(30) 6(30) 7(30) A(70) 0 5 15 6 7 18 19 B(70) 14 8 13 20 11 3 1 C(70) 16 17 2 4 12 9 10 Cộng thêm 86 vào mối số trong bảng trên ta điền được các số vào hình các đa giác 7 cạnh như sau. Bài 5. Đặt số ở các mặt ABCD, ABNM. ADQM, PNBC, PQMN, PQDC tương ứng là x r y, z, t, u, V, Theo giả thiết có x + y + z-=4(1)à đỉnh A, X - y
  272. t - 5 (2) ở đỉnh B, t + ư + V - 6 (3} ở đỉnh p. Tổng các số ò 6 đỉnh bằng tổng các sô ở đình A và đỉnh p nên bằng 10.
  273. Tổng ba số ở đỉnh Q bằng tổng các số ở 6 đỉnh trù đi tổng ba số ở đỉnh B nèn bằng 10-5=5.
  274. Dô 4 = 1 + 1 + 2 nẻn từ (1) chỉ có thể xảy ra các

    trưởng hợp sau đáy. • THI. X = y = 1 và z = 2 thì t = 3 theo (2), lúc đó 14 + V = 3 theo (3) nên có 2 cách điển số là (u, v) bằng (1, 2), hoặc là (2, 1).

    TH2. X = z = 1 và y = 2 thi t = 2 theo (2), lúc đó

    u + V = 4 theo (3) nên cỏ 3 cách điền số là (u, v) bằng (1,3), hoàc lè (3, 1), hoặc là (2, 2).

    TH3. ý = z = 1 và X = 2 thi t = 2 theo (2), lúc đó u + V = 4 theo (3) nên cỏ 3 cách điền số là (u, v) bằng (1, 3), hoàc là {3, 1), hoặc là (2. 2). Vậy tất cả có B cách điển các số vào các mặt của hình lập phương, Bải 6. Xét số gâm 6 chữ số khác nhau và khác 0 là n - abcdeg . Theo già thiết cóa + b + c + 1= d + e + g. Đât a + b + c = m thì tổng 6 chử số bằng 2m + 1 , là số lẻ. Do đó trong 6 chữ sổ có thể có 1, hoặc 3, hoặc 5 chữ số lẻ, xày ra 3 trường hợp sau đây. • THI . Giả sử a t bic + d + e I g = 1 + 2 I 3
  275. 4 + 5 + 6 = 21 thì m = 10 và m + 1 = 11 . Ta phân tích m-10-1 +3 + 6 = 1 + 4 + 5 -2
  276. 3 + 5 được 3 tồng 3 chữ sô.

    Vối mỗi tồng 3 chữ số, chẳng hạn 1 + 3 ) 6 có 6 cach viêt la 1 + 3 + 6, 1 — 6 + 3, 3+1 + 6, 3 + 6+1, 6 + 1 + 3, 6 + 3 + 1, với tổng 3 chữ số còn lại là 2 + 4 + 5 cũng có 6 cách viết nên có 6.6 = 36 cách viết số có 6 chữ số. Từ đó vâi 3 tổng cỏ m = 1 0 thì có tất cả là 3.36 = 108 cách viết. Kết quả tương tự với số gồm 6 chữ số liên tiếp như (2, 3, 4, 5, 6, 7) có 2m + 1 = 27, (3, 4, 5, 6, 7, 8) có 2m + 1 = 33, (4, 5, 6, 7, 3, 9) cỏ 2m + 1 = 39.

    TH2. Giả sử a + b + c + d + e- g-1+2+3
  277. 4 + 5 + 3 = 23 thi m = 11 và m + 1 = 12. Phân tích m = 11 = 1 + 2 + 8 = 2+ 4 + 5chì được 2 tồng 3 chữ số. Tương tự vâi mỗi tổng 3 chữ số có 6 cách viết, với tổng 3 chữ số còn lại cùng có 6 cách viết nẻn có 6.6 - 36 cách viết sổ có 6 chữ số. Từ đó với 2 tổng có m = 11 thì có tất cả là 2.36 = 72 cách viết. Kết quả tương tự với số gổm 6 chữ số như (1 , 2, 3. 4, 6, 7) có 2m' + 1 X 23, (1, 2. 3, 5, 6, 8) có 2m + 1 = 25, (1. 3, 5, 6, 7, 9) có 2m + 1 =31, ... • TH3. Giả sử a-rb+c + ct + eTg = 1 + 2+ 3
  278. 6 + 7 + 8 = 27thĩ m = 13 vè m + 1 = 14. 52 Phân tích m 13 2 + 3 + 8 chỉ được 1 tổng 3 chữ số. Tương tự với mồi tổng 3 chữ sô có 6 cách viết, vởi tống 3 chữ số còn lại cũng có 6 cách viết nên có 6.6 — 36 cách viết số' có 6 chữ số. Từ đó có tất cả là 36 cách viết. Kết quả tương tự với số gổm 6 chữ số như [1 , 2, 3, 6, 8. 9) có 2m + 1 = 29, (1, 2, 4, 7, 8, 9) có 2m + 1 =31, (2, 3, 4, 7, 8, 9) có 2m + 1 = 33. Vậy số cách viết các số góm 6 chữ Sũ khác nhau và khác 0 có thể là 36 cách, 72 cách hoặc 108 cách. Nếu xét chi7 sô 0 thi còn loại bỏ trường hợp chữ số đầu trải khác 0 nên số cách viết sẽ ít hơn. Bài 7. Gọi chiểu cao của 1 5 cây dừa theo thử tự là a T a 2 ’ — ' a l4' a is- Theo giả thiết có |a 1 -a 2 | - |a 2 — a 3 | — = |a 1+ -a 16 | = l a is - = k > 0. Từ đó aj - a j+1 = ke j, và a 1 - a t = ke 1 5 với 6j bằng 1 hoặc — 1 ( 1 - 1 , 2,..., 16). Cộng theo vế 15 đẳng thức trên được k(e 1 Hr e 2 + ... + e 15 ) = 0. Mà e 1 + e 2 + ... + e 15 khác 0 (vì tổng 15 số 1 và -1 là số lẻ), suy ra k = 0. Do đó a 1 - a 3 = a 2 - a 3 = ... = a 14 - a 1s = a 15 - a T = 0. Suy ra điểu phải chứng minh. Bải 8. Ta gọi là bạn Việt được thưởng m điểm nâu câu m đúng. Giả sử trong 20 câu trẽn bảng có a câu khẳng định sai với 1 < a < 19. Xét các phiếu theo thứ tự mà ban tổ chửc đưa cho bạn Việt thì tử phiếu số 1 đến phiếu số a cỏ nội dung đúng, còn từphiỂu a + 1 đến phiếu 20 (gồm 20 - a phiếu) có nội dung sai. Để nhận được tiền thuồng nhiều nhất thì bạn Việt cần chuyển các phiếu số 1 . 2, 3 a, a + 1 19, 20 trử thành các phiếu theo thứ tự là 20, 19, .... a + 1 , a 3, 2, 1 thì tất cà a phiếu nội dung dũng từ phiếu số 1 đến phiếu số a được thưâng với số điểm nhiều nhất từ 20 điểm, 19 điểm, ... lẩn lượt đến hết a phiếu. Tổng sô điểm bạn Việt có được là 20 + 19 + 18 +„.4 (21 -a) (20 + 21-a).a a(41-a) 2 2 Tổng số tiền bạn Việt nhận được là .200000 = 100000. a(41 -a) đồng. Đài 9. Sô đoạn ống đèn tuýp trong các hình H, E, dấu huyền, 2, 0,1,6 tương ứhg là S 0 = (5, 5, 1, 5, 6, 2, 6) vả cũng là sổ đoạn ống đèn tuýp không sáng lúc đầu. Gọi số đoạn ống đèn tuýp không sáng tại thỏi điểm t khống thao tác bật đèn là s t = (a, b, c, d, e, f, g). Tại thời điểm kết thúc mọi đèn đểu sáng thì s k = ( 0 , 0 , 0 , 0 , 0 , 0 , 0 ). ‘ Nêu chọn hình H và tại thơi điểm t + 1 bật p đèn sáng với 1 1 < p < a thì s t 1 = (a - p, b. c, d, e, f. g). Nếu chọn hai hình H, E và tại thài điểm t + 1 bật p đèn sáng vôi 1 < p < a, 1 < p < b thì s, t - (a - p, b - p, c, d, e, f, g) Nhận xét. Xét thủi điểm có cặp số bằng nhau (n. n), nếu một người chơi A bật p đèn với 1 < p Ể n chuyển vể cãp số (n - p, n) thì người chơi 8 cần bật p đèn ử hình thử hai để chuyển về cãp số nhỏ hơn bằng nhau (n - p, n - p). ... Cứ như thế cuối cùng người chơi B sẽ chuyển vé cặp số (0, 0) vả thắng cuộc. Với nhận xét trên, ta có thể bò qua các cặp số bằng nhau (5, 5, 6, 6) trong s ữ , tức là chỉ cần xét bộ ba số Rp - (1, 5, 2). Chiến thuật để ngưởí B đi trưâc có thể thắng cuộc là bật 2 đèn sáng à hình giữa (sổ 2) cỏ 5 đèn chưa sáng để chuyển về R 1 = (1, 3, 2). Đến đêy, nếu A chuyển đến bộ ba số R 2 nào đó thì 8 luôn chuyển về bộ ba số dạng R 3 - (0, 2, 2), hoặc Rj - (1 , 0, 1 ), hoệc R 3 = (1 , 1 , 0} và sẽ thắng cuộc theo nhận xét trên. Tuy nhiên, nếu 8 không đưa vể một trong 3 dạng đó thì không chắc thắng. Bài 10. Đáp án như sau 2016 603 207 621 621 81 126 63 162 36 63 36 NGUYÊN VIỆT HẢI 2 0 1 6 1 6 0 2 2 Ũ 1 6 0 6 2 1 6 2 0 1 6 1 2 0 22ans» SO ĐIEM ... (Tiếp theo trang 41) Vậy các trung điểm của 2015 đoạn thẳng A,|Aj với j bằng 2, 3 2016 và các trung điểm của 2014 đoạn thẳng A 2 A k với k bằng 3 , 4 2016 là khác nhau, do đó có ít nhất 4029 trung điểm khác nhau. Một cách vẽ 201 6 điểm chỉ có 4029 trung điểm khác nhau (bạn đạc tự kiểm tra) như sau. Trèn một đoạn thẳng A 1 A 2 lấy 201 6 điểm A 1 , A^. A + A £ũ1 5 , A 201 6 , A 2 theo thứ tự cách đều nhau, tức là A 1 Aj - AjA,
  279. — _rt 201 5^2016 rt 2016 rt 2' Vậy giá trị nhỏ nhất cùa T bằng 4029. Nhận xét. Tất cả các bạn gửi bài giải đã \ tỉnh được giá trị lớn nhất của T nhưng chưa chỉ ra cách vẽ hình Chỉ có bạn Trương Thị Thu Lan , 8A2, THCS Yên Phong, Yẻn Phong, Bắc Nính, chỉ ra được cách dựng 2016 điểm sao cho chỉ có 4029 trung điểm khác nhau, tuy nhiên chưa chứng minh được đó là giá trì nhỏ nhất của T. Bạn Lan là người được thưâng ki này. ANH COMPA A 2 TTCEíT BEt UIHIT 20. GAS LDlil nno PARTICLGS OP mnĩTER THEŨRV SECĨIOn (TTT2 số 161+162) cãu 1. Một khổi lượng khí nhất định chiếm 9.0 lít không khí 5 nhiệt dộ 300 K và áp suất 1.2 atm. Thể tích được giảm xuống còn 5 lít bằng cách tàng áp suất lên 2.3 atm.
  280. Giả sử rằng khí đó ở dạng khí lí tưởng, tính nhiệt độ của khí đó sau khl giảm thể tích.
  281. Chỉ ra lí do tại sao nhiệt độ thực có thê’ khác so vối nhiệt độ tính được. Câu 2. Sử dụng lí thuyết phân tử, giải thích ngắn gọn tại sao:
  282. Chất lủng bay hơi nhanh hơn khi có giỏ lùa.
  283. Sự bay hơi nhanh cúa chất lỏng làm mát chất lỏng đó.
  284. Nhận xét. Các bạn có bài dịch tốt và

    được nhận quà ki này là: Vù Trung Tiến, 8A2, THCS Trưng Vương, Mê Linh; Phan Anh Khánh, 9A2, THCS Giảng Võ, Hà Nộí; Nguyên Thị Thu Trang, 9A1, THCS Yèn Phong, Yên Phong, Bắc Nính; Nguyễn Thủy Mai, 8E1; Lê Tất Hoan, 7D, THCS Vĩnh Tưòng, Vĩnh Tưởng, Vĩnh Phúc. MAI MY lanm ĐIỀN SỐ Bài 1. Điền số thích hợp vào chỗ trống sao cho hợp lôgic. Bài 2. Điền số thích hớp vào chỗ trổng sao cho hợp lỏgic. 0 8 4 16 16 ?

    4 1 12 9 20 ? ■' NGUYỄN Đữc TẤN (TP. Hổ Chí Minh) SỐ NÀO? (TTT2 SỐ 161+162) Nhận xét. Quy luật của hai hài kỉ này tương đối khó r hầu hết các bạn tìm đủng kết quả nhưng chưa nêu rõ quy luật của các số. Quy luật. õài 1. Mỗi số của dãy là số chỉnh phương lán nhất có số chữ $ 6 bằng số thứ tự của nó trong dãy. Do vậy số tiếp theo (số thứ nám) là số chính phương lớn nhất có 5 chữ số, đó là số 99856 (- 316 2 }. Bài 2. Mỗi số ò hàng trên nằm chính giữa hai số ở hàng dưới, nó bằng tích hai số đò ở hàng dưới cộng vơi 3. Theo quy luật dó thì X = (-7) X 11 + 3 = (-74); y (-74) X (-8) + 3 595. , C ' ? Xin trao thưởng cho các bạn nêu rõ quy 7 --,/ luật của cả hai bài: Nguyễn Huy Quý, ——— 9A THCS Lf Tự Tr ọng' Bình Xuyên, Vĩnh Phúc: Nguyễn Thanh Sơn, 6A3, Vũ Linh Chi, 9A3, Bùi Trọng Vinh, 9A3, THCS Lâm Thao. Lâm Thao, Phú Thọ; Hà Đình Tâm, 8A2, THCS Yên Phong, Yên Phong, Bắc Ninh, Các bạn sau được tuyên dương: Phùng Thị Thùy Dung, Đàm Quang Anh, Hoàng Thị Ngọc Diệp, 8E1, THCS Vĩnh Tường, Vĩnh Tường, Vĩnh Phúc; Trân Tiến Đạt. 8A3.THCS Lâm Thao, Lâm Thao, Phú Thọ; Nguyễn Thị Quỳnh Anh, 9A1. THCSThị trấn Quán Hành, Nghi Lộc; Nguyễn Công Khanh. 6D, THCS Đăng Thai Mai, TP. Vinh, Nghệ An. NGUYỄN XUÂN BÌNH BẠN CO Y KIEN GI KHAC? Bài toán. Chú biểu thức p = - 1 24 VX+ 1 Vx-1 x-l' Tim các giá trị cũa X để biểu thút p có giá trị nguyên. Một học sinh đả giải như s au: ĐKXĐ Ta có p 'x>0 \x 1 Vx-1 + 2{Vx+1)-4 w* + IKvx-1) 3^-3 3
  285. (\'X + U{^ -1) - \'X +1 Để p có giá trị nguyên thi (Vx + 1) e { - 3; - 1 ; 1 : 3). Vì X > 0 nên Vx +1 > 1. Do đó \/x + 1 = 1 |~x = 0 Vx + 1 = 3 x = 4 {thỏa mãn £KXĐ). Vậy X = 0; X = 4 thỏa mãn bài toán. Các bạn có ý kiến gì khác không ? NGUYỄN NGỌC HỪNG (GV, THCS Hoàng Xuân Hãn, Đức Thọ, Hà Tĩnh ) 33HỈE* LỜI GIẢI DÃ DÚNG CHƯA? Lời giải đã đãng chưa ổn vì chia hai bất đẳng thức cùng chiều thì không thể được một bất đẳng thức! Một lời giải đúng như sau: Sử dụng giả thiết X 2 + y 2 = 4, ta có 2xy = (x + y) 2 - {x 2 + y 2 } = (x + y) 2 - 4 = (x + y - 2)(x + y + 2). Vãi X + y H- 2 khác 0 và (x ị y) 2 < 2(x 2 + y 2 ) thì A= xy _ xry-2 x+y+2 2 <|w'2(x 2 +y 2 )-2) = |(V^4-2) = 2-1.

    Đẳng thức xảy ra khi X “ y - V 2 . Vậy MaxA V2 -1 khí X - y - V 2 . — Nhặn xét. Nhiều bạn chì ra đưực cho sai của lởi giãi đá đang báo, nhưng chi có bạn Tran Quang Tà/. 8A1, THCS Yên Phong, Yên Phong, sắc Ninh đưa ra lài giải đúng. ANH KÍNH LÚP KIDH nBHIỆII) GÈG D|Ụ CÁC BÀI TOÁn OUỸ TÍCH NGUYỄN THỊ BỈNH (707 tòa 8, Muibeny Lane, Hà Đỏng, Hà Nội) N ói đến quỹ tích (tập hớp điểm), đa số học sinh sợ dạng toán này. Mệt vân đế đặt ra cho người thầy là làm thế nào xóa được ấn tượng đó trong học sinh. Điều đó chi đạt được khí học sinh nắm được phương pháp suy nghĩ và tính chất riéng của bài toán. Chính vì vậy trong quá trình giảng dạy lí thuyết cũng như trong bài tập, người thây cầh cố gang trình bày để học sinh nắm được nhũtig vấn đề sau:

    Hiểu được định nghĩa quỹ tích: quỹ tích

    những điểm M có tính chất « là một hình bao gốm tẩt cả những điểm cỏ tính chất re và chì gâm những điểm đứ. Tức lả M có tinh chất re thi suy ra M thuộc hình A và ngược lại nếu điểm M thuộc hình A thì M phải có tính chất a. Hiểu thấu đáo vấn để này học sinh mới thầy hết ý nghĩa và tầm quan trọng của cả hai phần thuận và đảo. Thưàng các em học sinh và giáo viên cũng hay coi thường và bò qua phần đảo (giáo viên chỉ chữa phần thuận, phần đảo để học sinh tự chứng minh nên đã tạo thói quen coi thưởng phần đảo),

    Nắm được các bưâc làm một bài toán quý

    tích gổm có:

    BƯỚC 1. Dự đoán quỹ tích. Bước 2. Chứng minh phần thuận. Bước 3. Giới hạn quỹ tích. Bưâc 4. Chứng minh phần đảo. Bưồc 5. Kết luận quỹ tích.

    Phẩn dự đoán quỹ tích có thể chĩ làm nháp mà khàng cẩn viết vào bái, nhưng nó là bước quan trọng để tim ra hưóng giải cho bài toán. Học sính nên nắm vững đặc trưng của từng dạng quỹ tích. Cho đến chương trình hình học lãp s, quỹ tích thường gặp chủ yếu là hai dạng: dạng thắng (đường thẳng, đoạn thẳng, nửa đường thẳng) và dạng tròn (đường tròn, cung tròn, một phần cung tròn).

    Dạng thẳng hay gặp là đưửng phân giác,

    đường trung trực, dường thẳng MA 2 - MB 2 = k 2 (các quỹ tích cớ bản).

    Dạng tròn: Cung chứa góc (bằng 90° hoặc nhỏ hơn 90' J hoặc lán hơn 90°), đường tròn Apôlâniút MA 77^7 “ k 1 , M nhin 2 đoạn kề thẳng hàng dưới 2 MB gôe bằng nhau... Việc chủng minh những bài toán quỹ tích sẽ đưa về dạng quỹ tích cơ bản. Vỉ vậy học sinh cần nắm được tính chất của các quỹ tích cơ bản ấy. Trên đây là một số vẩn đề khải quát cỏ tính chất li thuyết. Sau đây là phương pháp làm các bước của một bài toán quỹ tích qua mọt số vỉ dụ. Buởc 1. Dự đoán quỹ tích Nhu trên đã nói, ở cấp trung học cơ sở chủ yếu gập 2 dạng hình của quỹ tích là dạng thẳng và dạng tròn. Phải làm thế nào cho học sinh đoán được quỹ tích này sẽ là đưàng gì? Có mấy phương pháp để đoán nhận quỹ tích; đoán nhận bằng lí luận, đoán nhận bằng phần tử đặc biệt, phần tử vô tận, đoán nhận bàng vẽ nhiều vị trí khác nhau của điểm cần tìm quỹ tích. Đối vâi học sinh yếu, nên cho học sinh đoán bầng cách vẽ điểm cần tìm quỹ tích ở ít nhất 3 vị trí khác nhau. So sánh vị trí tương đối của các điểm đó ta sẽ dự đoán được hình dạng của quỷ tích (nếu các điểm thẳng hàng thi khi đó quỹ tích cỏ thể lá đường thẳng, nếu cỏ 3 điểm khủng thẳng hàng thì quỹ tích có thể là đưdng tròn,,,.). Đối với học sinh khá hơn có thể dự đoán quỹ tích bằng phần tử đặc biệt, phần tử vô tận. Ví dụ 1. Cho (O, R) vãi điểm p nằm trong đường tràn. APB = 90° , hai điểm A và B thuộc đuờng tròn (O). Điểm M thuộc AB sao cho MA= MB. Tìm quỹ tích điểm M khi APB quay quanh điểm p. B Dự đoản quỹ tích. Đối với bài toán này có thể dự đoán quỹ tích bằng phán tử đắc biệt như sau: Xét ÃPB ở vị tri khi cạnh PA đi qua tâm o. Nối OM ta được tam giác OMA vuông ở M. Khi đó MO 2 I MA 2 = R 2 . Trong tam giác vuỏng BPA có PM = MA. Từ trên suy ra MO 2 + MP 2 = R 2 Tử điều này ta có thể đoán quỹ tích là đường tròn dựa vào đính lí Apõlôniủt. Từ đó có hướng chứng minh đối với một vị tri bất kì, Ví dụ 2. Tìm quỹ tích các điểm chia các đoạn thằng tựa trên 2 đường thẳng song song cho trước theo một tì số cho trước, X Dự doản quỹ tích. Đỏi với bài này nên dự đoán bằng phần tử vỏ tận. Ta thấy 2 đường thẳng song song dài vô tận nên các đoạn thẳng tựa trên 2 đường thẳng ấy cũng có thể ồ xa vỏ tận. Do vậy phải có những điểm chia chúng ò xa vô tận, từ đỏ quỹ tích phải là đường thẳng hoặc nửa đường thảng. Nhưng trường hợp này, vô tận cả 2 phía nên ta có thể dự đoán quỹ tích là đường thẳng. Bước 2. Sau khi dự đoán quỹ tích, bát đlu bưỡc vào chứng minh phẩn thuận. Để chứng minh phần này cần xác định được những yếu tố cố định, không đổi và những yếu tố thay đổi. Trẽn cơ sà đó dựa vào tính chất các dạng quỹ tích đã phân loại ở trẽn và dự đoán của mình để chứhg minh. Nếu dự đoản là dạng thẳng thi xét xem điểm cần tìm quỹ tích có thể nằm trên đưởng phân giác của một góc Cố định nào đô, hoặc nằm trên đường trung trực của một đoạn thẳng cố định hay đường mà hiệu của các bình phương khoảng cách đến hai điểm cò định hay không? Nếu dự đoán là đường tròn thì hãy cố gắng chứng minh xem góc tại điểm đó nhìn một đoạn thẳng cố định có bằng 90° hay không? có thay đổi hay không? Hoặc có thỏa mân điều kiện của đường tròn Apôlỏniúl hay không? (xét tỉ số, hoặc 2 góc). Ví dụ 3. (Chứng minh phần thuận của ví dụ 1} Ta đã dự đoản quỹ tích trong ví dụ 1 là đường tròn, mà 2 điểm cố định là p và 0 nẻn bày giờ cần phải chũrig minh MP 2 + MO 2 - k 2 . Ở đáy trong bước dự đoán ta đã thấy k 2 = R 2 nên sê chứng minh MP 2 + MO 2 = R 2 . Điều này chứng minh được dựa vào cõng thức đường trung tuyến và hệ thức lượng trong tam giác vuông (bạn đọc tự chửng minh chi tiết). Chú ý. Trong phẩn thuận lưu ý cho học sinh: chưa được dùng từ ‘quỹ tích". Trong ví dụ trèn, sau khi chứng minh được MP 2 + MO 2 = R 2 nếu học sinh trả lời; “Do đó quỹ tích của M là đường tròn tâm I bán kính r, trong đó [ là trung điềm của Pũ. r--jV2R 2 -OP 2 ." là sai. Chỉ có thể được dùng tí/ ‘quỹ tích” khỉ đã chứng minh xong phấn đảo. Bước 3. Giói hạn quỹ tích Phương pháp thông thường để giãi hạn quỹ tích là xét các vị trí đặc biệt của điểm cẩn tim quỹ tích trong quá trình nó thay đổi. Ví dụ 5. Cho (O) vởi đương kính AB cố định và M di chuyển trẽn {O). Lấy điểm I thuộc tia đối của tia MA sao chù IM = 2MB. Tìm quỹ tích điểm I, Hướng dln. Sau khi chứng minh được điểm I thuộc 2 cung chứa góc MIB cố định, ta xét các vị trí đãc biệt sau: • Khi M chạy trên nửa đưàng tròn, thì điểm I có chạy hết cung chứa góc phía trên không? Ta thấy điểm M trùng với điểm B nên MB = 0. Má lại có IM = IB. Vậy ta cá IB = 2, 0 = 0 , điều này chứng tỏ I trùng vởí B. • Khí M di chuyển đến điểm A thì MA trở thành tiếp tuyến của đương tròn tại tiểp điểm A. Khi đó I trùng với l r • Cho M di chuyển tiếp trên nửa đường tròn dưới từ B đên khi M trùng với A. Khi đó I trùng với l 2 . Nhưvậy ở mọi vị trí không có điểm I nào rơi vào phần cung ỈÍA và Al.

    Vậy quỹ tích chỉ là 2 đoạn cung l,jB và Bl 2 . CHÚ ý. Nhiều khi học sinh làm phẩn giãi hạn sau khi đã chứng minh phần đảo. Nhung thiết nghĩ dứt khoát phần này phải làm trưốc phần đảo, bởi có như thế mới tránh được trường hợp điểm lấy để chửng mình phần đảo lại rơi đúng vào phần không nằm trong giới hạn, lúc đó không thể chứng minh đƯỢc. Bưởc 4. Chửng minh phẩn dảo. Thông thương nếu một bàì toán quỹ tích mà tính chất (X chỉ gốm 1 tính chất thì phần đào thương đƯỢe tiến hành là: lấy một điểm bất kì thuộc hình A, phải chứng minh nó có tính chất a. Dạng nảy tương đối đơn giản. Học sinh thường gặp khò khàn trong những bài có nhiều giả thiết. Tức là những điểm M có từ hai tinh chát trỏ lên. Giáo viên phải hương dẫn cho học sính cách đảo bộ phận. Nghĩa là: nếu phải tlm quỹ tích M thỏa mãn tính chất Cí, p phẩn thuận đã chúng minh M thuộc hình A. Vậy thì phần đảo sẽ là: lãy M thuộc A, M có tính chất ra {hoặc [H). Chứng minh M cỏ tính chất p (hoặc ra). Ví dụ 4. (Chứng minh phần đảo của ví dụ 1) Theo đề bài cho thỉ M thỏa mãn hai tính chất MA= MB vá ẤPB = 90°. Phần thuận đã Ghứríg minh M thuộc (I, r). Qua M kè đương thẳng vuông góc vơi MO cất (O) tại A và B. Nối A, B vơi p. Chứng minh ẨPB - 90° và MA = MB. Chú ý. Trong phẩn đào cẩn lưu ý cho học sinh rằng kết luận của phẩn thuận bao giở củng phải là điểm xuất phát của phần đảo. Trong ví dụ trên nếu đảo lá vẽ APB = 90°, AB cểt (I, r) tại M. Cần chứng minh MA = MB. Cách này không hựp lí vì nếu trương hợp AB không cẩt đường tròn (I) thì không tồn tại điểm M thuộc (I). VI vậy trong nhiều cách đảo bộ phận, học sinh phải biết chọn cách nào là hợp lí. Bước 5. Kết luận quỹ tích Phần này đơn thuần chỉ kết luận lại quỹ tích cùa bài toán là gì, dựa vảo phần thuận và phẩn đảo. TỦ SÁCH TOÁN TUỔI THƠ {ACMNỮIOAN CAC C4U £Õ CAP TiÉU NỌC Jẳ«. Số trang; 172; Khổ; 17 X 24 cm, Giá bìa; 21 ũũũ đổng. Sô' trang; 1S8; Khổ; 17 X £4 crn, Giá bìa: 21 000 đong. ■■iu BAI GIÀNG SÓ HỌC SÔ’ trang; 136; Khổ; 17 X 24 cm, Giá bìa: 23 000 đổng. Số trang; 216; Khổ: 17 X 24 cm. Giá bìa: 22 ŨŨO đũng. Đóng tập tập chí cà nám 2009 Khổ: 19 X 27 cm Giá bìa: 75 000 đống.

    mĩSỊ Số trang- 216; Khổ: 17 X £4 em. Giá bìa: Sỉ 000 dồng. Đống tập tạp chí cả năm 2013 Khổ 19 X 27 cm Giá bìa. T45 000 dóng. |-.F.rc-T|ii B 1-pniT-. nTỉNCi ĩẠr T{UI!ÚlTMfMM2DlS 1111 V, lllll Bủng tập tạp chí cả nãm 2015 Khổ: 19 X 27 em Giá bìa; 165 000 đông. BẠN ĐỌC CÔ THỀ ĐẶT MUA TẬP CHỈ CẢ nam học tại các Cơ Sỏ BƯU ĐIỆN TRONG CẢ NƯỚC VỚI MÃ ĐẶT CÁC ẤN PHẨM NHƯ SAU: Tạp chí Toán Tuổi thờ 1: C169; Tạp chí Toán Tuổi thơ 2; C169.1 Tổng tập Toán Tuổi thơ 1 năm 2015: C169.2: Tổng lộp Toán Tuổi thơ 2 nàm 2015: 0169.3: Tổng tập Toán Tuổi thơ 1 nàm 2014: C169.4; Tổng tập Toán Tuồi thơ2nàm2014: C169.5; Tuyển chọn 10 nầm Toán Tuổi thơ
  286. Cảc chuyên đế toán chọn lọc THCS; C169.7; 279 Bải toán hình học phảng Olympic các nước; C169.&; Bài giảng sđ học: C169.6. Vui cười J KHÔNG GIÔNG Tí: - Này Tèo, sao bài thơ cửa cậu lại giống hệt bài của người khác dã dâng trẽn báo? Tèo: - Không giống hệt đâu. Tớ kí tên khác. Người đó kí t.ẽn khác. □ ĐẸP Tú - Tèo ơi, cậu thấy chữ tớ cô dẹp không? Tèo: - Đẹp. Tí: - Đẹp thế nào? Tèo: - Bay bướm, các nét như hòa quyện vào nhau... Tĩ: - Ôi, cậu dúng là người hiểu tớ! Tèo: - Nhưng mà... Tí: - Nhưng mà sao? Tèo: - Không dịch ra được. Tí: - Tẻo này, cậu có biết người vụng chèo khéo chống lả người như thế nào không? Chị tớ đo tớ mà tớ chịu, không trả lời dược. Tèo; - Lả ngươi láỉ dò. Dề thế mà cậu cũng chịu à? J KHÔNG QUEN TAY TÚ - Này Tèo, bãi vãn hôm qua của cậu sao lấm lồi chính tá thế? Tèo: - Tại hôm qua bút của tớ bj hỏng, tớ phải mượn bút bạn khác. Tí: - Bút Thì liên quan gì? Tèo: - Bút lạ, viết không quen tay, khó viết lắm. The nên tớ dành phải viết dại di mà không nghĩ dốn lỗi chính ta nữa. 7T: - Cậu dũng lả vừa vụng chèo vừa vụng chống! DỖ HỔNG THỊNII (Xồm ỉ 1 , Xuân Thành., Xuân Trường, Nam Định) Tuổi của một sô' thiết bị. uật dụng vằ công nghẹ BỈNH NAM HÀ Năm 201 6 nảy là tròn: 70 nãm máy tính điện tử ra đời tại Mỹ 75 năm thuốc kháng sinh được tìm ra ở Anh 95 nãm cách tiêm phòng bệnh lao phát minh ra ô Pháp 100 năm ống catốt ra đởi tại Mỹ 115 nãm người Pháp chê tạo được thủy phi cơ 120 năm phóng xạ nguyên tử được phát hiện ở Pháp 130 năm người Anh chế ra ngư lỗi 135 năm tàu điện ra đời ở Đức 140 nàm điện thoại ra đài ở Mỹ 150 năm mỏ tô hữỉ ra đời ở Pháp 170 năm người Mỹ tiến hành gây mê để phẫu thuật 175 năm người Anh phát minh ra phim chụp ảnh 200 nảm người Pháp phát minh ra đèn hầm mỏ 220 năm công nghệ in litỏ ra đời ở Đức 225 nãm người Đức chế tạo được hóa phẩm sút 230 nãrrt người Anh chế tạo đưọc máy bơm nước 230 năm chữ Quốc tế ngữ xuất hiện 230 nám máy kéo chỉ ra đờì ở Anh 295 năm chuông giật được chê tạo ở Anh 305 năm người Pháp làm ra máy hơi nước 365 nâm ngưài Hà Lan chế ra đồng hổ quả lắc 410 năm máy tính kiểu Pascal cùa Pháp 480 năm ngành phẫu thuật ra đỡí ở Pháp 975 nãm Trung Quốc phát minh ra chữ in 1020 răm đồng hổ dây cót giật ra đời ở Trung Quốc Hỏi: Anh Phó ơi! Em muốn học giỏi cả môn toán, cả môn vãn và mân tiếng Anh nữa thì em phải học như thê nào để làm được điểu đó? VŨ THỊ MAI (THCS Yên Phong, Yến Phong, Bắc Ninh) Đáp: Chỉ cỏ chăm đọc sách íhôi Sách hay kim cổ hiểu đời qua văn Học Anh đọc sách tiếng Anh Làm trong sách toán bài thành dễ đi Viết điều mình nghĩ mỗi khi Tỉm ra cái mới ẳt thì giỏi hơn. Hỏi: Sắp đến ngày Nhà giảo Việt Nam 20-11 rồi, em muốn làm một việc thật ý nghĩa để chúc mừng các thầy cô giảo, anh gợi ý cho em được không ạ? ĐỖ THU HÀ (6A8, THCS Thành Công, Ba Đình, Hà Nội) Thầy vui khi trò giỏi Trò giỏi điểm phải cao Em phải gắng lảm sao Tháng này nhiều điểm tốt Và nuôi thêm mơ ước Mai sau thầy tự hào Trò ngày nảỡ là em. Hỏi: ở trưởng em có một anh học rất giỏi môn toán, khi được hỏi tại sao anh lại học tốt môn toán như vậy thì anh nóì rang: Anh thường xuyên tự đọc sách, đọc và giải bài trẽn tạp chí Toán Tuổi thơ, nhò vậy mà anh nắm chắc kiến thức trên lớp và tiếp thu các kiến thức mới. Em phải làm gì để cũng học giồì như anh ấy ạ? TTKC (THCS Nguyễn Chích, Đông Sơn, Thanh Hóa) Đáp: Kiền tố vừa đố vừa giải Em hỏi xong dáp đàng hoàng Thường xuyên đạc, đọc và làm Học đi đôi với hành là thế Anh chỉ góp thêm một ti Bên cạnh đọc Toán Tuổi thơ Còn cền đọc thêm sách nữa. ANH PHÓ Bạn hãy vào vvebsite: //olm.vn/hieu-sach-online đé đọc tạp chí Toán Tuổi thơ bản điện tử nhé. CÁC LỚP 6 & 7 Sài 1(164+165). Tìm số abcde, biết abcde = a.b.e.d.e.45. LẠI QUANG THỌ (Phòng Giáo dục và Đào tạo Tam Dương, Vĩnh Phúc) Bài 2(164+165}. Cho tam giác ABC vuông cân tại A. Trên cạnh BC lấy điểm M. Tìm giá trị nhỏ nhất của MA 2 I MB 2 + MC 2 . CAO NGỌC TOAN (GV. THPT Tam Giang, Phong Điền, Thừa Thiên - Huế) CÁC LỠP THCS Bải 3(164+165). Cho các sô thực a, b, c đôi một khác nhau thỏa mãn (a - b)1 - c 3 + (b - c)\/l -a 3 + (c - a)ặl - b 3 =0. Chứng minh rằng ỈJ'(1-a 3 )(1-b 3 )(1-c 3 ) +abc - 1. LƯU LÝ TƯỞNG (GV. THCS Văn Lang, TP. Việt Trì, Phú Thọ) Bài 4(164+165). Tìm giá trị nhô nhất của biểu thức 0 \''3a 2 +4ab + 3b 2 V3b 2 + 4bc + 3c 2 V3c 2 +4ca +3a 2 ab bc ca trong đó các số thực dương a, b f c thỏa mãn a 2 + b 2 + c 2 = 1 , MAI VẢN NĂM (GV. THCS Khánh Hồng, Yên Khánh, Ninh Bình) Bải 5(164+165). Trẽn một mặt bàn vẽ hình chữ nhật 3 X 6 ô vuông và trong IY 10 Ĩ ô vuông cố 1 viên sỏi. Họng muốn thực hiện trò chdi sau: cứ mói lần Hồng sẽ chọn hai ỏ vuông và bốc ở mỗi ô vuông 1 viên sài đặt sang ô kể bẽn (hai ô kể nhau là hai ô có 1 cạnh chung). Hồng hỏi Hà. phải thực hiện việc chuyển các viên sỏi ít nhất bao nhiêu lần thì đưa được 18 viên sỏi vào một ô vuông. Hà nghĩ mãi mà chưa có câu trả lời, bạn hãy giúp Hà tìm cáu trả lài nhé. TẠ TOÀN (TP. HỔ Ch! Minh) Bài 6(164+165). Cho tam giảo nhọn ABC nội tiếp đường tròn (O) với AB < AC. Tiếp tuyến tại A của đường tròn (0) cắt BC tại T. Gọi D là điểm đối xứng của A qua o. Đường thẳng DB cắt OT tại E và cắt AT tại F. Đường tròn ngoại tiếp tam giác AEF cắt EO tại G khác E, Chửng minh rằng tâm đường tròn nội tiếp tam giác AGB nằm trẽn đường tròn (0). TRẦN QUANG HỬNG (GV trường THPT chuyên ĐHSPHà Nội) SOLUE UIA IYIAIL COIYIPETITIOn QUESTI0I1S Translated by Nam Vũ Thành 1(1 64+165). Find the number ahũdtì given that abcde =axbxtxdxex 45. 2(164+165). Given a right ÍSQSceles triangle with the right angle at A. Let M be a point on the side BC. Fỉnd the minimum value of MA 2 + MB 2 + MC 2 . 3(164+165). Given the real numbers a, and c which are pair-wise distinct such that (a -b^l-c 3 + (ố-c) 3 i-a 3 + {c - a)\h - h' 1 = 0 Prove that (1-â 3 )(1-,b 3 )(1-c 3 ) + abc = 1. PHIÊU ĐĂNG KÍ THAM Dự Ị CUỘC THI GTQT I NĂM HỌC 2016-2017 4(164+165). Find the minimum value of the expression n ^3a 2 + 4 ab T 3b 2 y'3ỉ> 2 +4bc-r3c 2 v'3c 2 -4ca + 3a 2 p ■ — — + — , where a, b, and ab bc ca c are positive real numbers such that a 2 + b 2 + c 2 = 1 . 5(164+165). On a table top, a rectangle containing 3x6 small squares is drawn and a stone is put in each ùí the squares. Hổng plays a game as follows: In each turn, she chooses 2 squares, takes one stone from each square and puts it in an adjacent square (2 adjacent squares are ones sharing 3 side). Hồng asks Hà how many turns are needed to put all the 18 stones into one single square. Hà has been thinking but has not found the answer. Can you help her ỉĩnd the ansvver? 6(164+165). Let ABC be an acute triangle inscribing a circle (0) where AB < AC. The tangent to the circle at A intersects BC at r Let D be the reílection of the point A through the point ọ. The line DB intersects OT at E and intersects AT at F- The circumcircle ofthe triangle AEF intersects the line EO atanotherpoint G. Prove that the center of the incircle of the triangle A GB lies on the circle (O). • Ngây 6.10.2016, tại trường THCS Giảng Võ, Q. Ba Dĩnh, Hà Nội dã dỉcn ra chương trinh Ngày Hội sách, Tơi dự có TS- Vũ Hình Chuẩn, Vụ TrƯỏrig Vụ Giáo đục Trung học, Bộ Giáo dục và Đảo tạo; Ths. Vũ Kim Thủy, Tổng biên tập tạp chi Toán Tuổi thơ; ỏng Nguyên Quang Trung, Phó Chủ tịch UBND quặn Ba Đình, ... ThS. Vũ Kim Thúy, Tống biên lập tạp chi Toán Tuổi thơ đã cố bải nói chuyện vè toán. Sau dó, các em học sính tham gia chương trĩnh Toán học Muôn màu rà Lí thú với nhi cu trò chơi toán học hã'p dẫn. Tạp chí Toán Tuổi thơ dã có các câu dố toán vui có l hưởng dành cho cãc em học sính. Các em học sinh yẽu các mõn Khoa học Xâ hộ ỉ tham gia chương trình Ngày Hội sách, mở trang sách dệt ươc mơ vđi nhiều hoạt dộng bổ ích. Các em học sinh lớp 8 và lớp 9 tham gĩa giới thĩộu các cuốn sách vã tạp chi minh yêu thích. Tạp chí Toán Tuổi thơ đã có gian hàng tham gia triển lãm và gửỉ tạp chí tặng tất cả các em học sính, • Ngày 16.10.2016, trường THPT chuyên Dại học Vinh, Nghệ An dá tổ chức Lẻ kỉ niệm 50 năm thành lập. Đến dự cõ ông Uõng Chu Lựu, Phó Chủ tịch Quốc hội nươc CHXHCN Việt Nam; TS. Nguyền Thị Nghĩa, Thử trưởng Hộ Giáo dục vã Đảo tạo; cs. TS. Nguyên Hữu Dư, Chủ tịch Hội Toán học Việt Nam; ông Nguyễn Vãn Thõng, Phủ Hi Thư Tĩnh ủy Nghệ An cùng các vị dại bícu, các thảy cõ giáo dã từng giăng dạy tại trường vã các thế hệ học sinh. Tiền thản lả lứp chuyên Toán đặc biệt VỚI 36 học sinh bát dầu học từ tháng 10.1966 tại xã miền nũi Thạch Bình, Thách Thảnh, Thanh Hóa trong điều kiện rất khó khăn. Sau dỏ các nám 1970 đến 1973 Khối Chuyên toán lản lượt sơ tán ở Quỳnh Nghĩa, Quỳnh Minh huyện Quỳnh Lưu, Diễn Trường, huyện Diễn Châu, Phú Thành, Hậu Thảnh, huyện Yên Thành. Năm 1974 Khối trở vẻ TP, Vinh. Cựu học sinh chuyên toán Đại học Vinh khóa 5 Nhã trương dã cõ gần 280 học sinh giỏi eã*p Quổc gia; cú 12 học sinh đạt. giải Toán Quốc tế (IMO) và Toán Khu vực Châu Á Thái Binh Dương (APMO). Nguyên Chủ tịch Hội Toán học Việt Nam Phạm The Long, Viện trưởng Viện Toán học Việt Nam Lê Tuấn Hoa, nguyên Phô Tổng biẽn tập tạp chi Toán Tuổi thơ Lè Thống Nhất, Tổng biên tặp tạp chí Toán Tuổi thơ, Phó Tổng Thư ki Hội Toán học Hà Nội Vũ Kim Thủy, Chánh vãn phong Hội dồng Quốc gia giáo dục Trần Dĩnh châu, ... tửng lã học sinh chuyên toản Đại học Vinh. Ghi nhặn những thành tích đã dạt dưực, trường TI1PT chuyên Dại học Vinh dã được Chủ tịch nưởc Cộng hòa Xả hội Chũ nghĩa Việt Nam tặng thương Huân chương Lao dộng hạng Ba vã nhiều Bằng khen. PV Tữ Hà Lan và châu Àu. những cánh hoa tulip đẹp lạ lũng đã đến nước la vả ở lại dây. Cũng thật khô khăn cho loài hoa xứ lạnh sống trên đất. nước nhiệt dới. JViu« xuân, hoa khoe sác trong các gia dinh thảnh phố: Hà Nộí, Nam Định, Hải Phòng, Đã Lạt, ... Lá vươn cao, cuống hoa thẳng khôc khoắn. Cánh hoa cũng hiíỡng thẳng lên trôn mà thoạt nhìn ta cứ tường hoa nhựa, Một vẻ dẹp không lộng xx*iơng kiêu sa thu hút ánh nhìn cua ngưởi ngưỡng mộ tulip. Bạn hãy viết một bãi viết về vẻ đẹp của bức ánh nảy. Tòa soạn chờ bài viết tốt của bạn để dâng vã bạn sẽ có phần thưởng. MORIS VŨ ■Anh: Vũ Thanh Há CÁC HỌC SINH ĐƯỢC KHEN TRONG cuộc THI GIẢI TOÁN DÀNH CHO NỮ SINH Từ trái sang phá/: nguyên Thũy Dương, Khổng Thị Thu Thúy, Nguyên Thi Quỳnh Anh, Nguyên An Na, Bùi Thi Ninh Thu. Giấy phép xuất bản: số 31/GP-BVHTT, cấp ngày 23/1 /2003 của Bộ Vân hóa và Thông tin. Mã số: SBTT164M1Ê, In tại: Công ty co phần in Công Đoản Việt Nam. 167 Tây Sơn, Đổng Đa, Hà Nội. In xong và nộp lưu chiểu tháng 1 1 nám 2016. Công ty CP VPP Hồng Hà là nhà tàỉ trợ cho 2 cuộc thỉ: và CPVÂN PHỎNG Prỉúì HỔNG M 51 MCE 175? ẨhA-t Ítuịf/M ĩỉiấriỳ - r(ỉĩỉì tị/ểĩvry Ỉsiỉ

Chủ đề